You are on page 1of 778

UPSC Prelims

Previous Years' Questions


Paper I (GS)
Previous Year Questions
A B C D A B C D

Previous Years’ Questions


A B C D A B C D
A B C D A B C D
A B C D A B C D
A B C D A B C D

with Explanations
A B C D A B C D
A B C D A B C D
A B C D A B C D
A B C D A B C D
A B C D A B C D
A B C D A B C D
A B C D A B C D
A B C D A B C D

By Unacademy
A
A
B
B
C
C
D
D
A
A
B
B
C
C
D
D

UPSC CSE Prelims


Topic-wise Solved Paper GS I
(2013-2023)
PREFACE

Every year, the Union Public Service Commission (UPSC) conducts


the Civil Services Examination (CSE) to recruit for various
Civil Services of the Government of India, including the Indian
Administrative Service (IAS), Indian Foreign Service (IFS), and
Indian Police Service (IPS) and other Central Services.

One of the most difficult exams in the world, it aims to select the
best of the best through a rigorous testing process. Consequently,
it becomes challenging for the candidates to prepare for the exam
because the previous year's papers and guidance are not readily
available.

Therefore, we at Unacademy have designed this topic-wise UPSC


Prelims GS Paper I containing more than 1000 questions featured
in the last decade, along with answers and explanations.

While due care has been taken to ensure accuracy in the preparation
of this book. Nonetheless, if there are any corrections, feedback or
suggestions, email us at learnersupport.upsc@unacademy.com.

ANUJ GARG MRUNAL PATEL

Director, Vice President,


Unacademy Unacademy
Table of Contents

Modern India 7
1. India in the 18th Century 8
2. Indian Renaissance and Reform Movements 16
3. Early Uprising Against the British and Revolt of 1857: 23
4. Rise of Indian National Movement: Moderate and Extremists Phase 26
5. Phases of Revolutionary Nationalism 33
6. The Beginning of Gandhian Era 35
7. The National Movement in the 1940s 49
8. Development of Press, Education and Civil Services 54
9. Independence to Partition 57

Ancient India 60
1. Prehistoric Period and Indus Valley Civilisation 61
2. Vedic and Later Vedic Age 65
3. Mauryan and Post-Mauryan Age 67
4. Gupta and Post- Gupta Age 78
5. Sangam Age 84

Medieval India 87
1. Delhi Sultanate (1206 AD to 1526 AD) 88
2. Mughal Empire (1526 AD to 1761 AD) 92
3. Provincial Kingdoms in Medieval India 96
4. Religious movement during medieval period 100

Art & Culture 102


1. Architecture and Sculpture 103
2. Literature: Religious and Scientific 110
3. Performing Arts: Dance, Theatre and Music 113
4. Visual Arts: Painting, ceramics and drawing 118
5. Indian Philosophy and Bhakti & Sufi Movements 120
6. Indian Traditions, Festivals, and Calendars 125

Table of Contents 3
unacademy.com | Download the Unacademy app
Give your feedback here: Link
7. Miscellaneous 129

World Geography 134


1. The Earth and the Universe 135
2. Geomorphology 141
3. Climatology 144
4. Oceanography 151
5. World Climatic Regions 158
6. Human and Economic Geography 162
7. World Map 164

Indian Geography 171


1. Physiography of India 172
2. Drainage System of India 181
3. Indian Climate 189
4. Soils 191
5. Natural Vegetation in India 194
6. Mineral and Industries 199
7. Agriculture in India 205
8. Indian Map 214

Environment & Ecology and Disaster Management 223


1. Protected Area Network: NP, WS, BR, etc. 224
2. Ecosystem and Ecology 230
3. Environmental Pollution 250
4. Biodiversity 265
5. Global Conservation Efforts 384
6. National Conservation Efforts 300
7. Climate Change: Causes and Implications 314
8. Environment, Sustainable Development and General Issues 325
9. Agriculture 335

4 Table of Contents
unacademy.com | Download the Unacademy app
Give your feedback here: Link
Indian Polity and Governance 347
1. Historical Background & Making of Indian Constitution 348
2. Features of the Indian Constitution 354
3. Legislature 375
4. Executive 399
5. Judiciary 411
6. Local Self Government 422
7. Governance 427
8. Constitutional and Non-constitutional Bodies 432
9. Judicial & Quasi-Judicial Bodies 438

International Relations 441


1. India's Foreign Policy 442
2. India & Its Neighbors 444
3. International Groups and Political Organizations 446
4. Places in news (2022) 459

Indian Economy 465


1. 1A1: Money: Barter to Bitcoins 466
2. 1B1: Bank Classification 471
3. 1B2: NPA, Bad-Loans, BASEL 493
4. 1C: Sharemarket, Companies Act 498
5. 1D1: Insurance, Pension, Financial inclusion 505
6. 2A1: Budget Direct Taxes 511
7. 2A2: Budget Indirect Taxes GST 515
8. 2BC: Finance Commission, BlackMoney, Subsidies 517
9. 3A: BoP, CAD Currency Exchange 521
10. 3B: WTO, IMF & other Intl. Orgs & Agreeements 544
11. 4A: Sectors of Economy- Agri 552
12. 4B: Sectors- MFG, Services, Ease of Doing Biz, IPR, Startup, MSME 564
13. 4C: NITI, Planning Commission, FYP, Unemployment 569
14. 4D: GDP, GNP 575

Table of Contents 5
unacademy.com | Download the Unacademy app
Give your feedback here: Link
15. 4E: Inflation 580
16. 5A: Infra: Energy 587
17. 5B: Infra: Transport, Urban Rural, Communication, Investment, PPP 594
18. 6A: HRD: Census, Health Hunger 599
19. 6B: HRD: Education and Skill 603
20. 6C: HRD: POVERTY 607
21. 6D: HRD: Weaker Section, HDI, SDG 610
22. 7: MicroEconomics 614

Science & Tech and Basic Science 616


1. Biotechnology 617
2. Defence Technology 632
3. Space Science 634
4. IT & Communication Technology 645
5. Energy 655
6. Miscellaneous 660
7. Physics 671
8. Chemistry 678
9. Biology 680

Current Affairs and Miscellaneous 691


1. Current Affairs: India 692
2. Current Affairs: World 737
3. GK/Persons in News 763
4. Miscellaneous 768

6 Table of Contents
unacademy.com | Download the Unacademy app
Give your feedback here: Link
1 MODERN INDIA
1 India in the 18th Century

1. Who among the following rulers of medieval established a factory at Madras on a plot
Gujarat surrendered Diu to the Portuguese? of land leased from a representative of
(2023) the Vijayanagara empire.
(a) Ahmad Shah Which of the statements given above are
(b) Mahmud Begarha correct?

(c) Bahadur Shah (a) 1 and 2 only

(d) Muhammad Shah (b) 2 and 3 only


(c) 1 and 3 only

2. By which one of the following Acts was the (d) 1, 2 and 3


Governor General of Bengal designated as
the Governor General of India? (2023) 5. In the first quarter of the seventeenth
(a) The Regulating Act century, in which of the following was/
(b) The Pitt's India Act were the factory/factories of the English
East India Company located? (2021)
(c) The Charter Act of 1793
1. Broach
(d) The Charter Act of 1833
2. Chicacole
3. Trichinopoly
3. With reference to Indian History, Alexander
Rea, A.H. Longhurst, Robert Sewell, James Select the correct answer using the code
Burgess and Walter Elliot were associated given below:
with (2023) (a) 1 only
(a) archaeological excavations (b) 1 and 2
(b)
establishment of English Press in (c) 3 only
Colonial India (d) 2 and 3
(c) establishment of Churches in Princely
States
6. Which of the following statements
(d) construction of railways in Colonial correctly explains the impact of the
India Industrial Revolution on India during the
first half of the nineteenth century? (2020)
4. With reference to Indian history, consider (a) Indian handicrafts were ruined.
the following statements: (2022) (b) Machines were introduced in the Indian
1. The Dutch established their factories/ textile industry in large numbers.
warehouses on the east coast on lands (c) Railway lines were laid in many parts of
granted to them by the Gajapati rulers. the country.
2. Alfonso de Albuquerque captured Goa (d) Heavy duties were imposed on the
from the Bijapur Sultanate. imports of British manufactures.
3. The English East India Company

8 India in the 18th Century


unacademy.com | Download the Unacademy app
Give your feedback here: Link
7. With reference to the history of India, 2. It asserted the sovereignty of the British
consider the following pairs: (2020) Crown over the Indian territories held
by the Company.
1. Aurang: In-charge of treasury
of the State 3. The revenues of India were now
controlled by the British Parliament.
2. Banian: Indian agent of the
East India Company Which of the statements given above are
correct?
3. Mirasidar : Designated revenue
payer to the State (a) 1 and 2 only
Which of the pairs given above is/are (b) 2 and 3 only
correctly matched? (c) 1 and 3 only
(a) 1 and 2 only (d) 1, 2 and 3
(b) 2 and 3 only
(c) 3 only 11. The staple commodities of export by the
(d) 1, 2 and 3 English East India Company from Bengal in
the middle of the 18th century were: (2018)
(a) Raw cotton, oil-seeds and opium
8. Wellesley established the Fort William
College Calcutta because (2020) (b) Sugar, salt, zinc and lead
(a) He was asked by the Board of Directors (c) Copper, silver, gold, spices and tea
at London to do so (d) Cotton, silk, saltpetre and opium
(b) He wanted to revive interest in oriental
learning in India
12. Which one of the following statements
(c) He wanted to provide William Carey and does not apply to the system of Subsidiary
his associates with employment Alliance introduced by Lord Wellesley?
(d) He wanted to train British civilians for (2018)
administrative purpose in India (a) To maintain a large standing army at
others expense
9. Which one of the following groups of plants (b) To keep India safe from Napoleonic
was domesticated in the New world and danger
introduced in the Old World? (2019) (c) To secure a fixed income for the
(a) Tobacco, cocoa and rubber Company
(b) Tobacco, cotton and rubber (d) To establish British paramountcy over
the Indian States
(c) Cotton, coffee and sugarcane
(d) Rubber, coffee and wheat
13.
Who among the following was/were
associated with the introduction of
10. Consider the following statements about Ryotwari Settlement in India during the
‘the Charter Act of 1813’: (2019) British rule? (2017)
1. It ended the trade monopoly of the East 1. Lord Cornwallis
India Company in India except for trade
2. Alexander Read
in tea and trade with China.
3. Thomas Munro

India in the 18th Century 9


unacademy.com | Download the Unacademy app
Give your feedback here: Link
Select the correct answer using the code peasants to the government.
given below: 2. The government gave the Pattas to the
(a) 1 only Ryots.
(b) 1 and 3 only 3. The lands were surveyed and assessed
(c) 2 and 3 only before being taxed.

(d) 1, 2 and 3 Which of the statements given above is/are


correct?
(a) 1 only
14. Consider the following: (2012)
(b) 1 and 2 only
1. Assessment of land revenue on the basis
of nature of the soil and the quality of (c) 1, 2 and 3
crops. (d) None of the above
2. Use of mobile cannons in warfare.
3. Cultivation of tobacco and red chillies. 16. The tendency for increased litigation was
Which of the above was/were introduced visible after the introduction of the land
into India by the English? settlement system of Lord Cornwallis in
1793. The reason for this is normally
(a) 1 only traced to which of the following provisions?
(b) 1 and 2 only (2011)
(c) 2 and 3 only (a) Making Zamindar’s position stronger
(d) None vis-a-vis the ryot
(b) Making East India Company an overlord
of Zamindars
15. With reference to Ryotwari Settlement,
consider the following statements (2012) (c) Making judicial system more efficient

1. The rent was paid directly by the (d) None of the (a), (b) and (c) above

10 India in the 18th Century


unacademy.com | Download the Unacademy app
Give your feedback here: Link
India in the 18th Century-
1 Explanation
1. Answer: (c) called the Council of India.
In 1534, the Bahadur Shah signed the Treaty
of Bassein with the Portuguese. By this, he 3. Answer: (a)
ceded Diu to the Portuguese, as well as other
1. Alexander Rea: He was an archaeologist
territories of his empire such as Vasai and the
who served as the Director-General
islands that today form Mumbai.
of Archaeology in British India. Rea
conducted excavations and surveys
2. Answer: (d) in various regions of India, including
The Government of India Act 1833 or the Charter ancient sites like Taxila and Amaravati.
Act 1833 was an act of the British Parliament, He made important discoveries and
later renamed the St Helena Act 1833. This contributions to the understanding of
extended the charter granted to the East India Indian history and art.
Company for another 20 years. Reorganized 2. A.H. Longhurst: A.H. Longhurst was
the British Indian government. an archaeologist and art historian
Provision of the Charter Act 1833 who specialized in the study of
Indian architecture and sculpture.
y Governor-General of Bengal, renamed He conducted extensive research
Governor-General of India. This made Sir and documentation of historical sites
William Bentinck the first Governor General and monuments in India. Longhurst
of India. authored several books and publications
y In this way, the administration of the country on Indian art and architecture.
was unified under one administration. 3. Robert Sewell: Robert Sewell was a
y Governors of Bombay and Madras lose British civil servant and historian. He
legislative power. is known for his work on the history
y The governor-general had legislative power and administration of the Vijayanagara
over all of British India. Empire. Sewell’s book, “A Forgotten
Empire: Vijayanagar - A Contribution
y The Governor-General of the Council has
to the History of India,” is considered a
the power to change, repeal or amend laws
seminal work on the subject.
affecting all persons and places within the
British Indian Territory, whether British, 4. James Burgess: James Burgess
foreigners or Indians. Rice field. was a Scottish archaeologist and
epigraphist who played a crucial role
y The civil and military affairs of the Company
in the documentation and preservation
were administered by the Governor-General
of Indian archaeological sites. He
of the Council.
conducted excavations and surveys
y The Governor’s Council needs him to have across different regions of India and
4 members. A fourth member had limited made significant contributions to
powers. the understanding of Indian history,
y For the first time, the Government of architecture, and culture.
the Governor-General was called the 5. Walter Elliot: Walter Elliot was a British
Government of India and the Council was administrator and politician who also

India in the 18th Century-Explanation 11


unacademy.com | Download the Unacademy app
Give your feedback here: Link
had a keen interest in archaeology. He in the first quarter of the 17th century.
conducted archaeological excavations
in various parts of India, particularly in
the region of Tamil Nadu. Elliot’s work 6. Answer: (a)
contributed to the exploration and India was a major exporter for textiles in the
preservation of historical sites in South early 18th century, but by the middle of the
India. 19th century it had lost all of its export market
These individuals made important and much of its domestic market. At the
contributions to the study of Indian history beginning of the Industrial Revolution, cotton
and archaeology, helping to uncover and industries were developed in England which
document the rich cultural heritage of the made industrial groups worry about imports
country. from other countries. The government was
pressured to impose import duties on cotton
textiles so that Manchester goods could sell
4. Answer: (b) in Britain without facing any competition from
Statement 1 is incorrect: The first factory outside.
established by Dutch was in Masulipatnam Option (a) is correct: At the same time, the
in 1605, whereas Gajapati’s rule lasted till the East India Company was also persuaded by
middle of the 16th century. industrialists to sell British manufactures
Statement 2 is correct: Albuquerque acquired in Indian markets as well. Exports of British
Goa from the Sultan of Bijapur in 1510. cotton goods increased dramatically in the
early nineteenth century and the export market
Statement 3 is correct. The rulers of
of the cotton weavers collapsed. Produced by
Vijayanagara appointed chieftains called
machines at lower costs, the imported cotton
Nayaks to rule independently in their areas.
goods were so cheap that weavers could not
Darmala Venkatadri Nayaka was in charge of
easily compete with them. By the 1850s, reports
what is Chennai today. He gave land to the
from most weaving regions of India narrated
British on which they established settlements
stories of decline and desolation. Thus, Indian
of British factory workers and merchants.
handicrafts were ruined as one of the impacts
of the Industrial Revolution.
5. Answer: (a)
y Elimination Technique
Option (a) is correct: The English East India
y Due to the industrial Revolution
Company had very humble beginnings in
Indian handicrafts were ruined and
India. Captain Hawkins arrived in the court
not the other way around. Machines
of Jahangir in April 1609. But the mission to
were introduced in the later time.
establish a factory at Surat did not succeed
Railway line was for the support of
due to opposition from the Portuguese, and
industrialisation in Britain.
Hawkins left Agra in November 1611.
By 1623 it had established factories (trading
7. Answer: (b)
posts) at Surat, Broach, Ahmedabad, Agra, and
Masulipatam. From the very beginning, it tried Pair 1 is not correctly matched: Aurang, a
to combine trade and diplomacy with war and Persian term for a warehouse –a place where
control of the territory where its factories were goods are collected before being sold; also
situated. refers to a workshop.

The factories of the British East India Company Pair 2 is correctly matched: The use of
were not located in Chicacole and Trichinopoly intermediaries with expertise of the local

12 India in the 18th Century-Explanation


unacademy.com | Download the Unacademy app
Give your feedback here: Link
markets and languages became common in originated from the Neolithic Revolution. The
the 18th and 19th centuries because of the three continents shared common domesticated
linguistic difficulties and cultural barriers plants making it easy to group them together.
faced by European merchants, who opened Option (a) is correct: The Old-World crops
and deepened trade routes throughout Asia, include wheat, rye, oats, lentils, and barley.
Africa, and Latin America. The intermediaries, Such crops did not exist in America until their
called ‘banians’ in the Anglo-Indian trade, introduction in the 1490s by post-Columbian
fulfilled various internal and external roles contact. The famous New World crops include
for trading companies including, managing rubber, tobacco, sunflower, cocoa, and cashew.
treasury functions, securing credit, and acting Some plants such as cotton and yam as well as
as brokers in the local markets. They were the some animals like the dog are believed to have
interpreter, head bookkeeper, head secretary, existed in both worlds.
head broker, the supplier of cash and cash-
keeper. Also, they were known as the Indian
agent of the East India Company. 10. Answer: (a)
Pair 3 is correctly matched: Under the The British Parliament passed the Charter Act
ryotwari settlement system, the government 1813 which is also known as the East India
recognized mirasidars as the sole proprietors Company Act 1813, to renew the charter of
of land, dismissing tenants’ rights completely. British East India Company. The Act sought to
Mirasidars were the designated revenue payer redress these grievances by allowing all the
to the State. Only in villages where the mirasdar British merchants to trade with India under a
system was absent, the villagers holding strict license system.
permanent occupancy rights were recognized Key Provisions of the Charter Act of 1813:
as landholders responsible for the payment of
y The act regulated the company’s territorial
land revenue.
revenues and commercial profits by asking
territorial and commercial accounts to be
8. Answer: (d) kept separate.
Option (d) is correct: Fort William College was y The Company’s monopoly over trade in
established on 18 August 1800 by Lord Richard India ended, but the Company retained
Wellesley, Governor General of Bengal in order to the trade with China and the trade in tea.
provide instruction in the vernacular languages (Statement 1 is correct)
of India to the civil and military officials of the y The Company was to retain the possession
East India Company. It was named after King of territories and the revenue for 20 years
William III of England. It was aimed to train more, without prejudice to the sovereignty
British civilians for administrative purposes of the Crown. Thus, the constitutional
in India, towards fostering Indian languages position of the British territories in India
and making the British officials familiar with was defined explicitly for the first time.
the local languages so that it could ease the (Statement 2 is correct and statement 3 is
administrative work, as it involved interaction not correct)
with the Indian natives.
A provision that the Company should invest Rs.
1 Lakh every year on the education of Indians
9. Answer: (a) was also proposed. The local governments,
The New and Old World is often quoted in the under this Act, were empowered to impose
context of agricultural crops. Europe, Asia, and taxes on the persons subject to the jurisdiction
Africa share a common agricultural history that of the Supreme Court.

India in the 18th Century-Explanation 13


unacademy.com | Download the Unacademy app
Give your feedback here: Link
of Subsidiary Alliance are, to maintain a large
Elimination Technique standing army at others expense, to keep India
Statement 1 and statement 2 are closely safe from Napoleonic danger, to establish
related. British paramountcy over the Indian States,
etc. To secure a fixed income for the Company
11. Answer: (d) was not a part of it.

Option (d) is correct: Crucial to the emergence


of a powerful capitalist economy, British- 13. Answer: (c)
Indian territory was developed as a source of Option (c) is correct: The Ryotwari System was
food and raw material for Britain. In the 1750s, devised by Captain Alexander Read and Sir
fine cotton and silk was exported from India to Thomas Munro at the end of the 18th century
markets in Europe, Asia, and Africa. Also, raw and was introduced by Munro when he was the
materials which chiefly consisted of saltpetre, Governor of Madras Presidency (1819–26).
opium and indigo, accounted for most of India’s
exports. In this system, the ownership rights were
handed over to the peasants, and the British
Elimination Technique Government collected taxes directly from
y Bengal was not a producer of sugar or them. The Individual cultivator called Ryot
sugar cane and salt. Option (b) can be had full rights regarding the sale, transfer, and
eliminated. leasing of the land.

y Most of the production of spice The Ryots could not be evicted from their land
varieties were in southern Indian as long as they paid the rent. It was prevalent in
states. Even today, southern Indian most of southern India, first introduced in Tamil
states are known for the production Nadu. It was later extended to Maharashtra,
of spices. Bengal is less significant for Berar, East Punjab, Coorg, and Assam. The
spice production or export. So, option advantages of this system were the elimination
(c) can be easily eliminated. of middlemen, who often oppressed villagers.

y Bengal was also not significant for oil-


seeds production or export during the 14. Answer: (d)
British time. So, Option (a) can also be Statement 1 is not correct: Assessment of land
easily eliminated. revenue on the basis of the nature of soil and
y We can mark option (d) as the correct the quality of crops was introduced into India
answer. by Sher Shah Suri and further rationalized by
Akbar.

12. Answer: (c) Statement 2 is not correct: Use of mobile


cannons in warfare was first done by Babur.
The doctrine of Subsidiary Alliance was
introduced by Lord Wellesley, British Governor- Statement 3 is not correct: Tobacco was
General in India from 1798 to 1805. Under the introduced to India during the reign of Akbar
subsidiary alliance system, the ruler of the by the Portuguese. The first tobacco plant was
allying Indian State was obliged to accept established in Gujrat in 1613 and it was also
the permanent stationing of a British force exported to Arakan and Pegu.
within his territory and to pay a subsidy for its Potatoes and chillies were introduced in India
maintenance. by the Portuguese during early 17th century and
Option (c) is correct: Some Important features 15th century respectively. Both were originally
from South America.

14 India in the 18th Century-Explanation


unacademy.com | Download the Unacademy app
Give your feedback here: Link
cultivation. The advantages of this system
Elimination Technique were the elimination of middlemen, who often
y It is well known that Tea was introduced oppressed villagers, and an assessment of the
by the British in India. So, statement 3 tax on land actually cultivated and not merely
or option (c) can be easily eliminated. occupied. This system also gave much power to
y If we can recall, the British period subordinate revenue officials, whose activities
in India- use of mobile canon is not were inadequately supervised.
mentioned anywhere. That means
option (b) can also be easily eliminated. 16. Answer: (d)
y Assessment of land revenue was based Option (d) is correct: The main features of the
on soil and crop quality was introduced land settlement system of Lord Cornwallis
during the Mughal. So, option (a) can were as under:
also be eliminated.
y The Zamindars who collected land revenues
y We can mark option (d) as the correct were made owners of the land.
answer.
y The Zamindars had to pay a fixed amount
to the company.
15. Answer: (c) y It was decided that the Government would
Statement 1 is correct: Ryotwari system, one claim 10/11 of the gross revenue.
of the three principal methods of revenue y In case, any Zamindar failed to pay the
collection in British India. It was prevalent in fixed amount of revenue, the government
most of southern India, being the standard had the right to confiscate some part of his
system of the Madras Presidency. The system land holding to recover the amount due.
was devised by Capt. Alexander Read and Sir
Thomas Munro at the end of the 18th century y The farmers or the ryots were made the
and introduced by the latter when he was tenants of the Zamindars.
governor (1820–27) of Madras (now Chennai). y The Zamindars were deprived of their
The principle was the direct collection of the administrative and judicial powers.
land revenue from each individual cultivator by y The government assured that it would
government agents. not interfere with the traditions of the
Statement 2 is correct: The registered Zamindars
agreements called Pattas were given to the Cornwallis abolished the court fees which
Ryots to recognize their ownership rights. affected the earnings of the company and
Statement 3 is correct: For the purpose resulted in a massive increase in the number
all holdings were measured and assessed of cases. Abolition of Court fees encouraged
according to crop potential and actual litigation without any restraint.

India in the 18th Century-Explanation 15


unacademy.com | Download the Unacademy app
Give your feedback here: Link
Indian Renaissance and Reform
2 Movements
1. With reference to Madanapalle of Andhra 4. Consider the following pairs: (2019)
Pradesh, which one of the following Movement/Organisation Leader
statements is correct? (2021)
1. All India Anti-Untouchability League
(a) Pingali Venkayya designed the tricolour Mahatma Gandhi
Indian National Flag here.
2. All India Kisan Sabha Swami Sahajanand
(b) Pattabhi Sitaramaiah led the Quit India Saraswati
Movement of the Andhra region from
here. 3. Self-Respect Movement Naicker E.V.
Ramaswami
(c) Rabindranath Tagore translated the
National Anthem from Bengali to English Which of the pairs given above is/are
here. correctly matched?

(d) Madame Blavatsky and Colonel Olcott (a) 1 only


set up headquarters of Theosophical (b) 1 and 2 only
Society first here. (c) 2 and 3 only
(d) 1, 2 and 3
2. The Vital-Vidhwansak, the first monthly
journal to have the untouchable people
5. Which among the following events
as its target audience, was published by:
happened earliest? (2018)
(2020)
(a)
Swami Dayanand established Arya
(a) Gopal Baba Walangkar
Samaj.
(b) Jyotiba Phule
(b) Dinabandhu Mitra wrote Neel Darpan.
(c) Mohandas Karamchand Gandhi
(c) Bankim Chandra Chattopadhyay wrote
(d) Bhimrao Ramji Ambedkar Anandmath.
(d) Satyendranath Tagore became the first
3. In the context of Indian history, the Indian to succeed in the Indian Civil
Rakhmabai case of 1884 revolved around: Services Examination.
(2020)
1. Women’s right to gain education 6. Satya Shodhak Samaj organised: (2016)
2. Age of consent (a) a movement for upliftment of tribals in
3. Restitution of conjugal rights Bihar
Select the correct answer using the code (b) a temple-entry movement in Gujarat
given below: (c) an anti-caste movement in Maharashtra
(a) 1 and 2 only (d) a peasant movement in Punjab
(b) 2 and 3 only
(c) 1 and 3 only 7. Consider the following: (2016)
(d) 1, 2 and 3 1. Calcutta Unitarian Committee

16 Indian Renaissance and Reform Movements


unacademy.com | Download the Unacademy app
Give your feedback here: Link
2. Tabernacle of New Dispensation 1. The Peasants and Workers Party of India
3. Indian Reform Association 2. All India Scheduled Castes Federation
Keshab Chandra Sen is associated with the 3. The Independent Labour Party
establishment of which of the above? Select the correct answer using the code
(a) 1 and 3 only given below:
(b) 2 and 3 only (a) 1 and 2 only
(c) 3 only (b) 2 and 3 only
(d) 1, 2 and 3 (c) 1 and 3 only
(d) 1, 2 and 3
8. Annie Besant was: (2013)
1. Responsible for starting the Home Rule 10. Which of the following statements is/are
Movement. correct regarding Brahmo Samaj? (2012)
2. The founder of the Theosophical Society. 1. It opposed idolatry.
3. Once the President of the Indian 2. It denied the need for a priestly class
National Congress. for interpreting the religious texts.
Select the correct statement/statements 3. It popularised the doctrine that the
using the code given below: Vedas are infallible.
(a) 1 only Select the correct answer using the code
(b) 2 and 3 only given below:

(c) 1 and 3 only (a) 1 only

(d) 1, 2 and 3 (b) 1 and 2 only


(c) 3 only

9. Which of the following parties were (d) 1, 2 and 3


established by Dr. B.R. Ambedkar? (2012)

Indian Renaissance and Reform Movements 17


unacademy.com | Download the Unacademy app
Give your feedback here: Link
Indian Renaissance and Reform
2 Movements-Explanation
1. Answer: (c) historic socio-economic oppression. He is
Option (c) is correct: Madanapalle is a city in generally considered to be the pioneer of
the Annamayya district of the Indian state of that movement. He developed a racial theory
Andhra Pradesh. to explain oppression. In 1888, Walangkar
began publishing the monthly journal titled
Rabindranath Tagore translated “Jana Gana Vital-Vidhwansak (Destroyer of Brahmanical
Mana” from Bengali to English and also set it or Ceremonial Pollution), which was the first
to music in Madanapalle, Andhra Pradesh. to have the untouchable people as its target
Rabindranath Tagore was a Bengali poet, short- audience.
story writer, essayist, and painter. He was
highly influential in introducing Indian culture
to the West and vice versa, and he is generally 3. Answer: (b)
regarded as the outstanding creative artist of Option (b) is correct: Dr. RakhmabaiBhikaji was
early 20th-century India. In 1913 he became a 19th century pioneer in the field of medicine
the first non-European to receive the Nobel and women’s rights. She was instrumental in
Prize for Literature. The song for “Jana Gana raising the age of consent for women in 1891.
Mana” was composed by Margaret Cousins, the She became India’s first practicing lady doctor
wife of educationist Dr James Henry Cousins. after having studied in the London school of
Medicine for Women in 1889. Rakhmabai who
got married at 11-years of age to the 19-year-
2. Answer: (a) old DadajiBhikaji, sought a legal divorce,
Option (a) is correct: Gopal Baba Walangkar, demanding ‘freedom’ from her non-consensual
also known as Gopal Krishna, (1840-1900) marriage and sought restitution of conjugal
was an activist working to release the rights.
untouchable people of India from their

18 Indian Renaissance and Reform Movements-Explanation


unacademy.com | Download the Unacademy app
Give your feedback here: Link
This led to a nation-wide debate over infant 4. Answer: (d)
and non-consensual marriage. The legal Pair 1 is correctly matched: The All-India Anti-
and social controversies provoked by the Untouchability League was set up by Mahatma
case revolved round notions of colonial law, Gandhi in 1932 to lend a conducive environment
marriage and conjugality, and the prospect for the upliftment of the oppressed classes or
of state intervention. BehramjiMalabari and ‘Harijans’. Gandhi advised that the activities
PanditaRamabai came to her defense and of the League should be mainly directed
formed the RakhmabaiDefense Committee. towards the economic, social and educational
improvement of the depressed classes
rather than limiting it to the issue of temple

Indian Renaissance and Reform Movements-Explanation 19


unacademy.com | Download the Unacademy app
Give your feedback here: Link
entry and inter-dining. Henceforth, the word all the knowledge. It also believed that post-
untouchable was replaced by Harijan and the Vedic texts such as Puranas were responsible
Anti-Untouchability League was renamed as for polluting the Vedic religion. It opposed the
Harijan Sevak Sangh. idol-worship and reincarnation theory of God.
Pair 2 is correctly matched: To spearhead Satyendranath Tagore was selected for the
peasant movements in colonial India, All Indian Civil Service in June 1863. He completed
India Kisan Sabha was formed in 1936, at the his probationary training and returned to India
Lucknow session of the Congress, with Swami in November 1864.
Sahajanand Saraswati as its first President. It Bankim Chandra Chattopadhyay wrote
later issued a Kisan manifesto which demanded Anandamath in 1882. It was set in the
the abolition of Zamindari and occupancy background of the Sanyasi Bidroho (rebellion
rights for all tenants. Under pressure from its of monks in late 18th century) and is one of key
socialist members and leaders, the Congress works on Bengal’s nationalism.
adopted an Agrarian Programme in December
1936.
Pair 3 is correctly matched: Self-Respect 6. Answer: (c)
Movement, started by E.V. Ramaswamy Option (c) is correct: Satya Shodhak Samaj
Naicker in Tamil Nadu in 1925, was a dynamic was founded by Mahatma Jyotirao Govindrao
social movement aimed at destroying the Phule on 24 September 1873 with an objective
contemporary Hindu social order in its totality to liberate the Shudras and Ati Shudras and to
and creating a new, rational society without prevent their ‘exploitation’ by the upper caste
caste, religion, and God. It was an egalitarian like ruling caste Maratha. It was an anti-caste
movement that propagated the ideologies of movement in Maharashtra.
breaking down the Brahminical hegemony, Satya Shodhak Samaj refused to regard the
equal rights for the backward classes and Vedas as sacrosanct, opposed idol worship,
women in the society and revitalization of denounced the chaturvarnya system (the caste
the Dravidian languages like Telugu, Tamil, system) and rejected the need for a Brahman
Kannada, and Malayalam. priestly class as educational and religious
leaders. The existence of God was replaced
5. Answer: (b) by Nirmik. Deenbandhu was the name of the
mouthpiece of the Satya Shodhak Samaj. Satya
Option (b) is correct: Neel Darpan is a famous Shodhak Samaj wanted a Social Revolution
Bengali play which was written by Dinabandhu and had encountered the masses quite deeply.
Mitra in 1858-59, published in Dhaka in 1860. A drum in the hand and Dhoti, Turban and
The aim of this play was to protest against the blanket was the dress of the preachers of
imperial rule of British rule in India. The main Satya Shodhak Samaj.
context of the play is on the event of Indigo
Revolt in Bengal.
Swami Dayanand established the Arya Samaj 7. Answer: (b)
in April 1875, in Bombay. Formed with the Keshab Chandra Sen was a Hindu philosopher
motto Krinvanto Vishvam Aryam (Make this and social reformer who attempted to
world noble), the socio-cultural movement incorporate Christian theology within the
aimed to reform society by raising awareness framework of Hindu thought. He was also a
about Vedic knowledge among people. The member of the Brahmo Samaj.
movement believed in infallibility of Vedas Option 1 is not correct: The Calcutta Unitarian
and took them as the only truth and source of Committee, jointly founded by William Adam

20 Indian Renaissance and Reform Movements-Explanation


unacademy.com | Download the Unacademy app
Give your feedback here: Link
and Rammohun Roy in September 1821, sought Theosophical Society in the United States in
to bring together prominent Brahmins who were 1875. In India, the movement became popular
friends of Roy’s and supporters of his agenda with the election of Annie Besant (1847-1933)
for the promotion of religious monotheism as its president after the death of Olcott in
and social reform among Hindus with British 1907.
and European residents of Calcutta who were Statement 3 is correct: In the 1917 Calcutta
Unitarian Christians. Keshab Chandra Sen was session of the Indian National Congress, Annie
not associated with the establishment of the Besant (the first women) became the President
Calcutta Unitarian Committee. of Indian National Congress.
Option 2 is correct: In 1868, on the occasion
of Magh celebration on 24th January, Keshab
Chandra Sen laid the foundation stone of 9. Answer: (b)
his new church - the Tabernacle of New Statement 1 is not correct: The Peasants and
Dispensation (Naba Bidhan) for the purpose of Workers Party of India is a Marxist political
establishing the truth of all the great religions party in Maharashtra, founded in 1949.
in an institution that he believed would replace Statement 2 is correct: All India Scheduled
them all. Castes Federation, a political party, was founded
Option 3 is correct: The Indian Reform by Dr.B.R.Ambedkar in 1942 to fight for the
Association was formed on 29 October 1870 rights of the Dalit Community. This party was
with Keshub Chunder Sen as president. It the successor organisation of the Independent
represented the secular side of the Brahmo Labour Party also led by Ambedkar.
Samaj. The objective was to put into practice Statement 3 is correct: Dr.B.R.Ambedkar
some of the ideas Sen was exposed to during founded the Independent labour Party, it
his visit to Great Britain. Its objective also participated in the provincial elections and he
included to create public opinion against himself was elected to the Bombay Legislative
child marriage, get Brahmo form of marriage Assembly. Statement 3 is Correct.
legalized, promote status of women.

10. Answer: (b)


8. Answer: (c)
Brahmo Samaj was founded by Raja Rammohan
Statement 1 is correct: The Home Rule Roy in August 1828. It was later renamed
Movement was the Indian response to the First Brahmo Samaj. Through the Sabha he wanted
World War in a less charged but more effective to institutionalise his ideas and mission.
way than the response of Indians living abroad The Samaj was committed to “the worship
which took the form of the romantic Ghadr and adoration of the Eternal, Unsearchable,
adventure. Two Home Rule Leagues were Immutable Being who is the Author and
launched—one by Bal Gangadhar Tilak and the Preserver of the Universe”.
other by Annie Besant, both with the aim of
Option (b) is correct: The features of Brahmo
beginning a new trend of aggressive politics.
Samaj are:
She set up her All-India Home Rule League in
September 1916 in Madras and covered the y It denounced polytheism and idol worship.
rest of India (including Bombay city). y It discarded faith in divine avataras
Statement 2 is not correct: A group of (incarnations).
westerners led by Madame H.P. Blavatsky (1831- y It denied that any scripture could enjoy the
1891) and Colonel M.S. Olcott, who was inspired status of ultimate authority transcending
by Indian thought and culture, founded the human reason and conscience.

Indian Renaissance and Reform Movements-Explanation 21


unacademy.com | Download the Unacademy app
Give your feedback here: Link
y It took no definite stand on the doctrine of y The doctrine of infallibility of vedas, was
karma and transmigration of soul and left it not popularized by Brahmo Samaj. Though
to individual Brahmos to believe either way. the Brahmo samaj originated as a reformist
y It criticised the caste system. movement on the ancient foundations of
Vedic religion, yet there infallibility of vedas
y It denied the need for a priestly class for was questioned by many of its members.
interpreting the religious texts.

22 Indian Renaissance and Reform Movements-Explanation


unacademy.com | Download the Unacademy app
Give your feedback here: Link
Early Uprising Against the British
3 and Revolt of 1857
1. Indigo cultivation in India declined by the (c) Both 1 and 2
beginning of the 20th century because of: (d) Neither 1 nor 2
(2020)
(a) peasant resistance to the oppressive
conduct of planters 4. What was/were the object/objects of
Queen Victoria’s Proclamation (1858)?
(b) its unprofitability in the world market (2014)
because of new inventions
1. To disclaim any intention to annex
(c) national leaders’ opposition to the Indian States.
cultivation of indigo
2. To place the Indian administration under
(d) Government control over the planters the British Crown.
3. To regulate East India Company’s trade
2. With reference to the history of India, with India.
“Ulgulan” or the Great Tumult is the Select the correct answer using the code
description of which of the following given below:
events? (2020)
(a) 1 and 2 only
(a) The Revolt of 1857
(b) 2 only
(b) The Mappila Rebellion of 1921
(c) 1 and 3 only
(c) The Indigo Revolt of 1859-60
(d) 1, 2 and 3
(d) BirsaMunda’s Revolt of 1899-1900

5. Which amongst the following provided a


3. After the Santhal Uprising subsided, what common factor for tribal insurrection in
were the measure/measures taken by the India in the 19th century? (2011)
colonial government? (2018)
(a) Introduction of a new system of land
1. The territories called ‘Santhal Parganas’ revenue and taxation- of tribal products
were created.
(b) Influence of foreign religious
2. It became illegal for a Santhal to transfer missionaries in tribal areas
land to a non-Santhal.
(c) Rise of a large number of money
Select the correct answer using the code lenders, traders and revenue farmers as
given below: middlemen in tribal areas
(a) 1 only (d) The complete disruption of the old
(b) 2 only agrarian order of the tribal communities

Early Uprising Against the British and Revolt of 1857 23


unacademy.com | Download the Unacademy app
Give your feedback here: Link
Early Uprising Against the British
3 and Revolt of 1857-Explanation
1. Answer: (b) degree of legal protection for their land rights.
Option (b) is correct: New inventions took place
in Germany where the invention of scientific 3. Answer: (c)
techniques like modern synthetic chemistry
Santhals are a group of tribes concentrated
took place. By the late 19th century, nearly
in the state of Jharkhand. The Santhal Revolt
all indigo came from indigo bush plantations
took place in 1855-56. This was the first
in India. The Germans succeeded in making
peasant movement which took place in India.
it in the laboratory in 1878, but it took nearly
The revolt has reference to the establishment
three decades for the large-scale production
of the permanent land settlement of 1793.
of synthetic indigo.
Statement 1 is correct: Santhal rebellion
The breakthrough came in 1890, when Karl
was led by four Murmu brothers named
Heumann in Zurich found a way of making
Sindhu, Kanhu, Chand and Bhairav against the
indigo from aniline. A lucky accident at the
oppressive zamindari system. The Santhals
German firm had a revelation that mercury was
showed exemplary courage in fighting against
a catalyst for a key part of the synthesis which
the British, despite being beaten and harassed.
led to production of synthetic indigo in 1897.
In order to be able to rule the area, the British
The synthetic dye was much cheaper and the
finally agreed on demands of Santhals following
final blow to the naturally produced indigo dye.
which the district of Santhal Pargana was
Hence, indigo crop became a part of history for
created in 1885, carving out 5,500 square miles
its unprofitability in the world market because
from the districts of Bhagalpur and Birbhum.
of new inventions.
Statement 2 is correct: Also, the British
government enacted laws to ensure that the
2. Answer: (d) tribal land is not taken away by outsiders
Option (d) is correct: Munda Rebellion was one (dikus). This means, it became illegal for a
of the prominent 19th century tribal rebellions Santhal to transfer land to a non-Santhal.
in the subcontinent. It was led by BirsaMunda
in the region south of Ranchi in 1899-1900.
4. Answer: (a)
The Ulgulan, meaning ‘Great Tumult’, sought
to establish Munda Raj and independence. It The assumption of the Government of India by
is also known as the BirsaMunda’s Revolt of the sovereign of Great Britain was announced
1899-1900. The Mundas traditionally enjoyed by Lord Canning at a durbar at Allahabad in the
a preferential rent rate as the khuntkattidar ‘Queen’s Proclamation’ issued on November 1,
or the original clearer of the forest. But in 1858.
the 19th century, this khuntkatti land system y The era of annexations and expansion had
was eroded by the jagirdars and thikadars ended, and the British promised to respect
who came as merchants and moneylenders. the dignity and rights of the native princes.
The government attempted to redress the (Statement 1 is correct)
grievances of the Mundas through the survey
y The direct responsibility for the
and settlement operations of 1902-10. The
administration of the country was assumed
Chotanagpur Tenancy Act of 1908 provided
by the British Crown and Company rule was
some recognition to their khuntkatti rights and
abolished. (Statement 2 is correct)
banned beth begari. Chotanagpurtribals won a

24 Early Uprising Against the British and Revolt of 1857-Explanation


unacademy.com | Download the Unacademy app
Give your feedback here: Link
y The Governor-General acquired the 5. Answer: (d)
additional title of ‘Viceroy’. Option (d) is correct: The main cause of
y It proclaimed the end of an era of the East tribal insurrection was the rapid changes
India Company’s rule. (Statement 3 is not that the British introduced in the economy,
correct) administration, and land revenue system.
These changes led to the disruption of the
y People of India were promised freedom of
old agrarian order of the tribal communities
religion without interference from British
causing prolonged and widespread suffering
officials.
among tribal communities. Also, increasing
y The proclamation announced pardon to demands for land revenue and extracting as
Indians who had taken part in the Revolt of large an amount as possible by the British
1857 against the British. government proved to be devastating for
Elimination technique: Queen Victoria’s Indian villages, throwing millions on the verge
Proclamation (1858) proclaimed the end of of impoverishment. The British government
an era of the East India Company’s rule encroached upon the traditional tribal land
and not to regulate hence statement 3 is rendering tribals in a condition of destitution
not correct. in their own land.

Early Uprising Against the British and Revolt of 1857-Explanation 25


unacademy.com | Download the Unacademy app
Give your feedback here: Link
Rise of Indian National Movement:
4 Moderate and Extremists Phase
1. With reference to the book ‘Desher Katha’ 4. Consider the following pairs: (2017)
written by Sakharam Ganesh Deuskar 1. Radhakanta Deb — First President of
during the freedom struggle, consider the the British Indian Association
following statements: (2020)
2. Gazulu Lakshminarasu Chetty —
1. It warned against the Colonial State's Founder of the Madras Mahajana Sabha
hypnotic conquest of the mind.
3. Surendranath Banerjee — Founder of
2. It inspired the performance of swadeshi the Indian Association
street plays and folk songs.
Which of the above pairs is/are correctly
3. The use of ‘desh’ by Deuskar was in the matched?
specific context of the region of Bengal.
(a) 1 only
Which of the statements given above are
correct? (b) 1 and 3 only

(a) 1 and 2 only (c) 2 and 3 only

(b) 2 and 3 only (d) 1, 2 and 3

(c) 1 and 3 only


(d) 1, 2 and 3 5. Consider the following statements: (2017)
1. The Factories Act, 1881 was passed with
a view to fix the wages of industrial
2. Economically one of the results of the workers and to allow the workers to
British rule in India in the 19th century was form trade unions.
the: (2018)
2. N.M. Lokhande was a pioneer in
(a)
Increase in the export of Indian organising the labour movement in
handicrafts British India.
(b) Growth in the number of Indian owned Which of the above statements is/are
factories correct?
(c) Commercialization of Indian agriculture (a) 1 only
(d) Rapid increase in the urban population (b) 2 only
(c) Both 1 and 2
3. He wrote biographies of Mazzini, Garibaldi, (d) Neither 1 nor 2
Shivaji and Shrikrishna, stayed in America
for some time; and was also elected to the
Central Assembly. He was: (2018) 6. What was the main reason for the split in
(a) Aurobindo Ghosh the Indian National Congress at Surat in
1907? (2016)
(b) Bipin Chandra Pal
(a)
Introduction of communalism into
(c) Lala Lajpat Rai Indian politics by Lord Minto.
(d) Motilal Nehru (b) Extremists’ lack of faith in the capacity
of the moderates to negotiate with the
British Government.

26 Rise of Indian National Movement: Moderate and Extremists Phase


unacademy.com | Download the Unacademy app
Give your feedback here: Link
(c) Foundation of Muslim League. Select the correct answer using the code
(d)
Aurobindo Ghosh’s inability to be given below:
elected as the president of the Indian (a) 1 only
National Congress. (b) 1 and 2 only
(c) 2 and 3 only
7. The ‘Swadeshi’ and ‘Boycott’ were adopted (d) 1, 2 and 3
as methods of struggle for the first time
during the (2016)
(a) Agitation against the partition of Bengal 11. The Partition of Bengal made by Lord
Curzon in 1905 lasted until (2014)
(b) Home Rule Movement
(a) the First World War when Indian troops
(c) Non-Cooperation Movement were needed by the British and the
(d) Visit of Simon Commission to India partition was ended.
(b) King George V abrogated Curzon’s Act at
8. Which one of the following movements the Royal Durbar in Delhi in 1911.
has contributed to a split in the Indian (c) Gandhiji launched his Civil Disobedience
National Congress resulting in emergence Movement.
of ‘moderates’ and ‘extremists’? (2015) (d) the Partition of India in 1947 when East
(a) Swadeshi Movement Bengal became East Pakistan.
(b) Quit India Movement
(c) Non-Cooperation Movement 12. The Ilbert Bill controversy was related to
(d) Civil Disobedience Movement the (2013)
(a) imposition of certain restrictions to
carry arms by the Indians.
9. Consider the following statements: (2015)
(b) imposition of restrictions on newspapers
1. The first woman President of the Indian
and magazines published in Indian
National Congress was Sarojini Naidu.
languages.
2. The first Muslim President of the Indian
(c) removal of disqualifications imposed on
National Congress was Badruddin Tyabji.
the Indian Magistrates with regard to
Which of the statements given above is/are the trial of the Europeans.
correct?
(d) removal of a duty on imported cotton
(a) 1 only cloth.
(b) 2 only
(c) Both 1 and 2 13. During the Indian freedom struggle, the
(d) Neither 1 nor 2 National Social Conference was formed.
What was the reason for its formation?
(2012)
10. Who of the following was/were economic
(a)
Different social reform groups or
critic/critics of colonialism in India? (2015)
organisations of the Bengal region united
1. Dadabhai Naoroji to form a single body to discuss the
2. G. Subramania Iyer issues of larger interest and to prepare
appropriate petitions/ representations
3. R. C. Dutt

Rise of Indian National Movement: Moderate and Extremists Phase 27


unacademy.com | Download the Unacademy app
Give your feedback here: Link
to the government. 15. With reference to the period of colonial
(b)
Indian National Congress did not rule in India, “Home Charges” formed
want to include social reforms in its an important part of the drain of wealth
deliberations and decided to form a from India. Which of the following funds
separate body for such a purpose. constituted “Home Charges”? (2011)

(c) Behramji Malabari and M. G. Ranade 1. Funds used to support the India Office
decided to bring together all the social in London.
reform groups of the country under one 2. Funds used to pay salaries and pensions
organisation. of British personnel engaged in India.
(d) None of the statements (a), (b) and 3. Funds used for waging wars outside
(c) given above is correct in this India by the British.
context. Select the correct answer using the code
given below:
14. Consider the following statements: (2012) (a) 1 only
The most effective contribution made by (b) 1 and 2 only
Dadabhai Naoroji to the cause of Indian (c) 2 and 3 only
National Movement was that
(d) 1, 2 and 3
1. Exposed the economic exploitation of
India by the British.
2. Interpreted the ancient Indian texts and 16. What was the purpose with which Sir
restored the self-confidence of Indians. William Wedderburn and W.S.Caine had set
up the Indian Parliamentary Committee in
3. Stressed the need for eradication of all 1893? (2011)
the social evils before anything else.
(a) To agitate for Indian political reforms in
Which of the statement(s) given above is/ the House of Commons
are correct?
(b) To campaign for the entry of Indians
(a) 1 only into the Imperial Judiciary
(b) 2 and 3 only (c) To facilitate a discussion on India’s
(c) 1 and 3 only Independence in the British Parliament
(d) 1, 2 and 3 (d) To agitate for the entry of eminent
Indians into the British Parliament

28 Rise of Indian National Movement: Moderate and Extremists Phase


unacademy.com | Download the Unacademy app
Give your feedback here: Link
4 Rise of Indian National Movement: Moderate
and Extremists Phase-Explanation

1. Answer: (a) the number of landless labourers.


Statement 1 is correct: Sakharam Ganesh It also brought in many merchants, traders
Deuskar (1869-1912) a close associate of Sri and middlemen who further exploited the
Aurobindo was a marathi brahmin who had situation; There was an enormous drain of
settled in Bengal, Sakharam was born in wealth from India to Britain due to the various
Deoghar. His text, titled Desher Katha (Story economic policies. Additional financial burden
of the Nation/Country), written in 1904, warned was placed on India due to expenditures on
against the colonial state’s ‘hypnotic conquest salaries, pensions, and training of military and
of the mind’. civilian staff employed by the British to rule
Statement 2 is correct: This book had an India.
immense repercussion in Bengal, captured the
mind of young Bengal and assisted more than 3. Answer: (c)
anything else in the preparation of the Swadeshi
movement. The government of Bengal banned Option (c) is correct: Lajpat Rai was a famous
the book in 1910 and confiscated all the copies. nationalist in India who was also known for
But by the time DesherKatha was banned by his proficiency in writing. To rouse and inspire
the colonial state in 1910, it had sold over 15,000 the Punjabis with patriotic zeal, he wrote
copies, inspired swadeshi street plays and folk the biographies of Mazzini, Garibaldi, Shivaji,
songs, and had become a mandatory text for an Dayanand and Shri Krishna, besides other
entire generation of swadeshi activists. Deuskar important works in English, Hindi, and Urdu.
used ‘desh’ to mean nation. It is worth quoting He was famously called the “Lion of Punjab”.
part of an article “AmaderDesher Katha” [About
our Country], that appeared in the children’s 4. Answer: (b)
periodical Prakriti [Nature] in 1907.
Pair 1 is correctly matched: British Indian
Statement 3 is not correct: The interpretation Association was created after consolidating
of Bengal as ‘desh’ can be seen in the book. the “Landholders Society” and “British India
But, use of ‘desh’ by Deuskar was not in the Society” on 31 October 1851. It was the first
specific context of the region of Bengal. political organisation to bring the Indians
together. The President of the first committee
2. Answer: (c) of this organisation was Raja Radhakanta Deb,
while Debendranath Tagore was its secretary.
Option (c) is correct: Commercialization of The newspaper of this society was “Hindu
Indian agriculture was the result of British rule patriot”, which adopted a strongly critical
in India in the 19th century. Farmers were forced political tone.
to grow Indigo and other cash crops which
were exported to Great Britain as raw materials Pair 2 is not correctly matched: The Madras
of the English factories. In the 19th century, Mahajana Sabha was established in May 1884
agriculture became commercial due to the use by M. Veeraraghavachariar, G. Subramania Iyer,
of modern agricultural equipment and Indians and P. Anandacharlu
started producing crops for commercial use Pair 3 is correctly matched: Indian National
and on a large scale basis. Commercialisation Association was the first declared Nationalist
of agriculture further enhanced the speed of Organisation founded in British India by
transfer of ownership of land thereby increasing Surendranath Banerjee and Anand Mohan Bose

Rise of Indian National Movement: Moderate and Extremists Phase-Explanation 29


unacademy.com | Download the Unacademy app
Give your feedback here: Link
in 1876. It was originally established as Bharat Government.
Sabha and held its first annual conference in The Rift between these two sections became
Calcutta. It merged with the Indian National clearly visible at the Banaras Session of
Congress in 1885. It sought to bring Hindus and Congress (1905) when some nationalist led by
Muslims together for political action. Tilak denounced the method of the moderates
and suggested passive resistance. They also
5. Answer: (b) advocated the boycott of British goods and
government institutions. The moderates after
Statement 1 is not correct: The first Factory the Surat Split in 1907 demanded colonial self-
Act was passed in 1881. It aimed to improve government, as against the extremist demand
the working conditions of labour. The Act of complete independence.
prohibited the employment of children under
the age of seven, limited the number of working
hours for children below the age of twelve and 7. Answer: (a)
required that dangerous machinery should Option (a) is correct: The ‘Swadeshi’ and
be fenced properly. Thus, for the first time, ‘Boycott’ were adopted as methods of struggle
the British Government tried to improve the for the first time during the agitation against the
working conditions of labourers in factories. It partition of Bengal. The Swadeshi movement
neither fixed the wages of industrial workers of Bengal (1905-1908) is seen as an important
nor allowed the workers to form trade unions historical event in the episodic narrative of the
exclusively. Indian Nationalist Movement, which takes the
Statement 2 is correct: N.M Lokhande was story forward to its eventual climax in 1947. Lord
a pioneer of the labour movement in British Curzon’s unpopular decision to partition the
India. He is remembered for working towards province of Bengal in 1905, led to this popular
improving the working conditions of textile movement, which was organised around
mill-hands in the 19th century and for his the effective use of ‘swadeshi’ and ‘boycott’
courageous initiatives on caste and communal as methods of agitation, under Extremist
issues. leadership. The subsequent unification of
Bengal in 1911 came to be regarded as a marker
Elimination technique: We know that N.M of the movement’s success
Lokhande was a pioneer of the labour
movement in British India. Hence we are Mahatma Gandhi described Swadeshi as “a call
left with the b and c now if we see 1st to the consumer to be aware of the violence
statement closely it is too deep for a first he is causing by supporting those industries
labour code to talk about fixing the wages that result in poverty, harm to workers and to
of industrial workers, so 1st statement is humans and other creatures.” The Swadeshi
incorrect. Movement was an attempt to take economic
power from the British using domestic made
products.
6. Answer: (b)
Option (b) is correct: The Surat Split was a
turning point in the history of Nationalist 8. Answer: (a)
Movement in India. The Congress split into two Option (a) is correct: The Swadeshi Movement
groups i.e., moderates and extremists at the has contributed to a split in the Indian National
Surat in 1907. The main reason for the split in Congress resulting in emergence of ‘moderates’
the Indian National Congress (INC) is because and ‘extremists.
of the extremists’ lack of faith in the capacity y The Surat Split was a turning point in the
of the moderates to negotiate with the British

30 Rise of Indian National Movement: Moderate and Extremists Phase-Explanation


unacademy.com | Download the Unacademy app
Give your feedback here: Link
history of Nationalist Movement in India. carving the world’s first economic critique of
The Congress split into two groups, i.e., colonialism before Hobson and Lenin. The drain
moderates and extremists at Surat in theory was developed by Dadabhai Naoroji.
1907. The Rift between these two sections According to R.C. Dutt, increased poverty and
became clearly visible at the Banaras lower wages were among the indirect products
Session of Congress (1905) when some of colonial rule.
nationalists led by Tilak denounced the
method of the moderates and suggested Elimination technique: All were freedom
passive resistance. fighters and they all criticised foreign rule
(who will not criticise). Hence even if we
y They also advocated the boycott of British don’t know, there is a strong chance that
goods and government institutions. The all will be the answer.
moderates after the Surat Split in 1907
demanded colonial self-government, as
against the extremist demand of complete
independence. 11. Answer: (b)
The British government’s decision to partition
9. Answer: (b) Bengal had been made public in December
1903. The idea was to have two provinces:
Statement 1 is not correct: Annie Besant, of Bengal comprising Western Bengal as well as
Irish origin, was the first woman President of the provinces of Bihar and Orissa and Eastern
Indian National Congress (INC), who presided Bengal and Assam. Bengal retained Calcutta as
over the INC at the Calcutta Session of 1917. its capital, while Dacca became the capital of
Sarojini Naidu was appointed the President of Eastern Bengal.
the Indian National Congress in 1925 and later
became the Governor of the United Provinces Option (b) is correct: Bengal was officially
in 1947, becoming the first woman to hold the partitioned on October 16, 1905, by Viceroy
office of Governor in the Dominion of India. Curzon. King George V held a darbar in Calcutta
in 1911 and announced the annulment of the
Statement 2 is correct: Badruddin Tyabji was Partition of Bengal mainly to curb the menace
the first Indian to practice as a barrister of the of revolutionary activities and to shift the
High Court of Bombay who served as the third capital of British India to Delhi.
President of the Indian National Congress.
He was one of the founding members and
first Muslim President of the Indian National 12. Answer: (c)
Congress. Option (c) is correct: In 1883 Viceroy Ripon’s
Elimination technique: Annie Besant law member, lord Ilbert had sought to abolish
was the first woman President of Indian “judicial disqualification based on race
National Congress (INC) not Sarojini Naidu. distinctions” and to give the Indian members of
The 2nd statement is fact based. the covenanted civil service the same powers
and rights as those enjoyed by their European
colleagues. Ripon had to modify the bill, thus
almost defeating the original purpose, because
10. Answer: (d) of the stiff opposition from the European
Option (d) is correct: Dadabhai Naoroji, R. C. community.
Dutt, Ranade, Gokhale, G. Subramania Iyer were 13. Answer: (b)
among those who anchored Indian nationalism Option (b) is correct: Indian (National) Social
firmly on the foundation of anti-imperialism by Conference Founded by M.G. Ranade and

Rise of Indian National Movement: Moderate and Extremists Phase-Explanation 31


unacademy.com | Download the Unacademy app
Give your feedback here: Link
Raghunath Rao, the Indian Social Conference in the UK, purchases of military items and
met annually from its first session in Madras in railway equipment. Government procurement
1887 at the same time and venue as the Indian of civilian goods, armaments and shipping was
National Congress. It focussed attention on the carried out almost exclusively in the UK. By the
social issues of importance; it could be called 1930s these home charges were in the range of
the social reform cell of the Indian National £40 to £50 million a year. Some government
Congress, in fact. The conference advocated expenditure was on imports which an
inter-caste marriages, opposed polygamy and independent government would have bought
kulinism. It launched the ‘Pledge Movement’ to from local manufacturers. Of these official
inspire people to take a pledge against child payments, we can legitimately consider service
marriage. charges on non-productive debt, pensions, and
furlough payments as a balance of payment
drain due to colonialism.
14. Answer: (a)
Home Charges:
The early intellectuals of the first half of the
nineteenth century supported British rule y Cost of the Secretary of State’s India Office
under the impression that it would modernise in London. (Option 1 is correct)
the country based on the latest technology y East India Company’s military adventure.
and capitalist economic organisation. After the y Salaries and pensions to British Indian
1860s, disillusionment started to set in among officials and army officers. (Option 2 is
the politically conscious and they began to correct)
probe into the reality of British rule in India.
y The compensation of the Company’s
shareholders.
y Cost of army training.
y Transport, equipment, and campaigns
outside India.
y Guaranteed interest on railways.

Elimination technique: Funds used for


waging wars outside India by the British
Option (a) is correct: The foremost among these were not included in the Home Charges.
economic analysts was Dadabhai Naoroji, the
‘Grand Old Man of India’, who after a brilliant
16. Answer: (a)
analysis of the colonial economy put forward
the theory of economic drain in Poverty and Option (a) is correct: In 1893, Sir William
Under British Rule in India and exposed the Wedderburn became a member of British
economic exploitation of India by the British. Parliament. He along with W.S. Caine and Lord
Clywd, established the Indian Parliamentary
Committee to present the Indian problems in
15. Answer: (b) the British Parliament (House of Commons) in
During the period of direct British rule from their true shape.
1858 to 1947, official transfers of funds to the y Sir William Wedderburn served as the
UK by the colonial government were called President of the Indian National Congress
the “Home Charges”. They mainly represented in 1889 and 1910.
debt service, pensions, India Office expenses

32 Rise of Indian National Movement: Moderate and Extremists Phase-Explanation


unacademy.com | Download the Unacademy app
Give your feedback here: Link
Phases of Revolutionary
5 Nationalism
1. Consider the following freedom fighters: 2. The Ghadr (Ghadar) was a: (2014)
(2022) (a) Revolutionary association of Indians
1. Barindra Kumar Ghosh with headquarters at San Francisco
2. Jogesh Chandra Chatterjee (b) Nationalist organization operating from
3. Rash Behari Bose Singapore

Who of the above was/were actively (c) Militant organization with headquarters
associated with the Ghadar Party? at Berlin

(a) 1 and 2 (d)


Communist movement for India’s
freedom with headquarters at Tashkent
(b) 2 only
(c) 1 and 3
(d) 3 only

Phases of Revolutionary Nationalism 33


unacademy.com | Download the Unacademy app
Give your feedback here: Link
Phases of Revolutionary
5 Nationalism-Explanation
1. Answer: (d) JOGESH CHANDRA CHATTERJEE-: ANUSHILAN
Option d is correct: SAMITI
RASH BEHARI BOSE: GHADAR PARTY

2. Answer: (a)
Option (a) is correct: Ghadar Party was an
Indian revolutionary organization, which was
formed in 1913 in San Francisco, the United
States, by migrant Indians with the aim to
liberate India from British rule. The party was
established as the Hindi Association of Pacific
Coast under the leadership of Lala Har Dayal
BARINDRA KUMAR GHOSH: ANUSHILAN SAMITI with Baba Sohan Singh Bhakna as its president.

34 Phases of Revolutionary Nationalism-Explanation


unacademy.com | Download the Unacademy app
Give your feedback here: Link
6 The Beginning of Gandhian Era

1. In the Government of India Act 1919, the (c) Mohandas Karamchand Gandhi
functions of Provincial Government were (d) Sarojini Naidu
divided into “Reserved” and “Transferred”
subjects. Which of the following were
treated as “Reserved” subjects? (2022) 4. The Gandhi-Irwin Pact included which of
1. Administration of Justice the following? (2020)

2. Local Self-Government 1. Invitation to Congress to participate in


the Round Table Conference
3. Land Revenue
2. Withdrawal of Ordinances promulgated
4. Police in connection with the Civil Disobedience
Select the correct answer using the code Movement
given below: 3. Acceptance of Gandhi’s suggestion for
(a) 1, 2 and 3 enquiry into police excesses
(b) 2, 3 and 4 4. Release of only those prisoners who
(c) 1, 3 and 4 were not charged with violence

(d) 1, 2 and 4 Select the correct answer using the code


given below:
(a) 1 only
2. Consider the following statements: (2021)
(b) 1, 2 and 4 only
1. The Montagu-Chelmsford Reforms of
1919 recommended granting voting (c) 3 only
rights to all women above the age of 21. (d) 2, 3 and 4 only
2. The Government of India Act of 1935
gave women reserved seats in the 5. With reference to the British colonial rule
legislature. in India, consider the following statements:
Which of the statements given above is/are (2019)
correct? 1. Mahatma Gandhi was instrumental in the
(a) 1 only abolition of the system of ‘indentured
(b) 2 only labour’.

(c) Both 1 and 2 2. In Lord Chelmsford’s ‘War Conference’,


Mahatma Gandhi did not support the
(d) Neither 1 nor 2
resolution on recruiting Indians for
World War.
3. Who among the following is associated 3. Consequent upon the breaking of
with ‘Songs from Prison’, a translation of Salt Law by Indian people, the Indian
ancient Indian religious lyrics in English? National Congress was declared illegal
(2021) by the colonial rulers.
(a) Bal Gangadhar Tilak Which of the statements given above are
(b) Jawaharlal Nehru correct?

The Beginning of Gandhian Era 35


unacademy.com | Download the Unacademy app
Give your feedback here: Link
(a) 1 and 2 only 9. In the federation established by the
(b) 1 and 3 only Government of India Act of 1935, residuary
powers given to the (2018)
(c) 2 and 3 only
(a) Federal Legislature
(d) 1, 2 and 3
(b) Governor General
(c) Provincial Legislature
6. With reference to the Swadeshi Movement,
consider the following statements: (2019) (d) Provincial Governors

1. It contributed to the revival of the


indigenous artisan crafts and industries. 10. The Trade Disputes Act of 1929 provided
2. The National Council of Education was for (2017)
established as a part of the Swadeshi (a) The participation of workers in the
Movement. management of industries.
Which of the statements given above is/are (b) Arbitrary powers to the management to
correct? quell industrial disputes.
(a) 1 only (c) An intervention by the British Court in
(b) 2 only the event of a trade dispute.

(c) Both 1 and 2 (d) A system of tribunals and a ban on


strikes.
(d) Neither 1 nor 2

11. In the context of Indian history, the


7. Which one of the following is a very principle of `Dyarchy (diarchy)’ refers to
significant aspect of the Champaran (2017)
Satyagraha? (2018)
(a) Division of the central legislature into
(a) Active all-India participation of lawyers, two houses.
students and women in the National
Movement (b) Introduction of double government, i.e.,
Central and State governments.
(b) Active involvement of Dalit and Tribal
communities of India in the National (c) Having two sets of rulers, one in London
Movement and another in Delhi.

(c) Joining of peasant unrest to India's (d) Division of the subjects delegated to
National Movement the provinces into two categories.

(d) Drastic decrease in the cultivation of


plantation crops and commercial crops 12. The object of the Butler Committee of 1927
was to? (2017)

8. In 1920, which of the following changed its (a) Define the jurisdiction of the Central
name to "Swarajya Sabha"? (2018) and Provincial Governments.

(a) All India Home Rule League (b) Define the powers of the Secretary of
State for India.
(b) Hindu Mahasabha
(c) Impose censorship on the national
(c) South Indian Liberal Federation press.
(d) The Servants of India Society (d) Improve the relationship between the

36 The Beginning of Gandhian Era


unacademy.com | Download the Unacademy app
Give your feedback here: Link
Government of India and the Indian given below:
States. (a) 1 only
(b) 1 and 2 only
13. The Montague-Chelmsford Proposals were (c) 2 and 3 only
related to (2016)
(d) 1, 2 and 3
(a) Social reforms
(b) Education reforms
17. With reference to the Congress Socialist
(c) Reforms in public administration Party, consider the following statements:
(d) Constitutional reforms (2015)
1. It advocated the boycott of British
14. The Government of India Act of 1919 clearly goods and evasion of taxes.
defined (2015) 2. It wanted to establish the dictatorship
(a) the separation of power between the of the proletariat.
judiciary and the legislature 3. It advocated separate electorate for
(b) the jurisdiction of the central and minorities and oppressed classes.
provincial governments Which of the statements given above is/are
(c) the powers of the Secretary of State for correct?
India and the Viceroy (a) 1 and 2 only
(d) None of the above (b) 3 only
(c) 1, 2 and 3
15. Who of the following organised a march on (d) None of the above
the Tanjore coast to break the Salt Law in
April 1930? (2015)
18.
The 1929 Session of Indian National
(a) V.O Chidambaram Pillai Congress is of significance in the history of
(b) C. Rajagopalachari the Freedom Movement because the (2014)
(c) K. Kamaraj (a) attainment of Self-Government was
(d) Annie Besant declared as the objective of the
Congress
(b)
attainment of Poorna Swaraj was
16. With reference to Rowlatt Satyagraha, adopted as the goal of the Congress
which of the following statements is/are
correct? (2015) (c) Non-Cooperation Movement was
launched
1. The Rowlatt Act was based on the
recommendations of the ‘Sedition (d) decision to participate in the Round
Committee’. Table Conference in London was taken

2. In Rowlatt Satyagraha Gandhiji tried to


use the Home Rule League. 19. The people of India agitated against the
3. Demonstrations against the Simon arrival of the Simon Commission because
Commission coincided with the Rowlatt (2013)
Satyagraha. (a) Indians never wanted the review of the
Select the correct answer using the code working of the Act of 1919

The Beginning of Gandhian Era 37


unacademy.com | Download the Unacademy app
Give your feedback here: Link
(b) Simon Commission recommended the 3. A resolution was passed rejecting the
abolition of Dyarchy (Diarchy) in the two-nation theory in that Session.
provinces Which of the statement(s) given above is/
(c) there was no Indian member in the are correct?
Simon Commission (a) 1 only
(d) the Simon Commission suggested the (b) 2 and 3 only
partition of the country
(c) 1 and 3 only
(d) None of the above
20. Which of the following is/are the principal
feature(s) of the Government of India Act,
1919? (2012) 23. The Congress ministries resigned in the
1. Introduction of diarchy in the executive seven provinces in 1939, because (2012)
government of the provinces (a) The Congress could not form ministries
2. Introduction of separate communal in the other four provinces.
electorates for Muslims (b) Emergence of a ‘left wing’ in the
3. Devolution of legislative authority by Congress made the working of the
the center to the provinces ministries impossible.

Select the correct answer using the code (c)


There were widespread communal
given below: disturbances in their provinces.

(a) 1 only (d) None of the statements (a), (b) and (c)
given above is correct.
(b) 2 and 3 only
(c) 1 and 3 only
24. Mahatma Gandhi said that some of his
(d) 1, 2 and 3 deepest convictions were reflected in
a book titled, “Unto this Last” and the
21. The Rowlatt Act aimed at: (2012) book transformed his life. What was the
message from the book that transformed
(a) Compulsory economic support to war
Mahatma Gandhi? (2011)
efforts.
(a) Uplifting the oppressed and poor is the
(b) Imprisonment without trial and
moral responsibility of an educated man
summary procedures for trial.
(b) The good of individual is contained in
(c) Suppression of the Khilafat Movement.
the good of all
(d) Imposition of restrictions on freedom of
(c) The life of celibacy and spiritual pursuit
the press.
are essential for a noble life
(d) All the statements (a), (b) and (c) a r e
22. The Lahore Session of the Indian National correct in this context
Congress (1929) is very important in history,
because (2012)
25.
With reference to the period of India
1. The Congress passed a resolution
freedom struggle, which of the following
demanding complete independence.
was/were recommended by the Nehru
2. The rift between the extremists and Report? (2011)
moderates was resolved in that Session.
1. Complete Independence for India.

38 The Beginning of Gandhian Era


unacademy.com | Download the Unacademy app
Give your feedback here: Link
2. Joint electorates for reservation of peasants of Kheda? (2011)
seats for minorities. 1. The Administration did not suspend the
3. Provision of fundamental rights for the land revenue collection in spite of a
people of India in the Constitution. drought.
Select the correct answer using the code 2. The Administration proposed to
given below: introduce Permanent Settlement in
(a) 1 only Gujarat.

(b) 2 and 3 only Which of the statements given above is/are


correct?
(c) 1 and 3 only
(a) 1 only
(d) 1, 2 and 3
(b) 2 only
(c) Both 1 and 2
26. What was the reason for Mahatma Gandhi
to organise a satyagraha on behalf of the (d) Neither 1 nor 2

The Beginning of Gandhian Era 39


unacademy.com | Download the Unacademy app
Give your feedback here: Link
The Beginning of Gandhian Era-
6 Explanation
1. Answer: (c) on 5 March 1931. The pact placed the Congress
Option (c) is correct: Justice, the police, on an equal status with the Government.
land revenue, and irrigation were among the The terms of the agreement included
reserved subjects under the category of law immediate release of all political prisoners not
and order. Local self-government, education, convicted of violence, remission of all fines
public health, public works, agriculture, forests, not yet collected, withdrawal of emergency
and fisheries were among the transferred ordinances promulgated in connection with
topics (i.e., those under the control of Indian the Civil Disobedience Movement, return of all
ministers). lands not yet sold to third parties, to name a
few. The viceroy, however, turned down two
of Gandhi’s demands which included public
2. Answer: (b) inquiry into police excesses and commutation
Statement 1 is not correct: In line with the of Bhagat Singh and his comrades’ death
government policy contained in Montagu’s sentence to life sentence. Gandhi on behalf
statement of August 1917, the government of the Congress agreed to suspend the Civil
announced further constitutional reforms in Disobedience Movement and the Congress got
July 1918, known as Montagu-Chelmsford or the invitation to participate in the next Round
Montford Reforms. Women were also given Table Conference.
the right to vote, but it was not granted to
all women. It was not universal suffrage. It
5. Answer: (b)
granted a franchise to a limited number of
people based on property, tax or education. Statement 1 is correct: Indentured labour was
bonded labour hired on contract for working
Statement 2 is correct: The Government
on plantations in various colonies of western
of India Act 1935 extended the principle of
imperialists. Since India was going through
communal representation by providing separate
agricultural distress & stagnation, many farmers
electorates for depressed classes (Scheduled
ended up into the system of indentured labour
Castes), women and labour (workers). Women
because of luring advances & good salaries in
can also fight in general seats. So, this act gave
foreign states. A mass meeting was organised
women reserved seats in the legislature.
by Mahatma Gandhi on September 11, 1906 at
the Imperial Theatre in Johannesburg, where
3. Answer: (c) 3,000 people pledged to disregard the law.

Option (c) is correct: During his imprisonment at Statement 2 is not correct: Lord Chelmsford,
Yerwada Prison in 1930, Mohandas Karamchand the then viceroy of India invited Gandhi to Delhi,
Gandhi Ji translated hymns and lyrics from at an award conference in order to gain the
the Upanishads and other sacred texts. These trust of the empire and Gandhi agreed to move
texts were known as the ‘Songs from Prison’. people to enlist in the army for World War I. He
agreed to the resolution with the belief that
absolutely unconditional and whole-hearted
4. Answer: (b) cooperation with the government on the part
Option (b) is correct: Gandhi-Irwin Pact was of educated India will bring us within sight of
signed by Gandhiji on behalf of the Congress our goal of Swaraj as nothing else will’.
and by Lord Irwin on behalf of the Government, Statement 3 is correct: At the Calcutta congress

40 The Beginning of Gandhian Era-Explanation


unacademy.com | Download the Unacademy app
Give your feedback here: Link
of 1928, Gandhi declared that the British must Movement that opened a new phase in the
grant dominion status to India otherwise a national movement by joining it to the great
revolution for complete independence would struggle of the Indian peasantry for bread and
blow up in the country. The Salt March or the land. Gandhi led the struggle of indigo workers
Dandi March commenced on March 12th, 1930 as India’s first Satyagraha that set the pace for
and extended for a period of 24 days. The involvement of peasant unrest to the National
march was from Sabarmati Ashram to Dandi in Movement.
Gujarat. The salt law was broken by Gandhi on The Montagu-Chelmsford Reforms introduced
April 6th when he collected salt from mud. The Dyarchy in the provinces with the Government
police resorted to their usual brutal methods of Indian Act, 1919. The Moderates welcomed
in dealing with the breakers of the salt law these reforms, while the Extremists rejected
and the Indian National Congress was declared them. The Rowlatt Act was also passed in 1919
illegal. to suppress political violence. At this juncture
appeared a new face in the political arena of
6. Answer: (c) India’s freedom movement. This was Gandhi,
who filled the vacuum created in the top
The Swadeshi Movement in Bengal was
leadership of the Congress. Gandhi had led
announced by the Indian National Congress
the movement against discrimination meted
against the announcement of the partition of
out to Indians in South Africa. He had used the
Bengal in July 1905 by Lord Curzon. Launched
political weapon called Satyagraha (Truth force,
as a protest movement, it also paved the way
or Love force, or Soul force). His first triumph
for the Boycott movement in the country which
in India was the Champaran Satyagraha.
led to the use of goods produced in India.
Statement 1 is correct: The Swadeshi
Movement saw the growth of indigenous 8. Answer: (a)
industries by the establishment of swadeshi Option (a) is correct: The year 1919 opened
textile mills, soap and match factories, with two main political bodies in the field, viz.,
tanneries, banks, insurance companies, shops, the All India Home Rule League established in
etc. This inculcated a feeling of self-reliance 1916 mainly by Mrs. Besant and Tilak’s Indian
by reducing the dependence on foreign goods. Home Rule League started in 1917. The Home
These enterprises were based more on patriotic Rule League demanded self-government
zeal than on business acumen. based on the Irish model. Tilak’s League with
Statement 2 is correct: As an impact of the its stronghold in the Deccan. Mrs. Besant
movement, the demand for the establishment on the other hand had for some time been
of national education reached its peak which losing ground everywhere. All India Home Rule
resulted in the setting up of various national League which in October 1920 was renamed
schools and colleges. To impart education the “Swarajya Sabha” with the avowed object
through the vernacular medium, the National of securing complete Swaraj for India in
Council of Education was also set up on August accordance with the wishes of the people of
15, 1906 to organise a system of education— India.
literary, scientific, and technical—on national
lines and under national control. 9. Answer: (b)
Option (b) is correct: The Government of India
7. Answer: (c) Act of 1935 provided for the establishment of
Option (c) is correct: The Champaran Satyagraha an All-India Federation consisting of Provinces
of 1917 was India’s first Civil Disobedience and Princely States as units. The Act divided

The Beginning of Gandhian Era-Explanation 41


unacademy.com | Download the Unacademy app
Give your feedback here: Link
the powers between the Centre and units in 11. Answer: (d)
terms of three lists, Federal List (for Centre, Dyarchy was a system of double government
with 59 items), Provincial List (for provinces, introduced by the Government of India Act
with 54 items) and the Concurrent List (for (1919) for the provinces of British India. It
both, with 36 items). Residuary powers were marked the first introduction of the democratic
given to the Governor General. principle into the executive branch of the
The Government of India Act of 1935 abolished British administration of India. Though much-
dyarchy in the provinces and introduced criticised, it signified a breakthrough in British
‘provincial autonomy’ in its place and Indian government and was the forerunner
introduced bicameralism in six out of eleven of India’s full provincial autonomy (1935) and
provinces. Thus, the legislatures of Bengal, independence (1947). Dyarchy was introduced
Bombay, Madras, Bihar, Assam and the United as a Constitutional reform by Edwin Samuel
Provinces were made bicameral consisting of a Montagu (secretary of state for India, 1917–22)
legislative council (upper house) and a legislative and Lord Chelmsford (viceroy of India, 1916–21).
assembly (lower house), further extended Option (d) is correct: The principle of dyarchy
the principle of communal representation by was a division of the executive branch of each
providing separate electorates for depressed provincial government into authoritarian and
classes (scheduled castes), women and labour popularly responsible sections. The first was
(workers). It abolished the Council of India, composed of executive councillors, appointed,
established by the Government of India Act as before, by the crown. The second was
of 1858. The secretary of state for India was composed of ministers who were chosen by
provided with a team of advisors. It provided the governor from the elected members of
for the establishment of a Reserve Bank of the provincial legislature. The various fields,
India to control the currency and credit of the or subjects of administration were divided
country and provided for the establishment of between the councillors and the ministers,
not only a Federal Public Service Commission being named reserved and transferred subjects,
but also a Provincial Public Service Commission respectively. The reserved subjects came under
and Joint Public Service Commission for two the heading of law and order and included
or more provinces. justice, the police, land revenue, and irrigation.
The transferred subjects (i.e., those under the
10. Answer: (d) control of Indian ministers) included local self-
government, education, public health, public
Option (d) is correct: The main objective of
works, and agriculture, forests, and fisheries.
the Trade Dispute Act 1929 Act was to make
The system ended with the introduction of
provisions for the establishment of Courts of
provincial autonomy in 1935.
Inquiry and Boards of Conciliation (system of
tribunals) with a view to investigate and settle
trade disputes. The Act prohibited strikes 12. Answer: (d)
or lock-outs without notice in public utility Option (d) is correct: Harcourt Butler
services. Committee or Butler Commission was a three-
It also made any strike or lock-out illegal, member committee headed by Harcourt Butler,
which had any object other than advocating appointed on December 16, 1927, to examine
a trade dispute within the trade or industry. the relations between the native states and the
Prior to the year 1947 (India’s Independence), paramount power. The Committee was formed
the Trade Disputes Act, 1929 used to settle to inquire into the relationship between the
industrial disputes. Indian states and the paramount power and to
suggest ways and means for more satisfactory

42 The Beginning of Gandhian Era-Explanation


unacademy.com | Download the Unacademy app
Give your feedback here: Link
adjustments of the existing relations between Option (b) is correct: The Government of India
them and British India. The Committee visited Act 1919 clearly defined the jurisdiction of
16 States and submitted its report in 1929. the central and provincial governments in the
following ways:

13. Answer: (d) y It relaxed the central control over the


provinces by demarcating and separating
After Montagu took over as the Secretary
the central and provincial subjects. The
of State for India in July 1917, he made a
central and provincial legislatures were
historic declaration that British policy in India
authorised to make laws on their respective
would have an overall objective of “gradual
list of subjects. However, the structure of
development of self-governing institutions,
government continued to be centralised
with a view to the progressive realisation of
and unitary.
responsible government in India as an integral
part of the British empire”. y It further divided the provincial subjects
into two parts– transferred and reserved.
Option (d) is correct: The Montford (Montague-
The transferred subjects were to be
Chelmsford) proposals were related to
administered by the Governor with the aid
Constitutional reforms. The Montford
of Ministers responsible to the legislative
(Montague-Chelmsford) commission submitted
council. The reserved subjects, on the
its report in 1918. It professed to give a way
other hand, were to be administered by the
for self-government in India. However, it also
Governor and his executive council without
aimed at appeasing Indians to persuade them
being responsible to the legislative council.
to support the British during the First World
This dual scheme of governance was known
War (1914-18). This reform was important in
as ‘dyarchy’–which means a dual system of
a way that it was the first-time government
government.
showed its intention of gradual introduction of
responsible government in India. y It introduced bicameralism and direct
elections in the country. Thus, the Indian
It established parliamentary democracy in
legislative council was replaced by a
India and began the process of decolonisation.
bicameral legislature consisting of an Upper
Provisions of the Government of India Act of
House (Council of State) and a Lower House
1919 included Provincial Diarchy in the form
(Legislative Assembly). Most members of
of a dual form of government for the major
both the Houses were chosen by direct
provinces. It relaxed control over provinces by
election.
demarcating subjects as ‘central subjects’ and
‘provincial subjects. y It also separated, for the first time,
provincial budgets from the Central budget
and authorized the provincial legislatures
14. Answer: (b) to enact their budgets.
On August 20, 1917, the British Government
declared, for the first time, that its objective
15. Answer: (b)
was the gradual introduction of a responsible
Government in India. The Government of India On March 12, 1930, Mahatma Gandhi embarked
Act of 1919 was thus enacted, which came on a historic Salt March from Sabarmati Ashram
into force in 1921. This Act is also known as in Gujarat’s Ahmedabad to the village of Dandi
Montagu-Chelmsford Reforms (Montagu was in the state’s coastal area to protest the steep
the Secretary of State for India and Lord tax the British levied on salt. The Salt March
Chelmsford was the Viceroy of India). began on 12th March 1930 and continued till
6th April 1930. Upon reaching the seashore

The Beginning of Gandhian Era-Explanation 43


unacademy.com | Download the Unacademy app
Give your feedback here: Link
in Dandi, Mahatma Gandhi broke the law by 1934. The rise of this party was due to the
producing illegal salt. increased left influence in the Indian National
Option (b) is correct: C Rajagopalachari (the Congress. By 1935, one-third of the Congress
last Governor-General of India and had served members were Congress Socialists.
India during the freedom struggle) organized a y These leaders rejected the idea of Gandhi
March on the Tanjore coast to break the Salt (which they saw as anti-rational). Though
Law in April 1930. He was arrested in April 1930 they remained active in the workers and
for leading a salt march from Trichinopoly to peasant’s movement, they rejected the
Vedaranniyam on the Tanjore coast. sectarian attitude of the Communist Party
of India. It did not advocate the boycott
of British goods and the evasion of taxes.
16. Answer: (b) (Statement 1 is not correct)
Statement 1 is correct: Rowlatt Act was passed y They believed in Marxist Ideas, Liberal
on the recommendations of the Sedition and Social democracy of the west with
Committee Chaired by Sir Sidney Rowlatt. Nationalism and Independence as their
y This Act had been hurriedly passed in the goal. It advocated decentralised socialism
Imperial Legislative Council despite the in which co-operatives, trade unions,
united opposition of the Indian members. independent farmers, and local authorities
The purpose of the Committee was to would hold a substantial share of the
evaluate political terrorism in India, economic power. (Statement 2 is not
especially Bengal and Punjab. It gave the correct)
government enormous powers to repress y The CSP was not separate from Congress.
political activities and allowed the detention Its Constitution defined that the members
of political prisoners without trial for two were all required to be members of the Indian
years. National Congress. Members of communal
y In organizing his Satyagraha, Gandhi tried organisations or political organisations,
to utilize three types of political networks whose goals were incompatible with
- The Home Rule Leagues, Certain pan- the ones of CSP, were barred from CSP
Islamist groups and a Satyagraha Sabha membership. As secularists, they hoped to
which he himself started in Bombay on 24 surpass communal divisions through class
February. (Statement 2 is correct) solidarity. (Statement 3 is not correct)
y Rowlatt Satyagraha occurred in 1919 while
Simon Commission arrived in India in 1928. 18. Answer: (b)
(Statement 3 is not correct)
Option (b) is correct: In 1929 the Congress
Elimination technique: Rowlatt Satyagraha session was held at Lahore. This session was
occurred in 1919 while Simon Commission very significant because in the Lahore session
arrived in India in 1928. So Statement 3 is the prominent party Indian National Congress,
incorrect. took the resolution of Poorna Swaraj for
complete independence. In this, the Indian tri-
colour flag was hoisted by Pandit Jawahar Lal
17. Answer: (d)
Nehru on the bank of the Ravi River.
Congress Socialist Party, or (CSP), was a left-
The following major decisions were taken at
wing group within the Congress. It was formed
the Lahore Session:
with Acharya Narendra Deva as President and
Jay Prakash Narayan as General Secretary in y The Round Table Conference was to be
boycotted.

44 The Beginning of Gandhian Era-Explanation


unacademy.com | Download the Unacademy app
Give your feedback here: Link
y Complete independence or Poorna Swaraj 20. Answer: (c)
was declared as the aim of the Congress. On August 20, 1917, the British Government
y Congress Working Committee was declared, for the first time, that its objective
authorised to launch a programme of civil was the gradual introduction of a responsible
disobedience including non-payment of Government in India. The Government of India
taxes and all members of legislatures were Act of 1919 was thus enacted, which came
asked to resign their seats. into force in 1921. This Act is also known as
y January 26, 1930 was fixed as the first Montagu-Chelmsford Reforms (Montagu was
Independence (Swarajya) Day, to be the Secretary of State for India and Lord
celebrated everywhere. Chelmsford was the Viceroy of India).

y The Nehru report was declared to be null Statement 1 is correct: The Government of
and void. India Act 1919 divided the provincial subjects
into two parts—transferred and reserved. The
transferred subjects were to be administered
19. Answer: (c) by the governor with the aid of ministers
The Government of India Act, 1919 had a provision responsible to the legislative Council. The
that a Commission would be appointed ten reserved subjects, on the other hand, were
years from date to study the progress of the to be administered by the governor and his
governance scheme and suggest new steps. executive council without being responsible
An all-white, seven-member Indian Statutory to the legislative Council. This dual scheme of
Commission, popularly known as the Simon governance was known as ‘dyarchy’- double
Commission (after the name of its chairman, rule.
Sir John Simon), on 8th November 1927. Statement 2 is not correct: It extended the
Option (c) is correct: The Commission was to principle of communal representation by
recommend to the British government whether providing separate electorates for Sikhs, Indian
India was ready for further constitutional Christians, Anglo-Indians, and Europeans.
reforms and along what lines. The Indian Separate electorate was introduced for the
response to the Simon Commission was first time in 1909 for Muslims.
immediate and nearly unanimous. What Statement 3 is correct: It relaxed the central
angered the Indians most was the exclusion control over the provinces by demarcating and
of Indians from the Commission and the basic separating the central and provincial subjects.
notion behind the exclusion that foreigners The central and provincial legislatures were
would discuss and decide upon India’s fitness authorized to make laws on their respective
for self-government. list of subjects. However, the structure of
government continued to be centralised and
Elimination Technique
unitary.
Eliminating options in this question is
Other Features of the Government of India Act
not easy because options are very close
1919:
and analytical. Still, the options can
be eliminated if one has the idea of the y It introduced, for the first time, bicameralism
Government of India Act of 1919. and direct elections in the country. Thus,
the Indian legislative council was replaced
y As we know, it was decided in the Government
by a bicameral legislature consisting of
of India Act of 1919 that a commission will be
an Upper House (Council of State) and a
set up every 10 years to assess the working
Lower House (Legislative Assembly). The
of the Act. And, the Simon Commission was
majority of members of both the Houses
created for this.

The Beginning of Gandhian Era-Explanation 45


unacademy.com | Download the Unacademy app
Give your feedback here: Link
were chosen by direct election.
carefully, we will get a clue -related to
y It required that the three of the six members
division of power or authority.
of the Viceroy’s executive Council (other
than the Commander-in-Chief) were to be o Statement 1 talks about diarchy
Indian. which means government by
two independent authorities and
y It extended the principle of communal
statement 2 talks about devolution
representation by providing separate
of legislative authority by the center
electorates for Sikhs, Indian Christians,
to the provinces.
Anglo-Indians and Europeans.
y As both the statements are very close
y It granted franchise to a limited number
to that clue. We can easily mark Option
of people on the basis of property, tax or
(c) as the correct answer.
education.
y It created a new office of the High
Commissioner for India in London and 21. Answer: (b)
transferred to him some of the functions The popularly known Rowlatt Act was officially
hitherto performed by the Secretary of called the Anarchical and Revolutionary Crimes
State for India. Act. It was based on the recommendations
y It provided for the establishment of a public made in the previous year to the Imperial
service commission. Hence, a Central Public Legislative Council by the Rowlatt Commission,
Service Commission was set up in 1926 for headed by the British judge, Sir Sidney Rowlatt,
recruiting civil servants. to investigate the ‘seditious conspiracy’ of the
Indian people.
y It separated, for the first time, provincial
budgets from the Central budget and Option (b) is correct: The Rowlatt committee
authorised the provincial legislatures to had recommended that activists should be
enact their budgets. deported or imprisoned without trial for two
years, and that even possession of seditious
y It provided for the appointment of a
newspapers would be adequate evidence of
statutory commission to inquire into and
guilt. The act allowed political activists to be
report on its working after ten years of its
tried without juries or even imprisoned without
coming into force.
trial. It allowed the arrest of Indians without
Elimination Technique warrant on the mere suspicion of ‘treason’.
y In this question, statement 1 and Such suspects could be tried in secrecy
statement 3 are closely related. But without recourse to legal help.
statement 3 is a little confusing. If All the elected Indian members of the Imperial
we can recall then we will find that Legislative Council voted against the bill, but
introduction of separate communal they were in a minority and easily overruled
electorates for Muslims is one of the by the official nominees. All the elected Indian
recommendations of the Morley-Minto members—who included Mohammed Ali
reforms. So, statement 2 can be easily Jinnah, Madan Mohan Malaviya and Mazhar Ul
eliminated along with option (b) and Haq – resigned in protest.
option (d).
Elimination Technique
y After eliminating Statement 2, we are
Rowlett Act is a very famous event in
left with statement 1 and statement
modern Indian history. This question
3. If we read in between the lines
is directly asking about the aim of the

46 The Beginning of Gandhian Era-Explanation


unacademy.com | Download the Unacademy app
Give your feedback here: Link
Rowlatt Act. If any aspirant is not aware Elimination Technique
about the event, still he/she can easily hit
y It is very well known that in the Lahore
the question.
Session of 1929, the INC passed
y The Rowlatt Act is related to the a resolution demanding complete
Jallianwala Bagh Massacre. So, independence. So, option (d) and
option (a) and option (c) can be easily option (b) can be easily eliminated.
eliminated.
y Though, statement 3 remains unclear.
y After eliminating option (a) and Option But if we can recall the period of
(c), confusion still remains with option extremist and moderate leadership, we
(b) and Option (d). But if we read the find that-
statements carefully, we will find that
y Period of moderate leadership
the Jallianwala Bagh Massacre was
(1885-1905)
a disastrous event in Indian modern
history, a mere restriction on freedom y Period of extremist leadership
of press may not result in such events. (1905-1920)
Therefore, Option (b) is the correct y So, statement 3 or option (c) can be
answer. easily eliminated and the answer is
Option (a).

22. Answer: (a)


Jawaharlal Nehru, who had done more than 23. Answer: (d)
anyone else to popularise the concept of In February 1937, elections to the provincial
Purna Swaraj, was nominated the president for assemblies were held. Congress ministries
the Lahore session of the Congress (December were formed in Bombay, Madras, Central
1929) mainly due to Gandhi’s backing (15 out Provinces, Orissa, United Provinces, Bihar and
of 18 Provincial Congress Committees had later in the Northwest Frontier Province (NWFP)
opposed Nehru). and Assam also and won 716 out of 1,161 seats
Option (a) is correct: The following major it contested. (There were 1,585 seats in the
decisions were taken at the Lahore session: legislative assemblies of the eleven provinces).
It got a majority in all provinces, except in
y The Round Table Conference was to be
Bengal, Assam, Punjab, Sindh and the NWFP,
boycotted.
and emerged as the largest party in Bengal,
y Complete independence was declared as Assam and the NWFP.
the aim of the Congress.
Option (d) is correct: At the outbreak of
y Congress Working Committee was World War II, the Viceroy announced India’s
authorised to launch a programme of civil involvement without consulting the main
disobedience including non-payment of political parties. Congress demanded an
taxes and all members of legislatures were immediate transfer of power in return for
asked to resign their seats. cooperation in the war, however the British
y January 26, 1930, was fixed as the first Government refused. As a result, the Congress
Independence (Swarajya) Day, to be ministries resigned from power on December
celebrated everywhere. 22, 1939. Jinnah asked the Muslims to celebrate
this day as Day of Deliverance.
The rift between the extremists and moderates
was resolved in the 1916 Lahore Session of the
Congress.

The Beginning of Gandhian Era-Explanation 47


unacademy.com | Download the Unacademy app
Give your feedback here: Link
24. Answer: (b) in proportion to the Muslim population
The main teachings of “Unto This Last”: there with right to contest additional seats.
(Statement 2 is correct)
y That the good of the individual is contained
in the good of all. [Option (b) is correct] y Linguistic provinces.

y That a lawyer’s work has the same value y Nineteen fundamental rights including
as the barber’s in as much as all have the equal rights for women, right to form unions,
same right of earning their livelihood from and universal adult suffrage. (Statement 3
their work. is correct)

y That a life of labour, that is, the life of the y Responsible government at the Centre and
tiller of the soil and the handicraftsman is in provinces.
the life worth living. y Full protection of the cultural and religious
interests of Muslims.

25. Answer: (b) y Complete dissociation of State from


religion.
As an answer to Lord Birkenhead’s challenge,
an All-Parties Conference met in February y Hindi to be made the official language of
1928 and appointed a sub-committee under India.
the chairmanship of Motilal Nehru to draft a Elimination Technique: Dominion status
constitution. This was the first major attempt by on lines of self-governing dominions as the
the Indians to draft a constitutional framework form of government was demand of the
for the country. Nehru report. Statement 1 is eliminated.
The Committee included Tej Bahadur Sapru, Thus, Option (b) is the correct answer.
Subhash Bose, M.S.Aney, Mangal Singh, Ali
Imam, Shuaib Qureshi and G.R.Pradhan as its
members. The report was finalised by August 26. Answer: (a)
1928. Statement 1 is correct: Due to drought in 1918,
Statement 1 is not correct: In the December the crops failed in Kheda district of Gujarat.
1929 session of the Congress, presided by According to the Revenue Code, if the yield was
Jawaharlal Nehru, the Congress declared less than one-fourth of the normal produce, the
“Purna Swaraj” or Complete independence as farmers were entitled to remission, however
its ultimate goal. the government refused.

The Nehru Report confined itself to British India, Statement 2 is not correct: The Gujarat
as it envisaged the future link-up of British India Sabha, consisting of the peasants, submitted
with the princely states on a federal basis. For petitions to the highest governing authorities
the dominion, it recommended: of the province requesting that the revenue
assessment for the year 1919 be suspended.
y Dominion status on lines of self-governing The government, however, remained adamant
dominions as the form of government and said that the property of the farmers would
desired by Indians. be seized if the taxes were not paid. Gandhi
y Rejection of separate electorates, instead, asked the farmers not to pay the taxes.
a demand for joint electorates with y It was Sardar Vallabhbhai Patel and a
reservation of seats for Muslims at the group of other devoted Gandhians, namely,
Centre and in provinces where they were in Narahari Parikh, Mohanlal Pandya and Ravi
minority (and not in those where Muslims Shankar Vyas, who went around the villages
were in majority, such as Punjab and Bengal) and gave the necessary political leadership.

48 The Beginning of Gandhian Era-Explanation


unacademy.com | Download the Unacademy app
Give your feedback here: Link
The National Movement in the
7 1940s
1. With reference to 8th August 1942 in 1. Mutiny in Royal Indian Navy
Indian history, which one of the following 2. Quit India Movement launched
statements is correct? (2021)
3. Second Round Table Conference
(a) The Quit India Resolution was adopted
by the AICC. What is the correct chronological sequence
of the above events?
(b) The Viceroy’s Executive Council was
expanded to include more Indians. (a) 1-2-3

(c) The Congress ministries resigned in (b) 2-1-3


seven provinces. (c) 3-2-1
(d) Cripps proposed an Indian Union with (d) 3-1-2
fall Dominion Status once the Second
World War was over.
5. The plan of Sir Stafford Cripps envisaged
that after the Second World War: (2016)
2. In the context of Colonial India, Shah Nawaz (a)
India should be granted complete
Khan, Prem Kumar Sehgal and Gurbaksh independence
Singh Dhillon are remembered as (2021)
(b) India should be partitioned into two
(a)
leaders of Swadeshi and Boycott before granting independence
Movement
(c) India should be made a republic with
(b) members of the Interim Government in the condition that she will join the
1946 Commonwealth
(c) members of the Drafting Committee in (d) India should be given Dominion status
the Constituent Assembly
(d) officers of the Indian National Army
6. With reference to Cabinet Mission, which
of the following statements is/are correct?
3. Who among the following were the founders (2015)
of the "Hind Mazdoor Sabha" established in 1. It recommended a federal government.
1948? (2018)
2. It enlarged the powers of the Indian
(a) B. Krishna Pillai, E.M.S. Namboodiripad courts.
and K.C. George
3. It provided for more Indians in the ICS.
(b)
Jayaprakash Narayan, Deen Dayal
Select the correct answer using the code
Upadhyay and M.N. Roy
given below:
(c) C.P. Ramaswamy Iyer, K. Kamaraj and
(a) 1 only
Veeresalingam Pantulu
(b) 2 and 3
(d) Ashok Mehta, T.S. Ramanujam and G.G.
Mehta (c) 1 and 3
(d) None of the above
4. With reference to Indian freedom struggle,
consider the following events: (2017)

The National Movement in the 1940s 49


unacademy.com | Download the Unacademy app
Give your feedback here: Link
7. The demand for the Tebhaga Peasant 9. With reference to Indian freedom struggle,
Movement in Bengal was for (2013) Usha Mehta is well-known for (2011)
(a) the reduction of the share of the (a) Running the secret Congress Radio in
landlords from one-half of the crop to the wake of Quit India Movement
one-third (b) Participating in the Second Round Table
(b) the grant of ownership of land to Conference
peasants as they were the actual (c) Leading a contingent of Indian National
cultivators of the land Army
(c) the uprooting of Zamindari system and (d) Assisting in the formation of Interim
the end of serfdom Government under Pandit Jawaharlal
(d) writing off all peasant debts Nehru

8. Quit India Movement was launched in 10. Which one of the following observations is
response to (2013) not true about the Quit India Movement of
(a) Cabinet Mission Plan 1942? (2011)

(b) Cripps Proposals (a) It was a nonviolent movement

(c) Simon Commission Report (b) It was led by Mahatma Gandhi

(d) Wavell Plan (c) It was a spontaneous movement


(d) It did not attract the labor class in
general

50 The National Movement in the 1940s


unacademy.com | Download the Unacademy app
Give your feedback here: Link
The National Movement in the
7 1940s-Explanation
1. Answer: (a) as the General Secretary.
Option (a) is correct: In July 1942, the Congress
Working Committee met at Wardha and 4. Answer: (c)
resolved that it would authorise Gandhi to take
charge of the non-violent mass movement. Option (c) is correct: The Second Round Table
The resolution is generally referred to as the Conference was held in London in December
‘Quit India’ resolution. It was to be approved 1931. The Congress had agreed to attend
by the All India Congress Committee meeting the second-round table conference under
in Bombay in August. the Delhi pact, and hence Mahatma Gandhi
attended the second-round table conference
On August 8, 1942, the Quit India Resolution in London. The purpose of the conference
was adopted and ratified at the Congress was to discuss the Constitutional reforms of
meeting at Gowalia Tank, Bombay, on August 8, India. However, it failed to reach an agreement,
1942. Mahatma Gandhi was named the leader either constitutionally or on communal
of the struggle. representation. The session got a standstill on
The movement demanded an end to British the question of the minorities. All minorities
rule in India. Since the protest was held in came together in a “Minorities’ Pact”.
August, it also went on to be known as August On 8th August 1942, Mahatma Gandhi gave a
Kranti or August Movement. blaring call to end British rule and launched
the Quit India Movement at the session of the
2. Answer: (d) All-India Congress Committee in Mumbai. The
immediate cause for the movement was the
Option (d) is correct: After the Second World
collapse of Cripps Mission. Gandhi gave the
War, the British captured about 23,000 Indian
call “Do or Die” in his speech delivered at the
National Army soldiers and charged them with
Gowalia Tank Maidan, now popularly known as
treason. The Red Fort became the venue for
August Kranti Maidan. The slogan ‘Quit India’
the trials of Indian National Army (INA) soldiers.
was coined by Yusuf Meherally, a socialist and
These trials are popularly called the “Red Fort
trade unionist who also served as Mayor of
trials”. In November 1945, the first three senior
Mumbai.
INA officials, Shah Nawaz Khan, Prem Kumar
Sehgal and Gurbaksh Singh Dhillon, were tried. The Junior Commissioned Officers (JCOs) and
It was asserted that they went against the Non-Commissioned Officers (NCOs) had faced
British crown by siding with the Japanese and many hardships during the Second World War,
fighting against the British in the war. and when the war was coming to an end, the
ratings were left with no certainty about their
jobs, so they revolted against it on February 18,
3. Answer: (d) 1946, which came to be known as The Royal
Option (d) is correct: The Hind Mazdoor Sabha Indian Navy (RIN) uprising.
(HMS) was founded in Howrah in West Bengal
on 29 December 1948, by socialists, Forward
5. Answer: (d)
Bloc followers and independent unionists. It
was founded by, Ashok Mehta, T.S. Ramanujam, Option (d) is correct: In March 1942, a mission
G.G. Mehta, and other members. R.S. Ruikar headed by Stafford Cripps was sent to India
was elected as the President and Ashok Mehta with constitutional proposals to seek Indian

The National Movement in the 1940s-Explanation 51


unacademy.com | Download the Unacademy app
Give your feedback here: Link
support for the Second World War. In return, two-thirds’ share to the bargardars, the share-
it proposed to enable elections and give croppers also known as bagchasi or adyar,
Dominion status once the war was over. Sir instead of the one-half share.
Stafford Cripps was a senior left-wing politician The bargardars worked on lands rented from
and government minister in the war cabinet of the Jotedars. The communist cadres, including
Prime Minister Winston Churchill. But Cripps many urban student militias, went to the
Mission was a total failure as the Indians did countryside to organise the bargadars. The
not agree to it. central slogan was “nij khamare dhan tolo” i.e.,
sharecroppers taking the paddy to their own
threshing floor and not to the jotedar’s house,
6. Answer: (a)
as before, so as to enforce tebhaga.
In February 1946, the British Prime Minister
The storm center of the movement was north
Clement Richard Attlee decided to send a
Bengal, principally among Rajbanshis, a low
high-powered mission of three British cabinet
caste of tribal origin. Muslims also participated
members (Pethick Lawrence, Secretary of
in large numbers. The movement dissipated
State for India; Stafford Cripps, President of
soon, because of the League ministry’s sop of
the Board of Trade; and A.V. Alexander, First
the Bargardari Bill, an intensified repression,
Lord of Admiralty) to India to find out ways
the popularisation of the Hindu Mahasabha’s
and means for a negotiated, peaceful transfer
agitation for a separate Bengal and renewed
of power to India. The high-powered mission
riots in Calcutta which ended the prospects of
is famously known as the Cabinet Mission.
sympathetic support from the urban sections.
Pethick Lawrence was the Chairman of the
Mission.
y The Cabinet Mission plan proposed a 8. Answer: (b)
weak Centre with Provincial autonomy, Option (b) is correct: On 8 August 1942, at
essentially proposing a federal structure of the All-India Congress Committee Session
government. The cabinet mission plan of in Bombay, Gandhi launched the ‘Quit India’
1946 also proposed that there shall be a Movement and called for ‘Do or Die’. The
Union of India which was to be empowered movement was in response to the failure of
to deal with the defence, foreign affairs, and the Cripps Mission to solve the constitutional
communications. [Option (a) is correct] deadlock exposed Britain’s unchanged attitude
y The Union Government and its legislature on constitutional advance and made it clear
were to have limited powers, dealing that any more silence would be tantamount to
with Finance, Foreign Affairs and accepting the British right to decide the fate of
Communications. The Union would have Indians without consulting them.
powers necessary to raise the finances Other reasons for launching the Quit India
to manage the subjects. All subjects Movement:
other than the Union subjects and all the
y There was popular discontent because of
residuary powers would be vested in the
rising prices and shortage of food. There
provinces.
were fears of Britain following a scorched
earth policy in Assam, Bengal and Orissa
7. Answer: (a) against possible Japanese advance.
Option (a) is correct: In September 1946, the y News of reverses suffered by the British in
Bengal Provincial Kisan Sabha gave a call to South-East Asia and an imminent British
implement, through mass struggle, the Flood collapse enhanced popular willingness to
Commission recommendations of Tebhaga, give expression to discontent.

52 The National Movement in the 1940s-Explanation


unacademy.com | Download the Unacademy app
Give your feedback here: Link
y The leadership wanted to condition the Committee, thereby initiating India into the
masses for a possible Japanese invasion. mass uprising that would eventually lead to
y The rout of a European power by an the nation’s freedom. Gandhi had been striving
Asian power shattered white prestige and for self-determination of Indians at different
the British behavior towards the Indian levels for the last several decades. However,
subjects in South-East Asia exposed the the significance of the ‘Quit India Movement’
racist attitude of the rulers. lay in the broadening of his support base to
include peasants, students, and the lower
Wavell Plan (1945) and Cabinet Mission (1946)
middle class.
were introduced after the Quit Indian Movement
and Simon Commission (1927) was set up prior y Railway lines were disrupted, police
to the Movement. stations were burnt down, and telegraph
services destroyed. The British retaliated in
vehement terms using ‘lathi charges’ and
9. Answer: (a) making mass arrests.
Women participation in Quit India Movement: y An aspect of the movement that is rarely
y Women, especially school and college girls, spoken about is the way it encouraged
actively participated, and included Aruna women to come out of the thresholds of
Asaf Ali, Sucheta Kripalani and Usha Mehta. their homes and raise their voice against
British rule. With the majority of the men
y Usha Mehta actively supported the Quit
behind bars, women took to the streets,
India movement and was an important
raising slogans, holding public lectures
member of a small group that ran the
and demonstrations, and even making and
Congress Radio. [Option (a) is correct]
transporting explosives.
y The majority of the Quit India Movement
10. Answer: (d) was carried by the labour class as they
Mahatma Gandhi raised the slogan of ‘do or die’ agitated through bandhs and hartals.
at the Bombay session of the All India Congress Hence, statement 4 is not correct.

The National Movement in the 1940s-Explanation 53


unacademy.com | Download the Unacademy app
Give your feedback here: Link
Development of Press, Education
8 and Civil Services
1. Who among the following was associated 1. Charter Act of 1813
as Secretary with Hindu Female School 2. General Committee of Public Instruction,
which later came to be known as Bethune 1823
Female School? (2021)
3. Orientalist and Anglicist Controversy
(a) Annie Besant
Select the correct answer using the code
(b) Debendranath Tagore given below:
(c) Ishwar Chandra Vidyasagar (a) 1 and 2 only
(d) Sarojini Naidu (b) 2 only
(c) 1 and 3 only
2. Regarding Wood's Dispatch, which of the (d) 1, 2 and 3
following statements are true? (2018)
1. Grants-in-Aid system was introduced.
4. With reference to educational institutions
2. Establishment of universities was during colonial rule in India, consider the
recommended. following pairs: (2018)
3. English as a medium of instruction at all
Institution Founder
levels of education was recommended.
1. Sanskrit College at William Jones
Select the correct answer using the code
Benaras
given below:
2. Calcutta Madrasa Warren Hastings
(a) 1 and 2 only
3. Fort William College Arthur Wellesley
(b) 2 and 3 only
Which of the pairs given above is/are
(c) 1 and 3 only
correct?
(d) 1, 2 and 3
(a) 1 and 2 only
(b) 2 only
3. Which of the following led to the
(c) 1 and 3 only
introduction of English Education in India?
(2018) (d) 3 only

54 Development of Press, Education and Civil Services


unacademy.com | Download the Unacademy app
Give your feedback here: Link
Development of Press, Education
8 and Civil Services-Explanation
1. Answer: (c) also recommended the establishment of
three universities in the Presidency towns of
Calcutta, Bombay and Madras on the model of
the London University.
Statement 3 is not correct: It also wanted to
promote the study of English as well as Indian
regional languages and classical languages
like Arabic, Persian and Sanskrit. It did not
recommend English as a medium of instruction
at all levels of education, only for higher studies
it was recommended.
Option (c) is correct: Ishwar Chandra
Vidyasagar (1820-1891) was one of the pillars Elimination Technique: Wood’s Dispatch
of the Bengal renaissance who managed to recommended english as medium
continue the social reforms movement that of instruction for higher education.
was started by Raja Rammohan Roy in the early Statement 3 is eliminated. Thus, Option (a)
1800s. Vidyasagar was a well-known writer, is the correct answer.
intellectual and above all, a staunch supporter of
humanity. He supported John Elliot Drinkwater
Bethune to establish the first permanent girls’ 3. Answer: (d)
school in India, the Bethune School, in 1849. Option (d) is correct: The introduction of
In December 1850, Bethune appointed Ishwar English Education in India was first put forward
Chandra Vidyasagar as secretary to the school. by the Charter Act of 1813 and Orientalist
The Hindu Female School’s charge was taken and Anglicist Controversy. Then in 1823, the
up by the Government in 1856, and then later, Governor-General-in Council appointed a
it was renamed Bethune School. “General Committee of Public Instruction”,
which had the responsibility to grant one
lakh of rupees for education. That Committee
2. Answer: (a) consisted of ten European members of which
Charles Wood was a British Liberal politician Lord Macaulay was the President.
and Member of Parliament. He served as
Chancellor of the Exchequer from 1846 to 1852.
He later became the President of the Board of 4. Answer: (b)
Control of the East India Company. In 1854, the Pair 1 is not correctly matched: The first
“Wood’s despatch” was sent to the Governor generation of British administrators in India
General Lord Dalhousie by him. This document like Warren Hastings, William Jones and
is considered as the “Magna Carta of English Jonathan Duncan popularised the view
Education in India”. that India had a glorious past which had
Statement 1 is correct: As per this Wood’s subsequently degenerated. These scholars
Despatch an education department was to be and administrators were called Orientalists.
set up in every province along with the provision They were keen to learn and propagate Indian
of grants in aid to the affiliated private schools. languages and traditions. This, they thought,
would ensure a better understanding of India
Statement 2 is correct: The Wood’s Despatch which would eventually strengthen their rule

Development of Press, Education and Civil Services-Explanation 55


unacademy.com | Download the Unacademy app
Give your feedback here: Link
over this country. Hence, in 1794, Jonathan Pair 3 is not correctly matched: There was a
Duncan founded the Sanskrit College at strong urge to make local British administrators
Benares. familiar with Indian culture and tradition. Fort
William College founded by Richard Wellesley
Pair 2 is correctly matched: Warren Hastings
in 1801 to train the young British recruits to
set up Calcutta Madrasa in 1781 for the study
the civil service in India was meant to serve
and learning of Persian and Arabic. This was
primarily this purpose. This college became
done as the British hoped to win a place in
an important center for producing knowledge
the hearts of the “natives”; only then could the
on and about India. It had many departments
alien rulers expect to be respected by their
devoted exclusively to research on Indian
subjects.
languages and literature.

56 Development of Press, Education and Civil Services-Explanation


unacademy.com | Download the Unacademy app
Give your feedback here: Link
9 Independence to Partition

1. With reference to the proposals of Cripps 1. The first democratically elected


Mission, consider the following statements: communist party government formed in
(2022) a State in India.
1. The Constituent Assembly would have 2. India's then largest bank, 'Imperial Bank
members nominated by the Provincial of India', was renamed 'State Bank of
Assemblies as well as the Princely India'.
States. 3. Air India was nationalised and became
2. Any Province, which is not prepared to the national carrier.
accept the new Constitution would have 4. Goa became a part of independent
the right to sign a separate agreement India.
with Britain regarding its future status.
Which of the following is the correct
Which of the statements given above is/are chronological sequence of the above
correct? events?
(a) 1 only (a) 4 - 1 - 2 - 3
(b) 2 only (b) 3 - 2 - 1 - 4
(c) Both 1 and 2 (c) 4 - 2 - 1 - 3
(d) Neither 1 nor 2 (d) 3 - 1 - 2 - 4

2. With reference to Indian National 4. The Radcliffe Committee was appointed to


Movement, consider the following pairs: (2014)
(2019) (a) solve the problem of minorities in India
Person Position held (b) give effect to the Independence Bill
1. Sir Tej Bahadur President, All India (c) delimit the boundaries between India
Sapru Liberal Federation and Pakistan
2. K.C. Neogy Member, The (d) enquire into the riots in East Bengal
Constituent Assembly
3. P.C. Joshi General Secretary,
5. With reference to Indian History, the
Communist Party of
Member of the Constituent Assembly from
India
the provinces were (2013)
Which of the pairs given above is/are
(a) Directly elected by the people of those
correctly matched?
provinces
(a) 1 only
(b)
Nominated by the Indian National
(b) 1 and 2 only Congress and the Muslim league
(c) 3 only (c)
Elected by Provincial Legislative
(d) 1, 2 and 3 Assemblies
(d) Selected by the Government for their
3. Consider the following events: (2018) expertise in constitutional matters.

Independence to Partition 57
unacademy.com | Download the Unacademy app
Give your feedback here: Link
Independence to Partition-
9 Explanation
1. Answer: (b) 3. Answer: (b)
Statement 1 is incorrect: A Constituent Option (b) is correct: In 1953, India nationalised
Assembly was to be founded to frame a new all Indian airlines, creating two corporations—
Constitution. Members of this assembly were to one for domestic service, called Indian Airlines
be partly elected by the provincial assemblies Corporation (merging Air-India Limited with six
and partly nominated by the Princely states. lesser lines), and one for international service,
Statement 2 is correct: Any province not Air-India International Corporation.
willing to join the Union could have a separate The Reserve Bank of India, which is the Central
Constitution and would be eligible to sign an Bank of India, acquired a controlling interest
agreement with the British regarding its future in the Imperial Bank of India in 1955. On 1 July
status. 1955, the Imperial Bank of India became the
State Bank of India.

2. Answer: (d) The Kerala Legislative Assembly election of 1957


was the first Assembly election in the Indian
Pair 1 is correctly matched: When the Montague
state of Kerala. Indian state of Kerala became
report of 1918 was made public, there was
the first Communist Party barring the PCI in
a divide in the Congress. The moderates
San Marino to win elections in a parliamentary
welcomed it while the extremists opposed it.
democracy. The Communist Party formed the
This led to a rift in the Congress with moderate
government.
leaders forming the “Indian National Liberal
Federation (INLF)’’ in 1919. INLF was founded by On 19 December 1961, Goa was liberated
Surendra Nath Banerjee and Tej Bahadur Sapru from Portuguese rule and became a part of
was chosen as the President of the federation. independent India. After conferring statehood
on Goa on 30 May 1987, Daman and Diu was
Pair 2 is correctly matched: In 1946, the Cabinet
made a separate Union Territory.
Mission visited India to draft a Constitution for
the country. On the basis of the framework Elimination Technique: As India attained
provided by the Cabinet Mission, a Constituent independence, the Indian leaders opted
Assembly was constituted on 9th December for a mixed form of economy. Many banks
1946 which was a fully sovereign body. It and airlines were nationalized in the 1950s.
elected Dr Rajendra Prasad as the first Thus, 3 cannot be last in chronological
President of India. Kshitish Chandra Neogy, an order. Option (a) and (c) are eliminated.
Indian politician from West Bengal, was also a
member of the Constituent Assembly of India.
4. Answer: (c)
Pair 3 is correctly matched: The Communist
Party of India was formed on 26 December Option (c) is correct: The British government
1925 at the first Party Conference in Kanpur. appointed the Boundary Commission under the
S.V. Ghate was appointed the first General chairmanship of Sir Cyril Radcliffe to delimit
Secretary of CPI. The scattered Communist the boundaries between India and Pakistan.
Group again met in Surat in late 1935 and The Radcliffe Committee consisted of two
chose P.C. Joshi as the General Secretary of Muslims and two non-Muslim judges in each
the Communist Party of India. case and worked under serious constraints.
Radcliffe, with very limited knowledge of India,
and with the use of out-of-date maps and

58 Independence to Partition-Explanation
unacademy.com | Download the Unacademy app
Give your feedback here: Link
census materials, was required to draw the Option (c) is correct: A Constituent Assembly
boundaries and decide disputed points within was to be elected by provincial assemblies
a period of six weeks. by proportional representation (voting in
three groups—General, Muslims, Sikhs). This
Constituent Assembly would be a 389-member
5. Answer: (c) body with Provincial Assemblies sending 292,
The Cabinet Mission Plan, which set up the Chief Commissioner’s provinces sending 4, and
Constituent Assembly of India, discarded full princely states sending 93 members.
adult franchise for elections to the Constituent The seats given to a province were decided
Assembly and instead suggested indirect among three communities on the basis of
elections by members of the recently elected their number, the three communities being the
Provincial Legislative Assemblies. Muslims, Sikhs and General including Hindus
and all others who were not Muslims and Sikhs.

Independence to Partition-Explanation 59
unacademy.com | Download the Unacademy app
Give your feedback here: Link
2 ANCIENT INDIA
Prehistoric Period and Indus
1 Valley Civilisation
1. Which one of the following ancient towns 3. They employed horse-drawn chariots in
is well-known for its elaborate system warfare.
of water harvesting and management by Select the correct statement/statements
building a series of dams and channelizing using the code given below:
water into connected reservoirs? (2021)
(a) 1 and 2 only
(a) Dholavira
(b) 2 only
(b) Kalibangan
(c) 1, 2 and 3
(c) Rakhigarhi
(d) None of the statements given above is
(d) Ropar correct

2. Which one of the following is not a 4. Regarding the Indus Valley Civilization,
Harappan site? (2019) consider the following statements:(2011)
(a) Chanhudaro 1. It was predominantly a secular
(b) Kot Diji civilization and the religious element,
(c) Sohgaura though present, did not dominate the
scene.
(d) Desalpur
2. During this period, cotton was used for
manufacturing textiles in India.
3. Which of the following characterises/ Which of the statements given above is/are
characterise the people of the Indus correct?
Civilization? (2013)
(a) 1 only
1. They possessed great palaces and
temples. (b) 2 only

2. They worshipped both male and female (c) Both 1 and 2


deities. (d) Neither 1 nor 2

61 Prehistoric Period and Indus Valley Civilisation


unacademy.com | Download the Unacademy app
Give your feedback here: Link
Prehistoric Period and Indus
1 Valley Civilisation-Explanation
1. Answer: (a)
Option (a) is correct: The city of Dholavira
was an ancient town of the Indus Valley
Civilization (IVC). It is well-known for its
elaborate system of water harvesting and
management by building a series of dams and
channelizing water into connected reservoirs.
Planners in the ancient city of Dholavira had
conceptualised an amazing system of drains,
dams, and tanks to manage water. To conserve
every drop of water, they carefully considered
everything.
Unlike other Harappan antecedent towns, 2. Answer: (c)
Dholavira was strategically located on the
The history of India begins with the birth of
island of Khadir to harness various mineral and
the Indus Valley Civilization which flourished
raw material sources (copper, shell, agate-
around 2,500 BC, in the western part of South
carnelian, steatite, lead, banded limestone, and
Asia, in contemporary Pakistan and Western
others) and to facilitate internal and external
India. The first city to be unearthed was Harappa
trade to the Magan (modern Oman peninsula)
and hence Indus Valley Civilization, also known
and Mesopotamian regions.
as Harappan Civilization. The Indus Valley was
home to the largest of the four ancient urban

Prehistoric Period and Indus Valley Civilisation-Explanation 62


unacademy.com | Download the Unacademy app
Give your feedback here: Link
civilizations of Egypt, Mesopotamia, India, in western South Asia, which now is Pakistan
and China. Since it belongs to the Bronze/ and Western India. The Indus Valley civilization
Chalcolithic age, the Indus Valley Civilization is was essentially an urban civilization, and
also known as the Bronze Age Civilization. people lived in well-planned and well-built
Option (c) is correct: Sohgaura is not a Harappan cities, which were also centres of commerce.
Site. It is a village on the banks of the Rapti Statement 1 is not correct: When India’s
River, about 20 km south-east of Gorakhpur, Archeological Department conducted
in the Gorakhpur District, Uttar Pradesh, India. excavations in the Indus Valley, where the two
It is known for Indian copper plate inscriptions ancient towns, Mohenjodaro and Harappa,
written in Prakrit in the Brahmi script. were discovered. The remains of structures
and other items like domestic goods, military
Additional Information: weapons, gold and silver decorations, seals,
y Kot Diji: It was a pre-Harappan site located toys, ceramic products, etc., indicate that a
on the left bank of River Sindh. highly developed civilization existed in this area
four to five thousand years ago. Archaeologists
y Chanhudaro: It is situated 130 kms south of
have not found any structure resembling a
Mohenjo-Daro in Sindh and is the only site
temple or a palace or any monument. Great
without a fortified citadel.
temples and palaces were possessed by
y Desalpur: It is a mature Harappan site Egyptians and Mesopotamians rather than the
situated near Gunthali in Nakhatrana Harappan people.
Taluka of the Kutch district, famous for
Statement 2 is correct: Many figures of the
disproportionately large fortifications.
Mother Goddess made of clay have been found
at the Harappan site. Goddess was considered
3. Answer: (b)
as a symbol of fertility and was venerated by
India’s history starts with the emergence of
the people. Another figure of a male god in a
the Indus Valley Civilization, also known as the
seated position is also found. It was carved on
Harappan Civilization. It thrived about 2,500 BC
a small stone seal.

63 Prehistoric Period and Indus Valley Civilisation


unacademy.com | Download the Unacademy app
Give your feedback here: Link
Statement 3 is not correct: Evidence also 4. Answer: (c)
suggests that the Indus Valley Civilization may Statement 1 is correct: Indus Valley Civilization
have been the first to use wheeled transport. was highly developed, and it must have taken
These advances may have included bullock thousands of years to reach that stage. It was
carts that are identical to those seen today a predominantly secular civilization and even
in the South Asian region. According to Indus though religious elements were present, it did
seals, domesticated animals were mostly not dominate the scene.
goats, buffaloes, oxen, elephants, dogs, and
camels, but Harappans were not familiar with Statement 2 is correct: The economy of the
horses. Indus Valley Civilization depended upon crops
and animal husbandry. The main crops grown
Elimination Technique: IVC people did not were rice, wheat, melons, dates, vegetables,
possess great temples. So statement 1 mainly legumes, and cotton. Large granaries
can be eliminated. were also found. Cotton later became the
From seals and mother goddess statues stimulus for the growth of the textile Industry.
found in Harappan sites it is quite evident
that they were worshipers of both male
and female deities. So, option B is correct.

Prehistoric Period and Indus Valley Civilisation 64


unacademy.com | Download the Unacademy app
Give your feedback here: Link
2 Vedic and Later Vedic Age

1. With reference to the difference between (c) Agrahaarika


the culture of Rigvedic Aryans and Indus (d) Magadha
Valley people, which of the following
statements is/are correct? (2017)
1. Rigvedic Aryans used the coat of mail 3. The religion of early Vedic Aryans was
and helmet in warfare, whereas the primarily of: ( 2012)
people of the Indus Valley Civilization (a) Bhakti
did not leave any evidence of using (b) image worship and Yajnas
them.
(c) worship of nature and Yajnas
2. Rigvedic Aryans knew gold, silver, and
(d) worship of nature and Bhakti
copper, whereas Indus Valley people
knew only copper and iron.
3. Rigvedic Aryans had domesticated the 4. The “dharma” and “rita” depict a central
horse, whereas there is no evidence of idea of the ancient Vedic civilization of
Indus Valley people having been aware India. In this context, consider the following
of this animal. statements: (2011)
Select the correct answer using the code 1. Dharma was a conception of obligations
given below: and of the discharge of one’s duties to
oneself and to others.
(a) 1 only
2. Rita was the fundamental moral
(b) 2 and 3 only
law governing the functioning of the
(c) 1 and 3 only universe and all it contained.
(d) 1, 2 and 3 Which of the statements given above is/are
correct?
2. With reference to the cultural history (a) 1 only
of India, the memorising of chronicles, (b) 2 only
dynasty histories, and epic tales was the
(c) Both 1 and 2
profession of who of the following? (2016)
(d) Neither 1 nor 2
(a) Shramana
(b) Parivraajaka

65 Vedic and Later Vedic Age


unacademy.com | Download the Unacademy app
Give your feedback here: Link
Vedic and Later Vedic Age-
2 Explanation
1. Answer: (c) the Sutas and Magadhas (bards and chroniders),
Statement 1 is correct: The Ramayana has who were the descendants of priestly families
mention of chariots covered with leather. of the Vedic period.
The Kshatriyas used to go for combat on the
chariots that were equipped with asi (swords), 3. Answer: (c)
hanas (arrows), and ilhianus (bows). Rigvedic
Option (c) is correct: The Bhakti Cult in ancient
Aryans used the coat of mail and helmet in
India can be traced during and after the Gupta
warfare, whereas the people of the Indus
period, that is, 3rd- 4th Century AD. The Vedic
Valley Civilization did not leave any evidence
people worshipped various forces of nature
of using them.
as God, stress was on sacrifice but not on
Statement 2 is not correct: The metals which magico-ritual formulae, religion was based
were used by Indo-Aryans in the time of the on material gains, etc. Rituals and sacrifices
Rigveda were gold and copper or bronze, but in (yajnas) became a daily routine of early Vedic
the times of Yajurveda and Atharvaveda, these people.
metals were supplemented by silver and iron.
Among the people of the Indus Valley, silver
was more common than gold. The utensils 4. Answer: (c)
and the vessels were sometimes made out of Statement 1 is correct: Dharma means law or
stone which was a relic of the Neolithic Ageas natural law and it is a concept of importance
as well as copper and bronze. However, there in Indian religion and philosophy. In the Hindu
has been no evidence of Iron. context, it refers to a person’s personal
Statement 3 is correct: The Rig Veda mentions obligation or duties. A Hindu’s Dharma is
horse-drawn chariots with spoked wheels. affected by his age, class, caste, gender, and
However, there is very little archaeological occupation. Dharma is usually defined as
evidence of horses during the Harappan era “righteousness” or “duty”. The four aims of a
and none earlier. man’s life are
y Kama (gratification of desire)
Elimination Technique: During IVC people
were aware of bronze, the famous Lost y Artha (acquirement of wealth)
Wax Technique was used for Bronze y Dharma (discharge of duty)
Casting. So, we can eliminate the option
y Moksha (final emancipation)
B and D.
Statement 2 is correct: Rita is the physical
order of the universe, the order of the sacrifice
2. Answer: (d) and the moral law of the world. According to
Option (d) is correct: The memorising of Vedic Religion, Rita was guarded by Varuna,
chronicles, dynastic histories, or epic tales the god-sovereign, who was assisted by Mitra,
was the work of a group of people, the Sutas the god of honour and proper performance
and Magadhas. During the first stage (from the of sacrifices was necessary to guarantee its
4th century BC to the 4th century AD), the task continuance. Violation (anrita) of the order was
of collecting information and presenting it in considered sin and required careful expiation.
literary form was the special task assigned to

Vedic and Later Vedic Age-Explanation 66


unacademy.com | Download the Unacademy app
Give your feedback here: Link
3 Mauryan and Post-Mauryan Age

1. In which one of the following regions 4. Consider the following pairs: (2023)
was Dhanyakataka, which flourished as Site : Well Known for
a prominent Buddhist centre under the
Mahasanghikas, located? (2023) 1. Besnagar : Shaivite cave shrine

(a) Andhra 2. Bhaja : Buddhist cave shrine

(b) Gandhara 3. Sittanavasal : Jain cave shrine

(c) Kalinga How many of the above pairs are correctly


matched?
(d) Magadha
(a) Only one
(b) Only two
2. With reference to ancient India, consider
the following statements: (2023) (c) All three

1. The concept of Stupa is Buddhist in (d) None


origin.
2. Stupa was generally a repository of 5. According to Kautilya’s Arthashastra,
relics. which of the following are correct ? (2022)
3. Stupa was a votive and commemorative 1. A person could be a slave as a result of
structure in Buddhist tradition. a judicial punishment.
How many of the statements given above 2. If a female slave bore her master a son,
are correct? she was legally free.
(a) Only one 3. If a son born to a female slave was
(b) Only two fathered by her master, the son was
entitled to the legal status of the
(c) All three master’s son.
(d) None Which of the statements given above are
correct ?
3. "Souls are not only the property of animal (a) 1 and 2 only
and plant life, but also of rocks, running (b) 2 and 3 only
water and many other natural objects not
looked on as living by other religious sects." (c) 1 and 3 only

The above statement reflects one of the (d) 1, 2 and 3


core beliefs of which one of the following
religious sects of ancient India? (2023) 6. Consider the following pairs : (2022)
(a) Buddhism
Site of Ashoka’s Location in the
(b) Jainism major rock edicts State of
(c) Shaivism 1. Dhauli Odisha
(d) Vaishnavism 2. Erragudi Andhra Pradesh
3. Jaugada Madhya Pradesh

67 Mauryan and Post-Mauryan Age


unacademy.com | Download the Unacademy app
Give your feedback here: Link
4. Kalsi Karnataka correct description of the term 'paramitas'?
(2020)
How many pairs given above are correctly
matched ? (a)
The earliest Dharmashastra texts
written in aphoristic (sutra) style
(a) Only one pair
(b)
Philosophical schools that did not
(b) Only two pairs
accept the authority of Vedas
(c) Only: three pairs
(c) Perfections whose attainment led to
(d) All four pairs the Bodhisattva path
(d)
Powerful merchant guilds of early
7. Who among the following rulers advised his medieval South India
subjects through this inscription? (2020)
“Whosoever praises his religious sect 10. Consider the following: (2019)
or blames other sects out of excessive
1. Deification of the Buddha.
devotion to his own sect, with the view of
glorifying his own sect, he rather injures his 2. Treading the path of Bodhisattvas.
own sect very severely.” 3. Image worship and rituals.
(a) Ashoka Which of the above is/are the feature /
(b) Samudragupta features of Mahayana Buddhism?

(c) Harshavardhana (a) 1 only

(d) Krishanadeva Raya (b) 1 and 2 only


(c) 2 and 3 only

8. With reference to the religious history of (d) 1, 2 and 3


India, consider the following statements:
(2020) 11. In which of the following relief sculpture
1. Sthaviravadins belong to Mahayana inscriptions is ‘Ranyo Ashoka’ (King
Buddhism. Ashoka) mentioned along with the stone
2. Lokottaravadin sect was an offshoot of portrait of Ashoka? (2019)
Mahasanghika sect of Buddhism. (a) Kanganahalli
3. The deification of Buddha by (b) Sanchi
Mahasanghikas fostered Mahayana (c) Shahbazgarhi
Buddhism.
(d) Sohgaura
Which of the statements given above is/are
correct?
(a) 1 and 2 only 12. With reference to the religious practices
in India, the “Sthanakvasi” sect belongs to
(b) 2 and 3 only (2018)
(c) 3 only (a) Buddhism
(d) 1, 2 and 3 (b) Jainism
(c) Vaishnavism
9. With reference to the cultural history of (d) Shaivism
India, which one of the following is the

Mauryan and Post-Mauryan Age 68


unacademy.com | Download the Unacademy app
Give your feedback here: Link
13. With reference to India history, who among their path to it.
the following is a future Buddha, yet to Which of the statements given above is/are
come to save the world? (2018) correct?
(a) Avalokiteshvara (a) 1 only
(b) Lokesvara (b) 2 and 3 only
(c) Maitreya (c) 2 only
(d) Padmapani (d) 1, 2, and 3

14. With reference to the religious history of 17. Which of the following kingdoms were
India, consider the following statements: associated with the life of Buddha? (2015)
(2017)
1. Avanti
1. Sautrantika and Sammitiya were the
sects of Jainism. 2. Gandhara

2. Sarvastivadin held that the constituents 3. Kosala


of phenomena were not wholly 4. Magadha
momentary but existed forever in a Select the correct answer using the code
latent form. given below:
Which of the statements given above is/are (a) 1, 2 and 3 only
correct?
(b) 2 and 3 only
(a) 1 only
(c) 1, 3 and 4 only
(b) 2 only
(d) 3 and 4 only
(c) Both 1 and 2
(d) Neither 1 nor 2
18. Which of the following Kingdoms were
associated with the life of the Buddha?
15. Who of the following had first deciphered (2014)
the edicts of Emperor Ashoka? (2016) 1. Avanti
(a) Georg Buhler 2. Gandhara
(b) James Prinsep 3. Kosala
(c) Max Muller 4. Magadha
(d) William Jones Select the correct answer using the code
given below:
16. With the reference to the religious history (a) 1, 2 and 3
of India, consider the following statement: (b) 2 and 4
(2016)
(c) 3 and 4 only
1. The concept of Bodhisattva is central to
(d) 1, 3 and 4
the Hinayana sect of Buddhism.
2. Bodhisattva is the compassionate one
on his way to enlightenment. 19. Which one of the following describes best
the concept of Nirvana in Buddhism? (2013)
3. Bodhisattva delays achieving his own
salvation to help all sentient beings on (a) The extinction of the flame of desire

69 Mauryan and Post-Mauryan Age


unacademy.com | Download the Unacademy app
Give your feedback here: Link
(b) The complete annihilation of self (a) Buddha’s calling of the Earth to watch
(c) A state of bliss and rest over Mara and to prevent Mara from
disturbing his meditation.
(d) A mental stage beyond all comprehension
(b) Buddha’s calling of the Earth to witness
his purity and chastity despite the
20. Which of the following statements is/are temptations of Mara.
applicable to Jain doctrine? (2013)
(c) Buddha’s reminder to his followers that
1. The surest way of annihilating Karma is they all arise from the Earth and finally
to practice penance. dissolve into the Earth, and thus this
2. Every object, even the smallest particle, life is transitory.
has a soul. (d) Both the statements (a) and (b)
3. Karma is the bane of the soul and must are correct in this context
be ended.
Select the correct answer using the code 23. With reference to the guilds (Shreni) of
given below: ancient India that played a very important
(a) 1 only role in the country’s economy, which of
the following statements is/are correct?
(b) 2 and 3 only
(2012)
(c) 1 and 3 only
1. Every guild was registered with the
(d) 1, 2 and 3 central authority of the State and
the king was the chief administrative
authority on them.
21. With reference to the history of ancient
India, which of the following was/were 2. The wages, rules of work, standards
common to both Buddhism and Jainism? and prices were fixed by the guild.
(2012) 3. The guild had judicial powers over its
1. Avoidance of extremities of penance own members.
and enjoyment Select the correct answer using the code
2. Indifference to the authority of the given below:
Vedas (a) 1 and 2 only
3. Denial of efficacy of rituals (b) 3 only
Select the correct answer using the code (c) 2 and 3 only
given below:
(d) 1, 2 and 3
(a) 1 only
(b) 2 and 3 only
24. The Jain philosophy holds that the world is
(c) 1 and 3 only created and maintained by (2011)
(d) 1, 2 and 3 (a) Universal Law
(b) Universal Truth
22. Lord Buddha’s image is sometimes shown (c) Universal Faith
with the hand gesture called 'Bhumisparsha
(d) Universal Soul
Mudra'. It symbolises: (2012)

Mauryan and Post-Mauryan Age 70


unacademy.com | Download the Unacademy app
Give your feedback here: Link
Mauryan and Post-Mauryan Age-
3 Explanation
1. Answer: (a) and matter (pudgala). Souls are not only the
Dhanyakataka, also known as Amaravati, was property of animal and plant life, but also of
an important Buddhist centre that flourished entities such as stones, rocks, running water,
under the Mahasanghika sect. It was situated and many other natural objects not looked on
in present-day Guntur district of the Indian as living by other sects.
state of Andhra Pradesh.
Dhanyakataka was renowned for its Buddhist 4. Answer: (b)
stupas and monastic complexes. It played a 1. Besnagar: Shaivite cave shrine - This pair
significant role in the spread of Buddhism in is incorrect. Besnagar is not primarily
ancient South India and was a major centre for known for a Shaivite cave shrine. It is an
Buddhist learning and pilgrimage. archaeological site located in Madhya
The site is known for its ancient ruins and Pradesh, India, and is known for the
Buddhist sculptures, including the famous remains of ancient structures, including
Amaravati Stupa. These archaeological remains a heliodor pillar and an Ashokan pillar.
provide insights into the rich Buddhist heritage 2. Bhaja: Buddhist cave shrine - This pair is
that thrived in the region centuries ago. correct. Bhaja is a well-known Buddhist
cave shrine located in Maharashtra,
2. Answer: (b) India. It is famous for its rock-cut caves
and intricate sculptures depicting
Statement 1 is incorrect: The tradition of Buddhist art and architecture.
erecting stupas may have been pre-Buddhist,
but they came to be associated with Buddhism. 3. Sittanavasal: Jain cave shrine - This
pair is correct. Sittanavasal is a historic
Statement 2 is correct: Relics of the Buddha site in Tamil Nadu, India, known for
such as his bodily remains or objects used by its Jain cave temple complex. The
him were buried there. site contains rock-cut cave temples
Statement 3 is correct: It is a Buddhist and exquisite frescoes depicting Jain
commemorative monument usually housing religious themes.
sacred relics associated with the Buddha or
other saintly persons
5. Answer: (b)
In Chapter 13 of Kautilya’s Arthashastra.
3. Answer: (b)
Statement 1 is not correct: According to the
According to Jainism, the souls are found Kautilya’s Arthashastra, a person can primarily
not only in the living beings like animals and be ensalved by selling himself or being captured
plants, but also in stones, rocks, water, etc. in the war.
Soul is the core and the fundamental focus
of Jain philosophy. The universe functions Statement 2 is correct: According to the
through the interaction of living souls (jivas, Kautilya’s Arthashastra, children born to slave
literally “lives”), and five categories of non living and master when a slave gives birth to a child,
entities (ajiva): “ether” (akasa), the means or both the mother and child shall be recognised
condition of movement (dharma), the means as free and were no longer considered to be
or condition of rest ( adharma), time ( kala ), slaves.

71 Mauryan and Post-Mauryan Age-Explanation


unacademy.com | Download the Unacademy app
Give your feedback here: Link
Statement 3 is correct: If a son born to a female he established and the moral principles he
slave was fathered by her master, the son was recommended in his attempt to create a just
entitled to the legal status of the master’s son. and humane society.
Option (a) is correct: Major Rock Edict XII of
6. Answer: (b) Ashoka reads, “Whosoever praises his religious
sect or blames other sects out of excessive
Option (b) is correct:
devotion to his own sect, with the view of
glorifying his own sect, he rather injures his
own sect very severely.”

8. Answer: (b)
Statement 1 is not correct: The first division
in the Buddhist community occurred in 383
B.C. The order broke into Sthaviravada (in Pali
Theravada) and the Mahasanghikas. While the
Sthaviravadins were believers in the teaching
of elders, and orthodox school. They did not
belong to the Mahayana Buddhism.
Statement 2 is correct: Subdivisions of
the Mahasanghikas included aspects of
Lokottarvadin whose philosophy was based
on the concept of Lokottara Buddha or
Supernatural Buddha. Hence, Lokottaravadin
sect was an offshoot of Mahasanghika sect of
Buddhism.
The Major split took place during the reign of
Kanishka in 100 A.D. and there emerged two
sects: Hinayana or the Lesser Vehicle and
y Pair 1 is correctly matched: Dhauli is Mahayana or the Greater Vehicle.
located in Odisha.
Statement 3 is correct:Mahasanghika Paved
y Pair 2 is correctly matched: Erragudi or the way for the emergence of the Mahayana
Yerragudi is located in Andhra Pradesh sect of Buddhism during the 4th Buddhist
y Pair 3 is incorrectly matched: Jaugada is Council. Mahayanist gave prominence to the
located in Odisha. Bodhisattva ideal. Thus, the deification of
Buddha by Mahasanghikas fostered Mahayana
y Pair 4 is incorrectly matched: Kalsi is
Buddhism.
located in Uttarakhand.

7. Answer: (a) 9. Answer: (c)

Ashoka, also known as Ashoka the Great, was Option (c) is correct: To achieve enlightenment
an Indian Emperor of the Mauryan Dynasty. He and realise Nibbana, the Fourth Noble Truth,
ruled almost all of the subcontinent from c.268 the Noble Eightfold Path, gives a Buddhist
to 232 BCE. He adopted Buddhism in the 9th practitioner, ways to practice, and the path to
year of his reign after winning Kalinga. Ashoka’s follow. There also exists a parallel path that
edicts are mainly concerned with the reforms consists of perfecting certain qualities, in order

Mauryan and Post-Mauryan Age-Explanation 72


unacademy.com | Download the Unacademy app
Give your feedback here: Link
for a pilgrim to become a self-Enlightened 11. Answer: (a)
Universal Buddha. These qualities are termed Option (a) is correct: Kanaganahalli, an
as the Paramis (perfections) in the Southern important Buddhist site, dates from the 1st
traditions and the Paramitas in the Eastern century BC to the 3rd century AD. It is the
and Northern traditions in the context of the place where an ancient Buddhist Mahastupa
Bodhisattva path. site was found. It is a site in Karnataka with
an inscription in Brahmi script reading “Ranyo
10. Answer: (d) Ashoka” (King Ashoka) and a sculpture of King
Ashoka. The uniqueness of the Kanaganahalli
Buddhism is a religion which encompasses
image is that it is the only inscribed portrait of
a variety of traditions, beliefs and spiritual
the Emperor encountered so far.
practices largely based on the teachings,
life experiences of Siddhartha Gautam, born Additional Information:
in circa 563 BCE. It is one of the important y Sanchi: It is generally known for its three
religions of South and South-Eastern Asian stupas out of which the first and the oldest
countries. Buddhism was divided into two Great Stupa was originally commissioned
sects: Mahayana and Hinayana. by Asoka. The ‘Great Stupa’ at Sanchi is
Option (d) is correct: Mahayana is one of the two the oldest stone structure of India and
main schools of Buddhism. The term Mahayana was originally commissioned by the Ashoka
is a Sanskrit word which literally means “Great in the 3rd century BC. It was vandalised
Vehicle”. This school of Buddhism believes in by Pushyamitra Sunga and rebuilt by his
the heavenliness of Buddha and Idol worship son Agnimitra and again improvised by
of Buddha and Bodhisattvas embodying Saatavahana. Thus, this stupa represents a
Buddha Nature. It originated in northern India testimony to the Maurya, Sunga as well as
and Kashmir and then spread east into Central Satavahana art in India.
Asia, East Asia and some areas of Southeast y Shahbazgarhi: It is a village and a historic
Asia. site located in the Mardan District in the
North West Frontier Province of Pakistan.

12. Answer: (b)


Option (b) is correct: Sthanakvasi is a sect of
Svetambara Jainism founded by a merchant
named Lavaji in 1653 AD. It believes that idol
worship is not essential in the path of soul
purification and attainment of Nirvana or
Moksha. The sect is essentially a reformation
of the one founded on teachings of Lonka, a
fifteenth-century Jain reformer.

73 Mauryan and Post-Mauryan Age-Explanation


unacademy.com | Download the Unacademy app
Give your feedback here: Link
14. Answer: (b)
Statement 1 is not correct: Sautrantika and
Sammitiya are sects of Buddhism. Sautrantika
school belongs to Hinayana Buddhism, which
is one of the two schools of Buddhism, the
other one being Mahayana. The Sautrantikas
recognized the authority of the sutras (words
of the Buddha) but not of the Abhidharma, the
more schematic part of the canon.
Statement 2 is correct: This sect of Buddhism
believes that all things exist, and exist
continuously, in the past and the future as
well as in the present. Sarvastivada, also called
Vaibhashika, a school of early Buddhism. A
fundamental concept in Buddhist metaphysics
is the assumption of the existence of dharmas,
cosmic factors and events that combine
momentarily under the influence of a person’s
past deeds to form a person’s life flux, which
he considers his personality and career. Like
all Buddhists, the Sarvastivadins consider
everything empirical to be impermanent, they
maintain that the dharma factors are eternally
existing realities, that the constituents of
Jainism is an ancient religion that follows phenomena were not wholly momentary but
the philosophy of liberation and also a path existed forever in a latent form.
to spiritual purity and enlightenment through
disciplined nonviolence to all living creatures.
Jainism came into prominence in 6th century 15. Answer: (b)
B.C. when Lord Mahavira propagated the Option (b) is correct: In 1837, James Prinsep
religion. There were 24 great teachers, the last deciphered the edicts of Ashoka.
of whom was Lord Mahavira. Jain order has
been divided into two major sects: Digambar
and Svetambara. 16. Answer: (b)
Statement 1 is not correct: Bodhisattva is an
ideal in Mahayana Buddhism, Bodhisattva is the
13. Answer: (c)
individual who seeks enlightenment both for
Option (c) is correct: In Buddhist culture, him or herself and for others.
Maitreya is a bodhisattva who will arrive on Earth
Statement 2 is correct: A Bodhisattva is
in the future, and save the world, in order to
literally a living being (Sattva) who aspires to be
attain complete awareness and educate about
enlightened (Bodhi) and carries out altruistic
the pure dharma. According to the inscriptions,
practices. Compassion, an empathetic sharing
Maitreya will be an heir to the existing Buddha,
of the sufferings of others, is Bodhisattva’s
Gautama Buddha. The prediction of the
greatest characteristic.
appearance of Maitreya relates to a time in
the future when the dharma will have been Statement 3 is correct: Bodhisattva is an
neglected by most in the physical world. aspirant of Buddhahood (buddhatva) who works

Mauryan and Post-Mauryan Age-Explanation 74


unacademy.com | Download the Unacademy app
Give your feedback here: Link
for the enlightenment of all sentient beings in 19. Answer: (a)
contrast to that of one’s own emancipation of Buddhism is a religion and philosophy that
the Arhat (an ideal in non- Mahayana traditions). developed from the teachings of the Buddha, a
Elimination Technique: Bodhisattva is an teacher who lived in northern India between the
ideal in Mahāyāna Buddhism and Hinayana mid-6th and mid-4th centuries BCE. Ancient
don’t believe in Idol Worship.So, we can Buddhist scripture and doctrine developed in
eliminate the option A and D. several closely related literary languages of
ancient India, especially in Pali and Sanskrit.
Option (a) is correct: The Pali word Nibbana
17. Answer: (d) (nirvana in Sanskrit) was first used by the
Option (d) is correct: Magadha, Kosala, Vaishali, Buddha to describe the highest state of
Vajji, were the kingdoms and regions that were profound well-being a human is capable of
associated with the life of Buddha. Avanti attaining.
was not directly related to the life of Buddha. The fires of greed, hatred, and delusion—toxic
Pasenadi (Prasenajit), King of Kosala, was the unconscious mental and emotional dispositions
Buddha’s contemporary and is frequently that cause people to harm themselves and
mentioned in Pali texts. Kosala and Magadha one another and cause suffering— have been
were linked through matrimonial ties. Buddha extinguished (nibbuta) in a person who attains
wandered through the towns and villages in the nirvana.
kingdoms of Kosala and Magadha teaching his
philosophy. Gandhara is not directly associated
with the life of Buddha. It was through 20. Answer: (d)
the expansion of the Mauryan empire that Statement 1 is correct: According to Jain
Gandhara received much Buddhist influence, doctrine, the annihilation (nirjara) of karma
notably during the reign of Asoka. comes about through penance, and the
prevention (samvara) of the influx (asrava)
18. Answer: (c) and fixation (bandha) of karma in the soul is
ensured by carefully disciplined conduct, as a
According to Buddhist tradition, Siddhartha result of which it does not enter in dangerous
Gautama was born in Lumbini (modern-day quantities and is dispersed immediately. When
Nepal) and grew up, the son of a king. He left the soul has finally set itself free, it rises at
his palace when he encountered the sufferings once to the top of the universe, above the
people faced. highest heaven where it remains in inactive
Option (c) is correct: After the birth of his son, omniscient bliss 8 through all eternity.
Buddha left KapilaVastu at the age of 29 and Statement 2 is correct: Jiva or soul is a
headed for Magadha in search of spiritual truth. conscious substance. Consciousness is the
In Magadha, he studied in the communities essence of the soul. It is always present in the
of two teachers, Alara Kalama and Uddaka soul, though its nature and degree may vary.
Ramaputta. The soul is eternal, but it also undergoes a
Kosala and Magadha kingdoms were associated change of states. According to Jain doctrine,
with the life of Buddha. Buddha spent the Souls are not only the property of animal and
last 20 years of his life in Kosala and the King plant life, but also of entities such as stone,
Pasenadi, was his most devoted disciple. water and many other natural objects not
looked upon as living by other sects.
Statement 3 is correct: The Jain doctrine
holds that every living and non-living being is

75 Mauryan and Post-Mauryan Age-Explanation


unacademy.com | Download the Unacademy app
Give your feedback here: Link
gifted with souls. Each Jiva (soul) is eternally a must for Jain monks who take the lifelong
associated with Ajiva (non-sentient beings vow of diksha or monkhood, is excruciatingly
or non-conscious beings) because of karma. painful. (Statement 1 is not correct)
They are obstructed by karma, just as the
natural light of the sun is hindered by clouds. Elimination Technique: Jainism believes
By removing the karmas, a soul can remove in extremities of penance and Avoidance
bondage and regain its natural perfection. of enjoyment, while Buddhism believes
in ‘Madhyam Marg’ or the middle path.
So, we can eliminate Statement 1. Thus,
21. Answer: (b) Option (a) is correct.
The school of Buddhism evolved in the eastern
part of India in 563 BCE. More or less at the 22. Answer: (b)
same time (in & around 550 BCE) another
school, Jainism, with almost similar thoughts, Option (b) is correct: Bhumisparsha means
was developing in the same part of India. touching the earth. It is most commonly known
The founder of Jainism, Lord Mahavira was a as the “earth witness” mudra. This mudra
contemporary of Lord Buddha, the founder of involves all the five fingers of the right hand
Buddhism and even the Buddhist texts called extended to touch the ground. This posture
Lord Mahavira ‘an enlightened being’. of mudra symbolises the pose of Buddha at
the time of enlightenment under the Bodhi
Both were the products of intellectual, spiritual tree, when he summoned the earth goddess
and social forces of their age and therefore, Sthavara, to bear witness to his attainment of
both stood up as revolts against the prevalent enlightenment.
Brahmanical religion.
Both Mahavira and Buddha, the founders of
Jainism and Buddhism respectively, were
Kshatriya princes and were able to get support
for their cause from the contemporary
ruling class, different Kshatriya rulers and
economically prosperous Vaishyas.
Both opposed the ritualism and the sacrifices
of Brahmanism and also challenged the
supremacy of the Brahmanas. (Statement 3 is
correct)
Both denied the authenticity of the Vedas as
an infallible authority. (Statement 2 is correct)
Shakyamuni Buddha is commonly represented
Both accepted the doctrines of the upon his enlightenment throne with his right
transmigration of soul and Karma and hand touching the earth and his left hand
emphasised the effects of good and bad Karmas resting upon his lap in the gesture of meditation.
(deeds) upon an individual’s future births and This symbolises his method or skillful means
ultimate salvation which was accepted as the in overcoming Mara (right hand), through the
highest goal of life. perfect wisdom of his deep meditation on
Even though both emphasised Ahimsa or non- emptiness (left hand).
violence The earth touching gesture is also the mudra
However, Jainism believes in extremities of of Akshobya, the ‘unshakable’ blue Buddha of
penance, for example, the practice of Kaya klesh, the centre or eastern direction.

Mauryan and Post-Mauryan Age-Explanation 76


unacademy.com | Download the Unacademy app
Give your feedback here: Link
23. Answer: (c)
eliminate the options A and D.
Ancient Indian guilds are a unique and multi-
faceted form of organisation, which combined
the functions of a democratic government, 24. Answer: (a)
a trade union, a court of justice and a Jainism lays emphasis on non-violence
technological institution. The trained workers (ahimsa). Followers of Jainism follow a very
of the guilds provided a congenial atmosphere strict and well-disciplined life. The first
for work. Tirthankar, Rishabhdev, flourished before
Option (c) is correct: Guilds were autonomous the Indus Valley Civilization and is referred
socio-economic entities, and Each guild had to as Lord Vishnu in the Puranas and is also
its own chief, assisted by others. The guilds mentioned in the Vedas.
regulated the wages, rules of work, standards, Option (a) is correct: Jainism does not support
and prices, etc. The guilds had some judicial belief in a creator deity. According to the Jain
powers, some of them even had their own Doctrine, the universe and its constituents
army and forces known as Screnibal. have always existed, and their actions are
governed by Universal Natural Laws. The world,
Elimination Technique: The term ‘Every
according to the Jain Doctrine is not created,
guild’ in statement 1 makes it rigid. There
maintained or destroyed by any deity but
could be few exceptions to it. So, we can
functions according to Universal Law.

77 Mauryan and Post-Mauryan Age-Explanation


unacademy.com | Download the Unacademy app
Give your feedback here: Link
4 Gupta and Post- Gupta Age

1. Consider the following dynasties : (2023) (d) 3 only


1. Hoysala
2. Gahadavala 3. From the decline of Guptas until the rise
3. Kakatiya of Harshavardhana in the early seventh
century, which of the following kingdoms
4. Yadava were holding power in Northern India?
How many of the above dynasties (2021)
established their kingdoms in the early 1. The Guptas of Magadha
eighth century AD?
2. The paramaras of Malwa
(a) Only one
3. The Pushyabhutis of Thanesar
(b) Only two
4. The Maukharis of Kanauj
(c) Only three
5. The Yadavas of Devagiri
(d) None
6. The Maitrakas of Valabhi
Select the correct answer using the code
2. With reference to the history of ancient given below.
India, which of the following statements
is/are correct? (2021) (a) 1, 2 and 5

1. Mitakshara was the civil law for upper (b) 1, 3, 4 and 6


castes and Dayabhaga was the civil law (c) 2, 3 and 4
for lower castes. (d) 5 and 6
2. In the Mitakshara system, the sons can
claim right to the property during the
lifetime of the father, whereas in the 4. Which of the following phrases defines the
Dayabhaga system, it is only after the nature of the ‘Hundi’ generally referred to
death of the father that the sons can in the sources of the post-Harsha period?
claim right to the property. (2020)

3. The Mitakshara system deals with the (a) An advisory issued by the king to his
matters related to the property held by subordinates
male members only of a family, whereas (b) A diary to be maintained for daily
the Dayabhaga system deals with the accounts
matters rélated to the property held by (c) A bill of exchange
both male and female members of a
(d) An order from the feudal lord to his
family.
subordinates
Select the correct answer using the code
given below.
5. With reference to the period of the Gupta
(a) 1 and 2
dynasty in ancient India, the towns
(b) 2 only Ghantasala, Kadura, and Chaul were known
(c) 1 and 3 as (2020)

Gupta and Post- Gupta Age 78


unacademy.com | Download the Unacademy app
Give your feedback here: Link
(a) ports handling foreign trade weekly wages.
(b) capitals of powerful kingdoms (d) The eldest son of the labourer was sent
(c) places of exquisite stone art and as the forced labourer.
architecture
(d)
important of Buddhist pilgrimage 9. The Chinese traveller Yuan Chwang (Hiuen
centres Tsang) who visited India recorded the
general conditions and culture of India at
that time. In this context, which of the
6. With reference to the history of India, the
following statements is/are correct? (2013)
terms ‘kulyavapa’ and ‘dronavapa’ denote
(2020) 1. The roads and river routes were
completely immune from robbery.
(a) measurement of land
2. As regards punishment for offences,
(b) coins of different monetary value
ordeals by fire, water, and poison were
(c) classification of urban land the instruments for determining the
(d) religious rituals innocence or guilt of a person.

7. With reference to the scholars/litterateurs 3. The tradesmen had to pay duties at


of ancient India, consider the following ferries and barrier stations.
statements: (2020) Select the correct answer using the code
1. Panini is associated with Pushyamitra given below:

2.
Amarasimha is associated with (a) 1 only
Harshavardhana (b) 2 and 3 only
3. Kalidasa is associated with Chandra (c) 1 and 3 only
Gupta-II
(d) 1, 2 and 3
Which of the statements given above is/are
correct?
10. India maintained its early cultural contacts
(a) 1 and 2 only
and trade links with Southeast Asia across
(b) 2 and 3 only the Bay of Bengal. For this pre-eminence
(c) 3 only of early maritime history of Bay of Bengal,
which of the following could be the most
(d) 1, 2 and 3
convincing explanation/explanations?
(2011)
8. With reference to forced labour (Vishti) in (a) As compared to other countries, India
India during the Gupta period, which one of had better ship-building technology in
the following statements is correct? (2019) ancient and medieval times.
(a) It was considered a source of income (b) The rulers of southern India always
for the State, a sort of tax paid by the patronised traders, brahmin priests and
people. Buddhist monks in this context.
(b) It was totally absent in the Madhya (c) Monsoon winds across the Bay of Bengal
Pradesh and Kathiawar regions of the facilitated sea voyages.
Gupta Empire.
(d)
Both (a) and (b) are convincing
(c) The forced labourer was entitled to explanations in this context.

79 Gupta and Post- Gupta Age


unacademy.com | Download the Unacademy app
Give your feedback here: Link
Gupta and Post- Gupta Age-
4 Explanation
1. Answer: (d) While Mitakshara School is followed in most
1. Hoysala: The Hoysala dynasty ruled in parts of India, Dayabhaga School is confined
parts of present-day Karnataka from to Bengal. Certain areas in South India are
the 10th to the 14th century. Their governed by Marumakkatayam, Aliyasantana,
kingdom was established in the 10th and Nambudiri systems of law. Castes are not
century, which is later than the early specified in these laws.
eighth century. Statement 2 is correct: In the Mitakshara
2. Gahadavala: The Gahadavala dynasty school, which prevails in most parts of India,
was a medieval Indian dynasty that a male’s right to be a coparcener is by birth. In
ruled parts of present-day Uttar Pradesh the Mitakshara system, the sons can claim the
and Bihar. They rose to prominence in right to the property during the lifetime of the
the 11th century and their kingdom was father.
established around that time, not in the In the case of the Dayabhaga school of
early eighth century. inheritance, the property is inherited by the
3. Kakatiya: The Kakatiya dynasty was successors (coparceners) only when the
a medieval dynasty that ruled over holder of the property is dead.
Telangana and parts of Andhra Pradesh Statement 3 is not correct: Both laws deal
from the 12th to the 14th century. The with the matters related to the property held
Kakatiya kingdom was established in by both male and female members of a family.
the 12th century, making it later than Generally, it is seen that Mitakshara is biased
the early eighth century. against women & gave them very less right to
4. Yadava: The Yadava dynasty, also inherit property. Although Dayabhaga is also
known as the Seuna dynasty, ruled in biased, it gives more rights to women to inherit
present-day Maharashtra from the 12th property compared to Mitakshara.
to the 14th century. Their kingdom was
established in the 12th century, similar 3. Answer: (b)
to the Kakatiya dynasty, and not in the
Option (b) is correct:
early eighth century.
The Later Guptas succeeded as the rulers
of Magadha after the decline of the Guptas
2. Answer: (b) during the early seventh century. They ruled
Statement 1 is not correct: A Hindu joint family parts of the northern, including Magadha. (Pair
consists of lineal descendants of a common 1 is correctly matched)
ancestor. In other words, a male head and The Pushyabhutis of Thanesar (current day
his descendants, including their wives and Haryana) ruled various parts of Northern India
unmarried daughters. A coparcenary is a during the 6th and 7th century AD. (Pair 3 is
smaller unit of the family that jointly owns the correctly matched)
property. The English term itself is borrowed
The Maukharis dynasty ruled over parts of
from common law, but the concept is found in
Uttar Pradesh and Magadha, including Kannauj,
Hindu law.
during the 5th and 6th century AD. (Pair 4 is
There are two main schools of law followed correctly matched)
by Hindus — Mitakshara and Dayabhaga.

Gupta and Post- Gupta Age-Explanation 80


unacademy.com | Download the Unacademy app
Give your feedback here: Link
The Maitrakas of Valabhi ruled western and coast. There was a great maritime trade
northern parts of current-day Gujarat during between India and Southeast Asia and China.
the late 5th century to late 8th century AD.
(Pair 6 is correctly matched)
6. Answer: (a)
Pair 2 is not correctly matched: The Paramara
dynasty ruled the western and central parts of Option (a) is correct: Gupta Economy: The
India, including Malwa, between the 9th and agricultural crops constituted the main
14th century AD. resources which the society produced and a
major part of the revenue of the state came
Pair 5 is not correctly matched: Yadavas of from agriculture. There are various types of
Devagiri belonged to the Medieval Era. land mentioned in the inscriptions. Land
under cultivation was usually called Kshetra,
4. Answer: (c) Khila was the uncultivable land, Aprahata was
the jungle or forest land, Gopata Sarah was
Option (c) is correct: The movement of goods
the pastureland and Vasti was the habitable
during the Medieval period was facilitated by
land. Different land measures were known in
the growth of a financial system that permitted
different regions such as Nivartana, Kulyavapa,
easy transfer of money from one part of the
and Dronavapa. In the inscriptions of Bengal,
country to another. This was done using
terms like Kulyavapa and Dronavapa have been
Hundis, which were instruments of credit or
used. It is not possible to classify the regions
a bill of exchange. The Hundis was a letter of
precisely according to the crops grown, but all
credit payable after a period at a discount.
the major categories of crops – cereals like
barley, wheat, and paddy, different varieties of
5. Answer: (a) pulses, grams, and vegetables as well as cash
crops like cotton and sugarcane – were known
Option (a) is correct: Indian examples of
long before the Gupta period and continued to
Ancient submerged Port Towns: In India,
be cultivated.
evidence of shipbuilding, port, and warehouses
installations are datable to Harappan culture.
The same maritime traditions continued even 7. Answer: (c)
during the lifetime of Buddha, the Mauryas, the
Statement 1 is not correct: Paṇini was an
Gupta, and in later periods. During the historical
ancient Sanskrit grammarian, and a revered
period, India had trade and cultural contacts
scholar in ancient India. Considered the father
with Egypt, Rome, Greeks, Arabs, China, and
of linguistics, Paṇinilikely lived in the northwest
almost all Southeast Asian countries. through
Indian subcontinent during the Mahajanapada
these ports.
era. His major work is Astadhyayi (Astaka). His
The ports on the west coast were Barygaya, work is dated around the 3rd-4th century BC.
Suppara, Calliena, Semylla, Mandagore,
Pushyamitra Sunga (185 BC TO 151 BC):
Palaepatme, Malizigara, Aurranobbas,
Pushyamitra Sunga was the founder and first
Byzantine, Naura, Tyndis, Muziris and Nelcynda
ruler of the Shunga Empire in East India. He
and the ports on the east coast were Tamralipti,
was a follower of Hinduism. Pushyamitra was
Charitrapur, Paluru, Dantapur, Kalingapatnam,
originally a Senapati “General” of the Mauryan
Pithunda, Sopatma, Ghantasala, Poduca,
Empire. In 185 BCE he assassinated the last
Puhar, Korkai, and Camara. In the Andhra
Mauryan Emperor, BrihadrathaMaurya, during
region, the ports were Kadura and Ghantasala,
an army review, and proclaimed himself
Kaveripattanam (Puhar), and Tondail. These
emperor.
were the ports of the Pandya region. The ports
of Kottayam and Muziris were on the Malabar Statement 2 is not correct: Amarasimha Is a

81 Gupta and Post- Gupta Age-Explanation


unacademy.com | Download the Unacademy app
Give your feedback here: Link
Sanskrit grammarian and poet from ancient the extensive inventions and discoveries in
India and was one of the Navratnas during the science, technology, engineering, art, dialectic,
Gupta dynasty’s rule which happened around literature, logic, mathematics, astronomy,
the mid-to-late 3rd century (approximately) religion, and philosophy that illuminated the
to 543 AD. It was founded by Sri Gupta, but elements of Hindu Culture.
the dynasty rose to fame with rulers like Option (a) is correct: In the Gupta Age, the term
Chandragupta-I, Samudragupta, etc. Vishti referred to Forced labour, which was
King Harshavardhana, also known as Harsha, extracted by either state, provincial governor,
was the son of Prabhakara Vardhana. or local chief. It is also mentioned in the Gupta
Prabhakara Vardhana was the founder of the era copper inscriptions that enlist a variety of
Pushyabhuti Dynasty or the Vardhana Dynasty. other taxes. The Junagarh inscription mentions
Harshavardhana is considered as one of vishti as one form of tax, which suggests that
the most prominent Indian emperors in the it was extracted from Gujarat and the Malwa
7th century AD. He built a huge empire that region. It was not wage labour as it was forced
extended from north & northwestern India labour in lieu of tax. Wage labour did exist
till the Narmada in the South. His capital in ancient India, but it was not called vishti.
was Kannauj. His reforms and policies were Vishti could be extracted from anyone, not in
generous and were always aimed at boosting particular from the eldest son only.
the peace and prosperity of his people.
Statement 3 is correct: Chandragupta II, also 9. Answer: (b)
called Vikramaditya, was a powerful emperor
(reigned c. 380–c. 415 c) of northern India. Yuan Chwang (Hiuen Tsang) visited India during
He was the son of Samudra Gupta and the Harsha Vardhana’s reign in the 7th Century AD,
grandson of Chandragupta I. During his reign, in search of Buddhist scriptures. When he went
art, architecture, and sculpture flourished, back to China, he wrote a detailed description
and the cultural development of ancient India of India during the reign of Harsha in his book
reached its apex. Chandragupta-II was known ‘Si-yu-ki’ or ‘Record of the Western Countries’.
for his deep interest in art and culture. Kalidasa Statement 1 is not correct: According to Yuan
was court poet of Chandragupta Vikramaditya Chwang, the roads and river routes were not
ll. He was a great poet and dramatist. His work completely immune from robbery. He described
Abhijanasakuntalam is one of the 100 best that travelling was not very safe at that time,
literary works in the world. he himself was robbed twice.

Elimination Technique: Amarasimha is Statement 2 is correct: He described that the


associated with the Gupta Dynasty. So, kingdom was well-governed. It was- free from
by eliminating statement 2, we get the revolts, there were a few cases of law-breaking.
answer C. The narrative proceeds to describe the four
ordeals by which the innocence or guilt of an
accused person is determined. These are by
8. Answer: (a) water, by fire, by weighing, and by poison.
The Gupta Empire came into power in around Statement 3 is correct: Under the Revenue
275 AD. The ruling dynasty of the empire was and taxation system, Yuan Chwang notes
founded by king Sri Gupta and the most notable that tradesmen go to and fro bartering their
rulers of the dynasty were Chandragupta I, merchandise after paying light duties at ferries
Samudragupta, and Chandragupta II alias and barrier stations.
Vikramaditya. The Gupta Empire is referred
to as the Golden Age of India because of Elimination Technique: The word

Gupta and Post- Gupta Age-Explanation 82


unacademy.com | Download the Unacademy app
Give your feedback here: Link
the days of the composition of Yuktikalpataru
‘completely immune’ mentioned in the (around ninth century CE), it appears that ship-
statement 1 makes it rigid and incorrect. building was highly advanced. Hence option 1 is
So, by eliminating it we can get the answer B. correct. Yuktikalpataru written by King Bhoja.
The rules of southern India always patronized
traders, brahmin priests and Buddhist monks
10. Answer: (d)
in this context. Kanishka patronized Buddhist
The correct answer is (d) Both (a) and (b) are monks, Kushans patronized traders, Cholas
convincing explanations in this context. The patronized brahmin priests. Hence statement 2
maritime history of Indian shipbuilding begins is correct. ​Indians had Knowledge of Monsoon
right from the time of the Harappan Civilization. winds since the Gupta period. Monsoon
As compared to other countries, India had winds across the Bay of Bengal facilitated sea
better ship-building technology in ancient and voyages, but this was not the main reason that
medieval times. It was during the Medieval India maintained its early cultural contacts and
period that a number of Indian Vessels were trade links with Southeast Asia across the Bay
constructed purely for waging war at sea. During of Bengal. Hence, statement 3 is incorrect.

83 Gupta and Post- Gupta Age-Explanation


unacademy.com | Download the Unacademy app
Give your feedback here: Link
5 Sangam Age

1. With reference to ancient South India, 4. Consider the following events in the history
Korkai, Poompuhar and Muchiri were well of India: (2020)
known as (2023) 1. Rise of Pratiharas under King Bhoja
(a) capital cities 2. Establishment of Pallava power under
(b) ports Mahendravarman-I
(c) centres of iron-and-steel making 3. Establishment of Chola power by
(d) shrines of Jain Tirthankaras Parantaka-I
4. Pala dynasty founded by Gopala

2. Which one of the following explains the What is the correct chronological order of
practice of Vattakirutal' as mentioned in the above events, starting from the earliest
Sangam poems? (2023) time?

(a) Kings employing women bodyguards (a) 2 – 1 – 4 – 3

(b)
Learned persons assembling in (b) 3 – 1 – 4 – 2
royal courts to discuss religious and (c) 2 – 4 – 1 – 3
philosophical matters (d) 3 – 4 – 1 – 2
(c)
Young girls keeping watch over
agricultural fields and driving away birds
and animals 5. In the context of the history of India,
consider the following pairs: (2016)
(d) A king defeated in a battle committing
ritual suicide by starving himself to Term Description
death 1. Eripatti Land, revenue from
which was set apart for
the maintenance of the
3. Which one of the following statements
village tank
about Sangam literature in ancient South
India is correct ? (2022) 2. Taniyurs Villages donated to a
single Brahmin or a
(a) Sangam poems are devoid of any
group of Brahmins
reference to material culture.
3. Ghatikas Colleges generally
(b) The social classification of Varna was
attached to the temples
known to Sangam poets.
Which of the pairs given above is/are
(c) Sangam poems have no reference to correctly matched?
warrior ethics.
(a) 1 and 2
(d) Sangam literature refers to magical
(b) 3 only
forces as irrational.
(c) 2 and 3
(d) 1 and 3

Sangam Age 84
unacademy.com | Download the Unacademy app
Give your feedback here: Link
5 Sangam Age-Explanation

1. Answer: (b) Statement d is incorrect because


Chola, Chera and Pandyas were the early Sangam literature doesn’t refers
dynasties of the Sangam Age. Each of them to magical forces as irrational.
had ports which were used to trade with Other Aṇaṅku is a phenomenon of sacred power
countries. Example: Muziris, Korkai, Puhar described in ancient Tamil literature such as
(Poompuhar), Tondai, Arikamedu. the sangam literature. This sacred magical
forces were supposed to inhabit various
objects, which eventually determined their
association with the society.
Statement b is correct because, The varna
system was not practiced but it was known.
(Tolkappiyam refers to 4 castes)

4. Answer: (c)
The Pallavas were a powerful ancient dynasty
that ruled a huge part of Southern India,
including present day Tamil Nadu, between the
6th and 9th centuries AD, with Kanchipuram
as their capital. The Pallavas reached their
zenith during the reign of Mahendravarman I
(600–630 AD), a contemporary of Harsha and
Pulakeshin II.
The Pala Empire was an imperial power during
2. Answer: (d) the post-classical period (8th and 9th century
AD) which originated in the Bengal region.
Vatakkiruttal (Vaṭakkiruttal, ‘fasting facing
The empire was founded with the election of
north’), also Vadakiruthal and vadakiruttal,
Gopala as the emperor of Gauda in 750 CE.
was a Tamil ritual of fasting till death. It was
especially widespread during the Sangam age. The GurjaraPratihara dynasty was founded
The Tamil kings, in order to save their honour, by Nagabhatta I in the region of Malwa in the
and prestige, were prepared to meet their 8th century AD. He belonged to a Rajput clan.
death facing North (‘Vatakkiruttal’), and never The Gurjara-Pratihara dynasty ruled much of
would they turn their back in battle. It was a Northern India from the mid-8th to the 11th
Tamil martial. This was either done alone or as century. The greatest ruler of the Pratihara
a group with the supporters of the captured dynasty was Mihira Bhoja (836–885 CE). He
king. recovered Kannauj (Kanyakubja) by 836 A.D,
and it remained the capital of the Pratiharas
for almost a century.
3. Answer: (b)
The Cholas became prominent in the ninth
Sangam Literature has references to material century and established an empire comprising
culture and war tactics and ethics. Hence the major portion of South India. Their capital
statements a and c are incorrect. was Tanjore. Aditya I or Aditya Chola, the son

85 Sangam Age-Explanation
unacademy.com | Download the Unacademy app
Give your feedback here: Link
of VijayalayaChola, was the second ruler of the individuals, the revenue from which was set
Medieval Cholas. The Chola ruler Parantaka apart for the maintenance of the village tank.
succeeded his father Aditya I on the throne in This indicates that the village was dependent
907 AD and ruled for forty-eight years. Soon on the tank for irrigation.
after his accession, as early as 910 AD, he Pair 2 is not correctly matched: Taniyur, are
invaded the Pandyan country and assumed the the large villages under Chola administrative
title Maduraikonda (‘Capturer of Madura’). division. These were distinct revenue units.
Pair 3 is correctly matched: In South India
5. Answer: (d) centers of higher learning were known as
Pair 1 is correctly matched: A special category Ghatikas. Ghatikas and Mathas were attached
of land, Eripatti or tank land, was known only to temples. There was a famous Ghatika at
in south India. This was the land donated by Kanchi, drawing students from far and near.

Sangam Age-Explanation 86
unacademy.com | Download the Unacademy app
Give your feedback here: Link
3 MEDIEVAL
INDIA
Delhi Sultanate (1206 AD to 1526
1 AD)
1. Who among the following rulers of da Gama reached the coast of Kerala.
Vijayanagara Empire constructed a large Which of the statements given above is /
dam across Tungabhadra River and a canal- are correct?
cum-aqueduct several kilometres long
from the river to the capital city? (2023) (a) 1 only

(a) Devaraya I (b) 1 and 2

(b) Mallikarjuna (c) 3 only

(c) Vira Vijaya (d) 2 and 3

(d) Virupaksha
4. Consider the following statements: (2019)

2. With reference to Indian history, consider 1. In the revenue administration of the


the following statements : (2022) Delhi Sultanate, the in charge of revenue
collection was known as ‘Amil’.
1. The first Mongol invasion of India
happened during the reign of Jalal-ud- 2. The iqta system of Sultans of Delhi was
din Khalji. an ancient indigenous institution.

2. During the reign of Ala-ud-din Khalji, 3. The office of ‘Mir Bakshi’ came into
one Mongol assault marched up to Delhi existence during the reign of Khalji
and besieged the city. Sultans of Delhi.

3. Muhammad-bin-Tughlaq temporarily Which of the statements given above is/are


lost portions of north-west of his correct?
kingdom to Mongols. (a) 1 only
Which of the statements given above is/are (b) 1 and 2 only
correct? (c) 3 only
(a) 1 and 2 (d) 1, 2 and 3
(b) 2 only
(c) 1 and 3 5. Banjaras during the medieval period of
(d) 3 only Indian history were (2016)
(a) Agriculture
3. Consider the following statements: (2021) (b) Warriors
1. It was during the reign of Iltutmish (c) Weavers
that Chengiz Khan reached the Indus in (d) Traders
pursuit of the fugitive Khwarezm prince.
2. It was during the reign of Muhammad
6. With reference to the economic history of
bin Tughluq that Taimur occupied
medieval India, the term ‘Araghatta’ refers
Multan and crossed the Indus.
to (2016)
3. It was during the region of Deva Raya II
(a) bonded labor
of the Vijayanagara Empire that Vasco

88 Delhi Sultanate (1206 AD to 1526 AD)


unacademy.com | Download the Unacademy app
Give your feedback here: Link
(b) land grants made to military officers structure based on control and
(c) waterwheel used in the irrigation of possession of the land.
land 3. Creation of lord-vassal relationship
(d) wasteland converted to cultivated land between the feudal lord and his
overlord.
Select the correct answer using the code
7. With reference to Indian history, which of
given below:
the following is/are the essential element/
elements of the feudal system? (2015) (a) 1 and 2 only

1. A very strong centralized political (b) 2 and 3 only


authority and a very weak provincial or (c) 3 only
local political authority.
(d) 1, 2 and 3
2. The emergence of an administrative

Delhi Sultanate (1206 AD to 1526 AD) 89


unacademy.com | Download the Unacademy app
Give your feedback here: Link
Delhi Sultanate (1206 AD to 1526
1 AD)-Explanation
1. Answer: (a) while escaping from the Mongols, sought
Deva Raya-I from vijayanagar empire shelter at Iltutmish’s court, Iltutmish turned
constructed a dam across the Tungabhadra him away. He thus saved the Sultanate from
River to relieve the regions in the Vijayanagar destruction by the Mongols.
Empire that faced shortage of water and to Statement 2 is not correct: Muhammad bin
promote agriculture, which led to increase in Tughluq reigned from 1324 to 1351. Taimur
revenues by 350000 pardos. occupied Multan and crossed the river in 1398.
Statement 3 is not correct: Vasco da Gama
2. Answer: (b) reached the coast of Kozhikode (Calicut),
Kerala, in 1498. During this time, Narasimha
Statement 1 is incorrect: The first Mongol Raya II was the ruler of the Vijayanagara Empire.
invasion of India took place in the reign of He was the third and last ruler of the Saluva
Sultan Shams-ud-din lltutmish. dynasty. Deva Raya II died in 1446.
Statement 2 is correct: The Mongols besieged
the city during the reign of Alauddin Khilji.
4. Answer: (a)
Statement 3 is incorrect: Mongols did not
annex the North-West part of the kingdom of Statement 1 is correct: Under the Iqtadari
Muhammad-bin-Tughlaq. system, the lands were called iqta and their
holder was called iqtadar or muqti. Besides
Elimination Technique: this, during the sultanate of Iltutmish, the
During the reign of Ala-ud-din Khalji, one provinces were divided into six parts headed
Mongol assault marched up to Delhi and by shiqda. The Shiqda were further divided into
besieged the city. If you know the 2nd Parganas and had different officials which were:
statement is right, then options (c) and (d) Amil, Mushrif-i-mumalik, Hazamdars, Qazi-
can be easily eliminated. Civil officials, Shiqdar, Kotwal, Faujdar, Amin,
Qanungo, and Patwari. Amil is the officers who
collected land revenue and other taxes.
3. Answer: (a)
Statement 2 is not correct: The Iqta system
Statement 1 is correct: When Iltutmish ascended evolved in West Asia significantly in Persia. It
the throne, he found himself surrounded was not an ancient indigenous institution as it
by many problems. Other commanders of had been in force in the early Islamic world as
Muhammad Ghori rose in defiance again. The a form of reward for services to the state.
chief of Jalor and Ranthambore joined Gwalior
Statement 3 is not correct: Mir Bakshi was the
and Kalinjar in declaring their independence.
head of the military department that came
Apart from this, the rising power of Mongols
into existence in Mughal India and not Delhi
under Chengiz Khan threatened the North-
Sultanate. Mir Bakshi handled the military
West Frontier of the Sultanate.
pay and accounts and related duties and was
In AD 1220, when Chenghiz Khan destroyed considered the head of nobility. He not only
the Khwarezm empire, Iltutmish realised the was the Paymaster for all officers but also
political necessity of avoiding a confrontation played a role in the recruitment of soldiers,
with the Mongols. Thus, when Jalaluddin listing of mansabdars, and important officials.
Mangbarani, the son of the Shah of Khwarizm,

90 Delhi Sultanate (1206 AD to 1526 AD)-Explanation


unacademy.com | Download the Unacademy app
Give your feedback here: Link
5. Answer: (d) empire disappeared, to the 12th century,
Option (d) is correct: Banjaras during the when kingdoms began to emerge as effective
medieval period of Indian history were generally centralized units of government. It was a way
traders. Banjaras played a role in Alaudin khilji of structuring society around relationships
‘s market reforms. The Banjara are a community derived from the holding of land in exchange
of nomadic people with their origins in for service or labor (lord-vassal relationship).
Rajasthan. They were the trading groups that y Feudalism usually emerged because of
moved from one place to another in buying the decentralization of an empire. This
and selling goods. Their population is divided was especially the case in the Carolingian
into Kashmir, Bengal, Gujarat, and Karnataka, empires that lacked the bureaucratic
and are now found all over India. They are also infrastructure necessary to support cavalry
called Lamans, Lambadi, Lambhani, Lambani, without the ability to allocate land to
and Gormati. these mounted troops. (Statement 1 is not
correct)
Elimination Technique:
y Use of the term feudalism to describe India
Option (a), (b) and (c) can be easily applies a concept of medieval European
eliminated, as it is not related to warriors, origin, according to which the landed
agricultures and weavers. nobility held lands from the Crown in
exchange for military service, and vassals
6. Answer: (c) were in turn tenants of the nobles, while
the peasants (villeins or serfs) were
Option (c) is correct: The term ‘Araghatta’
obliged to live on their lord’s land and
comes from the combination of the words ‘ara’
give him homage, labor, and a share of the
meaning spoke and ‘ghatta’ meaning pot. It is
produce, notionally in exchange for military
a Persian wheel used in irrigation in medieval
protection. The term Indian feudalism is an
India. It is a mechanical device used to lift water.
attempt to classify Indian history according
It uses animal power to lift water from sources,
to a European model. (Statement 2 and
especially from open wells. The animals used
statement 3 are correct)
here are cattle, bullocks, etc. With its use also
in Iran, then Persia, and perhaps its discovery Elimination Technique:
there, it came to be called the Persian wheel. Statement 1 is not correct, A very strong
centralized political authority and a very
7. Answer: (b) weak provincial or local political authority.
It was a very common feature of the
The terms feudalism and feudal system were
feudal system. So options (a) and (d) can
generally applied to the early and central
be easily eliminated.
Middle Ages—the period from the 5th century,
when central political authority in the Western

Delhi Sultanate (1206 AD to 1526 AD)-Explanation 91


unacademy.com | Download the Unacademy app
Give your feedback here: Link
Mughal Empire (1526 AD to 1761
2 AD)
1. “Yogavasistha” was translated into Persian 5. With reference to Mughal India, what is/
by Nizamuddin Panipati during the reign of: are the difference/differences between
(2022) Jagirdar and Zamindar? (2019)
(a) Akbar 1. Jagirdars were holders of land
(b) Humayun assignments in lieu of judicial and police
duties, whereas Zamindars were holders
(c) Shahjahan of revenue rights without obligation to
(d) Aurangzeb perform any duty other than revenue
collection.

2. With reference to medieval India, which one 2. Land assignments to Jagirdars were
of the following is the correct sequence in hereditary and the revenue rights of
ascending order in terms of size? (2021) Zamindars were not hereditary.

(a) Paragana—Sarkar—Suba Select the correct answer using the code


given below:
(b) Sarkar—Paragana—Suba
(a) 1 only
(c) Suba—Sarkar—Paragana
(b) 2 only
(d) Paragana—Suba—Sarkar
(c) Both 1 and 2
(d) Neither 1 nor 2
3. Who among the following Mughal Emperors
shifted emphasis from illustrated
manuscripts to the album and individual 6. Consider the following: (2015)
portraits? (2019) The arrival of Babur into India led to the
(a) Humayun 1. introduction of gunpowder
(b) Akbar 2. introduction of the arch and dome in
(c) Jahangir the region’s architecture
(d) Shah Jahan 3. establishment of the Timurid dynasty
Select the correct answer using the code
4. With reference to Mian Tansen, which one given below:
of the following statements is not correct? (a) 1 and 2 only
(2019) (b) 3 only
(a) Tansen was the title given to him by (c) 1 and 3 only
Emperor Akbar.
(d) 1, 2 and 3
(b) Tansen composed Dhrupads on Hindu
gods and goddesses.
(c) Tansen composed songs for his patrons. 7. Ibadat Khana at Fatehpur Sikri was (2014)

(d) Tansen invented many Ragas. (a) the mosque for the use of the Royal
Family
(b) Akbar’s private prayer chamber

92 Mughal Empire (1526 AD to 1761 AD)


unacademy.com | Download the Unacademy app
Give your feedback here: Link
(c) the hall in which Akbar held discussions
with scholars of various religions
(d) the room in which the nobles belonging
to different religions gathered to discuss
religious affairs

Mughal Empire (1526 AD to 1761 AD) 93


unacademy.com | Download the Unacademy app
Give your feedback here: Link
Mughal Empire (1526 AD to 1761
2 AD)-Explanation
1. Answer: (a) 4. Answer: (a)
Option (a) is correct: Option (a) is not correct: Tansen was a court
YogaVasistha was translated into Persian musician in the darbar of Raja Ram Chandra
multiple times throughout the Mughal Dynasty, of Bandavagarh (Rewa). Tansen was the title
as commanded by Akbar, Jahangir, and Darah given to him by Raja Vikramjit of Gwalior.
Shikuh. Nizam al-Din Panipati completed Tansen is also known as the ‘Sangit Samrat’,
one of these translations, known as the Jug- according to Musical Heritage of India by Lalita
Basisht, in the late sixteenth century AD. Ramakrishna. He composed many dhrupads
on Ganesha, Shiva, Parvati, and Rama. He also
composed songs for his patrons. Kalpadruma
2. Answer: (a) is a compilation of 300 of his dhrupads that
Option (a) is correct: During the Mughal time, were in Gauhar Bani. Some noteworthy ragas
there were large provinces called Suba. Every were composed by Tansen- Multani, Bhairavi,
Suba was divided into several Sarkars or and Todi.
districts. Each Sarkar was divided into smaller Additional Information:
units called Parganas. So, the correct order is When Akbar heard of his prodigious talent, a
Paragana - Sarkar – Suba. ‘firman’ was sent by him to the king, asking
The sarkars, into which each suba was divided, for Tansen. Akbar then made him one of the
often overlapped with the jurisdiction of Navaratnas in his court. He gave him the title
faujdars (commandants), who were deployed of ‘Mian’.
with contingents of heavy cavalry and
musketeers in districts.
5. Answer: (d)
The local administration was looked after at
the level of the pargana (sub-district) by three The two main institutions, namely Mansabdari
semi-hereditary officers, the qanungo (keeper and the Jagirdari, were created by Akbar to
of revenue records), the chaudhuri (in charge consolidate and to expand his empire. These
of revenue collection), and the qazi. covered both civil and military sectors of
administration.
Statement 1 is not correct: Those nobles who
3. Answer: (c) joined Mughal service were commissioned as
Option (c) is correct: Akbar’s reign saw Mansabdars. Jagirdars were those mansabdars
illustrations of manuscripts, the prominent who were given salary in lieu of military services
being Razmnama which is the Persian in terms of jagirs (land) and Zamindars were the
translation of Mahabharata. It was during local elite or rural aristocracy who exercised
Jahangir’s reign that manuscripts became authority in their areas. The peasants paid
less important and there was more focus on taxes to the Mughals through the Zamindars
individual portraits. Under Jahangir, paintings and they also performed judicial and police
acquired greater charm, refinement, and functions locally.
dignity. Jahangir had a great fascination for Statement 2 is not correct: The land was not
nature and took pleasure in the portraiture of assigned to the Jagirdars but only the right to
birds, animals, and flowers. collect revenue or income from the piece of
land. They were allocated non-inheritable land

94 Mughal Empire (1526 AD to 1761 AD)-Explanation


unacademy.com | Download the Unacademy app
Give your feedback here: Link
areas equivalent to their fee amount called Afghanistan, much of Central Asia, as well
jagir. After the death of a mansabdar, all his as parts of contemporary Pakistan, India,
jagirs and wealth was returned to the state. Mesopotamia, Anatolia, and the Caucasus.
Zamindars also enjoyed certain social and The dynasty was founded by Timur
economic privileges because of their superior (Tamerlane) in the 14th century.
status in rural society. Caste was one factor that
resulted in the eminent status of zamindars. Elimination Technique:
Unlike Jagirdars, the post of Zamindars was Gunpowder was introduced by the
hereditary. Mongols. So, if you know statement 1 is
not correct, then you can easily mark the
answer (b) by eliminating all other options.
6. Answer: (b)
Statement 1 is not correct: Gunpowder was
introduced in the subcontinent, probably in
the 13th century by the Mongols, who had 7. Answer: (c)
conquered both China and some borderlands Muhammad Akbar, during his reign between
of India. The unification of a large single Mongol 1556-1605, did a lot of work for the ‘imposition
Empire resulted in the free transmission of of reason’. Throughout his reign, there was a
previously top-secret Chinese technology stress on reason (aql), which was to be given
into Mongol-conquered parts of India. It is precedence over traditionalism (taqlid).
also believed that the Mongols used Chinese y It is believed that Akbar was a big follower
gunpowder weapons during their invasions of of Sheikh Salim Chishti, for he gave credit
India. to the Sheik for winning battles and the
Statement 2 is not correct: Arch and dome birth of his son. The Sheikh’s predictions
architecture was widely prevalent during the strengthened the emperor’s faith in Islam
reign of the Delhi Sultanate, much earlier than and he became eager to learn more.
the arrival of Babur. y This led to the construction of ‘Ibadat
Statement 3 is correct: In the 16th century, Khana’ in 1575, where Muslim scholars
Babur, a Timurid Prince from Ferghana (modern would come together to have discussions
Uzbekistan), invaded Kabulistan (modern on Islam, thus enlightening the emperor.
Afghanistan) and established a small kingdom The ‘House of Worship’ soon became a place
there, and from there 20 years later, he invaded for arguments, which led to Akbar opening
Hindustan (present-day India) to establish the the gates for scholars of other religions,
Timurid dynasty in India which is known as the thus making it the first-ever attempt at
Mughal Empire. secularism, also known as ‘Din-i-Ilahi’.
y The Timurid dynasty was a Persianate, Here, the best elements from all religions
Sunni Muslim dynasty of Turco-Mongol were encouraged. [Option (c) is correct]
lineage that ruled over modern-day Iran,

Mughal Empire (1526 AD to 1761 AD)-Explanation 95


unacademy.com | Download the Unacademy app
Give your feedback here: Link
Provincial Kingdoms in Medieval
3 India
1. In medieval India, the term “Fanam” 4. Which one of the following foreign
referred to: (2022) travellers elaborately discussed diamonds
(a) Clothing and diamond mines of India? (2018)

(b) Coins (a) Francois Bernier

(c) Ornaments (b) Jean-Baptiste Tavernier

(d) Weapons (c) Jean de Thevenot


(d) Abbe Barthelemy Carre

2. Consider the following pairs : (2022)


King Dynasty 5. Which one of the following was a very
important seaport in the Kakatiya kingdom?
1. Nannuka Chandela (2017)
2. Jayashakti Paramara (a) Kakinada
3. Nagabhata II Gurjara-Pratihara (b) Motupalli
4. Bhoja Rashtrakuta (c) Machilipatnam (Masulipatnam)
How many pairs given above are correctly (d) Nelluru
matched?
(a) Only one pair
6. Regarding the taxation systems of Krishna
(b) Only two pairs Deva, the ruler of Vijayanagar, consider the
(c) Only three pairs following statements: (2016)
(d) All four pairs 1. The tax rate of land was fixed depending
on the quality of the land.

3. With reference to Indian history, which of 2. Private owners of workshops paid an


the following statements is/are correct? industry tax.
(2021) Which of the statements given above is/are
1. The Nizamat of Arcot emerged out of correct?
Hyderabad state. (a) 1 only
2. The Mysore kingdom emerged out of (b) 2 only
Vijayanagara Empire. (c) Both 1 and 2
3. Rohilkhand kingdom was formed out of (d) Neither 1 nor 2
the territories occupied by Ahmad Shah
Durrani.
Select the correct answer using the code 7. Consider the following pairs: (2015)
given below. Medieval Indian Present Region
(a) 1 and 2 State
(b) 2 only 1. Champaka Central India
(c) 2 and 3 2. Durgara Jammu

(d) 3 only

96 Provincial Kingdoms in Medieval India


unacademy.com | Download the Unacademy app
Give your feedback here: Link
3. Kuluta Malabar supposed to belong? (2015)

Which of the above pairs is/are correctly (a) Amoghavarsha I


matched? (b) Ballala II
(a) 1 and 2 only (c) Harihara I
(b) 2 only (d) Prataparudra II
(c) 1 and 3 only
(d) 3 only 9. In medieval India, the designations
'Mahattara' and 'Pattakila' were used for
(2014)
8. Who of the following founded a new city
on the south bank of a tributary to river (a) military officers
Krishna and undertook to rule his new (b) village headmen
kingdom as the agent of a deity to whom (c) specialists in Vedic rituals
all the land south of the river Krishna was
(d) chiefs of craft guilds

Provincial Kingdoms in Medieval India 97


unacademy.com | Download the Unacademy app
Give your feedback here: Link
Provincial Kingdoms in Medieval
3 India-Explanation
1. Answer: (b) into an autonomous state by the Wodeyar
Option (b) is correct: During the Vijaynagar dynasty. The Mysore Kingdom emerged out of
Empire, Fanams were coins which were used Vijayanagara Empire.
in day-to-day life. Fanam was usually a small Statement 3 is not correct: The states of
fraction of the main currency (Pagoda or Varaha). Rohilkhand and the kingdom of the Bangash
The Travancore Fanam was a form of money Pathans were a fallout of the Afghan migration
issued by the State of Travancore in South India, into India. Large-scale immigration of Afghans
which is today mostly a part of Kerala. Fanams into India took place in the mid-18th century
(sometimes spelled Fanoms) and Chuckrams because of political and economic turmoil
(or Chakrams) were renowned to be among the in Afghanistan. Ali Muhammad Khan took
world’s tiniest coins. Fanam appears to be an advantage of the collapse of authority in North
Anglo-Germanic sound shift from Panam, which India following Nadir Shah’s invasion to set up
in Dravidian languages denotes money. The a petty kingdom, Rohilkhand. This was the area
Fanam and Chuckram coins were the standard of the Himalayan foothills between Kumaon in
units of currency in medieval Travancore, and the north and the Ganga in the south.
they appear to have been widely utilised for
trade in the South Indian region. In Kerala’s Elimination Technique:
original language, Malayalam, the words Fanam
Here statement 3 is not correct, then you
and Panam literally mean money and are still
can easily eliminate (c) and (d) options. It
used as synonyms for riches.
was Ali Muhammad Khan not Ahmad Shah
Durrani.
2. Answer: (b)
Option (b) is correct: 4. Answer: (b)
Pair 1 is correct: Nanuka was the founder of Option (b) is correct: Jean-Baptiste Tavernier
the Chandela Dynasty. (1605‒89) was one of the most renowned
Pair 2 is incorrect: Jayashakti was also a travellers of 17th-century Europe. The son of
Chandela Ruler. a French Protestant who had fled Antwerp to
escape religious persecution, Tavernier was a
Pair 3 is correct: Nagabhata II belonged to the jewel merchant who between 1632 and 1668
Gurjara Pratihara Dynasty. made six voyages to the East. The countries
Pair 4 is incorrect: As Bhoja was a Parmara. he visited (most more than once) included
present-day Cyprus, Malta, Turkey, Syria, Iraq,
Iran, Afghanistan, Pakistan, India, Sri Lanka,
3. Answer: (b) and Indonesia. While on his travels to India
Statement 1 not is correct: Nizamat of Arcot he discovered and discussed diamonds and
was established by Mughal Emperor Aurangzeb diamond mines of India extensively.
and was the legal dependency of the Nizam
of Hyderabad; it did not emerge from the
Hyderabad state. 5. Answer: (b)
Statement 2 is correct: Mysore was transformed Option (b) is correct: The Kakatiya dynasty
from a viceroyalty of the Vijayanagara Empire was a South Indian dynasty, and the capital

98 Provincial Kingdoms in Medieval India-Explanation


unacademy.com | Download the Unacademy app
Give your feedback here: Link
of the Kakatiya dynasty was Orugallu which maintained their identity until recent years,
is known as Warangal. Motupalli was an despite their wars with each other and the
important seaport in the kingdom of Ganpati frequent raids from the men of the plains.
(an important Kakatiya ruler), frequented by States such as Champaka (Chamba in Himachal
foreign merchants. Italian traveler Marco Polo Pradesh), Durgara (Jammu), Trigarta (Jalandhar),
who had visited the port in 1287 AD, had given a Kuluta (Kulu in Himachal Pradesh), Kumaon
vivid account on the flourishing trade activities and Garhwal managed to remain outside the
at the port during the reign of Kakatiya queen main areas of conflict in the northern plains.
Rudrama Devi.
Plans are afoot for the integrated development 8. Answer: (c)
of Motupalli, a historic port in Chinnaganjam
mandal of the Prakasam district. The tourism Option (c) is correct: Vijayanagara or “city of
potential of the village, where the bronze idols victory”, was the name of both a city and an
of Lord Nataraja, his consort Parvati, their son empire. Harihara I and Bukka founded the
Ganesha and Alwars, the Bhakti movement Vijayanagar Empire in the fourteenth century.
saints dating back to the 12th century, were It stretched from the river Krishna in the north
found in the 1970s, has remained hitherto to the extreme south of the Peninsula (Krishna-
untapped. Tungabhadra doab).
y Harihara I established control over the
valley of the Tungabhadra River (a tributary
6. Answer: (c) of Krishna). Then gradually he expanded his
Statement 1 is correct: In the reign of Krishna control to certain regions of Konkan and
Deva, there existed a modern and fully Malabar Coast.
functional taxation system. Land revenue was y By that time Hoysalas had lost its last
the main source of income of the government. ruler Veera Ballala III, who died fighting
To assess the proper revenue, the entire land the Sultan of Madurai, and the vacuum
was classified into four parts: the wetland, the created facilitated Harihara I to emerge
dry land, the orchards, and the woods and the as a sovereign power. The entire Hoysala
tax rate of land was fixed depending on the territories came under his rule directly.
quality of the land.
Statement 2 is correct: Besides land tax, the
government also levied other taxes. Private 9. Answer: (b)
owners of workshops paid an industry tax. Option (b) is correct: Mahattara means the
Commercial taxes consisted of levies, duties, principal, chief, or oldest person, the most
and customs on manufactured articles of trade respectable person. In Medieval India Mahattara
were also levied. was Headman among the Agrahdrinas, that is,
the owner of an agrahara, a rent-free village in
the possession of Brahmanas.
7. Answer: (b)
Pattakila means village headman, the same as
The foothills of the Himalayas lent themselves modern-day Patel or Patil. Pattakila is believed
quite well to such small kingdoms, owing to to be the king’s representative in the village.
the nature of the country. Unlike Pattakila, Mahattara was a non-official
Option (b) is correct: The ninth century saw and possibly a representative of the people.
the rise of several hill states, some of which

Provincial Kingdoms in Medieval India-Explanation 99


unacademy.com | Download the Unacademy app
Give your feedback here: Link
Religious movement during
4 medieval period
1. According to Portuguese writer Nuniz, the Which of the statements given above is/are
women in the Vijayanagara Empire were correct?
experts in which of the following areas? (a) 1 only
(2021)
(b) 2 only
1. Wrestling
(c) Both 1 and 2
2. Astrology
(d) Neither 1 nor 2
3. Accounting
4. Soothsaying
3. With reference to the cultural history of
Select the correct answer using the code medieval India, consider the following
given below. statements: (2016)
(a) 1, 2 and 3 only 1. Siddhas (sittars) of the Tamil region were
(b) 1, 3 and 4 only monotheistic and condemned idolatry.
(c) 2 and 4 only 2. Lingayats of the Kannada region
(d) 1, 2, 3 and 4 questioned the theory of rebirth and
rejected the caste hierarchy.
Which of the statements given above is/are
2. Consider the following statements: (2019) correct?
1. Saint Nimbarka was a contemporary of (a) 1 only
Akbar.
(b) 2 only
2. Saint Kabir was greatly influenced by
Shaikh Ahmad Sirhindi. (c) Both 1 and 2
(d) Neither 1 nor 2

100 Religious movement during medieval period


unacademy.com | Download the Unacademy app
Give your feedback here: Link
Religious movement during
4 medieval period-Explanation
1. Answer: (d) the 13th century in South India. He was a yogi,
Option (d) is correct: Founded in 1336, philosopher, and prominent astronomer.
Vijayanagara, literally ‘city of victory’, attracted Statement 2 is not correct: Sant Kabir Das was
a number of international travellers such as a renowned saint, poet and social reformer of
the Italian, Niccolo di Conti, the Portuguese India who lived during the 15th century. His
Domingo Paes, Fernao Nuniz and Duarte writings significantly influenced the Bhakti
Barbosa and the Afghan Abd, al-Razzaq, who movement. Some of his famous writings
have left vivid accounts of the city. include ‘Sakhi Granth’, ‘Anurag Sagar’, ‘Bijak’
Fernao Nuniz was a Portuguese traveller, and ‘Kabir Granthawali’. He also founded a
chronicler and horse trader who spent three religious community named ‘Kabir Panth’ whose
years in Vijayanagara. Nuniz writes: “he (the members are referred to as ‘Kabir Panthis’,
king of Vijaynagar) has also women who implying the followers of Kabir Das. Vaishnava
wrestle, and others who are astrologers and saint Swami Ramananda, who accepted Kabir
soothsayers; and he has a woman who writes as his disciple, greatly influenced his ideologies.
all the accounts of expenses that are incurred Saint Kabir was 15th century poet whereas
inside the gates and others whose duty it is Sirhindi was in 17th century.
to write all the affairs of the kingdom and
compare their books with those of the writers 3. Answer: (c)
outside; he has women also for music, who
Statement 1 is correct: Siddhas (Sittars) of
play instruments and sing. Even the wives of
the Tamil region were monotheistic (who are
the king are well versed in music…it is said
characterized by the belief that there is only
that he has judges, as well as bailiffs and
one God) and they condemned idolatry, they
watchmen who every night guard the palace,
advocated renunciation of the world.
and these are women.”
Statement 2 is correct: Lingayats are the
followers of the 12th-century social reformer-
2. Answer: (d) philosopher poet, Basaveshwara. Basaveshwara
Statement 1 is not correct: Saint Nimbarka was against the caste system and Vedic rituals.
was not a contemporary of Akbar because the The term Lingayat denotes a person who wears
history of Akbar dates down to the 16th century a personal linga, an iconic form of the god
AD (1556 to 1605 AD). Akbar was the son of Shiva, on the body which is received during
Humayun, who ruled over India for 26 years. the initiation ceremony. The Lingayats are
The advent of the Mughal rule brought rich strict monotheists. They enjoin the worship of
culture and ethical changes in India. Nimbarka, only one God, namely, Linga (Shiva). They also
also called Nimbaditya or Niyamananda, was questioned the theory of rebirth. They believed
a Telugu-speaking Brahman who flourished in that on death, the devotee will be united with
Shiva and will not return to this world.

Religious movement during medieval period-Explanation 101


unacademy.com | Download the Unacademy app
Give your feedback here: Link
4 ART &
CULTURE
1 Architecture and Sculpture

1. The Prime Minister recently inaugurated 3. It was meant to promote the Vaishnava
the new Circuit House near Somnath cult in the region.
Temple at Veraval. Which of the following 4. Its design has given rise to a popular
statements are correct regarding Somnath belief that it was the inspiration behind
Temple? (2022) the Indian Parliament building.
1. Somnath Temple is one of the Jyotirlinga Which of the statements given above are
shrines. correct?
2. A description of the Somnath Temple (a) 1 and 2
was given by Al-Biruni.
(b) 2 and 3 only
3. Pran Pratishtha of Somnath Temple
(installation of the present-day temple) (c) 1 and 4
was done by President S. Radhakrishnan. (d) 2, 3 and 4
Select the correct answer using the code
given below : 4. Building ‘Kalyana Mandapam’ was a notable
(a) 1 and 2 only feature in the temple construction in the
(b) 2 and 3 only kingdom of: (2019)

(c) 1 and 3 only (a) Chalukya

(d) 1, 2 and 3 (b) Chandela


(c) Rashtrakuta

2. Which one of the following statements is (d) Vijayanagara


correct? (2021)
(a) Ajanta Caves lie in the gorge of Waghora 5. With reference to the cultural history of
river. India, consider the following statements:
(b) Sanchi Stupa lies in the gorge of (2018)
Chambal river. 1. White marble was used in making Buland
(c) Pandu-lena Cave Shrines lie in the gorge Darwaza and Khankah at Fatehpur Sikri.
of Narmada river. 2. Red sandstone and marble were used
(d) Amaravati Stupa lies in the gorge of in making Bara Imambara and Rumi
Godavari river. Darwaza in Lucknow.
Which of the statements given above is/are
correct?
3. With reference to Chausath Yogini Temple
situated near Morena, consider the (a) 1 only
following statements : (2021) (b) 2 only
1. It is a circular temple built during the (c) Both 1 and 2
reign of Kachchhapaghata Dynasty. (d) Neither 1 nor 2
2. It is the only circular temple built in
India.

103 Architecture and Sculpture


unacademy.com | Download the Unacademy app
Give your feedback here: Link
6. Which of the following is/are famous for (b) a religious sect
Sun temples? (2017) (c) a style of temple construction
1. Arasavalli (d) an administrative functionary
2. Amarkantak
3. Omkareshwar 10. With reference to the Indian history of art
Select the correct answer using the code and culture, consider the following pairs:
given below: (2014)
(a) 1 only Famous work of sculpture Site
(b) 2 and 3 only 1. A grand image of Buddha's Ajanta
(c) 1 and 3 only Mahaparinirvana with
numerous celestial
(d) 1, 2 and 3
musicians above and the
sorrowful figures of his
7. What is/are common to the two followers below
historical places known as Ajanta and 2. A huge image of Varaha Mount Abu
Mahabalipuram? (2016) Avatar (boar incarnation)
1. Both were built in the same period of Vishnu, as he rescues
Goddess Earth from the
2. Both belong to the same religious
deep and chaotic waters,
denomination
sculpted on rock
3. Both have rock-cut monuments
3. “Arjuna's Penance”/ “Descent Mamalla
Select the correct answer using the code of Ganga” sculpted on the Puram
given below. surface of huge boulders
(a) 1 and 2 only Which of the pairs given above is/are
(b) 3 only correctly matched?

(c) 1 and 3 only (a) 1 and 2 only

(d) None of the statements given above is (b) 3 only


correct (c) 1 and 3 only
(d) 1, 2 and 3
8. With reference to the art and archaeological
history of India, which one among the
11. With reference to the history of Indian rock-
following was made the earliest? (2015)
cut architecture, consider the following
(a) Lingaraja Temple at Bhubaneswar statements: (2013)
(b) Rock-cut Elephant at Dhauli 1. The caves at Badami are the oldest
(c) Rock-cut Monuments at Mahabalipuram surviving rock-cut caves in India.

(d) Varaha Image at Udayagiri 2. The Barabar rock-cut caves were


originally made for Ajivikas by Emperor
Chandragupta Maurya.
9. With reference to the cultural history of
3. At Ellora, caves were made for different
India, the term 'Panchayatan' refers to
faiths.
(2014)
Which of the statements given above is/are
(a) an assembly of village elders

Architecture and Sculpture 104


unacademy.com | Download the Unacademy app
Give your feedback here: Link
correct? (c) Chaitya is the stupa at the far end of the
(a) 1 only cave, while Vihara is the hall axial to it

(b) 2 and 3 only (d) There is no material difference between


the two
(c) 3 only
(d) 1, 2 and 3
13. The Nagara, the Dravida and the Vesara are
the (2012)
12. Some Buddhist rock-cut caves are called (a) three main racial groups of the Indian
Chaityas, while others are called Viharas. subcontinent
What is the difference between the two?
(2013) (b)
three main linguistic divisions into
which the languages of Indian can be
(a) Vihara is a place of worship, while classified
Chaitya is the dwelling place of the
monks (c) three main styles of Indian temple
architecture
(b) Chaitya is a place of worship, while
Vihara is the dwelling place of the (d) three main musical Gharanas prevalent
monks in India

105 Architecture and Sculpture


unacademy.com | Download the Unacademy app
Give your feedback here: Link
Architecture and Sculpture-
1 Explanation
1. Answer: (a) monarch Devapala (1055 – 1075), according to
Statement 1 is correct: Somnath Temple at an inscription dated 1323 CE. It is situated in
Somnath, Gujarat is one of the most sacred Morena district, Madhya Pradesh.
pilgrimage sites for Hindus and is believed to Statement 2 is not correct: It is not the only
be first among the twelve Jyotirlinga shrines circular temple built in India.
of Shiva. Statement 3 is not correct: It is reported that
Statement 2 is correct: A description of the the temple served as a centre for astrology
temple by Al-Biruni, an Arab traveller, was so and mathematical teaching based on the Sun’s
glowing that it prompted a visit in 1024 by a transit. A Shivalinga can also be found in the
most unwelcome tourist – the legendary looter central shrine. It features an outward circular
Mahmud of Ghazni from Afghanistan. shape with a radius of 170 feet and 64 tiny
Statement 3 is Incorrect: Then President chambers within its interior.
of India, Dr. Rajendra Prasad, did the Pran- Statement 4 is correct: It is popularly believed
Pratistha at the existing temple on 11 May 1951. that the round Chausath Yogini Temple served
as inspiration for the Indian Parliament House.
The temple is said to be dedicated to 64 yoginis
2. Answer: (a) and has 64 small chambers in its interior. The
Option (a) is correct: The caves at Ajanta are central shrine, of course, is dedicated to Lord
excavated out of a vertical cliff above the left Shiva, and there are slab coverings here that
bank of the river Waghora in the hills of Ajanta. have perforations to drain rainwater. The Indian
The U-shaped valley at Ajanta was carved by parliament, on the other hand, has 144 columns
the Waghora river over many centuries. They and is surrounded by gardens. History has it
are thirty in number, including the unfinished that the temple’s exterior surface had statues
ones, of which five (caves 9, 10, 19, 26 and of couples who were flanked by maidens.
29) are chaityagrihas (sanctuary) and the rest Unfortunately, this is now lost or damaged.
are sangharamas or viharas (monastery). The The similarities between the Chausath Yogini
caves are connected to the river by rock-cut Temple and the Indian Parliament are quite
staircases. evident. However, due to the lack of evidence,
The Sanchi Stupa is located nearly 46 historians and archaeologists remain sceptical.
kilometres northeast of Bhopal, the capital Elimination technique: It is the only
city of Madhya Pradesh. The Stupa is one of circular temple built in India. Statement
the oldest stone structures in India that was 2 is too extreme and it cannot be correct.
built during the Mauryan period. Thus, Option (c) is correct.
Pandu-lena or Pandava’s Caves are a group
of 24 cave excavations. They are located 8
km southwest of Nasik. Godavari river flows 4. Answer: (d)
through Nashik. Option (d) is correct: ‘Kalyana Mandapams’ was
a notable feature in the temple construction in
the kingdom of Vijayanagara. The Vijayanagara
3. Answer: (c) Empire was founded by Harihara and Bukka in
Statement 1 is correct: The Chausath Yogini AD 1336 and made Hastinavati (Hampi) as the
temple was built by the Kachchhapaghata capital city. The temple-building activity of the

Architecture and Sculpture-Explanation 106


unacademy.com | Download the Unacademy app
Give your feedback here: Link
Vijayanagar rulers produced a new style, called of the island is believed to be like the Hindu
the Vijayanagar Style. The Vitthala Temple is “om” symbol.
one of the greatest monuments exemplifying Omkareshwar is a Hindu temple dedicated to
the Vijayanagar Style (often characterised Shiva, located in Mandhata, nearby Khandwa
as Dravida Style) which dates from the 16th city in Khandwa district of the Indian state of
Century. It has several structures inside a Madhya Pradesh. It is one of the 12 revered
rectangular court, one of which is the Kalyana Jyotirlinga shrines of Shiva.
Mandapa with elaborately carved pillars.
Kalyana Mandapa is an open pavilion, which Elimination technique: Omkareshwar is
was used for ceremonies involving the symbolic a famous Hindu temple devoted to God
marriage of the temple’s divinity to his consort. Shiva. Thus, Option (a) is correct.
Additional Information:
They ruled over Andhra, Karnataka, Tamil Nadu, 7. Answer: (b)
and parts of Kerala in southern India. Hampi is Option (b) is correct: Both the historical places
a UNESCO World Heritage Site, located in east- known as Ajanta and Mahabalipuram have
central Karnataka. rock-cut monuments.
The Ajanta Caves is in the Aurangabad district of
5. Answer: (d) Maharashtra, with about 30 rock-cut Buddhist
Buland Darwaza: Buland Darwaza at Fatehpur cave monuments dating from the 2nd century
Sikri was built in 1602 A.D by the great Mughal BCE to about 480 or 650 CE. The caves include
emperor, Akbar, to commemorate his victory paintings and sculptures described by the
over Gujarat. Archaeological Survey of India as ‘the finest
surviving examples of Indian art, particularly
It is made of red and buff sandstone and painting’, which are masterpieces of Buddhist
decorated by carving and inlaying of white and religious art, with figures of the Buddha and
black marble. Mughals often use red sandstone depictions of the Jataka tales.
in their construction. However, according to
the official UPSC answer key, this statement is Mahabalipuram, an epitome of Hindu
WRONG. architecture has a group of sanctuaries, which
was carved out of rock along the Coromandel
coast in the 7th and 8th centuries. It has rathas
6. Answer: (a) (temples in the form of chariots), mandapas
Option (a) is correct: Arasavalli is famous for its (cave sanctuaries), and giant open-air rock
Sun Temple. It is a 7th-century AD Sun Temple reliefs such as the famous Descent of the
at Arasavalli in Andhra Pradesh. It is one of the Ganges, and the Shore Temple, with thousands
ancient sun God temples in our Country. of sculptures to the glory of Shiva. The Group
of Monuments at Mahabalipuram has been
Amarkantak is a pilgrim town and a Nagar
classified as a UNESCO World Heritage Site.
Panchayat in Anuppur, Madhya Pradesh, India.
It is a region that has a unique natural heritage
area and it is the meeting point of the Vindhya 8. Answer: (b)
and the Satpura Ranges, with the Maikal Hills Option (b) is correct: Rock-cut Elephant at
being the fulcrum. Dhauli was created during Ashoka’s reign
Omkareshwar is a Hindu temple devoted to (272-231 BC). Dhauli is in the ancient territory
God Shiva. It is one of the 12 revered Jyotirlinga of Kalinga, now the state of Orissa. The
shrines of Shiva on an island called Mandhata rock-cut elephant is above the Edicts. The
or Shivapuri in the Narmada River. The shape earliest Buddhist sculpture of Odisha. The

107 Architecture and Sculpture-Explanation


unacademy.com | Download the Unacademy app
Give your feedback here: Link
stone elephant shows the animal’s foreparts
only, though it has a fine sense of form and 10. Answer: (c)
movement.
Pair 1 is correctly matched: The
Additional Information: Mahaparinirvana of Lord Buddha in Cave 17,
y A sculpted panel at the Gupta-period (4th- with numerous celestial musicians above and
6th century CE) caves of Udayagiri, Madhya the sorrowful figures of his followers below,
Pradesh are rock-cut Hindu shrines, and is one the grandest and the most delicately
this panel shows Vishnu as the boar- expressive scenes ever made in stone at Ajanta
headed incarnation of Varaha. The god Caves. The grieving figure of Ananda near his
rises from the cosmic waters, defeating the feet is an exceptionally fine and thoughtful
primaeval serpent monster, and rescuing representation in Ajanta.
the goddess Bhudevi (earth), who hangs Pair 2 is not correctly matched: Rock cut
from his tusk. The site at Udayagiri Caves sculpture of Varaha is at Udaygiri caves near
was the patronage of Chandragupta II. Vidisha whereas Mount Abu is known for Jain
y The city of Mahabalipuram was founded by Temples.
the Pallava king Narasimhavarman I in the Pair 3 is correctly matched: Arjuna’s penance
7th century AD. The mandapa or pavilions is a massive open-air-bas-relief monolith,
and the rathas or shrines shaped as dating back to the 7th Century. It is situated
temple chariots are hewn from the granite in Mahabalipuram, near Chennai, Tamil
rock face, while the famed Shore Temple, Nadu. Standing tall at a height of 43 feet,
erected half a century later, is built from the Mahabalipuram monolith was carved
dressed stone. on the face of two huge adjoining boulders,
y Lingaraj Temple, built in the 11th century somewhere around the mid-7th Century. The
AD, is dedicated to Lord Shiva and is length of Arjuna’s penance is 96 feet and the
considered as the largest temple of the city monolith is also known by the name of ‘The
of Bhubaneswar. It is believed to have been Descent of Ganga’.
built by the Somvanshi King Yayati I. The
main tower of this temple measures 180-
feet in height. It is built in red stone and 11. Answer: (c)
is a classic example of the Kalinga style of Statement 1 is not correct: Dating back to
architecture. the Mauryan period, Barabar Caves are the
oldest rock-cut caves in India (not the caves
at Badami). Badami is a town in the Bagalkot
9. Answer: (c) district of Karnataka. Badami caves were
Option (c) is correct: A Hindu temple is constructed in the 6th century by the Chalukya
called Panchayatan when the main shrine Dynasty in their capital Badami, which was
is surrounded by four subsidiary shrines. Its earlier known as Vatapi. The cave significantly
original name comes from Sanskrit words, represents Indian rock-cut architecture but
Pancha meaning Five, and Yatana meaning God. is not the oldest surviving rock-cut cave. The
Generally, Hindu temples are along the west- caves house several Hindu and Jain temples.
east axis. So, the four subsidiary shrines are The caves are a UNESCO World Heritage Site.\
at the north-east, south-east, south-west, Statement 2 is not correct: Barabar caves
and north-west. Examples of this style are, were in the Jehanabad district in Bihar, and
Lakshmana Temple in Khajuraho, Lingaraja some of the caves have inscriptions that date
Temple in Bhubaneshwar and Dashavatara back to the Ashokan period. The Caves were
Temple in Deogarh, Uttar Pradesh. constructed by Emperor Ashoka and this fact

Architecture and Sculpture-Explanation 108


unacademy.com | Download the Unacademy app
Give your feedback here: Link
is strengthened by the Ashoka inscriptions worship. It is a rectangular hall with finely
engraved on the caves during his reign. The polished interior walls. There were several
caves were used by the Ajivikas. The Ajivika well-proportioned columns. A semicircular
sect was founded by Makkhali Gosala. roof stood over these columns. On either side
Statement 3 is correct: Ellora is a sacred site of the hall will be large entrances; opposite the
in Maharashtra, central India. The Ellora Caves entrance at one end of the hall stood a Stupa.
are listed by UNESCO as a World Heritage Site The Viharas were monasteries for monks to
and are celebrated for its Hindu, Buddhist, live in. These were built near the Chaityas at
and Jain temples and monuments which were Udayagiri in Orissa Jain cave temples belonging
carved from the local cliff rock in the 6th to 8th to the period of King Kharavela have been
century CE. The most spectacular example is excavated.
the 8th century CE Kailasa temple which, at 32
metres high, is the largest rock-cut monument
in the world. 13. Answer: (c)
Option (c) is correct: Temple architecture in
Elimination technique: Barabar Caves are
India can be classified into three types which
the oldest rock-cut caves in India. It was
are: Nagara, Dravida, and Vesara.
built during the period of Emperor Ashoka.
Statements 1 and 2 are not correct. Thus, The classification is based on the shapes, the
option (c) is correct. Nagara style or the North Indian Style is defined
as being quadrangular all over, from the base to
the Shikhara. This style is associated with the
12. Answer: (b) land between the Himalayas and the Vindhyas.
Buddhism and Jainism have introduced the art The Dravida style or the South Indian Style is
of rock-cut caves. The caves were cut out of associated with the land between the Krishna
huge solid rocks. The Buddhist and Jain monks and kaveri rivers. While Vesara or the Hybrid
lived in these caves and meditated. style is associated with the area between the
Vindhyas and the Krishna River.
Option (b) is correct: Chaitya is a hall of

109 Architecture and Sculpture-Explanation


unacademy.com | Download the Unacademy app
Give your feedback here: Link
2 Literature: Religious and Scientific

1. With reference to ancient Indian History, (b) playwrights


consider the following pairs : (2023) (c) temple architects
Literary work Author (d) philosophers
1. Devichandragupta : Bilhana
2. Hammira- Nayachandra 4. Which one of the following books of ancient
Mahakavya : Suri India has the love story of the son of the
3. Milinda-panha : Nagarjuna founder of Sunga dynasty? (2016)
4. Nitivakyamrita : Somadeva Suri (a) Swapnavasavadatta
How many of the above pairs are correctly (b) Malavikagnimitra
matched? (c) Meghadoota
(a) Only one (d) Ratnavati
(b) Only two
(c) Only three 5. The national motto of India, ‘Satyameva
(d) All four Jayate’ inscribed below the Emblem of
India is taken from (2014)
(a) Katha Upanishad
2. With reference to Indian history, consider
the following texts : (2022) (b) Chandogya Upanishad
1. Nettipakarana (c) Aitareya Upanishad
2. Parishishtaparvan (d) Mundaka Upanishad
3. Avadanashataka
4. Trishashtilakshana Mahapurana 6. Consider the following statements: (2014)
Which of the above are Jaina texts ? 1. ‘Bijak’ is a composition of the teachings
of Saint Dadu Dayal.
(a) 1,2 and 3
2. The Philosophy of PushtiMarg was
(b) 2 and 4 only
propounded by Madhavacharya.
(c) 1,3 and 4
Which of the statements given above is/are
(d) 2,3 and 4 correct?
(a) 1 only
3. With reference to the history of ancient (b) 2 only
India, Bhavabhuti, Hastimalla and
(c) Both 1 and 2
Kashemeshvara were famous (2021)
(d) Neither 1 nor 2
(a) Jain monks

Literature: Religious and Scientific 110


unacademy.com | Download the Unacademy app
Give your feedback here: Link
Literature: Religious and
2 Scientific-Explanation
1. Answer: (b) 3. Answer: (b)
1. Devi-Chandraguptam is an Indian Option (b) is correct: Bhavabhuti was an Indian
Sanskrit-language political drama dramatist, Playwriter and poet. His dramas were
attributed to Vishakhadatta. normally written in Sanskrit and noted for their
2. Hammira Mahakavya is a 15th-century suspense and vivid characterization. His plays
Indian Sanskrit epic poem written by are considered equal to the works of Kalidasa.
the Jain scholar Nayachandra Suri. It He wrote famous dramas Malatimadhava and
is a legendary biography of the 13th Mahaviracharita.
century Chahamana king Hammira. Hastimalla was a poet and writer in the Hoysala
3. Milinda Panho: It contains a dialogue Empire during the 13th century.
between King Meander (or Milinda) and Kshemendra was a Kashmiri poet of Sanskrit
Buddhist monk Nagasena. It means who lived in the 11th century.
“Questions of Milinda ‘’.
4. The one of the highest philosophical 4. Answer: (b)
enquiries Somadeva Suri composed are
Sunga dynasty, Indian ruling house founded by
the Yashastilaka and Nitivakyamrita
Pushyamitra about 185 BCE, which replaced the
Mauryan dynasty. Pushyamitra assassinated
2. Answer: (b) Brihadratha, the last Mauryan ruler, at a military
Option 1 is incorrect: The Nettipakaraa parade and assumed royal power. Pushyamitra
is a legendary Buddhist scripture that is was a Brahman, yet Buddhism still flourished
occasionally included in Theravada Buddhism’s in many areas under his control.
Pali Canon’s Khuddaka Nikaya. Option (b) is correct: The Malavikagnimitra is
The Parishishtaparvan, also known as the a Sanskrit play by Kalidas. It comprises of two
Sthaviravalicharitra, is a Sanskrit mahakavya words Malavika and Agnimitra. The play depicts
written by Hemachandra in the 12th century the love story of Agnimitra, son of Pushyamitra,
that describes the lives of the first Jain who falls in love with the picture of an exiled
instructors. servant girl named Malavika.

Option 3 is incorrect: The Avadanashataka, or


“Century of Noble Deeds,” is a Sanskrit collection 5. Answer: (d)
of one hundred Buddhist stories, roughly Option (d) is correct: Satyamev Jayate is the
contemporaneous with the Ashokavadana. national motto of India. It is inscribed in the
Trishashthilkshana or Mahapurana Mahapurana Devanagari script at the base of the National
is a famous Jain scripture mostly produced by Emblem, which is an adaptation of the Lion
“In the 9th century CE, his pupil Gunabhadra Capital of Ashoka at Sarnath, near Varanasi in
completed the work begun by Acharya” Uttar Pradesh. The words Satyameva Jayate
Jinasena during the reign of Rashtrakuta are taken from Mundaka Upanishad, meaning
monarch Amoghavarsha. ‘Truth Alone Triumphs’.

111 Literature: Religious and Scientific-Explanation


unacademy.com | Download the Unacademy app
Give your feedback here: Link
6. Answer: (d) Statement 2 is not correct: Vallabha, also
Statement 1 is not correct: Bijak is the best called Vallabhacharya, a Hindu philosopher was
known of the compilations of the Kabir, and the founder of the important Vallabhacharya
as such is the holy scripture for followers of (or Vallabha Sampradaya) devotional sect, also
the Kabirpanthi religion.It also has a number known as the Pushtimarg (from Sanskrit Pushti
of folk songs. The Bijak is one of the earliest of marga, “way of flourishing”).
the major texts in modern Bagheli.

Literature: Religious and Scientific-Explanation 112


unacademy.com | Download the Unacademy app
Give your feedback here: Link
Performing Arts: Dance, Theatre
3 and Music
1. With reference to cultural history of India, correctly matched?
consider the following statements: (2018) (a) 1 only
1. Most of the Tyagaraja Kritis are devotional (b) 2 and 3 only
songs in Praise of Lord Krishna.
(c) 1 and 3 only
2. Tyagaraja created several new ragas.
(d) 1, 2 and 3
3. Annamacharay and Tyagaraja are
contemporaries.
4. Annamacharya keertanas are devotional 4. With reference to the famous Sattriya
songs in praise of Lord Venkateshwara. dance, consider the following statements:
(2014)
Which of the statements given above are
correct? 1. Sattriya is a combination of music,
dance and drama.
(a) 1 and 3 only
2. It is a centuries-old living tradition of
(b) 2 and 4 only Vaishnavites of Assam.
(c) 1, 2 and 3 3. It is based on classical Ragas and
(d) 2, 3 and 4 Talas of devotional songs composed by
Tulsidas, Kabir and Mirabai.

2. With reference to Manipuri Sankirtana, Which of the statements above is/are


consider the following statements: (2017) correct?

1. It is a song and dance performance. (a) 1 only

2. Cymbals are the only musical (b) 1 and 2 only


instruments used in the performance. (c) 2 and 3 only
3. It is performed to narrate the life and (d) 1, 2 and 3
deeds of Lord Krishna.
Which of the statements given above is/are 5. In the context of cultural history of India,
correct? a pose in dance and dramatics called
(a) 1, 2 and 3. 'Tribhanga' has been a favourite of Indian
(b) 1 and 3 only artists from ancient times till today.
Which one of the following statements
(c) 2 and 3 only best describes this pose? (2013)
(d) 1 only (a) One leg is bent, and the body is slightly
but oppositely curved at waist and neck
3. Consider the following pairs: (2014) (b) Facial expressions, hand gestures and
1. Garba Gujarat make-up are combined to symbolise
certain epic or historic characters
2. Mohiniattam Odisha
(c) Movements of body, face and hands are
3. Yakshagana Karnataka
used to express oneself or to tell a story
Which of the pairs given above is/are
(d) A little smile, slightly curved waist and

113 Performing Arts: Dance, Theatre and Music


unacademy.com | Download the Unacademy app
Give your feedback here: Link
certain hand gestures are emphasised (c) 1, 2 and 3
to express the feelings of love or (d) None of the above is correct
eroticism

7. How do you distinguish between Kuchipudi


6. With reference to Dhrupad, one of the and Bharatanatyam dances? (2012)
major traditions of India that has been kept
alive for centuries, which of the following 1. Dancers occasionally speaking dialogues
statements are correct? (2012) are found in Kuchipudi dance but not in
Bharatanatyam.
1. Dhrupad originated and developed in
the Rajput kingdoms during the Mughal 2. Dancing on the brass plate by keeping
period. the feet on its edges is a feature
of Bharatanatyam but Kuchipudi
2. Dhrupad is primarily a devotional and dance does not have such a form of
spiritual music. movements.
3. Dhrupad Alap uses Sanskrit syllables Which of the statements given above is/are
from Mantras. correct?
Select the correct answer using the code (a) 1 only
given below:
(b) 2 only
(a) 1 and 2 only
(c) Both 1 and 2
(b) 2 and 3 only
(d) Neither 1 nor 2

Performing Arts: Dance, Theatre and Music 114


unacademy.com | Download the Unacademy app
Give your feedback here: Link
Performing Arts: Dance, Theatre
3 and Music-Explanation
1. Answer: (b) recognized by UNESCO. It was conceived by
Statement 1 is not correct: Saint Tyagaraja the famous Manipur king Bhagyachandra Singh
was a renowned composer of Carnatic in the 18th century. Sankirtana is maintained
music. He composed thousands of devotional by the Sangeet Natak Akademi. It encompasses
compositions, mostly in Telugu in the praise an array of arts performed to mark religious
of Lord Rama, many of which remain popular occasions and various stages in the life of the
even till this date. He saw the reigns of four Vaishnav people of Manipur plains. Sankirtana
kings of Maratha dynasty — Tulaja II (1763- practises centre on the temple, where
1787), Amarasimha (1787-1798), Serfoji II (1798- performers narrate the lives and deeds of
1832) and Sivaji II (1832-1855), but he served Krishna through song and dance.
none of them. Statement 2 is not correct: In a typical
Statement 2 is correct: Tyagaraja introduced performance, two drummers and about ten
and gave life to several unusual new ragas in singer-dancers perform in a hall or domestic
Carnatic music. Some he created while others courtyard encircled by seated devotees. The
he found as mere scales in books and infused dignity and flow of aesthetic and religious energy
musicality into them. is unparalleled, moving audience members to
tears and frequently to prostrate themselves
Statement 3 is not correct: Sri Tallapaka before the performers. Sankirtana has two main
Annamacharya was a saint composer in the social functions: it brings people together on
15th century, who hailed from the Tallapaka festive occasions throughout the year, acting
village, in the Rajampet mandal of Cuddapah as a cohesive force within Manipur’s Vaishnava
district, Andhra Pradesh, India. While Tyagaraja community; and it establishes and reinforces
belonged to the 18th century. Hence, they were relationships between the individual and the
not contemporaries. community through life-cycle ceremonies.
Statement 4 is correct.: Annamacharaya
was the earliest known Indian musician to
compose songs called Sankirtanas in praise of 3. Answer: (c)
god Venkateshwara, a form of Vishnu. Option 1 is correct: Garba is a form of dance,
as well as a religious and social event. It is a
Elimination technique: Annamacharay and community circle dance from the northwestern
Tyagaraja were not contemporaries(the Indian state of Gujarat. The word “Garba” is
former was a 15th-century saint while the also used to refer to the event at which the
later was an 18th-century saint). Thus, Garba is performed. Garba is performed during
Option (b) is correct. Navratri, the longest and largest dance festival
in the world.
2. Answer: (b) Option 2 is not correct: Mohiniattam is a
Statement 1 and statement 3 are correct: semiclassical dance form from the state
Manipuri Nat-Sankirtana is an Indian art of Kerala, India. The dance is performed by
form recognized by UNESCO as an intangible women in honour of the Hindu god Vishnu in
cultural heritage. Kutiyattam (Sanskrit Theatre), his incarnation as the enchantress Mohini. It is
Vedic Chanting and Ramlila are the other one of the eight Indian classical dance forms
masterpieces of oral and intangible heritage recognised by the Sangeet Natak Akademi.

115 Performing Arts: Dance, Theatre and Music-Explanation


unacademy.com | Download the Unacademy app
Give your feedback here: Link
Option 3 is correct: Yakshagana is a theatre oldest surviving form of Indian Classical music
form that combines dance, music, dialogue, and traces its origin to the chanting of vedic
costume, makeup and stage techniques with hymns and mantras. Though a highly developed
a unique style and form. This theatre style classical art with a complex and elaborate
resembling western opera is mainly found in grammar and aesthetics, it is also primarily a
the coastal districts and Malenadu region of form of worship, in which offerings are made
Karnataka. to the divine through sound or nada. Dhrupad
was originally sung in temples and only thrived
later under the patronage of Mughal and Rajput
4. Answer: (b)
kings.
Statement 1 is correct: The neo-Vaishnava
Statement 2 is correct: Dhrupad portrays
treasure of Assamese dance and drama has
a vast range of human emotions: serenity,
been, for centuries, nurtured and preserved
compassion, sensuality, pathos, strangeness,
with great commitment by the Sattras, that is,
anger and heroism and subtle shades of them
Vaishnava maths or monasteries. Because of
all. In Dhrupad of the Dagar tradition the
its religious character and association with the
notes are not treated as fixed points, but as
Sattras, this dance style has been aptly named,
fluid entities with infinite microtonal shades.
Sattriya.
The music is deeply spiritual and meditative.
Statement 2 is correct: The Sattriya dance form The Dagar style of Dhrupad is defined by 52
was introduced in the 15th century A.D by the musical concepts or Arkans, these are Udatta,
great Vaishnava saint and reformer of Assam, Anudatta, Svarita, Sapta Gupta, Sapta Prakata,
Mahapurusha Sankaradeva as a powerful Sakari etc.
medium for propagation of the Vaishnava faith.
Statement 3 is correct: A Dhrupad performance
Statement 3 is not correct: Kabir, Mirabai, starts with the Alap which is a slow and
and Tulsidas were all born around the start of elaborate development of a Raga (mode) using
the 16th century, whereas Sattriya dance was free flowing melodic patterns. The elaboration
introduced in the 15th Century. of Dhrupad alap is done using the syllables of a
mantric phrase ‘om antaran twam, taran taaran
twam, ananta hari narayan om’. The phrases of
5. Answer: (a)
Dhrupad alap are very slow and contemplative
Option (a) is correct: Tribhanga is a (tri-bent in the beginning, but the tempo increases in
pose) standing body position or stance used stages, and in the faster passages playful and
in the traditional Indian sculpture, art, and vigorous ornaments predominate. Dhrupad
Indian classical dance forms like the Odissi. Alap uses Sanskrit syllables from Mantras.
As, compared with the Contrapposto pose
Tribhanga, meaning three parts, consists of
three bends in the body; at the neck, waist, 7. Answer: (a)
knee, hence the body is oppositely curved Statement 1 is correct: Kuchipudi is an Indian
at waist and neck which gives it a gentle “S” Classical dance that has its roots in a village
shape and is considered the most graceful and situated in Krishna district of Andhra Pradesh.
the sensual of the Odissi positions. It is closely Kuchipudi is unique among the Indian Classical
associated with the Hindu deity, Krishna, who dance styles. It uses fast rhythmic footwork
is often portrayed in the posture. and sculpturesque body movements. Stylized
mime, using hand gestures and subtle facial
expressions, is combined with more realistic
6. Answer: (b)
acting, occasionally including dialogues spoken
Statement 1 is not correct: Dhrupad is the by the dancers.

Performing Arts: Dance, Theatre and Music-Explanation 116


unacademy.com | Download the Unacademy app
Give your feedback here: Link
In pure style, Bharatanatyam is classically clear (symbolic hand gestures) and no dialogues are
in technique. The feet beat out complicated spoken.
counter rhythms; the legs are bent in a Statement 2 is not correct: Another unique
characteristic low squat; arms, neck, and feature of Kuchipudi is the Tarangam, in which
shoulders are part of the movement. In the the performer dances on the edges of a brass
pantomime sections, the hands tell the story plate, deftly executing complicated rhythmic
through conventional gesture language, while patterns, while sometimes also balancing a pot
the face expresses the mood. In the pure of water on the head.
dance the hands are restricted to 11 mudras

117 Performing Arts: Dance, Theatre and Music-Explanation


unacademy.com | Download the Unacademy app
Give your feedback here: Link
Visual Arts: Painting, ceramics
4 and drawing
1. The well-known painting “Bani Thani” (b)
a handmade drawing on bamboo
belongs to the (2018) handicrafts in North-East India.
(a) Bundi School (c)
a block-painted woollen cloth in
(b) Jaipur School Western Himalayan region of India.

(c) Kangra School (d) a hand-painted decorative silk cloth in


North-Western India.
(d) Kishangarh School

4. Consider the following historical places:


2. The painting of Bodhisattva Padmapani is (2013)
one of the most famous and oft-illustrated
paintings at (2017) 1. Ajanta Caves

(a) Ajanta 2. Lepakshi Temple

(b) Badami 3. Sanchi Stupa

(c) Bagh Which of the above places is/are also


known for mural paintings?
(d) Ellora
(a) 1 only
(b) 1 and 2 only
3. Kalamkari painting refers to (2015)
(c) 1, 2 and 3
(a) a hand-painted cotton textile in South
India. (d) None of the above

Visual Arts: Painting, ceramics and drawing 118


unacademy.com | Download the Unacademy app
Give your feedback here: Link
Visual Arts: Painting, ceramics
4 and drawing-Explanation
1. Answer: (d) art practised in Andhra Pradesh, Southern
Option (d) is correct: The Kishangarh province Indian State.
in Rajasthan is famous for its Bani Thani y Kalamkari represents a unique art of
paintings. It is a style with highly exaggerated painting cotton fabrics with a Kalam (pen)
features like long necks, large, almond shaped which is generally done using a sharp,
eyes, and long fingers. This style of painting pointed pierced bamboo that regulates the
essentially depicts Radha and Krishna as divine flow of colour on the fabric.
lovers, and beautifully portrays their mystical y The name Kalamkari originates from two
love. Persian words, namely Qalam (Kalam or
Kishangarh miniature painting reached a peak pen) and Kari (craftsmanship). Kalamkari
in the eighteenth century, during the rule of art is considered unique as it uses natural
Raja Sawant Singh, who fell in love with a slave colours as a vibrant medium for portraying
girl, Bani Thani and commanded his artists to mythological characters from epics like
portray himself and her as Krishna and Radha. Ramayana and Mahabharat.
Other themes of Bani Thani paintings include y There are two identifiable styles of
portraits, court scenes, dancing, hunting, Kalamkari art – Srikalahasti style (Chittoor
music parties, nauka vihar (lovers travelling district) and Machilipatnam style (Krishna
in a boat), Krishna Lila, Bhagavata Purana and district).
various festivals like Holi, Diwali, Durga puja,
and Dussehra.
4. Answer: (b)
Mural is a painting applied to and made integral
2. Answer: (a)
with the surface of a wall or ceiling. The term
Option (a) is correct: The famous painting of may properly include painting on fired tiles but
the bodhisattva, Padmapani in Ajanta caves, is ordinarily does not refer to mosaic decoration
one of the best paintings remaining from the unless the mosaic forms part of the overall
prehistoric era when Buddhism was at its peak. scheme of the painting.
The Ajanta caves in Maharashtra were built in
Statement 1 is correct: Ajanta Caves are famous
two phases, the first group starting around
for its murals and are the finest surviving
the 2nd century BC, while the second group of
examples of Indian art, particularly painting.
caves built around 400–650 AD. In Ajanta caves,
Ajanta murals include sculptures of animals,
the walls were originally covered in paintings,
guards and deities. It also includes paintings
but today there are only nine surviving images
of court life, scenes of the life of Buddha etc.
present, Bodhisattva Padmapani being the
most famous one. Padmapani is a Sanskrit Statement 2 is correct: Lepakshi temple also
word, meaning “one who holds the lotus”. It is has the finest specimens of mural paintings of
one of the UNESCO world heritage sites. the Vijayanagara kings.
Statement 3 is not correct: Sanchi Stupa is
famous for architecture and sculptures only.
3. Answer: (a)
There is no evidence of mural paintings from
Option (a) is correct: Kalamkari is a popular the Sanchi Stupa.
hand-painted, or block-printed cotton textile

119 Visual Arts: Painting, ceramics and drawing-Explanation


unacademy.com | Download the Unacademy app
Give your feedback here: Link
Indian Philosophy and Bhakti &
5 Sufi Movements
1. With reference to Indian history, consider (b) 1 and 3 only
the following pairs : (2022) (c) 2 and 3 only
Historical person Known as (d) 1, 2 and 3
1. Aryadeva Jaina scholar
2. Dignaga Buddhist scholar 4. Which one of the following pairs does
3. Nathamuni Vaishnava scholar not form part of the six systems of Indian
How many pairs given above are correctly Philosophy? (2014)
matched ? (a) Mimamsa and Vedanta
(a) None of the pairs (b) Nyaya and Vaisheshika
(b) Only one pair (c) Lokayata and Kapalika
(c) Only two pairs (d) Sankhya and Yoga
(d) All three pairs
5. Consider the following Bhakti Saints:
2. The world’s second tallest statue in sitting (2013)
pose of Ramanuja was inaugurated by 1. Dadu Dayal
the Prime Minister of India at Hyderabad 2. Guru Nanak
recently. Which one of the following
statements correctly represents the 3. Tyagaraja
teachings of Ramanuja ? (2022) Who among the above was/were preaching
(a) The best means of salvation was when the Lodi dynasty fell, and Babur took
devotion. over?

(b) Vedas are eternal, self-existent and (a) 1 and 3


wholly authoritative. (b) 2 only
(c) Logical arguments were essential means (c) 2 and 3 only
for the highest bliss. (d) 1 and 2 only
(d) Salvation was to be obtained through
meditation.
6. With reference to the history of
philosophical thought in India, consider the
3. With reference to the cultural history of following statements regarding Sankhya
India, consider the following pairs: (2020) school: (2013)
1. Parivrajaka Renunciant and Wanderer 1. Sankhya does not accept the theory of
2. Shramana Priest with a high status rebirth or transmigration of soul.

3. Upasaka Lay follower of Buddhism 2. Sankhya holds that it is the self-


knowledge that leads to liberation and
Which of the pairs given above are correctly not any exterior influence or agent.
matched?
Which of the statements given above is/are
(a) 1 and 2 only correct?

Indian Philosophy and Bhakti & Sufi Movements 120


unacademy.com | Download the Unacademy app
Give your feedback here: Link
(a) 1 only 2. Severe ascetic exercises in a lonely
(b) 2 only place

(c) Both 1 and 2 3. Recitation of holy songs to arouse a


state of ecstasy in their audience
(d) Neither 1 nor 2
Select the correct answer using the code
given below:
7. With reference to the religious history (a) 1 and 2 only
of medieval India, the Sufi mystics were
known to pursue which of the following (b) 2 and 3 only
practices? (2012) (c) 3 only
1. Meditation and control of breath (d) 1, 2 and 3

121 Indian Philosophy and Bhakti & Sufi Movements


unacademy.com | Download the Unacademy app
Give your feedback here: Link
Indian Philosophy and Bhakti &
5 Sufi Movements-Explanation
1. Answer: (c) Pair 2 is not correctly matched: The Shramana
Option (c) is correct: group, consisting of Buddhism, Jainism and
various ‘heterodox’ sects, is a well-established
Pair 1 is incorrect: ARYADEVA- BUDDHIST group. Shramanas did not regard the Brahmans
SCHOLAR or a priest as being of the highest status. The
Aryadeva was born as a king’s son on the sramanas included a variety of ascetics, as well
island of Sinhala (Sri Lanka), but renounced his as the monks and lay followers of various sects
throne, became a monk, and travelled to South - Buddhist, Jaina, Ajivika and others.
India to study with Ngrjuna. The Four Hundred Pair 3 is correctly matched: Monks renounced
Verses (Catuhsataka) is Aryadeva’s greatest social obligations to take on an alternative life
book, and it offers the route to buddhahood as when they joined the Buddhism Order. They
built around these two requisites of merit and lived as equal members of the Order, denying
knowledge (jna). the caste distinctions. They lived in monasteries
Pair 2 is correct: Dignāga (c. 480 – c. 540 CE) near villages and towns so that they could
was an Indian Buddhist scholar and one of the draw on the support of the lay community,
Buddhist founders of Indian logic (hetu vidyā). namely those who were Buddhists or Jainas
Pair 3 is correct: Sri Ranganathamuni, popularly but were not initiated into renunciatory groups.
known as Sriman Nathamuni (823 CE–951 CE), Lay followers were referred to as upasaka and
was a Vaishnava theologian who collected and upasika.
compiled the Nalayira Divya Prabandham. Elimination Technique: Sramana was an
ancient Indian religious movement which
2. Answer: (a) rejected Vedic Hindu ritualism and the
authority of the Brahmins—the traditional
Option (a) is correct: Ramanuja tried to
priests of the Hindu religion. Pair 2 is not
assimilate bhakti to the tradition of Vedas.
correctly matched. Thus, Option (b) is
He argued that in order to attain salvation,
correct.
the grace of God was more important than
knowledge about him. He emphasised that the
path of prapatti or total reliance or surrender 4. Answer: (c)
to God was open to all, irrespective of caste. The Six Systems of Indian Philosophy are:
y Nyaya: The Nyaya school was founded
3. Answer: (b) by sage Gautama. Sixteen major topics
Pair 1 is correctly matched: The word were discussed in this system, the most
Parivrajaka means “roaming ascetic” or important of which is pramana, the source
a wandering religious mendicant. Thus, of valid knowledge. Nyaya is a school of logic,
Parivrajaka can be represented as Renunciant and all other schools of Indian philosophy
and Wanderer. Although this term occurs in the use the Nyaya system of logic, in whole or
early Brahmanic tradition of the Upaniṣhads, it in part, as a foundation for philosophical
is also applicable to Buddhist and Jain monks, reasoning and debate.
as well as to Hindu sannyasins. The Pāli y Vaisheshika: Kannada is the founder of
equivalent is paribbajaka. this school, which is associated with the
Nyaya system. This school discusses seven

Indian Philosophy and Bhakti & Sufi Movements-Explanation 122


unacademy.com | Download the Unacademy app
Give your feedback here: Link
major topics: substance, quality, action, 5. Answer: (b)
generality, uniqueness, inherence and non- Bhakti Movement was an important landmark
existence. in the cultural history of medieval India, it was
y Sankhya: Kapila is traditionally cited as the a silent revolution in society brought about
founder of this school, although his Samkhya by a galaxy of socio-religious reformers. This
Sutras have been lost. The Sankhya-karika movement was responsible for many rites and
of Isvara krsna, the oldest text on this rituals associated with the worship of God
philosophy, cites the names of Kapila, Asuri by Hindus, Muslims, and Sikhs of the Indian
and Pancasikha as previous teachers at this subcontinent. For example, Kirtan at a Hindu
school. It is considered to be the oldest Temple, Qawaali at a Dargah and singing of
of the philosophical systems. Sankhya is Gurbani at a Gurdwara. Its major achievement
a dualistic philosophy that believes in the was the abolition of idol worship.
coexistent and interdependent realities, Option (b) is correct: Lodi Dynasty fell in 1526
conscious Purusha and unconscious Prakrti. at the Battle of Panipat and at that time only
Sankhya philosophy explains the dynamics Guru Nanak preached. Dadu Dayal and Tyagaraja
of the body and nature of mind. were not even born when Babur founded the
y Yoga: Yoga and Sankhya are allied systems. Mughal Empire in India.
Although Yoga philosophy was known even Dadu Dayal (1554—1603) was a weaver
in the Vedic and pre-Vedic periods, it was from Ahmedabad, who made significant
not formally systematised until it was contributions to the Bhakti movement. He was
codified by Patanjali in about 200 BC. a mochi by caste and renounced the world
y Mimamsa: Jamini was the founder of this at an early age. He visited many places of
system that accepts the Veda as the final pilgrimage and became a saint.
authority on all questions. It provides a Guru Nanak (1469-1539) the founder of the
comprehensive method for interpreting and Sikh religion was another prominent bhakti
understanding the underlying meaning of saint, who greatly appreciated the teachings
the Veda. It lays great emphasis on rituals, of Kabir. Nanak decried the caste system and
worship and ethical conduct and provides a challenged the monopoly of spiritual evolution
systematic lifestyle and direction and religious sanctity of higher castes. Unity
y Vedanta: It was taught and practised by of God and unity of mankind were the two
the sages of the Vedas and Upanishads fundamental doctrines of his creed.
and was handed over through a long line Tyagaraja (1767-1847) was one of the greatest
of sages. The major schools of Vedanta composers of Carnatic music or Indian Classical
are Advaita (non-dualistic), Dvaita Music.
(dualistic), Dvaitadvaita (both dualistic
and non-dualistic), Visistadvaita (qualified
nondualism) and Visuddhadvaita (pure 6. Answer: (b)
non-dualism). Statement 1 is not correct: Sankhya is an
Option (c) is correct: Lokayata and Kapalika enumeration list philosophy that is strongly
do not form part of the six systems of Indian dualist. Sankhya denies the final cause of Ishvara
Philosophy. The Kapalika tradition was a non- (God). In Sankhya, the dualism is between the
puranic, tantric form of Shaivism in India. The real-self (Purusa) and matter (Prakriti). There
term Lokayata and Carvaka have historically are possible states of Purusa: the liberated
been used to denote the philosophical school state when Purusa has no connection with
of Indian Materialism. Lokayata means the Prakrit, the bonded state without life when
philosophy of the people. Purusa is bonded to 13 Karanas but does not

123 Indian Philosophy and Bhakti & Sufi Movements-Explanation


unacademy.com | Download the Unacademy app
Give your feedback here: Link
have a body. Therefore, the theory of rebirth or their spiritual leaders, or “murshid” (guide).
transmigration of soul is inherent in Samkhya. Statement 2 is correct: Among the early
Statement 2 is correct: Yoga presents a practical mystics who undertook hard ascetic exercises
path for the realisation of the self, whereas at his tomb was Khwaja Mu’in al-din Chisti,
the Samkhya emphasises the attainment of the founder of the Chisti Order in the Indian
knowledge of self by means of concentration Subcontinent.
and meditation. Statement 3 is correct: Some Sufi orders
engage in ritualised dhikr ceremonies, or sema.
7. Answer: (d) Sema includes various forms of worship such
as: recitation, singing (Qawwali), instrumental
Sufism is a mystical and ascetic Islam music, dance, incense, meditation, ecstasy and
practiced by Muslims. Known as “Tasawwuf” trance.
in the Muslim world.
Sufis in India adopted Sama and Rags as a mode
Statement 1 is correct: Followers of Sufism of invocation to God. Sama exhilarated their
believe they can become closer to Allah through spiritual spirit. Some of the saints practised
inner purification and introspection. They do severe exercises and controlled breath.
this by meditating and receiving guidance from

Indian Philosophy and Bhakti & Sufi Movements-Explanation 124


unacademy.com | Download the Unacademy app
Give your feedback here: Link
Indian Traditions, Festivals, and
6 Calendars
1. Consider the following statements: (2021) (b) 2 and 3 only
1. St. Francis Xavier was one of the (c) 1 and 3 only
founding members of the Jesuit Order. (d) None of the above
2. St Francis Xavier died in Goa and a
church is dedicated to him there.
4. Every year, a month-long ecologically
3. The Feast of St. Francis Xavier is important campaign/festival is held during
celebrated in Goa each year. which certain communities/tribes plant
Which of the statements given above are saplings of fruit-bearing trees.
correct? Which of the following are such
(a) 1 and 2 only communities/tribes? (2014)
(b) 2 and 3 only (a) Bhutia and Lepcha
(c) 1 and 3 only (b) Gond and Korku
(d) 1, 2 and 3 (c) Irula and Toda
(d) Sahariya and Agariya
2. Consider the following pairs: (2018)

Tradition State 5. A community of people called Manganiyars


is well-known for their (2014)
1. Chapchar Kut festival Mizoram
(a) Martial arts in North-East India
2. Khongjom Parba Ballad Manipur
3. Thang-Ta dance Sikkim (b) Musical tradition in North-West India

Which of the pairs given above is/are (c) Classical vocal music in South India
correct? (d) Pietra dura tradition in Central India
(a) 1 only
(b) 1 and 2 only 6. With reference to Buddhist history,
(c) 3 only tradition and culture in India, consider the
following pairs: (2014)
(d) 2 and 3 only
Famous Shrine Location
1. Tabo monastery and Spiti Valley
3. Consider the following pairs: (2017)
temple complex
Traditions Communities 2. Lhotsava Lhakhang Zanskar Valley
1. Chaliha Sahib Festival Sindhis temple, Nako
2. Nanda Raj Jaat Yatra Gonds 3. Alchi temple complex Ladakh
3. Wari-Warkari Santhals Which of the pairs given above is/are
correctly matched?
Which of the pairs given above is/are
correctly matched? (a) 1 only

(a) 1 only (b) 2 and 3 only


(c) 1 and 3 only

125 Indian Traditions, Festivals, and Calendars


unacademy.com | Download the Unacademy app
Give your feedback here: Link
(d) 1, 2 and 3 (d) It is an ancient martial art and a living
tradition in some parts of South India

7. With reference to India's culture and


tradition, what is `Kalaripayattu'? (2014) 8. Chaitra 1 of the national calendar based
(a) It is an ancient Bhakti cult of Shaivism on the Saka Era corresponds to which one
still prevalent in some parts of South of the following dates of the Gregorian
India calendar in a normal year of 365 days?
(2014)
(b) It is an ancient style bronze and brass
work still found in southern part of (a) 22nd March (or 21st March)
Coromandel area (b) 15th May (or 16th May)
(c) It is an ancient form of dance-drama (c) 31st March (or 30th March)
and a living tradition in the northern (d) 21st April (or 20th April)
part of Malabar

Indian Traditions, Festivals, and Calendars 126


unacademy.com | Download the Unacademy app
Give your feedback here: Link
Indian Traditions, Festivals, and
6 Calendars-Explanation
1. Answer: (c) or drum. This art form depicts stories of the
Statement 1 is correct: St. Francis Xavier was heroic battle of Khonjom, fought by the people
the greatest Roman Catholic missionary of of Manipur against the British empire in April
modern times who was instrumental in the 1891. It is one of the most popular musical
establishment of Christianity in India, the art forms of the state and spurs the spirit of
Malay Archipelago, and Japan. He was one of patriotism and nationalism in the audience.
the founding members of the Jesuit Order. Pair 3 is not correctly matched: Thang-Ta
In Paris, in 1534, he pronounced vows as one is a Manipuri art form. It combines ritual,
of the first seven members of the Society of demonstration and combat and involves a
Jesus, or Jesuits, under the leadership of St. variety of dance forms and warrior drills.
Ignatius of Loyola.
Statement 2 is not correct: St. Francis Xavier 3. Answer: (a)
died in 1552 in Shangchuan Island, China.
Pair 1 is correctly matched: The Chaliha is an
Statement 3 is correct: The Feast of St. important festival celebrated by the Sindhis
Francis Xavier is a time-honoured feast that is wherein they pray to their God, Jhulelal. There
celebrated mainly in Goa each year. Each year, is a special ‘jyot’ that is believed to have
on 3rd December marks the anniversary of St protected the Sindhi people and their culture
Francis Xavier’s death, when thousands gather during the ancient times, from around 1897 AD.
at the Basilica of Bom Jesus. This annual
It was brought from the Puj Chaliha Sahib
festival, known as the Feast of St Francis
Mandir Peergoath in Sindh by the Sindhis to
Xavier or Goinchea Saibache Fest (Lord of Goa
India during partition in 1947. This Akhand
Festival), is the biggest of all the Christian
jyot was then placed in the Chaliha Sahib Puj
festivals in Goa. The feast of the 16th-century
Jhulelal Mandir in Ulhasnagar. The jyot is kept
Spanish Jesuit, who is revered as Goencho Saib
lit in a structure similar to the one in Sindh
(Lord of Goa) in Goa, where his sacred relics
to this date. Sindhis from all over the country
remain, is celebrated annually on December 3
come to worship Jhulelal and offer their prayers
and draws thousands of devotees from across
here during the festival. It is customary to also
the state and beyond.
go on a 40 day fast during the festival.
Pair 2 and pair 3 are not correctly matched:
2. Answer: (b) Nanda Devi Raj Jaat Yatra is a festival of
Pair 1 is correctly matched: Chapchar Kut, a Gharwali and kumaoni people in Uttarakhand.
popular spring festival, is celebrated by the Wari Warkari or Varkari (Pandharpur Wari) is a
Mizo people of Mizoram. Celebrated in early Maharashtrian festival celebrated by Marathas.
March, every year, it is one of the 3 festivals that
marks the various stages of the agricultural
4. Answer: (b)
cycle. The other two are Mim Kut and Pawl Kut.
This festival celebrates the completion of the Option (b) is correct: Every monsoon, the Gond
jhum operations in the state. and Korku tribes of Madhya Pradesh’s Betul
and Harda districts celebrate Hari Jiroti. It is a
Pair 2 is correctly matched: Khongjom Parba
month-long festival of greenery, during which
ballad is a style of ballad singing from Manipur.
the tribals plant saplings of fruit-bearing
It is accompanied with the sounds of a dholak

127 Indian Traditions, Festivals, and Calendars-Explanation


unacademy.com | Download the Unacademy app
Give your feedback here: Link
trees. The festival starts with the green march Ladakh. The Alchi Monastery dates back to
celebrating the beginning of the rains. The nearly one thousand years and so the effect of
tribals still depend on forest-based tubers, the Tibetan influence can be seen in the local
leaves and flowers. culture.

5. Answer: (b) 7. Answer: (d)


Option (b) is correct: The Manganiyars are Option (d) is correct: Kalaripayattu is one of the
professional Muslim folk musicians belonging oldest (ancient) martial art forms, practised in
to Jaisalmer, Barmer, parts of Jalor, Bikaner Kerala and parts of Tamil Nadu.
and Jodhpur districts in Western Rajasthan. y Kalaripayattu’s influence can be seen in
For generations, they provide musical service most of the art forms of Kerala. In the past
to their patrons to receive cattle, camels, goats kalaris (combat arena or battlefield) formed
or cash as gifts. On the other side they function an integral part of the lives of the people.
as record keepers and keep the different family Without any gender discrimination boys
histories alive through their songs, based on and girls in their childhood would be sent
pure oral tradition. to the Kalaris for practising.
y Kalaripayattu includes strikes, kicks,
6. Answer: (c) grappling, preset forms, weaponry and
Pair 1 is correctly matched: Tabo Monastery, in healing methods.
Spiti Valley, Himachal Pradesh, is sometimes
referred to as the Ajanta of the Himalayas. It is 8. Answer: (a)
the oldest earthen Buddhist monastery in the
Himalayan region, established over 1,000 years The national calendar based on the Saka Era,
ago. Today, the old temple site is made up of with Chaitra as its first month and a normal
nine temples and several stupas, in addition year of 365 days was adopted from 22 March
to cave shrines along the mountain slope. The 1957 along with the Gregorian calendar for the
shrines are decorated with murals and are following official purposes:
traditionally used for meditation. y Gazette of India.
Pair 2 is not correctly matched: Lhotsava y News broadcast by All India Radio.
Lhakhang temple is located at Nako, Kinnaur y Calendars issued by the Government of
region of Himachal Pradesh. Zanskar Valley is India.
in Kargil, Ladakh.
y Government communications addressed to
Pair 3 is correctly matched: Alchi Monastery or the members of the public.
Alchi Gompa (monastery and temple complex)
Option (a) is correct: Dates of the national
lies about 65 km from Leh on the banks of
calendar have a permanent correspondence
the Indus River. Alchi is regarded as one of the
with dates of the Gregorian calendar, 1st
most important Buddhist centres in Ladakh
Chaitra falling on 22 March normally and on 21
and also as one of the monastic jewels of
March in a leap year.

Indian Traditions, Festivals, and Calendars-Explanation 128


unacademy.com | Download the Unacademy app
Give your feedback here: Link
7 Miscellaneous

1. With reference to Indian history, who of the Which of the above statements is/are
following were known as “Kulah-Daran”? correct?
(2022) (a) 1 only
(a) Arab merchants (b) 2 only
(b) Qalandars (c) Both 1 and 2
(c) Persian calligraphists (d) Neither 1 nor 2
(d) Sayyids

5. With reference to the history of India,


2. Consider the following pairs: (2022) consider the following pairs: (2020)

(Historical place) (Well – known Famous Place Present State


for) 1.
Bhilsa Madhya Pradesh
1. Burzahom Rock – cut shrines 2. Dwarasamudra Maharashtra
2. Chandra-ketugarh Terracotta art 3. Girinagar Gujarat
3. Ganeshwar Copper artifacts 4. Sthanesvara Uttar Pradesh
Which of the pairs given above is/are Which of the pairs given above are correctly
correctly matched? matched?
(a) 1 only (a) 1 and 3 only
(b) 1 and 2 (b) 1 and 4 only
(c) 3 only (c) 2 and 3 only
(d) 2 and 3 (d) 2 and 4 only

3. With reference to India, the terms ‘Halbi, 6. Consider the following pairs: (2018)
Ho and Kui’ pertain to (2021)
Craft Heritage of
(a) dance forms of Northwest India
1. Puthukkuli shawls Tamil Nadu
(b) musical instruments
2. Sujni embroidery Maharashtra
(c) prehistoric cave paintings
3. Uppada Jamdani Karnataka
(d) tribal languages sarees
Which of the pairs given above is/are
4. Consider the following statements: (2021) correct?
1. 21st February is declared to be the (a) 1 only
International Mother Language Day by (b) 1 and 2 only
UNICEF.
(c) 3 only
2. The demand that Bangla has to be one
of the national languages was raised in (d) 2 and 3 only
the Constituent Assembly of Pakistan.

129 Miscellaneous
unacademy.com | Download the Unacademy app
Give your feedback here: Link
7. Consider the following pairs: (2016) (a) 1 and 2 only
Famous place Region (b) 3 only
1. Bodhgaya Baghelkhand (c) 2 and 3 only
2. Khajuraho Bundelkhand (d) 1, 2 and 3
3.
Shirdi Vidarbha
4. Nasik (Nashik) Malwa 9. With reference to the scientific progress
5. Tirupati Rayalaseema of ancient India, which of the statements
given below are correct? (2012)
Which of the pairs given above are correctly
matched? 1. Different kinds of specialised surgical
instruments were in common use by
(a) 1, 2 and 4 only the 1st century AD.
(b) 2, 3, 4 and 5 only 2. Transplant of internal organs in the
(c) 2 and 5 only human body had begun by the beginning
(d) 1, 3, 4 and 5 only of 3rd century AD.
3. The concept of sine of an angle was
known in 5th century AD.
8. Consider the following languages (2014)
4. The concept of cyclic quadrilaterals
1. Gujarati
was known in the 7th century AD.
2. Kannada
Select the correct answer using the code
3. Telugu given below:
Which of the above has/have been declared (a) 1 and 2 only
as 'Classical Language / Languages' by the
(b) 3 and 4 only
Government?
(c) 1, 3 and 4 only
(d) 1, 2, 3 and 4

Miscellaneous 130
unacademy.com | Download the Unacademy app
Give your feedback here: Link
7 Miscellaneous-Explanation

1. Answer: (d) amongst others.


Option (d) is correct: The Sayyids claim descent
from the Prophet through his daughter Fatima. 3. Answer: (d)
The Sayyid put on a special pointed cap (Kulah)
Option (d) is correct: Some of the popular
and they were known as Kulah-Daran.
languages spoken by the tribes of India are as
follows: Bhatri, Bhilli, Halbi, Ho, Kui etc. Halabi
2. Answer: (d) is spoken by over 5.25 Lakh people in the states
Pair 1 is not correctly matched: The Burzahom of Maharashtra and Madhya Pradesh. It belongs
archaeological site is located in the Kashmir to the group of Indo-Aryan Tribal languages.
Valley of Jammu and Kashmir. The homes at Important Dravidian Tribal languages are
Burzahom were pits dug below ground level Kolami, Kui, Konda, Koya, Gondi, Oraon/Kurukh,
using stone tools. The sides of the pits were and Parji.
plastered with mud. The pits were usually round
or oval, narrow at the top and wide at the base. 4. Answer: (b)
It is a Neolithic Site. With this, India brings to
Statement 1 is not correct: International
light transitions in human habitation patterns
Mother Language Day is a worldwide annual
from Neolithic Period to the Megalithic period
observance held on 21 February to promote
to the Early Historic period. From the transition
awareness of linguistic and cultural diversity
in architecture to the development of tool-
and to promote multilingualism. The idea
making techniques to the introduction and
to celebrate International Mother Language
diffusion of lentils in north-western India, the
Day was the initiative of Bangladesh. It
site of Burzahom is a unique, comprehensive
was approved at the 1999 UNESCO General
storyteller of life between 3000 BCE to 1000
Conference and has been observed throughout
BCE. It is not famous for Rock-cut shrines.
the world since 2000. International Mother
Pair 2 is correctly matched: Archaeologist Language Day recognizes that languages and
Rakhaldas Banerji – the man who discovered multilingualism can advance inclusion, and
the ruins of Mohenjodaro – visited the Sustainable Development Goals focus on
Chandraketugarh and found a vast crop leaving no one behind.
of terracotta artifacts. He published his
The theme of the 2022 International Mother
impressions in the Bengal monthly, Basumati,
Language Day was “Using technology
in 1920. This site is in West Bengal.
for multilingual learning: Challenges and
Pair 3 is correctly matched: The Ganeshwar- opportunities”.
Jodhpura culture is a chalcolithic culture
Statement 2 is correct: Dhirendranath Datta
spread over the North-Eastern Rajasthan,
from East Pakistan demanded that Bangla
covering present districts such as Jhunjhunun,
has to be one of the national languages. He
Sikar, Jaipur, Tonk, Bhilwara, Sawai-Madhopur
raised this issue in the Constituent Assembly
and Bharatpur. The site of Ganeshwar yielded
of Pakistan in 1948. The Pakistani government-
a lot of copper objects such as arrowheads,
designated Urdu to be the only official language
spearheads, chisels, fishhooks, and razor
of Pakistan.
blades; ornaments like rings, hairpins, bangles,
antimony rods, and double spiral headed pins,

131 Miscellaneous-Explanation
unacademy.com | Download the Unacademy app
Give your feedback here: Link
5. Answer: (a) Cultural Heritage of Humanity.
Pair 1 is correctly matched: Vidisha is an
ancient city in Madhya Pradesh state in India. It 7. Answer: (c)
is also the administrative capital of the district
with the same name. It was known as Besnagar Pair 1 is not correctly matched: Bodh Gaya is in
in ancient times. During the Medieval period, the central part of the state of Bihar. It is part
Vidisha city was known as Bhilsa. Emperor of the great Ganges plains. Bihar is one of the
Ashoka the Great of Maurya Dynasty served four holy sites related to the life of the Lord
as the governor of Vidisha during his father’s Buddha, and particularly to the attainment of
regime. Enlightenment.

Pair 2 is not correctly matched: Hoysalas had Pair 2 is correctly matched: The temple at
their capital at Dwarasamudra, the modern Khajuraho, Madhya Pradesh (Bundelkhand)
Halebidu in Belur Taluk. It is in Karnataka. was made in the 10th century, about 400
years after the temple at Deogarh and the
Pair 3 is correctly matched: Girinagar is one complex is a UNESCO World Heritage Site.
of the most ancient 22nd Jain Tirthankar Lord The temples were patronized by Chandela
Neminath Prabhu Kshetras in India. It is in kings. The temples at Khajuraho are all made
Junagadh district of Gujarat. of Sandstone. These depict the style of the
Pair 4 is not correctly matched: Thanesar nagara temple architecture.
(sometimes called Thaneswar and, archaically, Pair 3 is not correctly matched: Shirdi is a town
Sthaneshwar) is a historic town and an in the state of Maharashtra, in western India.
important Hindu pilgrimage centre on the It is known as the former home of revered
banks of the Saraswati River in the state of spiritual leader Sai Baba, and as a major
Haryana in northern India. Prabhakara Vardhana pilgrimage site.
was a ruler of Thanesar in the early seventh-
century CE and was succeeded by his sons, Pair 4 is not correctly matched: Nashik is an
Rajyavardhana and Harshavardhana. ancient city and the largest city in the northern
region of the Indian state of Maharashtra. It is
situated on the banks of river Godavari. This
6. Answer: (a) place is well known for Kumbh Mela which is
Pair 1 is correctly matched: Puthukkuli shawls celebrated once in every twelve years and is
are made by Todas of Nilgiri hills in Tamil Nadu. known as Sinhasta. The city is also called the
It is locally called Pugur, meaning flower. The “Wine Capital of India”.
fine and intricate Toda embroidery is done by Pair 5 is correctly matched: The ancient and
tribal men and women on shawls. sacred temple of Sri Venkateswara is located
Pair 2 and pair 3 are not correctly matched: on the seventh peak, Venkatachala (Venkata
Sujini and Khatwa Embroidery is famous Hill) of the Tirupati Hill, lies on the southern
as well as beautiful work of handicraft in banks of Sri Swami Pushkarini what is known
Bihar. Jamdani style of weaving originated in popularly today as Tirupati Balaji Temple.
Bangladesh. Uppada Sari is a sari style woven Rayalaseema, meaning ‘rocky region’, includes
in Uppada of East Godavari district in Andhra the districts of Anantapur, Kurnool, Cuddapah
Pradesh. Jamdani is a ‘cotton’ fibre woven and Chittoor, south of the Krishna River in
delicately into the finest muslin. Uppada saree Andhra Pradesh. Tirupati Balaji temple has its
was registered by the Geographical Indications origins in Vaishnavism, which advocates the
Registry tag (GI Tag) for the jamdani technique principles of equality and love, and prohibits
in 2009. In 2013, the traditional art of weaving animal sacrifice. The sanctum sanctorum,
jamdani was declared a UNESCO Intangible which houses the awe-inspiring idol of the

Miscellaneous-Explanation 132
unacademy.com | Download the Unacademy app
Give your feedback here: Link
Lord of the Seven Hills, is situated in the main technology. The dependence on nature could
temple complex of Tirumala. be overcome by developments in science. In
ancient India, religion and science worked in
Elimination Technique: Malwa region close proximity.
generally refers to the region north of the Statement 1 is correct: There is evidence that
Vindhya Range. Nashik is not part of the the surgical knife dates back as far back as
Malwa region. Thus, Option (c) is correct. the Mesolithic era, around 8000 BC. One of the
oldest surgical procedures was trepanation or
trepanning, the art of drilling a hole into the
8. Answer: (c)
skull.
The guidelines for declaring a language as
Statement 2 is not correct: The first successful
‘Classical’ are:
internal organ transplant in the human body
y High antiquity of its early texts/recorded was done in the USA in the year 1954.
history over a period of 1500-2000 years.
Statement 3 is correct: The significant
y A body of ancient literature/texts, which development of trigonometry in India, was
is considered a valuable heritage by in the works known as Siddhantas (4th- 5th
generations of speakers. century) which first defined the sine as the
y The literary tradition is original and not modern relationship between half an angle
borrowed from another speech community. and half a chord.

y The classical language and literature being Statement 4 is correct: The formula for the
distinct from modern, there may also area of a cyclic quadrilateral was discovered
be a discontinuity between the classical by the 7th century Indian mathematician
language and its later forms or its offshoots. Brahmagupta.

Option (c) is correct: Currently, six languages


Elimination Technique: Transplant of
enjoy the ‘Classical’ status: Tamil (declared in
internal organs in the human body
2004), Sanskrit (2005), Kannada (2008), Telugu
requires modern technology which was
(2008), Malayalam (2013), and Odia (2014).
not available in 5 AD. Thus,Statement 2 is
not correct.
9. Answer: (c)
India has a rich heritage of science and

133 Miscellaneous-Explanation
unacademy.com | Download the Unacademy app
Give your feedback here: Link
5 WORLD
GEOGRAPHY
1 The Earth and the Universe

1. With reference to the Earth’s atmosphere, be interrupted.


which one of the following statements is Select the correct answer using the code
correct? (2023) given below:
(a) The total amount of insolation received (a) 1, 2, 4 and 5 only
at the equator is roughly about 10 times
that received at the poles. (b) 2, 3, 5, 6 and 7 only

(b) Infrared rays constitute roughly two- (c) 1, 3, 4, 6 and 7 only


thirds of insolation. (d) 1, 2, 3, 4, 5, 6 and 7
(c) Infrared waves are largely absorbed by
water vapour that is concentrated in 4. Consider the following pairs: (2020)
the lower atmosphere.
Rivers Flows into
(d) Infrared waves are a part of the visible
1. Mekong Andaman Sea
spectrum of electromagnetic waves of
solar radiation. 2. Thames Irish Sea
3. Volga Caspian Sea
2. In the northern hemisphere, the longest 4. Zambezi Indian Ocean
day of the year normally occurs in the: Which of the pairs given above is/are
(2022) correctly matched?
(a) First half of the month of June (a) 1 and 2 only
(b) Second half of the month of June (b) 3 only
(c) First half of the month of July (c) 3 and 4 only
(d) Second half of the month of July (d) 1, 2 and 4 only

3. If a major solar storm (solar flare) reaches 5. On 21st June, the Sun: (2019)
the Earth, which of the following are the
(a) does not set below the horizon at the
possible effects on the Earth? (2022)
Arctic Circle.
1. GPS and navigation systems could fail.
(b) does not set below the horizon at the
2. Tsunamis could occur at equatorial Antarctic Circle.
regions.
(c) shines vertically overhead at noon on
3. Power grids could be damaged. the Equator.
4. Intense auroras could occur over much (d) shines vertically overhead at the Tropic
of the Earth. of Capricorn.
5. Forest fires could take place over much
of the planet.
6. Consider the following statements: (2018)
6. Orbits of the satellites could be
1. The Earth's magnetic field has reversed
disturbed.
every few hundred thousand years.
7. Shortwave radio communication of the
2. When the Earth was created more than
aircraft flying over polar regions could

135 The Earth and the Universe


unacademy.com | Download the Unacademy app
Give your feedback here: Link
4000 million years ago, there was 54% life on the Earth? (2012)
oxygen and no carbon dioxide. (a) Hydrogen, Oxygen, Sodium
3. When living organisms originated, they (b) Carbon, Hydrogen, Nitrogen
modified the early atmosphere of the
Earth. (c) Oxygen, Calcium, Phosphorus

Which of the statements given above is/are (d) Carbon, Hydrogen, Potassium
correct? 10. A person stood alone in a desert on a dark
(a) 1 only night and wanted to reach his village which
was situated 5 km east of the point where
(b) 2 and 3 only he was standing. He had no instruments
(c) 1 and 3 only to find the direction, but he located the
(d) 1, 2 and 3 polestar. The most convenient way now to
reach his village is to walk in the: (2012)
(a) direction facing the polestar
7. Which of the following phenomena
might have influenced the evolution of (b) direction opposite to the polestar
organisms? (2014) (c) direction keeping the polestar to his left
1. Continental drift (d) direction keeping the polestar to his
2. Glacial cycles right

Select the correct answer using the code


given below: 11. What is the difference between asteroids
(a) 1 only and comets? (2011)

(b) 2 only 1. Asteroids are small rocky planetoids,


while comets are formed of frozen gases
(c) Both 1 and 2 held together by rocky and metallic
(d) Neither 1 nor 2 material.
2. Asteroids are found mostly between the
8. Variations in the length of daytime and orbits of Jupiter and Mars, while comets
night-time from season to season are due are found mostly between Venus and
to (2013) mercury.

(a) the earth’s rotation on its axis 3. Comets show perceptible glowing tails,
while asteroids do not.
(b) the earth’s revolution around the sun in
an elliptical manner Which of the statements given above is/are
correct?
(c) latitudinal position of the place
(a) 1 and 2 only
(d) revolution of the earth on a tilted axis
(b) 1 and 3 only
(c) 3 only
9. Which one of the following sets of elements
was primarily responsible for the origin of (d) 1, 2 and 3

The Earth and the Universe 136


unacademy.com | Download the Unacademy app
Give your feedback here: Link
The Earth and the Universe-
1 Explanation
1. Answer: (c)
atmosphere creating disruptions with signal
Infrared waves, particularly in certain
transmission from, say, a GPS satellite to Earth
wavelength ranges, are indeed absorbed by
causing it to be off by many yards. Another
water vapour that is concentrated in the
phenomenon produced by the sun could be
lower atmosphere. This characteristic of
even more disruptive. Known as a coronal
water vapour plays a significant role in the
mass ejection or CME, these solar explosions
Earth’s climate system and the greenhouse
propel bursts of particles and electromagnetic
effect.In the lower atmosphere, where water
fluctuations into Earth’s atmosphere. Those
vapour concentrations are typically higher,
fluctuations could induce electric fluctuations
the presence of water vapour absorbs a
at ground level that could blow out transformers
significant portion of the infrared radiation.
in power grids. A CME’s particles can also collide
This absorption contributes to the warming
with crucial electronics onboard a satellite and
of the lower atmosphere and helps regulate
disrupt its systems. (Source NASA website).
the Earth’s temperature. Water vapour acts as
a positive feedback mechanism in the climate A tsunami is a long, high sea wave caused by
system, as increased temperatures can lead an earthquake or oceanic volcanic activity.
to more water vapour, which further enhances Similarly, a forest fire can not be caused directly
the greenhouse effect. by solar flares as they are not too powerful to
burn the earth’s surface.

2. Answer: (b) Elimination Technique: Statement 2 is


very specific. There is no direct correlation
Option (b) is correct: Since a large portion of
between the solar storm and occurrence
the Northern Hemisphere is getting light from
of Tsunamis at equatorial regions. So, by
the sun, it is summer in the regions north of
eliminating that we are remaining with
the equator. The longest day and the shortest
only option C.
night at these places occur on 21st June (i.e.
second half of the month of June). At this
time in the Southern Hemisphere all these 4. Answer: (c)
conditions are reversed. It is winter there. The Pair 1 is not correctly matched: The Mekong
nights are longer than the days. This position River is one of the world’s great rivers. Covering
of the earth is called the Summer Solstice. a distance of nearly 5,000 km from its source
on the Tibetan Plateau in China to the Mekong
3. Answer: (c) Delta, the river flows through six countries:
China, Myanmar, Thailand, Lao PDR, Cambodia,
Option (c) is correct: A solar flare is an intense
and Vietnam. The Mekong Basin can be divided
burst of radiation coming from the release of
into two parts: the “upper Mekong basin”
magnetic energy associated with sunspots.
in Tibet, and the “lower Mekong basin” from
The explosive heat of a solar flare can’t make Yunnan downstream from China to the South
it all the way to our globe (Hence Statement China Sea.
5 can be said wrong), but electromagnetic
Pair 2 is not correctly matched: River Thames
radiation and energetic particles certainly can.
travels over 210 miles through the heart of
Solar flares can temporarily alter the upper
some of England’s most picturesque towns,

137 The Earth and the Universe-Explanation


unacademy.com | Download the Unacademy app
Give your feedback here: Link
including London, right into the centre of 6. Answer: (c)
London and eventually, out into the North Sea. Statement 1 is correct: The Earth’s magnetic
Pair 3 is correctly matched: The Volga River is field has flipped its polarity many times over
the longest in Europe. Its basin lies entirely the millennia. Reversals are the rule, not the
within the Russian Federation. It is often called exception. Earth has settled in the last 20
Mother Volga by the Russians. It is thought of million years into a pattern of a pole reversal
as the lifeblood of Russia. The Volga rises or about every 200,000 to 300,000 years, although
starts northwest of Moscow in the Valdai Hills. it has been more than twice that long since the
From there it flows north above Moscow, and last reversal. A reversal happens over hundreds
then south and southeast to the Caspian Sea. or thousands of years, and it is not exactly a
At the Caspian Sea, the Volga is an important clean backflip. Magnetic fields morph and
source of water for the sea and its famous push and pull at one another, with multiple
sturgeon fishery. poles emerging at odd latitudes throughout
Pair 4 is correctly matched: The Zambezi is the the process. Scientists estimate reversals have
fourth-longest river in Africa, the longest east- happened at least hundreds of times over the
flowing river in Africa and the largest flowing past three billion years. And while reversals
into the Indian Ocean from Africa. It serves as have happened more frequently in “recent”
the border between Zambia and Zimbabwe years, when dinosaurs walked Earth, a reversal
thundering over Victoria Falls. was more likely to happen only about every
few hundred thousand years. Earth’s polarity is
not constant.
5. Answer: (a)
Statement 2 is not correct: When the Earth
On 21st June, the Northern Hemisphere is was created more than 4000 million years ago,
tilted towards the Sun. The rays of the Sun fall it was a barren, rocky and hot object with a thin
directly on the Tropic of Cancer. As a result, atmosphere of hydrogen and helium. Between
these areas receive more heat. The areas near 4,600 million years and the present, led to the
the poles receive less heat as the rays of the evolution of life on the surface of the planet.
Sun are slanting. The early atmosphere was probably mostly
Option (a) is correct: The North Pole is inclined carbon dioxide, with little or no oxygen. There
towards the Sun and the places beyond the were smaller proportions of water vapour,
Arctic Circle experience continuous daylight ammonia, and methane. As the Earth cooled
for about six months, meaning the Sun does down, most of the water vapour condensed
not set below the horizon at the Arctic Circle. and formed the oceans.

Additional Information: Statement 3 is correct: There are three stages


in the evolution of the present atmosphere.
Since a large portion of the Northern
The first stage is marked by the loss of the
Hemisphere is getting light from the Sun, it is
primordial atmosphere. In the second stage,
summer in the regions north of the equator.
the hot interior of the earth contributed to
The longest day and the shortest night at these
the evolution of the atmosphere. Finally, the
places occur on 21st June.
composition of the atmosphere was modified
At this time in the Southern Hemisphere, all by the living world through the process of
these conditions are reversed. It is the winter photosynthesis.
season there. The nights are longer than the
days. This position of the Earth is called the Elimination Technique: Statement 2 is
Summer Solstice. fact loaded (i.e. 4000 million years ago,
54% oxygen and no carbon dioxide) which

The Earth and the Universe-Explanation 138


unacademy.com | Download the Unacademy app
Give your feedback here: Link
fixed path or orbit is called Revolution.
are vaguely mentioned. So, by eliminating
y The axis of the Earth, which is an imaginary
the statement 2 ae are remaining with the
line, makes an angle of 66° with its orbital
option A and C only.
plane. The plane formed by the orbit is
known as the orbital plane. The earth
7. Answer: (c) receives light from the sun. Due to the
spherical shape of the earth, only half of it
Physical factors upon which evolution depends
gets light from the sun at a time.
are Temperature, light, water, presence of
barriers, salinity, soil characteristics and y The portion, facing the sun, experiences
oxygen availability. day while the other half away from the sun
experiences night. The circle that divides
y Continental drift is the gradual movement
the day from night on the globe is called
of the continents across the earth’s surface
the circle of illumination. The earth takes
through geological time. As continents broke
about 24 hours to complete one rotation
apart from Pangaea, species got separated
around its axis. The period of rotation is
by seas and oceans and speciation
known as Earth Day. Throughout its orbit,
occurred. Individuals that were once able
the earth is inclined in the same direction.
to interbreed were reproductively isolated
from one another and eventually acquired
adaptations that made them incompatible 9. Answer: (b)
y This drives evolution by creating new Option (b) is correct: As per the Oparin-
species. Also, as the continents drift, they Haldane hypothesis, the atmosphere of the
move into new climates. What was once at early Earth may have been chemically reducing
the equator may now be near the poles. If in nature, composed primarily of methane
species did not adapt to these changes in (CH4), ammonia (NH3), water (H2O), hydrogen
the weather and temperature, then they sulphide (H2S), carbon dioxide (CO2) or carbon
would not survive and go extinct. New monoxide (CO), with phosphate (PO43-),
species would take their place and learn molecular oxygen (O2) and ozone (O3) either
to survive in the new areas. (Option 1 is rare or absent.
correct)
The present composition of the earth’s
y Glacial cycles affect the temperature atmosphere is chiefly contributed by nitrogen
range dramatically leading to variability in and oxygen. There are three stages in the
adaptations and natural selection takes evolution of the present atmosphere.
the lead. Therefore, evolution is influenced.
y The first stage is marked by the loss of the
(Option 2 is correct)
primordial atmosphere.
y In the second stage, the hot interior of the
8. Answer: (d) earth contributed to the evolution of the
Option (d) is correct: The variations in the atmosphere.
length of daytime and night-time from season y Finally, the composition of the atmosphere
to season are due to the revolution of the earth was modified by the living world through
on a tilted axis. the process of photosynthesis. The early
y Earth has two types of motion, namely atmosphere, with hydrogen and helium, is
rotation and revolution. Rotation is the supposed to have been stripped off as a
movement of the earth on its axis. The result of the solar winds. This happened
movement of the earth around the sun in a not only in the case of the earth but also

139 The Earth and the Universe-Explanation


unacademy.com | Download the Unacademy app
Give your feedback here: Link
in all the terrestrial planets, which were 11. Answer: (b)
supposed to have lost their primordial Statement 1 is correct: Comets are made up
atmosphere through the impact of solar mostly of frozen gases and a mass of dust. The
winds. surface is icy and as the comet approaches
During the cooling of the earth, gases and the sun, the ice evaporates. That makeup
water vapour were released from the interior distinguishes comets from asteroids because
solid earth. This started the evolution of the asteroids, on the other hand, are made up
present atmosphere. of rocky and metallic material. Asteroids are
The early atmosphere largely contained water warmer since they are much closer to the sun.
vapour, nitrogen, carbon dioxide, methane, Statement 2 is not correct: Asteroids and
ammonia and very little free oxygen. The process Comets are considered near-earth objects
through which the gases were outpoured from (NEO). They can be visible and there is always
the interior is called degassing. Continuous a possibility of earth collision but still, the
volcanic eruptions contribute to water vapour chances are very, very slim. Both asteroids
and gases. and comets orbit around the sun. However,
comets tend to have an extremely elongated
movement around the sun while asteroids
10. Answer: (c) have more circular orbits and most of them
Polaris, commonly known as the North Star, is are found in the asteroid belt where they orbit
the closest relatively bright star to the north the sun between the planets Mars and Jupiter.
celestial pole. Polaris is easily visible to the Statement 3 is correct: Comets have a distinct
naked eye, but not exceptionally bright. It is characteristic of forming a visible coma and
the brightest star in the constellation Ursa at times, a long tail of ions pointing opposite
Minor, but only the 48th brightest star in the the sun which the asteroid does not have. This
sky. This, however, does not mean that it is not is because of the comet’s icy surface but far
particularly luminous. As a yellow supergiant, away from the sun, it is hard to distinguish a
the star is immense and only appears faint comet from an asteroid.
because it is so distant from Earth.
Option (c) is correct: In ancient times, people
used to determine directions during the night
with the help of stars. The North Star indicates
the north direction. It is also called the Pole
State. It always remains in the same position
in the sky. The Pole Star is situated in the
direction of the earth’s axis. It is not visible
from the southern hemisphere. The convenient
way now to reach his village is to walk in the
direction keeping the pole star to his left.

The Earth and the Universe-Explanation 140


unacademy.com | Download the Unacademy app
Give your feedback here: Link
2 Geomorphology

1. Consider the following statements: (2023) currents


1. In a seismograph, P waves are recorded (c) Difference in salinity of water
earlier than S waves. (d) Occurrence of the belt of calm near the
2. In P waves, the individual particles equator
vibrate to and fro in the direction of
waves propagation whereas in S waves,
the particles vibrate up and down at 3. Consider the following: (2013)
right angles to the direction of wave 1. Electromagnetic radiation
propagation. 2. Geothermal energy
Which of the statements given above is/are 3. Gravitational force
correct?
4. Plate movements
(a) 1 only
5. Rotation of the earth
(b) 2 only
6. Revolution of the earth
(c) Both 1 and 2
Which of the above are responsible for
(d) Neither 1 nor 2 bringing dynamic changes on the surface
of the earth?
2. What explains the eastward flow of the (a) 1, 2, 3 and 4 only
equatorial counter-current? (2015) (b) 1, 3, 5 and 6 only
(a) The Earth's rotation on its axis (c) 2, 4, 5 and 6 only
(b)
Convergence of the two equatorial (d) 1, 2, 3, 4, 5 and 6

141 Geomorphology
unacademy.com | Download the Unacademy app
Give your feedback here: Link
2 Geomorphology-Explanation

1. Answer: (c) y Equatorial counter-currents are major


Statement 1 is correct: P waves, also known as surface flows that carry water eastward
primary waves or compressional waves, are the in the Atlantic, Indian, and Pacific Oceans.
fastest seismic waves and travel through the They are located near the equator and
Earth’s interior in a push-pull motion. They can are sandwiched between two westward-
travel through solid rock, liquids, and gases, flowing currents, the North Equatorial
and therefore arrive at a seismic station before Current and the South Equatorial Current.
other types of waves. Equatorial counter-currents are unique in
that they flow in the opposite direction of
S waves, also known as secondary waves or
the surface winds. The other major surface
shear waves, travel more slowly than P waves.
currents in the tropics flow in the same
Unlike P waves, S waves cannot travel through
direction as the prevailing winds.
liquids, so they are only observed in solid
materials. y The equatorial counter-currents are driven
by a distinct surface wind pattern in the
Due to their faster speed, P waves reach a
tropics. Strong westward trade winds
seismograph station before the arrival of S
result in westward surface flow in most
waves. The time interval between the arrival of
of the tropical Atlantic and Pacific Oceans.
P waves and S waves can be used to determine
The stronger winds to the south pile up
the distance between the seismic station and
water where the winds are weak. The
the earthquake’s epicenter.
excess water flows eastward under the
Statement 2 is correct:In P waves (primary influence of the Earth’s rotation, giving rise
waves), the individual particles vibrate in the to equatorial counter-currents.
same direction as the wave propagation. The
motion of the particles is in a push-pull or
compressional motion, similar to how a slinky 3. Answer: (d)
compresses and expands. This means that the Option (d) is correct: Earth’s surface is
particles move back and forth parallel to the constantly changing. Wind, water, and ice break
direction in which the wave is travelling. down large rocks and move sediments on the
On the other hand, in S waves (secondary surface. It usually takes years for weathering,
waves), the particles vibrate perpendicular to erosion, and deposition to cause noticeable
the direction of wave propagation. The motion changes. Some events, though, bring dynamic
of the particles is in a shearing or transverse changes on the Earth’s surface much more
motion, similar to the movement of a rope quickly, such as:
when you shake it side to side. This means y Temperature and precipitation are the two
that the particles move up and down or side to important climatic elements that control
side, but their motion is perpendicular to the various processes. Electro-magnetic
direction of wave travel. radiation causes temperature change which
induces the metamorphism of rocks.

2. Answer: (b) y Geothermal energy movements cause


volcanoes which bring dynamic change on
Option (b) is correct: Convergence of the two
the Earth surface.
equatorial currents is the reason behind the
eastward flow of the equatorial counter-current. y Tides occur due to the gravitational pull of

Geomorphology-Explanation 142
unacademy.com | Download the Unacademy app
Give your feedback here: Link
the Moon. Tides cause coastal erosion. and Night formation that create temperature
y Earthquakes (plate movements) bring differences. The Revolution of the Earth is
dynamic change on the earth surface. responsible for seasonal changes. Both are
responsible for bringing dynamic changes
y Rotation of the Earth is responsible for Day to the surface of the earth.

143 Geomorphology
unacademy.com | Download the Unacademy app
Give your feedback here: Link
3 Climatology

1. Consider the following statements: (2023) 3. Consider the following statements: (2020)
Statement-I: 1. Jet streams occur in the Northern
The temperature contrast between Hemisphere only.
continents and oceans is greater during 2. Only some cyclones develop an eye.
summer than in winter. 3. The temperature inside the eye of a
Statement-II: cyclone is nearly 10°C less than that of
The specific heat of water is more than that the surroundings.
of land surface. Which of the statements given above is/are
Which one of the following is correct in correct?
respect of the above statements? (a) 1 only
(a) Both Statement-I and Statement-II are (b) 2 and 3 only
correct and Statement-II is the correct (c) 2 only
explanation for Statement-I
(d) 1 and 3 only
(b)
Both Statement-I and Statement-II
are correct and Statement-II is not the
correct explanation for Statement-I 4. Why are dewdrops not formed on a cloudy
(c) Statement-I is correct but Statement-II night? (2019)
is incorrect (a) Clouds absorb the radiation released
(d) Statement-I is incorrect but Statement- from the Earth’s surface.
II is correct (b) Clouds reflect back the Earth’s radiation.
(c) The Earth’s surface would have low
2. Consider the following statements: (2022) temperatures on cloudy nights.

1. High clouds primarily reflect solar (d) Clouds deflect the blowing wind to
radiation and cool the surface of the ground level.
Earth.
2. Low clouds have a high absorption 5. During a thunderstorm, the thunder in the
of infrared radiation emanating from skies is produced by the (2013)
the Earth’s surface and thus cause a 1. Meeting of cumulonimbus clouds in the
warming effect. sky.
Which of the statements given above is/are 2. Lightning that separates the nimbus
correct? clouds.
(a) 1 only 3. Violent upward movement of air and
(b) 2 only water particles.
(c) Both 1 and 2 Select the correct answer using the code
(d) Neither 1 nor 2 given below:
(a) 1 only
(b) 2 and 3 only

Climatology 144
unacademy.com | Download the Unacademy app
Give your feedback here: Link
(c) 1 and 3 only km/sec can severely harm living beings if
(d) None of the above produces the thunder they reach the surface of the Earth. What
prevents them from reaching the surface
of the Earth? (2012)
6. The annual range of temperature in
(a) The Earth’s magnetic field diverts them
the interior of the continents is high as
towards its poles
compared to coastal areas. What is/are the
reason/reasons? (2013) (b) Ozone layer around the Earth reflects
them back to outer space
1. Thermal difference between land and
water (c)
Moisture in the upper layers of
atmosphere prevents them from
2. Variation in altitude between continents
reaching the surface of the Earth
and oceans
(d) None of the statements (a), (b) and (c)
3. Presence of strong winds in the interior
given above is correct
4. Heavy rains in the interior as compared
to coasts
9. The jet aircrafts fly very easily and smoothly
Select the correct answer using the code
in the lower stratosphere. What could be
given below:
the appropriate explanation? (2011)
(a) 1 only
1. There are no clouds or water vapour in
(b) 1 and 2 only the lower stratosphere.
(c) 2 and 3 only 2. There are no vertical winds in the lower
(d) 1, 2, 3 and 4 stratosphere.
Which of the statements given above is/are
correct in this context?
7. Normally, the temperature decreases with
the increase in height from the Earth’s (a) 1 only
surface, because: (2012) (b) 2 only
1. The atmosphere can be heated upwards (c) Both 1 and 2
only from the Earth’s surface.
(d) Neither 1 nor 2
2. There is more moisture in the upper
atmosphere.
10. Westerlies in the southern hemisphere
3. The air is less dense in the upper
are stronger and persistent than in the
atmosphere.
northern hemisphere. Why? (2011)
Select the correct answer using the code
1. The Southern Hemisphere has less
given below:
landmass as compared to the Northern
(a) 1 only Hemisphere.
(b) 2 and 3 only 2. Coriolis force is higher in the Southern
(c) 1 and 3 only Hemisphere as compared to the
Northern Hemisphere.
(d) 1, 2 and 3
Which of the statements given above is/are
correct?
8. Electrically charged particles from space
(a) 1 only
travelling at speeds of several hundred
(b) 2 only

145 Climatology
unacademy.com | Download the Unacademy app
Give your feedback here: Link
(c) Both 1 and 2 Pacific Ocean.
(d) Neither 1 nor 2 2. EI Nino has an adverse effect on the
southwest monsoon of India, but La
Nina has no effect on monsoon climate.
11. La Nina is suspected to have caused recent
floods in Australia. How is La Nina different Which of the statements given above is/are
from EI Nino? (2011) correct?

1. La Nina is characterised by unusually (a) 1 only


cold ocean temperature in the (b) 2 only
equatorial Indian ocean whereas EI Nino (c) Both 1 and 2
is characterised by unusually warm
ocean temperature in the equatorial (d) Neither 1 nor 2

Climatology 146
unacademy.com | Download the Unacademy app
Give your feedback here: Link
3 Climatology-Explanation

1. Answer: (a) is a relatively calm and clear area of sinking


Statement 1 is correct: The heat capacity of air and light winds that usually do not exceed
the ocean is much greater than that of the 15 mph (24 km/h) and is typically 20-40 miles
atmosphere or the land. As a result, the ocean (32-64 km) across. An eye will usually develop
slowly warms in the summer, keeping air cool, when the maximum sustained wind speeds go
and it slowly cools in winter, keeping the air above 74 mph (119 km/h) and is the calmest
warm. The temperature contrast between part of the storm. Hence, only some cyclones
continents and oceans is greater during develop an eye.
summer than in winter Statement 3 is not correct: The eye is the
Statement 2 is correct: Specific heat capacity is region of lowest surface pressure and warmest
a measure of how much heat energy is required temperatures. The eye temperature may be
to raise the temperature of a substance by a 10°C warmer or more at an altitude of 12 km
certain amount. Water has a relatively high than the surrounding environment, but only
specific heat capacity compared to most other 0-2°C warmer at the surface in the tropical
substances, including land surfaces. cyclone.

2. Answer: ( d) 4. Answer: (b)

Statement 1 is incorrect: High clouds are Option (b) is correct: Dew is formed when
often thin and do not reflect very much. They the atmospheric temperature draws the level
let lots of the Sun’s warmth in. They radiate to the dewpoint temperature. This often
less energy into space than the lower, warmer happens at ground level for two reasons. First,
clouds. Therefore, high clouds work to “trap” longwave emission causes the Earth’s surface
more energy than the low clouds. to cool at night and condensation requires the
temperature to decrease to the dew point.
Statement 2 is incorrect: Low clouds are often
Second, the soil often acts as the moisture
quite thick and reflect lots of sunlight back
source for dew. Warm and moist soils will help
to space. Low clouds are excellent reflectors.
with the formation of dew as the soil cools
But, they don’t stop the longwave energy from
overnight.
escaping to space. Therefore, low clouds help
to cool the Earth. Dewdrops are not formed on a cloudy night
because clouds reflect back the Earth’s
radiation.
3. Answer: (c)
Additional Information:
Statement 1 is not correct: Jet streams occur
The ideal conditions for its formation are clear
both in the Northern Hemisphere as well
sky, calm air, high relative humidity, and cold
as in the Southern Hemisphere. The actual
and long nights. For the formation of dew, it
appearance of jet streams results from the
is necessary that the dew point is above the
complex interaction between many variables -
freezing point.
such as the location of high- and low-pressure
systems, warm and cold air, and seasonal
changes. 5. Answer: (d)
Statement 2 is correct: The hurricane’s centre Thunderstorms develop in hot, humid tropical

147 Climatology-Explanation
unacademy.com | Download the Unacademy app
Give your feedback here: Link
areas like India very frequently. The rising
temperatures produce strong upward rising heating of land and sea creates different
winds. These winds carry water droplets air pressure zones in different seasons
upwards, where they freeze, and fall down in and around the Indian subcontinent.
again. The swift movement of the falling water Difference in air pressure causes reversal in
droplets along with the rising air creates the direction of monsoon winds.
lightning and sound. y Variation in altitude between continents
y A thunderstorm is a well-grown and oceans has little impact on the annual
cumulonimbus cloud producing thunder range of temp. But Temperature decreases
and lightning. When the clouds extend with height. Due to thin air, places in the
to heights where sub-zero temperature mountains are cooler than places on the
prevails, hails are formed, and they come plains. (Statement 2 is not correct)
down as hailstorms. If there is insufficient y Presence of strong winds in the interior is not
moisture, a thunderstorm can generate a main contributing factor for temperature
dust storms. variability. Rainfall is heavier in coastal
y A thunderstorm is characterised by an areas than interior generally. (Statement 3
intense updraft of rising warm air, which and statement 4 are not correct)
causes the clouds to grow bigger and rise to
Elimination Technique: Variation in altitude
a greater height. This causes precipitation.
between continents and oceans has
Later, downdraft brings down to earth the
nothing to do with the range of temperature
cool air and the rain.
in the interior of the continents. So, by
y From severe thunderstorms sometimes eliminating the statement 2, we can get
spiralling wind descends like a trunk of an the answer A.
elephant with great force, with very low
pressure at the centre, causing massive
destruction on its way. Such a phenomenon 7. Answer: (c)
is called a tornado. Tornadoes generally Option (c) is correct: In the troposphere,
occur in middle latitudes. The tornado over temperature decreases as altitude increases
the sea is called a waterspout. primarily because Earth’s atmosphere is heated
Option (d) is correct: None of the given reasons upward from the lowest level. The atmosphere
explains the production of thunder in the sky is warmed from the ground up, and since the
during a thunderstorm. air is at its most dense near the surface of the
Earth, the air near the surface is going to be
able to remain much more heat than the air at
6. Answer: (a) higher elevations due to the increased amounts
Statement 1 is correct: As compared to the of air molecules; higher elevations have fewer
landmass, water heats up or cools down slowly. air molecules and consequently cannot remain
Due to this thermal difference between land much heated.
and water, the annual range of temperature Simply putting, as elevation increases, there
in the interior of the continents is high as is less air above thus the pressure decreases.
compared to coastal areas. As the pressure decreases, air molecules
y India is flanked by the Indian Ocean on three spread out further (i.e., air expands) and
sides in the south and girdled by a high and the temperature decreases. Temperature
continuous mountain wall in the north. As decreases with altitude only in the troposphere
compared to the landmass, water heats and the mesosphere.
up or cools down slowly. This differential

Climatology-Explanation 148
unacademy.com | Download the Unacademy app
Give your feedback here: Link
Statement 2 is correct: As one moves upward
through the stratosphere Temperatures rise
Elimination Technique: Statement 2 and 3 opposite of the behaviour in the troposphere in
are contradictory and can not be correct which we live, where temperatures drop with
simultaneously. In the upper atmosphere increasing altitude. Because of this temperature
moisture is less. So, by eliminating the stratification, there is little convection and
statement 2 we can get the answer C. mixing in the stratosphere, so the layers of air
there are quite stable. Jet aircraft fly in the
lower stratosphere to avoid the turbulence
8. Answer: (a) which is common in the troposphere below.
Option (a) is correct: The Earth itself is a magnet,
with a magnetic north pole and south pole. 10. Answer: (a)
The origin of the Earth’s magnetic field is
said to be a result of the dynamo effect. The westerly winds, also known as the
Electric currents produced by the rotation westerlies, occur at two regions on Earth:
of iron-nickel cores. The Earth’s magnetic between 30 and 60 degrees latitude in the
field continually traps moving charged Northern Hemisphere and between 30 and 60
particles coming from the sun, called the degrees latitude in the Southern Hemisphere.
solar wind. It Protects us from solar winds The name of these unique winds comes from
containing electrically charged Particles. the direction of their origin; the westerlies run
west to east while other winds run east to
west.
9. Answer: (c) Statement 1 is correct: In the Southern
Statement 1 is correct: The amount of water Hemisphere, westerlies are particularly strong,
vapour in the stratosphere is very low, especially in areas where land is absent,
only 5 out of one million air molecules are because land amplifies the flow pattern,
water molecules. This means that under making the current more north-south oriented,
normal conditions there are no clouds in slowing the westerlies.
the stratosphere. Clouds are found almost Statement 2 is not correct: Coriolis force is
exclusively in the troposphere. strongest in the polar regions and zeroes at
the equator. At the intermediate levels, it varies
directly as the sine of the latitude.

149 Climatology-Explanation
unacademy.com | Download the Unacademy app
Give your feedback here: Link
11. Answer: (d) rains in India. A La Nina is actually beneficial
Statement 1 is not correct: La Nina is a cold for the Indian monsoon.
event. Here, the water temperature in the Statement 2 is not correct: Since 1950, out of
Eastern Pacific gets colder than normal. As a the 13 droughts that India faced, 10 have been
result of this, there is a strong high pressure during El Nino years and one in a La Nina year. El
over the eastern equatorial Pacific. Now, Nino means lesser than average rains for India.
there is low pressure in the Western Pacific Indian agriculture is heavily dependent on the
and off Asia. La Nina causes drought in Peru monsoons and because of this, lesser rainfall
and Ecuador, heavy floods in Australia, high during the monsoons generally translates to
temperatures in Western Pacific, Indian Ocean, below-average crop yields.
off the Somalian coast and good monsoon

Climatology-Explanation 150
unacademy.com | Download the Unacademy app
Give your feedback here: Link
4 Oceanography

1. Consider the following statements: (2021) 2. OMT collected during January March
1. In the tropical zone, the western can be used in assessing whether the
sections of the oceans are warmer amount of rainfall in monsoon will be
than the eastern sections owing to the less or more than a certain long-term
influence of trade winds. mean

2. In the temperate zone, westerlies make Select the correct answer using the code
the eastern sections of oceans warmer given below
than the western sections. (a) 1 only
Which of the statements given above is/are (b) 2 only
correct? (c) Both 1 and 2
(a) 1 only (d) Neither 1 nor 2
(b) 2 only
(c) Both 1 and 2 4. Consider the following statements: (2018)
(d) Neither 1 nor 2 1. Most of the world's coral reefs are in
tropical waters.
2. With reference to the water on the planet 2. More than one-third of the world's coral
Earth, consider the following statements: reefs are located in the territories of
1. The amount of water in the rivers Australia, Indonesia and Philippines.
and lakes is more than the amount of 3. Coral reefs host far more animal
groundwater. phyla than those hosted by tropical
2. The amount of water in polar ice caps rainforests.
and glaciers is more than the amount of Which of the statements given above is/are
groundwater. correct?
Which of the statements given above is/are (a) 1 and 2 only
correct? (b) 3 only
(a) 1 only (c) 1 and 3 only
(b) 2 only (d) 1, 2 and 3
(c) Both 1 and 2
(d) Neither 1 nor 2 5. Which of the following has/have shrunk
immensely/dried up the recent past due to
3. With reference to Ocean Mean Temperature human activities? (2018)
(OMT), which of the following statements 1. Aral Sea
is/are correct? (2020) 2. Black Sea
1. OMT is measured up to a depth of 3. Lake Baikal
26°C isotherm which is 129 metres in
the south-western Indian Ocean during Select the correct answer using the code
January – March. given below:
(a) 1 only

151 Oceanography
unacademy.com | Download the Unacademy app
Give your feedback here: Link
(b) 2 and 3 only (a) Warm and cold atmospheric currents
(c) 2 only meet

(d) 1 and 3 only (b)


Rivers drain out large amounts of
freshwater into the sea
(c) Warm and cold oceanic currents meet
6. In the South Atlantic and South-Eastern
Pacific regions in tropical latitudes, (d) Continental shelf is undulating
cyclones do not originate. What is the
reason? (2015) 10. The acidification of oceans is increasing.
(a) Sea surface temperatures are low Why is this phenomenon a cause of
(b) Inter-Tropical Convergence Zone seldom concern? (2012)
occurs 1. The growth and survival of calcareous
(c) Coriolis force is too weak phytoplankton will be adversely
affected.
(d) Absence of land in those regions
2. The growth and survival of coral reefs
will be adversely affected.
7. Tides occur in the oceans and seas due to 3. The survival of some animals that have
which among the following? (2015) phytoplanktonic larvae will be adversely
1. Gravitational force of the Sun affected.
2. Gravitational force of the Moon 4. The cloud seeding and formation of
3. Centrifugal force of the Earth clouds will be adversely affected.

Select the correct answer using the code Which among the statements given above
given below: is/are correct?

(a) 1 only (a) 1, 2 and 3 only

(b) 2 and 3 only (b) 2 only

(c) 1 and 3 only (c) 1 and 3 only

(d) 1, 2 and 3 (d) 1, 2, 3 and 4

8. On the planet earth, most of the freshwater 11. Consider the following factors (2012)
exists as ice caps and glaciers. Out of 1. Rotation of the Earth
the remaining freshwater, the largest 2. Air pressure and wind
proportion (2013)
3. Density of ocean water
(a) is bound in atmosphere as moisture
and clouds 4. Revolution of the Earth

(b) is found in freshwater lakes and rivers Which of the above factors influence the
ocean currents?
(c) exists as groundwater
(a) 1 and 2 Only
(d) exists as soil moisture
(b) 1, 2 and 3
(c) 1 and 4
9. The most important fishing grounds of
the world are found in the regions where (d) 2, 3 and 4
(2013)

Oceanography 152
unacademy.com | Download the Unacademy app
Give your feedback here: Link
4 Oceanography-Explanation

1. Answer: (c) varying from 50 –100 metres. During January


Statement 1 is correct: Trade wind generates –March, the mean 26 degree C isotherm depth
the equatorial current which pushes eastern in the Southwestern Indian Ocean is 59 metres.
section water in the western section. Statement 2 is correct: Ocean heat content
Statement 2 is correct: Warm water comes in (OHC) and Ocean Mean Temperature (OMT) are
contact with the Westerlies. They are moved important climatic parameters required for
into eastern part so In the temperate zone, atmospheric and oceanic studies like cyclone
westerlies make the eastern sections of oceans and monsoon predictions and ocean heat
warmer than the western sections. transport estimations. Sea surface temperature
(SST) is routinely used for predicting if the
total amount of rainfall that India receives
2. Answer: (b) during the monsoon season will be less or
Earth is known as the “Blue Planet’’ because 71 more than the long term mean of 887.5 mm.
per cent of the Earth’s surface is covered with Now, scientists from Pune’s Indian Institute
water. Water also exists below the land surface of Tropical Meteorology (IITM) find that ocean
and as water vapour in the air. The earth has mean temperature (OMT) has better ability to
an abundance of water, but unfortunately, only predict this than the sea surface temperature.
a small percentage (about 0.3 per cent) is even
usable by humans. The other 99.7 percent is in
4. Answer: (d)
the oceans, soils, icecaps, and floating in the
atmosphere. Coral Reefs are most commonly known as the
rainforest of oceans. They are the underwater
Distribution of the water on Earth
structures that are formed of coral polyps,
y Ocean water: 97.2 percent held together by calcium carbonate.
y Glaciers and other ice: 2.15 percent Coral polyps are short-lived microscopic
y Groundwater: 0.61 percent organisms, which live in colonies. They flourish
in shallow, mud free and warm waters. They
y Freshwater lakes: 0.009 percent
secrete calcium carbonate. The coral secretion
y Inland seas: 0.008 percent and their skeletons from coral deposits, in the
y Soil Moisture: 0.005 percent form of reefs, are mainly of three kinds: barrier
reef, fringing reef and atolls. The Great Barrier
y Atmosphere: 0.001 percent
Reef of Australia is a good example of the first
y Rivers: 0.0001 percent. kind of coral reefs.
Hence, statement 1 is not correct, but Statement 1 is correct: Mostly the coral reefs
statement 2 is correct. are in tropical waters, located between the
Tropics of Cancer and Capricorn, in the Pacific
Ocean, the Indian Ocean, the Caribbean Sea,
3. Answer: (b)
the Red Sea, and the Persian Gulf.
Statement 1 is not correct: The Ocean Mean
Statement 2 is correct: Indonesia, followed by
Temperature (OMT) is measured up to a depth
Australia and the Philippines are the largest
of 26 degree C isotherm, is more stable and
reef nations, while France comes in fourth.
consistent, and the spatial spread is also less.
More than one-third of the world’s coral reefs
The 26 degree C isotherm is seen at depths

153 Oceanography-Explanation
unacademy.com | Download the Unacademy app
Give your feedback here: Link
are located in the territories of Australia, Habitable area in Black Sea is shrinking, but
Indonesia, and Philippines. the actual volume is not shrinking immensely.
Statement 3 is correct: Coral reefs are the most In 2015, Russia officially declared Lake Baikal
diverse of all marine ecosystems. Reefs cover an emergency zone because of a catastrophic
just a tiny fraction (less than one percent) of drop in its water level. Baikal is the globe’s
the earth’s surface and less than two percent of largest freshwater lake and is on the UNESCO
the ocean bottom. Because they are so diverse, World Heritage list. Industrial area of the
coral reefs are often called the rainforests of closed Baikal Pulp and Paper Mill that has been
the sea. Coral reefs host a greater number of polluting the lake for decades. building more
animal phyla than those hosted by tropical hydroelectric dams near Lake Baikal can cause
rainforests. the unique lake to dry out. It is shrinking but
the fate is not immense. It is not dried up.

5. Answer: (a)
Option (a) is correct: Decades ago, the Aral 6. Answer: (b)
Sea was the world’s fourth largest body of Tropical cyclones are violent storms that
inland water, but it has shrunk immensely originate over oceans in tropical areas and move
due to diversion of rivers for irrigation (human over to the coastal areas bringing about large-
activities). Two decades ago, it split into the scale destruction caused by violent winds,
small northern and larger southern sections, very heavy rainfall, and storm surges. This is
and further divisions have been happening ever one of the most devastating natural calamities.
since. Moreover, in the 1960s, the Soviet Union They are known as Cyclones in the Indian
undertook major water diversion projects on Ocean, Hurricanes in the Atlantic, Typhoons in
the Syr Darya and Amu Darya rivers, capturing the Western Pacific and South China Sea, and
water that once fed into the Aral Sea. Willy-willies in Western Australia.
Dust blowing from the exposed lakebed Tropical cyclones originate and intensify
eventually degraded the soils, forcing further over warm tropical oceans. The conditions
water diversion efforts to revive them. On favourable for the formation and intensification
a larger scale, loss of the Aral Sea’s water of tropical storms are
influenced regional climate, making the winters y Large sea surface with a temperature
even colder and the summers much hotter. higher than 27° C.
Fifty years later, the lake is virtually gone.
y Presence of the Coriolis force.
Black Sea is often seen in the news in the context
of Russian affairs. Past 60 years, the oxygen- y Small variations in the vertical wind speed.
rich top layer of the Black Sea decreased from y A pre-existing weak low-pressure area or
140 metres to 90 metres deep, which amounts low-level-cyclonic circulation.
to an over 40 percent dip in habitable waters. y Upper divergence above the sea level
Two existing causes behind the shrinkage: an system.
abundance of nutrients, particularly algae that
Option (b) is correct: In the South Atlantic
led to great consumption of oxygen and global
and South-Eastern Pacific regions in tropical
warming. With warmer winters, there is a lower
latitudes, cyclones do not originate due to
volume of dense water created and this lowers
the Inter-tropical Convergence Zone, which
the oxygen content. Warmer waters also mean
seldom occurs.
the sea is now able to accumulate less dissolved
gas, including oxygen. The Black Sea has lost y The Intertropical Convergence Zone
more than a third of its habitable volume. The (ITZC), which shifts toward the north in

Oceanography-Explanation 154
unacademy.com | Download the Unacademy app
Give your feedback here: Link
our summer and toward the south in the
southern summer, usually stays at or north 8. Answer: (c)
of the equator in the Atlantic Ocean.
Option (c) is correct: About 71 per cent of the
y A broad convergence zone like this need to planetary water is found in the oceans. The
reach about 5 degrees of latitude away from remaining is held as freshwater in glaciers
the equator in order for the Coriolis Force to and icecaps, groundwater sources, lakes, soil
have sufficient intensity to organise a full- moisture, atmosphere, streams and within
fledged tropical cyclone, and the Atlantic life. Nearly 59 per cent of the water that falls
ITCZ almost never shift that far south, on land returns to the atmosphere through
while the Monsoon trough in the Indian and evaporation from over the oceans as well as
Australian basins, and the western South from other places.
Pacific, regularly do so.
The remainder runs off on the surface, infiltrates
The eastern South Pacific is just about as void into the ground or a part of it becomes glacier.
of activity as the South Atlantic, due largely to Renewable water on the earth is constant
the ITCZ also having a tendency to stay near while the demand is increasing tremendously.
or north of the equator and to climatologically This leads to water crises in different parts
cool sea surface temperatures. of the world — spatially and temporally. The
pollution of river waters has further aggravated
7. Answer: (d) the crisis.

The periodical rise and fall of the sea level, once


or twice a day, mainly due to the attraction of the 9. Answer: (c)
sun and the moon, is called a tide. Movement Ocean currents have a number of direct
of water caused by meteorological effects and indirect influences on human activities.
(winds and atmospheric pressure changes) are West coasts of the continents in tropical and
called surges. Surges are not regular like tides. subtropical latitudes (except close to the
y The moon’s gravitational pull to a great equator) are bordered by cool waters. Their
extent and to a lesser extent the sun’s average temperatures are relatively low with a
gravitational pull, are the major causes for narrow diurnal and annual ranges. There is fog,
the occurrence of tides. Another factor is but generally the areas are arid.
the centrifugal force of the Earth, which is y West coasts of the continents in the middle
the force that acts to counterbalance the and higher latitudes are bordered by warm
gravity. [Option (d) is correct] waters which cause a distinct marine
y Together, the gravitational pull and the climate. They are characterised by cool
centrifugal force are responsible for creating summers and relatively mild winters with a
the two major tidal bulges on the earth. On narrow annual range of temperatures.
the side of the earth facing the moon, a y Warm currents flow parallel to the east
tidal bulge occurs while on the opposite coasts of the continents in tropical and
side though the gravitational attraction of subtropical latitudes. This results in
the moon is less as it is farther away, the warm and rainy climates. These areas lie
centrifugal force causes tidal bulge on the in the western margins of the subtropical
other side. anticyclones.
y The ‘tide-generating’ force is the difference y The mixing of warm and cold currents helps
between these two forces, i.e. the to replenish the oxygen and favour the
gravitational attraction of the moon and the growth of planktons, the primary food for
centrifugal force.

155 Oceanography-Explanation
unacademy.com | Download the Unacademy app
Give your feedback here: Link
fish population. The best fishing grounds
of the world exist mainly in these mixing
zones. [Option (c) is correct]

10. Answer: (a)


Ocean acidification refers to a reduction in
the pH of the ocean over an extended period
of time, caused primarily by uptake of carbon
dioxide (CO2) from the atmosphere.
For more than 200 years, or since the industrial
revolution, the concentration of carbon dioxide
(CO2) in the atmosphere has increased due to 11. Answer: (b)
the burning of fossil fuels and land use change.
Oceanic currents describe the movement of
The ocean absorbs about 30 percent of the
water from one location to another. Currents
CO2 that is released in the atmosphere, and
are generally measured in metres per second
as levels of atmospheric CO2 increase, so do
or in knots (1 knot = 1.85 kilometres per hour
the levels in the ocean. When CO2 is absorbed
or 1.15 miles per hour). Oceanic currents are
by seawater, a series of chemical reactions
driven by the following factors:
occur resulting in the increased concentration
of hydrogen ions. This increase causes the Option (b) is correct:
seawater to become more acidic and causes The rise and fall of the tides: Tides create a
carbonate ions to be relatively less abundant. current in the oceans, which are strongest
Option (a) is correct: Ocean acidification near the shore, and in bays and estuaries along
is already impacting many ocean species, the coast. These are called “tidal currents.”
especially organisms like oysters and corals that Tidal currents change in a very regular pattern
make hard shells and skeletons by combining and can be predicted for future dates. In some
calcium and carbonate from seawater. locations, strong tidal currents can travel at
However, as ocean acidification increases, speeds of eight knots or more.
available carbonate ions (CO32-) bond with Wind: Winds drive currents that are at or near
excess hydrogen, resulting in fewer carbonate the ocean’s surface. Near coastal areas winds
ions available for calcifying organisms to build tend to drive currents on a localised scale
and maintain their shells, skeletons, and and can result in phenomena like coastal
other calcium carbonate structures. It further upwelling. On a more global scale, in the open
affects the growth and survival of calcareous ocean, winds drive currents that circulate
phytoplankton, the growth and survival of water for thousands of miles throughout the
coral reefs, and the survival of some animals ocean basins.
that have phytoplanktonic larvae.
Thermohaline circulation: This is a process
driven by density differences in water due
to temperature (thermo) and salinity (haline)
variations in different parts of the ocean.
Currents driven by thermohaline circulation
occur at both deep and shallow ocean levels
and move much slower than tidal or surface
currents.
The rotation of the Earth: The rotation of the

Oceanography-Explanation 156
unacademy.com | Download the Unacademy app
Give your feedback here: Link
Earth causes Coriolis force which affects the of density and salinity in ocean water
direction of movement of water and leads to motivates denser waters to sink and move as
formation of ocean currents. undercurrents, whereas lighter waters move
Difference of density and salinity: Difference towards the denser water as surface currents.

157 Oceanography-Explanation
unacademy.com | Download the Unacademy app
Give your feedback here: Link
5 World Climatic Regions

1. Consider the following States: (2015) 3. Thorns instead of leaves


1. Arunachal Pradesh Select the correct answer using the code
2. Himachal Pradesh given below:

3. Mizoram (a) 1 and 2 only

In which of the above States do 'Tropical (b) 2 only


Wet Evergreen Forests' occur? (c) 1 and 3 only
(a) 1 only (d) 1, 2 and 3
(b) 2 and 3 only
(c) 1 and 3 only 5. “Climate is extreme, rainfall is scanty, and
(d) 1, 2 and 3 the people used to be nomadic herders.”
The above statement best describes which
of the following regions? (2013)
2. "Each day is more or less the same, the (a) African Savannah
morning is clear and bright with a sea
breeze; as the Sun climbs high in the sky, (b) Central Asian Steppe
heat mounts up, dark clouds form, then rain (c) North American Prairie
comes with thunder and lightning. But the (d) Siberian Tundra
rain is soon over." Which of the following
regions is described in the above passage?
(2015) 6. Which of the following is/are unique
(a) Savannah characteristics / characteristics of
equatorial forests? (2013)
(b) Equatorial
1. Presence of tall, closely set trees with
(c) Monsoon crowns forming a continuous canopy
(d) Mediterranean 2. Coexistence of a large number of
species
3. The seasonal reversal of winds is the 3. Presence of numerous varieties of
typical characteristic of: (2014) epiphytes
(a) Equatorial climate Select the correct answer using the code
(b) Mediterranean climate given below:

(c) Monsoon climate (a) 1 only

(d) All of the above climates (b) 2 and 3 only


(c) 1 and 3 only

4. Which of the following leaf modifications (d) 1, 2 and 3


occurs/occur in desert areas to inhibit
water loss? (2013) 7. Which one of the following is the
1. Hard and waxy leaves characteristic climate of the Tropical
2. Tiny leaves or no leaves Savannah Region? (2012)

World Climatic Regions 158


unacademy.com | Download the Unacademy app
Give your feedback here: Link
(a) Rainfall throughout the year pressure cells.
(b) Rainfall in winter only 2. It is under the influence of warm ocean
(c) An extremely short dry season currents.

(d) A definite dry and wet season Which of the statements given above is/are
correct in this context?
(a) 1 only
8. What could be the main reason/reasons
for the formation of African and Eurasian (b) 2 only
desert belts? (2011) (c) Both 1 and 2
1. It is located in the subtropical high- (d) Neither 1 nor 2

159 World Climatic Regions


unacademy.com | Download the Unacademy app
Give your feedback here: Link
World Climatic Regions-
5 Explanation
1. Answer: (c) temperature is negligible. The maximum
Tropical evergreen forests are well stratified, temperature on any day is around 30°C,
with layers closer to the ground and are while the minimum temperature is around
covered with shrubs and creepers, with short, 20°C. Tropical evergreen forests with dense
structured trees followed by a tall variety of canopy cover and large biodiversity are
trees. In these forests, trees reach great heights found in this climate.
up to 60 m or above. There is no definite time y Twice a year, during the spring and autumn
for trees to shed their leaves, flowering, and equinoxes, the sun passes directly over the
fruition. As such, these forests appear green all Equator. Even during the rest of the year,
the year-round. Species found in these forests equatorial regions often experience a hot
include rosewood, mahogany, aini, ebony, etc. climate with little seasonal variation.
Option (c) is correct: The Tropical Wet Evergreen
Forests are found in the western slope of the 3. Answer: (c)
Western Ghats, hills of the north-eastern
region (Assam, Arunachal Pradesh, Meghalaya, Option (c) is correct: The seasonal reversal of
Mizoram, Nagaland, Tripura, West Bengal) and winds is the typical characteristic of Monsoon
the Andaman and Nicobar Islands. They are climate. Monsoon climate is identified with a
found in warm and humid areas with annual major wind system that seasonally reverses
precipitation of over 200 cm and mean annual its direction—such as one that blows for
temperature above 22oC. approximately six months from the northeast
and six months from the southwest. It can
cause heavy rainfall during the summer and
2. Answer: (b) dry spells in the winter.
Option (b) is correct: In the equatorial region, y Equatorial climate: It is a hot-wet climate,
the annual rainfall is high as it rains almost it remains uniform throughout the year. It is
every day. The temperatures are constant all found between 5° and 10° north and south
year round – the temperature range is usually of the equator. Its greatest extent is found
only a few degrees. There are no seasons. Each in the lowlands of the Amazon, the Congo,
day is more or less the same, the morning is Malaysia, and the East Indies.
clear and bright with a sea breeze; as the Sun y Mediterranean climate: It is characterised
climbs high in the sky, heat mounts up, dark by hot, dry summers and cool, wet winters.
clouds form, then rain comes with thunder and It is located between about 30° and 45°
lightning. But the rain is soon over. latitude north and south of the Equator and
Additional Information: on the western sides of the continents.
y Tropical wet climate is found near the
equator. The major areas are the Amazon 4. Answer: (d)
Basin in South America, western equatorial
Option (d) is correct: Desert plants lose very
Africa and the islands of the East Indies.
little water through transpiration. The leaves
Significant amount of rainfall occurs
in desert plants are either absent, very small,
in every month of the year as thunder
or they are present in the shape of spines.
showers in the afternoon. The temperature
This helps in reducing loss of water from the
is uniformly high, and the annual range of
leaves through transpiration. Plants growing

World Climatic Regions-Explanation 160


unacademy.com | Download the Unacademy app
Give your feedback here: Link
in extreme dry conditions throughout the year crowns of other trees. Hardwood trees like
adopted thorns instead of leaves to prevent rosewood, ebony, mahogany are common
loss of water. Photosynthesis in these plants here. (Statement 1 is correct)
is usually carried out by the stems. The stem y They are centres of biodiversity, holding an
is also covered with a thick waxy layer, which estimated half of the world’s plants and
helps to retain water. Most desert plants animals. The coexistence of a large number
have roots that go very deep into the soil for of species is an important characteristic of
absorbing water. these forests. (Statement 2 is correct)
y In equatorial forests, numerous varieties
5. Answer: (b) of epiphytes were present. (Statement 3 is
Option (b) is correct: A steppe is a dry, grassy correct)
plain. Steppes occur in temperate climates,
which lie between the tropics and polar regions. 7. Answer: (d)
Temperate regions have distinct seasonal
temperature changes, with cold winters and Option (d) is correct: By distinct wet and dry
warm summers. Climate is extreme, rainfall seasons the Savanna climate is characterised
is scanty, and the people used to be nomadic by high temperature throughout the year
herders is the best description of Steppe. The (ranging between 24°C and 27°C), and
best-known pastoral and hunter-gatherer tribe abundant insolation. In any month of the year
in history were the Mongols. They inhabited the Temperature does not fall below 20°C. Savanna
grasslands (steppes) of Central Asia. climate is similar to equatorial climate as
regards temperature but the annual range of
The largest temperate grassland in the world is temperature ranging between 3°C and 8°C is
the Eurasian steppe, extending from Hungary greater than in the equatorial climate.
to China. The Eurasian steppe has historically
been one of the most important routes for
travel and trade. The flat expanse provides an 8. Answer: (a)
ideal route between Asia and Europe. Statement 1 is correct: The desert occurs in
two broad belts viz. at 20-30° north and south
6. Answer: (d) of the Equator along the Tropics of Cancer and
Capricorn. At the equator hot air would rise
Equatorial forests are also called tropical and spread north and south before cooling and
rainforests. These thick forests occur in the it would condense and release its moisture
regions near the equator and close to the over the tropical zones leading to development
tropics. These regions are hot and receive of an equatorial zone of low atmospheric
heavy rainfall throughout the year. As there is pressure. The two tropical zones are at high
no particular dry season, the trees do not shed pressure and nearer to the poles are two low-
their leaves altogether. This is the reason they pressure belts of cold. As the denser air sinks
are called evergreen. towards the ground to the two subtropical
y The thick canopies of the closely spaced high-pressure belts, wind is created which is
trees do not allow the sunlight to penetrate hot and completely lacking in moisture. This
inside the forest even in the daytime. A blows across the Middle East, the Sahara, and
mature lowland tropical forest consists of North America.
several layers. The top layer of vegetation Statement 2 is not correct: Eurasian Belt is
consists of scattered tall trees which tower related to cold currents not warm current.
above a closed canopy layer formed by the

161 World Climatic Regions-Explanation


unacademy.com | Download the Unacademy app
Give your feedback here: Link
6 Human and Economic Geography

1. Consider the following statements : (2023) (a) Argentina


1. India has more arable area than China. (b) Botswana
2. The proportion of irrigated area is more (c) the Democratic Republic of the Congo
in India as compared to China. (d) Kazakhstan
3. The average productivity per hectare in
Indian agriculture is higher than that in
China. 3. Consider the following specific stages of
demographic transition associated with
How many of the above statements are economic development: (2012)
correct?
1. Low birth rate with low death rate
(a) Only one
2. High birth rate with high death rate
(b) Only two
3. High birth rate with low death rate
(c) All three
Select the correct answer using the code
(d) None given below:
(a) 1, 2, 3
2. About three-fourths of world's cobalt, (b) 2, 1, 3
a metal required for the manufacture of
batteries for electric motor vehicles, is (c) 2, 3, 1
produced by (2023) (d) 3, 2, 1

Human and Economic Geography 162


unacademy.com | Download the Unacademy app
Give your feedback here: Link
Human and Economic Geography-
6 Explanation
1. Answer: (b) demand for cobalt for use in batteries will grow
Statement 1 is correct: India and China,the fourfold in 2030 as a result of this electric
world’s most populous countries, have limited vehicle boom.
arable land — China has about 120 million
hectares (mha) and India 156 million hectares 3. Answer: (c)
(mha).
Frank W Notestein in his demographic
Statement 2 is correct: China’s irrigation cover transition theory mentions that all countries
is 41 per cent of the country’s cultivated area, pass through stages of demographic transition,
while India’s irrigation cover is 48 per cent. which is accompanied by industrialization and
Statement 3 is not correct: As a result of this economic development. Notestein gave the
irrigation, China’s total sown area is 166 m ha following stages of demographic transition.
compared to India’s gross cropped area of Option (c) is correct: With the economic
198 m ha. Even with much lesser land under development, the demographic transition
cultivation, China produces agricultural output follows:
valued at $1,367 billion—more than three times
that of India’s $407 billion. China’s productivity High birth rate with high death rate: In this
in most crops is 50 to 100% higher than India’s. stage birth and death rates are both high.
Population growth is slow and fluctuating.
Birth rate is high as a result of lack of family
2. Answer: (C) planning, high infant mortality rate, children
Half of the world’s available cobalt supply is in as economic assets, etc. Death rate is high
the Democratic Republic of the Congo. Australia because of high levels of disease, famine, and
is the next cobalt-rich country, followed by lack of healthcare. Then follows,
Indonesia and Cuba.15 to 30 percent of the High birth rate with low death rate: In this
Congolese cobalt is produced by artisanal and stage birth rate remains high, death rate
small-scale mining (ASM). falls. Population begins to rise rapidly. Death
Cobalt is an essential mineral used for batteries rate falls as a result of improved healthcare,
in electric cars, computers, and cell phones. improved hygiene sanitation, etc. Then follows,
Demand for cobalt is increasing as more Low birth rate with low death rate: In this
electric cars are sold, particularly in Europe, stage of demographic transition, a low birth
where governments are encouraging the rate and low death rate lead to a stationary
sales with generous environmental bonuses. or declining population. It is called a stage of
According to recent projections by the World stationary population. Birth and death rates
Economic Forum’s Global Battery Alliance, the are both low.

163 Human and Economic Geography-Explanation


unacademy.com | Download the Unacademy app
Give your feedback here: Link
7 World Map

1. Which one of the following is a part of the (d) All four pairs
Congo Basin? (2023)
(a) Cameroon 4. Consider the following countries: (2022)
(b) Nigeria 1. Azerbaijan
(c) South Sudan 2. Kyrgyzstan
(d) Uganda 3. Tajikistan
4. Turkmenistan
2. Consider the following countries: (2023) 5. Uzbekistan
1. Bulgaria Which of the above have borders with
2. Czech Republic Afghanistan?
3. Hungary (a) 1, 2 and 5 only
4. Latvia (b) 1, 2, 3 and 4 only
5. Lithuania (c) 3, 4 and 5 only
6. Romania (d) 1, 2, 3, 4 and 5
How many of the above-mentioned
countries share a land border with Ukraine? 5. The term “Levant” often heard in the
(a) Only two news roughly corresponds to which of the
(b) Only three following regions? (2022)

(c) Only four (a) Region along the eastern Mediterranean


shores
(d) Only five
(b)
Region along North African shores
stretching from Egypt to Morocco
3. Consider the following pairs: (2022) (c) Region along Persian Gulf and Horn of
Region often Country Africa
mentioned in the (d) The entire coastal areas of Mediterranean
news Sea
1. Anatolia Turkey
2. Amhara Ethiopia 6. Which one of the following lakes of West
3. Cabo Delgado Spain Africa has become dry and turned into a
4. Catalonia Italy desert? (2022)

How many pairs given above are correctly (a) Lake Victoria
matched? (b) Lake Faguibine
(a) Only one pair (c) Lake Oguta
(b) Only two pairs (d) Lake Volta
(c) Only three pairs

World Map 164


unacademy.com | Download the Unacademy app
Give your feedback here: Link
7. Consider the following pairs: (2019) (a) 1 and 2 only
Sea Bordering country (b) 1 and 4 only
1. Adriatic Sea Albania (c) 2 and 3 only
2. Black Sea Croatia (d) 3 and 4 only
3. Caspian Sea Kazakhstan
4. Mediterranean Sea Morocco 10. The Mediterranean Sea is a border of which
of the following countries? (2017)
5. Red Sea Syria
1. Jordan
Which of the pairs given above are correctly
matched? 2. Iraq
(a) 1, 2 and 4 only 3. Lebanon
(b) 1, 3 and 4 only 4. Syria
(c) 2 and 5 only Select the correct answer using the code
given below:
(d) 1, 2, 3, 4 and 5
(a) 1, 2 and 3
(b) 2 and 3 only
8. Consider the following pairs: (2018)
(c) 3 and 4 only
Regions sometimes Country
(d) 1, 3 and 4 only
mentioned in news
1. Catalonia Spain
2. Crimea Hungary 11. Which one of the following countries of
South-West Asia does not open out to the
3. Mindanao Philippines
Mediterranean Sea? (2015)
4. Oromia Nigeria
(a) Syria
Which of the pairs given above are correctly
(b) Jordan
matched?
(c) Lebanon
(a) 1, 2 and 3
(d) Israel
(b) 3 and 4 only
(c) 1 and 3 only
12. Turkey is located between (2014)
(d) 2 and 4 only
(a) Black Sea and Caspian Sea
(b) Black Sea and Mediterranean Sea
9. Consider the following pairs: (2018)
(c) Gulf of Suez and Mediterranean Sea
Towns sometimes Country
(d) Gulf of Aqaba and Dead Sea
mentioned in news
1. Aleppo Syria
2. Kirkuk Yemen 13. What is the correct sequence of occurrence
of the following cities in South-East Asia as
3. Mosul Palestine
one proceeds from south to north? (2014)
4. Mazar-i-sharif Afghanistan
1. Bangkok
Which of the pairs given above are correctly
matched?

165 World Map


unacademy.com | Download the Unacademy app
Give your feedback here: Link
2. Hanoi (c) Guiana South-Western
3. Jakarta Highlands Africa
4. Singapore (d) Okavango Basin Patagonia
Select the correct answer using the code
given below: 15. Between India and East Asia, the navigation
(a) 4-2-1-3 time and distance can be greatly reduced
by which of the following? (2011)
(b) 3-2-4-1
1. Deepening the Malacca straits between
(c) 3-4-1-2
Malaysia and Indonesia.
(d) 4-3-2-1
2. Opening a new canal across the Kra
Isthmus between the Gulf of Siam and
14. Which one of the following pairs is correctly Andaman Sea.
matched? (2013) Which of the statements given above is/are
Geographical Region correct?
Feature (a) 1 only
(a) Abyssinian Arabia (b) 2 only
Plateau
(c) Both 1 and 2
(b) Atlas Mountains North-Western
(d) Neither 1 nor 2
Africa

World Map 166


unacademy.com | Download the Unacademy app
Give your feedback here: Link
7 World Map-Explanation

1. Answer: (a) 2. Amhara—Ethiopia (pair 2 is correctly


The Congo Basin is the sedimentary basin of matched)
the Congo River. The Congo Basin is located 3. Cabo Delgado—Mozambique (pair 3 is
in Central Africa, in a region known as west not correctly matched)
equatorial Africa. The Congo Basin region 4. Catalonia—Spain (pair 4 is not correctly
is sometimes known simply as the Congo. matched)
It contains some of the largest tropical
rainforests in the world and is an important
source of water used in agriculture and energy 4. Answer: (c)
generation. Option (c) is correct: Afghanistan shares
Countries wholly or partially in the Congo borders with Iran, Pakistan, Turkmenistan,
region: Angola, Gabon, Burundi, Cameroon, Tajikistan, Uzbekistan, China and India.
Central African Republic, Democratic Republic
of Congo, Republic of the Congo, Rwanda,
Tanzania, Zambia.

2. Answer: (a)
With an area of 600,000 km², Ukraine is
the second-largest country in Europe after
European Russia, twice the size of Italy or
slightly smaller than the US state of Texas.
Ukraine borders Belarus, Hungary, Moldova,
Poland, Romania, Russia, and Slovakia.

5. Answer: (a)
Option (a) is correct: From the beginning of
civilization, the Levant, the coastal region of
the eastern Mediterranean from Syria in the
north to Egypt in the south, was the crossroads
of various peoples and cultures. From this
important trade region, fundamental social and
economic changes began spreading across the
3. Answer: (b) Middle East and the Mediterranean, leaving
1. Anatolia—Turkey (pair 1 is correctly behind a rich heritage of unique material
matched) remains.

167 World Map-Explanation


unacademy.com | Download the Unacademy app
Give your feedback here: Link
Russia and Georgia to the East, Turkey to the
South, and Bulgaria and Romania to the West.
Pair 3 is correctly matched: Caspian Sea is
located between south-eastern Europe and
Western Asia. It is the largest inland body
of water that can only be accessed through
Russia’s Volga River and the canals connecting
it to the Black Sea, the Baltic Sea, and the Sea
of Azov. It is supplied by freshwater sources
and has no salt-water connection to the
open seas of the world. The bordering states
are Azerbaijan, Iran, Kazakhstan, Russia, and
Turkmenistan.
Pair 4 is correctly matched: Mediterranean
Sea is an intercontinental sea placed between
Western Asia, North Africa, and Europe. 21
countries share the coastline with it. These
are- Albania, Algeria, Bosnia and Herzegovina,
Croatia, Cyprus, Egypt, France, Greece,
6. Answer: (b)
Israel, Italy, Lebanon, Libya, Malta, Monaco,
Option (b) is correct: The Lake Faguibine Montenegro, Morocco, Slovenia, Spain, Syria,
System, four interlinked lakes 80 km west of Tunisia, and Turkey.
Timbuktu, was historically one of Mali’s most
Pair 5 is not correctly matched: Red Sea
fertile areas. But over seven years, droughts in
is located between North-east Africa and
the 1970s dried up the lakes.
Western Asia. The six countries bordering it are
Then sand filled the channels connecting the Saudi Arabia, Yemen, Egypt, Sudan, Eritrea, and
lakes to the River Niger, with the result that Djibouti.
when rain finally returned the water could no
longer reach the lakes. The region’s prosperity
evaporated along with the water. 8. Answer: (c)
In the past, during prolonged rainfall in the Pair 1 is correctly matched: Catalonia is an
Fouta Djallon highlands in Guinea, the river autonomous community in the north-eastern
flooded and forced water to flow through two corner of Spain. It is one of Spain’s wealthiest
channels into the lake. regions. It consists of four provinces, such as
Barcelona, Girona, Lleida, and Tarragona.
Pair 2 is not correctly matched: Crimea is a
7. Answer: (b)
peninsula located on the northern coast of
Pair 1 is correctly matched: Adriatic Sea is a the Black Sea. In 2014, Russia seized Crimea
water body that acts as a divide between the from Ukraine in an illegal move that violated
Italian Peninsula and the Balkan Peninsula. the territorial integrity of the former Soviet
Albania, Croatia, Bosnia and Herzegovina, Italy, republic and sparked a war that has displaced
Slovenia, and Montenegro share borders with nearly 2 million people and destroyed the
the Adriatic Sea. country’s infrastructure.
Pair 2 is not correctly matched: Black Sea is Pair 3 is correctly matched: Mindanao is the
located between Europe and Asia. Six countries second-largest island in the Philippines, after
border the Black Sea, Ukraine to the North, Luzon.

World Map-Explanation 168


unacademy.com | Download the Unacademy app
Give your feedback here: Link
Pair 4 is not correctly matched: The Oromo Croatia, Cyprus, Egypt, France, Greece,
people are the native inhabitants of Ethiopia, Israel, Italy, Lebanon, Libya, Malta, Monaco,
Eastern Africa. Their population is estimated to Montenegro, Morocco, Slovenia, Spain, Syria,
be 55 million, which makes it the largest ethnic Tunisia, and Turkey. Jordan and Iraq do not
group in Eastern Africa. share a coastline with the Mediterranean Sea.

9. Answer: (b) 11. Answer: (b)


Pair 1 and pair 4 are correctly matched: Option (b) is correct: 21 countries share a
Aleppo is in Syria. Mazar-i-sharif is located in coastline with the Mediterranean Sea. They
Afghanistan. are Albania, Algeria, Bosnia and Herzegovina,
Pair 2 and pair 4 are not correctly matched: Croatia, Cyprus, Egypt, France, Greece,
Kirkuk and Mosul, both are in Iraq. All these Israel, Italy, Lebanon, Libya, Malta, Monaco,
places appeared in news in context of war and Montenegro, Morocco, Slovenia, Spain, Syria,
violence. Terrorism and Ethic conflict are the Tunisia, and Turkey. Jordan does not share a
main reasons of violence. The ethnic dimension border with the Mediterranean Sea.
to social relations can be distinguished from
dimensions based on neighbourliness, national
origin, race, and religion etc., but ethnic
conflicts are not a special class of conflicts.
The significance of a shared ethnic origin varies
with the location and scale of social relations.

12. Answer: (b)


Option (b) is correct: Turkey is located between
Black Sea and Mediterranean Sea. Turkey
occupies a unique geographic position, lying
partly in Asia and partly in Europe. Throughout
10. Answer: (c) its history it has acted as both a barrier and a
bridge between the two continents.

Option (c) is correct: 21 countries share a


coastline with the Mediterranean Sea. These
are Albania, Algeria, Bosnia and Herzegovina,

169 World Map-Explanation


unacademy.com | Download the Unacademy app
Give your feedback here: Link
13. Answer: (c) a series of mountain ranges in north-western
Option (c) is correct: If one proceeds from Africa, generally running southwest to northeast
south to north in South-East Asia, the correct to form the geologic backbone of the countries
sequence is Jakarta, Singapore, Bangkok, of the Maghrib (the western region of the Arab
Hanoi. world)—Morocco, Algeria, and Tunisia.

y Jakarta, earlier named Batavia and Djakarta, Pair (c) is not correctly matched: Guiana
is the largest city and capital of Indonesia. Highlands, plateau and low-mountain region
It lies on the northwest coast of Java at the of South America located north of the Amazon
mouth of the Ciliwung (Liwung River), on and south of the Orinoco River. They cover the
Jakarta Bay, Java Sea. southern half of Venezuela, all of the Guianas
except for the low Atlantic coastal plain, the
y Singapore, a city-state located at the
northern part of Brazil, and a portion of south-
southern tip of the Malay Peninsula, north
eastern Colombia.
of the Equator. It consists of the diamond-
shaped Singapore Island and some 60 Pair (d) is not correctly matched: Okavango
small islets Basin is an endorheic basin, shared between
Angola, Namibia and Botswana. It comprises
y Bangkok is the capital and most populous
permanent marshlands and seasonally flooded
city of Thailand. It is located on the delta of
plains. It is one of the very few major interior
the Chao Phraya River.
delta systems that do not flow into the sea or
y Hanoi is the capital of Vietnam. It is situated ocean, with a wetland system that is almost
in northern Vietnam on the western bank intact.
of the Red River.

15. Answer: (b)


Statement 1 is not correct: Malacca strait is
“linked” to its economic importance rather
than “time of navigation and distance”. The
deepening of the strait would certainly help in
Increasing the volume of the business because
ships of larger sizes can pass through it, there
is no significance of distance and navigation.
Statement 2 is correct: Kra isthmus canal,
also known as Kra canal, a proposed canal
that would connect the Gulf of Thailand with
the Andaman Sea across the Kra Isthmus in
14. Answer: (b) southern Thailand. It is envisaged that such
Pair (a) is not correctly matched: Abyssinian a canal would improve transportation in the
is a plateau in Ethiopia. It is located at an region, similar to the Panama Canal and Suez
elevation of 1,388 metres above sea level. Canal. The canal would provide an alternative
Hundreds of the world’s only grass-eating to transit through the Straits of Malacca.
monkeys thrive on this plateau in the central Opening Kra isthmus canal will greatly reduce
Ethiopian Highlands. navigation time and distance between India
Pair (b) is correctly matched: Atlas Mountains, and East-Asia.

World Map-Explanation 170


unacademy.com | Download the Unacademy app
Give your feedback here: Link
6 INDIAN
GEOGRAPHY
1 Physiography of India

1. Which one of the following is the best given below:


example of repeated falls in sea level, giving (a) 1 and 2 only
rise to present-day extensive marshland?
(2023) (b) 3 and 4 only

(a) Bhitarkanika Mangroves (c) 1, 3 and 4 only

(b) Marakkanam Salt Pans (d) 1, 2, 3 and 4

(c) Naupada Swamp


(d) Rann of Kutch 5. Consider the following statements: (2017)
1. In India, the Himalayas are spread over
five states only.
2. Consider the following pairs: (2022)
2. The Western Ghats are spread over five
Peak Mountains states only.
1. Namcha Barwa Garhwal Himalaya 3. Pulicat Lake is spread over two States
2. Nanda Devi Kumaon Himalaya only.
3. Nokrek Sikkim Himalaya Which of the statements given above is/are
Which of the pairs given above is/are correct?
correctly matched? (a) 1 and 2 only
(a) 1 and 2 (b) 3 only
(b) 2 only (c) 2 and 3 only
(c) 1 and 3 (d) 1 and 3 only
(d) 3 only
6. Which of the following is geographically
3. Siachen Glacier is situated in the: (2020) closest to Great Nicobar? (2017)

(a) East of Aksai Chin (a) Sumatra

(b) East of Leh (b) Borneo

(c) North of Gilgit (c) Java

(d) North of Nubra Valley (d) Sri Lanka

4. Which of the following Protected Areas are 7. From the ecological point of view, which
located in the Cauvery basin? (2020) one of the following assumes importance
in being a good link between the Eastern
1. Nagarhole National Park Ghats and the Western Ghats? (2017)
2. Papikonda National Park (a) Sathyamangalam Tiger Reserve
3. Sathyamangalam Tiger Reserve (b) Nallamala Forest
4. Wayanad Wildlife Sanctuary (c) Nagarhole National Park
Select the correct answer using the code (d) Seshachalam Biosphere Reserve

172 Physiography of India


unacademy.com | Download the Unacademy app
Give your feedback here: Link
8. Which one of the following pairs of States (a) 1 only
of India indicates the easternmost and (b) 2 and 3 only
westernmost State? (2015)
(c) 1 and 3 only
(a) Assam and Rajasthan
(d) None
(b) Arunachal Pradesh and Rajasthan
(c) Assam and Gujarat
11. When you travel in Himalayas, you will see
(d) Arunachal Pradesh and Gujarat the following: (2012)
1. Deep gorges
9. Consider the following pairs: (2014) 2. U-turn river courses
Hills Region 3. Parallel mountain ranges
1. Cardamom Hills Coromandel Coast 4. Steep gradients causing land sliding
2. Kaimur Hills Konkan Coast Which of the above can be said to be the
3. Mahadeo Hills Central India evidence for Himalayas being young fold
4. Mikir Hills North-East India mountains?

Which of the above pairs are correctly (a) 1 and 2 only


matched? (b) 1, 2 and 4 only
(a) 1 and 2 (c) 3 and 4 only
(b) 2 and 3 (d) 1, 2, 3 and 4
(c) 3 and 4
(d) 2 and 4 12. The Brahmaputra, Irrawaddy and Mekong
Rivers originate in Tibet's narrow and
parallel mountain ranges in their upper
10. The Narmada River flows to the west, while
reaches. Of these rivers, Brahmaputra
most other large peninsular rivers flow to
makes a “U” turn in its course to flow into
the east. Why? (2013)
India. This “U” turn is due to (2011)
1. It occupies a linear rift valley.
(a) Uplift of folded Himalayan series
2. It flows between the Vindhyas and the
(b) Syntaxial bending of geologically young
Satpuras.
Himalayas
3. The land slopes to the west from
(c) Geo-tectonic disturbance in the tertiary
Central India.
folded mountain chains
Select the correct answer using the code
(d) Both (a) and (b) above
given below:

Physiography of India 173


unacademy.com | Download the Unacademy app
Give your feedback here: Link
1 Physiography of India-Explanation

1. Answer: (d) 2. Answer: (b)


A. Bhitarkanika Mangroves is a mangrove Pair 1 is not correct: Assam Himalayas, eastern
wetland in Odisha, India, covering an area of section of the Great Himalayas, extending
650 km (400 mi) in the Brahmani River and eastward across Sikkim state (India) and
Baitarani River deltas. Bhutan, into northern Assam and Arunachal
B. The Marakkanam salt pans are spread over Pradesh states (India), and along the border
an area of 4,000 acres and is one of the largest with the Tibet Autonomous Region (China).
producers of salt in Tamil Nadu. These salt Important peaks include Kula, Chomo, and
pans support the livelihood of around 1,000 Kangto; the highest is Namjagbarwa (Namcha
workers every day during peak season, starting Barwa; 25,445 feet [7,756 metres]) in Tibet.
January each year. Pair 2 is correct: Kumaun Himalayas, west-
C. A nematode infestation has led to mass central section of the Himalayas in northern
mortality of spot-billed pelicans (Pelicanus India. The range, comprising part of the Siwalik
philippensis) at Telineelapuram Important Bird Range in the south and part of the Great
Area (IBA) in Naupada swamp of Srikakulam Himalayas in the north, lies largely within the
district in Andhra Pradesh. state of Uttarakhand, northwest of Nepal. It
rises to 25,646 feet (7,817 metres) at Nanda
D.Rann of Kutch was once a shallow part of Devi, the range’s highest peak, and to 25,446
the Arabian Sea, until a geological shift closed feet (7,756 metres) at Kamet, near the Chinese
off the connection with the sea. The region border.
became a seasonal marshy salt desert over the
years. Pair 3 is not correct: The Nokrek is the highest
peak of the Garo hills, rising up 1,412 metres.
which forms part of the Meghalaya Plateau.

174 Physiography of India-Explanation


unacademy.com | Download the Unacademy app
Give your feedback here: Link
3. Answer: (d) and elephants to other areas of the Western
The Siachen Glacier is in the Eastern Karakoram Ghats through Brahmagiri wildlife sanctuary
range in the Himalayas. It is situated to the and to the Eastern Ghats through Bandipur
North of Nubra Valley. It is the Second-Longest Tiger reserve. It is located in the Cauvery basin.
glacier in the World’s Non-Polar areas. It is part Statement 2 is not correct: Papikonda National
of Ladakh which has now been converted into Park is located in the East Godavari and West
a Union Territory. Siachen Glacier is the world’s Godavari districts of Andhra Pradesh and
highest battlefield. Khammam district of Telangana. The Park lies
Option (d) is correct: The Nubra Valley acts in the River Godavari basin.
as the gateway to the strategically important Statement 3 is correct: The Sathyamangalam
Siachen Glacier and Karakoram Pass. The Tiger Reserve is located in Kerala. It is located
Karakoram Pass on the northwest side in the Eastern and Western Ghats. Sandalwood
connects the valley with Xinjiang, China. The trees are a common find here. It lies in the
area was once part of the ancient Himalayan Cauvery basin.
trade path, the Silk Route that connected Statement 4 is correct: Wayanad Wildlife
China to the Middle East. The Nubra River is a Sanctuary is located in Kerala and was set
river in the Nubra Valley of Ladakh in India. It up in 1973. These are located in the southern
is a tributary of the Shyok River (a part of the trenches of the famous Western Ghats, ranked
Indus River system) and originates from the 8th in the list of the World’s Biodiversity
Siachen Glacier. Hotspots. The protected areas here share their
Siachen glacier
r boundaries with the protected area network
Hush
Th

ie
ac
Keres gl S
alle

im
o hy
ok of Nagarhole and Bandipur Tiger Reserves of
e

R
Ind

CHINA Karnataka in the northeast and Mudumalai


us

Khapalu
Tiger Reserve of Tamil Nadu in the southeast.
Nu
bra

PAKISTAN It lies in the Cauvery basin.


Sh
yo
k
Diskit Satti 5. Answer: (b)
INDIA
Chang-Chen-Mo
The Himalayas, geologically young and
Indu
s structurally fold mountains, stretch over the
Lo
ng

northern borders of India. These mountain


Ko
nm
a

4. Answer: (c) ranges run in a west-east direction from the


Indus to the Brahmaputra. The Himalayas
Cauvery is an easterly flowing river of Peninsular
represent the loftiest and one of the most
India that runs across three of the southern
rugged mountain barriers in the world.
Indian states i.e., Karnataka, Tamil Nadu,
Kerala, and a Union Territory of Puducherry. Statement 1 is not correct: The Indian
The point of origin of Cauvery, Talakaveri is in Himalayan Region is spread across 12 Indian
the Brahmagiri ranges of the Western Ghats at States (namely, Jammu & Kashmir, Uttarakhand,
an elevation of 1341m. Himachal Pradesh, Arunachal Pradesh, Manipur,
Meghalaya, Mizoram, Nagaland, Sikkim, Tripura,
Statement 1 is correct: Nagarhole Tiger
two districts of Assam, namely Dima Hasao and
Reserve, previously known as Rajiv Gandhi
Karbi Anglong and Darjeeling and Kalimpong
(Nagarhole) National Park, was named after the
in West Bengal) stretching across a length of
river ‘Nagarhole’ which means ‘Serpent River’ in
2500 km and width of 250 to 300 km. Some
the Kannada language. Nagarhole tiger reserve
extensions of Shiwaliks are also present in
forms a critical connecting habitat for tigers
Punjab and Haryana.

Physiography of India-Explanation 175


unacademy.com | Download the Unacademy app
Give your feedback here: Link
Statement 2 is not correct: The Western Ghats Statement 3 is correct: Pulicat Lake is the
and the Eastern Ghats mark the western and second-largest brackish water lake in the
the eastern edges of the Deccan Plateau, country lies at the border of the states of Andhra
respectively. The Western Ghats lie parallel Pradesh and Tamil Nadu on the Barricade Island
to the western coast. They are continuous of Sriharikota. Also called Pazhaverkadu, the
and can be crossed through passes only. The lake is popular as a flamingo-watching site and
Western Ghats traverse the States of Kerala, for water activities. It is home to the Satish
Tamil Nadu, Karnataka, Goa, Maharashtra, and Dhawan Space Centre.
Gujarat.
Elimination technique: The Indian
Himalayan Region is spread from J&K to
north-east India, across 12 Indian States
not 5. The Western Ghats traverse the
States of Kerala, Tamil Nadu, Karnataka,
Goa, Maharashtra, and Gujarat.

6. Answer: (a)
The Andaman and Nicobar Islands are the
largest archipelago system in the Bay of
Bengal, consisting of 306 islands and 206
rocks and rocky outcrops and covering a total
area of about 8200 sq km. Only 38 of these

176 Physiography of India-Explanation


unacademy.com | Download the Unacademy app
Give your feedback here: Link
islands are inhabited, 11 in the Andaman group Wildlife Sanctuary, Sigur Plateau, Mudumalai
and 13 in the Nicobars. This large archipelago National Park and Bandipur National Park.
is separated from mainland India by about Additional Information:
1000 km. The nearest landmass in the north is
Myanmar, roughly 280 km from Landfall Island, y Nallamala Forest: It is one of the largest
the northernmost island in the group. undisturbed stretches of forest in South
India. It is located in Nallamala Hill, which
Option (a) is correct: The closest landmass to is a part of the Eastern Ghats. The forest
Great Nicobar in the south is Sumatra, which is has a good tiger population, and a part of
located at a distance of 145 km. the forest belongs to the Nagarjunsagar-
Srisailam Tiger Reserve.
y Nagarhole National Park: It is also known
as Rajiv Gandhi National Park. The word
‘Nagarahole’ means ‘Cobra-river’ in Kannada.
It is spread between the Kodagu and Mysore
districts of Karnataka. Nagarahole River
flows through the park and gradually joins
the Kabini River, which also is a boundary
between Nagarahole and Bandipur.
Nagarhole has the highest congregation of
Additional Information: the Asian elephant in the world Nagarhole
y Indira Point, the southernmost point of has the highest congregation of the Asian
India, is the southern point of Great Nicobar elephant in the world.
Island. y Seshachalam Biosphere Reserve:
y Andaman and Nicobar Islands are separated Seshachalam hills, the first Biosphere
by a water body which is called the Ten- Reserve in Andhra Pradesh, is located in
degree channel. the southern Eastern Ghats of Chittoor and
Kadapa districts. It is the richest floristic
y Barren island, the only active volcano in
hotspot harbouring many endemic and rare
India, is also situated in the Nicobar Islands.
plants.
y The coastal line has some coral deposits
and beautiful beaches. These islands
receive convectional rainfall and have an 8. Answer: (d)
equatorial type of vegetation. Option (d) is correct: The latitudinal and
Some important mountain peaks in Andaman longitudinal extent of mainland India is about
and Nicobar Islands are Saddle peak, Mount 30°. The east-west extent appears to be
Diavolo, Mount Koyob and Mount Thuiller. smaller than the north-south extent. Arunachal
Pradesh and Gujarat are the easternmost and
westernmost States of India.
7. Answer: (a)
Additional Information:
Option (a) is correct: Sathyamangalam Tiger
Lying entirely in the Northern hemisphere, the
Reserve is the largest wildlife sanctuary
mainland extends between latitudes 8°4’N
located in the Erode district of Tamil Nadu. It is
and 37°6’N and longitudes 68°7’E and 97°25’E.
a part of the Nilgiri Biosphere Reserve and lies
The Tropic of Cancer (23° 30’N) divides the
between the Western Ghats and the Eastern
country into almost two equal parts. To the
Ghats. The reserve also adjoins four other
southeast and southwest of the mainland lie
protected areas: Biligiriranga Swamy Temple

Physiography of India-Explanation 177


unacademy.com | Download the Unacademy app
Give your feedback here: Link
the Andaman and Nicobar Islands and the The Southernmost point of the Indian Union is
Lakshadweep Islands in the Bay of Bengal and ‘Indira Point’.
the Arabian Sea, respectively.

178 Physiography of India-Explanation


unacademy.com | Download the Unacademy app
Give your feedback here: Link
9. Answer: (c) y The Narmada River is bounded by the
Pair 1 is not correctly matched: Cardamom Vindhyas on the north, by the Maikala range
Hills, mountainous area Malabar Coast in on the east, by the Satpuras on the south
South-eastern Kerala state, southern India, and by the Arabian Sea on the west. It is
forming part of the Western Ghats range. the largest west flowing river of peninsular
India.
Pair 2 is not correctly matched: Kaimur Hills,
eastern portion of the Vindhya Range, starting
near Katangi in the Jabalpur district of Madhya 11. Answer: (d)
Pradesh. There are three types of mountains- Fold
Pair 3 is correctly matched: Mahadeo Hills, Mountains, Block Mountains, and the Volcanic
sandstone hills located in the northern part Mountains. The Himalayan Mountains and the
of the Satpura Range, in southern Madhya Alps are young fold mountains with rugged
Pradesh state, central India. relief and high conical peaks.
Pair 4 is correctly matched: Mikir Hills are a Statement 1 is correct: A gorge is a narrow
group of hills located in Assam. Karbi plateau valley with steep, rocky walls located between
or Mikir Hills is known as the oldest landform hills or mountains. The Himalayas being young
in Assam. fold mountains are known for deep georges.

Elimination Technique: Kaimur Hills, Statement 2 is correct: The main Himalayan


eastern portion of the Vindhya Range is River systems are the Ganga, the Indus and the
located in central India. Thus, Option (c) Brahmaputra River systems. The Himalayan
is correct. rivers form large basins. The Brahmaputra
originates in the Man Sarovar lake, also the
source of the Indus and the Satluj. It is slightly
10. Answer: (a) longer than the Indus, but most of its course
Option (a) is correct: The Narmada rises in the lies outside India. It flows eastward, parallel
Amarkantak hills in Madhya Pradesh. It flows to the Himalayas. Reaching Namcha Barwa
towards the west in a rift valley formed due to (7757 m), it takes a U-turn around it and enters
faulting. On its way to the sea, the Narmada India in Arunachal Pradesh known as dihang.
creates many picturesque locations. The undercutting done by this river is of the
order of 5500 metres. In India, it flows through
The ‘Marble rocks’, near Jabalpur where the
Arunachal Pradesh and Assam, and is joined by
Narmada flows through a deep gorge, and the
several tributaries.
‘Dhuandhar falls’ where the river plunges over
steep rocks, are some of the notable ones. All Statement 3 is correct: The Himalayas are a
the tributaries of the Narmada are very short series of parallel mountain ranges extending
and most of these join the mainstream at right along the North-West to the South-East
angles. Its principal tributaries are the Burner, direction (known as the Strike of the Himalayas).
the Halon, the Heran, the Banjar, the Dudhi, These ranges are separated by longitudinal
the Shakkar, the Tawa, the Barna, the Kolar, the valleys. They include,
Ganjal, the Beda, the Goi, and the Orsang. y Trans-Himalayas
Additional Information: y The Greater Himalayas or Himadri
y The Narmada River is the fifth largest river in y The Lesser Himalayas or Himachal
the country and the largest one in Gujarat. y Shiwaliks or the Outer Himalayas
It traverses Madhya Pradesh, Maharashtra
and Gujarat and meets the Gulf of Cambay. y The Eastern Hills of Purvanchal

Physiography of India-Explanation 179


unacademy.com | Download the Unacademy app
Give your feedback here: Link
Statement 4 is correct: Himalayan mountains Option (b) is correct: Brahmaputra makes ‘U’
have a high altitude, steep gradients, snow- turn in its course to flow into India near Namcha
capped mountains, youthful river drainage Barwa and it is due to Syntaxial bending of
systems, and dissected topography. The geologically young Himalayas.
southern slope of the Himalayas is very steep Additional Information:
compared to its northern slope forming a hog
back shape. The steep gradient renders the y The gently arching ranges of the Himalayan
region prone to landslides. mountains on their Western and Eastern
extremities are sharply bent southward in
deep Knee-bend flexures that are called
12. Answer: (b) syntaxial bends.
Brahmaputra River has its source in southwest y On both the ends, the great mountains
Tibet. It flows there with the name Tsangpo appear to bend around a pivotal point.
River (flowing from the crest/ summit). The The western point is situated south of the
river runs across south Tibet to penetrate the Pamir where the Karakoram meets the
Himalayan Mountain Ranges in huge canyons Hindu Kush.
and subsequently into Arunachal Pradesh y A similar sharp, almost hairpin bend occurs
(India). In Arunachal Pradesh, it is called Dihang. on the eastern limit of Arunachal Pradesh
The river runs to the southwest across the where the strike of the mountain changes
Assam Basin as Brahmaputra and towards the sharply from the Easterly to Southerly trend.
south through Bangladesh as the Jamuna (you Besides these two major bends, there are a
should not confuse it with Yamuna of India). number of minor syntaxial bends in other
parts of Himalayas.

180 Physiography of India-Explanation


unacademy.com | Download the Unacademy app
Give your feedback here: Link
2 Drainage System of India

1. Consider the following statements : (2023) 4. Consider the following rivers: (2021)
1. Jhelum River passes through Wular 1. Brahmani
Lake. 2. Nagavali
2. Krishna River directly feeds Kolleru 3. Subarnarekha
Lake.
4. Vamsadhara
3. Meandering of the Gandak River formed
Kanwar Lake. Which of the above rise from the Eastern
Ghats?
How many of the statements given above
are correct? (a) 1 and 2

(a) Only one (b) 2 and 4

(b) Only two (c) 3 and 4

(c) All three (d) 1 and 3

(d) None
5. With reference to the Indus river system,
of the following four rivers, three of them
2. Consider the following pairs: (2022) pour into one of them which joins the Indus
Reservoirs States directly. Among the following, which one is
such a river that joins the Indus directly?
1 Ghataprabha Telangana
(2021)
2. Gandhi Sagar Madhya Pradesh
(a) Chenab
3. Indira Sagar Andhra Pradesh
(b) Jhelum
4. Maithon Chhattisgarh
(c) Ravi
How many pairs given above are not
correctly matched? (d) Sutlej

(a) Only one pair


(b) Only two pairs 6. Consider the following pairs: (2019)

(c) Only three pairs Famous place River


(d) All four pairs 1. Pandharpur Chandrabhaga
2. Tiruchirappalli Cauvery

3. Gandikota canyon of South India was 3. Hampi Malaprabha


created by which one of the following Which of the pairs given above are correctly
rivers? (2022) matched?

(a) Cauvery (a) 1 and 2 only

(b) Manjira (b) 2 and 3 only

(c) Pennar (c) 1 and 3 only

(d) Tungabhadra (d) 1, 2 and 3

Drainage System of India 181


unacademy.com | Download the Unacademy app
Give your feedback here: Link
7. Consider the following pairs: (2019) (b) 2 and 3 only

Glacier River (c) 1 and 3 only
1. Bandarpunch Yamuna (d) 1, 2 and 3
2. Bara Shigri Chenab
3.
Milam Mandakini 10. Consider the following rivers: (2015)
4. Siachen Nubra 1. Vamsadhara
5.
Zemu Manas 2. Indravati
Which of the pairs given above are correctly 3. Pranahita
matched? 4. Pennar
(a) 1, 2 and 4 only Which of the above are tributaries of the
(b) 1, 3 and 4 only River Godavari?
(c) 2 and 5 only (a) 1, 2 and 3 only
(d) 3 and 5 only (b) 2, 3 and 4 only
(c) 1, 2 and 4 only
8. With reference to river Teesta, consider (d) 2 and 3 only
the following statements: (2017)
1. The source of the river Teesta is the 11. What are the benefits of implementing
same as that of Brahmaputra, but it the ‘Integrated Watershed Development
flows through Sikkim. Programme’? (2014)
2. River Rangeet originates in Sikkim, and 1. Prevention of soil runoff
it is a tributary of the river Teesta.
2. Linking the country’s perennial rivers
3. River Teesta flows into the Bay of Bengal with seasonal rivers
on the border of India and Bangladesh.
3. Rainwater harvesting and recharge of
Which of the statements given above is/are groundwater table
correct?
4. Regeneration of natural vegetation
(a) 1 and 3 only
Select the correct answer using the code
(b) 2 only given below:
(c) 2 and 3 only (a) 1 and 2 only
(d) 1, 2 and 3 (b) 2, 3 and 4 only
(c) 1, 3 and 4 only
9. Which of the following is/are tributary/ (d) 1, 2, 3 and 4
tributaries of Brahmaputra? (2016)
1. Dibang
12. Consider the following rivers: (2014)
2. Kameng
1. Barak
3. Lohit
2. Lohit
Select the correct answer using the code
given below. 3. Subansiri

(a) 1 only Which of the above flows/flow through


Arunachal Pradesh?

182 Drainage System of India


unacademy.com | Download the Unacademy app
Give your feedback here: Link
(a) 1 only a short distance from the coast of Bay
(b) 2 and 3 only of Bengal before flowing into the sea.
This is an important site of wildlife and
(c) 1 and 3 only biodiversity and a protected area. Which
(d) 1, 2 and 3 one of the following could be this? (2011)
(a) Bhitarkanika
13. Two important rivers- one with its source (b) Chandipur-on-sea
in Jharkhand (and known by a different (c) Gopalpur-on-sea
name in Odisha), and another, with its
source in Odisha- merge at a place only (d) Simlipal

Drainage System of India 183


unacademy.com | Download the Unacademy app
Give your feedback here: Link
Drainage System of India-
2 Explanation
1. Answer: (b) from Dhanbad, in the state of Jharkhand. It
Option (b) is correct: is located on the banks of Barakar River. The
Barakar River is the tributary of the Damodar
Statement 1 is correct: The Jhelum, an River.
important tributary of the Indus, rises from a
spring at Verinag situated at the foot of the Pir Additional Information: Polavaram Irrigation
Panjal in the south-eastern part of the valley Project (Andhra Pradesh) is located on river
of Kashmir. It flows through Srinagar and the Godavari in West Godavari district of Andhra
Wular lake before entering Pakistan through a Pradesh. The project is a multipurpose major
deep narrow gorge. terminal reservoir project on river Godavari
for development of Irrigation, Hydro power
Statement 2 is not correct: Krishna River and drinking water facilities to East Godavari,
does not feed Kolleru directly. Kolleru, one Visakhapatnam, West Godavari and Krishna
of the largest freshwater lakes in India, (it districts of Andhra Pradesh.
was designated a sanctuary in October 1999)
is situated between the Krishna and West
Godavari districts of the state. . Budameru, 3. Answer: (c)
Tammileru, Ramileru, Gaderu and Bulusuvagu Option (c) is correct: Gandikota Canon is
rivers as well as a number of streams drain situated on the Pennar River in Andhra Pradesh
their waters into the lake. and is known as the Grand Canyon of India.
Statement 3 is correct: The Kanwar lake in It is formed between the Erramala range of
Bihar, Asia’s largest freshwater oxbow lake, is hills which are also known as Gandikota hills.
today a dying wetland ecosystem. It is a residual The river Pennar that flows at its foot and the
oxbow lake, formed due to meandering of area is marked by deep valleys and massive
Gandak river, a tributary of Ganga. boulders of granite
A gorge is a narrow valley between hills or
2. Answer: (c) mountains, typically with steep rocky walls
and a stream running through it. Gorges are
Ghataprabha- Karnataka (Pair 1 is not correctly formed because of rock erosion over a long
matched) period of time.
Gandhi Sagar- Madhya Pradesh (Pair 2 is
correctly matched)
4. Answer: (b)
Indira Sagar- Madhya Pradesh (there is one
irrigation project with the similar name in Option (b) is correct: Vamsadhara River and
Andhra as the ‘Indira Sagar Polavaram-project’ Nagavali River originate from the Eastern
commonly known as the Polavaram Irrigation Ghats.
Project) (Pair 3 is not correctly matched) The Brahmani river rises near Nagri village
Maithon- Jharkhand (Pair 4 is not correctly in the Ranchi district of Jharkhand (not the
matched) Eastern Ghats) at an elevation of about 600 m
and has a total length of 799 km.
The Gandhi Sagar Dam is built on India’s
Chambal River. The dam is located in the The Subarnarekha River rises near Nagri village
Mandsaur district, Madhya Pradesh. in the Ranchi District of Jharkhand at an
elevation of 600 m. It flows for a length of 395
The Maithon Dam is located at Maithon, 48 km

184 Drainage System of India-Explanation


unacademy.com | Download the Unacademy app
Give your feedback here: Link
km before outfalling into the Bay of Bengal. Its 6. Answer: (a)
principal tributaries joining from the right are Pair 1 is correctly matched: Pandharpur Wari
the Kanchi, the Karkari and the Kharkai. is an annual pilgrimage (yatra) to Pandharpur,
which is the seat of the Hindu god Vithal.
5. Answer: (d) The Journey involves Palakhis (palanquin
processions) carrying the paduka (footprints) of
Option (d) is correct: The Indus River System
various saints and Upon reaching Pandharpur
is one of the largest river basins in the world.
on Ashadi Ekadashi, these devotees take a holy
The Indus, also known as the Sindhu, is the
dip in the sacred Chandrabhaga River/Bhima
westernmost of the Himalayan rivers in India.
River before proceeding to visit the Vitthal
It originates from a glacier near Bokhar Chu
temple.
in the Tibetan region in the Kailash Mountain
range. Pair 2 is correctly matched: Tiruchirapalli
District is located along the Kaveri (Cauvery)
The Sutlej River is the most extensive of the five
River in Tamil Nadu, India.
rivers that run through the famous crossroad
area of the state of Punjab in North India and Pair 3 is not correctly matched: The site of
Pakistan. The river is situated on the north Hampi comprises mainly the remains of the
of the Vindhya Mountain Range, to the south capital city of the Vijayanagara Empire (14th-
of the Hindu Kush division of the Himalayan 16th century CE), the last great Hindu Kingdom.
Mountain Ranges and to the east of the Central It is a UNESCO World Heritage Site in India.
Sulaiman range in Pakistan. Chenab, Jhelum Hampi’s spectacular setting is dominated by
and Ravi pour into Sutlej, which joins the Indus river Tungabhadra, rugged hill ranges and open
directly. plains with widespread physical remains.

The Sutlej is joined by the Beas River in Elimination techniques: Hampi is located
Hari-Ke-Patan, Amritsar, Punjāb, India, and at the bank of river Tungabhadra. Pair 3 is
continues southwest into Pakistan to unite not correctly matched.Thus, Option (a) is
with the Chenab River, near Bahawalpur. The correct.
Sutlej joins the Indus River at Mithankot.

7. Answer: (a)
Pair 1 is correctly matched: Bandarpunch
glacier is located in the Garhwal region of
Uttarakhand. The River Yamuna originates
on the West end of the Bandarpunch cirque
Glacier. The Eastern slopes of the Bandarpunch
mountain feed the River Ganga. This is a unique
mountain that feeds both Yamuna and Ganga.
Pair 2 is correctly matched: Bara Shigri glacier
is located in Lahaul and Spiti Valley of Himachal
Pradesh. The Bara Shigri glacier attracted
much attention for many years because of
the valuable antimony deposits found there. It
feeds the Chandra River and after the junction
at Tandi with the Bhaga River, it is known as
the Chenab river.
Pair 3 is not correctly matched: Milam is

Drainage System of India-Explanation 185


unacademy.com | Download the Unacademy app
Give your feedback here: Link
situated in the Kumaon Himalayas, originating the Yamuna. Teesta River ultimately drains
from the eastern slopes of the Trishuli. The into the Brahmaputra at Teestamukh Ghat
river Gori Ganga has its source here. Mandakini in Kamarjani- Bahadurabad in the Rangpur
River is a tributary of the Alakananda River. district of Bangladesh.
Pair 4 is correctly matched: The Siachen Additional Information:
Glacier is located in Northern Ladakh in the y The Teesta Basin in India extends over
eastern Karakoram Range of the Himalayas. It an area of 9,855 sq km, which is nearly
is the source of the Nubra River, a tributary of 0.28% of the total geographical area of
the Indus River flowing into Pakistan and the the country. The basin lies in the states of
Arabian Sea. The water of the Nubra drains into Sikkim (72.43%) and West Bengal (27.57%).
the Shyok River and then into the Indus. The
Siachen Glacier is the highest battleground on Elimination Technique: Teesta river is a
Earth. tributary of the Brahmaputra. Statement
Pair 5 is not correctly matched: The Zemu 3 is eliminated. Thus, Option (b) is the
glacier is the largest glacier in the Eastern correct answer.
Himalayas located in Sikkim. It is situated
at the base of the Kanchenjunga and is one 9. Answer: (d)
of the sources for the Teesta River that joins
The Brahmaputra, one of the largest rivers of
the River Brahmaputra. The Manas River is a
the world, has its origin in the Chemayungdung
trans-boundary river in the Himalayas between
glacier of the Kailash range near the Man
Bhutan and India.
Sarovar lake. From here, it traverses eastward
longitudinally for a distance of nearly 1,200 km
8. Answer: (b) in a dry and flat region of southern Tibet, where
Statement 1 is not correct: The Brahmaputra, it is known as the Tsangpo, which means ‘the
one of the largest rivers of the world, has purifier.’
its origin in the Chemayungdung glacier of The Rango Tsangpo is the major right bank
the Kailash range near the Mansarovar lake. tributary of this river in Tibet. It emerges as a
The Teesta River originates from the Pahunri turbulent and dynamic river after carving out
(or Teesta Kangse) glacier above 7,068 m and a deep gorge in the Central Himalayas near
flows southward through gorges and rapids in Namcha Barwa (7,755 m). The river emerges
the Sikkim Himalaya. from the foothills under the name of Siang or
The river then flows past the town of Rangpo, Dihang.
where the Rangpo River joins and where it Option (d) is correct: Brahmaputra enters
forms the border between Sikkim and West India in the west of Sadiya town in Arunachal
Bengal. Pradesh. Flowing southwest, it receives its
Statement 2 is correct: The Rangeet River is main left bank tributaries, viz., Dibang or
the main tributary of Teesta, and it originates in Sikang and Lohit; thereafter, it is known as the
Sikkim. It changes course southwards, flowing Brahmaputra.
entirely into West Bengal. The river hits the The Brahmaputra receives numerous tributaries
plains at Sevoke, where it is spanned by the in its 750 km long journey through the Assam
Coronation Bridge which links the north east valley. Its major left bank tributaries are the
states to the rest of India. Burhi Dihing and Dhansari (South) whereas
Statement 3 is not correct: In Bangladesh, the important right bank tributaries are the
the Teesta joins the Brahmaputra on its Subansiri, Kameng, Manas and Sankosh.
right bank from where the river is known as The Subansiri which has its origin in Tibet, is an

186 Drainage System of India-Explanation


unacademy.com | Download the Unacademy app
Give your feedback here: Link
antecedent river. The Brahmaputra enters into harnessing, conserving and developing
Bangladesh near Dhubri and flows southward. degraded natural resources such as soil,
In Bangladesh, the Tista joins it on its right vegetative cover and water.
bank from where the river is known as the y Prevention of soil erosion.
Yamuna. It finally merges with the river Padma,
which falls in the Bay of Bengal. y Rainwater harvesting.
y Recharging of the groundwater table.

10. Answer: (d) y Regeneration of natural vegetation.

The Godavari is the largest Peninsular River. It This enables multi-cropping and the
rises from the slopes of the Western Ghats in introduction of diverse agro-based activities,
the Nasik district of Maharashtra. Its length is which help to provide sustainable livelihoods
about 1500 km. It drains into the Bay of Bengal. to the people residing in the watershed area.
Its drainage basin is also the largest among the The National River Linking Project (NRLP)
peninsular rivers. formally known as the National Perspective
Plan, envisages the transfer of water from
Option (d) is correct: The basin covers parts of water ‘surplus’ basins where there is flooding
Maharashtra (about 50 per cent of the basin to water ‘deficit’ basins where there is drought/
area lies in Maharashtra), Madhya Pradesh, scarcity through inter-basin water transfer
Odisha and Andhra Pradesh. The Pravara, projects.
Manjira and Maner are right bank tributaries
covering about 16.14%, the Purna, Pranahita, The National River Linking Project (NRLP)
Indravati and Sabari are important left bank formally known as the National Perspective
tributaries, covering nearly 59.7% of the total Plan, envisages the transfer of water from
catchment area of the basin. Because of its water ‘surplus’ basins where there is flooding
length and the area it covers, it is also known to water ‘deficit’ basins where there is drought/
as the ‘Dakshin Ganga’. scarcity through inter-basin water transfer
projects.

11. Answer: (c)


12. Answer: (b)
Watershed is defined as a hydro-geological
unit of area from which the rainwater drains Option (b) is correct: Five major river valleys of
through a single outlet. Watershed development Arunachal Pradesh: the Kameng, the Subansiri,
refers to the conservation, regeneration, and the Siang, the Lohit and the Tirap. All these
judicious use of all the natural resources (like rivers are fed by snow from the Himalayas
land, water, plants, animals) by human beings. and countless rivers and rivulets except Tirap
Watershed Management brings about the best which is fed by Patkai Range. The mightiest of
possible balance between natural resources on these rivers is Siang, called Tsangpo in Tibet,
the one side and human beings on the other. which becomes Brahmaputra after it is joined
by the Dibang and the Lohit in the plains of
The Integrated Watershed Management Assam.
Programme (IWMP) is implemented by the
Department of Land Resources of the Ministry y The Lohit River is a tributary to the
of Rural Development. Brahmaputra River. The river rises in Tibet
Autonomous Region, in the Kangri Garpo
Option (c) is correct: The main benefits of the range, where it is known as the Zayü River.
Integrated Watershed Development Programme
are: y The Barak River rises in the Manipur Hills
in Manipur, India and flows west and
y Restoration of the ecological balance by then southwest into Mizoram state. The

Drainage System of India-Explanation 187


unacademy.com | Download the Unacademy app
Give your feedback here: Link
river flows through the states of Manipur, of Orissa is surrounded by rivers Baitarani,
Nagaland, Mizoram and Assam in India and Brahmani, Dhamara, and is crisscrossed by
into the Bay of Bengal via Bangladesh. It several creeks-creeklets. Geographically
does not pass through Arunachal Pradesh. situated in Kendrapara district, Bhitarkanika
y Subansiri, also known as the “Gold River”. forms the deltaic region of the river Brahmani
It is the biggest tributary of the Upper and Baitarani. Bhitarkanika is home to the
Brahmaputra. It originated from the Tibetan largest population of saltwater crocodile,
Himalayas and makes its way to India water monitor lizard, king cobra, python and
through Arunachal Pradesh. many other reptiles and amphibians besides
herbivores
Bhitarkanika National Park is situated in
13. Answer: (a) Kendrapara district, which is about 162 km
Option (a) is correct: Bhitarkanika, a from Bhubaneswar, the state capital. The Park
storehouse of nature’s bounty, harbours a rich encompasses an area of 672 km².
and unique biodiversity. This unique ecosystem

188 Drainage System of India-Explanation


unacademy.com | Download the Unacademy app
Give your feedback here: Link
3 Indian Climate

1. With reference to 'Indian Ocean Dipole rains in North-Western region of India


(IOD)' sometimes mentioned in the news are part of westerlies.
while forecasting Indian monsoon, which Which of the statements given above is/are
of the following statements is/are correct? correct?
(2017)
(a) 1 only
1. The IOD phenomenon is characterised by
a difference in sea surface temperature (b) 2 only
between tropical Western Indian Ocean (c) Both 1 and 2
and tropical Eastern Pacific Ocean. (d) Neither 1 nor 2
2. An IOD phenomenon can influence an
El Nino's impact on the monsoon.
3. Consider the following statements: (2012)
Select the correct answer using the code
1. The duration of the monsoon decreases
given below:
from southern India to northern India.
(a) 1 only
2. The amount of annual rainfall in the
(b) 2 only northern plains of India decreases from
(c) Both 1 and 2 east to west.
(d) Neither 1 nor 2 Which of the statements given above is /
are correct?

2. Consider the following statements: (2015) (a) 1 Only

1. The winds which blow between 30 N (b) 2 Only


and 60 S latitudes throughout the year (c) Both 1 and 2
are known as westerlies. (d) Neither 1 nor 2
2. The moist air masses that cause winter

Indian Climate 189


unacademy.com | Download the Unacademy app
Give your feedback here: Link
3 Indian Climate-Explanation

1. Answer: (b) coasts of continents, especially in the southern


Statement 1 is not correct: The Indian Ocean hemisphere because of its vast oceanic
Dipole (IOD) is the Indian Ocean counterpart of expanse.
the Pacific El Niño and La Niña. The term dipole Statement 2 is correct: The western
means two “poles” or two areas of differences. cyclonic disturbances which enter the Indian
The IOD measures differences in SSTs between subcontinent from the west and the northwest
the Arabian Sea (western pole) and the eastern during the winter months originate over the
Indian Ocean south of Indonesia (eastern pole). Mediterranean Sea and are brought into India
Statement 2 is correct: About 50% per cent by the westerly jet stream. An increase in the
of the IOD events in the past 100 years have prevailing night temperature generally indicates
co-occurred with El Nino Southern Oscillation an advance in the arrival of these cyclones’
(ENSO) and the other half independently. El disturbances.
Nino and negative or neutral IOD in the Indian
Ocean, the chances of a bad monsoon are 3. Answer: (c)
very high. During the positive IOD phase, India
Option (c) is correct: In India Monsoons arrive
experiences anomalously high rainfall along the
at the Bay of Bengal branch and Arabian Sea
latitude belts covering Central India, and during
branch. Bay of Bengal branch of south west
the negative phase of the IOD, the rainfall is
monsoon reaches north east of India. The
anomalously high along the longitudinal belt,
Arabian Sea Branch of the Southwest Monsoon
with the western part of the country receiving
first hits the Western Ghats of Kerala making
high rainfall. Positive IOD events are often
this area first to receive rain from the Southwest
associated with El Niño and negative events
Monsoon moving northwards towards western
with La Niña.
side. The duration of the monsoon is between
100 to 120 days. The low pressure system over
2. Answer: (b) north and north-west India gradually weakens,
Statement 1 is not correct: Westerlies are and this leads to the retreat of the monsoon
prevailing winds that blow from the west at winds by the end of this period. With large
midlatitudes (between 30 and 60 degrees areas of western India, summer monsoon lasts
latitudes). The strongest westerlies blow from June-September and during the period
through the “Roaring Forties,” a wind zone central India receives more than 90% of their
between 40 and 50 degrees latitude in the total annual precipitation, and southern and
Southern Hemisphere. They tend to blow northwestern India receive 50%-75% of their
from the high-pressure area in the horse total annual rainfall. Overall, monthly totals
latitudes towards the poles. The Westerlies average 200-300 mm over the country as a
play an important role in carrying the warm, whole, with the largest values observed during
equatorial waters and winds to the western the heart of the monsoon season in July and
August.

190 Indian Climate-Explanation


unacademy.com | Download the Unacademy app
Give your feedback here: Link
4 Soils

1. Consider the following statements (2023) 3. Tropical climate


Statement-I: Select the correct answer using the code
The soil in tropical rain forests is rich in given below:
nutrients. (a) 1 and 2 only
Statement-II: (b) 2 only
The high temperature and moisture of (c) 1 and 3 only
tropical rain forests cause dead organic (d) 1, 2 and 3
matter in the soil to decompose quickly.
Which one of the following is correct in
respect of the above statements? 4. Contour bunding is a method of soil
conservation used in (2013)
(a) Both Statement-I and Statement-II are
correct and Statement-II is the correct (a) desert margins, liable to strong wind
explanation for Statement-I action

(b)
Both Statement-I and Statement-II (b) low flat plains, close to stream courses,
are correct and Statement-II is not the liable to flooding
correct explanation for Statement-I (c) scrublands, liable to spread of weed
(c) Statement-I is correct but Statement-II growth
is incorrect (d) None of the above
(d) Statement-I is incorrect but Statement-
II is correct 5. Which of the following statements
regarding laterite soils of India are correct?
2. The black cotton soil of India has been (2013)
formed due to the weathering of (2021) 1. They are generally red in colour.
(a) brown forest soil 2. They are rich in nitrogen and potash.
(b) fissure volcanic rock 3. They are well-developed in Rajasthan
(c) granite and schist and UP.

(d) shale and limestone 4. Tapioca and cashew nuts grow well on
these soils.
Select the correct answer using the code
3. In India, the problem of soil erosion is given below:
associated with which of the following?
(2014) (a) 1, 2 and 3

1. Terrace cultivation (b) 2, 3 and 4

2. Deforestation (c) 1 and 4


(d) 2 and 3 only

Soils 191
unacademy.com | Download the Unacademy app
Give your feedback here: Link
4 Soils-Explanation

1. Answer: (d) Option (b) is correct: In India, the problem of


Statement 1 is incorrect: Tropical rainforest soil erosion is associated with deforestation
soils may not be considered rich in all nutrients, and not with terrace cultivation and tropical
they are highly productive and support climate.
diverse and abundant plant life. The complex y Deforestation is the permanent removal of
interactions between plants, microorganisms, trees to make room for something besides
and nutrient cycling processes contribute to forest. This can include clearing the land for
the overall fertility and resilience of rainforest agriculture or grazing, or using the timber
ecosystems. for fuel, construction or manufacturing.
Statement 2 is correct: The combination of Without plant cover, erosion can occur and
high temperature and moisture in tropical sweep the land into rivers. The agricultural
rainforests creates favourable conditions plants that often replace the trees cannot
for the rapid decomposition of dead organic hold onto the soil and many of these plants,
matter in the soil. This process is known as such as coffee, cotton, palm oil, soybean
decomposition or decay.The warm and humid and wheat, can actually worsen soil erosion.
climate in tropical rainforests accelerates the y Terrace cultivation: It is a method of growing
activities of decomposer organisms such as crops on sides of hills or mountains by
bacteria, fungi, and other microorganisms. planting on graduated terraces built into
These decomposers break down organic matter, the slope. It is a labour-intensive method.
including fallen leaves, twigs, dead plants, and Aims to maximise arable land area in
animal remains, into simpler compounds and variable terrains and to reduce soil erosion
nutrients. and water loss.
y Tropical climate: Tropical climate with
2. Answer: (b) healthy soils can support multiple
ecosystem services. Regions in the tropical
Option (b) is correct: The black soils are found climate zones suffer the greatest rainfall-
in the lava-covered areas. The Black soil is related soil erosion. But tropical climate in
often referred to as “black cotton soils” since itself is not a cause of soil erosion.
cotton has been the most common traditional
crop in areas where they are found. Black soils
are derivatives due to the weathering of fissure 4. Answer: (d)
volcanic rock. The soil is spread mostly across Option (d) is correct: Contour bunding is a
interior Gujarat, Maharashtra, Karnataka, and proven sustainable land management practice
Madhya Pradesh on the Deccan lava plateau for marginal, sloping, and hilly land where the
and the Malwa Plateau, where there is both soil productivity is very low. It can be adopted in
moderate rainfall and underlying basaltic rock. light and medium-textured soils. It can be laid
up to 6% slopes. It helps to retain moisture in
3. Answer: (b) the field. It has a high probability of replication
because it is simple to implement, is low cost,
Soil erosion is a gradual process of movement and makes maximum use of local resources.
and transport of the upper layer of soil (topsoil)
by different erosional agents like water and
wind.

192 Soils-Explanation
unacademy.com | Download the Unacademy app
Give your feedback here: Link
5. Answer: (c) in excess. Hence, laterites are not suitable for
Statement 1 is correct: Laterite has been cultivation.
derived from the Latin word ‘Later’ which Statement 3 is not correct: These soils have
means brick. They are generally red in colour. mainly developed in the higher areas of the
The laterite soils develop in areas with high Peninsular plateau. The laterite soils are
temperatures and high rainfall. These are the commonly found in Karnataka, Kerala, Tamil
result of intense leaching due to tropical rains. Nadu, Madhya Pradesh and the hilly areas of
With rain, lime and silica are leached away, Odisha and Assam.
and soils rich in iron oxide and aluminium Statement 4 is correct: Application of manures
compounds are left behind. Humus content of and fertilisers are required for making the soils
the soil is removed fast by bacteria that thrive fertile for cultivation. Red laterite soils in Tamil
well in high temperatures. Nadu, Andhra Pradesh and Kerala are more
Statement 2 is not correct: These soils are suitable for tree crops like Tapioca and cashew
poor in organic matter, nitrogen, phosphate nut. Laterite soils are widely cut as bricks for
and calcium while iron oxide and potash are use in house construction.

Soils-Explanation 193
unacademy.com | Download the Unacademy app
Give your feedback here: Link
5 Natural Vegetation in India

1. Consider the following statements: (2023) 1. Oak


1. Amarkantak Hills are at the confluence 2. Rhododendron
of Vindhya and Sahyadri Ranges. 3. Sandalwood
2. Biligirirangan Hills constitute the Select the correct answer using the code
easternmost part of Satpura Range. given below:
3. Seshachalam Hills constitute the (a) 1 and 2 only
southernmost part of Western Ghats
(b) 3 only
How many of the statements given above
are correct? (c) 1 and 3 only

(a) Only one (d) 1, 2 and 3

(b) Only two


(c) All three 5. Which of the following have coral reefs?
(2014)
(d) None
1. Andaman and Nicobar Islands
2. Gulf of Kachchh
2. In a particular region in India, the local
people train the roots of living trees into 3. Gulf of Mannar
robust bridges across the streams. As 4. Sunderbans
the time passes, these bridges become Select the correct answer using the code
stronger. These unique 'living root bridges' given below:
are found in (2015)
(a) 1, 2 and 3 only
(a) Meghalaya
(b) 2 and 4 only
(b) Himachal Pradesh
(c) 1 and 3 only
(c) Jharkhand
(d) 1, 2, 3 and 4
(d) Tamil Nadu

6. With reference to the wetlands of India,


3. In India, in which one of the following types consider the following statements: (2012)
of forests is teak a dominant tree species?
1. The country’s total geographical area
(2015)
under the category of wetlands is
(a) Tropical moist deciduous forest recorded more in Gujarat as compared
(b) Tropical rainforest to other States.
(c) Tropical thorn scrub forest 2. In India, the total geographical area of
(d) Temperate forest with grasslands coastal wetlands is larger than that of
inland wetlands.
Which of the statements given above is/are
4. If you travel through the Himalayas, you are correct?
likely to see which of the following plants
are naturally growing there? (2014) (a) 1 only

194 Natural Vegetation in India


unacademy.com | Download the Unacademy app
Give your feedback here: Link
(b) 2 only hedge against coastal calamities. How do
(c) Both 1 and 2 mangroves function as a safety hedge?
(2011)
(d) Neither 1 nor 2
(a) The mangrove swamps separate the
human settlements from the sea by a
7. In which one among the following wide zone in which people neither live
categories of protected areas in India are nor venture out.
local people not allowed to collect and use (b) The mangroves provide both food and
biomass? (2012) medicines which people are in need of
(a) Biosphere Reserves after any natural disaster.
(b) National Parks (c) The mangrove trees are tall with dense
(c)
Wetlands declared under Ramsar canopies and serve as an excellent
Convention shelter during a cyclone or Tsunami.

(d) Wildlife Sanctuaries (d) The mangrove trees do not get uprooted
by storms and tides because of their
extensive roots.
8. The 2004 Tsunami made people realise that
mangroves can serve as a reliable safety

Natural Vegetation in India 195


unacademy.com | Download the Unacademy app
Give your feedback here: Link
Natural Vegetation in India-
5 Explanation
1. Answer: (d) the area, to grow laterally across a stream bed,
Statement 1 is not correct: The Amarkantak resulting in a living bridge of roots. As the time
region is a unique natural heritage area and passes, these bridges become stronger.
is the meeting point of the Vindhya and the
Satpura Ranges, with the Maikal Hills being the 3. Answer: (a)
fulcrum. This is where the Narmada River, the Tropical deciduous forests are the most
Son River and Johilla River (tributary of Son) widespread forests in India. They are also
originate. called the monsoon forests.
Statement 2 is not correct: The Biligirirangan y They spread over regions which receive
Hills is a hill range situated in south-western rainfall between 70-200 cm. On the basis
Karnataka, at its border with Tamil Nadu (Erode of the availability of water, these forests
District) in South India. The hills are located at are further divided into moist and dry
the north-west of the Western Ghats and the deciduous.
westernmost edge of the Eastern Ghats.
y The Moist deciduous forests are more
Statement 3 is not correct: Seshachalam pronounced in the regions which record
Hills are hilly ranges part of the Eastern rainfall between 100-200 cm.
Ghats in southern Andhra Pradesh state, in
southeastern India. The Seshachalam hill y These forests are found in the northeastern
ranges are predominantly present in Tirupati states along the foothills of Himalayas,
district of the Rayalaseema region in Andhra eastern slopes of the Western Ghats and
Pradesh, India. Odisha. Teak, sal, shisham, hurra, mahua,
amla, semul, kusum, and sandalwood etc.,
are the main species of these forests.
2. Answer: (a) [Option (a) is correct]

4. Answer: (a)
The Himalayan ranges show a succession of
vegetation from the tropical to the tundra, which
change with the altitude. Deciduous forests
are found in the foothills of the Himalayas. It
is succeeded by the wet temperate type of
forests between an altitude of 1,000-2,000 m.
Option 1 is correct: In the higher hill ranges of
northeastern India, hilly areas of West Bengal
and Uttaranchal, evergreen broadleaf trees
Option (a) is correct: The Jing Kieng Jri or living
such as oak and chestnut are predominant.
root bridges — aerial bridges built by weaving
and manipulating the roots of the Indian rubber Option 2 is correct: Between 1,500-1,750 m,
tree — have been serving as connectors for pine forests are also well-developed in this
generations in Meghalaya. A root bridge uses zone, with Chir Pine as a very useful commercial
traditional tribal knowledge to train roots of tree. Deodar, a highly valued endemic species,
the Indian rubber tree, found in abundance in grows mainly in the western part. Blue pine

196 Natural Vegetation in India-Explanation


unacademy.com | Download the Unacademy app
Give your feedback here: Link
and spruce appear at altitudes of 2,225- 6. Answer: (a)
3,048 m. Silver firs, junipers, pines, birch and Wetlands occur wherever water meets land.
rhododendrons occur between 3,000-4,000 m. These unique habitats include mangroves,
At higher altitudes, mosses and lichens form peatlands and marshes, rivers and lakes,
part of the tundra vegetation. deltas, floodplains and flooded forests, rice-
Option 3 is not correct: Sandalwood is found fields, and even coral reefs. Wetlands exist in
in tropical deciduous forests which are found every country and in every climatic zone, from
in western ghats, Deccan plateau and northern the polar regions to the tropics, and from high
plains. It is not a natural vegetation of the altitudes to dry regions.
Himalayan range. Statement 1 is correct: Gujarat is ahead of other
states in the country in ecological richness. This
5. Answer: (a) is evident from the recent ‘National wetland
inventory and assessment’ report prepared by
Coral reefs are one of the most biologically
the Space Applications Centre, Ahmedabad. In
diverse marine ecosystems on the Earth. Coral
the report, Gujarat comes out on top with the
polyps, the animals primarily responsible for
largest amount of its landmass identified as
building reefs, can take many forms: large reef
wetlands of different kinds.
building colonies, graceful flowing fans, and
even small, solitary organisms. Statement 2 is not correct: According to the
2011 Report of the Ministry of Environment,
Option (a) is correct: Coral reefs are present
Forest and Climate Change, India has a total
in the areas of Gulf of Kutch, Gulf of Mannar,
2,01,503 wetlands, of which 1,88,470 are inland
Andaman & Nicobar, Lakshadweep Islands and
wetlands and 13,033 are coastal wetlands. Area
Malvan.
wise inland wetlands consist of 69.23 percent
and coastal wetlands consist of 27.13 percent,
while other wetlands occupy 3.64 percent.
Additional Information:
y The Ramsar Convention is the
intergovernmental treaty that provides the
framework for the conservation and wise
use of wetlands and their resources.
y The Convention was adopted in the Iranian
city of Ramsar in 1971 and came into force
in 1975. Since then, almost 90% of UN
member states have become “Contracting
Parties” and committed to the Convention’s
Coral reef enables the formation of associated three pillars:
eco-systems which allow the formation of y Work towards the wise use of all their
essential habitats, fisheries, and livelihoods. wetlands.
In addition, coral reefs are climatologically
important because they provide an accurate y Designate suitable wetlands for the list
long-term record of climate change and help in of Wetlands International Importance and
extending our knowledge of seasonal climate ensure their effective management.
variability in many remote tropical oceans. y Cooperate internationally on transboundary
wetlands, shared wetland systems and
shared species.

Natural Vegetation in India-Explanation 197


unacademy.com | Download the Unacademy app
Give your feedback here: Link
7. Answer: (b) 8. Answer: (d)
Option (b) is correct: National Parks are among Mangroves are shrubs or small trees that grow
the categories of protected areas in India in coastal saline or brackish water. Mangroves
where local people are not allowed to collect are salt-tolerant trees, also called halophytes,
and use biomass. National Parks are formed by and are adapted to harsh coastal conditions.
the State or central Legislature. A national park Mangrove vegetation facilitates more water
cannot be downgraded to a sanctuary. loss.
National Park is an area which is strictly Option (d) is correct: Mangroves are highly
reserved for the betterment of the wildlife productive ecosystems, and the trees may
& biodiversity, and where activities like vary in height from 8 to 20 m. They protect
developmental, forestry, poaching, hunting the shoreline from the effect of cyclones and
and grazing on cultivation are not permitted. In tsunamis. It is due to the fact that mangrove
these parks, even private ownership rights are trees do not get uprooted by storms and tides
not allowed. Their boundaries are well marked because of their extreme roots.
and circumscribed. Mangrove belts and their extensive roots
Additional Information: several hundred metres wide have been
There are 104 existing national parks in India shown to reduce tsunami height by between
covering an area of 43,716 km2, which is 5 and 30%. Wider mangrove forests are more
1.33% of the geographical area of the country effective at reducing tsunami height, as well
(National Wildlife Database, Dec. 2020). as speed of the water and the area flooded by
the tsunami.

198 Natural Vegetation in India-Explanation


unacademy.com | Download the Unacademy app
Give your feedback here: Link
6 Mineral and Industries

1. Ilmenite and rutile, abundantly available in 4. Consider the following statements: (2021)
certain coastal tracts of India, are rich sources 1. The Global Ocean Commission grants
of which one of the following? (2023) licences for seabed exploration and
(a) Aluminum mining in international waters.
(b) Copper 2. India has received licences for seabed
(c) Iron mineral exploration in international
waters.
(d) Titanium
3. 'Rare earth minerals' are present on the
seafloor in international waters.
2. Consider the following heavy industries: Which of the statements given above are
(2023) correct?
1. Fertilizer plants (a) 1 and 2 only
2. Oil refineries (b) 2 and 3 only
3. Steel plants (c) 1 and 3 only
Green hydrogen is expected to play a (d) 1, 2 and 3
significant role in decarbonizing how many
of the above industries?
(a) Only one 5. Consider the following minerals: (2020)

(b) Only two 1. Bentonite

(c) All three 2. Chromite

(d) None 3. Kyanite


4. Sillimanite

3. With reference to India, consider the In India, which of the above is/are officially
following statements: (2022) designated as major minerals?

1. Monazite is a source of rare earths. (a) 1 and 2 only

2. Monazite contains thorium. (b) 4 only

3. Monazite occurs naturally in the entire (c) 1 and 3 only


Indian coastal sands in India. (d) 2, 3 and 4 only
4. In India, Government bodies only can
process or export monazite. 6. With reference to the management of minor
Which of the statements given above are minerals in India, consider the following
correct? statements: (2019)
(a) 1, 2 and 3 only 1. Sand is a ‘minor mineral’ according to
(b) 1, 2 and 4 only the prevailing law in the country.

(c) 3 and 4 only 2. State Governments have the power


to grant mining leases of minor
(d) 1, 2, 3 and 4 minerals, but the powers regarding the

Mineral and Industries 199


unacademy.com | Download the Unacademy app
Give your feedback here: Link
formation of rules related to the grant 9. In India, the steel production industry
of minor minerals lie with the Central requires the import of (2015)
Government. (a) saltpetre
3. State Governments have the power to (b) rock phosphate
frame rules to prevent illegal mining of
minor minerals. (c) coking coal

Which of the statements given above is/are (d) All of the above
correct?
(a) 1 and 3 only 10. Which one among the following industries
(b) 2 and 3 only is the maximum consumer of water in
India? (2013)
(c) 3 only
(a) Engineering
(d) 1, 2 and 3
(b) Paper and pulp
(c) Textiles
7. Consider the following statements: (2018)
(d) Thermal power
1. In India, State Governments do not have
the power to auction non-coal mines.
2. Andhra Pradesh and Jharkhand do not 11. Consider the following statements: (2013)
have gold mines. 1. Natural gas occurs in the Gondwana beds.
3. Rajasthan has iron ore mines. 2. Mica occurs in abundance in Kodarma.
Which of the statements given above is/are 3. Dharwars are famous for petroleum.
correct? Which of the statements given above is/are
(a) 1 and 2 correct?
(b) 2 only (a) 1 and 2
(c) 1 and 3 (b) 2 only
(d) 3 only (c) 2 and 3
(d) None
8. In which of the following regions of India
are shale gas resources found? (2016) 12.
Which of the following is/are the
1. Cambay Basin characteristics/characteristics of Indian
2. Cauvery Basin coal? (2013)

3. Krishna-Godavari Basin 1. High ash content

Select the correct answer using the code 2. Low sulphur content
given below. 3. Low ash fusion temperature
(a) 1 and 2 only Select the correct answer using the code
(b) 3 only given below:

(c) 2 and 3 only (a) 1 and 2 only

(d) 1, 2 and 3 (b) 2 only


(c) 1 and 3 only
(d) 1, 2 and 3

200 Mineral and Industries


unacademy.com | Download the Unacademy app
Give your feedback here: Link
Mineral and Industries-
6 Explanation
1. Answer: (d) decarbonization efforts.
A. Bauxite is a sedimentary rock with a relatively 2. Oil refineries: Green hydrogen can be
high aluminum content. It is the world’s main utilized in oil refineries as a clean energy
source of aluminum and gallium. Bauxite source for various processes. It can be
consists mostly of the aluminium minerals used in hydrogenation processes to
gibbsite (Al(OH)3), boehmite (Y-AlO(OH)) and produce cleaner fuels with lower sulfur
diaspore (α-AlO(OH)), mixed with the two content, as well as for desulfurization
iron oxides goethite (FeO(OH)) and haematite and hydrocracking operations. By
(Fe2O3), the aluminium clay mineral kaolinite incorporating green hydrogen, oil
(Al2Si2O5(OH)4) and small amounts of anatase refineries can reduce their reliance on
(TiO2) and ilmenite (FeTiO3 or FeO.TiO2). fossil fuels and minimize their carbon
B.Main sources of copper are Antimony, Azurite, footprint.
Bornite, Chalcocite, Chalcopyrite, Chrysocolla, 3. Steel plants: Green hydrogen has the
Copper extraction, Copper monosulfide, potential to revolutionize the steel
Copper(I) sulfide, Covellite, Cuprite, Digenite, industry by replacing the conventional
Dioptase, Malachite, Mineral, Tennantite, use of coal in blast furnaces. Currently,
Tetrahedrite ores. the production of steel through the
C.The iron is usually found in the form of traditional blast furnace route is a
magnetite (Fe3O4, 72.4% Fe), hematite (Fe2O3, significant source of carbon dioxide
69.9% Fe), goethite (FeO(OH), 62.9% Fe), emissions. By using green hydrogen
limonite (FeO(OH)·n(H2O), 55% Fe) or siderite as a reducing agent instead of coal, it
(FeCO3, 48.2% Fe). becomes possible to produce “green
steel” with substantially lower carbon
D. Ilmenite (FeO.TiO2 ) and rutile (TiO2 ) are emissions. This process, known as
the two chief minerals of titanium. Along direct reduction, has the potential
with other heavy minerals are important to significantly decarbonize the steel
constituents of beach sand deposits found industry.
right from Saurashtra coast (Gujarat) in the
west to Digha coast, West Bengal in the east.
3. Answer: (b)

2. Answer: (c) Statement 1 is correct: Monazite in general,


contains about 55 – 60% total Rare Earth Oxide.
1. Fertilizer plants: Green hydrogen can be
used as a clean source of energy for the Statement 2 is correct: Monazite is a mineral
production of fertilizers. Hydrogen is a mainly containing rare earths and thorium-a
key component in the manufacturing of prescribed substance to be handled by the
ammonia, which is used as a primary Department of Atomic Energy (DAE).
raw material in the production of Statement 3 is incorrect: Monazite doesn’t
nitrogen-based fertilizers. By utilizing occur naturally in the entire Indian coastal
green hydrogen instead of traditional sands in India.
hydrogen derived from fossil fuels, Statement 4 is correct: A licence from the
the fertilizer industry can reduce its Department of Atomic Energy (DAE) under the
carbon emissions and contribute to Atomic Energy (Working of the Mines. Minerals

Mineral and Industries-Explanation 201


unacademy.com | Download the Unacademy app
Give your feedback here: Link
and Handling of Prescribed Substances) Rules
1984 promulgated under the Atomic Energy Act 5. Answer: (d)
1962 is necessary for exporting monazite.
Bentonite is essentially a highly plastic clay
Indian Rare Earths Limited (IREL), a wholly containing not less than 85% clay mineral,
owned Public Sector Undertaking of the montmorillonite. It gets its name from Fort
Government of India (GOI) under DAE, is the Benton, America, the place where its presence
only entity which has been permitted to and usages were first discovered. The
produce and process monazite, and handle it Government of India declared Bentonite as
for domestic use as well as for export. ‘Minor Mineral’.
Elimination Technique: Monazite occurs Option (d) is correct: Chromite (Cr) is the single
naturally in the entire Indian coastal sands commercially viable ore of chromium which is
in India. This statement is too broad and chemically known as iron chromium oxide (Fe
extreme. Thus, Option (b) is correct. Cr2O4). Kyanite, Sillimanite and andalusite are
anhydrous aluminosilicate minerals that have
the same chemical formula Al2O3 but differ
4. Answer: (b) in crystal structure and physical properties.
Statement 1 is not correct: With the Chromite, Kyanite, and Sillimanite are major
establishment of the United Nations minerals.
Convention on the Law of the Sea in 1982 and
the entry into force of the International Seabed
Authority (ISA) in 1994, exploration activities 6. Answer: (a)
for mineral resources in the international Statement 1 is correct: As per the Mines and
waters are now regulated under exploration Minerals (Development and Regulation) Act,
contracts, allocated by the International 1957 “Minor Minerals” means building stones,
Seabed Authority (ISA). gravel, ordinary clay, ordinary sand other than
Statement 2 is correct: India has received sand used for prescribed purposes.
licences for seabed mineral exploration in Statement 2 is not correct: The power to frame
international waters in the Central Indian policy and legislation relating to minor minerals
Ocean Basin (CIOB). These rights are over 75000 is entirely delegated to the State Governments
sq. km of area in international waters allocated while policy and legislation relating to the
by the International Seabed Authority (ISA) major minerals are dealt by the Ministry of
for developmental activities for polymetallic Mines under the Central Government. Various
nodules. In 2017, India’s exclusive rights to State Governments have indeed prescribed
explore polymetallic nodules from the seabed rules for the grant of mineral concessions in
in the Central Indian Ocean Basin (CIOB) have respect of minerals classified as minor minerals
been extended by five years. under the MMDR Act, 1957. Minor Minerals get
Statement 3 is correct: The vast repository specified in the schedule appended in Minor
of minerals, including the precious cobalt, Mineral concession Rules issued by States.
zinc, manganese and rare earth materials Statement 3 is correct: As opposed to major
that are needed for smartphones, laptops minerals, the regulatory and administrative
and hybrid cars, is present in three forms of jurisdiction of minor minerals falls under the
ore—polymetallic manganese nodules that purview of State governments. These include
remain strewn across the ocean floor; cobalt- the powers to frame rules, prescribe rates
rich ferromanganese crusts that cover the of royalty, contribution to District Mineral
seamounts; and massive polymetallic sulphide Foundation, the procedure for grant of mineral
deposits around hydrothermal vents. concessions, regulation of their mining, control

202 Mineral and Industries-Explanation


unacademy.com | Download the Unacademy app
Give your feedback here: Link
of illegal mining etc. y Cambay Basin
y Gondwana Basin
7. Answer: (d) y Krishna-Godavari Basin
Statement 1 is not correct: As per the Mines y Cauvery Basin
and Minerals (Development and Regulation) y Indo-Gangetic Basin
Amendment Act, 2015 the non-coal mines
have to be auctioned by the respective State y Assam & Assam-Arakan Basin
governments.
Statement 2 is not correct: The largest 9. Answer: (c)
resources in terms of gold ore (primary) are Option (c) is correct: In order to support steel
located in Bihar (44%) followed by Rajasthan (25%), production, the required quantities of critical
Karnataka (21%), West Bengal (3%), Andhra Pradesh inputs are iron ore, coking and non-coking
(3%), Jharkhand (2 %). The remaining 2% resources coal. In India, the steel production industry
of ore are in Chhattisgarh, Madhya Pradesh, Kerala, requires the import of coking coal.
Maharashtra, and Tamil Nadu. Andhra Pradesh and
y The other raw materials besides iron ore
Jharkhand do have gold mines.
and coking coal, essential for iron and
Statement 3 is correct: About 2621 million steel industry are limestone, dolomite,
tons of resources of Iron–Ore (both Hematite manganese and fire clay. All these raw
& Magnetite) are estimated in Rajasthan. The materials are gross (weight losing),
Iron-Ore deposits are located in the district therefore, the best location for the iron
of Jaipur, Udaipur, Jhunjhunu, Sikar, Bhilwara, and steel plants is near the source of raw
Alwar, Bharatpur, Dausa and Banswara. The materials.
important localities of Iron-Ore are Morija-
y In India, there is a crescent-shaped region
Neemala (Jaipur), Lalsot (Dausa), Rampura,
comprising parts of Chhattisgarh, Northern
Dabla (Sikar), Taonda (Jhunjhunu), Pur-Banera,
Odisha, Jharkhand and western West
Bigod (Bhilwara), Nathara-Ki-Pal, Thur (Udaipur),
Bengal, which is extremely rich in high
Indergarh, Mohanpura (Bundi), Dedrauli, Liloti,
grade iron ore, good quality coking coal and
Todupura, Khora (Karauli).
other supplementing raw materials.

8. Answer: (d)
10. Answer: (d)
Shale gas refers to natural gas that is trapped
According to the Central Pollution Control
within shale formations. Shales are fine-
Board (CPCB) of India, about 500 billion cubic
grained sedimentary rocks that can be rich
metres of water out of the total available
sources of petroleum and natural gas. Over the
freshwater is used in industries annually. Out
past decade, the combination of horizontal
of this, about 10 billion cubic metres water is
drilling and hydraulic fracturing has allowed
used by processing industries and 30 billion
access to large volumes of shale gas that were
cubic metres is used for refrigeration purposes.
previously uneconomical to produce.
Option (d) is correct: Thermal power industries
Option (d) is correct: The following sedimentary
are the maximum consumer of water in india.
basins are considered prospective from Shale
oil and gas point of view:

Mineral and Industries-Explanation 203


unacademy.com | Download the Unacademy app
Give your feedback here: Link
11. Answer: (b) Naharkatiya and Moran-Hugrijan are the
Statement 1 is not correct: large reserves important oil fields in the state.
of natural gas have been discovered in the
Krishna-Godavari basin. Along the west coast, 12. Answer: (a)
the reserves of the Mumbai High and allied
fields are supplemented by finds in the Gulf Option 1 and option 2 are correct: Indian coals
of Cambay. Andaman and Nicobar Islands are are of high ash content due to the nature of
also important areas having large reserves formation of the coal deposits. Indian coals
of natural gas. Gondwana beds are the main have low sulphur content.
reserve of coal. y The behaviour of ash fusibility is an important
Statement 2 is correct: Mica deposits are factor for the calculation of the efficiency of
found in the northern edge of the Chota the boiler plant and gas producers.
Nagpur plateau. Koderma Gaya – Hazaribagh y Ash Fusion Temperature helps in estimating
belt of Jharkhand is the leading producer. the coal ash slagging behaviour. Slagging
In Rajasthan, the major mica producing area behaviour of the ash depends upon the
is around Ajmer. Nellore mica belt of Andhra constituents present in it, which are
Pradesh is also an important producer in the divided into acidic (SiO2, FeO, TiO2), basic
country. (CaO, MgO, Fe2O3, Na2O and K2O) and
Statement 3 is not correct: Mumbai High, amphoteric (Al2O3).
Gujarat and Assam are major petroleum y Higher the presence of acidic compounds
production areas in India. Ankleshwar is the in coal ash higher is the melting point.
most important field of Gujarat. Assam is the So Indian coals have a high ash fusion
oldest oil-producing state in India. Digboi, temperature. (Option 3 is not correct)

204 Mineral and Industries-Explanation


unacademy.com | Download the Unacademy app
Give your feedback here: Link
7 Agriculture in India

1. Consider the following States: (2022) (a) 1 only


1. Andhra Pradesh (b) 2 and 3 only
2. Kerala (c) 2 only
3. Himachal Pradesh (d) 1, 2 and 3
4. Tripura
How many of the above are generally known 4. With reference to the cultivation of Kharif
as tea-producing States? crops in the last five years, consider the
(a) Only one State following statements: (2019)

(b) Only two States 1. Area under rice cultivation is the highest.

(c) Only three States 2. Area under cultivation of jowar is more


than that of oilseeds.
(d) All four States
3. Area of cotton cultivation is more than
that of sugarcane.
2. "The crop is subtropical in nature. A hard 4. Area under sugarcane cultivation has
frost is injurious to it. It requires at least 210 steadily decreased.
frost -free days and 50 to 100 centimetres
of rainfall for its growth. A light well- Which of the statements given above is/are
drained soil capable of retaining moisture correct?
is ideally suited for the cultivation of the (a) 1 and 3 only
crop." Which one of the following is that (b) 2, 3 and 4 only
crop? (2020)
(c) 2 and 4 only
(a) Cotton
(d) 1, 2, 3 and 4
(b) Jute
(c) Sugarcane
5. Consider the following pairs: (2014)
(d) Tea
Region Well-known for the
production of
3. With reference to pulse production in India,
1. Kinnaur Areca nut
consider the following statements: (2020)
2. Mewat Mango
1. Black gram can be cultivated as both
kharif and rabi crops. 3. Coromandel Soya bean

2. Green-gram alone accounts for nearly Which of the above pairs is/are correctly
half of pulse production. matched?

3. In the last three decades, while (a) 1 and 2 only


the production of kharif pulses has (b) 3 only
increased the production of rabi pulses (c) 1, 2 and 3
has decreased.
(d) None
Which of the statements given above is/are
correct?

Agriculture in India 205


unacademy.com | Download the Unacademy app
Give your feedback here: Link
6. Consider the following pairs: (2014) 8. Improper handling and storage of cereal
Programme/Project Ministry grains and oilseeds result in the production
of toxins known as aflatoxins which are
1. Drought-Prone Area Programme not generally destroyed by normal cooking
Ministry of Agriculture process. Aflatoxins are produced by (2013)
2. Desert Development Programme (a) bacteria
Ministry of Environment and Forests
(b) protozoa
3. National Watershed Development
Project for Rainfed Areas Ministry of (c) moulds
Rural Development (d) viruses
Which of the above pairs is/are correctly
matched? 9. Consider the following crops: (2013)
(a) 1 and 2 only 1. Cotton
(b) 3 only 2. Groundnut
(c) 1, 2 and 3 3. Rice
(d) None 4. Wheat
Which of these are Kharif crops?
7. In the context of food and nutritional (a) 1 and 4
security of India, enhancing the 'Seed
Replacement Rates' of various crops (b) 2 and 3 only
helps in achieving the food production (c) 1, 2 and 3
targets of the future. But what is/are the (d) 2, 3 and 4
constraints/constraints in its wider/greater
implementation? (2014)
10. Consider the following crops of India:
1. There is no National Seeds Policy in
(2012)
place.
1. Groundnut
2. There is no participation of private
sector seed companies in the supply of 2. Sesamum
quality seeds of vegetables and planting 3. Pearl millet
materials of horticultural crops.
Which of the above is / are predominantly
3. There is a demand-supply gap regarding rainfed crop/crops?
quality seeds in case of low value and
(a) 1 and 2 only
high-volume crops.
(b) 2 and 3 only
Select the correct answer using the code
given below: (c) 3 only
(a) 1 and 2 only (d) 1, 2 and 3
(b) 3 only
(c) 2 and 3 only 11. Consider the following crops of India:
(2012)
(d) None
1. Cowpea
2. Green gram

206 Agriculture in India


unacademy.com | Download the Unacademy app
Give your feedback here: Link
3. Pigeon pea (c) Karnataka
Which of the above is/are used as pulse, (d) Tamil Nadu
fodder and green manure?
(a) 1 and 2 only 15. With reference to micro-irrigation, which
(b) 2 only of the following statements is/are correct?
(c) 1 and 3 only (2011)

(d) 1, 2 and 3 1. Fertiliser/nutrient loss can be reduced.


2. It is the only means of irrigation in dry
land farming.
12.
Which of the following is the chief
characteristic of ‘mixed farming’? (2012) 3. In some areas of farming, receding of
the groundwater table can be checked.
(a) Cultivation of both cash crops and food
crops Select the correct answer using the code
given below:
(b) Cultivation of two or more crops in the
same field (a) 1 only

(c) Rearing of animals and cultivation of (b) 2 and 3 only


crops together (c) 1 and 3 only
(d) None of the above (d) 1, 2 and 3

13. Among the following states, which one has 16. The lower Gangetic plain is characterised
the most suitable climatic conditions for by a humid climate with high temperature
the cultivation of a large variety of orchids throughout the year. Which one among the
with minimum cost of production, and can following pairs of crops is most suitable
develop an export-oriented industry in this for this region? (2011)
field? (2011) (a) Paddy and cotton
(a) Andhra Pradesh (b) Wheat and Jute
(b) Arunachal Pradesh (c) Paddy and Jute
(c) Madhya Pradesh (d) Wheat and cotton
(d) Uttar Pradesh

17. Salinization occurs when the irrigation


14.
A state in India has the following water accumulated in the soil evaporates,
characteristics: (2011) leaving behind salts and minerals. What
1. Its northern part is arid and semi-arid. are the effects of Salinization on irrigated
land? (2011)
2. Its central part produces cotton.
(a) It greatly increases the crop production
3. Cultivation of cash crops is predominant
over food crops. (b) It makes some soils impermeable

Which one of the following states has all of (c) It raises the water table
the above characteristics? (d) It fills the air spaces in the soil with
(a) Andhra Pradesh water

(b) Gujarat

Agriculture in India 207


unacademy.com | Download the Unacademy app
Give your feedback here: Link
7 Agriculture in India-Explanation

1. Answer:(d) cultivated in kharif and rabi seasons. Coconut


Tea cultivation is an example of plantation is the leading plantation crop grown. Pulses
agriculture. It is also an important beverage (black gram and lentil) and oilseed crops
crop introduced in India initially by the British. (sunflower and groundnut) are grown after rice
The tea plant grows on residual moisture in the soil. Hence, black
gram can be cultivated as both kharif and rabi
well in tropical and sub-tropical climates crops.
endowed with deep and fertile well-drained
soil, rich in humus and organic matter. Statement 2 is not correct: Green gram grown
in nearly 8 percent of the total pulse area of
Option (d) is correct: Major tea producing the country. India’s pulses production is driven
states are Assam, hills of Darjeeling and by chickpea and pigeon pea. The contribution
Jalpaiguri districts, of chickpea in total production is around 48
West Bengal, Tamil Nadu and Kerala. per cent, followed by pigeon pea at 17 per cent.
Apart from these, Himachal Pradesh, Other pulses include moong (nine per cent),
Uttarakhand, Meghalaya, Andhra Pradesh and urad (11 per cent) and others.

Tripura are also tea-producing states in the Statement 3 is not correct: The country’s
country. pulses production stood at 234 lakhs tonnes
in the 2018-19 crop year, down from the
record level of 254 lakh tonnes in 2017-18.
2. Answer: (a) Due to MSP increases, some improvement in
India is believed to be the original home of the public procurement and buffer stocks norms
cotton plant. Cotton is one of the main raw announced, India’s pulses production saw a big
materials for the cotton textile industry. In increase in 2016-17 to 231 lakhs tonnes from
2008 India was the second largest producer of 163 lakh tonnes in 2015-16, the year that saw
cotton after China. a spike in prices. The government policy over
the last few years was guided by an intent to
Option (a) is correct: Cotton grows well in
reduce import dependence (which was as high
drier parts of the black cotton soil of the
as around 40% of the domestic consumption).
Deccan plateau. It is subtropical in nature. It
In the last three decades, the production of
requires high temperature, rainfall of 50-100
kharif pulses has decreased due to monsoon
centimetres, or irrigation, 210 frost-free days
delay and production of rabi pulses has
and bright sun-shines for its growth. A light
increased.
well-drained soil capable of retaining moisture
is desirable for its growth. Elimination Technique : We know that
It is a kharif crop and requires six to eight months Chickpea and Pigeon pea are major pulses
to mature. Major cotton-producing states that are consumed in India. So, Statement
are– Maharashtra, Gujarat, Madhya Pradesh, 2 is not correct. Thus, Option (a) is correct.
Karnataka, Andhra Pradesh, Telangana, Tamil
Nadu, Punjab, Haryana, and Uttar Pradesh.
4. Answer: (a)
As per the annual report of Ministry of
3. Answer: (a) Agriculture, area, production and yield of major
Statement 1 is correct: Rice is dominantly crops are as follow:

208 Agriculture in India-Explanation


unacademy.com | Download the Unacademy app
Give your feedback here: Link
Statement 1 is correct: With reference to the Andhra Pradesh, Karnataka, Chhattisgarh, and
cultivation of Kharif crops in the last five years, Gujarat contribute the remaining 11 per cent
the area under rice cultivation is the highest. production. Coromandel Coast, broad coastal
Statement 2 is not correct: Area under Coarse plain in eastern Tamil Nadu state, southern
cereals is less than area under oilseeds. Jowar India.
is a coarse cereal.
Statement 3 is correct: Area of cotton 6. Answer: (d)
cultivation (108.45 lakh hectare in 2016-17) Pair 1 is not correctly matched: Drought-
is more than that of sugarcane (43.89 lakh Prone Area Programme is implemented by
hectare in 2016-17). the Department of Land Resources, Ministry
Statement 4 is not correct. In 2013-14 to 2014- of Rural Development. The basic objective of
15, area under sugarcane cultivation increased the programme is to minimise the adverse
while it decreased for the next two years. But effects of drought on production of crops
during the last five years, it has not decreased and livestock and productivity of land, water
steadily. and human resources ultimately leading to
drought proofing of the affected areas. The
Elimination Technique: India is one of programme also aims to promote overall
largest consumers of oil due to its huge economic development and improve the
population. Jowar is a coarse cereal socio-economic conditions of the resource
whose consumption is quite low in India poor and disadvantaged sections inhabiting
compared to other cereal crops. So, the programme areas.
Statement 2 is not correct. Thus, Option
(a) is correct. Pair 2 is not correctly matched: Desert
Development Programme (DDP) is implemented
by the Department of Land Resources, Ministry
5. Answer: (d) of Rural Development. DDP aims to minimise
Pair 1 is not correctly matched: The areca nut the adverse effect of drought and control
palm is the source of a common chewing nut, desertification through rejuvenation of the
popularly known as betel nut or Supari. India natural resource base of the identified desert
is the largest producer of areca nuts and at areas. The scheme is funded based on 75:25 by
the same time largest consumer also. Major the Central and the State government.
states cultivating this crop are Karnataka Pair 3 is not correctly matched: National
(40%), Kerala (25%), Assam (20%), Tamil Nadu, Watershed Development Project in Rainfed
Meghalaya, and West Bengal. Kinnaur is in Areas (NWDPRA) implemented by Department
Himachal Pradesh. of Agriculture & Cooperation, Ministry of
Pair 2 is not correctly matched: India ranks Agriculture.
first among world’s mango producing countries y NWDPRA is a special central assistance
accounting for about 50% of the world’s programme for the benefit of cultivator’s
mango production. About 80% of Mango is families living below poverty line through
commercially cultivated in Andhra Pradesh, treatment of arable and non-arable land,
West Bengal, Karnataka, Kerala, Bihar, Uttar drainage line treatment, creation of water
Pradesh and Uttarakhand. Mewat is in Haryana. bodies, development of agriculture/
Pair 3 is not correctly matched: Production of horticulture crops, forestry and land based/
soya bean in India is dominated by Maharashtra household production system as packages
and Madhya Pradesh which contribute 89 of rehabilitation components.
per cent of the total production. Rajasthan,

Agriculture in India-Explanation 209


unacademy.com | Download the Unacademy app
Give your feedback here: Link
7. Answer: (b) Statement 2 is not correct: The National Seed
Seed Replacement Rate is the percentage of Policy, 2002 led to the opening up of FDI in
area sown out of the total area of crop planted seeds. This led to high participation of private
in the season by using certified/quality seeds sector seed companies in the supply of quality
other than the farm saved seed. In simple seeds of vegetables and planting materials of
terms it is a measure of cropped area covered horticulture crops.
with quality seed. Enhancement in Seed Statement 3 is correct: There is a huge gap
Replacement Rate (SRR) SRR is a measure of between the requirement and supply of the
cropped area covered with quality seed. Hence seeds. Seed firms which provide quality seeds
SRR is having a direct bearing on productivity are not able to diffuse into the seed market
augmentation and enhancement in farmers’ due to the cheaper seeds supplied by the
income and is one of the means for achieving unorganised sector.
doubling the farmers’ income.
Elimination Technique: Quality Seeds
Statement 1 is not correct: Prior to the National are an important input to achieve food
Seed Policy 2002, the Government of India had production targets. It is obvious that
adopted a “New Policy on Seed Development” the Indian government would have a
in 1988 which has been evolved with special seed policy in 2014 and private sector
emphasis on the import of high-quality seeds, participation as a part of policy. Thus,
a time bound-programme to strengthen/ statements 1 and 2 are not correct. Thus,
modernise plant quarantine facilities, effective Option (b) is correct.
observance of procedures for quarantine/
post entry quarantine (PEQ), and incentives
to encourage the domestic seed industry. But 8. Answer: (c)
this policy was flawed due to nonexistence Option (c) is correct: Aflatoxins are poisonous
of IPR laws and several restrictions on import substances produced by certain kinds of fungi
and exports. Thus, the National Seed Policy (moulds) that are found naturally all over the
2002 was launched to – provide intellectual world; they can contaminate food crops and
property protection to new varieties; usher this pose a serious health threat to humans and
sector into planned development; protect the livestock. Aflatoxins also pose a significant
interest of farmers and encourage conservation economic burden, causing an estimated 25% or
of agro-biodiversity. This policy had ten thrust more of the world’s food crops to be destroyed
areas as follows: annually.
y Varietal Development and Plant Varieties Long-term or chronic exposure to aflatoxins
Protection has several health consequences including:
y Seed Production y They are potent carcinogens and may affect
y Quality Assurance all organ systems, especially the liver and
y Seed Distribution and Marketing. kidneys; they cause liver cancer.

y Infrastructure facilitiesBHNS y They are mutagenic in bacteria (affect the


DNA), genotoxic, and have the potential to
y Transgenic Plant Varieties cause birth defects in children. Children may
y Import of seeds and planting material become stunted, although these data have
y Export of seeds yet to be confirmed because other factors
also contribute to growth faltering, e.g. low
y Promotion of Domestic Seed
socioeconomic status, chronic diarrhoea,
y Strengthening of monitoring system infectious diseases, malnutrition;

210 Agriculture in India-Explanation


unacademy.com | Download the Unacademy app
Give your feedback here: Link
y They cause immunosuppression, therefore crops in India grown mostly under rain-fed
may decrease resistance to infectious conditions. Gujarat, Andhra Pradesh, Tamil
agents (e.g. HIV, tuberculosis). Nadu, Karnataka, Maharashtra, Rajasthan,
Madhya Pradesh, Orissa, and Uttar Pradesh are
the major Groundnut growing states.
9. Answer: (c)
Sesamum or Sesame is the oldest indigenous
Option (c) is correct: Kharif crops are grown
oilseed crop in India. India ranks first in the
with the onset of monsoon in different parts
world with 19.47 Lakh ha area and 8.66 Lakh
of the country, and these are harvested in
tonnes production. The average yield of sesame
September-October. Important crops grown
(413 kg/ha) in India is low as compared with
during this season are rice (paddy), maize,
other countries in the world (535 kg / ha). The
jawar, bajra, tur (arhar), moong, urad, cotton,
main reasons for low productivity of sesame
jute, groundnut and soybean.
are its rainfed cultivation in marginal and
y Some of the most important rice-growing submarginal lands under poor management
regions are Assam, West Bengal, coastal and input starved conditions. Some of the
regions of Odisha, Andhra Pradesh, Sesame growing States are Gujarat, Madhya
Telangana, Tamil Nadu, Kerala and Pradesh, Maharashtra, etc.
Maharashtra, particularly the (Konkan
Pearl millet is the major millet occupying first
coast) along with Uttar Pradesh and Bihar.
position among all the millets in India followed
y Recently, paddy has also become an by Sorghum and Ragi. Pearl millet occupies the
important crop of Punjab and Haryana. In prime position in the rainfed farming in arid
states like Assam, West Bengal and Odisha, and semi-arid regions of India.
three crops of paddy are grown in a year.
These are Aus, Aman and Boro.
11. Answer: (a)
Option (a) is correct: Green manure is a type
10. Answer: (d)
of organic fertiliser where an entire fresh plant
Option (d) is correct: In India, Groundnut, or plant part (e.g., leaf) is directly used as
Sesamum and Pearl millet- all these are rainfed manure in agricultural lands, without any prior
crops, grown in several Indian States. Nearly decomposition or composting. Pulses are the
80% of the area sown to groundnuts is rain edible seeds of plants in the legume family.
fed and relies entirely on summer monsoon fodder crops are the agricultural crops that
rainfall. In India Sesamum is mainly a rain fed can be used to feed livestock. Cow pea and
crop and is grown on marginal, dry and sloppy green gram can be used as pulse, fodder and
lands. The main rain fed cereals grown in India green manure.
are sorghum (Ragi).
Cowpea (Vigna unguiculata) is also known as
Groundnut or Peanut (Arachis hypogaea) is black-eyed pea or southern pea etc. and has
a species in the legume or “bean” family. multiple uses like food, feed, forage, fodder,
The peanut was probably first domesticated green manuring and vegetable. It is known
and cultivated in the valleys of Paraguay. as drought hardy nature, its wide and droopy
Groundnut is the major oil seed crop in leaves keeps soils and soil moisture conserved
India, and it plays a major role in bridging the due to shading effect. Cowpea is essentially
vegetable oil deficit in the country. Groundnuts a warm weather crop adapted to tropics and
in India are available throughout the year due subtropics. It tolerates heat and dry conditions
to a two-crop cycle harvested in March and and can be successfully grown even in regions
October. Ground Nuts are important protein with low rainfall. In Indian context, it is a minor

Agriculture in India-Explanation 211


unacademy.com | Download the Unacademy app
Give your feedback here: Link
pulse cultivated mainly in arid and semi arid conditions (temperature not less than 16-17oC),
tracts grown in pockets of Punjab, Haryana, they should last eight weeks with flowers,
Delhi, and West UP along with considerable after which the blooms slowly start dropping
area in Rajasthan, Karnataka, Kerala, Tamil off. Among all the states, Arunachal Pradesh
Nadu, Maharashtra and Gujarat. has the most suitable climatic conditions for
Green gram is popularly known as “Moong the cultivation of a large variety of orchids with
Dal “in India and is basically a tiny circular minimum cost of production and can develop
shaped bean that is green in colour. Green an export-oriented industry in this field.
gram is one of the main pulse crops in India.
It requires a hot and warm climate. Green 14. Answer: (b)
gram is considered to be hardiest among all
pulse crops and can tolerate drought to a great Statement 1 is correct: The northern arid
extent. Green gram is also used as a green regions in India comprised largely the desert of
manure and fodder crop. Green gram is mainly Rajasthan, the Rann of Kutch and the semi-arid
cultivated in states of Madhya Pradesh, Punjab, regions of Punjab and Gujarat. The Southern
Haryana, Maharashtra Rajasthan, Karnataka, arid regions are in the rain shadow of the
Andhra Pradesh, and Tamil Nadu. Western Ghats covering states of Maharashtra,
Karnataka and Tamil Nadu.
Statement 2 is correct: India is the country
12. Answer: (c) to grow all four species of cotton. Gossypium
Option (c) is correct: Mixed farming is a system Hirsutum represents 88% of the hybrid cotton
of farming in agriculture, which involves the production in India and all the current BT
growing or cultivation of crops and rearing of cotton hybrids are Gossypium Hirsutum. In
animals (livestock) on the same piece of land, India, majority of the cotton production comes
on a particular season. This system of farming from nine major cotton growing states, which
is practised all around the world, it helps to are grouped into three diverse agro-ecological
ensure sustainable agriculture. A mixed farm zones, as under:
may grow cereal like maize, wheat and also y Northern Zone: Punjab, Haryana, and
rear cattle, pig and poultry. Rajasthan
A few advantages of mixed farming are: y Central Zone: Gujarat, Maharashtra, and
y In mixed farming, all products are utilised. Madhya Pradesh
y Money is saved as there is reduced cost of y Southern Zone: Telangana, Andhra Pradesh,
feeding farm animals and reduced or no and Karnataka
cost of fertiliser application. Statement 3 is correct: The economy of
y If a crop does poorly in a season, the farmer Gujarat is largely dependent upon agriculture.
can fall back on income from sales of meat, More than 50 percent of the total available
milk and eggs. production is cash crop. The main food
crops are bajra, jowar, rice and wheat. Major
commercial crops or cash crops are groundnut,
13. Answer: (b)
tobacco and cotton, linseed, sugarcane, etc.
Option (b) is correct: There are more than Gujarat is the state where cultivation of cash
1,300 species of orchids found in India, crops is predominant over food crops.
primarily in the Northeast: Assam, Meghalaya,
and Arunachal Pradesh and also in Sikkim and
south India. Most orchids are perennial herbs 15. Answer: (c)
with simple leaves and if raised in the right Micro-irrigation is the process of slow

212 Agriculture in India-Explanation


unacademy.com | Download the Unacademy app
Give your feedback here: Link
application of water to the soil so that moisture to drip or sprinkler irrigation systems (micro-
is retained in the upper layer rather than irrigation).
percolating underground or causing surface This will help in conserving groundwater as
run-off. well as save on the cost of irrigation. Using
Statement 1 is correct: Micro-irrigation can such micro-irrigation systems has also been
increase yields and decrease water, fertiliser, linked to an increase in the yield of crops.
and labour requirements. Using Micro-irrigation
fertiliser or nutrient loss can be reduced as
less water is applied hence fertilisers are easily 16. Answer: (c)
absorbed into plants. Option (c) is correct: The low and deltaic plains
Moreover, by applying water directly to the of the Ganges and Brahmaputra rivers that
root zone, the practice reduces loss of water are characterised by humid climate with high
through conveyance, run-off, deep percolation, temperature throughout the year. Paddy and
and evaporation. Jute is the most suitable crops for this region.

Statement 2 is not correct: It is not the only Paddy is the first-ranking crop in the vast
means of irrigation in dry land farming. Micro region stretching from lower Gangetic Plain to
Irrigation can be applied to any form of land Brahmaputra Valley in the east and the circum-
because it saves water in Irrigation. coastal alluvial tracts of the Peninsula region.
Paddy cultivation has a complete dominance
Statement 3 is correct: Agriculture consumes over all fertile plains around Bay of Bengal.
the largest share of groundwater in India - the
biggest user of groundwater in the world. The
past few decades have witnessed an alarming 17. Answer: (b)
depletion of groundwater resources in the Option (b) is correct: Salinization makes it
country. more difficult for plants to absorb soil moisture
While almost half of the agricultural area making it impermeable. It has detrimental
cultivating food grains in India depends on effects on plant growth and yield. Sea levels
irrigation, as high as 65 percent of these irrigated are rising, which further accelerates the
land holdings depend on groundwater to meet process of salinization due to climate change.
their water needs. It has been recommended Vast areas of farmland are lost every day due
that farmers should move from flood irrigation to salinization, land that is desperately needed
to feed the world’s growing population.

Agriculture in India-Explanation 213


unacademy.com | Download the Unacademy app
Give your feedback here: Link
8 Indian Map

1. Consider the following pairs : (2023) (d) saline lakes

Port Well Known as


1. Kamarajar Port : First major port in 4. What is common to the places known as
India registered as Aliyar, Isapur and Kangsabati? (2019)
a company (a) Recently discovered uranium deposits
2. Mundra Port : Largest privately (b) Tropical rain forests
owned port in India
(c) Underground cave systems
3. Visakhapatnam Largest container
(d) Water reservoirs
Port : port in India
How many of the above pairs are correctly
matched? 5. Consider the following statements: (2018)

(a) Only one pair 1. The Barren Island volcano is an active


volcano located in Indian territory.
(b) Only two pairs
2. Barren Island lies about 140 km east of
(c) All three pairs
Great Nicobar.
(d) None of the pairs
3. The last time the Barren Island volcano
erupted was in 1991 and it has remained
2. Consider the following pairs: (2022) inactive since then.
Wetland/Lake Location Which of the statements given above is/are
correct?
1. Hokera Wetland Punjab
(a) 1 only
2. Renuka Wetland Himachal Pradesh
(b) 2 and 3
3. Rudrasagar Lake Tripura
(c) 3 only
4. Sasthamkotta Lake Tamil Nadu
(d) 1 and 3
How many pairs given above are correctly
matched?
(a) Only one pair 6. Which one of the following is an artificial
lake? (2018)
(b) Only two pairs
(a) Kodaikanal (Tamil Nadu)
(c) Only three pairs
(b) Kolleru (Andhra Pradesh)
(d) All four pairs
(c) Nainital (Uttarakhand)
(d) Renuka (Himachal Pradesh)
3. With reference to India, Didwana,
Kuchaman, Sargol and Khatu are the names
of (2021) 7. Among the following cities, which one lies
(a) glaciers on a longitude closest to that of Delhi?
(2018)
(b) mangrove areas
(a) Bengaluru
(c) Ramsar sites

214 Indian Map


unacademy.com | Download the Unacademy app
Give your feedback here: Link
(b) Hyderabad 11. With reference to 'Changpa' community of
(c) Nagpur India, consider the following statements:
(2014)
(d) Pune
1. They live mainly in the State of
Uttarakhand.
8. If you travel by road from Kohima to 2. They rear the Pashmina goats that yield
Kottayam, what is the minimum number fine wool.
of States within India through which you
can travel, including the origin and the 3. They are kept in the category of
destination? (2017) Scheduled Tribes.

(a) 6 Which of the statements given above is/are


correct?
(b) 7
(a) 1 only
(c) 8
(b) 2 and 3 only
(d) 9
(c) 3 only
(d) 1, 2 and 3
9. At one of the places in India, if you stand
on the seashore and watch the sea, you
will find that the seawater recedes from 12. Which one of the following pairs of islands
the shoreline a few kilometres and comes is separated from each other by the 'Ten
back to the shore, twice a day, and you can Degree Channel'? (2014)
actually walk on the seafloor when the (a) Andman and Nicobar
water recedes. This unique phenomenon is
seen at (2017) (b) Nicobar and Sumatra

(a) Bhavnagar (c) Maldives and Lakshadweep

(b) Bheemunipatnam (d) Sumatra and Java

(c) Chandipur
(d) Nagapattinam 13. Consider the following towns of India:
(2014)
1. Bhadrachalam
10. Consider the following pairs: (2015)
2. Chanderi
Place of Pilgrimage Location
3. Kancheepuram
1. Srisailam Nallamala Hills
4. Karnal
2. Omkareshwar Satmala Hills
Which of the above are famous for the
3. Pushkar Mahadeo Hills production of traditional sarees/fabric?
Which of the above pairs is/are correctly (a) 1 and 2 only
matched?
(b) 2 and 3 only
(a) 1 only
(c) 1, 2 and 3 only
(b) 2 and 3 only
(d) 1, 3 and 4 only
(c) 1 and 3 only
(d) 1, 2 and 3
14. Consider the following pairs: (2014)
1. Dampa Tiger Reserve Mizoram

Indian Map 215


unacademy.com | Download the Unacademy app
Give your feedback here: Link
2. Gumti Wildlife Sanctuary Sikkim 1. Nokrek Biosphere Garo Hills
3. Saramati Peak Nagaland Reserve
Which of the above pairs is/are correctly 2. Logtak (Loktak) Lake Barail Range
matched? 3. Namdapha National Dafla Hills
(a) 1 only Park
Which of the above pairs is/are correctly
(b) 2 and 3 only
matched?
(c) 1 and 3 only
(a) 1 only
(d) 1, 2 and 3
(b) 2 and 3 only
(c) 1, 2 and 3
15. Consider the following pairs: (2014)
(d) None
National Cities connected
Highway
18. Consider the following pairs: (2013)
1. NH 4 Chennai and Hyderabad

Tribe State
2. NH 6 Mumbai and Kolkata
1. Limboo (Limbu) Sikkim
3. NH 15 Ahmedabad and Jodhpur
2.
Karbi Himachal Pradesh
Which of the above pairs is/are correctly
matched? 3. Dongaria Kondh Odisha

(a) 1 and 2 only 4.


Bonda Tamil Nadu

(b) 3 only Which of the above pairs are correctly


matched?
(c) 1, 2 and 3
(a) 1 and 3 only
(d) None
(b) 2 and 4 only
(c) 1, 3 and 4 only
16. Consider the following pairs: (2013)
(d) 1, 2, 3 and 4
National Park River flowing through the
park
1. Corbett National Park Ganga 19. A particular State in India has the following
characteristics: (2012)
2. Kaziranga National Park Manas
1. It is located on the same latitude which
3. Silent Valley National Park Kaveri
passes through northern Rajasthan.
Which of the above pairs is/are correctly
2. It has over 80% of its area under forest
matched?
cover.
(a) 1 and 2 only
3. Over 12% of forest cover constitutes the
(b) 3 only Protected Area Network in this State.
(c) 1 and 3 only Which one among the following States has
(d) None all the above characteristics?
(a) Arunachal Pradesh

17. Consider the following pairs: (2013) (b) Assam


(c) Himachal Pradesh
(d) Uttarakhand

216 Indian Map


unacademy.com | Download the Unacademy app
Give your feedback here: Link
8 Indian Map-Explanation

1. Answer: (b) 4. SASTHAMKOTTA LAKE: KERALA


Pair one is correct: Kamarajar Port: The port
was declared as a major port under the Indian 3. Answer: (d)
Ports Act, 1908 in March 1999 and incorporated
Option (d) is correct: A salt lake or saline
as Ennore Port Limited under the Companies
lake is a landlocked body of water that has
Act, 1956 in October 1999. The Kamarajar Port
a concentration of salts (typically sodium
is the only corporatized major port and is
chloride) and other dissolved minerals
registered as a company.
significantly higher than most lakes (often
Pair two is correct: Mundra Port - It is India’s defined as at least three grams of salt per
biggest commercial port by volume, has litre). The Didwana (Rajasthan), Kuchaman
overtaken state-run Jawaharlal Nehru Port (Rajasthan), Sargol (Rajasthan) and Khatu
Trust (JNPT) to become the country’s largest (Rajasthan) are the names of some of the
container gateway by handling 5.65 million important saltwater lakes in India.
twenty-foot equivalent units (TEUs) in FY21. It
is Largest privately owned port in India.
4. Answer: (d)
Pair three is not correct: Jawaharlal Nehru Port
Trust –(JNPT) Known as Nhava Sheva, JNPT is Option (d) is correct: Aliyar, Isapur and
the largest container port in India. Kangsabati are related to water reservoirs.
**India has 12 major ports -- Deendayal y The Kangsabati Reservoir Project is a
(Kandla), Mumbai, Mormugao, New Mangalore, project started in the state of West Bengal
Cochin, Chennai, Ennore (Kamarajar), Tuticorin in 1956 as part of the Second Five-year Plan
(V O Chidambaranar), Visakhapatnam, Paradip to provide water to 3,484.77 km² of land in
and Kolkata (including Haldia) and Jawaharlal the districts of Paschim Medinipur, Purba
Nehru Port. Medinipur, Bankura, and Hooghly.
Cargo traffic at the country’s major ports y Aliyar Reservoir is a reservoir located
increased 11.79 per cent to 65.45 million tonnes in Aliyar village near Pollachi town in
in February 2023 from 58.55 million tonnes in Coimbatore District, Tamil Nadu, South
the same month a year ago, according to Indian India. The dam is located in the foothills
Ports Association (IPA). The positive growth in of Valparai, in the Anaimalai Hills of the
cargo traffic was recorded by all major ports Western Ghats.
except Visakhapatnam, Kamarajar and Chennai y Isapur reservoir is an earthfill dam on
Ports Penganga river near Kalamnuri in the state
of Maharashtra in India. It is one of the
biggest dams in Maharashtra. This dam is
2. Answer: (b)
ranked at 4th position having around 3.5
Option (b) is correct: km long earthwall.
Only 2 pairs are correctly matched.
1. HOKERA WETLAND: JAMMU AND KASHMIR 5. Answer: (a)
2. RENUKA WETLAND: HIMACHAL PRADESH Statement 1 is correct: Andaman and Nicobar
3. RUDRASAGAR LAKE-: TRIPURA Islands are separated by a water body which
is called the Ten-degree channel. It is believed

Indian Map-Explanation 217


unacademy.com | Download the Unacademy app
Give your feedback here: Link
is one of the finest man-made (artificial) lakes
that these islands are an elevated portion of in India, is located in the state of Tamil Nadu.
submarine mountains. However, some smaller
Kolleru lake, one of the largest freshwater
islands are volcanic in origin. Barren island, the
lakes in India, (it was designated a sanctuary in
only active volcano in India is also situated in
October 1999) is situated between the Krishna
the Nicobar Islands.
and West Godavari districts of Andhra Pradesh.
Statement 2 is not correct: The Barren Island Kolleru is one of the most important wetlands
lies about 140 Km from Port Blair. of India. The atapaka Bird Sanctuary, part of the
Statement 3 is not correct: The last time the Kolleru Lake, has been identified as the world’s
Barren Island volcano erupted was in 2018. It largest home for the spot-billed pelican. It is a
was a stratovolcano composed of lava, rock Ramsar site.
fragments, and volcanic ash. On the west side Naini Lake is the sole source of drinking
of the island is a caldera formed by an explosive water for Nainital town, an important tourist
eruption in the Pleistocene era. destination in Uttarakhand state. It is a natural
lake.
Renuka lake is situated in the Sirmour district
of Himachal Pradesh in India. One of the
popular temples of Himachal Pradesh, Renukaji
is situated on the bank of this lake. It is a
Ramsar site.

7. Answer: (a)
Option (a) is correct: The longitude of Delhi is
77.1025° E. Bengaluru with longitude 77.5946°
E lies on the longitude closest to that of Delhi.
Longitude of Hyderabad: 78.4867° E
Longitude of Nagpur: 79.0882° E
Longitude of Pune: 73.8567° E
Parallels of Latitude: The parallels of latitude
refer to the angular distance, in degrees,
minutes and seconds of a point north or south
of the Equator. Lines of latitude are often
referred to as parallels.
Meridians of Longitude: The meridians of
Elimination Technique: The first recorded
longitude refer to the angular distance, in
eruption of the volcano dates back to 1787.
degrees, minutes, and seconds, of a point east
Since then, the volcano has erupted more
or west of the Prime (Greenwich) Meridian.
than ten times, with the most recent one
Lines of longitude are often referred to as
being in 2020. Statement 3 is eliminated.
meridians.
Thus, Option (a) is correct.

8. Answer: (b)
6. Answer: (a)
Option (b) is correct: Kohima is the capital of
Option (a) is correct: Kodaikanal Lake, which
Nagaland and Kottayam is a city in the Indian

218 Indian Map-Explanation


unacademy.com | Download the Unacademy app
Give your feedback here: Link
state of Kerala. If we travel by road from Kohima The beach is widely known for being the
to Kottayam, the shortest route will pass base to explore ruins of a Dutch Fort
through the minimum seven states. These are: and cemetery. Bheemunipatnam was an
Nagaland-Assam-West Bengal-Odisha-Andhra important Buddhist centre. Evidence of
Pradesh-Tamil Nadu-Kerala. Buddhism dating back to the second century
has been found on the Pavuralakonda
hillock, which surrounds the beach.
y Nagapattinam Beach is located in
Tamilnadu. The towering lighthouse is the
most prominent attraction of Nagapattinam
beach.

10. Answer: (a)


Pair 1 is correctly matched: Srisailam is located
in the Nallamala Hills. ‘Srisailam’ the abode of
Siva and Sakthi is on the right side of the river
Krishna in Kurnool district of Andhra Pradesh.
The temple at Srisailam is the ancient and
sacred place of South India.
Pair 2 is not correctly matched: Omkareshwar
9. Answer: (c) temple in Mandhata hills is located on a pranva
shaped island at Narmada. Mamaleshwar
Option (c) is correct: The Chandipur beach (ancient name Amareshwar) temple is located
is located in the Balasore district of Odisha. on the south coast. One of the 12 Jyotirlingas
During low tides, the beach recedes up to 5 of Omkar-Mamaleswar and sidhdhawar kut of
KM offering visitors the opportunity to walk on Jain are located in this place.
the seabed. During the ebb, one can see the
water receding massively from the beach and Pair 3 is not correctly matched: Pushkar is
then coming back during the high tide in order one of the oldest cities in India. Located in
to fill up the emptiness, twice in a day. It is Rajasthan’s Ajmer district in the midst of the
also known as Odisha’s Hide and Seek Beach. Aravali Range, Pushkar is often called tirtha-
It is home to the endangered horseshoe crabs, raj, which literally means the king of pilgrim
starfish, sea urchins, to name a few. The water sites. The city of Pushkar is home to the only
of the beach is muddy, making it unsuited for temple dedicated to Lord Brahma in the whole
bathing. world. It is also among the five pilgrim sites or
dhams for people following Hinduism. Pushkar
Additional Information: is also famous for the Pushkar Lake. Pushkar
y Bhavnagar is located in Gujarat. Mahuva is also called ‘the rose garden of Rajasthan’,
Beach of Bhavnagar is located close to because of the flower farming in and around
the ancient Bhavani Temple. It is well the city. The mountain range Nag Pahar, which
known for its tranquil waters and natural literally means Snake Mountain, separates
beautification. Pushkar from Ajmer.
y Bheemunipatnam Beach is one of the Elimination Technique: The Pushkar Fair,
longest beaches in India. It is located at also called the Pushkar Camel Fair is
the mouth of the Gosthani river in the located in Rajasthan. Mahadeo hills are
Visakhapatnam district of Andhra Pradesh.

Indian Map-Explanation 219


unacademy.com | Download the Unacademy app
Give your feedback here: Link
150 km wide, running essentially along an
located in Madhya Pradesh. Pair 3 is not east-west orientation. It is so named as it lies
correctly matched. Thus, Option (a) is on the 10-degree line of latitude, north of the
correct. equator.
Additional Information:
11. Answer: (b) y Nine Degree Channel: It separates the island
Statement 1 not correct: The Changpa of of Minicoy from the main Lakshadweep
Ladakh are high altitude pastoralists. Among Archipelago.
the Ladakh Changpa, are semi-nomadic tribes y Eight Degree Channel: The maritime
known as Phalpa. Hanley Valley is home to the boundary between the Maldives and India
non-nomadic Changpa (Fangpa). runs through the Eight Degree Channel.
Statement 2 is correct: They mainly rear the It separates the islands of Minicoy and
yaks and Pashmina goats. Despite their different Maldives.
lifestyles, both these groups intermarry. They
speak a dialect of Tibetan Changskhat.
13. Answer: (b)
Statement 3 is correct: The Main religion of
Option 1 and option 4 are not correct: Karnal
the ‘Changpa’ community of India is Tibetan
is famous for the shoe industry and institutes
Buddhism. They are of the Scheduled Tribe
like National Dairy Research Institute (NDRI)
categories in the whole Indian Subcontinent.
and Indian Agricultural Research Institute
(IARI). Bhadrachalam, Telangana is famous for
12. Answer: (a) its Lord Ram temple.
Option 2 is correct: Chanderi, is a well-known
place in Ashok Nagar, Madhya Pradesh, famous
for its hand-woven sarees. It is a renowned
centre for traditional weavers of Sarees. The
production of Chanderi has been provided with
a Geographical Indication Tag.
Option 3 is correct: Kancheepuram is a city
in Tamil Nadu, India. The city is renowned for
its hand-woven silk sarees and most of the
city’s workforce is involved in the weaving
industry. Kancheepuram Sarees received the
Geographical Indication Tag in 2005, the first
product in India to receive this tag.

Elimination Technique: Bhadrachalam is a


key town of Pilgrimage importance located
in the Khammam District of Telangana. The
town is synonymous for its famous temple
devoted to Lord Rama. Thus, Option (b) is
correct.

Option (a) is correct: Ten degrees Channel


which lies between Little Andaman Island and 14. Answer: (c)
the Car Nicobar. The Channel is approximately Pair 1 is correctly matched: The Dampa Tiger

220 Indian Map-Explanation


unacademy.com | Download the Unacademy app
Give your feedback here: Link
Reserve, the largest wildlife sanctuary of 16. Answer: (d)
Mizoram. It occupies an area of 500 sqkm. Pair 1 is not correctly matched: Jim Corbett
The rich rainforests of Dampa harbour several National Park, which is a part of the larger
species such as the swamp deer, tiger, leopard, Corbett Tiger Reserve, a Project Tiger Reserve
elephant and hoolock gibbon. The Dampa Tiger lies in the Nainital district of Uttarakhand.
Reserve is a safe-haven for the tiger and a host Established in the year 1936 as Hailey National
of other animal and bird species. Park, Corbett has the glory of being India’s
Pair 2 is not correctly matched: The Gumti oldest and most prestigious National Park.
Wildlife Sanctuary is located in the South- It is also being honoured as the place where
eastern extreme of Tripura (not in Sikkim-a Project Tiger was first launched in 1973. The
north-eastern State of India). The sanctuary is Ramganga River (West) along with its significant
famous for its elephants, Barking deer, Sambar tributaries Sonanadi, Palain and Mandal
and Bison. The sanctuary encompasses an area forms the prominent hydrological resource
of 390 sq km. for the Corbett. The river Kosi also flows by
Pair 3 is correctly matched: Saramati is the national park and serves as the significant
highest peak(3841m) in the state of Nagaland water resources for the nearby area. The major
and occupies a place of pride in the heart portion of the Corbett wild is situated on the
of its residents. This peak is located on the bank of River Kosi.
Nagaland-Myanmar border. Pair 2 is not correctly matched: Kaziranga
National Park is located in the Golaghat and
Nagaon regions of Assam in India. It is also
15. Answer: (d)
placed beside the Brahmaputra River on the
Option (d) is correct: The Golden Quadrilateral North and the Karbi Anglong mounts on the
(GQ) is a national highway network connecting South India. It is famous for the Great Indian
most of the major industrial, agricultural, one-horned rhinoceros.
and cultural centres of India. In 1999, then
Pair 3 is not correctly matched: Silent Valley
prime minister Atal Bihari Vajpayee laid the
National Park is located in Kerala. It is shielded
foundation stone for the Golden Quadrilateral
by the Nilgiri Plateau to the North and the
(GQ) Highway project.
Mannarkkad Plains to the South. It constitutes
y It is the longest road project in India and the centrepiece of the Nilgiri Biosphere
the fifth-longest highway in the world. It Reserve, an integral part of the Western Ghats,
connects four major cities: Delhi, Mumbai, christened a World Heritage Site by UNESCO
Chennai, and Kolkata. The GQ project in 2012. The Kunthipuzha River flows through
is managed by the National Highways this park.
Authority of India (NHAI) under the Ministry
of Road, Transport and Highways.
17. Answer: (a)
y National Highway-4 (NH4) connects
Mumbai, Chennai via Bangalore. National Pair 1 is correctly matched: The Nokrek
Highway-6 (NH6) connects Kolkata and Biosphere Reserve is located in the northeast
Surat via Raipur. The National Highway-15 of India on the Tura Range, which forms part of
(NH15) is one of the major highways of the Meghalaya Plateau. Nokrek is the highest
Northwestern India, it connects Samakhiali peak of the Garo hills, rising up 1,412 metres.
of Gujrat to Pathankot in Punjab via UNESCO added Nokrek National Park to its
Rajasthan (Jodhpur). list of Biosphere Reserves in May 2009. The
national park is also a sanctuary for several
highly vulnerable and threatened fauna species

Indian Map-Explanation 221


unacademy.com | Download the Unacademy app
Give your feedback here: Link
like the Slow Loris, Petaurista philippensis Pair 4 is not correctly matched: The Bonda
(Giant flying squirrel) and Macaca leonina (Pig- tribe live in the isolated hill regions of the
tailed macaque). Malkangiri district of Odisha.
Pair 2 is not correctly matched: Loktak Lake is
the largest freshwater lake in the northeastern 19. Answer: (a)
region of India. It is Known for its floating
circular swamps, which are called phumdis Option (a) is correct: Arunachal Pradesh
in the local tongue. The lake houses the only is one among the States which has all the
floating national park in the world, the Keibul characteristics as mentioned.
Lamjao National Park, which is the last refuge of y Arunachal Pradesh is the largest state in
the endangered brow-antlered deer or sangai, the North-Eastern region of India. It lies
Manipur’s state animal. It is a Ramsar site. The between the latitude of 26° 28’N - 29°
Barail is the highest hill range in Assam. 30’N and longitudes of 91° 30’E-97° 30’E.
Pair 3 is not correctly matched: Namdapha Arunachal Pradesh is the state which
National Park is located in Arunachal Pradesh. is located on the same latitude which
Namdapha and its adjoining areas, is flanked passes through northern Rajasthan. Some
by the Patkai hills to the south and south-east of the districts of northern Rajasthan are
and by the Himalaya in the north. The area lies Hanumangarh (latitude: 29° 35’ N), Churu
close to the Indo-Myanmar-China trijunction. (28° 19’ N), Jhunjhunu (28° 06’ N), Alwar
(27° 34’ N).
y As per the India State of Forest Report 2011,
18. Answer: (a) released by the Forest Survey of India (FSI),
Pair 1 is correctly matched: Limbu, the second the forest cover in the state is 67,410km2
most numerous tribes of the indigenous which works out as 80.50% of the States
people called Kiranti, living in Nepal, on the geographical area.In terms of the forest
easternmost section of the Himalayas east of canopy density classes, the area covered by
the Arun River, and in northern India, mostly in very dense forests is 20,868km2 that with
the states of Sikkim, West Bengal, and Assam. moderately dense forests is 31,519 km2 and
Pair 2 is not correctly matched: Karbis open forests are 15,023Km2.
represents one of the prominent indigenous y Arunachal Pradesh has a large network
tribes of Northeast India, with unique traditions of protected areas covering an area of
and cultures distinct from other ethnic groups 9,527.995 sq km, which constitutes 18.48%
of the region. The Karbi community is the of recorded Forest area. However, in addition
principal indigenous community in the Karbi to this there are one Biosphere reserve i.e.,
Anglong district and West Karbi Anglong district Dibang Dihang Biosphere Reserve covers
of the Indian State of Assam. an area of 5112 sqkm, two tiger reserves as
Pair 3 is correctly matched: The Niyamgiri Hill Namdapha Tiger Reserve and Pakke Tiger
Range in Odisha state, eastern India, is home Reserve as one Elephant Reserve named
to the Dongria Kondh tribe. Kameng Elephant Reserve.

222 Indian Map-Explanation


unacademy.com | Download the Unacademy app
Give your feedback here: Link
7 ENVIRONMENT & ECOLOGY
AND DISASTER MANAGEMENT
Protected Area Network: NP, WS,
1 BR, etc.
1. Which of the following are the most likely 4. With reference to India’s Desert National
places to find the musk deer in its natural Park, which of the following statements
habitat? (2020) are correct? (2020)
1. Askot Wildlife Sanctuary 1. It is spread over two districts.
2. Gangotri National Park 2. There is no human habitation inside the
3. Kishanpur Wildlife Sanctuary Park.

4. Manas National Park 3. It is one of the natural habitats of the


Great Indian Bustard.
Select the correct answer using the code
given below: Select the correct answer using the code
given below.
(a) 1 and 2 only
(a) 1 and 2 only
(b) 2 and 3 only
(b) 2 and 3 only
(c) 3 and 4 only
(c) 1 and 3 only
(d) 1 and 4 only
(d) 1, 2 and 3

2. Which one of the following protected


areas is well-known for the conservation 5. Which one of the following National Parks
of a sub-species of the Indian swamp deer lies completely in the temperate alpine
(Barasingha) that thrives well on hard zone? (2019)
ground and is exclusively graminivorous? (a) Manas National Park
(2020) (b) Namdapha National Park
(a) Kanha National Park (c) Neora Valley National Park
(b) Manas National Park (d) Valley of Flowers National Park
(c) Mudumalai Wildlife Sanctuary
(d) Tal Chhapar Wildlife Sanctuary 6. Which of the following are in Agasthyamala
Biosphere Reserve? (2019)
3. Among the following Tiger Reserves, which (a) Neyyar, Peppara and Shendurney Wildlife
one has the largest area under “Critical Sanctuaries; and Kalakad Mundanthurai
Tiger Habitat”? (2020) Tiger Reserve.
(a) Corbett (b)
Mudumalai, Sathyamangalam and
(b) Ranthambore Wayanad Wildlife Sanctuaries; and
Silent Valley National Park.
(c) Nagarjunsagar-Srisailam
(c) Kaundinya, Gundla Brahmeswaram and
(d) Sunderbans Papikonda Wildlife Sanctuaries; and
Mukurthi National Park.
(d) Kawal and Sri Venkateswara Wildlife
Sanctuaries; and Nagarjunasagar-
Srisailam Tiger Reserve.

224 Protected Area Network: NP, WS, BR, etc.


unacademy.com | Download the Unacademy app
Give your feedback here: Link
7. In which one of the following States is 1. Harike Wetlands: Confluence of Beas
Pakhui Wildlife Sanctuary located? (2018) and Satluj/ Sutlej
(a) Arunachal Pradesh 2. Keoladeo Ghana Confluence of
(b) Manipur National Park: Banas and Chambal
(c) Meghalaya 3. Kolleru Lake: Confluence of Musi
and Krishna
(d) Nagaland
Which of the above pairs is/are correctly
matched?
8. Which one of the following National Parks
(a) 1 only
has a climate that varies from tropical to
subtropical, temperate, and arctic? (2015) (b) 2 and 3 only

(a) Khangchendzonga National Park (c) 1 and 3 only

(b) Nanda Devi National Park (d) 1, 2 and 3

(c) Neora Valley National Park


(d) Namdapha National Park 11. Consider the following protected areas;
(2012)
1. Bandipur
9. Which of the following National Parks is
unique in being a swamp with floating 2. Bhitarkanika
vegetation that supports a rich biodiversity? 3. Manas
(2015)
4. Sunderbans
(a) Bhitarkanika National Park
Which of the above are declared Tiger
(b) Keibul Lamjao National Park Reserves?
(c) Keoladeo Ghana National Park (a) 1 and 2 Only
(d) Sultanpur National Park (b) 1, 3 and 4 Only
(c) 2, 3 and 4 only
10. Consider the following pairs: (2014) (d) 1,2, 3 and 4
Wetlands Conference of
rivers

Protected Area Network: NP, WS, BR, etc. 225


unacademy.com | Download the Unacademy app
Give your feedback here: Link
Protected Area Network: NP, WS,
1 BR, etc.-Explanation
1. Answer: (a) Maikal range of the Satpura hills and is spread
The state animal of Uttarakhand, the white- over an area of 940 square kilometers between
bellied musk deer (Moschus leucogaster) the Mandla and Balaghat districts of Madhya
roams the high alpine region of the Himalayas Pradesh. The swamp deer is endemic to KNPTR.
at an elevation range of 2,500-5,000 meters, is
extremely vulnerable during the harsh season. 3. Answer: (c)
The Musk pod is highly valuable. So, the
communities would continue to poach despite The Wildlife Protection Act (WLPA), 1972
proper livelihood opportunities. identifies ‘Critical Tiger habitats’ (CTHs), also
known as core areas of tiger reserves. They are
Option (a) is correct: The musk deer is most based on scientific evidence that “such areas
likely found in Askot Wildlife Sanctuary are required to be kept as inviolate for the
(also known as Askot Musk Deer Sanctuary), purpose of tiger conservation, without affecting
located in the Pithoragarh district of Kumaun, the rights of the Scheduled Tribes or such
Uttarakhand and the Gangotri Wildlife National other forest dwellers”. The notification of CTH
Park in Uttarkashi District of Uttarakhand in is done by the state government in dialogue
India. with the expert committee constituted for the
purpose.
Option (c) is correct: Nagarjunsagar-Srisailam
Tiger Reserve is the largest tiger reserve in
India. The reserve spreads over five districts,
Kurnool District, Prakasam District, Guntur
District, Nalgonda District and Mahbubnagar
district. The total area of the tiger reserve is
3,728 km2 (1,439 sq mi).

4. Answer: (c)
Statement 1 is correct: Desert National Park
is situated in the Indian state of Rajasthan. It
covers an area of 3162 km² of which 1900 km²
is in Jaisalmer district and the remaining 1262
km² is in Barmer district of Rajasthan State. The
Desert National Park is an excellent example of
the ecosystem of the Thar Desert.
Statement 2 is not correct: The Thar desert is
2. Answer: (a)
the most thickly populated desert in the world
Option (a) is correct: Hard ground swamp deer with an average density of 83 persons/km²
(Barasingha or Rucervus duvaucelii), the state (compared to 7km² of other deserts, Baqri and
animal of Madhya Pradesh, is seeing a revival Kankane 2001). The human population, though,
in the Kanha National Park and Tiger Reserve is low (4-5 persons per km²) within the DNP.
(KNPTR) after having been perilously close to There are 73 villages and settlements or Dhanis
extinction for a long time. KNPTR is on the existing within the Park. These communities

226 Protected Area Network: NP, WS, BR, etc.-Explanation


unacademy.com | Download the Unacademy app
Give your feedback here: Link
have inhabited this area for hundreds of years. Tamil Nadu. It is located at the southernmost
They form an integral part of this ecosystem end of the Western Ghats. It is listed on the
with their rich culture and tradition. World Network of Biosphere Reserves. The
Statement 3 is correct: The DNP is the most reserve includes three wildlife sanctuaries,
important site for the long-term survival of the Shendurney, Peppara and Nayar, as well as the
Globally Threatened Great Indian Bustard and Kalakad Mundanthurai Tiger reserve. It consists
other endemic fauna and flora. It is one of the mostly of tropical forests and possesses a
natural habitats of the Great Indian Bustard. unique genetic reservoir of cultivated plants
Other birds of significance identified by BirdLife such as cardamom, Jamun, nutmeg, pepper
International, include the Eleven bird species and plantain.
that are representative of Biome-13.
7. Answer: (a)
5. Answer: (d) Option (a) is correct: Pakhui Wildlife Sanctuary
Option (d) is correct: The Valley of Flowers lies in the foothills of the Eastern Himalaya, in
national park is located in the Chamoli district the East Kameng District of Arunachal Pradesh.
of Uttarakhand and is known for its meadows It was declared a Tiger Reserve in 2002. The
of endemic alpine flowers and the variety of habitat comprises lowland semi-evergreen,
flora. It lies completely in the temperate alpine evergreen, and Eastern Himalayan broadleaf
zone. It is a richly diverse area and is home forests.
to rare and endangered animals, including the National Parks in Arunachal Pradesh: Mouling
Asiatic black bear, snow leopard, musk deer, and Namdapha, etc.
brown bear, red fox, and blue sheep. Wildlife Sanctuary in Arunachal Pradesh: D’
Additional Information: Ering Memorial, Dibang, Eagle Nest, Itanagar,
y Valley of Flowers National Park and Nanda Kane, Mehao, Pakhui, Sessa Orchid, etc.
Devi National Park parks are encompassed
in the Nanda Devi Biosphere Reserve. This 8. Answer: (d)
Reserve has been in the UNESCO World
Network of Biosphere Reserves since 2004. Namdapha National Park is located in Arunachal
Pradesh. It was declared as a tiger reserve
y The valley was declared a national park in under Project Tiger.
1982 and now it is a World Heritage Site.
The major forest types of the reserve are y The main vegetation of Namdapha is
temperate. The entire Nanda Devi Biosphere evergreen forests, moist deciduous forests,
Reserve lies within the Western Himalayas subtropical forests, temperate forests and
Endemic Bird Area (EBA). alpine. It has a climate that varies from
tropical to subtropical, temperate, and
y The Valley of Flowers National Park is arctic. [Option (d) is correct]
the second core zone of the Nanda Devi
Biosphere Reserve. Seven restricted-range y The Namdapha Protected area falls within
bird species are endemic to this part of the the geographical sub-tropical zone and
EBA. enjoys the sub-tropical climate. However,
the climate of the area varies from place
to place inside the Protected Area due to
6. Answer: (a) variations of altitude from 200 m to 4571m
Option (a) is correct: The Agasthyamalai and the area also being the zone of heavy
Biosphere Reserve was established in 2001 and rainfall.
is spread across the two states of Kerala and Additional Information:

Protected Area Network: NP, WS, BR, etc.-Explanation 227


unacademy.com | Download the Unacademy app
Give your feedback here: Link
y Khangchendzonga National Park (KNP): Additional Information:
Situated in the northern Indian State of y Bhitarkanika is India’s second-largest
Sikkim. It has been declared as a World mangrove ecosystem after the Sunderbans.
Heritage Site by UNESCO. This is for the The National Park is essentially a network
first time that any Indian destination has of creeks and canals which are inundated
been under the Mixed criteria of UNESCO’s with waters from rivers Brahmani, Baitarani,
heritage sites list, thus recognising the Dhamra and Patasala forming a unique
outstanding universal values for both ecosystem.
natural and cultural significance.
y Keoladeo National Park is in the State of
y Nanda Devi Biosphere Reserve: It is located Rajasthan. Formerly known as the Bharatpur
in the Himalayan Mountains in the northern Bird Sanctuary, the Keoladeo National
part of the country, and includes as core Park is recognised as one of the world’s
areas the Nanda Devi and Valley of Flowers most important bird breeding and feeding
National Parks, which are one World grounds. In 1982, Keoladeo was declared
Heritage site. a national park and then later listed as a
y Neora Valley National Park (NVNP): It is World Heritage Site by UNESCO in 1985. It
a compact patch of virgin forest, rich was also known as the breeding ground for
in biodiversity, located in the Eastern the rare and elusive Siberian crane.
Himalayas, a global ‘biodiversity hotspot’. It y Sultanpur National Park & Bird Sanctuary
was designated as a National Park based on is in the Gurgaon district of Haryana. The
the provisions of the Wildlife (Protection) Sultanpur National Park is lush with trees,
Act, 1972, in 1992. It is contiguous with shrubs, and clusters of bougainvillea.
Sikkim and Bhutan at its northern and
north-eastern boundaries, respectively and
links the Pangolakha Wildlife Sanctuary in 10. Answer: (a)
Sikkim and the Toorsa Strict Reserve of Pair 1 is correctly matched: Harike Wetlands
Bhutan. is one of the largest man-made wetlands
of northern India. It came into existence
9. Answer: (b) in 1952 after the construction of a barrage
near the confluence of rivers Sutlej and
Option (b) is correct: Keibul Lamjao National Beas. Considered a wetland of international
Park is the only floating park in the world. importance, especially as a waterfowl refuge,
The Park is located in the southwestern part this site was accorded wetland status in 1990
of Loktak Lake. The Loktak Lake has a unique by the Ramsar Convention.
ecosystem called ‘Phumdi’ (a Manipuri word
meaning floating mats of soil and vegetation). Pair 2 is not correctly matched: Keoladeo Ghana
National Park is formed by the confluence of
y The largest area of the Phumdi in the the Gambira and Banganga rivers. Keoladeo
Loktak lake is in the Keibul Lamjao National National Park is also known as Bharatpur Bird
Park, which is home to Manipur brow- Sanctuary or Keoladeo Ghana Bird Sanctuary. It
antlered deer, also popularly known as the is a UNESCO world heritage site and is included
Sangai. The habitat exclusively consists of in the Montreux Record.
floating meadows and an elevated strip
of hard ground that dissects the park into Pair 3 is not correctly matched: Kolleru Lake
northern and southern zones. Loktak Lake is the confluence of Godavari and Krishna.
is the largest natural freshwater lake in the Currently, it covers an area of 308 sq km. Kolleru
north-eastern region of India. has a sanctuary status under the Wildlife
Protection Act of 1972. It was designated a

228 Protected Area Network: NP, WS, BR, etc.-Explanation


unacademy.com | Download the Unacademy app
Give your feedback here: Link
Ramsar site in 2002. It was also identified as Brahmani, Baitarani, Dhamra and Patasala
an “Important Bird Area” of India by the Bombay forming a unique ecosystem. Bhitarkanika
Natural History Society. is home to the largest congregation of the
endangered Saltwater Crocodile in the country.

11. Answer: (b) Statement 3 is correct: Located in the state


of Assam at the foothills of the Bhutan hills,
Statement 1 is correct: Bandipur National Park
Manas Tiger Reserve was created in the year of
established in 1974 as a tiger reserve under
1973. This was done with the launch of Project
Project Tiger, is a national park located in the
Tiger in India. It has Manas Sanctuary as its
Indian state of Karnataka, Bandipur National
core which was established in the year of 1928.
Park has the second-highest Tiger population
in India. largest protected area in Southern Statement 4 is correct: In the year 1978,
India and the largest habitat of Wild Elephants Sundarbans was declared a national park, and
in South Asia. in 1973, they have declared a tiger reserve under
Project Tiger. It was in 1875 under the Forest
Statement 2 is not correct: Bhitarkanika is
Act, 1865 (Act VIII of 1865) that a large part
India’s second-largest mangrove ecosystem
of these forests was declared as “reserved”.
after the Sunderbans. The National park is
Post-independence, it was declared a wildlife
essentially a network of creeks and canals
sanctuary in 1977 and established as a national
which are inundated with waters from rivers
park on 4th May 1984.

Protected Area Network: NP, WS, BR, etc.-Explanation 229


unacademy.com | Download the Unacademy app
Give your feedback here: Link
2 Ecosystem and Ecology

1. Consider the following statements : (2023) which one of the following situations?
1. Some microorganisms can grow in (2022)
environments with temperature above (a) Restoration of damaged coral reefs
the boiling point of water. (b) Development of building materials using
2. Some microorganisms can grow in plant residues
environments with temperature below (c) Identification of areas for exploration/
the freezing point of water. extraction of shale gas
3. Some microorganisms can grow in a (d) Providing salt licks for wild animals in
highly acidic environment with a pH forests/protected areas
below 3.
How many of the above statements are
correct? 5. Which of the following are nitrogen-fixing
plants? (2022)
(a) Only one
1. Alfalfa
(b) Only two
2. Amaranth
(c) All three
3. Chickpea
(d) None
4. Clover
5. Purslane (Kulfa)
2. Certain species of which one of the
following organisms are well known as 6. Spinach
cultivators of fungi? (2022) Select the correct answer using the code
(a) Ant given below :

(b) Cockroach (a) 1, 3 and 4 only

(c) Crab (b) 1, 3, 5 and 6 only

(d) Spider (c) 2, 4, 5 and 6 only


(d) 1, 2, 4, 5 and 6

3. The “Miyawaki method” is well known for


the : (2022) 6. “If rainforests and tropical forests are the
(a) Promotion of commercial farming in lungs of the Earth, then surely wetlands
arid and semi-arid areas function as its kidneys.” Which one of
the following functions of wetlands best
(b)
Development of gardens using reflects the above statement ? (2022)
genetically modified flora
(a) The water cycle in wetlands involves
(c) Creation of mini forests in urban areas surface runoff, subsoil percolation and
(d) Harvesting wind energy on coastal areas evaporation.
and on sea surfaces (b) Algae form the nutrient base upon
which fish, crustaceans, molluscs,
4. “Biorock technology” is talked about in birds, reptiles and mammals thrive.

230 Ecosystem and Ecology


unacademy.com | Download the Unacademy app
Give your feedback here: Link
(c) Wetlands play a vital role in maintaining 10. In the nature, which of the following is/
sedimentation balance and soil are most likely to be found surviving on a
stabilization. surface without soil? (2021)
(d) Aquatic plants absorb heavy metals and 1. Fern
excess nutrients. 2. Lichen
3. Moss
7. What is blue carbon? (2021) 4. Mushroom
(a) Carbon captured by oceans and coastal Select the correct answer using the code
ecosystems given below.
(b) Carbon sequestered in forest biomass (a) 1 and 4 only
and agricultural soils
(b) 2 only
(c) Carbon contained in petroleum and
natural gas (c) 2 and 3

(d) Carbon present in atmosphere (d) 1, 3 and 4

8. In case of which one of the following 11. Which of the following are detritivores?
biogeochemical cycles, the weathering (2021)
of rocks is the main source of release of 1. Earthworms
nutrients to enter the cycle? (2021) 2. Jellyfish
(a) Carbon cycle 3. Millipedes
(b) Nitrogen cycle 4. Seahorses
(c) Phosphorus cycle 5. Woodlice
(d) Sulphur cycle Select the correct answer using the code
given below.
9. Consider the following kinds of organisms : (a) 1, 2 and 4 only
(2021) (b) 2, 3, 4 and 5 only
1. Copepods (c) 1, 3 and 5 only
2. Cyanobacteria (d) 1, 2, 3, 4 and 5
3. Diatoms
4. Foraminifera 12. Which of the following have species that
Which of the above are primary producers can establish a symbiotic relationship with
in the food chains of oceans? other organisms? (2021)
(a) 1 and 2 1. Cnidarians
(b) 2 and 3 2. Fungi
(c) 3 and 4 3. Protozoa
(d) 1 and 4 Select the correct answer using the
code given below.
(a) 1 and 2 only

Ecosystem and Ecology 231


unacademy.com | Download the Unacademy app
Give your feedback here: Link
(b) 2 and 3 only trees, the vegetation is largely composed
(c) 1 and 3 only of plant forms that reach up into the
canopy vicariously, by climbing the trees or
(d) 1, 2 and 3 growing as epiphytes, rooted on the upper
branches of trees." This is the most likely
13. Consider the following animals : (2021) description of (2021)

1. Hedgehog (a) coniferous forest

2. Marmot (b) dry deciduous forest

3. Pangolin (c) mangrove forest

To reduce the chance of being captured by (d) tropical rainforest


predators, which of the above organisms
rolls up/roll up and protects/ protect its/ 16. Why is a plant called Prosopis juliflora
their vulnerable parts? often mentioned in the news? (2018)
(a) 1 and 2 (a) Its extract is widely used in cosmetics.
(b) 2 only (b) It tends to reduce the biodiversity in the
(c) 3 only area in which it grows.
(d) 1 and 3 (c) Its extract is used in the synthesis of
pesticides.

14. The vegetation of savannah consists of (d) None of the above


grassland with scattered small trees, but
extensive areas have no trees. The forest 17. Which of the following leaf modifications
development in such areas is generally kept occur(s) in desert areas to inhibit water
in check by one or more or a combination loss? (2018)
of some conditions. Which of the following
are such conditions? (2021) 1. Hard and waxy leaves

1. Burrowing animals and termites 2. Tiny leaves

2. Fire 3. Thorns instead of leaves

3. Grazing herbivores Select the correct answer using the code


given below:
4. Seasonal rainfall
(a) 2 and 3 only
5. Soil properties
(b) 2 only
Select the correct answer using the code
given below. (c) 3 only

(a) 1 and 2 (d) 1, 2 and 3

(b) 4 and 5
(c) 2, 3 and 4 18. Due to some reasons, if there is a huge fall
in the population of species of butterflies,
(d) 1, 3 and 5 what could be its likely consequence/
consequences? (2017)
15. "Leaf litter decomposes faster than in 1. Pollination of some plants could be
any other biome and as a result the soil adversely affected.
surface is often almost bare. Apart from 2. There could be a drastic increase in the

232 Ecosystem and Ecology


unacademy.com | Download the Unacademy app
Give your feedback here: Link
fungal infections of some cultivated Select the correct answer using the code
plants. given below:
3. It could lead to a fall in the population (a) 1 and 3 only
of some species of wasps, spiders, and (b) 2 only
birds.
(c) 1, 2 and 4 only
Select the correct one using the code given
below: (d) 1, 2, 3 and 4

(a) 1 only
(b) 2 and 3 only 22. Consider the following: (2014)

(c) 1 and 3 only 1. Bats

(d) 1, 2 and 3 2. Bears


3. Rodents

19. Which one of the following is the best The phenomenon of hibernation can be
description of the term 'ecosystem'? (2015) observed in which of the above kinds of
animals?
(a) A community of organisms interacting
with one another (a) 1 and 2 only

(b) That part of the Earth that is inhabited (b) 2 only


by living organisms (c) 1, 2 and 3
(c) A community of organisms together (d) Hibernation cannot be observed in any
with the environment in which they live of the above
(d) The flora and fauna of a geographical
area 23. Which one of the following is the correct
sequence of a food chain? (2014)
20. Which one of the following regions of India (a) Diatoms-Crustaceans-Herrings
has a combination of mangrove forest, (b) Crustaceans-Diatoms-Herrings
evergreen forest, and deciduous forest?
(c) Diatoms-Herrings-Crustaceans
(2015)
(d) Crustaceans-Herrings-Diatoms
(a) North Coastal Andhra Pradesh
(b) South-West Bengal
24. Among the following organisms, which one
(c) Southern Saurashtra
does not belong to the class of the other
(d) Andaman and Nicobar Islands three? (2014)
(a) Crab
21. Which of the following adds/add carbon (b) Mite
dioxide to the carbon cycle on the planet
(c) Scorpion
Earth? (2014)
(d) Spider
1. Volcanic action
2. Respiration
25. Which one of the following is the process
3. Photosynthesis
involved in photosynthesis? (2014)
4. Decay of organic matter
(a) Potential energy is released to form free

Ecosystem and Ecology 233


unacademy.com | Download the Unacademy app
Give your feedback here: Link
energy. of old and used computers or their parts,
(b) Free energy is converted into potential which of the following are released into
energy and stored. the environment as e-waste? (2013)

(c) Food is oxidised to release carbon 1. Beryllium


dioxide and water. 2. Cadmium
(d) Oxygen is taken, and carbon dioxide and 3. Chromium
water vapour are given out. 4. Heptachlor
5. Mercury
26.
With reference to the food chains in 6. Lead
ecosystems, which of the following kinds
of organism is/are known as decomposer 7. Plutonium
organism/organisms? (2013) Select the correct answer using the code
1. Virus given below:

2. Fungi (a) 1, 3, 4, 6, and 7 only

3. Bacteria (b) 1, 2, 3, 5, and 6 only

Select the correct answer using the code (c) 2, 4, 5, and 7 only
given below: (d) 1, 2, 3, 4, 5, 6 and 7
(a) 1 only
(b) 2 and 3 only 30. Acid rain is caused by the pollution of
(c) 1 and 3 only environment by (2013)

(d) 1, 2 and 3 (a) carbon dioxide and nitrogen


(b) carbon monoxide and carbon dioxide

27. In the grasslands, trees do not replace (c) ozone and carbon dioxide
the grasses as a part of an ecological (d) nitrous oxide and sulphur dioxide
succession because of (2013)
(a) insects and fungi 31. With reference to food chains in ecosystems,
(b) limited sunlight and paucity of nutrients consider the following statements: (2013)
(c) water limits and fire 1. A food chain illustrates the order in
(d) None of the above which a chain of organisms feeds upon
each other.
2. Food chains are found within the
28. Which one of the following is the correct populations of a species.
sequence of ecosystems in the order of
decreasing productivity? (2013) 3. A food chain illustrates the numbers of
each organism that are eaten by others.
(a) Oceans, lakes, grasslands, mangroves
Which of the statements given above is/are
(b) Mangroves, oceans, grasslands, lakes correct?
(c) Mangroves, grasslands, lakes, oceans (a) 1 only
(d) Oceans, mangroves, lakes, grasslands (b) 1 and 2 only
(c) 1, 2 and 3
29. Due to improper/indiscriminate disposal

234 Ecosystem and Ecology


unacademy.com | Download the Unacademy app
Give your feedback here: Link
(d) None supporting, regulating, preserving and
cultural. Which one of the following is
supporting service? (2012)
32. Which of the following adds/add nitrogen
to the soil? (2013) (a) Production of food and water

1. Excretion of urea by animals (b) Control of climate and disease

2. Burning of coal by man (c) Nutrient cycling and crop pollination

3. Death of vegetation (d) Maintenance of diversity

Select the correct answer using the code


given below: 36. What would happen if phytoplankton of an
(a) 1 only ocean is completely destroyed for some
reason? (2012)
(b) 2 and 3 only
1. The ocean as a carbon sink would be
(c) 1 and 3 only adversely affected.
(d) 1, 2 and 3 2. The food chains in the ocean would be
adversely affected.
33. Which one of the following terms describes 3. The density of ocean water would
not only the physical space occupied by an drastically decrease.
organism but also its functional role in the Select the correct answer using the code
community of organisms? (2013) given below:
(a) Ecotone (a) 1 and 2 only
(b) Ecological niche (b) 2 only
(c) Habitat (c) 3 only
(d) Home range (d) 1, 2 and 3

34. Consider the following kinds of organisms: 37. In the context of ecosystem productivity,
(2012) marine upwelling zones are important
1. Bat as they increase marine productivity by
2. Bee bringing the: (2011)

3. Bird 1. Decomposer microorganisms to the


surface
Which of the above is/are pollinating agent/
agents? 2. Nutrients to the surface

(a) 1 and 2 only 3. Bottom-dwelling organisms to the


surface
(b) 2 only
Which of the statements given above is/are
(c) 1 and 3 only
correct?
(d) 1, 2 and 3
(a) 1 and 2 only
(b) 2 only
35.
The Millennium Ecosystem Assessment
(c) 2 and 3 only
describes the following major categories
of ecosystem services-provisioning, (d) 3 only

Ecosystem and Ecology 235


unacademy.com | Download the Unacademy app
Give your feedback here: Link
38. If a tropical rainforest is removed, it does (b) Propagules of the trees in a rainforest
not regenerate quickly as compared to a have poor viability.
tropical deciduous forest. This is because (c) The rain forest species are slow growing.
(2011)
(d) Exotic species invade the fertile soil of
(a) The soil of the rainforest is deficient in the rainforest.
nutrients.

236 Ecosystem and Ecology


unacademy.com | Download the Unacademy app
Give your feedback here: Link
Ecosystem and Ecology-
2 Explanation
1. Answer: (c) several different agricultural techniques to
Statement 1 is correct: Some microorganisms cultivate mutualistic fungi which they raise
have adapted to thrive in extreme environments, in fungus gardens. The more basal lineages
including those with temperatures above raise their crop on organic detritus, while the
the boiling point of water. These organisms, more advanced leaf-cutting genera Atta and
known as thermophiles or hyperthermophiles, Acromyrmex, which evolved only 8-12 million
have remarkable adaptations that allow them years ago, harvest vegetation and process
to survive and even flourish in such extreme it into pulp to feed to the fungi they raise in
conditions. Thermophiles are typically found monocultures.
in geothermal areas, such as hot springs,
hydrothermal vents, and volcanic areas, where 3. Answer: (c)
temperatures can exceed 100°C (212°F).
Option (c) is correct: The method involves
Statement 2 is correct: Some microorganisms planting two to four trees per square meter.
have the ability to grow and thrive in Miyawaki forests grow in two to three years
environments with temperatures below the and are self-sustaining. They help lower
freezing point of water. These organisms are temperatures in concrete heat islands, reduce
known as psychrophiles or cryophiles and air and noise pollution, attract local birds and
have adapted to survive in extremely cold insects, and create carbon sinks.
conditions, including polar regions, high-
altitude environments, and even within glaciers
and ice sheets.Psychrophiles have evolved 4. Answer: (a)
specialised adaptations that allow them to Option (a) is correct: Biorock technology greatly
withstand and function at low temperatures. accelerates coral settlement, growth, healing,
These adaptations include membrane survival, and resistance to environmental
flexibility,enzyme activity,Antifreeze proteins stresses such as high temperature, sediment,
and Osmoprotectants and pollution. All other marine organisms
Statement 3 is correct: Some microorganisms examined also benefit.
have the remarkable ability to grow and thrive
in highly acidic environments with a pH below
5. Answer: (a)*
3. These organisms are known as acidophiles
and have adapted to survive and carry out Option (a) is correct: Rhizobium is rod-shaped
their life processes in extreme acidity, which nitrogen-fixing bacteria that live symbiotically
would be inhospitable or even deadly for most within the roots of several leguminous plants
other organisms.Acidophiles can be found in such as alfalfa, sweet clover, sweet pea, lentils,
a range of acidic environments, including acid etc.
mine drainage, volcanic areas, sulphur-rich hot The amaranth plant is a grain and green
springs, and acidic soils. crop plant because it is not a pulse crop so
does not have a nitrogen-fixing mechanism.
Purslane plant is one of the few vegetables
2. Answer: (a)
that are rich in omega-3 fatty acids, which are
Option (a) is correct: The attine ants are a important to support healthy arteries and can
group of more than 200 species that use help prevent strokes, heart attacks, and other

Ecosystem and Ecology-Explanation 237


unacademy.com | Download the Unacademy app
Give your feedback here: Link
forms of heart disease. It also does not have on Earth is tied up in rock and sedimentary
a nitrogen-fixing mechanism at the root zone. deposits, from which it is released by
weathering, leaching, and mining.
6. Answer: (d) Elimination Technique: The carbon cycle
Option (d) is correct: Heavy metals in aquatic and Nitrogen cycle are the gaseous cycles
environments, such as lakes and rivers, and are not related to the weathering
have been studied extensively because of of rocks. So, options A and B can be
their toxicity, persistence, and tendency to eliminated.
bioaccumulate. Many studies have shown that
aquatic plants are sinks for heavy metals in
aquatic ecosystems. Because of their capacity 9. Answer: (b)
to accumulate heavy metals, several species The marine ecosystem is made up of a
of aquatic macrophytes such as H. verticillata complicated series of interconnected energy
and water hyacinth have been used to remove producers—like plants and phytoplankton and
heavy metals from wastewater. consumers, from plant-eaters to meat-eaters,
both big and small.

7 Answer: (a) Option (b) is correct: Primary producers —


including bacteria, phytoplankton, and algae
Option (a) is correct: The term “blue carbon” — form the lowest trophic level, the base
refers to carbon captured by the ocean and of the aquatic food web. Primary producers
coastal ecosystems around the world. Through synthesize their own energy without needing
sequestration, our oceans and coasts provide to eat. Cyanobacteria are essential primary
a natural means to reduce the influence producers in aquatic environments. The
of greenhouse gases on our atmosphere. cyanobacteria (also referred to as blue-green
Seagrasses, mangroves, and salt marshes along algae) have chlorophyll similar to green plants
our coast operate as carbon sinks, capturing and are photosynthetic autotrophs. Diatoms
and storing carbon. are one of the ocean’s most important primary
producers. Diatoms are the chief ‘producers’ in
8. Answer: (c) the oceans.

Nutrient cycles are of two types: (a) gaseous Copepods are important secondary producers
and (b) sedimentary. The reservoir for the in the oceans. Copepod is any member of
gaseous type of nutrient cycle (e.g., nitrogen, the widely distributed crustacean subclass
carbon cycle) exists in the atmosphere, and for Copepoda.
the sedimentary cycle (e.g., phosphorus cycle), Foraminifera are single-celled organisms and
the reservoir is located in Earth’s crust. are members of a phylum or class of amoeboid
The phosphorus cycle is the process by which protists characterized by streaming granular
phosphorus moves through the lithosphere, ectoplasm for catching food and other uses,
hydrosphere, and biosphere. Phosphorus is and commonly an external shell (called a
essential for plant and animal growth, as well as “test”) of diverse forms and materials. They are
the health of microbes inhabiting the soil, but is not primary producers.
gradually depleted from the soil over time. The
main biological function of phosphorus is that 10. Answer: (c)
it is required for the formation of nucleotides,
which comprise DNA and RNA molecules. Option (c) is correct:

Option (c) is correct: Much of the phosphorus y Lichens may grow on practically any

238 Ecosystem and Ecology-Explanation


unacademy.com | Download the Unacademy app
Give your feedback here: Link
surface and can grow from sea level to high 12. Answer: (d)
alpine heights under a variety of climatic Option (d) is correct: A symbiosis is an evolved
situations. interaction or close living relationship between
y Mosses are a type of bryophyte, which is a organisms from different species, usually with
group of non-vascular plants. This means benefits to one or both of the individuals
they lack the vascular tissue, known as the involved. Some symbiotic relations are:
xylem and phloem, that most plants have y The link between cnidarians and
for transporting water from roots to leaves dinoflagellate algae.
and stems. Instead, mosses have rhizoids
or small hair-like appendages that anchor y Mycorrhiza and lichen
the moss and take in water. As a result, y Symbiosis in protozoa mostly represents
mosses grow low and flat along the ground a close mutualistic association between
because they do not have the support a protozoan and unicellular symbionts
system necessary to grow upwards. (bacteria, cyanobacteria or/and unicellular
y Ferns need indirect sunlight, moist soil, algae) or protozoans and a multicellular
and a humid environment to thrive. organism (ruminants, lower termites,
wood-eating cockroaches, plants).
y A mushroom is a fleshy, spore-bearing
fruiting body of a fungus that grows above
ground, on soil, or its food source. 13. Answer: (d)
Option (d) is correct:
11. Answer: (c) y Hedgehogs have spines which are known
A detritivore is a heterotrophic organism that as quills, and they know how to use them.
obtains its nutrition by feeding on detritus. When they feel threatened, their quills
Detritivores and decomposers contribute to immediately go up in full force. To protect
the breakdown of all of the dead and decaying himself, a hedgehog coils his head and feet
material in any ecosystem. In this way, they play into his body and creates a ball.
an important role in the cycling of nutrients and y When threatened, a pangolin will curl up
are an essential part of most biogeochemical into a tight ball and defend itself with its
cycles. sharp-scaled tails.
Option (c) is correct: Some examples of When the yellow-bellied marmot sees a
detritivores are: predator, it whistles to alert the others in the
y Earthworms area, then hides under a nearby rock pile until
the threat has passed. They do not roll up.
y Millipedes
y Dung Beetles
14. Answer: (c)
y Woodlice
Option (c) is correct: The Savanna or Sudan
y Cushion Stars Climate is a transitional type of climate found
The seahorse is a carnivore. between the equatorial forests and the trade
Jellyfish are carnivores and can increase in wind hot deserts. Savanna vegetation includes
size rapidly and procreate in large numbers scrub, grasses, and occasional trees, which
when food is abundant. grow near water holes, seasonal rivers, or
aquifers. Plants and animals must adapt to
long dry periods.
The vegetation of the savannah consists of

Ecosystem and Ecology-Explanation 239


unacademy.com | Download the Unacademy app
Give your feedback here: Link
grassland with scattered small trees, but The tropical rainforest is a hot, moist biome
extensive areas have no trees. The forest where it rains all year long. The bottom layer
development in such areas is generally kept or floor of the rainforest is covered with wet
in check by one or more or a combination of leaves and leaf litter. This material decomposes
some conditions. The forest development in rapidly in wet, warm conditions (like a compost
savannah areas is generally kept in check by pile), sending nutrients back into the soil. Few
Fire, Grazing herbivores, and Seasonal rainfall. plants are found on the floor of the forest due
This climate is characterized by an alternate to the lack of sunlight. However, the hot, moist
hot, rainy season and cool, dry season. In atmosphere and all the dead plant material
the northern hemisphere, the hot, rainy create the perfect conditions in which bacteria
season normally begins in May and lasts until and other microorganisms can thrive.
September, as in Kano, Nigeria. The rest of the Elimination Technique: The terms ‘Faster
year is cool and dry. The annual rainfall is not decomposition of leaf litter, growing of
sufficient and scattered over the year, which is plants by climbing the trees or growing
not enough to develop forests. as epiphytes’ are related to tropical
There are two main groups of animals in the rainforests only.
savanna, the grass-eating herbivorous animals
and the fleshing-eating carnivorous animals.
The herbivorous animals are often very alert 16. Answer: (b)
and move swiftly from place to place in search Option (b) is correct: Prosopis juliflora (vilayati
of green pastures. They check the growth of kikar or vilayati babul), an exotic tree, is one
forests. of the top invaders in India. It was introduced
The tall brown bushes are burnt down by the into India from South America and has become
farmers in preparation for new fields for the naturalised all-over north India. This tree is
following year. Sometimes fires may be caused also affecting the nesting success of birds. It
by the dry, dusty Harmattan. They also check tends to reduce the biodiversity in the area in
the growth of forests. which it grows.
Invasive plants are those plants which-spread
to regions other than their native and thrive to
15. Answer: (d) the extent of becoming a threat to local species.
Tropical rainforests are luxuriant forests found Alien invasive species occur in all groups of
in wet tropical lands around the Equator. plants and animals. They include competitors,
Tropical rainforests, which worldwide make up predators, pathogens, and parasites. They have
one of Earth’s largest biomes (major life zones), invaded almost every type of native ecosystem
are dominated by broad-leaved trees that form and have caused hundreds of extinctions.
a dense upper canopy (layer of foliage) and
contain a diverse array of vegetation and other
life. 17. Answer: (d)

Option (d) is correct: In tropical rainforests, Option (d) is correct: Desert plants lose very
leaf litter decomposes faster than in any other little water through transpiration. The leaves in
biome, and as a result, the soil surface is often desert plants are either absent, very small (tiny),
almost bare. Apart from trees, the vegetation or they are present in the shape of spines. This
is largely composed of plant forms that reach helps in reducing the loss of water from the
up into the canopy vicariously by climbing the leaves through transpiration plants growing in
trees or growing as epiphytes, rooted on the extremely dry conditions throughout the year
upper branches of trees. adopt thorns instead of leaves to prevent loss

240 Ecosystem and Ecology-Explanation


unacademy.com | Download the Unacademy app
Give your feedback here: Link
of water. caterpillars. The growth of a butterfly depends
Photosynthesis in these plants is usually entirely on the larval stage: if the larva is well-
carried out by the stems. The stem is also fed, the size of the adult will be bigger. As such,
covered with a thick waxy layer, which helps to female butterflies are picky about which leaves
retain water. Hence, they have hard and waxy to lay eggs on. Some, for instance, would want
leaves as well. Most desert plants have roots their offspring caterpillars to be fed only on
that go very deep into the soil for absorbing lime leaves. Each species has a distinct palate
water. for nectar.

Xerophytes are plants growing in extremely dry Elimination Technique: The term ‘drastic
conditions throughout the year. For example, increase’ seems vague in the statement.
plants growing in deserts (psammophytes), So, by eliminating statement 2, we are
on rock (lithophytes), or alpine plants growing remaining with options A and C.
above 14000 feet altitude.

Elimination Technique: Hard and waxy 19. Answer: (c)


leaves are a well-known feature of desert Option (c) is correct: An ecosystem can be
plants. So, statement 1 must be part of visualised as a functional unit of nature, where
the answer. It makes answer D. living organisms interact among themselves
and with the surrounding physical environment.
18. Answer: (c) Ecosystem varies greatly in size from a small
pond to a large forest or a sea. The interaction
The Western Ghats, along with the Himalayas, of biotic and abiotic components results in
the country’s northeast region, and the a physical structure that is characteristic of
Andaman Islands, are home to butterflies each type of ecosystem. The components of
unique to India. With 1,800 known species and the ecosystem are seen to function as a unit
subspecies, the country boasts an impressive when we consider the aspects of Productivity,
10 percent of the global biodiversity in butterfly. Decomposition, Energy flow, and Nutrient
Statement 1 is correct: Butterflies are crucial cycling. Thus, an ecosystem is a community
pollinators, and their contribution to global of organisms together with the environment in
agriculture is valued only next to honeybees. which they live.
Butterflies probe for nectar, their flight fuel,
and typically favour the flat, clustered flowers
that provide a landing pad and abundant 20. Answer: (d)
rewards. Option (d) is correct: The vegetation of
Statement 2 is not correct: No such concrete Andaman and Nicobar Islands may be broadly
evidence is found that ensures a drastic classified into beach forests, mangrove forests,
increase in the fungal infections of some wet evergreen forests, semi-evergreen forests,
cultivated plants. The leaf-feeding activity of moist deciduous forests, and grasslands.
leaf beetles (Not Butterflies) may increase the Additional Information:
plant’s susceptibility to bacterial or fungal y Indira Point, the southernmost point of
infections. India is the southern point of Great Nicobar
Statement 3 is correct. A fall in their population Island.
would also lead to a fall in their predator y Andaman and Nicobar Islands are separated
population like the wasps, spiders, and birds. by a water body which is called the Ten-
A butterfly needs two types of plants: Nectar degree channel.
plants for adults and larval host plants for

Ecosystem and Ecology-Explanation 241


unacademy.com | Download the Unacademy app
Give your feedback here: Link
y Barren Island, the only active volcano in below.
India is also situated in the Nicobar Islands.
Elimination Technique: During
y The coastal line has some coral deposits photosynthesis carbon dioxide is taken
and beautiful beaches. These islands out of the atmosphere and converted into
receive convectional rainfall and have an organic compounds and oxygen. We can
equatorial type of vegetation. eliminate options A and D. So, the most
y Some important mountain peaks in appropriate option is C.
Andaman and Nicobar Islands are Saddle
peak, Mount Diavolo, Mount Koyob, and
Mount Thuillier. 22. Answer: (c)
Animals can be divided into two groups based
on how they regulate body temperature:
21. Answer: (c) endotherms and ectotherms. Endotherms
The carbon cycle describes the process in generate most of the heat they need internally.
which carbon atoms continually travel from When it’s cold out, they increase metabolic
the atmosphere to the Earth and then back heat production to keep their body temperature
into the atmosphere. Since our planet and its constant. Because of this, the internal body
atmosphere form a closed environment, the temperature of an endotherm is independent
amount of carbon in this system does not of the temperature of the environment.
change. Option (c) is correct: Hibernation is a state
of inactivity and metabolic depression in
endotherms. There are several animals that
hibernate: skunks, bees, snakes, groundhogs
(type of rodents) and amongst the well-known
bears and bats.

23. Answer: (a)


Organisms in the ecosystem are related to
each other through feeding mechanisms or
y Volcanic action, and decay of organic trophic levels, where one organism becomes
matter add carbon dioxide (CO2) to the food for the other. A sequence of organisms
carbon cycle on the planet Earth. [Option that feed on one another, forms a food chain.
(c) is correct] A food chain starts with producers and ends
with top carnivores.
y During photosynthesis, carbon dioxide is
converted into organic compounds and Option (a) is correct: Phytoplankton are
oxygen. Carbon is released back into the the primary producers in the oceans.
atmosphere due to respiration, organisms’ They include diatoms (unicellular algae),
death, volcanoes erupting, fires blazing, coccolithophores (unicellular, eukaryotic
fossil fuels being burned, and through a protist), Cyanobacteria (Bluegreen algae)-
variety of other mechanisms. Synechococcus, Prochlorococcus, Nostoc,
spirogyra, etc. Crustaceans form a very large
y Moreover, in the case of volcanoes, dormant
group of arthropods which includes crabs,
volcanoes as well as active volcanoes
lobsters, crayfish, shrimp, krill, and barnacles.
emit large fluxes of CO2, derived from the
Herrings are fish, and they eat crustaceans.
degassing of magma bodies in the crust

242 Ecosystem and Ecology-Explanation


unacademy.com | Download the Unacademy app
Give your feedback here: Link
and NADPH. This energy generated is used
in fixing CO and for carbohydrates, which is
glucose and stored for future use.

Elimination Technique: During


photosynthesis, energy is created, carbon
dioxide is absorbed (not released) and
Oxygen is added to (not taken from) the
atmosphere.

26. Answer: (b)


Decomposers break down complex organic
matter into inorganic substances like carbon
dioxide, water, and nutrients and the process
is called decomposition. Decomposition is
largely an oxygen-requiring process. The rate
24. Answer: (a) of decomposition is controlled by the chemical
Arachnids include a diverse group of composition of detritus and climatic factors.
invertebrates: spiders, scorpions, ticks, mites, Option 1 is not correct. The virus is neither
etc. Many arachnids are spiders. Most of them dead nor living. Most viruses need a living host
are carnivorous, typically preying on insects, organism; they are more like a parasite. They
and keeping insect populations under control. cannot function as decomposers.
They are terrestrial, living on land and lack both
Option 2 and 3 are correct: Dead plant remains
wings and antennae. Crustaceans form a large,
such as leaves, bark, flowers, and dead
diverse arthropod taxon which includes such
remains of animals, including faecal matter,
animals as crabs, lobsters, crayfish, shrimps,
constitute detritus, which is the raw material
prawns, krill, woodlice, and barnacles.
for decomposition. The important steps in the
Option (a) is correct: A Mite, Spider and Scorpion process of decomposition are fragmentation,
belong to the category of Arachnids, while leaching, catabolism, humification, and
Crabs belong to the category of crustaceans. mineralisation. Detritivores (e.g., earthworms)
break down detritus into smaller particles. This
25. Answer: (b) process is called fragmentation. Through the
process of leaching, water-soluble inorganic
Option (b) is correct: Photosynthesis is the nutrients go down into the soil horizon and get
process in which autotrophs such as green precipitated as unavailable salts. Bacterial and
plants use carbon dioxide and water in fungal enzymes degrade detritus into simpler
presence of sunlight to synthesise nutrients inorganic substances. This process is called
such as carbohydrates in the form of glucose catabolism.
and release oxygen. In this carbon dioxide
is reduced to glucose and water is oxidised
to release oxygen. During the process of 27. Answer: (c)
photosynthesis in plants, the free energy of Option (c) is correct: In the grasslands, trees
light is harvested by photosynthetic pigments do not replace the grasses as a part of an
which is chlorophyll in the chloroplast. This ecological succession because of water limits
free energy of light is trapped and used in the and fire. Grassland vegetation grows in areas
synthesis of chemical energy in molecules ATP of moderate to low amounts of rainfall. This

Ecosystem and Ecology-Explanation 243


unacademy.com | Download the Unacademy app
Give your feedback here: Link
scarcity of water limits the growth of grasses 28. Answer: (c)
and also the conversion of grasslands in the Net ecosystem production (NEP), defined
forest. as the difference between gross primary
y Succession is a universal process of production and total ecosystem respiration,
directional change in vegetation, on an represents the total amount of organic carbon
ecological time scale. Succession occurs in an ecosystem available for storage, export
when a series of communities replace one as organic carbon, or non-biological oxidation
another due to large-scale destruction to carbon dioxide through fire or ultraviolet
either natural or manmade. oxidation.
y This process continues - one community y Net Primary Production (NPP) is the net
replacing another community until a stable, amount of primary production after the
mature community develops. Succession costs of plant respiration are included.
is a progressive series of changes that y The correct sequence of ecosystems in
leads to the establishment of a relatively the order of decreasing productivity is
stable climax community. The first plant Mangroves, grasslands, lakes, and oceans.
to colonise an area is called the pioneer [Option (c) is correct]
community. The final stage of succession
is called the climax community. The stages Elimination Technique: Mangroves have
leading to the climax community are called greater productivity than the oceans. So,
successional stages. we can eliminate the options A and D.
y Succession is characterised by the
following: increased productivity, the shift 29. Answer: (b)
of nutrients from’ the reservoirs, increased
Electronic waste (e-waste) typically includes
diversity of organisms with increased niche
discarded computer monitors, motherboards,
development, and a gradual increase in the
mobile phones and chargers, compact discs,
complexity of food webs.
headphones, television sets, air conditioners,
and refrigerators.
Option (b) is correct:

244 Ecosystem and Ecology-Explanation


unacademy.com | Download the Unacademy app
Give your feedback here: Link
30. Answer: (d)
Option (d) is correct: Acid rain refers to the ways in which acid from the atmosphere is deposited
on the earth’s surface. Oxides of nitrogen and sulphur which are acidic in nature can be blown by
the wind along with solid particles in the atmosphere and finally settle down either on the ground
as dry deposition or in water, fog and snow as wet deposition.

y Acid rain is a byproduct of a variety of such as iron, lead, and copper into the
human activities that emit the oxides of drinking water. Acid rain damages buildings
sulphur and nitrogen into the atmosphere. and other structures made of stone or
As mentioned earlier, the burning of metal. The Taj Mahal in India has been
fossil fuels (which contain sulphur and affected by acid rain.
nitrogenous matter) such as coal and oil in
power stations and furnaces or petrol and
31. Answer: (a)
diesel in motor engines produces sulphur
dioxide and nitrogen oxides. SO2 and NO2 Statement 1 is correct: Organisms in the
after oxidation and reaction with water are ecosystem are related to each other through
major contributors to acid rain because feeding mechanisms or trophic levels, i.e.,
polluted air usually contains particulate one organism becomes food for the other.
matter that catalyses oxidation. A sequence of organisms that feed on one
another, form a food chain. A food chain starts
y Acid rain is harmful to agriculture, trees,
with producers and ends with top carnivores.
and plants as it dissolves and washes away
nutrients needed for their growth. Statement 2 is not correct: A food chain
represents only one part of the food or energy
y It causes respiratory ailments in human
flow through an ecosystem and implies a
beings and animals. It corrodes water pipes
resulting in the leaching of heavy metals simple, isolated relationship, which seldom

Ecosystem and Ecology-Explanation 245


unacademy.com | Download the Unacademy app
Give your feedback here: Link
occurs in ecosystems. Food chains are not
found within the populations of a species (dogs
would not eat dogs).
Statement 3 is not correct: A food web
illustrates all possible transfers of energy Elimination Technique: Food chains are
and nutrients among the organisms in an not found within the populations of a
ecosystem, whereas a food chain traces only species. Ex- Tiger won’t eat Tiger. Hence
one pathway of the food. statement 2 is not correct. So, option A is
the most appropriate answer.
Picture of a food web, consisting of many food
chains:
32. Answer: (c)
Plants convert sunlight into usable energy
by the process of photosynthesis, and a
key component is chlorophyll, which is the
compound that makes the leaves and stems
look green. Nitrogen is an important element
in this compound, and if the plant lacks this
nutrient, the leaves turn yellow and wilt, and
the plant dies.
Option (c) is correct: Excretion of urea by
animals adds nitrogen to the soil, as urea is
the main nitrogen-containing substance in the
urine of mammals, therefore urea is widely
used in fertilizers as the convenient source
of nitrogen. Burning coal does not produce
Nitrogen or its compounds. Death of vegetation
adds up in soil organic matter including humus.
As organic matter decomposes inorganic
nitrogen is released into the soil.

246 Ecosystem and Ecology-Explanation


unacademy.com | Download the Unacademy app
Give your feedback here: Link
33. Answer: (b) pollen grains from the particular flowers visit
Option (b) is correct: Each organism has an the stigma and become a pollen tube with a
invariably defined range of conditions that style length, which links both the stigma and
it can tolerate, diversity in the resources it ovary. After the accomplishment of the pollen
utilizes,, and a distinct functional role in the tube, the pollen grain begins sharing sperm
ecological system, all these together comprise cells from the grain to the ovary. After some
its niche (Ecological niche). time, fertilisation in plants occurs when the
sperm cells are placed at the ovary and egg
y The term niche means the sum of all the cells. The seed comes out from the parent
activities and relationships of a species by plant enabling it to flourish into a plant and
which it uses the resources in its habitat continue reproductive production with the use
for its survival and reproduction. of the pollination approach.
y No two species in a habitat can have the
same niche. This is because, if two species
occupy the same niche, they will compete 35. Answer: (c)
with one another until one is displaced. The The Millennium Ecosystem Assessment
most important resources available in the (MA) was called for by the United Nations
niches of animals are food and shelter while Secretary-General Kofi Annan in 2000. Initiated
in the case of plants, they are moisture and in 2001, the objective of the MA was to assess
nutrients (phosphorus and nitrogen). the consequences of ecosystem change for
Additional Information: human well-being and the scientific basis for
action needed to enhance the conservation
y Habitat: Habitat is thus the physical and sustainable use of those systems and their
environment in which an organism lives. contribution to human well-being.
Earth has four major habitats: Terrestrial,
Freshwater, Estuarine (where rivers meet The findings provide a state-of-the-art
the ocean), and Oceanic. scientific appraisal of the condition and trends
in the world’s ecosystems and the services
y Ecotone: Ecotone is a transitional zone they provide such as clean water, food, forest
between two ecosystems. E.g., the products, flood control, and natural resources
mangrove forests represent an ecotone and the options to restore, conserve or enhance
between marine and terrestrial ecosystems. the sustainable use of ecosystems.
Other examples are - grassland, estuary,
and riverbank. Option (c) is correct: The category of support
services is an important distinction to make
when discussing the ecosystem services
34. Answer: (d) provided by a natural area. The Millennium
Option (d) is correct: Pollination is the act of Ecosystem Assessment has defined four
transferring pollen grains from the male anther broad categories of ecosystem services. These
of a flower to the female stigma. The process are: supporting, regulating, provisioning and
results in fertilisation which produces seeds cultural services. The supporting services
and, in some cases, fruits. Pollinators are which are necessary for the production of all
Bees, Flies, Beetles, Birds, bats, butterflies, other ecosystem services are food, nutrient
moths, wasps, and small mammals. They place cycling and primary production.
themselves on flowers to drink nectar or feed
off of pollen and ferry pollen grains as they 36. Answer: (a)
shift from one place to another.
Phytoplankton are microscopic marine algae.
The process of pollination starts when the Phytoplankton is the base of several aquatic

Ecosystem and Ecology-Explanation 247


unacademy.com | Download the Unacademy app
Give your feedback here: Link
food webs. In a balanced ecosystem, they Microorganisms are natural decomposers as
provide food for a wide range of sea creatures. they possess an enormous ability to utilise
Phytoplankton, also known as microalgae, are diverse types of organic substances as a source
similar to terrestrial plants in that they contain of energy and convert toxic ones into harmless
chlorophyll and require sunlight in order to live by-products. They do not have any relevance
and grow. Scientists have, for the first time, in the context of ecosystem productivity.
found evidence that microscopic marine algae Organisms and decomposer microorganisms
known as “phytoplankton” have been declining would remain where they are, no movement
globally over the 20th century. to the surface can be noticed at the upwelling
Statement 1 is correct: Like land plants, zone.
phytoplankton have chlorophyll to capture Statement 2 is correct: Upwelling is an
sunlight, and they use photosynthesis to turn oceanographic phenomenon that involves
it into chemical energy. They consume carbon the wind-driven motion of dense, cooler,
dioxide and release oxygen. All phytoplankton and usually, nutrient-rich water towards the
photosynthesize, but some get additional ocean surface, replacing the warmer, usually
energy by consuming other organisms. nutrient-depleted surface water. The increased
Therefore, if the phytoplankton of an ocean is availability in upwelling regions results in high
completely destroyed for some reason, then levels of primary productivity and thus fishery
the ocean as a carbon sink would be adversely production. Approximately 25% of the total
affected. global marine fish catches come from five
Statement 2 is correct: Phytoplankton forms upwellings that occupy only 5% of the total
the basis of the marine food chain and ocean area.
sustains diverse assemblages of species
ranging from tiny zooplankton to large marine 38. Answer: (a)
mammals, seabirds, and fish. “Phytoplankton
is the fuel on which marine ecosystems run. Option (a) is correct: The soil of the rainforest
A decline of phytoplankton affects everything is deficient in nutrients. Here rainforest means
up the food chain, including humans. Long- Rainfall throughout the year. It rains almost
term phytoplankton declines were negatively every day during this the topsoil is continuously
correlated with rising sea surface temperatures washed away which may result in nutrients
and changing oceanographic conditions. being washed away.

Statement 3 is not correct: Density of ocean y It will start leaching of nutrients and very
water at the sea surface is about 1027 kg/m 3. little fertility remains in the topsoil. In this
There are two main factors that make ocean case, most of the seeds don’t germinate for
water more or less dense than about 1027 years.
kg/m 3: the temperature of the water and the y The Regeneration Process will be very slow
salinity of the water. Ocean water gets denser (it may take decades). But the layer below
as temperature goes down and vice versa. topsoil (subsoil) is very fertile. So, plants
grow very quickly once their roots reach
the sub-soil and if they receive enough
37. Answer: (b) sunlight.
Statement 1 and statement 3 are not correct:

248 Ecosystem and Ecology-Explanation


unacademy.com | Download the Unacademy app
Give your feedback here: Link
3 Environmental Pollution

1. Consider the following statements 4.


With reference to polyethylene
regarding mercury pollution: (2023) terephthalate, the use of which is so
1. Gold mining activity is a source of widespread in our daily lives, consider the
mercury pollution in the world. following statements : (2022)

2. Coal-based thermal power plants cause 1. Its fibres can be blended with wool
mercury pollution. and cotton fibres to reinforce their
properties.
3. There is no known safe level of exposure
to mercury. 2. Containers made of it can be used to
store any alcoholic beverage.
How many of the above statements are
correct? 3. Bottles made of it can be recycled into
other products.
(a) Only one
4. Articles made of it can be easily
(b) Only two
disposed of by incineration without
(c) All three causing greenhouse gas emissions.
(d) None Which of the statements given above are
correct?

2. Consider the following : (2022) (a) 1 and 3

1. Carbon monoxide (b) 2 and 4

2. Nitrogen oxide (c) 1 and 4

3. Ozone (d) 2 and 3

4. Sulphur dioxide
Excess of which of the above in the 5. In the context of WHO Air Quality Guidelines,
environment is/are cause(s) of acid rain ? consider the following statements : (2022)

(a) 1, 2 and 3 1. The 24-hour mean of PM2.5 should not


exceed 15 μg/m3 and annual mean of
(b) 2 and 4 only
PM2.5 should not exceed 5μg/m3.
(c) 4 only
2. In a year, the highest levels of ozone
(d) 1, 3 and 4 pollution occur during the periods of
inclement weather.

3. Among the following crops, which one is 3. PM10 can penetrate the lung barrier and
the most important anthropogenic source enter the bloodstream.
of both methane and nitrous oxide ? (2022) 4. Excessive ozone in the air can trigger
(a) Cotton asthma.

(b) Rice Which of the statements given above are


correct?
(c) Sugarcane
(a) 1, 3 and 4
(d) Wheat
(b) 1 and 4 only

Environmental Pollution 249


unacademy.com | Download the Unacademy app
Give your feedback here: Link
(c) 2, 3 and 4 power.
(d) 1 and 2 only 3. Its use causes sulphur emissions into
the environment.

6. Magnetite particles, suspected to cause Which of the statements given above are
neurodegenerative problems, are generated correct?
as environmental pollutants from which of (a) 1 and 2 only
the following? (2021) (b) 2 and 3 only
1. Brakes of motor vehicles (c) 1 and 3 only
2. Engines of motor vehicles (d) 1, 2 and 3
3. Microwave stoves within homes
4. Power plants 9. Consider the following statements: (2020)
5. Telephone lines 1. Coal ash contains arsenic, lead and
Select the correct answer using the code mercury.
given below. 2. Coal-fired power plants release sulphur
(a) 1, 2, 3 and 5 only dioxide and oxides of nitrogen into the
(b) 1, 2 and 4 only environment.

(c) 3, 4 and 5 only 3. High ash content is observed in Indian


coal.
(d) 1, 2, 3, 4 and 5
Which of the statements given above is/are
correct?
7. Why is there a concern about copper (a) 1 only
smelting plants? (2021)
(b) 2 and 3 only
1. They may release lethal quantities of
carbon monoxide into the environment. (c) 3 only

2. The copper slag can cause the (d) 1, 2 and 3


leaching of some heavy metals into the
environment. 10. Which of the following are the reasons/
3. They may release sulphur dioxide as a factors for exposure to benzene pollution?
pollutant. (2020)
Select the correct answer using the code 1. Automobile exhaust
given below. 2. Tobacco smoke
(a) 1 and 2 only 3. Wood burning
(b) 2 and 3 only 4. Using varnished wooden furniture
(c) 1 and 3 only 5. Using products made of polyurethane
(d) 1, 2 and 3 Select the correct answer using the code
given below:
8. With reference to furnace oil, consider the (a) 1, 2 and 3 only
following statements : (2021) (b) 2 and 4 only
1. It is a product of oil refineries. (c) 1, 3 and 4 only
2. Some industries use it to generate

250 Environmental Pollution


unacademy.com | Download the Unacademy app
Give your feedback here: Link
(d) 1, 2, 3, 4 and 5 nitrogen compounds into the
environment.

11. In India, the use of carbofuran, methyl Which of the statements given above is/are
parathion, phorate and triazophos is viewed correct?
with apprehension. These chemicals are (a) 1 and 3 only
used as (2019) (b) 2 and 3 only
(a) pesticides in agriculture. (c) 2 only
(b) preservatives in processed foods. (d) 1, 2 and 3
(c) fruit-ripening agents.
(d) moisturising agents in cosmetics. 15. Why is there a great concern about the
‘microbeads' that are released into the
12. In the context of which one of the following environment? (2019)
are the terms ‘pyrolysis and plasma (a) They are considered harmful to marine
gasification’ mentioned? (2019) ecosystems.
(a) Extraction of rare earth elements (b) They are considered to cause skin
(b) Natural gas extraction technologies cancer in children.

(c) Hydrogen fuel-based automobiles (c) They are small enough to be absorbed
by crop plants in irrigated fields.
(d) Waste-to-energy technologies
(d) They are often found to be used as food
adulterants.
13. Consider the following: (2019)
1. Carbon monoxide 16. Which of the following is/are the possible
2. Methane consequence/s of heavy sand mining in
3. Ozone riverbeds? (2018)

4. Sulphur dioxide 1. Decreased salinity in the river

Which of the above are released into the 2. Pollution of groundwater


atmosphere due to the burning of crop/ 3. Lowering of the water - table
biomass residue? Select the correct answer using the code
(a) 1 and 2 only given below:
(b) 2, 3 and 4 only (a) 1 only
(c) 1 and 4 only (b) 2 and 3 only
(d) 1, 2, 3 and 4 (c) 1 and 3 only
(d) 1, 2 and 3
14. Consider the following statements: (2019)
1. Agricultural soils release nitrogen oxides 17. In the context of solving pollution problems,
into the environment. what is/are the advantage/advantages of
2. Cattle release ammonia into the the bioremediation technique? (2017)
environment. 1. It is a technique for cleaning up pollution
3. The poultry industry releases reactive by enhancing the same biodegradation
process that occurs in nature.

Environmental Pollution 251


unacademy.com | Download the Unacademy app
Give your feedback here: Link
2. Any contaminant with heavy metals such 20. Brominated flame retardants are used in
as cadmium and lead can be readily and many household products like mattresses
completely treated by bioremediation and upholstery. Why is there some concern
using microorganisms. about their use? (2014)
3. Genetic engineering can be used to 1. They are highly resistant to degradation
create microorganisms specifically in the environment.
designed for bioremediation. 2. They are able to accumulate in humans
Select the correct option using the given and animals.
code below: Select the correct answer using the code
(a) 1 only given below:
(b) 2 and 3 only (a) 1 only
(c) 1 and 3 only (b) 2 only
(d) 1, 2 and 3 (c) Both 1 and 2
(d) Neither 1 nor 2
18.
Biological Oxygen Demand (BOD) is a 21.
There is some concern regarding the
standard criterion for (2017) nanoparticles of some chemical elements
(a) Measuring oxygen levels in the blood that are used by the industry in the
manufacture of various products. Why?
(b)
Computing oxygen levels in forest (2014)
ecosystems
1. They can accumulate in the environment
(c) Pollution assay in aquatic ecosystems and contaminate water and soil.
(d) Assessing oxygen levels in high-altitude 2. They can enter the food chains.
regions
3. They can trigger the production of free
radicals.
19. In the cities of our country, which among Select the correct answer using the code
the following atmospheric gases are given below:
normally considered in calculating the
value of the Air Quality Index? (2016) (a) 1 and 2 only

1. Carbon dioxide (b) 3 only

2. Carbon monoxide (c) 1 and 3 only

3. Nitrogen dioxide (d) 1, 2 and 3

4. Sulphur dioxide
5. Methane 22. Which of the following are some important
pollutants released by the steel industry in
Select the correct answer using the code India? (2014)
given below.
1. Oxides of sulphur
(a) 1, 2 and 3 only
2. Oxide of nitrogen
(b) 2, 3 and 4 only
3. Carbon monoxide
(c) 1, 4 and 5 only
4. Carbon dioxide
(d) 1, 2, 3, 4 and 5
Select the correct answer using the code
given below:

252 Environmental Pollution


unacademy.com | Download the Unacademy app
Give your feedback here: Link
(a) 1, 3 and 4 only Which of the statements given above is/are
(b) 2 and 3 only correct?

(c) 1 and 4 only (a) 1, 2 and 3 only

(d) 1, 2, 3 and 4 (b) 4 only


(c) 1, 3 and 4 only

23. Which of the following can be found as (d) 1, 2, 3 and 4


pollutants in the drinking water in some
parts of India? (2013) 26.
Lead, ingested or inhaled, is a health
1. Arsenic hazard. After the addition of lead to petrol
2. Sorbitol has been banned, what still are the sources
of lead poisoning? (2012)
3. Fluoride
1. Smelting units
4. Formaldehyde
2. Pens and pencils
5. Uranium
3. Paints
Select the correct answer using the code
given below: 4. Hair oils and cosmetics

(a) 1 and 3 only Select the correct answer using the code
given below:
(b) 2, 4 and 5 only
(a) 1, 2 and 3 only
(c) 1, 3 and 5 only
(b) 1 and 3 only
(d) 1, 2, 3, 4 and 5
(c) 2 and 4 only
(d) 1, 2, 3 and 4
24. Photochemical smog is a resultant of the
reaction among (2013)
(a) NO2, O3 and peroxyacetyl nitrate in the 27. There is a concern over the increase in
presence of sunlight harmful algal blooms in the seawater of
India. What could be the causative factors
(b) CO, O2 and peroxyacetyl nitrate in the for this phenomenon? (2011)
presence of sunlight
1. Discharge of nutrients from the
(c) CO, CO2 and NO2 at low temperature estuaries.
(d) high concentration of NO2, O3 and CO in 2. Run-off from the land during the
the evening monsoon.
3. Upwelling in the seas.
25. Consider the following statements: (2012) Select the correct answer from the code
Chlorofluorocarbons, known as ozone given below:
depleting substances, are used (a) 1 only
1. In the production of plastic foams (b) 1 and 2 only
2. In the production of tubeless tyres (c) 2 and 3 only
3.
In cleaning certain electronic (d) 1, 2 and 3
components
4. As pressurizing agents in aerosol cans

Environmental Pollution 253


unacademy.com | Download the Unacademy app
Give your feedback here: Link
28. Consider the following: (2011) (b) Presence of prominent polar front and
1. Carbon dioxide stratospheric clouds; and inflow of
chlorofluorocarbons
2. Oxides of nitrogen
(c) Absence of polar front and stratospheric
3. Oxides of Sulphur clouds; and inflow of methane and
Which of the above is/are the emission/ chlorofluorocarbons
emissions from coal combustion at thermal (d) Increased temperature at polar region
power plants? due to global warming
(a) 1 only
(b) 2 and 3 only 30. Recently, “oilzapper’’ was in the news.
(c) 1 and 3 only What is it? (2011)
(d) 1, 2, and 3 (a) It is an eco-friendly technology for the
remediation of oily sludge and oil spills.

29.
The formation of ozone holes in the (b) It is the latest technology developed for
Antarctic Region has been a cause of undersea oil exploration.
concern. What could be the reason for the (c) It is a genetically engineered high
formation of this hole? (2011) biofuel-yielding maize variety.
(a) Presence of prominent tropospheric (d) It is the latest technology to control
turbulence; and inflow of the accidentally caused flames from oil
chlorofluorocarbons wells.

254 Environmental Pollution


unacademy.com | Download the Unacademy app
Give your feedback here: Link
Environmental Pollution-
3 Explanation
1. Answer: (c) 3. Answer: (b)
Statement 1 is correct: Mercury is commonly Option (b) is correct: Rice cultivation has been
used in small-scale and artisanal gold identified as one of the important sources
mining processes to extract gold from ore. of GHGs, namely methane (CH4) and nitrous
Unfortunately, this gold mining technique oxide (N2O).
releases mercury into the environment, leading According to a global analysis by the
to serious environmental and health concerns. Environmental Defense Fund (EDF) in the US,
Statement 2 is correct: When coal is burned methane and nitrous oxide emissions from rice
for electricity generation in these power plants, farms could have the same long-term warming
trace amounts of mercury present in the coal impact as about 600 coal plants.
are released into the atmosphere. This mercury The researchers investigated greenhouse gas
can then undergo long-range atmospheric emissions from rice farms across southern
transport and deposit into ecosystems, posing India. They found that nitrous oxide emissions
environmental and health risks. from rice can contribute up to 99 % of the total
Statement 3 is correctThere is no known safe climate impact of rice cultivation at a variety of
level of exposure to mercury. Mercury is a highly intermittently flooded farms
toxic substance that can have detrimental
effects on human health, even at low levels
of exposure.Once inside the body, mercury 4. Answer: (a)
can accumulate and persist for a long time, Statement 1 is correct: Polyethylene
particularly in the form of methylmercury, terephthalate is a polymer created by the
which is the most concerning form of mercury combination of two monomers: modified
in terms of its health effects. ethylene glycol and purified terephthalic acid.
Its fibers can be blended with wool and cotton
fibers to reinforce their properties.
2. Answer: (b)
Statement 2 is not correct: PET Containers
Option (b) is correct: Acid rain, or acid are widespread in our lives But it is not
deposition, is a broad term that includes any recommended for storage of any alcoholic
form of precipitation with acidic components, beverage.
such as sulfuric or nitric acid that fall to the
ground from the atmosphere in wet or dry Statement 3 is correct: Polyethylene
forms. This can include rain, snow, fog, hail or terephthalate (PET) is a highly recyclable
even dust that is acidic. plastic resin and a form of polyester.

Acid rain results when sulfur dioxide (SO2) Statement 4 is not correct: Emission of
and nitrogen oxides (NOX) are emitted into the greenhouse gases is one of the major
atmosphere and transported by wind and air disadvantages associated with the disposal of
currents. The SO2 and NOX react with water, PET by incineration process.
oxygen and other chemicals to form sulfuric Elimination Technique: Polyethylene
and nitric acids. These then mix with water and Terephthalate (PET or PETE) is plastic,
other materials before falling to the ground. so while incinerating the PET it will emit
greenhouse gasses.

Environmental Pollution-Explanation 255


unacademy.com | Download the Unacademy app
Give your feedback here: Link
5. Answer: (b) copper from copper concentrates through
Statement 1 is correct: WHO’s new guidelines multiple sulphide oxidizing stages.
recommend air quality levels for six pollutants Option (b) is correct: The copper slag can
— particulate matter (PM), ozone (O₃), nitrogen cause the leaching of some heavy metals into
dioxide (NO₂) sulfur dioxide (SO₂), and carbon the environment. Organic vapours and sulfur
monoxide (CO) — that are not just health oxides resulting from secondary smelting
hazards but they also give rise to other roasting operations and fuel combustion can
damaging pollutants. The recommendation for cause smog, containing ozone, fine airborne
PM 2.5 is that the annual average should not particles, nitrogen oxides, sulfur dioxide and
exceed 5 micrograms per cubic meter or 15 carbon monoxide. So, it may release sulphur
micrograms per cubic meter in a day. dioxide as a pollutant.
Statement 2 is not correct: Ozone levels are Copper smelters use carbon monoxide as
typically highest during the afternoon hours a reducing agent for copper oxide. Carbon
of the summer months when the influence monoxide is not an end product of the
of direct sunlight is the greatest. Inclement reaction. Hence, it is unlikely that a smelter
weather is unpleasant, especially with cold will discharge dangerous levels of carbon
wind and rain. So, Ozone pollution will be low monoxide into the atmosphere.
during inclement weather.
CuO + CO → Cu + CO2.
Statement 3 is not correct: Particulate matter
2.5 (PM2.5) (not PM10) are such small particles
that can penetrate the human lung barrier and 8. Answer: (d)
enter the blood system. Statement 1 is correct: Furnace oil, commonly
Statement 4 is correct: Excessive Ozone known as fuel oil, is a viscous black residual fuel
like other air pollutants in the air can trigger predominantly composed of heavy components
asthma. from the crude distillation unit, short residue,
and clarified oil from the fluidized catalytic
cracker unit.
6. Answer: (b)
Statement 2 is correct: Furnace oil is commonly
Option (b) is correct: Magnetite particles used in Steam boilers which in turn are used in
suspected to cause neurodegenerative power plants, ships, and industrial operations.
problems are generated from the following:
Statement 3 is correct: Using furnace oil
y Brakes of motor vehicles contributes to high sulphur dioxide (SO2)
y Engines of motor vehicles emission and secondary sulphate formation as
particulate matter (PM).
y Power plants
Other sources of magnetite nanoparticles
include open fires and poorly sealed stoves 9. Answer: (d)
within homes. Coal ash, also referred to as coal combustion
Currently, there is no study to suggest that residuals or CCRs, is produced primarily
they are produced by Microwave stoves within from the burning of coal in coal-fired power
homes and Telephone lines plants. Coal ash includes several by-products
produced from burning coal, including Fly
Ash, a very fine, powdery material composed
7. Answer: (b) mostly of silica made from the burning of
Copper smelting plants separate elemental finely ground coal in a boiler; Bottom Ash, a
coarse, angular ash particle that is too large

256 Environmental Pollution-Explanation


unacademy.com | Download the Unacademy app
Give your feedback here: Link
to be carried up into the smoke stacks, so it exposure to tobacco smoke is also a significant
forms in the bottom of the coal furnace; Boiler source of exposure to benzene pollution.
Slag, molten bottom ash from slag taps and Benzene is highly volatile, and exposure occurs
cyclone type furnaces that turn into pellets mostly through inhalation.
that have a smooth glassy appearance after it Statement 3 is correct: Wood burning is also a
is cooled with water; Flue Gas Desulfurization source of benzene pollution.
Material, a material leftover from the process
of reducing sulphur dioxide emissions from
a coal-fired boiler that can be a wet sludge 11. Answer: (a)
consisting of calcium sulphite or calcium Option (a) is correct: Pesticides are chemical
sulphate or dry powdered material that is a compounds that are used to kill pests;
mixture of sulphites and sulphates. including insects, rodents, fungi, and unwanted
Statement 1 is correct: Coal ash commonly plants (weeds). Pesticides include herbicides
contains some of the earth’s deadliest toxics: for destroying weeds and other unwanted
arsenic, lead, mercury, cadmium, chromium, vegetation, insecticides for controlling a wide
and selenium. variety of insects, fungicides used to prevent
Statement 2 is correct: Thermal power plants the growth of moulds and mildew, disinfectants
produce large amounts of nitrogen oxides and for preventing the spread of bacteria, and
sulphur dioxide—the pollutants that cause acid compounds used to control mice and rats.
rain—when they burn fossil fuels, especially Carbofuran, methyl parathion, phorate and
coal, to produce energy. triazophos are used as pesticides in Agriculture.
Statement 3 is correct: India’s domestic coal Additional Information:
reserves have a high ash content—up to 40 to The use of these pesticides is banned in India as
45 percent. As a result, more coal is required these pesticides are highly hazardous with the
to generate one-kilowatt hour of electricity. potential to cause severe health problems such
This in turn means higher emissions. Reducing as hormonal changes, carcinogenic, neurotoxic,
the ash content will mean lower amounts of reproductive and developmental health effects
coal will be burnt for every unit of electricity as well as environmental impacts such as toxic
generated, and that would mean lower to bees. Acephate, carbaryl, dimethoate and
emissions of pollutants. Coal washing is one flubendiamide are less hazardous pesticides.
possible way to tackle the problem of high ash
content in coal.
12. Answer: (d)
Option (d) is correct: Waste-to-Energy (WtE)
10. Answer: (a)
technologies consist of any waste treatment
Statement 1 is correct: Human exposure to process that creates energy in the form of
benzene has been associated with a range of electricity, heat, or transport fuels (e.g. diesel)
acute and long-term adverse health effects from a waste source.
and diseases, including cancer and aplastic
Pyrolysis is a common technique used to
anemia. Exposure can occur occupationally and
convert plastic waste into energy, in the form
domestically as a result of the ubiquitous use
of solid, liquid and gaseous fuels. Pyrolysis is
of benzene-containing petroleum products,
the thermal degradation of plastic waste at
including motor fuels and solvents. Automobile
different temperatures (300–900°C), in the
exhaust accounts for the largest source of
absence of oxygen, to produce liquid oil.
benzene pollution in the general environment.
Municipal solid waste (MSW) is considered
Statement 2 is correct: Active and passive

Environmental Pollution-Explanation 257


unacademy.com | Download the Unacademy app
Give your feedback here: Link
one source of renewable energy, and plasma air pollution, namely nitric oxide and nitrogen
gasification technology is one of the leading- dioxide) and about 7 million kg of nitrous oxide
edge technologies available to harness this (N2O) per year.
energy. Plasma gasification technology treats Agriculture remains the largest contributor
wastes and generates syngas to produce to nitrogen emissions, the non-agricultural
electricity or heat. emissions of nitrogen oxides and nitrous oxide
are growing rapidly, with sewage and fossil-
13. Answer: (d) fuel burning for power, transport and industry-
leading the trend.
Option (d) is correct: Burning of crop or
biomass residue means burning of living or Statement 1 is correct: Agricultural soils
dead vegetation including grassland, forest, contributed to over 70% of N2O emissions from
and agricultural waste for fuel. Recent India in 2010, followed by wastewater (12%)
crop harvesting practices use mechanical and residential and commercial activities (6%).
harvesters in the rice and wheat farm system Statement 2 is correct: Cattle account for 80%
in India which leave behind large quantities of of the ammonia production, though their annual
crop residue in the field. However, there is no growth rate is 1%, due to a stable population.
suitable method available for managing crop India is globally the biggest source of ammonia
residues. emission, nearly double that of NOx emissions.
Crop residues/biomass burning is a cheap and Statement 3 is correct: The poultry industry,
easiest method to dispose of the leftover crop on the other hand, with an annual growth
residues (wheat, rice, sugarcane, etc.) after rate of 6%, recorded an excretion of reactive
harvesting, for land clearing and pest control. nitrogen compounds of 0.415 tonnes in 2016.
Burning of crop residues is a common approach That is anticipated to increase by almost 250%
to eliminate waste after harvesting all over the to 1.089 tonnes by 2030.
world. Burning of these residues emit gases
like sulphur dioxide (SO2), oxides of nitrogen
(NOx), carbon dioxide (CO2), carbon monoxide 15. Answer: (a)
(CO), black carbon (BC), organic carbon (OC), Option (a) is correct: Microbeads are small,
methane (CH4), volatile organic compounds solid, manufactured plastic particles that
(VOC), non-methane hydrocarbons (NMHCs), are less than 5mm in diameter and do not
ozone (O3), and aerosols, etc which affect the degrade or dissolve in water. They may be
global atmospheric chemistry and climate. added to a range of products, including rinse-
off cosmetics, personal care and cleaning
products. Microbeads are used in these
14. Answer: (d) products for a variety of purposes. This
The Indian Nitrogen Assessment assesses the includes as an exfoliant, a bulking agent, to
sources, impacts, trends and future scenarios prolong shelf life, or for the controlled release
of reactive nitrogen in the Indian environment. of active ingredients.
Nitrogen particles make up the largest fraction Microbeads are not captured by most
of PM 2.5, the class of pollutants closely linked wastewater treatment systems. If washed down
to cardiovascular and respiratory illness. drains after use, they can end up in our rivers,
While the burning of crop residue is said to be lakes and oceans. These tiny plastics persist in
a key contributor to winter smog in many parts the environment and have a damaging effect
of North India, it contributes over 240 million on marine life, the environment and human
kg of nitrogen oxides (NOx: a generic term for health.
the nitrogen oxides that are most relevant for

258 Environmental Pollution-Explanation


unacademy.com | Download the Unacademy app
Give your feedback here: Link
from a contaminated site. It is a ‘treatment
16. Answer: (b) technique’ that uses naturally occurring
organisms to break down harmful materials
“Sand mining” is a practice that is used to into less toxic or non-toxic materials.
extract sand from various environments, such
as beaches, and inland dunes and dredged from Statement 1 is correct: Bioremediation
ocean beds and riverbeds of deltaic regions. provides a technique for cleaning up pollution
Mining is in operation on all the continents by enhancing the same biodegradation
of the Globe. Environmental problems occur processes that occur in nature. Microorganisms
when the rate of extraction of sand, gravel and (primarily bacteria and fungi) are nature’s
other materials exceeds the rate of deposition. original recyclers. Their capability to transform
natural and synthetic chemicals into sources
Option (b) is correct: Sand is vital for the of energy and raw materials for their growth
sustenance of rivers. It has several impacts on suggests that expensive chemical or physical
the river environment: remediation processes might be replaced with
y It disturbs and completely removes the biological processes that are lower in cost and
habitat from the mined zones. Moreover, more environmentally friendly.
it leads to changes in its channel form, Statement 2 is not correct. Bioremediation is
physical habitats, and food webs – the a technique used to remove environmental
river’s ecosystem. contaminants from the ecosystem. It utilises
y It also increases the velocity of flow in the biological mechanisms inherent in
rivers which destroys the flow-regime and microbes and plants to eradicate hazardous
eventually erodes the riverbanks. pollutants and restore the ecosystem to its
y The sediment in the mining site, if it original condition. The basic principles of
contains toxic material due to long time bioremediation involve reducing the solubility of
accumulation, will cause groundwater these environmental contaminants by changing
pollution. pH, the redox reactions and the adsorption of
contaminants from polluted environments. Not
y The riverbed becomes dry due to exposure
all contaminants are readily treated through
to solar radiation, decreasing the surface
the use of bioremediation; heavy metals such
and groundwater. It causes a lowering of
as cadmium, and lead are not readily absorbed
the water tables.
or captured by microorganisms.
y Depletion of sand in the streambed causes
Statement 3 is correct. Biotechnology utilises
the deepening of rivers and estuaries,
the application of genetic engineering to
and the enlargement of river mouths and
improve efficiency and cost, which are key
coastal inlets. It leads to decreased salinity
factors in the future widespread exploitation of
in the river.
microorganisms to reduce the environmental
Elimination Technique: Heavy sand mining burden of toxic substances. Genetic engineering
in riverbeds increased salinity in the river. can be used to create microorganisms
Hence, statement 1 is incorrect and we are specifically designed for bioremediation.
remaining with option B only. Genetically modified microorganisms have
shown the potential for bioremediation
applications in groundwater, soil and activated
17. Answer: (c)
sludge environments, exhibiting enhanced
Bioremediation is a waste management degradative capabilities encompassing a wide
technique that includes the use of living range of chemical contaminants.
organisms to eradicate or neutralise pollutants

Environmental Pollution-Explanation 259


unacademy.com | Download the Unacademy app
Give your feedback here: Link
are PM10, PM2.5, Nitrogen dioxide (NO2),
Elimination Technique: The ‘Any Sulphur dioxide (SO2), Carbon monoxide (CO),
contaminant with heavy metals’ mentioned Ozone (O3), Ammonia (NH3), Lead (Pb).
in statement 2 makes it difficult to stand
true. So we can eliminate options B and D.
20. Answer: (c)
Brominated flame retardants (BFRs) are
18. Answer: (c)
mixtures of man-made chemicals that are
Water pollution by organic wastes is measured added to a wide variety of products, including
in terms of Biochemical Oxygen Demand (BOD). for industrial use, to make them less flammable.
BOD is the amount of dissolved oxygen needed They are used commonly in plastics, textiles,
by bacteria in decomposing the organic wastes and electrical/electronic equipment.
present in water. It is expressed in milligrams
Statement 1 is correct: Brominated flame
of oxygen per liter of water.
retardants (BFRs), which were detected
The higher value of BOD indicates the low extensively in environmental and biota samples
dissolved oxygen content of water. Since worldwide, have raised significant concerns
BOD is limited to biodegradable materials during past decades for their persistence,
only, therefore, it is not a reliable method of bioaccumulation and potential toxicity to the
measuring pollution load in the water. ecological environment and human health.
Option (c) is correct: The higher amounts of Statement 2 is correct: BFR can enter the
waste increase the rates of decomposition food chain and tend to bioaccumulate in living
and O2 consumption, thereby decreasing the organisms. BFR-treated products, whether in
dissolved oxygen content of water. The demand use or waste, leach BFRs into the environment
for O2 is directly related to the increasing and contaminate the air, soil, and water. These
input of organic wastes and is expressed as contaminants may then enter the food chain
the Biological Oxygen Demand (BOD) of water. where they mainly occur in food of animal
Thus, Biological Oxygen Demand (BOD) is a origin, such as fish, meat, milk, and derived
standard criterion for pollution assay in aquatic products.
ecosystems.
Additional Information:
There are main classes of BFRs, listed here
19. Answer: (b) with their common uses:
Air Quality Index (AQI) is a tool for effective Polybrominated diphenyl ethers (PBDEs):
communication of air quality status to people Plastics, textiles, electronic castings, circuitry.
in terms, which are easy to understand. It
Hexabromocyclododecane (HBCDDs): Thermal
transforms complex air quality data of various
insulation in the building industry.
pollutants into a single number (index value),
nomenclature and colour. Tetrabromobisphenol A (TBBPA) and other
phenols: Printed circuit boards, thermoplastics
There are six AQI categories, namely Good,
(mainly in TVs).
Satisfactory, Moderately Polluted, Poor, Very
Poor, and Severe. Each of these categories Polybrominated biphenyls (PBBs): Consumer
is decided based on ambient concentration appliances, textiles, plastic foams.
values of air pollutants and their likely health
impacts (known as health breakpoints).
21. Answer: (d)
Option (b) is correct: Eight pollutants or
Nanoparticles are ultrafine units with
atmospheric gases included in AQI calculation
dimensions measured in nanometers. They can

260 Environmental Pollution-Explanation


unacademy.com | Download the Unacademy app
Give your feedback here: Link
be introduced into the environment through groundwater quality monitoring indicate that
anthropogenic and natural routes; the natural the groundwater in major parts of the country
nanoparticles are considered to be non-toxic, is potable. However, some parts of various
whereas anthropogenic nanoparticles can states are contaminated by Salinity, Arsenic,
reveal a toxic effect. Fluoride, Iron, Nitrate, and Heavy metals
Statement 1 and statement 2 are correct: beyond the permissible limits of BIS. Moreover,
Nanoparticles can accumulate in the an international study has found widespread
environment and contaminate water and Uranium contamination in groundwater from
soil, from here they enter the food chain and aquifers in 16 Indian states.
bioaccumulate. Materials that by themselves
are not very harmful could be toxic if they 24. Answer: (a)
are inhaled in the form of nanoparticles. The
effects of inhaled nanoparticles in the body The word smog is derived from smoke and
may include lung inflammation and heart fog. This is the most common example of air
problems. pollution that occurs in many cities throughout
the world. There are two types of smog:
Statement 3 is correct: Most of these are due Classical smog which occurs in cool humid
to the high surface-to-volume ratio, which can climates and Photochemical smog occurs in
make the particles very reactive or catalytic warm, dry and sunny climates.
and make them trigger the production of free
radicals. For example, Iron oxide nanoparticles Option (a) is correct: The main components
act as a catalyst for the degradation of hydrogen of the photochemical smog resulting from the
peroxide into free radicals. action of sunlight on unsaturated hydrocarbons
and nitrogen oxides produced by automobiles
and factories. Photochemical smog has a
22. Answer: (d) high concentration of oxidising agents and
Option (d) is correct: The steel industry is, therefore, called as oxidising smog. The
releases large amounts of pollutants into the common components of photochemical smog
air during all its processes be it while handling are ozone (O3), nitric oxide (NO2), acrolein,
raw material, producing iron and steel, or formaldehyde and peroxyacetyl nitrate
disposing of solid waste. The main pollutants (PAN). When fossil fuels are burnt, a variety
are particulate matter, oxides of sulphur and of pollutants are emitted into the earth’s
nitrogen, carbon dioxide and carbon monoxide. troposphere. Two of the pollutants that are
emitted are hydrocarbons (unburned fuels) and
As per the TERI report in 2020, carbon dioxide
nitric oxide (NO). When these pollutants build
emissions from the steel industry are projected
up to sufficiently high levels, a chain reaction
to jump to 837 million tons over the next three
occurs from their interaction with sunlight in
decades from 242 million tons now as India’s
which NO is converted into nitrogen dioxide
demand for steel more than quadruples to
(NO2).
about 490 million tons.

25. Answer: (c)


23. Answer: (c)
Chlorofluorocarbons are non-toxic, non-
Central Ground Water Board (CGWB) regularly
flammable chemicals containing atoms of
monitors the ground water quality of shallow
carbon, chlorine and fluorine. They are used
aquifers on a regional scale, once every year.
in the manufacture of aerosol sprays, blowing
Option (c) is correct: Groundwater quality data agents for foams and packing materials,
generated during various scientific studies and solvents and as refrigerants.

Environmental Pollution-Explanation 261


unacademy.com | Download the Unacademy app
Give your feedback here: Link
Option (c) is correct: Chlorofluorocarbons, into water bodies, feeding more algal blooms.
known as ozone-depleting substances, are Statement 3 is correct: The process by which
used in the production of plastic foams, winds push surface water offshore and deep
in cleaning electronic components and as water moves towards the coast, bringing
pressurizing agents in aerosol cans. nutrients from the ocean floor to the surface is
Coastal upwelling. Climate change is expected
26. Answer: (b) to alter the timing and intensity of coastal
upwelling. Along the East and west coast of
Lead poisoning is one of the most common
India, excess nutrients delivered by upwelling
diseases of toxic environmental origin and
might lead to more algal blooms.
accounts for about 0.6% of the global burden
of disease. Lead is a naturally occurring
heavy metal found in the earth’s crust. It is 28. Answer: (d)
soft, malleable, and melts at a relatively low More than 70% of total energy consumed in India
temperature. It is easily moulded and can be is shared by thermal power producing energy
combined with other metals to form alloys. from coal, gas and diesel which constitute
Option (b) is correct: After the addition of lead 69.52% of the installed capacity of India. India’s
to petrol has been banned, but still lead is Total 80% of coal produced is consumed by
found in many sources including lead-based coal-based power plants only and despite that,
paints (paint on the walls of old houses and we need to continuously import coal.
toys), art supplies, contaminated dust, lead Option (d) is correct: The main emissions from
crystal glassware, ammunition, ceramic glazes, coal combustion at thermal power plants are
jeweller, and traditional medicines. Besides carbon dioxide (CO2), nitrogen oxides (NOx),
this, the most common route of lead exposure sulphur oxides (SOx), chlorofluorocarbons
is caused by lead smelting, which is through (CFCs), carbonaceous material, and air-borne
inhalation or ingestion of lead dust, particles, inorganic particles such as fly ash, also known
or exhaust from the burning process. Workers as suspended particulate matter (SPM) and
in the smelting factories are particularly at risk, other trace gas species. Carbon dioxide, nitrous
as they can be exposed to prolonged and direct oxide, and chlorofluorocarbons are greenhouse
inhalation of gaseous emissions and dust. gases.

27. Answer: (d) 29. Answer: (b)


Statement 1 is correct: Algal blooms are the Ozone layer depletion is the gradual thinning
result of an excess of nutrients, particularly of the earth’s ozone layer in the upper
some phosphates. Algal blooms are caused atmosphere caused due to the release of
by a major influx of nutrient-rich runoff into chemical compounds containing gaseous
a water body, programs to treat wastewater, bromine or chlorine from industries or other
reduce the overuse of fertilisers in agriculture human activities.
and reduce the bulk flow of runoff can be
effective for reducing severe algal blooms at y Ozone depletion occurs when ozone
river mouths, estuaries, and the ocean directly molecules in the upper atmosphere are
in front of the river’s mouth. destroyed by chemical reactions with
halogen gases, such as chlorine. To form
Statement 2 is correct: Monsoon and Climate chlorine the nitric acid in polar stratospheric
Change might affect rainfall patterns, leading clouds reacts with chlorofluorocarbon
to alternating periods of drought and intense carbon (CFCs), which catalyses the
storms. This can cause more nutrient runoff

262 Environmental Pollution-Explanation


unacademy.com | Download the Unacademy app
Give your feedback here: Link
photochemical destruction of ozone. In microbial strains that are capable of eating
the winter months, Cold conditions above up oil, they take in the pollutants and give
Antarctica encourage the formation of out carbon dioxide without producing any
extremely cold, high-altitude clouds. harmful residues.
[Option (b) is correct] y Oilzapper feeds on hydrocarbon
compounds present in crude oil and the
30. Answer: (a) hazardous hydrocarbon waste generated
by oil refineries, known as Oil Sludge and
Option (a) is correct: ‘Oilzapper’ is a bio- converts them into harmless CO2 and water.
remediation technique involving the use of ‘oil
zapping’ bacteria to clean up oil spills in the y Oilzapper is neatly packed into sterile
sea. It is considered as the most eco-friendly polythene bags and sealed aseptically for
technology for the remediation of oil sludge. safe transport. The shelf life of the product
is three months at ambient temperature.
y Oilzapper refers to a mixture of five

Environmental Pollution-Explanation 263


unacademy.com | Download the Unacademy app
Give your feedback here: Link
4 Biodiversity

1. Consider the following statements : (2023) (a) Only one


Statement-I: (b) Only two
Marsupials are not naturally found in India. (c) All three
Statement-II: (d) None
Marsupials can thrive only in montane
grasslands with no predators. 4. Which of the following organisms perform
Which one of the following is correct in waggle dance for others of their kin to
respect of the above statements? indicate the direction and the distance to a
(a) Both Statement-I and Statement-II are source of their food? (2023)
correct and Statement-II is the correct (a) Butterflies
explanation for Statement-I (b) Dragonflies
(b)
Both Statement-I and Statement-II (c) Honey Bees
are correct and Statement-II is not the
correct explanation for Statement-I (d) Wasps

(c) Statement-I is correct but Statement-II


is incorrect 5. Consider the following statements : (2023)
(d) Statement-I is incorrect but Statement- 1. Some mushrooms have medicinal
II is correct properties.
2. Some mushrooms have psychoactive
2. Invasive Species Specialist Group' (that properties.
develops Global Invasive Species Database) 3. Some mushrooms have insecticidal
belongs to which one of the following properties.
organizations? (2023) 4. Some mushrooms have bioluminescent
(a) The International Union for Conservation properties.
of Nature How many of the above statements are
(b)
The United Nations Environment correct?
Programme
(a) Only one
(c) The United Nations World Commission
(b) Only two
for Environment and Development
(c) Only three
(d) The World Wide Fund for Nature
(d) All four

3. Consider the following fauna: (2023)


6. Consider the following statements
1. Lion-tailed Macaque
regarding the Indian squirrels :(2023)
2. Malabar Civet
1. They build nests by making burrows in
3. Sambar Deer the ground.
How many of the above are generally 2. They store their food materials like nuts
nocturnal or most active after sunset?

264 Biodiversity
unacademy.com | Download the Unacademy app
Give your feedback here: Link
and seeds in the ground. 10. Consider the following statements:(2021)
3. They are omnivorous. 1. Moringa (drumstick trees) is a
How many of the above statements are leguminous evergreen trees.
correct? 2. The Tamarind tree is endemic to South
(a) Only one Asia.

(b) Only two 3. In India, most of the tamarind is


collected as minor forest produce.
(c) All three
4. India exports tamarind and seeds of
(d) None moringa.
5. Seeds of moringa and tamarind can be
7. Which one of the following makes a tool used in the production of biofuels.
with a stick to scrape insects from a hole Which of the statements given above are
in a tree or a log of wood?(2023) correct?
(a) Fishing cat (a) 1, 2, 4 and 5
(b) Orangutan (b) 3, 4 and 5
(c) Otter (c) 1, 3 and 4
(d) Sloth bear (d) 1, 2, 3 and 5

8. With reference to “Gucchi” sometimes 11. Which one of the following is a filter
mentioned in the news, consider the feeder? (2021)
following statements : (2022)
(a) Catfish
1. It is a fungus.
(b) Octopus
2. It grows in some Himalayan forest areas.
(c) Oyster
3. It is commercially cultivated in the
Himalayan foothills of north-eastern (d) Pelican
India.
Which of the statements given above is/are 12. Which one of the following is used in
correct? preparing a natural mosquito repellent?
(a) 1 only (2021)

(b) 3 only (a) Congress grass

(c) 1 and 2 (b) Elephant grass

(d) 2 and 3 (c) Lemon grass


(d) Nut grass

9. Which of the following is not a bird? (2022)


(a) Golden Mahseer 13.
With reference to India's biodiversity,
Ceylon frogmouth, Coppersmith barbet,
(b) Indian Nightjar Gray-chinned minivet and White-throated
(c) Spoonbill redstart are (2020)
(d) White Ibis (a) Birds
(b) Primates

Biodiversity 265
unacademy.com | Download the Unacademy app
Give your feedback here: Link
(c) Reptiles herbivores.
(d) Amphibians 4. Some species of snakes are viviparous.
Which of the statements given above is/are
14.
With reference to Indian elephants, correct?
consider the following statements: (2020) (a) 1 and 3 only
1. The leader of an elephant group is a (b) 2, 3, and 4 only
female. (c) 2 and 4 only
2. The maximum gestation period can be (d) 1, 2, 3 and 4
22 months.
3. An elephant can normally go on calving
till the age of 40 years only. 17. Consider the following statements: (2019)

4. Among the States in India, the highest 1. Asiatic lions are naturally found in India
elephant population is in Kerala. only.

Which of the statements given above is/are 2. Double-humped camels are naturally
correct? found in India only.

(a) 1 and 2 only 3. One-horned rhinoceros are naturally


found in India only.
(b) 2 and 4 only
Which of the statements given above is/are
(c) 3 only correct?
(d) 1, 3, and 4 only (a) 1 only
(b) 2 only
15. Consider the following pairs: (2019) (c) 1 and 3 only
Wildlife Naturally found (d) 1, 2 and 3
in
1. Blue-finned Cauvery River
18.
The term "sixth mass extinction/sixth
Mahseer
extinction" is often mentioned in the news
2. Irrawaddy Dolphin Chambal River in the context of the discussion of (2018)
3. Rusty Spotted Cat Eastern Ghats (a) Widespread monoculture practices in
Which of the pairs given above is/are agriculture and large-scale commercial
correctly matched? farming with indiscriminate use of
(a) 1 and 2 only chemicals in many parts of the world
that may result in the loss of good
(b) 2 and 3 only native ecosystems.
(c) 1 and 3 only (b)
Fears of a possible collision of a
(d) 1, 2 and 3 meteorite with the Earth in the near
future in the manner it happened 65
million years ago that caused the mass
16. Consider the following statements: (2019) extinction of many species including
1. Some species of turtles are herbivores. those of dinosaurs.
2. Some species of fish are herbivores. (c) Large-scale cultivation of genetically
3. Some species of marine mammals are modified crops in many parts of the
world and promoting their cultivation

266 Biodiversity
unacademy.com | Download the Unacademy app
Give your feedback here: Link
in other parts of the world which may
cause the disappearance of good 22. Which one of the following is the national
native crop plants and the loss of food aquatic animal of India? (2015)
biodiversity.
(a) Saltwater crocodile
(d) Mankind's overexploitation/misuse
of natural resources, fragmentation/ (b) Olive Ridley turtle
loss of natural habitats, destruction (c) Gangetic dolphin
of ecosystems, pollution, and global (d) Gharial
climate change.

23. With reference to 'dugong', a mammal found


19. If you want to see gharials in their natural in India, which of the following statements
habitat, which one of the following is the is/are correct? (2015)
best place to visit? (2017)
1. It is a herbivorous marine animal.
(a) Bhitarkanika Mangroves
2. It is found along the entire coast of
(b) Chambal River India.
(c) Pulicat Lake 3. It is given legal protection under
(d) Deepor Beel Schedule I of the Wildlife (Protection)
Act, 1972.

20.
In which of the following regions of Select the correct answer using the code
India are you most likely to come across given below:
the ‘Great Indian Hornbill’ in its natural (a) 1 and 2 only
habitat? (2016) (b) 2 only
(a) Sand deserts of northwest India (c) 1 and 3 only
(b) Higher Himalayas of Jammu and Kashmir (d) 3 only
(c) Salt marshes of western Gujarat
(d) Western Ghats 24. With reference to Neem tree, consider the
following statements: (2014)
21. What is/are unique about ‘Kharai camel’, a 1. Neem oil can be used as a pesticide
breed found in India? (2016) to control the proliferation of some
1. It is capable of swimming up to three species of insects and mites.
kilometers in seawater. 2. Neem seeds are used in the manufacture
2. It survives by grazing on mangroves. of biofuels and hospital detergents.

3. It lives in the wild and cannot be 3. Neem oil has applications in the
domesticated. pharmaceutical industry.

Select the correct answer using the code Which of the statements given above is/are
given below. correct?

(a) 1 and 2 only (a) 1 and 2 only

(b) 3 only (b) 3 only

(c) 1 and 3 only (c) 1 and 3 only

(d) 1, 2 and 3 (d) 1, 2 and 3

Biodiversity 267
unacademy.com | Download the Unacademy app
Give your feedback here: Link
2. Leatherback turtle
25. Other than poaching, what are the possible 3. Swamp deer
reasons for the decline in the population of Which of the above is/are endangered?
Ganga River Dolphins? (2014)
(a) 1 and 2 only
1. Construction of dams and barrages on
rivers. (b) 3 only

2. Increase in the population of crocodiles (c) 1, 2 and 3


in rivers. (d) None
3. Getting trapped in fishing nets
accidentally. 28. Consider the following: (2013)
4. Use of synthetic fertilisers and other 1. Star tortoise
agricultural chemicals in crop fields in
2. Monitor lizard
the vicinity of rivers.
3. Pygmy hog
Select the correct answer using the code
given below: 4. Spider monkey
(a) 1 and 2 only Which of the above are naturally found in
India?
(b) 2 and 3 only
(a) 1, 2 and 3 only
(c) 1, 3 and 4 only
(b) 2 and 3 only
(d) 1, 2, 3 and 4
(c) 1 and 4 only
(d) 1, 2, 3 and 4
26. If you walk through the countryside, you are
likely to see some birds stalking alongside
the cattle to seize the insects disturbed by 29. Consider the following animals: (2013)
their movement through grasses. Which
1. Sea cow
one of the following is/are such bird/birds?
(2014) 2. Sea horse
1. Painted Stork 3. Sea lion
2. Common Myna Which of the above is/are mammal/
mammals?
3. Black-necked Crane
(a) 1 only
Select the correct answer using the code
given below: (b) 1 and 3 only
(a) 1 and 2 only (c) 2 and 3 only
(b) 2 only (d) 1, 2 and 3
(c) 2 and 3 only
(d) 3 only 30. In which of the following States is lion-
tailed macaque found in its natural habitat?
(2013)
27. Consider the following fauna and India:
1. Tamil Nadu
(2013)
2. Kerala
1. Gharial
3. Karnataka

268 Biodiversity
unacademy.com | Download the Unacademy app
Give your feedback here: Link
4. Andhra Pradesh (a) Oryx is adapted to live in hot and arid
Select the correct answer using the code areas whereas Chiru is adapted to live
given below: in steppes and semi-desert areas of
cold high mountains.
(a) 1, 2 and 3 only
(b) Oryx is poached for its antlers whereas
(b) 2 only Chiru is poached for its musk.
(c) 1, 3 and 4 only (c) Oryx exists in western India only
(d) 1, 2, 3 and 4 whereas Chiru exists in north-cast India
only.

31. The Government of India encourages the (d) None of the statements (a), (b) and (c)
cultivation of ‘sea buckthorn ‘. What is the given above is correct.
importance of this plant? (2012)
1. It helps in controlling soil erosion and in 34. Which of the following can be threats to
preventing desertification. the biodiversity of a geographical area?
2. It is a rich source of biodiesel. (2012)

3. It has nutritional value and is well- 1. Global warming


adapted to live in cold areas of high 2. Fragmentation of habitat
altitudes. 3. Invasion of alien species
4. Its timber is of great commercial value. 4. Promotion of vegetarianism
Which of the statements given above is/are Choose the correct answer using the code
correct? given below:
(a) 1 only (a) 1, 2 and 3 only
(b) 2, 3 and 4 only (b) 2 and 3 only
(c) 1 and 3 only (c) 1 and 4 only
(d) 1, 2, 3 and 4 (d) 1, 2, 3 and 4

32.
Which one of the following groups of 35. Consider the following: (2012)
animals belongs to the category of
endangered species? (2012) 1. Black-necked crane

(a) Great Indian Bustard, Musk Deer, Red 2. Cheetah


Panda and Asiatic Wild Ass 3. Flying squirrel
(b) Kashmir Stag, Cheetal, Blue Bull and 4. Snow leopard
Great Indian Bustard Which of the above are naturally found in
(c) Snow Leopard, Swamp Deer, Rhesus India?
Monkey and Saras (Crane) (a) 1, 2 and 3 only
(d)
Lion-tailed Macaque, Blue Bull, (b) 1, 3 and 4 only
Hanuman Langur and Cheetal
(c) 2 and 4 only
(d) 1, 2, 3 and 4
33.
What is the difference between the
antelopes Oryx and Chiru? (2012)

Biodiversity 269
unacademy.com | Download the Unacademy app
Give your feedback here: Link
36. Vultures which used to be very common in 39. Biodiversity forms the basis for human
the Indian countryside some years ago are existence in the following ways: (2011)
rarely seen nowadays. This is attributed to: 1. Soil formation
(2012)
2. Prevention of soil erosion
(a) the destruction of their nesting sites by
new invasive species 3. Recycling of waste

(b) a drug used by cattle owners for treating 4. Pollination of crops


their diseased cattle Select the correct answer using the code
(c) scarcity of food available to them given below:

(d)
a widespread, persistent and fatal (a) 1, 2, and 3 only
disease among them (b) 2, 3, and 4 only
(c) 1 and 4 only
37. A sandy and saline area is the natural (d) 1, 2, 3 and 4 only
habitat of an Indian animal species. The
animal has no predators in that area, but
40. Which one of the following is not a site for
its existence is threatened due to the
in-situ method of conservation of flora?
destruction of its habitat. Which one of the
(2011)
following could be that animal? (2011)
(a) Biosphere Reserve
(a) Indian wild buffalo
(b) Botanical Garden
(b) Indian wild ass
(c) National Park
(c) Indian wild boar
(d) Wildlife Sanctuary
(d) Indian gazelle

41. The Himalayan Range is very rich in species


38. Consider the following statements: (2011)
diversity. Which one among the following
1. Biodiversity is normally greater in the is the most appropriate reason for this
lower latitudes as compared to the phenomenon? (2011)
higher latitudes.
(a) It has a high rainfall that supports
2.
Along the mountain gradients, luxuriant vegetative growth.
biodiversity is normally greater in the
(b) It is a confluence of different
lower altitudes as compared to the
biogeographical zones.
higher altitudes.
(c) Exotic and invasive species have not
Which of the statements given above is/are
been invasive species and have not
correct?
been introduced in this region.
(a) 1 only
(d) It has less human interference.
(b) 2 only
(c) Both 1 and 2
(d) Neither 1 nor 2

270 Biodiversity
unacademy.com | Download the Unacademy app
Give your feedback here: Link
4 Biodiversity-Explanation

1. Answer: (c) 3. Answer: (b)


Statement 1 is correct: Marsupials, a distinctive 1. Lion-tailed macaque (Macaca silenus),
group of mammals, are not naturally found a primate endemic to small and severely
in India. Evolutionarily, the distribution of fragmented rainforests of the Western Ghats
marsupials is primarily restricted to the in Karnataka, Kerala and Tamil Nadu, continues
continent of Australia, which has long been to be in the ‘endangered’ category in the IUCN
isolated from other land masses. Marsupials are Red List of Threatened Species. It is diurnal,
believed to have originated in South America, meaning it is active exclusively in daylight
and from there, they dispersed to Australia via hours.
Antarctica during a time when these continents 2. The Malabar large-spotted civet, also known
were connected in the supercontinent called as the Malabar civet, is a viverrid endemic
Gondwana. to the Western Ghats of India. It is listed as
Statement 2 is incorrect: While it is true that Critically Endangered on the IUCN Red List.
some marsupial species are found in grassland The Malabar civet is considered nocturnal and
habitats, such as kangaroos and wallabies in so elusive that little is known about its biology
Australia, it is incorrect to suggest that they can and ecology apart from habitat use.
only thrive in montane grasslands. Marsupials 3. The sambar is a large deer native to the Indian
exhibit diverse ecological preferences and can subcontinent, South China and Southeast Asia
be found in a wide range of ecosystems with that has been listed as a vulnerable species
varying levels of predation. on the IUCN Red List since 2008. Sambar are
nocturnal or crepuscular.
2. Answer: (a)
The Global Invasive Species Database (GISD) 4. Answer: (c)
is managed by the Invasive Species Specialist Astonishingly, honeybees possess one of the
Group (ISSG) of the IUCN Species Survival most complicated examples of nonhuman
Commission. It was developed between 1998 communication. They can tell each other
and 2000 as part of the global initiative on where to find resources such as food, water,
invasive species led by the erstwhile Global or nest sites with a physical “waggle dance.”
Invasive Species Programme (GISP). This dance conveys the direction, distance and
The Global Invasive Species Database is a quality of a resource to the bee’s nestmates.
free, online searchable source of information Essentially, the dancer points recruits in the
about alien and invasive species that correct direction and tells them how far to go
negatively impact biodiversity. The GISD aims by repeatedly circling around in a figure eight
to increase public awareness about invasive pattern centered around a waggle run, in which
species and to facilitate effective prevention the bee waggles its abdomen as it moves
and management activities by disseminating forward. Dancers are pursued by potential
specialist’s knowledge and experience to a recruits, bees that closely follow the dancer,
broad global audience. It focuses on invasive to learn where to go to find the communicated
alien species that threaten native biodiversity resource.
and natural areas and covers all taxonomic
groups from microorganisms to animals and
plants.

Biodiversity-Explanation 271
unacademy.com | Download the Unacademy app
Give your feedback here: Link
5. Answer: (d) 6. Answer: (c)
Statement 1 is correct: Mushrooms possess Statement 1 is correct: The Indian palm squirrel
medicinal properties and have been used for is a species of rodent in the family Sciuridae
their therapeutic benefits in various traditional found naturally in the south of the Vindhyas and
and modern systems of medicine. For example- in Sri Lanka. Indian Palm squirrels construct
reishi mushroom (Ganoderma lucidum). Reishi their nests in tree branches and hollow trunks.
mushrooms have a long history of use in Statements 2 and 3 are correct: Indian palm
traditional Chinese medicine, where they are squirrels are omnivores. They feed mainly on
believed to promote overall well-being and nuts and fruits but will also eat seeds, insects,
boost the immune system. Shiitake mushroom small mammals and reptiles, eggs, and even
(Lentinula edodes).It contains compounds like sometimes chicks of birds.
lentinan, which has been shown to enhance
immune function and exhibit potential
anticancer properties. 7. Answer: (b)
Statement 2 is correct: some mushrooms Orangutans, which are large arboreal great apes
possess psychoactive properties, meaning native to the rainforests of Southeast Asia, have
they contain compounds that can alter been observed using sticks as tools in various
consciousness, mood, perception, and ways. One of their notable tool-use behaviours
cognition when ingested. These mushrooms are involves employing sticks to scrape or probe
commonly referred to as “magic mushrooms” insect nests or crevices. They may modify the
or “psychedelic mushrooms.” The primary sticks by stripping off leaves or bark to create
psychoactive compound found in these a more effective tool for extracting insects.This
mushrooms is called psilocybin. behaviour demonstrates their problem-solving
Statement 3 is correct: Certain mushrooms abilities and adaptive intelligence.
possess insecticidal properties, meaning they
contain compounds that can repel, inhibit the 8. Answer: (c)
growth of, or directly kill insects. These natural
Statement 1 is correct: Guchhi mushroom is a
insecticidal properties of mushrooms have
species of fungus in the family Morchellaceae
been recognized and studied for their potential
of the Ascomycota. One of the most sought-
use in pest control and agriculture.
after edible mushrooms, guchhi is known for
Statement 4 is correct: some mushrooms its spongy, honeycombed head and savoury
exhibit bioluminescent properties, meaning flavour but all of that comes at a high price.
they have the ability to produce and emit light.
Statement 2 is correct and Statement 3 is not
These mushrooms are often referred to as
correct: Apart from its flavour, guchhi also owes
“glow-in-the-dark” mushrooms and are known
its high price to the challenges in cultivation. The
for their captivating and magical appearance
mushrooms cannot be cultivated commercially
in the dark.Bioluminescence in mushrooms
and grow in conifer forests across temperature
is a result of a chemical reaction that occurs
regions, and the foothills in Himachal Pradesh,
within their cells. The exact mechanisms and
Uttaranchal, and Jammu and Kashmir. And it
compounds responsible for the bioluminescent
takes months for villagers to collect enough of
properties vary among different species
these mushrooms, dry them and bring them to
of bioluminescent mushrooms. However,
the market.
a common feature is the presence of a
bioluminescent enzyme called luciferase and
a substrate known as luciferin.

272 Biodiversity-Explanation
unacademy.com | Download the Unacademy app
Give your feedback here: Link
9. Answer: (a) water through their bodies. Water is pumped
Option (a) is correct: Golden Mahseer is a fish, through the oyster’s gills. Plankton, algae, and
whose scientific name is Tor putitora. Mahseer other particles become trapped in the mucus
roughly translates as mahi – fish and sher – of the gills. From there, these particles are
tiger, and hence is also referred as tiger among transported to the oyster’s mouth.
fish. It is a large cyprinid and known to be the
toughest among freshwater sport fish. 12. Answer: (c)
Lemongrass is a herb with a lemony scent. The
10. Answer: (b) culinary herb is produced from the stalk of the
Statement 1 is not correct: Drumstick is one of lemongrass plant (Cymbopogon citratus). This
those rare plant species whose seeds, flowers, plant grows in many tropical climates, most
leaves, and stems are edible and extremely notably in Southeast Asia. It is a common
nutritious. Moringa, commonly referred to ingredient in Thai cooking and can also be
as drumstick, is treated as a ‘super plant’ found in dishes from Indonesia, Sri Lanka, and
for its unique yet powerful properties that India.
combat different ailments in our system. This Option (c) is correct: Lemongrass acts as a
nutrient-dense plant has its roots in Ayurveda natural mosquito repellent. It helps one to get
and was used as a medicine by our ancestors. rid of mosquitoes and keep them at bay. All you
Drumstick is an easily available vegetable need to do is crush the leaves of lemongrass
which is cultivated mostly in Asian and African and then apply the oily mixture to your skin.
regions. It is a staple vegetable in Southern
India. It can be evergreen and semi-deciduous.
13. Answer: (a)
Statement 2 is not correct: The tamarind tree
is endemic to Tropical Africa. Option (a) is correct: Ceylon frogmouth,
Coppersmith barbet, Grey-chinned minivet,
Statement 3 is correct: In India, most of the and White-throated redstart are birds.
tamarind is collected as minor forest produce.
Coppersmith Barbet is a short, stocky bird, with
Statement 4 is correct: India exports tamarind a leaf-green body that allows it to remain well-
and seeds of moringa. Moringa has been in camouflaged in the canopy. The Coppersmith
great demand in many countries. The major Barbet feeds mainly on the fruits of common
countries that import moringa leaves and fig trees, such as banyan, peepul, and cluster
seeds are the US, Germany, China, Canada, fig, as well as the occasional insect.
South Korea, and other European countries.
White-throated redstart is found in the Indian
Statement 5 is correct: Seeds of moringa and Subcontinent and Southeast Asia, as well
tamarind can be used in the production of as some adjoining areas. The species ranges
biofuels. across Afghanistan, Bhutan, Cambodia, India,
Laos, Myanmar, Nepal, Pakistan, Tajikistan,
11. Answer: (c) Thailand, Tibet, and Vietnam. Its natural habitat
is temperate forests.
Filter feeders are suspension-feeding animals
that feed by straining suspended matter and Grey-chinned minivet is a small minivet
food particles from water, typically by passing species distributed in the Indian subcontinent
the water over a specialized filtering structure. and southeast Asia. These minivet species are
common resident birds of the thorn jungles
Option (c) is correct: Oysters are filter feeders,
and shrubs in their range. These minivets are
meaning they eat by pumping large volumes of
polytypic species.

Biodiversity-Explanation 273
unacademy.com | Download the Unacademy app
Give your feedback here: Link
14. Answer: (a)
very specific. So, by eliminating statement
The Indian elephant (Elephas maximus indicus) 3 we are remaining with options A and B.
is a subspecies of the Asian elephant found on
the Asian mainland. Other subspecies of Asian
elephants are found on the islands of Sri Lanka 15. Answer: (c)
and Sumatra. Indian elephants are considered Pair 1 is correctly matched: Blue-finned
an endangered species. Mahseer belongs to the genus tor. It is a
Statement 1 is correct: Indian elephants are freshwater fish found in the Cauvery river
both diurnal and nocturnal. They are highly basin in Karnataka. Pollution, loss of habitat,
social animals, gathering into matriarchal units: destructive fishing, and sand extraction have
these are stable groups, which may consist of taken this fish to the brink of extinction.
more than 20 related females. The leader of Pair 2 is not correctly matched: The Irrawaddy
the group is the oldest female. dolphin is a species of river dolphin found in
Statement 2 is correct: The Indian elephants parts of South Asia. Chilika lake, in Odisha,
have a polygynous mating system. These is home to the only known population of
animals breed year-round. Before mating, Irrawaddy Dolphins in India.
males usually engage in aggressive fights, Pair 3 is correctly matched: The rusty-spotted
which occasionally result in serious injuries cat inhabits the forests of Andhra Pradesh and
or even death. The winner joins an all-female Eastern Ghats. It is a threatened species as
group, driving away other males. A breeding per IUCN, listed in Schedule I of the Wildlife
pair remains together for about 3 weeks. The Protection Act, 1972.
gestation period lasts for 22 months, yielding
a single baby. y Elimination Technique: Irrawaddy
Dolphin in pair 2 can be eliminated.
Statement 3 is not correct: Females between
As Irrawaddy Dolphins are naturally
14 - 45 years may give birth to calves
found in the Irrawaddy river and not in
approximately every four years with the mean
the Chambal River. So, we can get the
interbirth intervals increasing to five years
answer C.
by age 52 and six years by age 60. Hence, an
elephant can go on calving even after the age
of 40 years. Interbirth intervals of up to 13 years 16. Answer: (d)
may occur depending on habitat conditions
Statement 1 is correct: Whether or not a turtle
and population densities.
is a carnivore, a herbivore, or an omnivore
Statement 4 is not correct: The population of depends on the particular species. For instance,
India’s national heritage animal— the elephant— tortoises are almost always exclusively
has dipped in the country in the last five years herbivores. Their diets usually consist of plants.
(2012-2017). Among the states, the highest Most aquatic turtles are carnivores, it however
population was recorded in Karnataka (6,049), depends on the species of aquatic turtle. Semi-
followed by Assam (5,719) and Kerala (3,054). aquatic turtles are mostly omnivores and their
As far as regions are concerned, the highest nutritional requirements are usually best met
population was in the southern region (11,960) with a diet i.e. meat and vegetables.
followed by the northeast region (10,139), east-
Statement 2 is correct: Fish are of two types
central region (3,128), and northern region
mainly, there are carnivorous fish that eat
(2,085).
the meat of other animals, and there are
Elimination Technique: The ‘till the age herbivorous fish that eat herbs, seaweeds,
of 40 years only’, in statement 3 makes it and coral reefs, etc. Herbivore fish are usually

274 Biodiversity-Explanation
unacademy.com | Download the Unacademy app
Give your feedback here: Link
small in number as compared to carnivore fish. 18. Answer: (d)
Statement 3 is correct: The Earth’s vast Mass extinction refers to a substantial increase
network of oceans supports millions of marine in the degree of extinction or when the Earth
animals. Among them, some marine animals are loses more than three-quarters of its species
carnivores who eat the flesh of other animals, in a geologically short period of time. So far,
some are omnivores who eat a combination of during the entire history of the Earth, there have
flesh and plant matter and some are herbivores been five mass extinctions. The sixth, which
who are adapted to eat only plant matter. is ongoing, is referred to as the Anthropocene
Marine herbivores are found within four groups extinction.
of species in the animal kingdom which are: The ‘Sixth Extinction’ is presently in progress,
invertebrates, fish, reptiles, and mammals. different from the previous episodes. The
Statement 4 is correct: Snakes are reptiles. The difference is in the rates; the current species
species of snake can be classified as viviparous extinction rates are estimated to be 100 to
(no eggs) and ovoviviparous (egg retained inside 1,000 times faster than in pre-human times
the body). Most species of snakes will follow and our activities are responsible for the faster
the general rules of reptilian reproduction, the rates. Ecologists warn that if the present trends
mother will lay a clutch of eggs that will hatch continue, nearly half of all the species on earth
into baby snakes. Such snakes are referred might be wiped out within the next 100 years.
to as oviparous. A few exceptional species of Humanity needs a relatively stable climate,
snakes diverge from this general rule. They flows of fresh water, agricultural pest and
are referred to as viviparous because they give disease-vector control, and pollination for
birth to live young. crops, all services that will be impacted as the
sixth mass extinction accelerates.
17. Answer: (a) Option (d) is correct: The term “sixth mass
Statement 1 is correct: Asiatic lions are extinction/sixth extinction” is in the context of
naturally found in India only. In India, they overexploitation/misuse of natural resources,
are found in Gir National Park (Gujarat). It is fragmentation/loss of natural habitats,
part of the Schedule I species of the Wildlife destruction of ecosystems, pollution, and
Protection Act, 1972 that covers endangered global climate change.
species.
Statement 2 is not correct: In India double- 19. Answer: (b)
humped camels are found in Nubra Valley, Gharial prefers deep fast flowing rivers;
Ladakh. Besides India, they are found however, adult gharial has also been observed
throughout the highlands of central Asia from in still-water branches (jheel) of rivers
Turkistan to Mongolia and their total population and comparatively velocity-free aquatic
is around 20 million. environments of deep holes (kunds) at river
Statement 3 is not correct: One-horned bends and confluences.
rhinoceros are naturally found in Bhutan, Option (b) is correct: Historically, gharial was
Nepal, Indonesia, Malaysia, and India. These found in the river system of India, Pakistan,
countries signed a declaration- ‘The New Bangladesh, and the southern part of Bhutan
Delhi Declaration on Asian Rhinos 2019’ for the and Nepal. Today, they survive only in the
conservation and protection of the species. waters of India and Nepal. The surviving
population can be found within the tributaries
of the Ganges River system: Girwa (Uttar
Pradesh), Son (Madhya Pradesh), Ramganga

Biodiversity-Explanation 275
unacademy.com | Download the Unacademy app
Give your feedback here: Link
(Uttarakhand), Gandak (Bihar), Chambal (Uttar marine food, they sometimes swim for hours.
Pradesh, Madhya Pradesh, and Rajasthan) and Statement 3 is not correct: The number of
Mahanadi (Orissa). these camels, spread out across six coastal
It is listed in Schedule I of the Wildlife districts of Gujarat, has dwindled to 5,000.
(Protection) Act, 1972, and as Critically Kharai Camels are bred by two distinct
Endangered on the IUCN Red List. communities —the Fakirani Jats, who are
the handlers, and the Rabaris, who own the
animals. While the Rabaris are spread across
20. Answer: (d)
Bhachau and Mundra talukas, the Fakirani Jats
India is home to nine species of hornbills, of live in Bhachau, Mundra, Lakhpat, and Abdasa
which two are endemic. India is positioned talukas as well as in other coastal districts of
between two bio-geographic realms: the Gujarat such as Ahmedabad, Bharuch, Anand,
Afrotropical and Indomalayan. The north- and Bhavnagar. The nomadic communities
eastern region of India has the highest diversity move in search of mangroves for their camels
of Hornbill species (5), though the number to feed on and they can be domesticated.
of sympatric species is not as high as in the
Special Features of Kharai Camel:
Southeast Asian forests. The Great Hornbill
occurs in north, northeast, and south India, y Kharai Camel is an Eco-tonal breed. They
apart from Nepal, Bhutan, and Bangladesh. survive on dry land ecosystems as well as
coastal systems.
Option (d) is correct: Great Indian Hornbills are
found in Western Ghat. They are fondly referred y They are less affected by common skin
to as the ‘farmers of the forest’ because of the diseases (Khaji/Dermatitis).
great distances to which they help disperse y Excellent feeding and digesting of saline
seeds. trees and shrubs.
There are four species of hornbills in the y This breed can tolerate water with high TDS
Western Ghats. The prominent among them is (Total Dissolved Solid) up to 10000 ppm.
the Great Indian Hornbill and the others are y Smooth and long hair can be used for the
the Malabar Pied Hornbill, the Common Grey preparation of soft clothes/stoles.
Hornbill, and the Malabar Grey Hornbill.

22. Answer: (c)


21. Answer: (a)
Option (c) is correct: Ganges River Dolphin
Statement 1 is correct: The Kharai is a unique is the national aquatic animal of India. This
breed of camel found only in Kutch. They have mammal is also said to represent the purity of
the special ability to survive on both, dry land the holy Ganga as it can only survive in pure
and in the sea. They swim in seawater and feed and fresh water. This species inhabits parts of
on saline plants and mangroves. The Kharai the Ganges, Meghna, and Brahmaputra rivers in
was recognized as a separate breed in 2015 India, Nepal, Bhutan, and Bangladesh, and the
by the Indian Council of Agricultural Research Karnaphuli River in Bangladesh.
(ICAR). This breed can swim more than 3 km in
search of mangroves for grazing. y River dolphin is a critically endangered
species in India and therefore, has been
Statement 2 is correct: The Kharai breed has included in Schedule I for the Wildlife
the special ability to survive on both dry land (Protection) Act, 1972. The main reasons for
and in the sea, making it an ecotonal breed. the decline in the population of the species
Kharai camels are known to feed on mangroves are poaching and habitat degradation
on the island offshore. And to eat this salty due to declining flow, heavy siltation, and

276 Biodiversity-Explanation
unacademy.com | Download the Unacademy app
Give your feedback here: Link
construction of barrages causing physical y Neem oil is used in the pharmaceutical
barriers for this migratory species. industry because it contains various
compounds that have medicinal, and
cosmetic purposes.
23. Answer: (c)
Dugongs are mammals. They graze on seagrass,
especially young shoots and roots in shallow 25. Answer: (c)
coastal waters. They can consume up to 40 The Gangetic River dolphin is India’s national
kilograms of seagrass in a day. aquatic animal and is also known as ‘Susu’. The
y Dugong (Dugong), also called as ‘Sea Cow’ Ganga River Dolphin is found in Assam, Uttar
is one of the four surviving species in the Pradesh, Madhya Pradesh, Rajasthan, Bihar,
Order Sirenia and it is the only existing Jharkhand, and West Bengal. The upper Ganga
species of herbivorous mammal that lives River, Chambal River, Ghagra and Gandak
exclusively in the sea including in India. Rivers, Son and Kosi rivers, Brahmaputra and
(Statement 1 is correct) Kulsi River, a tributary of the Brahmaputra
River, form ideal habitats for the Ganges River
y Dugongs are protected in India and found
dolphin.
in the Gulf of Mannar, PalkBay, the Gulf of
Kutch, Andaman and the Nicobar Islands. Option (c) is correct: Once present in tens
They are not found along the entire coast of thousands of numbers, the Ganges River
of India. (Statement 2 is not correct) dolphin has dwindled abysmally to less than
2000 during the last century owing to direct
y They are protected in India under Schedule
killing, habitat fragmentation by dams and
I of the Wild (Life) Protection Act, 1972.
barrages, and indiscriminate fishing. Moreover,
(Statement 3 is correct)
the pollution entering their habitat from
y Human activities such as the destruction agricultural fields is another threat. It is for
and modification of habitat, pollution, these reasons that despite the high level of
rampant illegal fishing activities, vessel protection, its numbers continue to decline.
strikes, unsustainable hunting or poaching, Crocodiles are not a threat to dolphins.
and unplanned tourism are the main threats
The absence of a coordinated conservation
to dugongs. Moreover, the loss of seagrass
plan, lack of awareness, and continuing
beds due to ocean floor trawling was the
anthropogenic pressure, are posing incessant
most important factor behind dwindling
threats to the existing dolphin population.
dugong populations in many parts of the
world. Additional Information:
y The Ganga River Dolphin is classified under
Schedule 1, Wildlife (Protection) Act, 1972
24. Answer: (c)
providing absolute protection as offenses
Option (c) is correct: Coating urea with Neem under these are prescribed the highest
oil or Neem cake has been proved to be an penalties. Vikramshila Gangetic Dolphin
effective natural alternative to chemical Sanctuary (VGDS) in the Bhagalpur district
pesticides. is the only dedicated sanctuary for Gangetic
y Neem urea-coated agents contain an all- dolphins.
natural unaltered form of a nutrient, which
enables it to be an effective denitrifying
26. Answer: (b)
agent and as well as a natural soil
insecticide. Option (b) is correct: The Common Myna is
brown with a blackhead. It has a yellow bill,

Biodiversity-Explanation 277
unacademy.com | Download the Unacademy app
Give your feedback here: Link
legs, and bare eye skin. In flight, it shows Swamp deer is a highly endangered species
large white wing patches. The Common Myna protected under Schedule I of the Wildlife
is a member of the starling family and is also Protection Act of India 1972. It is also included
known as the Indian Myna or Indian Mynah. in the red data list of the rare animals of the
The Common Myna is closely associated with International Union for Conservation of Nature
human habitation. In the evening, large groups (IUCN).
of Common Mynas gather in communal roosts,
mainly in the non-breeding season, in roof 28. Answer: (a)
voids, bridges, and large trees, and numbers
can reach up to several thousand. Option 1 is correct: The Indian star tortoise is
one of the most trafficked tortoise species in
the world. Indian star tortoises are found in
27. Answer: (c) the forest areas closer to villages in Andhra
Option 1 is correct: Gharials (Gavialis Pradesh, Bihar, Delhi, Gujarat, Karnataka,
gangeticus) have been identified as the most Kerala, Madhya Pradesh, Maharashtra, Odisha,
Critically Endangered crocodilian species in the Tamil Nadu, Uttar Pradesh, and West Bengal.
world. Gharial prefers deep fast flowing rivers. The star tortoise is a Schedule IV species under
Historically, gharial was found in the river the Wildlife (Protection) Act, 1972 which means
system of India, Pakistan, Bangladesh and the it is illegal to keep them as pets or trade them
southern part of Bhutan and Nepal. Today they commercially. It is a CITES Appendix II species.
survive only in the waters of India and Nepal. Option 2 is correct: India is home to four
The surviving population can be found within species: Bengal Monitor Varanus bengalensis,
the tributaries of the Ganges River system: Common Water Monitor Varanus Salvator,
Girwa (Uttar Pradesh), Son (Madhya Pradesh), Yellow Monitor Varanus flavescens, and Desert
Ramganga (Uttarakhand), Gandak (Bihar), Monitor Varanus griseus. Monitor Lizards are
Chambal (Uttar Pradesh, Madhya Pradesh, and protected under India’s Wildlife (Protection)
Rajasthan) and Mahanadi (Orissa). Act, 1972 and listed under CITES. Monitor
Option 2 is correct: Leatherback turtles are Lizards helped the Maratha General Tanaji
named for their shell, which is leather-like rather Malusare scale the steep hilltop Sinhagad Fort
than hard, like other turtles. The leatherback in Pune in 1670.
(Dermochelys coriacea) is the largest turtle and Option 3 is correct: The pygmy hog is the
the largest living reptile in the world. Globally, smallest member of the pig family and is a
leatherback status according to IUCN is listed critically endangered species. Once found
as Vulnerable, but many subpopulations (such along a narrow strip of tall and wet grassland
as in the Pacific and Southwest Atlantic) are plains on the Himalayan foothills – from Uttar
Critically Endangered. Pradesh to Assam, through Nepal’s terai areas
Option 3 is correct: Swamp deer (Barasingha and Bengal’s duars – it was thought to have
or Rucervus duvaucelii) is the State Animal of become extinct in the 1960s. But in 1971 it was
Madhya Pradesh. There are three subspecies of “re-discovered” with a small population in the
swamp deer found in the Indian Subcontinent. Bornadi Wildlife Sanctuary. The only viable
The western swamp deer (Rucervus duvaucelii) population of the pygmy hog in the wild is in
is found in Nepal, the southern swamp deer the Manas Tiger Reserve in Assam.
(Rucervus duvaucelii branderi) is found in Option 4 is not correct: Spider monkeys are
central and north India, and the eastern swamp found in healthy tropical rainforests in Central
deer (Rucervus duvaucelii ranjitsinhi) found and South America. Black spider monkeys are
in the Kaziranga and Dudhwa National Parks. one of the largest primates in South America.

278 Biodiversity-Explanation
unacademy.com | Download the Unacademy app
Give your feedback here: Link
They exhibit anatomical and locomotory is a ‘staccato’ call emitted by the female which
adaptations, not unlike those of apes. coincides with a rather conspicuous swelling
of the skin near the females’ sexual organs at
the time of attaining sexual maturity. It is an
29. Answer: (b)
endangered species in the IUCN red data book.
Option 1 is correct: The sea cow also known
as Dugong (Dugong dugon), is one of the four
surviving species in the Order Sirenia and 31. Answer: (c)
it is the only existing species of herbivorous Statement 1 is correct: Sea buckthorn is a
mammal that lives exclusively in the sea hardy shrub with thorny branches, delicate
including in India. Dugongs are protected in and willowy leaves, and bright orange-yellow
India and occur in the Gulf of Mannar, PalkBay, berries. This shrub is also known as seaberry,
the Gulf of Kutch and the Andaman and Nicobar Siberian pineapple, sandthorn, and sallowthorn.
Islands. It restores degraded sites by preventing soil
Option 2 is not correct: Seahorses are erosion and fixes nitrogen with the help of soil
charismatic fish with a snout like a horse. They microbes.
are widespread in marine ecosystems across Statement 2 is not correct: It is found to have
the world. Most seahorses are marine except low viscosity in Seabuckthorn which is an
a few estuarine species. 46 recognised species important property of biodiesel. But this is not
of seahorses are found in the world’s oceans. only the criteria to extract biodiesel from it.
7 of these are found in India. All seahorses Statement 3 is correct: Sea buckthorn grows
give birth to young ones through the unique mostly wild throughout a few of India’s cold
breeding behaviour of male pregnancy. They and dry regions like Ladakh, Kumaon-Garhwal
are not mammals. in Uttaranchal, Lahaul-Spiti and Kinnaur in
Statement 3 is correct: Seals and sea lions are Himachal Pradesh, and the sacred forests of
marine mammals called ‘pinnipeds’ that differ Sikkim and Arunachal Pradesh. Seabuckthorn
in physical characteristics and adaptations. is a medicinal plant found in the Himalayan
Sea lions haul out in large colonies on rocks region also which is a cold region of India.
and sandy shores on the Islands. They move Statement 4 is not correct: Sea buckthorn is
into the water to feed and cool off as needed. a medium- sized, hardy, deciduous shrub (not
timber) that can grow between 2 and 6 m in
30. Answer: (a) height. It is found in the Northern Hemisphere
along riversides, in mountainous areas, and in
Option (a) is correct: Lion-tailed macaque
sandy and gravel ground at elevations of 3,300
(Macaca silenus), a primate endemic to small
to 4,500 m.
and severely fragmented rainforests of the
Western Ghats in Karnataka, Kerala, and Tamil
Nadu. This monkey is not only an indicator of 32. Answer: (a)
the health of rainforests, but its management Option (a) is correct: An endangered species
in the fragmented habitats also makes it an is a type of organism that is threatened by
umbrella species for the conservation of other extinction. Species become endangered for
wildlife species in the region. two main reasons: loss of habitat and loss of
The macaque is a typically arboreal species genetic variation. Great Indian Bustard, Musk
spending over 99 percent of its time in the Deer, Red Panda, and Asiatic Wild Ass belong
rainforest canopy. One of the most unusual to the category of endangered species.
features of the species’ reproductive behaviour y The Great Indian Bustard is the heaviest

Biodiversity-Explanation 279
unacademy.com | Download the Unacademy app
Give your feedback here: Link
flying bird and can weigh up to 15 kg sand dunes. Oryx live in desert conditions for
which grows up to one meter in height. In long periods and can survive without water.
July 2011 it was categorised as “critically Chiru is a Tibetan antelope that lives in cold
endangered” by the International Union for high mountains.
Conservation of Nature (IUCN).
y A musk deer species occurs in the Himalayas 34. Answer: (a)
of Nepal, Bhutan, India, Pakistan, and China.
It is listed as endangered on the IUCN Red In our biosphere immense diversity (or
List because of overexploitation. heterogeneity) exists not only at the species
level but at all levels of biological organisation
y The Red Panda is an endangered species of ranging from macromolecules within cells to
arboreal mammal found in the Himalayan biomes.
region and in south-western China. Its
range of countries includes India, Nepal, The biological wealth of our planet has been
Bhutan, Tibet, Myanmar, and China. declining rapidly which is mainly due to human
activities. The colonisation of tropical Pacific
y The IUCN Red List also moved the Islands by humans is said to have led to the
Indian wild ass from the ‘vulnerable’ to extinction of more than 2,000 species of native
‘endangered’ category, indicating the need birds. Presently, 12 percent of all bird species,
for heightened protection measures in 23 percent of all mammal species, 32 percent
August 2015. The Kiang or Tibetan Wild Ass of all amphibian species, and 31 percent of
is the largest of all African and Asiatic wild all gymnosperm species in the world face the
asses. threat of extinction.
Option (a) is correct: Global warming,
33. Answer: (a) fragmentation of habitat, invasion of alien
Oryx is a genus consisting of four large antelope species, and promotion of vegetarianism- all
species called oryxes. Their fur is pale with can be the threats to the biodiversity of a
contrasting dark markings on the face and geographical area.
on the legs, and their long horns are almost y Global Warming: Global warming is harmful
straight. The Arabian oryx was only saved from to the earth as it increases the water level
extinction through a captive breeding program in the sea but now it is also endangering the
and reintroduction to the wild. The scimitar biodiversity present on the planet. Studies
oryx, which is now listed as extinct in the wild, suggest that an increase in temperature
also relies on a captive breeding program for leads to the loss of biodiversity, mainly
its survival. The chiru is the only genus of large marine life as the elevated temperature
mammals endemic to the Tibetan Plateau. reduces the amount of dissolved oxygen
Despite its antelope-like appearance, the and hence leads to the unavailability of
chiru is not related to antelopes or to gazelles oxygen for marine animals.
but represents the last remnant of a group of y Fragmentation of habitat: Loss of
ruminants that split off from the sheep, goat, biodiversity stems largely from the habitat
and goat antelope lineages during the Neogene loss and fragmentation produced by human
Period. appropriation of land for development,
Option (a) is correct: Oryx live primarily in forestry and agriculture as natural capital
arid regions with low rainfall. Arabians live in is progressively converted to human-made
deserts, plains, savanna, and desert edges. capital.
Scimitars and East Africans live in similar y Invasion of alien species: Invasive alien
habitats, like steppe, desert, semi-desert, and species are known for endangering

280 Biodiversity-Explanation
unacademy.com | Download the Unacademy app
Give your feedback here: Link
biodiversity. Invasive species can take of the western Himalayas including the states
over that environment and create a new of Jammu and Kashmir, Himachal Pradesh,
functioning habitat. However, not all alien Uttarakhand and Sikkim, and Arunachal
species are harmful. Some non-native Pradesh in the eastern Himalayas.
species bring benefits to their new area of
habitation.
36. Answer: (b)
Option (b) is correct: In India Vultures are on
35. Answer: (b) the verge of extinction because a banned drug
Option (b) is correct: Black-necked crane, is still being used illegally to treat suffering
Flying squirrel, Snow leopard- they are naturally cattle. The endangered birds eat the remains of
found in India but Cheetah is mostly found in the drugged animals and suffer kidney failure
Africa but not in India, especially after 1952, and visceral gout, which is usually fatal. The
when it was declared extinct from India. drug was banned five years ago, but pharmacies
Black-necked crane: The black-necked crane continue to sell it under the counter, a study
is found at extremely high elevations, typically has revealed. More than 97 percent of vultures
3000-4900m, on the central and northern have disappeared from India’s skies in the past
Tibetan Plateau where it nests in high-altitude 15 years - the fastest decline ever recorded in
wetlands throughout Qinghai and the northern a bird population anywhere in the world.
Tibet Autonomous Region as well as in adjacent
areas of Xinjiang, Gansu, western Sichuan, and 37. Answer: (b)
the Ladakh region of India.
Option (b) is correct: The Indian wild ass
Cheetah: The Cheetah is the world’s fastest (Equus hemionus khur) also known as Ghor
land mammal. It is listed as vulnerable in IUCN Khar or Ghud Khur is found in the Little Rann
red-listed species. The cheetah was declared of Kutch and its surrounding areas in Gujarat.
extinct from India in 1952 and is considered Its preferred environment are Saline deserts
the only large mammal that has gone extinct (Rann), arid grasslands and shrublands. It is
since the country’s independence. The also found in Afghanistan, southern Pakistan,
cheetah lives in dry and shrub forests and and south-eastern Iran.
savannas in Africa with the largest populations
occurring in South-Western Angola, Botswana, y Indian wild buffalo: Terai region
Malawi, South-Western Mozambique, Namibia, y Indian wild boar: Tt can survive in different
Northern Mozambique, Northern South Africa, types of habitats: grasslands, taiga, and
Southern Zambia, and Zimbabwe. tropical rainforests, but they prefer life in
Flying squirrel: A species of rodent, the flying deciduous forests.
squirrel, is found in China, India, Indonesia, y Chinkara (Indian gazelle): Thar desert
Myanmar, Sri Lanka, Taiwan, Vietnam and
Thailand. In India, all flying squirrels are a rare
38. Answer: (c)
species and are placed under Schedule 1 of the
Indian Wildlife (Protection) Act, 1972. Statement 1 is correct: Latitudinal gradients
of biodiversity are biogeographic patterns
Snow leopards: Snow leopards are considered
that quantify the ways in which taxonomic,
medium-sized cats, standing about 24 inches
phylogenetic, functional, genetic, or phonetic
at the shoulder and weighing around 30-55kg.
biodiversity changes with the latitudinal
Snow leopards live in the mountainous regions
position on the surface of the earth. Species
of central and southern Asia. In India, their
richness, or biodiversity, increases from the
geographical range encompasses a large part

Biodiversity-Explanation 281
unacademy.com | Download the Unacademy app
Give your feedback here: Link
poles to the tropics. So, the latitude range activities such as excessive pesticide and land
with maximum species diversity is the equator use, would heavily impact human beings when
region which is at lower latitude. they die out.
Statement 2 is correct: A decrease in species
diversity takes place from lower to higher 40. Answer: (b)
altitudes on mountains. On Mountain Higher
seasonal variations and fall in temperature Option (b) is not correct: Botanical Garden
leads to a reduction in biodiversity with a 1000 is not a site for the ‘in-situ’ method of
m increase in altitude leading to a temperature conservation of flora. In-situ conservation
drop of about 6.5°C. is on-site conservation or the conservation
of genetic resources in natural populations
of plant or animal species, such as forest
39. Answer: (d) genetic resources in natural populations of
Statement 1 is correct: Biodiversity helps tree species.
in the maintenance and formation of soil
structure and the retention of moisture and
nutrient levels. Clearing of vegetation has
contributed to the leaching of nutrients,
salinisation of soils, lateralization of minerals
and accelerated erosion of topsoil, reducing the
land’s productivity. On the other hand, Trees
lower the water table and remove deposited
salt from the upper soil horizons.
Statement 2 is correct: The relationship
between erosion and biodiversity is reciprocal.
Soil organisms can both reduce soil loss,
by improving porosity, and increase it, by
diminishing soil stability as a result of their
mixing activities.
41. Answer: (b)
Statement 3 is correct: Deforestation due to
The Himalayas have three biogeographical
raw materials destroys the natural habitat and
zones which have different biodiversity as:
causes global warming. Recycling reduces the
Northwest Himalayas, West Himalayas, Central
need for raw materials and hence preserves
Himalayas, and East Himalayas, which together
the rainforests.
constitute about 6.4% of the country’s area.
Statement 4 is correct: Insects are responsible It is a confluence of different biogeographical
for the pollination of 3/4th of all the plants zones due to which the Himalayan region is
cultivated for human consumption. Bees, very rich in species diversity.
which are severely endangered by human

282 Biodiversity-Explanation
unacademy.com | Download the Unacademy app
Give your feedback here: Link
5 Global Conservation Efforts

1. Consider the following statements: (2023) 3. It is a legally binding international


1. In India, the Biodiversity Management declaration.
Committees are key to the realisation of 4. It is endorsed by governments, big
the objectives of the Nagoya Protocol. companies and indigenous communities.
2.
The Biodiversity Management 5. India was one of the signatories at its
Committees have important functions in inception.
determining access and benefit sharing, Select the correct answer using the code
including the power to levy collection given below.
fees on the access of biological
resources within its jurisdiction. (a) 1, 2 and 4

Which of the statements given above is/are (b) 1, 3 and 5


correct? (c) 3 and 4
(a) 1 only (d) 2 and 5
(b) 2 only
(c) Both 1 and 2 4. 'R2 Code of Practices’ constitutes a tool
(d) Neither 1 nor 2 available for promoting the adoption of
(2021)
(a) environmentally responsible practices
2. The ‘Common Carbon Metric’, supported by in electronics recycling industry
UNEP, has been developed for (2021)
(b) ecological management of ‘Wetlands
(a)
assessing the carbon footprint of of International Importance’ under the
building operations around the world Ramsar Convention
(b) enabling commercial farming entities (c) sustainable practices in the cultivation
around the world to enter carbon of agricultural crops in degraded lands
emission trading
(d) ‘Environmental Impact Assessment’ in
(c) enabling governments to assess the the exploitation of natural resources
overall carbon footprint caused by their
countries
(d) assessing the overall carbon footprint 5. Consider the following statements: (2021)
caused by the use of fossil fuels by the Statement 1: The United Nations Capital
world in a unit time Development Fund (UNCDF) and the Arbor
Day Foundation have recently recognized
Hyderabad as 2020 Tree City of the World.
3. With reference to the 'New York Declaration
on Forests’, which of the following Statement 2: Hyderabad was selected for
statements are correct? (2021) the recognition for a year following its
commitment to grow and maintain the
1. It was first endorsed at the United urban forests.
Nations Climate Summit in 2014.
Which one of the following is correct in
2. It endorses a global timeline to end the respect of the above statements?
loss of forests.

Global Conservation Efforts 283


unacademy.com | Download the Unacademy app
Give your feedback here: Link
(a) Both Statement 1 and Statement 2 are Janeiro
correct and Statement 2 is the correct (c)
The United Nations Framework
explanation for Statement 1 Convention on Climate change 2015,
(b) Both Statement 1 and Statement 2 Paris
are correct but Statement 2 is not the (d) The World Sustainable Development
correct explanation for Statement 1 Summit 2016, New Delhi
(c) Statement 1 is correct but Statement 2
is not correct
8. "Momentum for Change: Climate Neutral
(d) Statement 1 is not correct but Statement Now" is an initiative launched by (2018)
2 is correct
(a) The Intergovernmental Panel on Climate
Change
6. Consider the following statements: (2019) (b) The UNEP Secretariat
1. Under Ramsar Convention, it is (c) The UNFCCC Secretariat
mandatory on the part of the Government
of India to protect and conserve all the (d) The World Meteorological Organisation
wetlands in the territory of India.
2. The Wetlands (Conservation and 9. With reference to the 'Global Alliance for
Management) Rules, 2010 were framed Climate-Smart Agriculture (GACSA)', which
by the Government of India based on of the following statements is/are correct?
the recommendation of the Ramsar (2018)
Convention. 1. GACSA is an outcome of the Climate
3. The Wetlands (Conservation and Summit held in Paris in 2015.
Management) Rules, 2010 also 2. Membership of GACSA does not create
encompass the drainage area or any binding obligations.
catchment regions of the wetlands as
3. India was instrumental in the creation
determined by the authority.
of GACSA.
Which of the statements given above is/are
Select the correct answer using the code
correct?
given below:
(a) 1 and 2 only
(a) 1 and 3 only
(b) 2 and 3 only
(b) 2 only
(c) 3 only
(c) 2 and 3 only
(d) 1, 2 and 3
(d) 1, 2 and 3

7. The Partnership for Action on Green


10. Consider the following statements: (2017)
Economy (PAGE), a UN mechanism to assist
countries transition towards greener and 1. Climate and Clean Air Coalition (CCAC) to
more inclusive economies, emerged at Reduce Short Lived Climate Pollutants
(2018) is a unique initiative of G2O group of
countries.
(a)
The Earth Summit on Sustainable
Development 2002, Johannesburg 2. The CCAC focuses on methane, black
carbon, and hydrofluorocarbons.
(b)
The United Nations Conference on
Sustainable Development 2012, Rio de Which of the statements given above is/are

284 Global Conservation Efforts


unacademy.com | Download the Unacademy app
Give your feedback here: Link
correct? Select the correct answer using the code
(a) 1 only given below.

(b) 2 only (a) 1 and 3 only

(c) Both 1 and 2 (b) 2 only

(d) Neither 1 nor 2 (c) 2 and 3 only


(d) 1, 2 and 3

11. Consider the following statements in


respect of Trade Related Analysis of Fauna 13. With reference to an initiative called ‘The
and Flora in Commerce (TRAFFIC): (2017) Economics of Ecosystems and Biodiversity
1. TRAFFIC is a bureau under the United (TEEB)’, which of the following statements
Nations Environment Programme is/are correct? (2016)
(UNEP). 1. It is an initiative hosted by UNEP, IMF
2. The mission of TRAFFIC is to ensure and World Economic Forum.
that trade in wild plants and animals 2. It is a global initiative that focuses on
is not a threat to the conservation of drawing attention to the economic
nature. benefits of biodiversity.
Which of the above statements is/are 3. It presents an approach that can
correct? help decision makers recognize,
(a) 1 only demonstrate, and capture the value of
ecosystems and biodiversity.
(b) 2 only
Select the correct answer using the code
(c) Both 1 and 2 given below.
(d) Neither 1 nor 2 (a) 1 and 2 only
(b) 3 only
12. The FAO accords the status of the Globally (c) 2 and 3 only
Important Agricultural Heritage System
(GIAHS) to traditional agricultural systems. (d) 1, 2 and 3
What is the overall goal of this initiative?
(2016) 14. Which of the following statements is/are
1. To provide modern technology, training correct? (2016)
in modern farming methods and Proper design and effective implementation
financial support to local communities of UNREDD+ Programme can significantly
of identified GIAHS so as to greatly contribute to
enhance their agricultural productivity.
1. Protection of biodiversity
2. To identify and safeguard eco-friendly
traditional farm practices and their 2. Resilience of forest ecosystems
associated landscapes, agricultural 3. Poverty reduction
biodiversity, and knowledge systems of Select the correct answer using the code
the local communities. given below.
3. To provide Geographical Indication (a) 1 and 2 only
status to all the varieties of agricultural
(b) 3 only
produce in such identifies GIAHS.

Global Conservation Efforts 285


unacademy.com | Download the Unacademy app
Give your feedback here: Link
(c) 2 and 3 only (d) 1, 2 and 3
(d) 1, 2 and 3
17. Which one of the following is associated
15. What is/are the importance/importances of with the issue of control and phasing out
the ‘United Nations Convention to Combat of the use of ozone-depleting substances?
Desertification’? (2016) (2015)

1. It aims to promote effective action (a) Bretton Woods Conference


through innovative national programmes (b) Montreal Protocol
and supportive international (c) Kyoto Protocol
partnerships.
(d) Nagoya Protocol
2. It has a special/particular focus on
South Asia and North Africa regions, and
its Secretariat facilitates the allocation 18. With reference to the International Union
of major portions of financial resources for Conservation of Nature and Natural
to these regions. Resource (IUCN) and the Convention of
3. It is committed to a bottom-up approach, International Trade in Endangered Species
encouraging the participation of local of Wild Fauna and Flora (CITES), which of
people in combating desertification. the following statements is/are correct?
(2015)
Select the correct answer using the code
given below. 1. IUCN is an organ of the United Nations
and CITES is an international agreement
(a) 1 only between governments.
(b) 2 and 3 only 2. IUCN runs thousands of field projects
(c) 1 and 3 only around the world to better manage
(d) 1, 2 and 3 natural environments.
3. CITES is legally binding on the States
that have joined it, but this Conventional
16. With reference to an organisation known
does not take the place of national laws.
as 'BirdLife International', which of the
following statements is/are correct? Select the correct answer using the code
(2015) given below:

1. It is a Global Partnership of Conservation (a) 1 only


Organisations. (b) 2 and 3 only
2. The concept of 'biodiversity hotspots' (c) 1 and 3 only
originated from this organisation. (d) 1, 2 and 3
3. It identifies the sites known/referred
to as 'Important Bird and Biodiversity
Areas'. 19. If a wetland of international importance is
brought under the 'Montreux Record', what
Select the correct answer using the code does it imply? (2014)
given below:
(a) Changes in ecological character have
(a) 1 only occurred, are occurring or are likely
(b) 2 and 3 only to occur in the wetland as a result of
(c) 1 and 3 only human interference.

286 Global Conservation Efforts


unacademy.com | Download the Unacademy app
Give your feedback here: Link
(b) The country in which the wetland is (c) Both 1 and 2
located should enact a law to prohibit (d) Neither 1 nor 2
any human activity within five kilometres
from the edge of the wetland.
(c) The survival of the wetland depends on 22.
Consider the following statements
the cultural practices and traditions of regarding 'Earth Hour' (2014)
certain communities living in its vicinity 1. It is an initiative of UNEP and UNESCO.
and therefore the cultural diversity 2. It is a movement in which the
therein should not be destroyed. participants switch off the lights for one
(d) It is given the status of 'World Heritage hour on a certain day every year.
Site'. 3. It is a movement to raise awareness
about climate change and the need to
20.
With reference to 'Global Environment save the planet.
Facility', which of the following statements Which of the statements given above is/are
is/are correct? (2014) correct?
(a) It serves as a financial mechanism for (a) 1 and 3 only
'Convention on Biological Diversity' and (b) 2 only
'United Nations Framework Convention
(c) 2 and 3 only
on Climate Change'.
(d) 1, 2 and 3
(b) It undertakes scientific research on
environmental issues at global level.
(c) It is an agency under OECD to facilitate 23.
Consider the following international
the transfer of technology and funds to agreements: (2014)
underdeveloped countries with specific 1. The International Treaty on Plant Genetic
aims to protect their environment. Resources for Food and Agriculture.
(d) Both (a) and (b) 2. The United Nations Convention to
Combat Desertification.
21.
With reference to a conservation 3. The World Heritage Convention.
organisation called 'Wetlands International', Which of the above has/have a bearing on
which of the following statements is/are biodiversity?
correct? (2014)
(a) 1 and 2 only
1. It is an intergovernmental organisation
(b) 3 only
formed by the countries which are
signatories to the Ramsar Convention. (c) 1 and 3 only
2. It works at the field level to develop (d) 1, 2 and 3
and mobilise knowledge and use the
practical experience to advocate for
24. The “Red Data Books’’ published by the
better policies.
International Union for Conservation of
Select the correct answer using the code Nature and Natural Resources (IUCN)
given below: contain lists of (2011)
(a) 1 only 1. Endemic plant and animal species
(b) 2 only present in the biodiversity hotspots.

Global Conservation Efforts 287


unacademy.com | Download the Unacademy app
Give your feedback here: Link
2. Threatened plant and animal species. as hotspots of biodiversity: (2011)
3. Protected sites for conservation of 1. Species richness
nature and natural resources in various 2. Vegetation density
countries.
3. Endemism
Select the correct answer using the code
given below: 4. Ethno-botanical importance

(a) 1 and 3 only 5. Threat perception

(b) 2 only 6. Adaptation of flora and fauna to warm


and humid conditions
(c) 2 and 3 only
Which three of the above are correct criteria
(d) 3 only in this context?
(a) 1, 2 and 6 only
25.
Three of the following criteria have (b) 2, 4 and 6 only
contributed to the recognition of western
Ghats, Sri Lanka and Indo-Burma regions (c) 1, 3 and 5 only
(d) 3, 4 and 6 only

288 Global Conservation Efforts


unacademy.com | Download the Unacademy app
Give your feedback here: Link
Global Conservation Efforts-
5 Explanation
1. Answer: (c) GHG emissions related to the operation of
Statement 1 is correct: In India, the different types of buildings in different climate
Biodiversity Management Committees (BMCs) zones.
play a crucial role in realising the objectives The CCM is intended to create a uniform
of the Nagoya Protocol. The Nagoya Protocol system for defining the climate impact of
on Access to Genetic Resources and the Fair buildings through a consistent protocol,
and Equitable Sharing of Benefits Arising from which can, in turn, help develop international
their Utilization is an international agreement baselines for use by architects, designers and
under the Convention on Biological Diversity the construction industry.
(CBD). It aims to promote the conservation and
sustainable use of biodiversity and ensure fair
3. Answer: (a)
and equitable sharing of benefits arising from
the utilization of genetic resources. Statements 1 and 2 are correct: The New York
Declaration on Forests (NYDF) was adopted in
Statement 2 is correct: The Biodiversity
2014 as a political declaration calling for the
Management Committees have important
end of natural forest loss and the restoration
functions related to access and benefit
of 350 million hectares of degraded landscapes
sharing. They have the power to determine
and forestlands by 2030. It was launched at
access to biological resources within their
the UN Secretary-General’s Climate Summit in
jurisdiction and regulate the utilization of those
September 2014.
resources. They are responsible for ensuring
that the benefits derived from the utilization Statement 3 is not correct: It is not a legally
of biodiversity resources are shared fairly binding international declaration but entirely
and equitably among various stakeholders, voluntary.
including local communities and indigenous Statement 4 is correct: It was endorsed
people. The BMCs also have the authority to by nearly 200 governments, multinational
levy collection fees on the access of biological companies, Indigenous Peoples, and civil
resources to generate revenue that can be used society organizations.
for biodiversity conservation and sustainable
Statement 5 is not correct: India is not a
development activities.
signatory to the treaty.

2. Answer: (a)
4. Answer: (a)
The Common Carbon Metric is supported by
Option (a) is correct: Responsible Recycling
the United Nations Environment Program.
(R2) is a standard specifically created for the
The purpose of a Common Carbon Metric for
electronics recycling industry by Sustainable
buildings is to give the sector a way to measure,
Electronics Recycling International (SERI).
report, and verify reductions consistently and
comparably. This sector consumes 40% of the SERI is the housing body and ANSI-accredited
world’s energy and releases 1/3rd of global Standards Development Organization for the
greenhouse gas (GHG). R2 Standard: Responsible Recycling Practices
for Use in Accredited Certifications Programs.
Option (a) is correct: The Common Carbon
Metric is used to measure, report, and verify An updated version of the R2 Code of Practices

Global Conservation Efforts-Explanation 289


unacademy.com | Download the Unacademy app
Give your feedback here: Link
(COP 2.1) was released on July 22, 2021, to clarify on the part of the Government of India to
areas of the COP. The changes were based on protect and conserve all the wetlands in the
questions that arose from the Certification territory of India.
Bodies as they started to implement the COP The Ramsar Convention is not a regulatory regime
for R2v3 certification programs. and has no punitive sanctions for violations
of or defaulting upon treaty commitments –
5. Answer: (d) nevertheless, its terms do constitute a solemn
treaty and are binding in international law in
Option (d) is correct: Hyderabad has become
that sense. The whole edifice is based upon an
the only city in India to be recognised as a ‘Tree
expectation of common and equitably shared
City of the World’ by the Arbor Day Foundation
transparent accountability.
and the Food and Agriculture Organization
(FAO) of the United Nations. Hyderabad is Statement 2 is not correct: India is a signatory
placed alongside 119 other cities from 63 to the Ramsar Convention which slots irrigated
countries. agricultural lands, and canals under wetlands.
But the Wetlands Conservation Rules, 2010,
The ‘Tree City of the World’ programme
does not have river canals, paddy fields, and
provides direction, assistance, and worldwide
coastal wetlands in their ambit. The Ministry
recognition for communities’ dedication to
of Environment, Forest and Climate Change
their urban forest and provides a framework
prepared the Wetlands (Conservation and
for a healthy, sustainable urban forestry
Management) Rules, 2010. Thus, it is not
programme.
based on the recommendation of the Ramsar
To be eligible as a ‘Tree City’, cities need to Convention.
conform to the following five standards:
Statement 3 is correct: Under the Wetlands
y Standard 1: Establish Responsibility (Conservation and Management) Rules,
y Standard 2: Set the Rules 2010, “wetland” means an area of marsh,
fen, peatland, or water; natural or artificial,
y Standard 3: Know What You Have
permanent or temporary, with water that
y Standard 4: Allocate the Resources is static or flowing, fresh, brackish or salt,
y Standard 5: Celebrate Achievements including areas of marine water, the depth of
which at low tide does not exceed six meters
and includes all inland waters such as lakes,
6. Answer: (c) reservoir, tanks, backwaters, lagoon, creeks,
The Ramsar Convention is an international treaty estuaries, and man-made wetland and the
for the conservation and sustainable utilisation zone of direct influence on wetlands that is to
of wetlands, recognizing the fundamental say the drainage area or catchment region of
ecological functions of wetlands and their the wetlands as determined by the authority
economic, cultural, scientific, and recreational but does not include main river channels,
value. The Ramsar Convention came into force paddy fields, and the coastal wetland.
in 1975. India became a contracting party to
the Ramsar Convention in 1981. Currently, India
7. Answer: (b)
has 42 Ramsar Sites.
Option (b) is correct: The Partnership for Action
Statement 1 is not correct: Ramsar Convention
on Green Economy (PAGE), a United Nation
does not enforce the parties, but rather guides
mechanism to assist countries’ transition
them to protect and ensure that there is a
towards greener and more inclusive economies,
proper policy in place for the conservation of
emerged at The United Nations Conference on
the wetlands in the country. It is not mandatory

290 Global Conservation Efforts-Explanation


unacademy.com | Download the Unacademy app
Give your feedback here: Link
Sustainable Development 2012, Rio de Janeiro. 9. Answer: (b)
PAGE seeks to put sustainability at the heart Statement 1 is not correct: The Global Alliance
of economic policies and practices to advance for Climate-Smart Agriculture (GACSA) is
the 2030 Agenda for Sustainable Development an inclusive, voluntary, and action-oriented
and supports nations and regions in reframing multi-stakeholder platform on Climate-Smart
economic policies and practices around Agriculture (CSA). GACSA was launched in
sustainability to foster economic growth, September 2014 on the margins of the UN
create income and jobs, reduce poverty and Climate Summit. It was launched at COP 21 in
inequality, and strengthen the ecological Paris.
foundations of their economies. GACSA claims that it aspires to improve
PAGE brings together five UN agencies which farmers’ agricultural productivity and incomes
are the UN Environment, International Labour in a sustainable way. It also hopes to build
Organisation, UN Development Programme, UN farmers’ resilience to extreme weather and
Industrial Development Organisation, and UN changing climate and reduce greenhouse gas
Institute for Training and Research. emissions associated with agriculture.
Statement 2 is correct: Membership in GACSA
8. Answer: (c) does not create any binding obligations.
Members determine their particular voluntary
Option (c) is correct: “Momentum for Change: actions according to their needs and priorities.
Climate Neutral Now” is an initiative launched Individuals cannot be members, but they can
by the UNFCCC Secretariat. join GACSA’s Action Groups.
The UNFCCC secretariat launched its Climate Statement 3 is not correct. India was not
Neutral Now initiative in 2015. The following instrumental in the creation of GACSA. Though,
year, the secretariat launched a new pillar few Indian NGOs are its members.
under its Momentum for Change initiative
focused on Climate Neutral Now, as part of Climate-Smart Agriculture (CSA) is an
larger efforts to showcase successful climate approach to developing the technical,
action around the world. policy and investment conditions to achieve
sustainable agricultural development for food
Climate neutrality is a three-step process, security under climate change. The magnitude,
which requires individuals, companies, immediacy, and broad scope of the effects
and governments to measure their climate of climate change on agricultural systems
footprint, reduce their emissions as much as create a compelling need to ensure the
possible, and offset what they cannot reduce comprehensive integration of these effects into
with UN-certified emission reductions. national agricultural planning, investments,
Elimination Technique: The UNFCCC and programs.
secretariat (UN Climate Change) is The concept of Climate-Smart Agriculture
the United Nations entity tasked with (CSA) was originally developed by FAO.
supporting the global response to the Climate-smart agriculture includes proven
threat of climate change. UNFCCC stands practical techniques by increasing the organic
for United Nations Framework Convention content of the soil through conservation tillage,
on Climate Change. Thus, Option (c) is its water-holding capacity increases, making
correct. yields more resilient and reducing erosion,
by promoting soil carbon capture also helps
mitigate climate change. Another example
is integrated soil fertility management which

Global Conservation Efforts-Explanation 291


unacademy.com | Download the Unacademy app
Give your feedback here: Link
can lower fertiliser costs, increase soil carbon (TRAFFIC) is a leading non-governmental
and improve yields. Climate-smart agriculture organisation working globally on trade in wild
gives attention to landscape approaches, animals and plants in the context of both
for example, integrated planning of land, biodiversity conservation and sustainable
agriculture, forests, fisheries and water to development. TRAFFIC was established in 1976.
ensure synergies are captured. TRAFFIC is a joint conservation programme of
the World Wide Fund for Nature (WWF) and the
Elimination Technique: India is against International Union for Conservation of Nature
expanding the scope for mitigation of (IUCN). It is not a bureau under the United
climate change to agriculture, thereby Nations Environment Programme (UNEP).
compromising the very foundation of food
security in the world. So, statement 3 can Statement 2 is correct: The mission of TRAFFIC
be eliminated. Thus, Option (b) is correct. is to ensure that trade in wild plants and
animals is not a threat to the conservation of
nature. TRAFFIC also works in close cooperation
10. Answer: (b) with the Secretariat of the Convention on
Statement 1 is not correct: The Climate and International Trade in Endangered Species of
Clean Air Coalition (CCAC) is a voluntary Wild Fauna and Flora (CITES).
partnership of governments, intergovernmental
organisations, businesses, scientific 12. Answer: (b)
institutions, and civil society organisations
committed to protecting the climate and Globally Important Agricultural Heritage
improving air quality through actions to reduce System (GIAHS) is a programme of the UN Food
short-lived climate pollutants. The Coalition’s and Agricultural Organisation (FAO), founded
activities are financed through a multi- as a UN partnership initiative for sustainable
donor trust fund administered through UN development in 2002.
Environment. It is not an initiative of the G2O Statement 1 is not correct: GIAHS aims to
group of countries. identify, support and safeguard globally
Statement 2 is correct: The CCAC focuses on important agricultural heritage systems and
methane, black carbon, and hydrofluorocarbons. their livelihoods, agricultural and associated
At the same time, partners recognize that biodiversity, knowledge systems, cultures and
action on short-lived climate pollutants must landscapes around the world. GIAHS sites are
complement and supplement, not replace, not living museums, but places where people
global action to reduce carbon dioxide, in practice ‘dynamic conservation’. They retain
particular efforts under the UNFCCC. the best of the past to build a sustainable
future. GIAHS focus on traditional methods
Additional Information: (not modern methods).
The short-lived climate pollutants such as Statement 2 is correct: Capacity building
black carbon, methane, tropospheric ozone, of local farming communities and local and
and hydrofluorocarbons are the most important national institutions to conserve and manage
contributors to the artificial global greenhouse GIAHS, generate income and add economic
effect after carbon dioxide, responsible for up value to goods and services of such systems
to 45% of current global warming. in a sustainable fashion is an important
goal of GIAHS. It will identify and safeguard
11. Answer: (b) eco-friendly traditional farm practices and
their associated landscapes, agricultural
Statement 1 is not correct: Trade Related
biodiversity and knowledge systems of the
Analysis of Fauna and Flora in Commerce

292 Global Conservation Efforts-Explanation


unacademy.com | Download the Unacademy app
Give your feedback here: Link
local communities. system, Saffron is also a cash crop. Saffron
Statement 3 is not correct. Geographical is part of the cultural heritage of the Region,
Indication status is not associated with GIAHS. associated with the famous Kashmiri cuisine,
its medicinal values and the rich cultural
GIAHS focuses on biodiversity at a genetic
heritage of Kashmir.
level, looking at the conservation and use of
indigenous varieties and species. They also Elimination Technique: They provide
focus on biodiversity at a landscape level, the Geographical Indication status to all the
mosaic of different land uses which produce varieties of agricultural produce in such
ecosystem services such as clean water and identifies GIAHS. Geographical Indication
food security. status is not associated with GIAHS and
Additional Information: also statement 3 is too extreme, so it can
be eliminated. Thus, Option (b) is correct.
In Asia and the Pacific region, 40 agricultural
heritage systems were designated so far in 8
countries and one territory: one in Bangladesh, 13. Answer: (c)
15 in China, 2 in India, 3 in Iran, 11 in Japan, The Economics of Ecosystems and Biodiversity
one in the Philippines and 5 in the Republic of (TEEB) is a global initiative focused on “making
Korea, one in Sri Lanka and one in the Territory nature’s values visible”.
of Jammu and Kashmir.
Statement 1 is not correct: The initiative was
Koraput Traditional Agriculture (Odisha): launched at the G8 meeting of environment
Traditional systems in the Koraput Region ministers in Potsdam, Germany, in 2007. The
are strongly linked to the local traditional major international initiative was funded by
communities. From their knowledge and the European Commission, Germany, the
practices, high biodiversity has been United Kingdom, Norway, the Netherlands and
conserved through in-situ conservation Sweden, and managed by the United Nations
preserving endemic species. Strongly linked Environment Programme as part of its Green
as a cultural trait, sacred grove is an effective Economy Initiative (GEI).
method of preserving plant genetic resources.
It is a biological heritage as well as a social Statement 2 is correct: It seeks to draw
mechanism by which a forest patch is protected attention to the global economic benefits of
with religious significance. biodiversity, to highlight the growing costs of
biodiversity loss and ecosystem degradation,
Kuttanad Below Sea Level Farming System and to draw together expertise from the fields
(Kerala): It is unique, as it is the only system of science, economics and policy to enable
in India that favours rice cultivation below practical actions moving forward.
sea level in the land created by draining delta
swamps in brackish waters. The Kuttanad Statement 3 is correct: Its principal objective
system is a complex mosaic of fragmented is to mainstream the values of biodiversity
agricultural landscapes divided into three and ecosystem services into decision-making
structures: wetlands used for paddy activities at all levels. It aims to achieve this goal by
and fish catching, garden lands used for following a structured approach to valuation
coconut, tubers and food crops plantation and that helps decision-makers recognize the wide
water areas used for inland fishing and shells. range of benefits provided by ecosystems
and biodiversity, demonstrates their values
Saffron Heritage of Kashmir: Saffron is in economic terms and, where appropriate,
mentioned in the 5th century B.C. in Kashmiri captures those values in decision-making.
records and is still part of the agricultural
economy. Integrated with an annual crop Additional Information:

Global Conservation Efforts-Explanation 293


unacademy.com | Download the Unacademy app
Give your feedback here: Link
The Economics of Ecosystems and Biodiversity resources, thus contributing to the global fight
TEEB-India Initiative (TII): against climate change.
y The Government has launched The Statement 1 is correct: Countries that
Economics of Ecosystems and Biodiversity reduce emissions and undertake sustainable
TEEB-India Initiative (TII) to highlight the management of forests will be entitled to
economic consequences of the loss of receive funds and resources as incentives.
biological diversity and the associated REDD+ approach incorporates important
decline in ecosystem services. benefits of livelihood improvement, biodiversity
y The Initiative focussed on three ecosystems, conservation and food security services.
namely forests, inland wetlands, and Statements 2 and 3 are correct: The objective
coastal and marine ecosystems. TII has of the REDD+ program is not only to reduce
been implemented under the Indo-German deforestation and forest degradation but also
Biodiversity Programme. to improve livelihoods, reduce poverty and
consequently, to mitigate climate change.

14. Answer: (d) However, it was agreed in the 2007 Bali Action
Plan that the program should go beyond
REDD (Reducing Emissions from Deforestation
mitigation. It should also deliver environmental
and Forest Degradation) is the global endeavour
and social co-benefits that would contribute
to create an incentive for developing countries
to poverty reduction.
to protect, better manage and save their forest

Elimination Technique: All the options are correct as it is obvious that deforestation would
result in these benefits. Thus, Option (d) is correct.

294 Global Conservation Efforts-Explanation


unacademy.com | Download the Unacademy app
Give your feedback here: Link
15. Answer: (c) parties work together to improve the living
Established in 1994, the United Nations conditions for people in drylands, to maintain
Convention to Combat Desertification (UNCCD) and restore land and soil productivity, and to
is the sole legally binding international mitigate the effects of drought. The UNCCD
agreement linking environment and is particularly committed to a bottom-up
development to sustainable land management. approach, encouraging the participation of
The Convention addresses specifically the arid, local people in combating desertification and
semi-arid and dry sub-humid areas, known land degradation.
as the drylands, where some of the most Elimination Technique: The Secretariat
vulnerable ecosystems and peoples can be plays an instrumental role in addressing
found. the causes and devastating impact of
Statement 1 is correct: The UNCCD plays a land degradation and drought. It does not
key role in global efforts to eradicate poverty, focus on particular regions and allocates
achieve sustainable development and reach the financial resources to regions. Statement
Millennium Development Goals, in particular 2 can be eliminated. Statement 3 is clearly
with regard to the eradication of extreme correct. Thus, Option (c) is correct.
poverty. It aims to promote effective action
through innovative national programmes and
supportive international partnerships. 16. Answer: (c)

The objective of this Convention is to combat Statement 1 is correct: BirdLife International


desertification and mitigate the effects of is a global partnership of conservation
drought in countries experiencing serious organisations (NGOs) that strives to conserve
drought and/or desertification, particularly in birds, their habitats and global biodiversity,
Africa, through effective action at all levels, working with people towards sustainability in
supported by international cooperation and the use of natural resources.
partnership arrangements, in the framework y Conservation International was a pioneer
of an integrated approach which is consistent in defining and promoting the concept of
with Agenda 21, with a view to contributing to biodiversity hotspots. In 1989, just one year
the achievement of sustainable development after scientist Norman Myers wrote the
in affected areas. The long-term integrated paper that introduced the hotspots concept,
strategies will improve productivity, Conservation International adopted the
rehabilitation, and conservation, particularly at idea of protecting these incredible places
the community level. as the guiding principle of our investments.
Statement 2 is not correct: The UNCCD (Statement 2 is not correct)
secretariat facilitates cooperation between y Since the late 1970s, the BirdLife Partnership
developed and developing countries (and has worked collectively to identify,
does not have any special focus on South document and protect the places of the
Asia and North Africa Region), particularly greatest significance for the conservation
around knowledge and technology transfer for of the world’s birds called Important Bird
sustainable land management. The key function and Biodiversity Areas (IBAs). (Statement 3
of the secretariat is to service the Conference is correct)
of Parties (COP) and its subsidiary bodies in a
Elimination Technique: The concept of
manner that enables well-founded decision-
‘biodiversity hotspots’ originated from
making and successful action in advancing the
Conservation International. Statement 2
implementation of the UNCCD.
can be eliminated. BirdLife International
Statement 3 is correct: The Convention’s 197

Global Conservation Efforts-Explanation 295


unacademy.com | Download the Unacademy app
Give your feedback here: Link
Protocol commits industrialised countries
is a global partnership of conservation to stabilise greenhouse gas emissions
organisations (NGOs) so, Statement 1 is based on the principles of the Convention.
correct. Thus, Option (c) is correct.
y The Nagoya Protocol on Access to Genetic
Resources and the Fair and Equitable
17. Answer: (b) Sharing of Benefits Arising from their
Utilisation (ABS) to the Convention on
Option (b) is correct: Recognising the
Biological Diversity is a supplementary
deleterious effects of ozone depletion, an
agreement to the Convention on Biological
international treaty, known as the Montreal
Diversity.
Protocol, was signed in Montreal (Canada) in
1987 (effective in 1989) to control the emission
of ozone-depleting substances. 18. Answer: (b)
y Ozone depletion refers to the phenomenon Statement 1 is not correct: IUCN was founded
of reductions in the amount of ozone in in October 1948 as the International Union for
the stratosphere. The problem of ozone the Protection of Nature (or IUPN) following
depletion is caused by high levels of an international conference in Fontainebleau,
chlorine and bromine compounds in the France. The organisation changed its name to
stratosphere. the International Union for Conservation of
y The origins of these compounds are Nature and Natural Resources in 1956 with the
chlorofluorocarbons (CFC), used as acronym IUCN (or UICN) with its headquarters
cooling substances in air conditioners and in Gland, Switzerland. It is not a UN organ.
refrigerators, or as aerosol propellants, and Statement 2 is correct: IUCN supports scientific
bromofluorocarbons (halons), used in fire research, manages field projects globally
extinguishers. and brings governments, non-government
y As a result of the depletion of the ozone organisations, United Nations agencies,
layer, more ultraviolet (UV) radiation comes companies and local communities together
to Earth and causes damage to living to develop and implement policy. IUCN runs
organisms. UV radiation seems responsible thousands of field projects around the world
for skin cancer in humans; it also lowers to better manage natural environments.
the production of phytoplankton and thus CITES (the Convention on International
affects other aquatic organisms. It can also Trade in Endangered Species of Wild Fauna
influence the growth of terrestrial plants. and Flora) is an international agreement
A reduction of approximately 5 percent in between governments. It aims to ensure
the ozone layer was detected from 1979 to that international trade in specimens of wild
1990. animals and plants does not threaten the
Additional Information: survival of the species.
y The Bretton Woods Conference held in 1944
set up the International Monetary Fund (IMF) Statement 3 is correct: It became the only
and the World Bank and re-established a treaty to ensure that international trade in
system of fixed exchange rates. plants and animals does not threaten their
y The Kyoto Protocol was adopted in Kyoto, survival in the wild. CITES is legally binding
Japan, on 11 December 1997. Due to a on the Parties – in other words, they have to
complex ratification process, it entered implement the Convention – it does not take
into force on 16 February 2005. The Kyoto the place of national laws. Rather it provides
a framework to be respected by each Party,

296 Global Conservation Efforts-Explanation


unacademy.com | Download the Unacademy app
Give your feedback here: Link
which has to adopt its own domestic legislation y Montreux Record sites in India: Keoladeo
to ensure that CITES is implemented at the National Park and Loktak Lake.
national level.

Elimination Technique: IUCN is not an 20. Answer: (a)


organ of the United Nations. IUCN has Option (a) is correct: The Global Environment
observer and consultative status at Facility (GEF) was established on the eve of
the United Nations. Statement 1 can be the 1992 Rio Earth Summit. It is a financial
eliminated. Thus, Option (b) is not correct. mechanism that provides grants to developing
countries for projects that benefit the global
19. Answer: (a) environment.

The Montreux Record was established by the Since then, the GEF has provided more
recommendation of the Conference of the than $21.1 billion in grants and mobilised an
Contracting Parties (1990). During the fourth additional $114 billion in co-financing for more
meeting held at Montreux, Switzerland in 1990, than 5,000 projects in 170 countries. It serves
it was decided that mere listing of wetlands as a financial mechanism for ‘Convention
as Ramsar sites was not enough. The purpose on Biological Diversity’ and ‘United Nations
would be fulfilled if the conservation activities Framework Convention on Climate Change’.
could be prioritised where ecological damage Through its Small Grants Programme, the GEF
has begun or is likely to occur. has provided support to more than 25,000
civil society and community initiatives in 133
Option (a) is correct: The Montreux Record countries.
is a register of wetland sites on the List of
Wetlands of International Importance where
changes in ecological character have occurred, 21. Answer: (b)
are occurring, or are likely to occur as a result Statement 1 is not correct: Wetlands
of technological developments, pollution or International is the only global not-for-
other human interference. It is maintained as profit organisation (not an intergovernmental
part of the Ramsar List. organisation) dedicated to the conservation
Additional Information: and restoration of wetlands. It is concerned
y The Convention on Wetlands is an about the loss and deterioration of wetlands
intergovernmental treaty that provides the such as lakes, marshes, and rivers. Wetlands
framework for the conservation and wise International is dedicated to maintaining and
use of wetlands and their resources. The restoring wetlands for their environmental
Convention was adopted in the Iranian city values as well as for the services they provide
of Ramsar in 1971 and came into force in to people.
1975. Statement 2 is correct: Wetland International-a
y There are 171 contracting parties to the network organisation working locally and
Ramsar Convention. globally. The organisation has a network
of offices around the world; these often-
y India signed the Ramsar Convention on 1 independent entities share the same Strategy.
February 1982. They work at the field level to develop and
y There are 42 Ramsar sites in India. Chilika mobilise knowledge and use this practical
Lake (Odisha) and Keoladeo National Park experience to advocate for better policies.
(Rajasthan) were recognized as the first
Ramsar Sites of India. Elimination Technique: Statement
1 is not correct as there is no such

Global Conservation Efforts-Explanation 297


unacademy.com | Download the Unacademy app
Give your feedback here: Link
y Establishing a global system to provide
intergovernmental organisation formed by farmers, plant breeders and scientists with
the countries which are signatories to the access to plant genetic materials.
Ramsar Convention. Statement 2 is clearly
correct. Option (b) is correct. y Ensuring that recipients share the benefits
they derive from the use of these genetic
materials with the countries where they
22. Answer: (c) have originated.
Statement 1 is not correct: Earth Hour, Option 2 is correct: The United Nations
organised by Worldwide Fund for Nature Convention to Combat Desertification (UNCCD)
(WWF), is a global grassroots movement uniting was established in 1994. It is the sole legally
people to take action on environmental issues binding international agreement linking
and protect the planet. environment and development to sustainable
Statement 2 is correct: Engaging a massive land management. The Convention addresses
mainstream community, Earth Hour was specifically the arid, semi-arid and dry sub-
famously started as a light-out event in Sydney, humid areas, known as the drylands, where
Australia in 2007. Since then, it has grown to some of the most vulnerable ecosystems
engage millions of supporters in more than 185 and peoples can be found. The new UNCCD
countries and territories, inspiring individuals 2018-2030 Strategic Framework is the most
and organisations worldwide to take action for comprehensive global commitment to achieve
the environment and driving major legislative Land Degradation Neutrality (LDN) in order to:
changes by harnessing the power of the crowd. y restore the productivity of vast expanses of
Statement 3 is correct: As the movement degraded land,
grows, the one-hour lights-out event continues y improve the livelihoods of more than 1.3
to be the symbol of a broader commitment billion people,
toward nature and our planet. y reduce the impacts of drought on vulnerable
Elimination Technique: Earth Hour is an populations, and
initiative of the Worldwide Fund for Nature y maintain biodiversity.
(WWF) to increase awareness about Option 3 is correct: The World Heritage
our Earth and its importance through Convention is concerned about the Protection
a one-hour lights-out event every year. of World Cultural and Natural Heritage is an
Statement 1 is not correct and Statement international agreement. It was adopted by the
2 is correct. Thus, Option (c) is correct. General Conference of UNESCO in 1972.
y The Convention is governed by the World
23. Answer: (d) Heritage Committee.
Option 1 is correct: The International Treaty y IUCN is the Advisory Body on natural
on Plant Genetic Resources for Food and heritage. It monitors listed sites and
Agriculture was adopted by the Thirty-First evaluates sites nominated to the World
Session of the Conference of the Food and Heritage List.
Agriculture Organisation of the United Nations y The World Heritage Convention ensures
on 3 November 2001. global commitment for the conservation of
The Treaty aims at: biodiversity-rich places, while respecting
y Recognizing the enormous contribution of their cultural and local connections.
farmers to the diversity of crops that feed
the world.

298 Global Conservation Efforts-Explanation


unacademy.com | Download the Unacademy app
Give your feedback here: Link
24. Answer: (b) International (CI) adopted Myers’ hotspots as
Option (b) is correct: “Red Books”, a public its institutional blueprint and the organisation
document which is created for recording made the decision to undertake a reassessment
endangered threatened and rare species of of the hotspots concept.
plants, animals, fungi as well as some local India is known for its rich biodiversity and has
subspecies that are present in a particular 23.39% of its geographical area covered by
region is considered as a Red Data Book. forests and trees.
y It gives complete information about Biodiversity hotspot in India:
research, and studies and, also for y Himalaya: Includes the entire Indian
monitoring of the programs on rare and Himalayan region (and that falling in
endangered species and their habitats. It Pakistan, Tibet, Nepal, Bhutan, China and
is mainly created to identify and protect Myanmar).
those species which are on the verge of
extinction. y Indo-Burma: Includes entire North-eastern
India, except Assam and Andaman group of
Elimination Technique: Red list does not Islands (and Myanmar, Thailand, Vietnam,
contain the list of protected sites for Laos, Cambodia and southern China).
the conservation of nature and natural y Sundalands: Includes Nicobar group of
resources in various countries. Statement Islands (and Indonesia, Malaysia, Singapore,
3 is eliminated, Thus, Option (b) is correct. Brunei, Philippines).
y Western Ghats and Sri Lanka: Includes the
25. Answer: (c) entire Western Ghats (and Sri Lanka).
Option (c) is correct: The term “Biodiversity Elimination Technique: Adaptation of flora
hotspots” Coined by Norman Myers can be and fauna to warm and humid conditions
defined as the regions which are known for is not criteria for biodiversity hotspots. So,
their high species richness, threat perception It can be eliminated. Thus, Option (c) is
and endemism. In 1989 and 1996, Conservation correct.

Global Conservation Efforts-Explanation 299


unacademy.com | Download the Unacademy app
Give your feedback here: Link
6 National Conservation Efforts

1. Consider the following statements : (2023) animal becoming a danger to human


Once the Central Government notifies an life is sufficient ground for its capture
area as a 'Community Reserve' or killing.

1. the Chief Wildlife Warden of the State Which of the statements given above is/are
becomes the governing authority of correct?
such forest (a) 1 and 2
2. Hunting is not allowed in such area (b) 2 only
3. People of such area are allowed to (c) 1 and 3
collect non-timber forest produce (d) 3 only
4. People of such area are allowed
traditional agricultural practices
4. If a particular plant species is placed under
How many of the above statements are Schedule VI of The Wildlife Protection Act,
correct? 1972, what is the implication? (2020)
(a) Only one (a) A licence is required to cultivate that
(b) Only two plant.
(c) Only three (b) Such a plant cannot be cultivated under
(d) All four any circumstances.
(c) It is a Genetically Modified crop plant.

2. Which one of the following has been (d) Such a plant is invasive and harmful to
constituted under the Environment the ecosystem.
(Protection) Act, 1986? (2022)
(a) Central Water Commission 5. Consider the following statements: (2020)
(b) Central Ground Water Board 1. 36% of India’s districts are classified
(c) Central Ground Water Authority as “overexploited” or “critical” by the
Central Ground Water Authority (CGWA).
(d) National Water Development Agency
2. CGWA was formed under the
Environment (Protection) Act.
3. With reference to Indian laws about 3. India has the largest area under
wildlife protection, consider the following groundwater irrigation in the world.
statements : (2022)
Which of the statements given above is/are
1. Wild animals are the sole property of correct?
the government.
(a) 1 only
2. When a wild animal is declared
protected, such animal is entitled for (b) 2 and 3 only
equal protection whether it is found in (c) 2 only
protected areas or outside. (d) 1 and 3 only
3. Apprehension of a protected wild

300 National Conservation Efforts


unacademy.com | Download the Unacademy app
Give your feedback here: Link
6. As per the Solid Waste Management Rules, of Forest Rights) Act, 2006, bamboo is a
2016 in India, which one of the following minor forest produce.
statements is correct? (2019) 3. The Scheduled Tribes and Other
(a) Waste generators have to segregate Traditional Forest Dwellers (Recognition
waste into five categories. of Forests Rights) Act, 2006 allows
(b) The Rules are applicable to notified ownership of minor forest produce to
urban local bodies, notified towns and forest dwellers.
all industrial townships only. Which of the statements given above is/are
(c) The Rules provide for exact and elaborate correct?
criteria for the identification of sites for (a) 1 and 2 only
landfills and waste processing facilities. (b) 2 and 3 only
(d) It is mandatory on the part of the waste (c) 3 only
generator that the waste generated in
one district cannot be moved to another (d) 1, 2 and 3
district.
9. Consider the following states: (2019)
7. Consider the following statements: (2019) 1. Chhattisgarh
The Environment Protection Act, 1986 2. Madhya Pradesh
empowers the Government of India to 3. Maharashtra
1. state the requirement of public 4. Odisha
participation in the process of
With reference to the States mentioned
environmental protection, and the
above, in terms of percentage of forest
procedure and manner in which it is
cover to the total area of the State, which
sought
one of the following is the correct ascending
2. lay down the standards for emission or order?
discharge of environmental pollutants
(a) 2-3-1-4
from various sources
(b) 2-3-4-1
Which of the statements given above is/are
correct? (c) 3-2-4-1
(a) 1 only (d) 3-2-1-4
(b) 2 only
(c) Both 1 and 2 10. Consider the following statements: (2019)
(d) Neither 1 nor 2 1. As per law, the Compensatory
Afforestation Fund Management and
Planning Authority exists at both
8. Consider the following statements: (2019) National and State levels.
1. As per recent amendment to the Indian 2. People’s participation is mandatory
Forest Act, 1927, forest dwellers have in the compensatory afforestation
the right to fell the bamboos grown on programmes carried out under the
forest areas. Compensatory Afforestation Fund Act,
2. As per the Scheduled Tribes and Other 2016.
Traditional Forest Dwellers (Recognition Which of the statements given above is/are

National Conservation Efforts 301


unacademy.com | Download the Unacademy app
Give your feedback here: Link
correct? Rights Act, 2006.
(a) 1 only 2. For the first time in India, Baigas have
(b) 2 only been given Habitat Rights.

(c) Both 1 and 2 3. Union Ministry of Environment, Forest


and Climate Change officially decides
(d) Neither 1 nor 2
and declares Habitat Rights for Primitive
and Vulnerable Tribal Groups in any part
11. In India, ‘extended producer responsibility’ of India.
was introduced as an important feature in Which of the statements given above is/are
which of the following? (2019) correct?
(a) The Bio -medical Waste (Management (a) 1 and 2 only
and Handling) Rules, 1998
(b) 2 and 3 only
(b) The Recycled Plastic (Manufacturing
(c) 3 only
and Usage) Rules, 1999
(d) 1, 2 and 3
(c)
The e -Waste (Management and
Handling) Rules, 2011
(d)
The Food Safety and Standard 14. The term ‘M-STrIPES’ is sometimes seen in
Regulations, 2011 the news in the context of (2017)
(a) Captive breeding of Wild Fauna

12. How is the National Green Tribunal (NGT) (b) Maintenance of Tiger Reserves
different from the Central Pollution Control (c) Indigenous Satellite Navigation System
Board (CPCB)? (2018) (d) Security of National Highways
1. The NGT has been established by an Act
whereas the CPCB has been created by
an executive order of the Government. 15. In India, if a species of tortoise is declared
protected under Schedule I of the Wildlife
2. The NGT provides environmental justice (Protection) Act, 1972, what does it imply?
and helps reduce the burden of litigation (2017)
in the higher courts whereas the CPCB
promotes cleanliness of streams and (a) It enjoys the same level of protection as
wells, and aims to improve the quality the tiger.
of air in the country. (b) It no longer exists in the wild, a few
Which of the statements given above is/are individuals are under captive protection;
correct? and now it is impossible to prevent its
extinction.
(a) 1 only
(c) It is endemic to a particular region of
(b) 2 only India.
(c) Both 1 and 2 (d) Both (b) and (c) stated above are
(d) Neither 1 nor 2 correct in this context.

13. Consider the following statements: (2018) 16.


According to the Wildlife (Protection)
1. The definition of "Critical Wildlife Act, 1972, which of the following animals
Habitat" is incorporated in the Forest cannot be hunted by any person except
under some provisions provided by law?

302 National Conservation Efforts


unacademy.com | Download the Unacademy app
Give your feedback here: Link
(2017) (Registration and Protection) Act, 1999
1. Gharial (c) Environment (Protection) Act, 1986
2. Indian wild ass (d) Wildlife (Protection) Act, 1972
3. Wild buffalo
Select the correct answer using the code 20. With reference to Bombay Natural History
given below: Society (BNHS), consider the following
(a) 1 only statements: (2014)

(b) 2 and 3 only 1. It is an autonomous organisation under


the Ministry of Environment and Forests.
(c) 1 and 3 only
2. It strives to conserve nature through
(d) 1, 2 and 3 action-based research, education, and
public awareness.
17. With reference to ‘Red Sanders’, sometimes 3. It organises and conducts nature trails
seen in the news, consider the following and camps for the general public.
statements: (2016) Which of the statements given above is/are
1. It is a tree species found in a part of correct?
South India. (a) 1 and 3 only
2. It is one of the most important trees (b) 2 only
in the tropical rainforest areas of South
India. (c) 2 and 3 only

Which of the statements given above is/are (d) 1, 2 and 3


correct?
(a) 1 only 21.
The most important strategy for the
(b) 2 only conservation of biodiversity together with
traditional human life is the establishment
(c) Both 1 and 2 of (2014)
(d) Neither 1 nor 2 (a) Biosphere Reserves
(b) Botanical gardens
18. ‘Gadgil Committee Report’ and (c) National Parks
‘Kasturirangan Committee Report’,
sometimes seen in the news, are related to (d) Wildlife Sanctuaries
(2016)
(a) Constitutional reforms 22. Consider the following statements: (2014)
(b) Ganga Action Plan 1. Animal Welfare Board of India was
(c) Linking of rivers established under the Environment
(Protection) Act, 1986.
(d) Protection of Western Ghats
2. National Tiger Conservation Authority is
a Statutory Body.
19.
The Genetic Engineering Appraisal 3. The National Ganga River Basin Authority
Committee is constituted under the (2015) is chaired by the Prime Minister.
(a) Food Safety and Standards Act, 2006 Which of the statements given above is/are
(b)
Geographical Indications of Goods correct?

National Conservation Efforts 303


unacademy.com | Download the Unacademy app
Give your feedback here: Link
(a) 1 only (c) 1 and 3 only
(b) 2 and 3 only (d) 1, 2 and 3
(c) 2 only
(d) 1, 2 and 3 25. The National Green Tribunal Act, 2010 was
enacted in consonance with which of the
following provisions of the Constitution of
23. With reference to 'Eco-Sensitive Zones', India? (2012)
which of the following statements is/are
correct? (2014) 1. Right to healthy environment, construed
as a part of Right to life under Article 21.
1. Eco-Sensitive Zones are the areas
that are declared under the Wildlife 2. Provision of grants for raising the level
(Protection) Act, 1972. of administration in the Scheduled
Areas for the welfare of Scheduled
2. The purpose of the declaration of Eco Tribes under Article 275 (1).
Sensitive Zones is to prohibit all kinds of
human activities in those zones except 3. Powers and functions of Gram Sabha as
agriculture. mentioned under Article 243(A).

Select the correct answer using the code Select the correct answer using the code
given below: given below:

(a) 1 only (a) 1 Only

(b) 2 only (b) 2 and 3 Only

(c) Both 1 and 2 (c) 1 and 3 only

(d) Neither 1 nor 2 (d) 1, 2 and 3

24.
How does the National Biodiversity 26.
With reference to India, consider the
Authority (NBA) help in protecting Indian following Central Acts: (2011)
agriculture? (2012) 1. Import and Export (Control) Act, 1947
1. NBA checks the biopiracy and protects 2. Mining and Mineral Development
the indigenous and traditional genetic (Regulation) Act, 1957
resources. 3. Customs Act, 1962
2. NBA directly monitors and supervises 4. Indian Forest Act, 1927
scientific research on the genetic
modification of crop plants. Which of the above Acts have relevance to/
bearing on the biodiversity conservation in
3. Application for Intellectual Property the country?
Rights related to resources genetic/
biological cannot be made without the (a) 1 and 3 only
approval of the NBA. (b) 2, 3 and 4 only
Which of the statements given above is/are (c) 1, 2, 3 and 4
correct? (d) None of the above Acts
(a) 1 Only
(b) 2 and 3 only

304 National Conservation Efforts


unacademy.com | Download the Unacademy app
Give your feedback here: Link
National Conservation Efforts-
6 Explanation
1. Answer: (C) 3. Answer: (b)
Statement 1 is correct: Once an area is notified Statement 1 is not correct: Hunted wild animals
as a ‘Community Reserve’ by the Central and not all wild animals ar considered state
Government in India, the Chief Wildlife Warden property.
of the respective state becomes the governing Statement 2 is correct and 3 is not correct:
authority responsible for managing and
The Wildlife Protection Act protects all kinds of
overseeing the activities within the reserve.
animals from amphibians to birds, mammals,
Statement 2 is correct:In a ‘Community and reptiles under Section 2(1). The Chief
Reserve,’ hunting and any form of wildlife Wildlife Warden may if he is satisfied that any
exploitation are strictly prohibited to protect wild animal specified in Schedule I has become
the biodiversity and ensure the conservation dangerous to human life or is so disabled or
of wildlife species. diseased as to be beyond recovery, by order
Statement 3 is correct: ‘Community Reserves’ in writing and stating the reasons, therefore,
are established to involve local communities in permit any person to hunt such animal or
conservation efforts and promote sustainable cause such animal to be hunted.
utilisation of forest resources. Therefore,
people living in the area designated as a
4. Answer: (a)
‘Community Reserve’ are generally allowed to
collect non-timber forest produce for their The Wildlife Protection Act, 1972 provides
livelihoods, provided it is done in a sustainable for the protection of the country’s wild
manner and does not harm the ecosystem. animals, birds, and plant species, to ensure
environmental and ecological security. Among
Statement 4 is incorrect:’Community Reserves’
other things, the Act lays down restrictions on
primarily focus on conserving wildlife and
hunting many animal species.
biodiversity rather than facilitating traditional
agricultural practices. The primary objective Option (a) is correct: Schedule VI has been
is to protect and restore natural ecosystems added to include the specified plant species
and promote community participation in to be protected by the Wildlife (Protection)
conservation activities.However, traditional Amendment Act of 1991. Placement in Schedule
agricultural practices are not typically VI provides for regulation in the cultivation of
permitted in such areas. a specified plant and restricts its possession,
sale, and transportation. Both cultivation and
trade of specified plants can only be carried
2. Answer: (c) out with the prior permission of the competent
Option (c) is correct: Central Ground Water authority. Thus, a license is required to cultivate
Authority has been constituted under Section that plant.
3 (3) of the Environment (Protection) Act, 1986
to regulate and control the development and
5. Answer: (b)
management of groundwater resources in the
country. Statement 1 is not correct: Out of the 5723
assessment units assessed jointly by State
Ground Water Departments and CGWB
in the country, as per the report of Niti

National Conservation Efforts-Explanation 305


unacademy.com | Download the Unacademy app
Give your feedback here: Link
Aayog, the following is the data - Safe-71%, avoided. A transfer station at a suitable
Semi-critical-10%, Critical-4%, and Over enclosed location shall be set up to collect
Exploited-15%. Hence statement 1 is not residual waste from the processing facility
correct. and inert waste. Suitable land shall also be
Statement 2 is correct: Central Ground identified in the plain areas. (Option (c) is
Water Authority (CGWA) was constituted for correct)
the purposes of regulation and control of y Waste deposition centers for domestic
groundwater development and management in hazardous waste shall be established in a
the country, under sub-section (3) of Section 3 city or town in a manner that one center is set
of the Environment (Protection) Act, 1986. up for an area of twenty square kilometers
Statement 3 is correct: At 39 million hectares or part thereof and notify the timings of
(67% of its total irrigation), India has the world’s receiving domestic hazardous waste at
largest groundwater well-equipped irrigation such centers. So, it is not mandatory on
system. India, Pakistan, and Bangladesh are, the part of the waste generator that the
respectively, the first, fourth and sixth largest waste generated in one district cannot be
users of groundwater globally. India pumps moved to another district. (Option (d) is not
more than the US and China combined - the correct)
second and third-largest users, respectively.
7. Answer: (b)
6. Answer: (c) Statement 1 is not correct. The Environment
Salient features of Solid Waste Management Protection Act, 1986 empowers the Government
Rules, 2016: of India to take all measures as it deems
necessary or expedient for the purpose of
y Responsibilities of generators have been
protecting and improving the quality of the
introduced to segregate waste into three
environment and preventing controlling and
streams, Wet (Biodegradable), Dry (Plastic,
abating environmental pollution. It does not
Paper, metal, wood, etc.) and domestic
talk about public participation.
hazardous wastes. (Option (a) is not
correct) The Environmental Impact Assessment (EIA)
procedure is currently the central arena of public
y The Rules are now applicable beyond
decision-making in granting environmental
Municipal areas and extend to urban
clearances in India, and the procedure for
agglomerations, census towns, notified
conducting EIAs is provided through various
industrial townships, areas under the
executive notifications under the Environment
control of Indian Railways, airports, airbases,
Protection Act, 1986 (EPA).
Port and harbour, defence establishments,
special economic zones, State and Central Statement 2 is correct. The Environment
government organisations, places of Protection Act, 1986 empowers the Government
pilgrims, religious & historical importance. of India to lay down standards for the emission
(Option (b) is not correct) or discharge of environmental pollutants from
various sources.
y The Rules provide also for exact and
elaborate criteria for the identification of It also empowers the Government of India to
sites for landfills and waste processing plan and execute a nationwide programme
facilities. Special provision for the for the prevention, control and abatement of
management of solid waste in hilly areas: environmental pollution.
construction of landfill on the hill shall be

306 National Conservation Efforts-Explanation


unacademy.com | Download the Unacademy app
Give your feedback here: Link
8. Answer: (b) y Maharashtra (16.50%)
The Government of India recently amended the y Madhya Pradesh (25.14%)
Indian Forest Act, 1927, and the new changes y Odisha (33.15%)
can transform the bamboo sector. Earlier,
Bamboo was categorised as a tree. After y Chhattisgarh (41.13%)
amending Section 2(7) of the Indian Forest Act, Additional Information:
1927, bamboo is no longer a tree and felled y Area-wise Madhya Pradesh has the largest
bamboo too is not timber. As a result, felled or forest cover in the country followed by
extracted bamboo, whether found in or brought Arunachal Pradesh, Chhattisgarh, Odisha
from a forest, was considered as “timber”. The and Maharashtra.
Act empowered state governments to regulate
y In terms of forest cover as a percentage of
the trade and movement of bamboo.
total geographical area, the top five States
Statement 1 is not correct: The Bill permits are Mizoram (85.41%), Arunachal Pradesh
the felling and transit of bamboo grown in (79.63%), Meghalaya (76.33%), Manipur
non-forest areas. However, bamboo grown on (75.46%) and Nagaland (75.31%).
forest lands would continue to be classified
as a tree and would be guided by the existing
legal restrictions. 10. Answer: (a)
Statements 2 and 3 are correct: Scheduled The Compensatory Afforestation Fund
Tribes and Other Traditional Forest Dwellers Management and Planning Authority (CAMPA)
(Recognition of Forest Rights) Act, 2006, is meant to promote afforestation and
recognises bamboo as Minor Forest Produce regeneration activities as a way of compensating
(MFP) and vests the “right of ownership, access for forest land diverted to non-forest uses.
to collect, use and dispose of minor forest National CAMPA Advisory Council has been
produce” with Scheduled Tribes and traditional established as per orders of The Hon’ble
forest dwellers. Supreme Court with the following mandate:
The Forest Rights Act (FRA), 2006 recognizes the y Laying down broad guidelines for State
rights of the forest-dwelling tribal communities CAMPA.
and other traditional forest dwellers to forest
y Facilitating scientific, technological and
resources, on which these communities were
other assistance that may be required by
dependent for a variety of needs, including
State CAMPA.
livelihood, habitation and other socio-cultural
needs. y Making recommendations to State CAMPA
based on a review of their plans and
The Gram Sabha is a highly empowered body
programmes.
under the Act, enabling the tribal population
to have a decisive say in the determination of y Providing a mechanism to State CAMPA to
local policies and schemes impacting them. resolve issues of an inter-state or Centre-
State character.
Statement 1 is correct: As per law, the
9. Answer: (c)
Compensatory Afforestation Fund Management
Option (c) is correct: According to ‘The India and Planning Authority exists at both National
State of Forest Report (ISFR) 2019’, the total and State levels.
forest cover is 21.67% of the total geographical
Statement 2 is not correct: The provisions
area of the country. In terms of the percentage
laid out in the rules that limit compliance
of forest cover to the total area of the State,
with the FRA only to those areas where
the correct ascending order as follows:

National Conservation Efforts-Explanation 307


unacademy.com | Download the Unacademy app
Give your feedback here: Link
rights holders have been formally recognised. environmental matters provides speedy
Moreover, it does not provide for mandatory environmental justice and helps reduce
people’s participation in the compensatory the burden of litigation in the higher courts.
afforestation programmes carried out under Principal Functions of the CPCB, as spelled
the Compensatory Afforestation Fund Act, out in the Water (Prevention and Control of
2016. Pollution) Act, 1974, and the Air (Prevention and
Control of Pollution) Act, 1981, is to promote the
cleanliness of streams and wells in different
11. Answer: (c)
areas of the States by prevention, control, and
Extended producer responsibility is a practice abatement of water pollution, and to improve
and a policy approach in which producers the quality of air and to prevent, control or
take responsibility for the management of the abate air pollution in the country.
disposal of products they produce once those
products are designated as no longer useful by
consumers. 13. Answer: (a)

A collection mechanism-based approach Statement 1 is correct: The definition ‘Critical


has been adopted, which includes collection Wildlife Habitat’ is incorporated in the
centers, collection points, take-back systems, Scheduled Tribes and Other Traditional Forest
etc for the collection of e-waste by producers Dwellers (Recognition of Forest Rights) Act,
under Extended Producer Responsibility. 2006.

Option (c) is correct: In India, ‘extended Statement 2 is correct: Baiga Tribals became
producer responsibility’ was introduced as an India’s first community to get Habitat Rights.
important feature in the e-Waste (Management Baigas are considered as a Particularly
and Handling) Rules, 2011. Vulnerable Tribal Group (PVTG) in the Indian
Constitution and rely mostly on shifting
cultivation, forest produce and fishing for
12. Answer: (b) sustenance. The tribe numbers only 150,000
Statement 1 is not correct: The National Green people spread over forested areas of Madhya
Tribunal has been established under the Pradesh and Chhattisgarh.
National Green Tribunal Act 2010 for effective Statement 3 is not correct: The definition of
and expeditious disposal of cases relating to Habitat Rights was incorporated through an
environmental protection and conservation of Amendment in the FRA in 2012. As per the
forests and other natural resources including amendment, the District Level Committee
enforcement of any legal right relating to the under the Act shall ensure that all PVTGs
environment and giving relief and compensation receive Habitat Rights, in Consultation with
for damages to persons and property and for the concerned traditional institutions of these
matters connected therewith or incidental groups, after filing claims before the Gram
thereto. Whereas, the Central Pollution Control Sabha.
Board (CPCB), a statutory organisation, was
constituted in September 1974 under the Water
(Prevention and Control of Pollution) Act, 1974. 14. Answer: (b)
Further, CPCB was entrusted with the powers Option (b) is correct: M-STrIPES (Monitoring
and functions under the Air (Prevention and System for Tigers-Intensive Protection and
Control of Pollution) Act, 1981. Ecological Status) is a platform where modern
Statement 2 is correct: The National technology is used to assist in effective
Green Tribunal’s dedicated jurisdiction in patrolling, assess ecological status, and
mitigate human-wildlife conflict in and around

308 National Conservation Efforts-Explanation


unacademy.com | Download the Unacademy app
Give your feedback here: Link
tiger reserves. 16. Answer: (d)
The MSTrIPES program uses Global Positioning The Wildlife Protection Act was implemented
System (GPS), General Packet Radio Services in 1972 in India. The thrust of the programme
(GPRS), and remote sensing to collect was towards protecting the remaining
information from the field, create a database population of certain endangered species
using modern Information Technology (IT) by banning hunting, giving legal protection
based tools, analyses the information using to their habitats, and restricting trade in
GIS and statistical tools to provide inferences wildlife. Subsequently, central and many state
that allow tiger reserve managers to manage governments established national parks and
their wildlife resources better. wildlife sanctuaries.
Other software used for tiger estimation were: Option (d) is correct: The central government
Spatially Explicit Capture-Recapture (SECR) also announced several projects for protecting
and Extract Compare, which helps differentiate specific animals, which were gravely threatened,
between the stripes of tigers as each has a including the tiger, the one-horned rhinoceros,
different pattern. the Kashmir stag or hangul, three types of
crocodiles – freshwater crocodile, saltwater
crocodile and the Gharial, the Asiatic lion, and
15. Answer: (a)
others. Most recently, the Indian elephant,
The Wildlife Protection Act was implemented blackbuck (chinkara), the great Indian bustard
in 1972 in India. The thrust of the programme (godawan) and the snow leopard, etc., have
was towards protecting the remaining been given full or partial legal protection against
population of certain endangered species hunting and trade throughout India. Indian Wild
by banning hunting, giving legal protection Ass (Equus hemionus khur) and Wild Buffalo
to their habitats, and restricting trade in (Bubalus bubalis) are also protected under
wildlife. Subsequently, central and many state Schedule I of the act.
governments established national parks and
wildlife sanctuaries.
17. Answer: (a)
Option (a) is correct: In India, if a species of
tortoise is declared protected under Schedule Red Sanders, botanical name Pterocarpus
I of the Wildlife (Protection) Act, 1972, it means santalinus, is a non-fragrant variety of
it enjoys the same level of protection as the sandalwood that mostly grows in rocky, hilly
tiger. No person can hunt that species except regions.
as provided by Law. Statement 1 is correct: Red Sanders wood
Additional Information: fetches huge prices in the international
market for its use in making luxury products,
No person shall hunt any wild animal specified
musical instruments, and medicine. species is
in Schedule I, II, III and IV except as provided
endemic to India. It is found in Andhra Pradesh,
under section 11 and section 12. The rating of
Karnataka, and Tamil Nadu.
Schedules I to V is in accordance with the risk
of survival of the wildlife (fauna) enlisted in Statement 2 is not correct: It is found in the
them. Animals included in these Schedules thorny scrub/dry deciduous forests of the
are provided for total protection from hunting, central Deccan and not in the tropical rainforest
and the trade and commerce related to such areas of South India.
animals are strictly regulated. Schedule VI The International Union for Conservation of
has been added to include the specified Nature (IUCN), an international organisation
plant species to be protected by the Wildlife for nature conservation, classified red sanders
(Protection) Amendment Act of 1991. (Pterocarpus santalinus) as ‘near threatened’.

National Conservation Efforts-Explanation 309


unacademy.com | Download the Unacademy app
Give your feedback here: Link
Red sanders remain listed in Appendix II of Climate Change (erstwhile, the Ministry of
CITES (Convention on International Trade in Environment and Forest) in 2010, created
Endangered Species of Wild Fauna and Flora). an Expert Committee to study the situation
in the Western Ghat and recommend how
best to reconcile the sometimes-competing
18. Answer: (d) claims of environment and development. The
Option (d) is correct: ‘Gadgil Committee Committee was headed by Madhav Gadgil, a
Report’ and ‘Kasturirangan Committee Report’ world-renowned ecologist and a pragmatic
are related to the protection of Western Ghats. scientist. The Kasturirangan Panel was set up
The Ministry of Environment, Forest, and to study the Gadgil Committee report on the
Western Ghats.

310 National Conservation Efforts-Explanation


unacademy.com | Download the Unacademy app
Give your feedback here: Link
The Gadgil panel report had faced unanimous under the Environment (Protection) Act,
opposition from the State government for 1986.
recommending that almost three-fourths of
the hills, including plantations, cultivated lands
and large habitations, be turned into restricted 20. Answer: (c)
development zones. The Kasturirangan The Bombay Natural History Society (BNHS),
Committee has in contrast advised against a pan-India wildlife research organisation,
bringing cultivated lands, plantations, and has been promoting the cause of nature
habitations outside the ambit of such a conservation since 1883.
restrictive regime called Ecologically Sensitive Statement 1 is not correct: It is a premier
Area (ESA) under the Environment Protection independent scientific non-governmental
Act, 1976. Instead, it has suggested that 90% organisation (not an autonomous organisation
of the natural forests left in the Western under the Ministry of Environment and Forests),
Ghats complex- adding up to 60,000 S. Km with a broad-based constituency, excelling in
and constituting 37% of the entire hilly belt- the conservation of threatened species and
be conserved under the ESA Provision of the habitats.
green law.
Statement 2 and statement 3 are correct: It
The Panel has recommended that there should aims at the conservation of nature, primarily
be a complete ban on mining activity in this biological diversity through action based on
zone and current mining activities should be research, education, and public awareness
phased out within five years. through camps. It also organises and conducts
nature trails and camps for the general public.
19. Answer: (c)
Option (c) is correct: The Genetic Engineering 21. Answer: (a)
Appraisal Committee is a statutory body
Option (a) is correct: The most important
constituted under the ‘Rules for the
strategy for the conservation of biodiversity
Manufacture, Use /Import /Export and Storage
together with traditional human life is
of Hazardous Microorganisms / Genetically
the establishment of biosphere reserves.
Engineering Organisms or Cells, 1989’ notified
Biosphere reserves are ‘learning places for
under the Environment (Protection) Act,
sustainable development’. They are sites
1986, is mandatory before commercial use of
for testing interdisciplinary approaches to
genetically modified organisms and products
understanding and managing changes and
derived therefrom.
interactions between social and ecological
The major functions of GEAC as prescribed in systems, including conflict prevention and
the Rules, 1989 are: management of biodiversity.
y To appraise activities involving large-scale Biosphere Reserves involve local communities
use of hazardous microorganisms GE and all interested stakeholders in planning
organisms or cells in research and industrial and management. They integrate three main
production from the environmental angle. “functions”:
y To appraise proposals relating to the y Conservation of biodiversity and cultural
release of GE organisms and products into diversity
the environment, including experimental
y Economic development that is socio-
field trials.
culturally and environmentally sustainable
y The committee or any persons authorised
y Logistic support, underpinning development
by it has the powers to take punitive action

National Conservation Efforts-Explanation 311


unacademy.com | Download the Unacademy app
Give your feedback here: Link
through research, monitoring, education 23. Answer: (d)
and training Statement 1 is not correct: Eco-Sensitive Zones
These three functions are pursued through the or Ecologically Fragile Areas are areas within 10
Biosphere Reserves’ three main zones km around Protected Areas, National Parks and
y Core Areas: It comprises a strictly protected Wildlife Sanctuaries. ESZs are notified by the
zone that contributes to the conservation Ministry of Environment Forest and Climate
of landscapes, ecosystems, species and Change (MoEFCC), Government of India under
genetic variation the Environment Protection Act 1986.

y Buffer Zones: These surrounds or adjoins Statement 2 is not correct: Commercial mining,
the core area(s) and is used for activities setting of sawmills and industries causing
compatible with sound ecological practices pollution, commercial use of firewood and
that can reinforce scientific research, major hydro-power projects, are prohibited in
monitoring, training, and education. such areas. Activities permitted in the areas
include ongoing agriculture and horticulture
y Transition Area: The transition area is
practices by local communities, rainwater
where community foster socio-culturally
harvesting, organic farming, adoption of green
and ecologically sustainable economic and
technology and use of renewable energy
human activities.
sources.

22. Answer: (b)


24. Answer: (c)
Statement 1 is not correct: Animal Welfare
The National Biodiversity Authority (NBA) was
Board of India is a Statutory advisory Body
established in 2003 by the Central Government
created under the Prevention of Cruelty to
to implement India’s Biological Diversity
Animals Act. It was set up in 1962 to advise on
Act (2002). The NBA is a Statutory body that
animal welfare laws and policymaking.
performs a facilitative, regulatory and advisory
Statement 2 is correct: National Tiger function for the Government of India on the
Conservation Authority (NTCA) is a Statutory issue of Conservation, sustainable use of
Body under the Ministry of Environment, Forests biological resources and fair equitable sharing
and Climate Change. It was established in 2005 of benefits of use.
following the recommendations of the Tiger
Statement 1 is correct: The NBA is a Statutory
Task Force. It was constituted under enabling
body set up with the main objective of
provisions of the Wildlife (Protection) Act, 1972,
conservation of biological diversity, sustainable
as amended in 2006, for strengthening tiger
use of its components and fair and equitable
conservation, as per powers and functions
sharing of the benefits arising out of utilisation
assigned to it.
of genetic resources.
Statement 3 is correct: National Ganga River
Statement 2 is not correct: The National
basin authority was constituted under the
Biodiversity Authority is not responsible for
Environment Protection Act in 2009 as a
monitoring and supervising the scientific
Statutory Body headed by the Prime Minister.
research on the genetic modification of crop
The main functions are conservation and
plants. In India, the top biotech regulator for
abatement of pollution of Ganga, the national
Genetically Modified Organisms is the Genetic
river. Chief Ministers of a few states through
Engineering Appraisal Committee (GEAC).
which the river passes are also members.
Statement 3 is correct: Under Section 6(1) of
the Biological Diversity Act, 2002 and Biological
Diversity Rules 2004, no person shall apply for

312 National Conservation Efforts-Explanation


unacademy.com | Download the Unacademy app
Give your feedback here: Link
any intellectual property right, by whatever of administration in the Scheduled Areas for
name called, in or outside India for any the welfare of Scheduled Tribes under Article
invention based on any research or information 275 (1) and Powers and functions of Gram
on a biological resource obtained from India Sabha as mentioned under Article 243(A).
without obtaining the previous approval of the
Elimination Technique: The National
National Biodiversity Authority before making
Green Tribunal Act, 2010 was enacted in
such application. Provided that if a person
consonance with the right to a healthy
applies for a patent, permission of the National
environment. It is not related to the
Biodiversity Authority may be obtained after
Powers and functions of Gram Sabha as
the acceptance of the patent but before the
mentioned under Article 243(A). Statement
sealing of the patent by the patent authority
3 is not correct. Thus, Option (a) is correct.
concerned: Provided further that the National
Biodiversity Authority shall dispose of the
application for permission made to it within a 26. Answer: (c)
period of ninety days from the date of receipt
Option (c) is correct: In India there are various
thereof.
Central Acts that have relevance to biodiversity.
Elimination Technique: Genetic Some of them are:
Engineering Appraisal Committee (GEAC) is y Import and Export (Control) Act, 1947 and
regulating Genetically Modified Organisms. Customs Act, 1962: Under these Acts,
Statement 2 is not correct. So, we are left exotic species were prohibited for import
with Option (a) and Option (c). and export, and plants with medicinal
importance attracted high customs duty.
Foreign Trade Act has replaced the Import
25. Answer: (a)
and Export (Control) Act 1947.
Option (a) is correct: The National Green
y Mining and Mineral Development
Tribunal Act, 2010 was enacted in consonance
(Regulation) Act, 1957: It regulates the
with Article 21 of the Indian Constitution. In the
mining sector in India and specifies the
judicial pronouncement in India, the right to a
requirement for obtaining and granting
healthy environment has been constructed as
mining leases for mining operations. Under
a part of the right to life under Article 21 of the
the Act, a person could acquire one mining
Indian Constitution.
lease for a maximum area of 10 sq. km. Thus,
The National Green Tribunal Act, 2010, is an Act preserving the areas where mining can be
to provide for the establishment of a National done, it helps protect the environment.
Green Tribunal for the effective and expeditious
y Indian Forest Act, 1927: The Indian Forest
disposal of cases relating to environmental
Act of 1927 empowered Indian state
protection and conservation of forests and
governments to enact rules regulating
other natural resources including enforcement
various aspects of forest management, the
of any legal right relating to the environment
rules differ from state to state.
and giving relief and compensation for damages
to persons and property and for matters y There are several other Acts which is related
connected therewith or incidental thereto. to biodiversity, such as Wildlife Protection
Act, Environment Protection Act, Seeds
The said Act was not enacted in consonance
Act, Fisheries Act, etc.
with the Provision of grants for raising the level

National Conservation Efforts-Explanation 313


unacademy.com | Download the Unacademy app
Give your feedback here: Link
Climate Change: Causes and
7 Implications
1. Consider the following statements : (2023) non-profit organization that drives
Statement-I: climate action by building large
networks and runs them.
Carbon markets are likely to be one of the
most widespread tools in the fight against 2. The International Energy Agency in
climate change. partnership with the Climate Group
launched a global initiative “EP100”.
Statement-II :
3. EP100 brings together leading companies
Carbon markets transfer resources from
committed to driving innovation
the private sector to the State.
in energy efficiency and increasing
Which one of the following is correct in competitiveness while delivering on
respect of the above statements? emission reduction goals.
(a) Both Statement-I and Statement-II are 4. Some Indian companies are members
correct and Statement-II is the correct of EP100.
explanation for Statement-I
5. The International Energy Agency is the
(b)
Both Statement-I and Statement-II Secretariat to the “Under2 Coalition”.
are correct and Statement-II is not the
Which of the statements given above are
correct explanation for Statement-I
correct ?
(c) Statement-I is correct but Statement-II
(a) 1, 2, 4 and 5
is incorrect
(b) 1, 3 and 4 only
(d) Statement-I is incorrect but Statement-
II is correct (c) 2, 3 and 5 only
(d) 1, 2, 3, 4 and 5

2. “Climate Action Tracker” which monitors


the emission reduction pledges of different 4. Which one of the following statements best
countries is a : (2022) describes the term 'Social Cost of Carbon'?
(a)
Database created by coalition of (2020)
research organisations It is a measure, in monetary value, of the
(b) Wing of “International Panel of Climate (a) long-term damage done by a tonne of
Change” CO2 emissions in a given year.
(c)
Committee under “United Nations (b) requirement of fossil fuels for a country
Framework Convention on Climate to provide goods and services to its
Change” citizens, based on the burning of those
(d) Agency promoted and financed by fuels.
United Nations Environment Programme (c) efforts put in by a climate refugee to
and World Bank adapt to live in a new place.
(d) contribution of an individual person
3. Consider the following statements: (2022) to the carbon footprint on the planet
Earth.
1. “The Climate Group” is an international

314 Climate Change: Causes and Implications


unacademy.com | Download the Unacademy app
Give your feedback here: Link
5. Which of the following statements are the increased concentration of carbon
correct about the deposits of ‘methane dioxide in the atmosphere
hydrate’? (2019)
1. Global warming might trigger the release 8. With reference to 'Global Climate Change
of methane gas from these deposits. Alliance', which of the following statements
2. Large deposits of ‘methane hydrate’ are is/are correct? (2017)
found in Arctic Tundra and under the 1. It is an initiative of the European Union.
seafloor.
2. It provides technical and financial
3. Methane in the atmosphere oxidised to support to targeted developing countries
carbon dioxide after a decade or two. to integrate climate change into their
Select the correct answer using the code development policies and budgets.
given below. 3. It is coordinated by World Resources
(a) 1 and 2 only Institute (WRI) and World Business
(b) 2 and 3 only Council for Sustainable Development
(WBCSD).
(c) 1 and 3 only
Select the correct answer using the code
(d) 1, 2 and 3 given below:
(a) 1 and 2 only
6. In the context of which of the following do (b) 3 only
some scientists suggest the use of cirrus
cloud thinning technique and the injection (c) 2 and 3 only
of sulphate aerosol into the stratosphere? (d) 1, 2 and 3
(a) Creating artificial rains in some regions. 9. In the context of mitigating the impending
(b) Reducing the frequency and intensity of global warming due to anthropogenic
tropical cyclones. emissions of carbon dioxide, which of the
following can be potential sites for carbon
(c) Reducing the adverse effects of solar sequestration? (2017)
wind on the Earth.
1. Abandoned and uneconomic coal seams
(d) Reducing global warming.
2. Depleted oil and gas reservoirs
3. Subterranean deep saline formations
7. Which of the following statements best
describes "carbon fertilisation"? (2018) Select the correct answer using the code
given below:
(a) Increased plant growth due to increased
concentration of carbon dioxide in the (a) 1 and 2 only
atmosphere (b) 3 only
(b) Increased temperature of Earth due (c) 1 and 3 only
to increased concentration of carbon (d) 1, 2 and 3
dioxide in the atmosphere
(c) Increased acidity of oceans as a result
of increased concentration of carbon 10. The term ‘Intended Nationally Determined
dioxide in the atmosphere Contributions’ is sometimes seen in the
news in the context of (2016)
(d) Adaptation of all living beings on Earth
to the climate change brought about by (a) pledges made by the European countries

Climate Change: Causes and Implications 315


unacademy.com | Download the Unacademy app
Give your feedback here: Link
to rehabilitate refugees from the war- Select the correct answer using the code
affected Middle East given below.
(b) plan of action outlined by the countries (a) 1 only
of the world to combat climate change (b) 2 and 3 only
(c) capital contributed by the member (c) 3 only
countries in the establishment of Asian
Infrastructure Investment Bank (d) 1, 2 and 3

(d) plan of action outlined by the countries


of the world regarding Sustainable 13. What is the ‘Greenhouse Gas Protocol’?
Development Goals (2016)
(a) It is an international accounting tool
11. Consider the following pairs: (2016) for government and business leaders
to understand, quantify and manage
Terms sometimes seen in the news Their greenhouse gas emissions.
origin
(b) It is an initiative of the United Nations to
1. Annex-I Countries : Cartagena Protocol offer financial incentives to developing
2. Certified Emissions Reductions : Nagoya countries to reduce greenhouse gas
Protocol emissions and to adopt eco-friendly
3. Clean Development Mechanisms: Kyoto technologies
Protocol (c) It is an intergovernmental agreement
Which of the pairs given above is/are ratified by all the member countries
correctly matched? of the United Nations to reduce
greenhouse gas emissions to specified
(a) 1 and 2 only levels by the year 2022
(b) 2 and 3 only (d) It is one of the multilateral REDD+
(c) 3 only initiatives hosted by the World Bank
(d) 1, 2 and 3
14. With reference to the agreement at the
12. Which of the following best describes the UNFCCC Meeting in Paris in 2015, which of
aim of the ‘Green India Mission’ of the the following statements is/are correct?
Government of India? (2016) (2016)

1. Incorporating environmental benefits 1. The agreement was signed by all the


and costs into the Union and State member countries of the UN, and it will
Budgets thereby implementing the go into effect in 2017.
‘green accounting’. 2. The agreement aims to limit greenhouse
2. Launching the second green revolution gas emissions so that the rise in average
to enhance agricultural output so as to global temperature by the end of this
ensure food security to one and all in century does not exceed 2oC or even
the future. 1.5oC above pre-industrial levels.

3. Restoring and enhancing forest cover 3. Developed countries acknowledged


and responding to climate change their historical responsibility in global
by a combination of adaptation and warming and committed to donate $
mitigation measures. 1000 billion a year from 2020 to help

316 Climate Change: Causes and Implications


unacademy.com | Download the Unacademy app
Give your feedback here: Link
developing countries to cope with given below:
climate change. (a) 1 only
Select the correct answer using the code (b) 1 and 2 only
given below.
(c) 2, 3 and 4 only
(a) 1 and 3 only
(d) 1, 2, 3 and 4
(b) 2 only
(c) 2 and 3 only
17. The increasing amount of carbon dioxide in
(d) 1, 2 and 3 the air is slowly raising the temperature of
the atmosphere, because it absorbs (2012)
15.
Which of the following statements (a) the water vapour of the air and retains
regarding 'Green Climate Fund' is/are its heat.
correct? (2015) (b) the UV part of the solar radiation.
1. It is intended to assist developing (c) all the solar radiations.
countries in adaptation and mitigation
practices to counter climate change. (d) the infrared part of the solar radiation.

2. It is founded under the aegis of UNEP,


OECD, Asian Development Bank and 18. Regarding “carbon credits’’, which one of
World Bank. the following statements is not correct?
Select the correct answer using the code (2011)
given below: (a) The carbon credit system was ratified
(a) 1 only in conjunction with the Kyoto protocol.

(b) 2 only (b) Carbon credits are awarded to countries


or groups that have reduced greenhouse
(c) Both 1 and 2 gases below their emission quota.
(d) Neither 1 nor 2 (c) The goal of the carbon credit system is
to limit the increase of carbon dioxide
16. The scientific view is that the increase in emission.
global temperature should not exceed 20C (d) Carbon credits are traded at a price
above pre-industrial level. If the global fixed from time to time by the United
temperature increases beyond 30C above Nations environment programs.
the pre-industrial level, what can be its
possible impact/impacts on the world?
(2014) 19. Human activities in the recent past have
caused the increased concentration of
1. Terrestrial biosphere tends toward a net carbon dioxide in the atmosphere, but
carbon source. a lot of it does not remain in the lower
2. Widespread coral mortality will occur. atmosphere because (2011)
3. All the global wetlands will permanently 1. its escape into the outer stratosphere
disappear. 2. the photosynthesis by phytoplankton in
4. Cultivation of cereals will not be the oceans
possible anywhere in the world. 3. the trapping of air in the polar ice caps
Select the correct answer using the code Which of the statements given above is/are

Climate Change: Causes and Implications 317


unacademy.com | Download the Unacademy app
Give your feedback here: Link
correct?
(a) 1 and 2 only
(b) 2 only
(c) 2 and 3 only
(d) 3 only

318 Climate Change: Causes and Implications


unacademy.com | Download the Unacademy app
Give your feedback here: Link
Climate Change: Causes and
7 Implications-Explanation
1. Answer: (b) Statement 2 is not correct: E100 is a global
Statement 1 is correct: Carbon markets are initiative led by the international non-profit
indeed poised to become one of the most Climate Group,
widespread tools in the global efforts to Statements 3 and 4 are correct: EP100 is a
combat climate change. A carbon market is global corporate energy efficiency initiative
a mechanism that enables the buying and bringing together over 120 ambitious
selling of carbon credits, which represent the businesses committed to measuring and
right to emit a certain amount of greenhouse reporting on efficiency improvements. Energy
gases (GHGs).These markets create a financial efficiency is one of the fastest, cheapest and
incentive for reducing emissions by putting a most accessible energy resources. Automobile
price on carbon. giant Mahindra & Mahindra has become the
Statement 2 is correct: Carbon markets involve first Indian company to join a global energy
financial transactions between entities in the campaign, EP 100.
private sector, however, it is important to note Statement 5 is not correct: The Climate Group
that they do not necessarily transfer resources acts as a secretariat to the Under 2 Coalition.
exclusively from the private sector to the It is made up of over 260 governments globally,
state. The functioning of carbon markets can representing 1.75 billion people and 50% of
vary depending on their design and regulatory the global economy, committed to ambitious
framework. climate action in line with the Paris Agreement.
The Co-Chairs are supported by a Steering
Group, which determines and oversee the
2. Answer: (a) strategic direction.
Option (a) is correct: The Climate Action
Tracker is an independent scientific analysis
that tracks government climate action and 4. Answer: (a)
measures it against the globally agreed Paris Option (a) is correct: The social cost of carbon
Agreement aim of “holding warming well below is a measure of the economic harm from those
2°C and pursuing efforts to limit warming to impacts, expressed as the dollar value of the
1.5°C.” total damages from emitting one ton of carbon
It is a collaboration of two organizations, dioxide into the atmosphere in a given year.
Climate Analytics and New Climate Institute. Carbon dioxide emissions are costing the
It aggregates country’s actions to the global Indian economy up to $210 billion every year. It
level and evaluates climate change mitigation is likely to suffer the highest economic damage
targets, policies and actions. from climate change after the US. The country-
level SCC for India alone is estimated to be
about $86 per tonne of CO2. For the US, the
3. Answer: (b) cost is about $50 billion per tonne. This means
Statement 1 is correct: “The Climate Group” that the nearly five billion metric tonnes of
is an international non-profit organization CO2 the US emits each year is costing the US
that drives climate action by building large economy about $250 billion.
networks and running them.

Climate Change: Causes and Implications-Explanation 319


unacademy.com | Download the Unacademy app
Give your feedback here: Link
5. Answer: (d) y Thinning of the high-altitude ice clouds
Statement 1 is correct: Methane hydrate is a allows more terrestrial radiation to be
crystalline solid that consists of a methane emitted to space. Additionally, the potent
molecule surrounded by a cage of interlocking greenhouse gas water vapour is removed
water molecules. Methane hydrate is an from the upper troposphere. Thus, global
“ice” that only occurs naturally in subsurface climatic cooling can be achieved.
deposits where temperature and pressure y A controlled and targeted release of certain
conditions are favourable for its formation. As aerosols, such as sulphur, will roughly offset
Global Warming led to a rise in temperature the effect of several thousand kilograms of
therefore which might trigger the release of carbon dioxide.
methane gas from these deposits. y SAI could cause droughts or extreme
Statement 2 is correct: Enormous amounts weather in other parts of the World; harm
of methane hydrate have been found beneath crop yields as well as cause potential public
Arctic permafrost, beneath Antarctic ice, and health and governance issues.
in sedimentary deposits along continental
margins worldwide.
7. Answer: (a)
Statement 3 is correct: When methane is
anthropogenically emitted, it is oxidised in Option (a) is correct: A higher concentration
the atmosphere a decade or two later. Once of carbon dioxide in the atmosphere aids
oxidised, the carbon in each methane molecule photosynthesis, which in turn contributes to
is converted to CO2, which then stays in the increased plant growth. This is known as the
carbon fertilisation effect.
atmosphere as CO2 for another century or
more. Under higher CO2, plants do not lose so
much water through their leaves, thereby less
impacted by drier conditions. Carbon dioxide
6. Answer: (d) fertilisation is not the only cause of increased
Option (d) is correct: Cirrus clouds are known plant growth- nitrogen, land cover change and
as high-altitude clouds, as they are at a height climate change by way of global temperature,
of 6-18 km above the Earth’s surface. They precipitation and sunlight changes all
trap the heat of the atmosphere, and thinning contribute to the greening effect.
them reduces their heat-trapping capacity
hence could bring down the heat of the Earth’s
8. Answer: (a)
climate.
Statement 1 is correct: The Global Climate
Stratospheric Aerosol Injection (SAI) proposes
Change Alliance Plus (GCCA+) is a European
spraying particles into the upper atmosphere to
Union flagship initiative that is helping the
block sunlight. SAI acts as a reflective barrier
world’s most vulnerable countries to address
against incoming sunlight ultimately resulting
climate change. Having started with just four
in cooling the planet. The use of sulphate is
pilot projects in 2008, it has become a major
due to its property to be able to reflect all
climate initiative that has funded over 80
radiations.
projects of national, regional, and worldwide
Thus, the Cirrus cloud thinning technique scope in Africa, Asia, the Caribbean and the
and the injection of sulphate aerosol into Pacific.
the stratosphere are used to reduce global
Statement 2 is correct: The GCCA+ also
warming.
supports these groups of countries in
Additional Information: implementing their commitments resulting

320 Climate Change: Causes and Implications-Explanation


unacademy.com | Download the Unacademy app
Give your feedback here: Link
from the 2015 Paris Agreement on Climate Option (d) is correct: Carbon sequestration
Change (COP21), in line with the 2030 Agenda may be carried out by pumping carbon into
for Sustainable Development and the new 'carbon sinks'— an area that absorbs carbon.
European Consensus on Development. It Natural sinks - Oceans, forests, soil, etc.
provides technical and financial support to Artificial sinks - Geologic formations suitable
targeted developing countries to integrate for sequestration include depleted (abandoned
climate change into their development policies and uneconomic) oil and gas fields (reservoirs),
and budgets. deep (abandoned and uneconomic) coal seams,
Statement 3 is not correct: Technical support and subterranean deep saline formations.
and knowledge sharing are provided by the
global EU GCCA+ Support Facility – within the 10. Answer: (b)
thematic Global Public Goods and Challenges
(GPGC) programme under the Development Option (b) is correct: Nationally Determined
Cooperation Instrument (DCI) – and by Contributions (NDC) or Intended Nationally
the Intra-ACP Support Facility – under the Determined Contributions (INDC) are at
European Development Fund (EDF). the heart of the Paris Agreement and the
achievement of these long-term goals. NDCs
Additional Information embody efforts by each country to reduce
Greenhouse Gas Protocol: national emissions and adapt to the impacts
y It establishes comprehensive global of climate change.
standardised frameworks to measure and The Paris Agreement requires each Party to
manage greenhouse gas (GHG) emissions prepare, communicate and maintain successive
from private and public sector operations, nationally determined contributions (NDCs)
value chains and mitigation actions. that it intends to achieve. Parties shall pursue
y The GHG Protocol arose when the domestic mitigation measures, with the aim of
World Resources Institute (WRI) and achieving the objectives of such contributions.
World Business Council for Sustainable Additional Information:
Development (WBCSD) recognized the need India’s INDC: India has pledged to improve
for an international standard for corporate the emissions intensity of its GDP by 33 to 35
GHG accounting and reporting in the late percent by 2030 below 2005 levels. It has also
1990s. pledged to increase the share of non-fossil
fuels-based electricity to 40 percent by 2030.
9. Answer: (d) It has agreed to enhance its forest cover which
will absorb 2.5 to 3 billion tonnes of carbon
Carbon capture and storage, also known
dioxide (CO2, the main gas responsible for
as carbon sequestration, describes the
global warming) by 2030.
technologies designed to tackle global
warming by capturing CO2 at power stations,
industrial sites or even directly from the air 11. Answer: (c)
and permanently storing it underground. It Pair 3 is correctly matched: The Kyoto
describes the long-term storage of carbon Protocol was adopted in Kyoto, Japan, on 11
dioxide or other forms of carbon to either December 1997. Due to a complex ratification
mitigate or defer global warming. It has been process, it entered into force on 16 February
proposed as a way to slow the atmospheric 2005. It commits industrialised countries to
and marine accumulation of greenhouse gases, stabilise greenhouse gas emissions based
which are released by burning fossil fuels. on the principles of the Convention. Annex-I

Climate Change: Causes and Implications-Explanation 321


unacademy.com | Download the Unacademy app
Give your feedback here: Link
countries, Certified emissions reductions, and Plan on Climate Change (NAPCC). The Mission
clean development mechanism, all three are recognizes that climate change phenomena
provisions of Kyoto Protocol. will seriously affect and alter the distribution,
Key outcomes of Kyoto Protocol: type, and quality of natural resources of the
country and the associated livelihoods of the
y Emission reduction commitment: The people.
first was binding emissions reduction
commitments for developed country The Green India Mission acknowledges the
parties. This meant the space to pollute influences that the forestry sector has on
was limited. environmental amelioration through climate
mitigation, food security, water security,
y Flexible market mechanism: It is based on biodiversity conservation and livelihood
the trade of emissions permits. Countries security of forest-dependent communities.
are bound to targets that must meet
largely through domestic action— that is, Option (c) is correct: The aim of the National
to reduce their emissions onshore. But they Mission for a Green India is to increase
can meet part of their targets through three forest/tree cover on 5 m ha of forest/non-
“market-based mechanisms” that ideally forest lands and improve the quality of forest
encourage GHG abatement to start where cover on another 5 m ha (a total of 10 m
it is most cost-effective-- for example, in ha), Improved ecosystem services including
the developing world. biodiversity, hydrological services, and carbon
sequestration as a result of treatment of 10 m
Additional Information: ha., Increased forest-based livelihood income
Nagoya Protocol: The Nagoya Protocol on for 3 million forest-dependent households and
“Access to Genetic Resources and the Fair and enhanced annual CO2 sequestration of 50-60
Equitable Sharing of Benefits Arising from their million tonnes by the year 2020.
Utilisation to the Convention on Biological
Respond to climate change by a combination
Diversity” is an international agreement that
of adaptation and mitigation measures,
aims at sharing the benefits arising from the
which would help in enhancing carbon sinks
utilisation of genetic resources in a fair and
in sustainably managed forests and other
equitable way.
ecosystems, the adaptation of vulnerable
Cartagena Protocol: The Cartagena Protocol species/ecosystems to the changing climate
on Biosafety to the Convention on Biological and adaptation of forest-dependent local
Diversity is an international agreement that communities in the face of climatic variability.
aims to ensure the safe handling, transport,
and use of living-modified organisms (LMOs)
resulting from modern biotechnology that may 13. Answer: (a)
have adverse effects on biological diversity, Option (a) is correct: Greenhouse Gas
taking also into account risks to human health. Protocol establishes comprehensive global
standardised frameworks to measure and
Elimination Technique: Certified emissions
manage greenhouse gas emissions from private
reduction is one of the provisions of the
and public sector operations, value chains
Kyoto Protocol. Pair 2 is not correctly
and mitigation actions. It is an international
matched. Thus, Option (c) is correct.
accounting tool for government and business
leaders to understand, quantify and manage
12. Answer: (c) greenhouse gas emissions.

The National Mission for a Green India is one The Protocol arose when the World Resources
of the eight Missions under the National Action Institute (WRI) and World Business Council for

322 Climate Change: Causes and Implications-Explanation


unacademy.com | Download the Unacademy app
Give your feedback here: Link
Sustainable Development (WBCSD) recognized 15. Answer: (a)
the need for an international standard for Statement 1 is correct: The Green Climate
corporate Greenhouse Gas accounting and Fund (GCF) – a critical element of the historic
reporting in the late 1990s. Paris Agreement - is the world’s largest
climate fund, mandated to support developing
14. Answer: (b) countries raising and realizing their Nationally
Determined Contributions (NDC) ambitions
Statement 1 is not correct: The Paris Agreement towards low-emissions, climate-resilient
is a legally binding international treaty on pathways.
climate change. It was adopted by 196 Parties
at the Conference of Parties (COP 21) in Paris, y The Green Climate Fund was established
in December 2015 and entered into force on 4 by 194 countries in the UN Framework
November 2016. Convention on Climate Change in 2010. The
Fund pays particular attention to the needs
Statement 2 is correct: Its goal is to limit of societies that are highly vulnerable to
global warming to well below 2, preferably to the effects of climate change, in particular
1.5 degrees Celsius, compared to pre-industrial Least Developed Countries (LDCs), Small
levels. Island Developing States (SIDS), and the
The Paris Agreement is a landmark in the African States. The World Bank was invited
multilateral climate change process because, by the COP to serve as the interim trustee of
for the first time, a binding agreement brings the GCF, subject to review three years after
all nations into a common cause to undertake operationalization of the Fund. (Statement
ambitious efforts to combat climate change 2 is not correct)
and adapt to its effects. The Paris Agreement y GCF can structure its financial support
works on a 5- year cycle of increasingly through a flexible combination of grants,
ambitious climate action carried out by concessional debt, guarantees or equity
countries. By 2020, countries submit their instruments to leverage blended finance
plans for climate action known as nationally and crowd-in private investment for
determined contributions (NDCs). climate action in developing countries.
Statement 3 is not correct: Through the Cancun This flexibility enables the Fund to pilot
Agreements in 2010 developed country Parties new financial structures to support green
committed, in the context of meaningful market creation.
mitigation actions and transparency on
implementation, to a goal of mobilising jointly
USD 100 billion per year by 2020 to address the 16. Answer: (b)
needs of developing countries. Impact of the global temperature increases
beyond 30C above the pre-industrial level:
Elimination Technique: Developed
countries acknowledged their historical y Melting of ice and emission of carbon
responsibility in global warming and trapped beneath them will make a large
committed to donate $ 1000 billion a year carbon source instead of a sink. (Statement
from 2020 to help developing countries to 1 is correct)
cope with climate change. $1000 billion is y Global warming has affected the coral reefs
too extreme so, statement 3 is eliminated. which can lead to a loss of plant and animal
Option (b) is correct. lives. An increase in global temperatures
has made the fragility of coral reefs
even worse. Both aquatic and terrestrial
ecosystems will be affected. (Statement 2

Climate Change: Causes and Implications-Explanation 323


unacademy.com | Download the Unacademy app
Give your feedback here: Link
is correct) many times. The above-mentioned gases –
y Rising temperatures and acidity within our carbon dioxide and methane – are commonly
oceans are contributing to extreme coral known as greenhouse gases and are mainly
bleaching events, like the 2016 event that responsible for the greenhouse effect. An
destroyed more than one-third of the Great increase in the level of greenhouse gases has
Barrier Reef. led to considerable heating of the Earth leading
to global warming.
y Wetlands are drying up and shrinking around
the world. (Statement 3 is not correct)
y Cultivation of crops will face shifts in terms 18. Answer: (d)
of location e.g., wheat cultivation will A carbon credit is a tradable permit or
shift northwards. Also, productivity will be certificate that provides the holder of the credit
affected. (Statement 4 is not correct) the right to emit one ton of carbon dioxide or
y There are droughts at some places and an equivalent of another greenhouse gas – it
floods at some. This climatic imbalance is is essentially an offset for producers of such
the result of global warming. gases.

y Global warming leads to a change in the Option (d) is not correct: Carbon credit prices
patterns of heat and humidity. This has led are never fixed because they are traded on an
to the movement of mosquitoes that carry exchange. It is a permit that allows the company
and spread diseases. that holds it to emit a certain amount of carbon
dioxide or other greenhouse gases. One credit
y Due to an increase in floods, tsunamis and
permits the emission of a mass equal to one
other natural calamities, the average death
ton of carbon dioxide. Credit is one-half of a
toll usually increases. Also, such events can
so-called “cap-and-trade” program.
bring about the spread of diseases that can
hamper human life.
19. Answer: (b)

17. Answer: (d) Option (b) is correct: Phytoplankton is


responsible for most of the transfer of
Option (d) is correct: The increasing amount
carbon dioxide from the atmosphere to the
of carbon dioxide in the air is slowly raising
ocean. Carbon dioxide is consumed during
the temperature of the atmosphere because it
photosynthesis, and the carbon is incorporated
absorbs the infrared part of the solar radiation.
in the phytoplankton, just as carbon is stored
Clouds and gases reflect about one-fourth in the wood and leaves of a tree.
of the incoming solar radiation and absorb
y Most of the carbon is returned to near-
some of it but almost half of the incoming
surface waters when phytoplankton is
solar radiation falls on Earth’s surface heating
eaten or decomposed, but some fall into the
it, while a small proportion is reflected back.
ocean depths. Worldwide, this “biological
Earth’s surface re-emits heat in the form of
carbon pump” transfers about 10 gigatons
infrared radiation but part of this does not
of carbon from the atmosphere to the deep
escape into space as atmospheric gases
ocean each year.
(e.g., carbon dioxide, methane, etc.) absorb a
major fraction of it. The molecules of these y Even small changes in the growth of
gases radiate heat energy, and a major part phytoplankton may affect atmospheric
of which again comes to Earth’s surface, thus carbon dioxide concentrations, which would
heating it up once again. This cycle is repeated feed back to global surface temperatures.

324 Climate Change: Causes and Implications-Explanation


unacademy.com | Download the Unacademy app
Give your feedback here: Link
Environment, Sustainable
8 Development and General Issues
1. Consider the following: (2023) form of carbonated waters
1. Aerosols How many of the above activities are
2. Foam agents often considered and discussed for carbon
capture and sequestration?
3. Fire retardants
(a) Only one
4. Lubricants
(b) Only two
In the making of how many of the above are
hydrofluorocarbons used? (c) All three

(a) Only one (d) None

(b) Only two


(c) Only three 4. Which one of the following best describes
the term “greenwashing” ? (2022)
(d) All four
(a) Conveying a false impression that a
company’s products are eco-friendly
2. With reference to green hydrogen, consider and environmentally sound
the following statements: (2023) (b)
Non-inclusion of ecological/
1. It can be used directly as a fuel for environmental costs in the Annual
internal combustion. Financial Statements of a country
2. It can be blended with natural gas and (c)
Ignoring the disastrous ecological
used as fuel for heat or power generation consequences while undertaking
. infrastructure development
3. It can be used in the hydrogen fuel cell (d)
Making mandatory provisions for
to run vehicles. environmental costs in a government
How many of the above statements are project/programme
correct?
(a) Only one 5. According to India's National Policy on
(b) Only two Biofuels, which of the following can be
used as raw materials for the production
(c) All three
of biofuels? (2020)
(d) None
1. Cassava
2. Damaged wheat grains
3. Consider the following activities: (2023)
3. Groundnut seeds
1. Spreading finely ground basalt rock on
4. Horse grams
farmlands extensively
5. Rotten potatoes
2. Increasing the alkalinity of oceans by
adding lime 6. Sugar beet

3. Capturing carbon dioxide released by Select the correct answer using the code
various industries and pumping it into given below:
abandoned subterranean mines in the (a) 1, 2, 5 and 6 only

Environment, Sustainable Development and General Issues 325


unacademy.com | Download the Unacademy app
Give your feedback here: Link
(b) 1, 3, 4 and 6 only emissions.
(c) 2, 3, 4 and 5 only 2. H-CNG as fuel reduces carbon dioxide
(d) 1, 2, 3, 4, 5 and 6 and hydrocarbon emissions.
3. Hydrogen up to one -fifth by volume can
be blended with CNG as fuel for buses.
6. In rural road construction, the use of which
of the following is preferred for ensuring 4. H-CNG makes the fuel less expensive
environmental sustainability or to reduce than CNG.
carbon footprint? (2020) Which of the statements given above is/are
1. Copper slag correct?

2. Cold mix asphalt technology (a) 1 only

3. Geotextiles (b) 2 and 3 only

4. Hot mix asphalt technology (c) 4 only

5. Portland cement (d) 1, 2, 3 and 4

Select the correct answer using the code


given below: 9. It is possible to produce algae-based
(a) 1, 2 and 3 only biofuels, but what is/are the likely
limitation(s) of developing countries in
(b) 2, 3 and 4 only
promoting this industry? (2017)
(c) 4 and 5 only
1. Production of algae-based biofuels
(d) 1 and 5 only is possible in seas only and not on
continents.

7. Steel slag can be the material for which of 2. Setting up and engineering the algae-
the following? (2020) based biofuels production requires a
high level of expertise/ technology until
1. Construction of base road
the construction is complete.
2. Improvement of agricultural soil
3. Economically viable production
3. Production of cement necessitates the setting up of large-
Select the correct answer using the code scale facilities, which may raise
given below: ecological and social concerns.

(a) 1 and 2 only Select the correct answer using the code
given below:
(b) 2 and 3 only
(a) 1 and 2 only
(c) 1 and 3 only
(b) 2 and 3 only
(d) 1, 2 and 3
(c) 3 only
(d) 1, 2 and 3
8. In the context of proposals to the use of
hydrogen - enriched CNG (H-CNG) as fuel
for buses in public transport, consider the 10. Consider the following statements: (2016)
following statements: (2019)
1. The Sustainable Development Goals
1. The main advantage of the use of H-CNG were first proposed in 1972 by a global
is the elimination of carbon monoxide think tank called the ‘Club of Rome’.

326 Environment, Sustainable Development and General Issues


unacademy.com | Download the Unacademy app
Give your feedback here: Link
2. The Sustainable Development Goals (c) 1 and 3 only
have to be achieved by 2030. (d) 3 only
Which of the statements given above is/are
correct?
13. What is the Rio+20 Conference, often
(a) 1 only mentioned in the news? (2015)
(b) 2 only (a) It is the United Nations Conference on
(c) Both 1 and 2 Sustainable Development.
(d) Neither 1 nor 2 (b) It is a Ministerial Meeting of the World
Trade Organisation.

11. With reference to ‘Agenda 21’, sometimes (c)


It is a Conference of the
seen in the news, consider the following Intergovernmental Panel on Climate
statements: (2016) Change.

1. It is a global action plan for sustainable (d) It is a Conference of the Member


development. Countries of the Convention on
Biological Diversity.
2. It originated in the World Summit
on Sustainable Development held in
Johannesburg in 2002. 14. With reference to the usefulness of the
Which of the statements given above is/are by- products of the sugar industry, which
correct? of the following statements is/are correct?
(2013)
(a) 1 only
1. Bagasse can be used as biomass fuel
(b) 2 only
for the generation of energy.
(c) Both 1 and 2
2. Molasses can be used as one of the
(d) Neither 1 nor 2 feedstocks for the production of
synthetic chemical fertilisers.

12. With reference to ‘fly ash’ produced by the 3. Molasses can be used for the production
power plants using the coal as fuel, which of ethanol.
of the following statements is/are correct? Select the correct answer using the code
(2015) given below:
1. Fly ash can be used in the production of (a) 1 only
bricks for building construction.
(b) 2 and 3 only
2. Fly ash can be used as a replacement for
(c) 1 and 3 only
some of the Portland cement contents
of concrete. (d) 1, 2 and 3

3. Fly ash is made up of silicon dioxide


and calcium oxide only and does not 15. Biomass gasification is considered to be
contain any toxic elements. one of the sustainable solutions to the
Select the correct answer using the code power crisis in India. In this context which
given below: of the following statements is/are correct?
(2012)
(a) 1 and 2 only
1. Coconut shells, groundnut shells and
(b) 2 only
rice husk can be used in biomass

Environment, Sustainable Development and General Issues 327


unacademy.com | Download the Unacademy app
Give your feedback here: Link
gasification internal combustion engines.
2. The combustible gases generated Select the correct answer using the code
from biomass gasification consist of given below:
hydrogen and carbon dioxide only (a) 1 only
3. The combustible gases generated (b) 2 and 3 only
from biomass gasification can be used
for direct heat generation but not in (c) 1 and 3 only
(d) 1, 2 and 3

328 Environment, Sustainable Development and General Issues


unacademy.com | Download the Unacademy app
Give your feedback here: Link
8 Environment, Sustainable Development
and General Issues-Explanation

1. Answer: (d) deep geological formations is achieved by


Option (d) is correct: Hydrofluorocarbons pumping fluids down into a well. The part of
(HFC) are potent greenhouse gasses (GHG) the well in the storage zone is either perforated
that have global warming potentials that range or covered with a permeable screen to enable
from hundreds to thousands of times that the CO2 to enter the formation.
of carbon dioxide. HFCs are entirely man-
made and are primarily produced for use 4. Answer: (a)
in refrigeration, air-conditioning, insulating
Option (a) is correct: Greenwashing is the
foams and aerosol propellants, with minor
process of conveying a false impression or
uses as solvents and for fire protection. The
providing misleading information about how a
HFC-type hydraulic fluids are water-glycol
company’s products are more environmentally
hydraulic fluids. They contain around 40-45%
sound. Greenwashing is considered an
water. Condat offers CONDAT GUARD C46,
unsubstantiated claim to deceive consumers
with a formulation specially adapted to meet
into believing that a company’s products are
customer requirements in terms of lubrication,
environmentally friendly.
fire-resistance and biodegradability.

5. Answer: (a)
2. Answer: (c)
‘Biofuels’ are fuels produced from renewable
resources and used in place of or in blend with,
3. Answer: (c) diesel, petrol or other fossil fuels for transport,
Statement 1 is correct: Sprinkling powdered stationary, portable and other applications.
basalt over natural ecosystems would remove Option (a) is correct: Biofuels can be produced
vast amounts of carbon dioxide from the from biomass such as sugar-containing
Earth’s atmosphere while also improving soils. materials, like sugar cane, sugar beet, sweet
That’s the finding of a new study that evaluates sorghum, etc.; starch-containing materials
this proposed geoengineering scheme and such as corn, cassava, rotten potatoes, algae,
estimates the costs involved. etc.; and cellulosic-materials such as bagasse,
Statement 2 is correct: Ocean alkalinization is wood waste, agricultural and forestry residues,
an approach to carbon removal that involves or other renewable resources like an industrial
adding alkaline substances to seawater to waste; rice & wheat straw/corn cobs (Damaged
enhance the ocean’s natural carbon sink. These wheat grains).
substances could include minerals, such as
olivine, or artificial substances, such as lime or
6. Answer: (a)
some industrial byproducts. Adding alkalinity
to the ocean removes carbon dioxide from the In order to encourage locally available
atmosphere through a series of reactions that materials and the use of green technologies
convert dissolved CO2 into stable bicarbonate for the construction of roads under Pradhan
and carbonate molecules, which in turn causes Mantri Gram Sadak Yojana (PMGSY), guidelines
the ocean to absorb more CO2 from the air to were issued by the Ministry, wherein the
restore equilibrium. State Governments are required to propose
a minimum 15% of the total length of annual
Statement 3 is correct: Injection of fluids into

Environment, Sustainable Development and General Issues-Explanation 329


unacademy.com | Download the Unacademy app
Give your feedback here: Link
proposals under new technologies such as The slag occurs as a molten liquid melt and is
Cement stabilisation, Lime stabilisation, a complex solution of silicates and oxides that
Cold mix, Waste plastics, Cell filled concrete, solidifies upon cooling.
Panelled cement concrete pavement, Fly ash, Statement 1 is correct: Steel slags can be used
etc. in the construction of base roads in order to
Statement 1 is correct: The use of copper slag replace natural aggregates, which reduces the
in cement and concrete provides potential environmental impact.
environmental as well as economic benefits Statement 2 is correct: Steel slag can also
for all related industries, particularly in areas be used in the agricultural sector due to its
where a considerable amount of copper slag ability to correct soil acidity, as it contains
is produced. some nutrients for the plants and also as
Statement 2 is correct: Cold asphalt mix silicate fertiliser that is capable of providing
is produced by mixing unheated mineral silicon to the plants. Hence, it is a factor in the
aggregate with either emulsified bitumen or improvement of agricultural soil.
foamed bitumen. cold asphalt mix does not Statement 3 is correct: Steel slag used in
require any heating of aggregate which makes cement and concrete production as aggregate
it economical and relatively pollution-free (no or as additional cementing materials has a
objectionable fumes or odours). significant positive impact on the environment
Statement 3 is correct: Geotextiles are due to its long-term strength and durability.
commonly used to improve soils over which
roads, embankments, pipelines, and earth
retaining structures are built. There are several 8. Answer: (b)
types of geotextile material, including open- CNG is compressed natural gas. With natural gas
mesh, warp-knitted, and closed fabric, or non- mainly composed of methane, CNG emits less
woven textiles. It is preferred for ensuring air pollutants such as carbon dioxide, carbon
environmental sustainability or to reduce monoxide, nitrogen oxides and particulate
carbon footprint. matter than petrol or diesel.
Statement 4 is not correct: Hot mix asphalt Statement 1 is not correct: H-CNG is a blend
(HMA), requires heating of aggregate which is of hydrogen and CNG, the ideal hydrogen
prone to pollution. concentration being 18%. Compared to
Statement 5 is not correct: Portland Cement conventional CNG, the use of H-CNG can
is an important construction ingredient around reduce the emission of carbon monoxide
the world, and as a result, cement production up to 70%, (elimination of carbon monoxide
is a significant source of global carbon dioxide emissions does not happen by using H-CNG)
(CO2) emissions. besides enabling up to 5% savings in fuel.
Statement 2 is correct: H-CNG as fuel also
Elimination Technique: Cold mix asphalt reduces carbon dioxide and hydrocarbon
technology is relatively pollution-free emissions.
while Hot mix asphalt technology is prone
Statement 3 is correct: Worldwide Hydrogen is
to pollution. So, Option (a) is correct.
being blended (20-30%) with natural gas and
then compressed to dispense into vehicles.
7. Answer: (d) The US, Brazil, Canada, and South Korea have
Steel slag, a by-product of steel making, is all conducted trials and found that they get
produced during the separation of the molten a reduction in emissions from buses using
steel from impurities in steel-making furnaces. H-CNG.

330 Environment, Sustainable Development and General Issues-Explanation


unacademy.com | Download the Unacademy app
Give your feedback here: Link
Statement 4 is not correct: Physical blending of examples producing only bioenergy. Energy
CNG and hydrogen involves a series of energy- and biofuels tend to have a low value. Algae
intensive steps that would make H-CNG more cultivation and processing systems require a
expensive than CNG. high capital input (higher than agriculture). For
Additional Information: developing countries, large-scale facilities are
more economically viable but are also more
Compact Reforming Process’ directly produces likely to have higher social and ecological
a Hydrogen-CNG mixture from natural gas, impacts.
using a single step. The cost of production is
significantly lower than physical blending.
10. Answer: (b)

9. Answer: (b) The ‘Club of Rome’ is a global think tank that


deals with a variety of international political
Algae are organisms that grow in aquatic issues. It was founded in 1968 at Accademia
environments and use light and carbon dei Lincei in Rome, Italy. In 1972, the Club’s
dioxide (CO2) to create biomass. Microalgae first major Report, the Limits to Growth was
are measured in micrometers and are tiny, published.
unicellular algae that normally grow in
suspension within a body of water. Microalgae The notion of sustainable development
have long been recognized as potentially good emerged in the wake of a general rise in the
sources for biofuel production because of their awareness of environmental issues in the
relatively high oil content and rapid biomass late 1960s in the Western World. It reflected
production. Microalgae grow very quickly the concern of people about the undesirable
compared to terrestrial crops; the practice of effects of industrial development on the
algal mass culture can be performed on non- environment.
arable lands using non-potable saline water Statement 1 is not correct: Concerned with
and wastewater. the growing opinion of the world community
Statement 1 is not correct: Algae-based on environmental issues, the United
biofuels are suitable for many land and water Nations established a World Commission on
types, with widely varying opportunities and Environment and Development (WCED) headed
restrictions. They can grow on marginal or by Norwegian Prime Minister Gro Harlem
non-cropland and also on brackish or polluted Brundtland. The Commission gave its report
water. Hence, the production of algae-based (also known as Brundtland Report) entitled ‘Our
biofuels is possible in seas as well as on Common Future’ in 1987. The report defines
continents. sustainable development as a “development
that meets the needs of the present without
Statement 2 is correct: Setting up and compromising the ability of future generations
engineering the algae-based biofuel production to meet their own needs.”
requires a high level of expertise/ technology
until the construction is complete. Innovation Statement 2 is correct: The Sustainable
for higher productivity also requires some Development Goals (SDGs), also known as
knowledge and/or experience. Operation and the Global Goals, were adopted by all United
maintenance, as well as processing, can be Nations Member States in 2015 as a universal
mostly done without specific educational call to action to end poverty, protect the
requirements. planet and ensure that all people enjoy peace
and prosperity by 2030. In 2015, world leaders
Statement 3 is correct: The economic viability agreed to 17 Global Goals.
of algae-based biofuels is still in its nascent
phase. There are no commercial-scale

Environment, Sustainable Development and General Issues-Explanation 331


unacademy.com | Download the Unacademy app
Give your feedback here: Link
11. Answer: (a) Environment and Development, Agenda
Statement 1 is correct: Agenda 21 is an action 21 and Forest Principles. Moreover, two
plan of the United Nations (UN) related to important legally binding agreements
sustainable development. It is a comprehensive Convention on Biological Diversity and
plan of action to be taken globally, nationally Framework Convention on Climate Change
and locally by organisations of the United (UNFCCC) were also adopted here.
Nations System, Governments, and Major
Groups in every area in which human impacts 12. Answer: (a)
on the environment.
Fly ash is a by-product of coal after it is burnt
Statement 2 is not correct: It was an at thermal power plants, and is usually dumped
outcome of the United Nations Conference on at the site or ponds, resulting in depletion of
Environment and Development (UNCED) held topsoil, groundwater deterioration and air
in Rio de Janeiro, Brazil, in 1992. pollution.
The number 21 refers to an agenda for the y Fly ash is used in making bricks and
21st century. It is a comprehensive blueprint concrete. Fly Ash Bricks can be used as
of action to be taken globally, nationally, and an alternative to ordinary burnt clay bricks
locally by organisations of the UN, governments, with equivalent strength. (Statement 1 is
and major groups in every area in which humans correct)
directly affect the environment.
y As a part of the concrete Fly ash saves water
Additional Information: it lowers the heat of hydration; reduces the
y United Nations Conference on Environment corrosion of steel and makes the concrete
and Development (UNCED), also known as strong quickly. Due to this fly ash reaches
the Rio Summit, Rio Conference, or Earth its maximum strength more slowly than
Summit held in Rio de Janeiro in June 1992. concrete made with only Portland cement.
y The Earth Summit resulted in the (Statement 2 is correct)
documents related to Rio Declaration on y Fly Ash and is made of Silica, Aluminum

332 Environment, Sustainable Development and General Issues-Explanation


unacademy.com | Download the Unacademy app
Give your feedback here: Link
oxide, calcium oxide and some potentially boards, cattle feed, etc. Bagasse can be used
toxic elements such as arsenic, beryllium, as biomass fuel for the generation of energy.
cadmium, barium, chromium, copper, lead, Statement 2 is not correct: The main products
mercury, molybdenum, nickel, radium, that can be produced from molasses on a
selenium, thorium, uranium, vanadium, and commercial scale are ethyl alcohol, citric acid,
zinc, etc. (Statement 3 is not correct) lactic acid, cattle feed, oxalic acid, baker’s
Elimination Technique: Fly ash is made up yeast, monosodium glutamate, torula yeast,
of silicon dioxide and calcium oxide only. lysine, acetone-butanol-alcohol. In addition
Statement 3 is eliminated. Statement 1 is to this, many alcohol-based chemicals like
clearly correct. Thus, Option (b) is correct. acetic acid, acetic anhydride, acetone, ethyl
acetate, ethyl benzene, styrene, polystyrene,
poly-ethylene and synthetic rubber are also
13 Answer: (a) produced using cane molasses. It is also used
Option (a) is correct: Rio+20 was one of the for the production of organic fertiliser.
biggest international gatherings of 2012 and Statement 3 is correct: Sugarcane ethanol
the largest event in the history of the United is an alcohol-based fuel produced by the
Nations. It presented an opportunity to redirect fermentation of sugarcane juice and molasses.
and re-energise political commitment to the Because it is a clean, affordable and low-carbon
three dimensions of Sustainable Development: biofuel, sugarcane ethanol has emerged as a
economic growth, social improvement, and leading renewable fuel for the transportation
environmental protection. sector. In India, Bioethanol can be produced
The United Nations Conference on Sustainable from multiple sources like sugar-containing
Development (Rio+20) was held in Rio de materials, starch-containing materials,
Janeiro, Brazil, from 20th to 22nd June 2012. cellulose and lignocellulose material including
Rio+20 focused on two themes: petrochemical route. However, the present
policy of the Ethanol Blended Petrol (EBP)
y A Green economy in the context of
Programme allows bioethanol to be procured
sustainable development and poverty
from non-food feedstock like molasses,
eradication.
cellulose and lignocellulose material including
y The institutional framework for sustainable the petrochemical route. Similarly, biodiesel
development. can be produced from any edible/non-edible
oil.

14. Answer: (c) Elimination Technique: Molasses being


Sugarcane today is considered as one of the can be organic, it will be used as one of the
best converters of solar energy into biomass feedstocks for the production of organic
and sugar. It is a rich source of food (sucrose, fertilisers. Statement 2 is eliminated.
jaggery and syrup), fiber (cellulose), fodder Statement 3 is clearly correct. Thus,
(green leaves and tops of cane plant, bagasse, Option (c) is correct.
and molasses and to some extent press mud),
fuel and chemical. The main by-products are
15. Answer: (a)
bagasse, molasses and press mud.
Biomass gasification is a mature technology
Statement 1 is correct: Bagasse is a fibrous
pathway that uses a controlled process
residue left over after the sugarcane is crushed.
involving heat, steam, and oxygen to convert
The fiber content of bagasse (cellulose) is used
biomass to hydrogen and other products,
in cellulose industries like pulp, paper, particle
without combustion.

Environment, Sustainable Development and General Issues-Explanation 333


unacademy.com | Download the Unacademy app
Give your feedback here: Link
Statement 1 is correct: Biomass, a renewable carbon dioxide (CO2).
organic resource, includes agriculture crop Statement 3 is not correct: Syngas, or synthesis
residues (such as corn stover or wheat gas, is a fuel gas mixture consisting primarily
straw), forest residues, special crops grown of hydrogen, carbon monoxide, and very often
specifically for energy use (such as switchgrass some carbon dioxide. The name comes from
or willow trees), organic municipal solid waste, its use as intermediate in creating synthetic
and animal wastes. This renewable resource natural gas (SNG) and for producing ammonia
can be used to produce hydrogen, along or methanol. Syngas is usually a product of
with other byproducts, by gasification. When coal gasification, and its main application is
Biomass fields are developed coconut shells, electricity generation. Syngas is combustible
Ground nut shells, Sugar cane bagasse, and and can be used as a fuel of internal combustion
Rice husk, can be used as fuels. The output engines.
will be Producer-gas directly used for heating
applications. Elimination Technique: The combustible
Statement 2 is not correct: Biomass gasification gasses generated from biomass gasification
is a thermal process that converts organic can be used for direct heat generation
carbonaceous materials (such as wood waste, and also in internal combustion engines.
shells, pellets, agricultural waste, and energy Statement 3 is not correct. Thus, Option
crops) into a combustible gas comprised of (a) is correct.
carbon monoxide (CO), hydrogen (H) and

334 Environment, Sustainable Development and General Issues-Explanation


unacademy.com | Download the Unacademy app
Give your feedback here: Link
9 Agriculture

1. Consider the following trees : (2023) (a) 1 and 2 only


1. Jackfruit (Artocarpus heterophyllus) (b) 2 and 3 only
2. Mahua (Madhuca indica) (c) 1 and 3 only
3. Teak (Tectona grandis) (d) 1, 2 and 3
How many of the above are deciduous
trees? 4. How is permaculture farming different from
(a) Only one conventional chemical farming? (2021)
(b) Only two 1. Permaculture farming discourages
(c) All three monocultural practices but in
conventional chemical farming,
(d) None
monoculture practices are predominant.
2. Conventional chemical farming can
2. With reference to the role of biofilters cause an increase in soil salinity but the
in Recirculating Aquaculture System, occurrence of such phenomenon is not
consider the following statements: (2023) observed in permaculture farming.
1. Biofilters provide waste treatment by 3. Conventional chemical farming is
removing uneaten fish feed. easily possible in semi-arid regions but
2. Biofilters convert ammonia present in permaculture farming is not so easily
fish waste to nitrate. possible in such regions.

3. Biofilters increase phosphorus as a 4. Practice of mulching is very important


nutrient for fish in water. in permaculture farming but not
necessarily so in conventional chemical
How many of the statements given above
farming.
are correct?
Select the correct answer using the code
(a) Only one
given below.
(b) Only two
(a) 1 and 3
(c) All three
(b) 1, 2 and 4
(d) None
(c) 4 only
(d) 2 and 3
3. “System of Rice Intensification” of
cultivation, in which alternate wetting and
drying of rice fields is practised, results in 5. With reference to ‘palm oil’, consider the
(2022) following statements: (2021)

1. Reduced seed requirement 1. The palm oil tree is native to Southeast


Asia.
2. Reduced methane production
2. The palm oil is a raw material for
3. Reduced electricity consumption
some industries producing lipstick and
Select the correct answer using the code perfumes.
given below :

Agriculture 335
unacademy.com | Download the Unacademy app
Give your feedback here: Link
3. The palm oil can be used to produce 8. What is the use of biochar in farming?
biodiesel. (2020)
Which of the statements given above are 1. Biochar can be used as a part of the
correct? growing medium in vertical farming.
(a) 1 and 2 only 2. When biochar is a part of the growing
(b) 2 and 3 only medium, it promotes the growth of
nitrogen-fixing microorganisms.
(c) 1 and 3 only
3. When biochar is a part of the growing
(d) 1, 2 and 3 medium, it enables the growing medium
to retain water for a longer time.
6. Among the following, which one is the least Which of the statements given above is/are
water-efficient crop? (2021) correct?
(a) Sugarcane (a) 1 and 2 only
(b) Sunflower (b) 2 only
(c) Pearl millet (c) 1 and 3 only
(d) Red gram (d) 1, 2 and 3

7. In the context of India's preparation for 9. In the context of India, which of the
Climate-Smart Agriculture, consider the following is/are considered to be practice(s)
following statements: (2021) of eco-friendly agriculture? (2020)
1. The 'Climate-Smart Village' approach 1. Crop diversification
in India is a part of a project led by 2. Legume intensification
the Climate Change, Agriculture and
Food Security (CCAFS), an international 3. Tensiometer use
research programme. 4. Vertical farming
2. The project of CCAFS is carried Select the correct answer using the code
out under Consultative Group on given below:
International Agricultural Research (a) 1, 2 and 3 only
(CGLAR) headquartered in France.
(b) 3 only
3. The International Crops Research
Institute for the Semi-Arid Tropics (c) 4 only
(ICRISAT) in India is one of the CGIAR's (d) 1, 2, 3 and 4
research centres.
Which of the statements given above are 10. What is/are the advantage/advantages of
correct? zero tillage in agriculture? (2020)
(a) 1 and 2 only 1. Sowing of wheat is possible without
(b) 2 and 3 only burning the residue of the previous
(c) 1 and 3 only crop.

(d) 1, 2 and 3 2. Without the need for a nursery of rice


saplings, direct planting of paddy seeds
in the wet soil is possible.
3. Carbon sequestration in the soil is

336 Agriculture
unacademy.com | Download the Unacademy app
Give your feedback here: Link
possible. (c) 1 and 3 only
Select the correct answer using the code (d) 1, 2 and 3
given below:
(a) 1 and 2 only 13.
With reference to the circumstances
(b) 2 and 3 only in Indian agriculture, the concept of
(c) 3 only "Conservation Agriculture" assumes
significance. Which of the following fall
(d) 1, 2 and 3 under the Conservation Agriculture? (2018)
1. Avoiding the monoculture practices
11. What are the advantages of fertigation in 2. Adopting minimum tillage
agriculture? (2020)
3. Avoiding the cultivation of plantation
1. Controlling the alkalinity of irrigation crops
water is possible.
4. Using crop residues to cover soil surface
2. Efficient application of Rock Phosphate
and all other phosphatic fertilizers is 5. Adopting spatial and temporal crop
possible. sequencing/crop rotations

3. Increased availability of nutrients to Select the correct answer using the code
plants is possible. given below:

4. Reduction in the leaching of chemical (a) 1, 3 and 4 only


nutrients is possible. (b) 2, 3, 4 and 5 only
Select the correct answer using the code (c) 2, 4 and 5 only
given below: (d) 1, 2, 3 and 5 only
(a) 1, 2 and 3
(b) 1, 2 and 4 only 14. Which of the following statements can
(c) 1, 3 and 4 only help in water conservation in agriculture?
(d) 2, 3 and 4 only (2017)
1. Reduced or zero tillage of the land

12.
With reference to agricultural soils, 2. Applying gypsum before irrigating the
consider the following statements: (2018) field

1. A high content of organic matter in soil 3. Allowing crop residue to remain in the
drastically reduces its water holding field
capacity. Select the correct answer using the code
2. Soil does not play any role in the sulphur given below:
cycle. (a) 1 and 2 only
3. Irrigation over a period of time can (b) 3 only
contribute to the salinization of some (c) 1 and 3 only
agricultural lands.
(d) 1, 2 and 3
Which of the statements given above is/are
correct?
(a) 1 and 2 only 15. What can be the impact of excessive/
inappropriate use of nitrogenous fertilisers
(b) 3 only

Agriculture 337
unacademy.com | Download the Unacademy app
Give your feedback here: Link
in agriculture? (2015) 3. Spirogyra
1. Proliferation of nitrogen fixing Which of the above is/are used as
microorganisms in soil can occur. biofertilizer/biofertilizers?
2. Increase in the acidity of soil can take (a) 1 and 2 only
place. (b) 2 only
3. Leaching of nitrate to the groundwater (c) 2 and 3 only
can occur.
(d) 3 only
Select the correct answer using the code
given below:
(a) 1 and 3 only 18. Consider the following kinds of organisms:
(2012)
(b) 2 only
1. Bacteria
(c) 2 and 3 only
2. Fungi
(d) 1, 2 and 3
3. Flowering plants
Some species of which of the above kinds of
16. What is/are the significance of a practical organisms are employed as biopesticides?
approach to sugarcane production known
as 'Sustainable Sugarcane Initiative’? (a) 1 only
(2014) (b) 2 and 3 only
1. Seed cost is very low in this compared to (c) 1 and 3 only
the conventional method of cultivation. (d) 1, 2 and 3
2. Drip irrigation can be practised very
effectively in this.
19.
Consider the following agricultural
3. There is no application of chemical/ practices: (2012)
inorganic fertilisers at all in this.
1. Contour bunding
4. The scope for intercropping is more
2. Relay cropping
in this compared to the conventional
method of cultivation. 3. Zero tillage
Select the correct answer using the code In the context of global climate change,
given below: which of the above helps/help in carbon
sequestration/ storage in the soil?
(a) 1 and 3 only
(a) 1 and 2 only
(b) 1, 2 and 4 only
(b) 3 only
(c) 2, 3 and 4 only
(c) 1, 2 and 3
(d) 1, 2, 3 and 4
(d) None of them

17. Consider the following organisms: (2013)


1. Agaricus
2. Nostoc

338 Agriculture
unacademy.com | Download the Unacademy app
Give your feedback here: Link
9 Agriculture-Explanation

1. Answer: (b) impact water quality if not properly managed.


1. Jackfruit(Artocarpus heterophyllus) - It is Biofilters facilitate the nitrification process,
found in Wet evergreen forests that are located which involves two key steps performed by
in the south along the Western Ghats and the specific groups of beneficial bacteria. The first
Nicobar and Andaman Islands and all along the group consists of ammonia-oxidising bacteria
north-eastern region. It is characterized by tall, (AOB), which convert ammonia into nitrite
straight evergreen trees that have a buttressed (NO2-). The second group includes nitrite-
trunk or root on three sides like a tripod that oxidising bacteria (NOB), which further convert
helps to keep a tree upright during a storm. nitrite into nitrate (NO3-).
These trees often rise to a great height before Statement 3 is incorrect: Phosphorus is an
they open out like a cauliflower. The more essential nutrient for fish and other aquatic
common trees that are found here are the organisms, but it is not directly influenced by
jackfruit, betel nut palm, jamun etc. the biofiltration process. Phosphorus levels
2. Mahua(Madhuca indica) - The Mahua tree in water are typically managed through other
(Madhuca indica) belongs to the Sapotaceae means, such as water quality monitoring,
family.It thrives well in deep clay soil and grows nutrient management, and appropriate fish
quickly with a height reaching up to 20 m with feed formulations.
high adaptability to arid environments.
**The biodiesel obtained from mahua oil has 3. Answer: (d)
properties similar to the conventional diesel Option (d) is correct: The System of Rice
of American and European standards. Under Intensification involves cultivating rice with
optimized conditions, about 98% conversion as much organic manure as possible, starting
of mahua oil to biodiesel is possible. with young seedlings planted singly at a
3. Teak (Tectona grandis) - Teak (Tectona wider spacing in a square pattern; and with
grandis) is a tropical hardwood tree species in intermittent irrigation that keeps the soil
the family Lamiaceae. It is a large, deciduous moist but not inundated, and frequent inter
tree that occurs in mixed hardwood forests. cultivation with weeder that actively aerates
the soil.

2. Answer: (b) In SRI paddy cultivation, less quantity of seeds


- 2 kg/acre is required compared to 20 kg/acre
Statement 1 is correct: Biofilters are an in conventional cultivation.
essential part of an aquaponics system,used in
waste treatment systems to remove uneaten By stopping flooding and relying mostly on
fish feed and other organic waste from aquatic organic fertilization, methane emissions are
environments. Biofiltration is a natural and greatly reduced
effective method of water treatment commonly SRI Paddy Cultivation requires less water. So,
employed in aquaculture systems, such as fish less electricity consumption.
farms or aquariums.
Statement 2 is correct: In fish waste and 4. Answer: (b)
uneaten feed, one of the primary waste products
Option (b) is correct: The concept is not new. It
is ammonia (NH3). Ammonia is toxic to fish and
was first propagated in the 1970s by Australian
other aquatic organisms and can negatively

Agriculture-Explanation 339
unacademy.com | Download the Unacademy app
Give your feedback here: Link
biologist Bill Mollison. It gained acceptance in
India after several enthusiasts were influenced Statement 1 is clearly correct. So, Option
by Mollison during his visit to the country in (b) is correct.
1987. Permaculture is the conscious design
and maintenance of agriculturally productive
5. Answer: (b)
ecosystems which have the diversity, stability,
and resilience of natural ecosystems. It is Statement 1 is not correct: Palm is an African
the harmonious integration of landscape and tree, cultivated as a source of oil. The oil
people — providing their food, energy, shelter, palm is grown extensively in its native West
and other material and non-material needs in and Central Africa, as well as in Malaysia and
a sustainable way. Indonesia. It reached global markets with the
Atlantic slave trade five centuries ago.
Statement 1 is correct: Permaculture integrates
patterns in ecosystems to replicate biodiversity Statement 2 is correct: Palm oil is cheap and
into agriculture to minimalize human energy versatile. It is found in everything from lipstick
intervention. Monoculture efficiently produces to ice cream. For Indians, though, its primary
a single crop or livestock type grown in a specific use has been as a cooking oil. Palm oil, obtained
area, relying heavily on human involvement and from fruits, is used in making soaps, cosmetics
energy-intensive machinery. So, Permaculture (lipstick and perfumes), candles, biofuels, and
farming discourages monocultural practices. lubricating greases and in processing tinplate
Whereas in conventional chemical farming, and coating iron plates.
monoculture practices are predominant. Statement 3 is correct: Edible vegetable oils
Statement 2 is correct: In Conventional such as soybean oil, palm oil and sunflower
chemical farming, soil salinity increases due oil are first-generation biodiesel feedstocks
to an increase in salts in the soil. Soil salinity, because they were the first type of crop used
due to intense chemical use, demands at to produce biodiesel. Palm biodiesel, a type
least four times more water. Chemical runoffs of liquid bioenergy fuel (biofuel), is gaining
destroy not just marine ecosystems but life prominence as renewable energy that can
forms in the soil, which makes soil sterile. In either be used together with petrol or diesel in
permaculture, the whole is greater than the any proportion or as a full substitute for diesel
sum of its parts. So, the occurrence of soil engines without modification in transport and
salinity is not observed in permaculture. industrial applications.

Statement 3 is not correct: Permaculture is Elimination Technique: The palm oil


suitable in semi-arid regions using specific tree is native to West and Central Africa.
crops and water harvesting structures. Statement 1 is eliminated. Thus, Option (b)
Statement 4 is correct: Mulching basically is correct.
involves the application of a protective layer
of material to the field soil surface just after
6. Answer: (a)
sowing any crop. Mulching is an essential
component of ecologically rich permaculture. Option (a) is correct: Sugarcane is one of
Though it is not important in conventional the most water-intensive crops. Crop water
chemical farming. needs (mm/total growing period): Sugarcane:
1500-2500, Sunflower: 600-1000, Millet: 450-
Elimination Technique: Conventional 650, Gram (Chickpea): 350-500. So, Sugarcane
chemical farming requires high water requires a higher amount of water than other
availability which is not available in semi- crops. So, Sugarcane is the least water-
arid regions. Statement 3 is eliminated. efficient crop.

340 Agriculture-Explanation
unacademy.com | Download the Unacademy app
Give your feedback here: Link
7. Answer: (d) y It can be used as a part of the growing
Climate-smart agriculture (CSA) is an approach medium in vertical farming. (Statement 1 is
that helps guide actions to transform agri- correct)
food systems towards green and climate- y When biochar is a part of the growing
resilient practices. CSA supports reaching medium, it promotes the growth of nitrogen-
internationally agreed goals such as the SDGs fixing microorganisms. (Statement 2 is
and the Paris Agreement. It aims to tackle correct)
three main objectives: sustainably increasing y Biochar holds the potential for long-term
agricultural productivity and incomes; adapting carbon sequestration, improving soil health
and building resilience to climate change, and and water-holding capacity, and further
reducing and/or removing greenhouse gas reducing emissions of greenhouse gases
emissions. associated with fertiliser application.
Statement 1 is correct: The ‘Climate-Smart (Statement 3 is correct)
Village’ approach in India is a part of a project Adding biochar to soil increases its carbon
led by the Climate Change, Agriculture and content and can mitigate greenhouse gas
Food Security (CCAFS), an international emissions. This mitigation can occur by several
research programme. This program provides means, long-term transfer of carbon into
technological, organizational and systemic biochar, which would otherwise decompose
support to farmers in association with naturally and emit carbon dioxide and methane,
institutions to help them cope with climate production of syngas andbio–oil, which can
change. be used as energy alternatives to fossil fuels,
Statement 2 is correct: The project of CCAFS and reduced emissions of nitrous oxide from
is carried out under the Consultative Group on fertiliser application.
International Agricultural Research (CGIAR),
headquartered in France. The CGIAR Research
Program on Climate Change, Agriculture and 9. Answer: (a)
Food Security (CCAFS) marshals the science Statement 1 is correct: Crop diversification
and expertise of CGIAR and its partners to refers to the addition of new crops or cropping
catalyse positive change for climate-smart systems to agricultural production on a
agriculture (CSA). particular farm considering the different returns
Statement 3 is correct: The International from value-added crops with complementary
Crops Research Institute for the Semi-Arid marketing opportunities. It is an eco-friendly
Tropics (ICRISAT) in India is one of the CGIAR’s practice.
research centers. ICRISAT is headquartered in Statement 2 is correct: Improved varieties of
Patancheru near Hyderabad, Telangana, India. legumes adapted to the nutrient deficiency
have the potential to improve food security
for the poorest farmers. Tolerant varieties
8. Answer: (d) could be an inexpensive and biologically
Biochar is a charcoal-like product that is the smart technology that improves soils while
residue of a renewable energy production minimising fertiliser costs.
process called slow pyrolysis. Products like Statement 3 is correct: A tensiometer is
paper mill waste, green waste, animal manure a device for measuring soil water tension.
or other biomass can be recycled by heating to Tensiometers have five applications: They are
550 degrees Celsius in the absence of oxygen. used to determine rooting depth. They are
The process generates energy and biochar. used for the timing of field irrigation. They are
Usages of biochar: used to determine the timing of greenhouse

Agriculture-Explanation 341
unacademy.com | Download the Unacademy app
Give your feedback here: Link
irrigations for potted plants and greenhouse reduced due to the presence of mulch.
beds. The water table level is determined
using tensiometers. The hydraulic gradient is
determined from measurements using two 11. Answer: (c)
tensiometers. It is an eco-friendly practice. Fertigation is a method of fertilizer application
Statement 4 is not correct: In vertical farming, in which fertilizer is incorporated within the
crops are grown indoors, under artificial irrigation water by the drip system. In this
conditions of light and temperature. It aims system, fertilizer solution is distributed evenly
at higher productivity in smaller spaces. It in irrigation.
uses soil-less methods such as hydroponics, Statement 1 is correct: Drip irrigation also
aquaponics and aeroponics. Vertical farming avoids water spillage on the field which could
uses significantly less water and pesticides have promoted weed growth or increased soil
than traditional agricultural methods. Being alkalinity due to water logging. So, controlling
indoors, the crops are not subject to seasons the alkalinity of irrigation water is possible
and hence give high productivity year-round. through it.
Lettuces, tomatoes, and green crops can be Statement 2 is not correct: Fertigation is the
produced through this practice. technique of supplying dissolved fertilizers
The biggest downside of vertical farms is the to crops through an irrigation system. But
large amount of energy it takes to grow crops: phosphatic fertilizers are insoluble in water
between 30-176 kWh per kg more than the and thus cannot be applied to the field by
greenhouses method of crop production. fertigation. Hence, efficient application of Rock
Phosphate and all other phosphatic fertilizers
is not possible.
10. Answer: (d)
Statement 3 is correct: By this method, fertilizer
In Zero tillage (No tillage), a new crop is planted use efficiency is increased from 80 to 90
in the residues of the previous crop without any percent as they are delivered directly to roots
prior soil tillage or seed bed preparation, and it via a drip. As water and fertilizer are supplied
is possible when all the weeds are controlled evenly to all the crops through fertigation there
by the use of herbicides. is the possibility of getting 25-50 percent higher
Advantages of zero tillage in agriculture: yield. So, increased availability of nutrients to
y In this method, the sowing of wheat is done plants is possible.
by a special machine (zero tillage machine) Statement 4 is correct: Leaching Process
without preparing the field. Sowing of wheat describes the release of organic and inorganic
is possible without burning the residue of contaminants or radionuclides from a solid
the previous crop. (Statement 1 is correct) phase into a water phase when influenced by
y In this method, there is no need for a processes such as desorption, complexation,
nursery of rice saplings, direct planting of and mineral dissolution. This is correct since
paddy seeds in the wet soil is possible. overapplication, and soil waterlogging is
(Statement 2 is correct) avoided. Therefore, a reduction in the leaching
of chemical nutrients is possible.
y Zero tillage helps in carbon sequestration
in the soil. (Statement 3 is correct)
y Zero-tilled soils are homogenous in structure 12. Answer: (b)
with a greater number of earthworms, Organic matter affects both the chemical and
organic matter content increases due to physical properties of the soil and its overall
less mineralization, and surface runoff is health. Properties influenced by organic

342 Agriculture-Explanation
unacademy.com | Download the Unacademy app
Give your feedback here: Link
matter include soil structure; moisture holding bacterial waste products, organic gels, fungal
capacity; diversity and activity of soil organisms, hyphae and worm secretions and casts.
both those that are beneficial and harmful to Moreover, organic matter intimately mixed
crop production; and nutrient availability. with mineral soil materials has a considerable
Statement 1 is not correct: Organic matter influence in increasing water holding capacity.
contributes to the stability of soil aggregates Especially in the topsoil, where the organic
and pores through the bonding or adhesion matter content is greater, more water can be
properties of organic materials, such as stored.

Statement 2 is not correct: Sulphur is one The salt lodged in the lower profiles of the
of three nutrients that are cycled between soil comes up to the surface and destroys its
the soil, plant matter and the atmosphere. fertility. Chemical fertilisers in the absence of
The Sulphur cycle describes the movement organic manures are also harmful to the soil.
of Sulphur through the atmosphere, mineral Unless the soil gets enough humus, chemicals
and organic forms, and through living things. harden it and reduce its fertility in the long
Although Sulphur is primarily found in run. This problem is common in all the com-
sedimentary rocks, it is particularly important mand areas of the river valley projects, which
to living things because it is a component of were the first beneficiaries of the Green Rev-
many proteins. Hence, soil plays a role in the olution. According to estimates, about half of
sulphur cycle. the total land of India is under some degree of
Statement 3 is correct: A fairly large area of degradation.
arable land in the irrigated zones of India is
becoming saline because of overirrigation. Elimination Technique: A high content of

Agriculture-Explanation 343
unacademy.com | Download the Unacademy app
Give your feedback here: Link
slow the flow of water, increasing the time
organic matter in soil drastically increases during which infiltration can take place.
its water-holding capacity. Statement 1 is
Statement 2 is not correct: An infiltration
eliminated. Thus, Option (b) is correct.
problem often occurs in farms if the irrigation
water does not enter the soil rapidly enough
13. Answer: (c) during a normal irrigation cycle. Gypsum helps
in the dissolution of the salt clay pan found
Option (c) is correct: Conservation Agriculture
in soils that hinder the percolation of water.
is a set of soil management practices that
This increases the efficiency of irrigation as
minimise the disruption of the soil’s structure,
less water is needed to irrigate the same soil.
composition, and natural biodiversity. Despite
Adding gypsum to irrigation water further
high variability in the types of crops grown
increases its infiltration rate.
and specific management regimes, all forms
of conservation agriculture share three core Statement 3 is correct: Crop residues or other
principles, which are, adopting minimum organic matter left in the field will improve
tillage (just enough to get the seed into the water penetration and is becoming a more
ground), using crop residues to cover soil widely accepted practice. It is one of the
surface, and adopting spatial and temporal easiest methods to improve water infiltration,
crop sequencing/crop rotations. especially for small farmers who do not have
the resources to implement more costly
Conservation agriculture is largely the product
corrective measures. Thus, it helps in water
of the collective efforts of several previous
conservation in agriculture.
agricultural movements, including no-till
agriculture, agroforestry, green manures/cover
crops, direct planting/seeding, integrated pest 15. Answer: (c)
management, and conservation tillage among
The applications of inorganic nitrogen fertilisers
many others.
to various crops have been continuously
Monoculture is the agricultural practice in increasing since the last many decades globally.
which only one type of crop is grown at a time. Although nitrogen fertiliser contributes
It deplete the soil's nutrients and destroy the substantially to yield enhancement, excessive
soil microorganism diversity and reduces the use of this manure has posed serious threats
biodiversity of a specific area so, it is not related to the environment and human health.
to concept of "Conservation Agriculture".
y Statement 1 is not correct: More use
Elimination Technique: Avoiding the of nitrogenous fertilisers in agriculture
cultivation of plantation crops is not does not contribute to the growth of
related to ‘conservation agriculture’. So, microorganisms. The proliferation of
Thus, Option (c) is correct. nitrogen-fixing microorganisms in soil
cannot occur due to the inappropriate use
of nitrogenous fertilisers in agriculture.
14. Answer: (c)
y The rate of nitrogen fertiliser application
Statement 1 is correct: In zero tillage (no- has a close relationship with nitrate
tillage), a new crop is planted in the residues accumulation in the surrounding
of the previous crop without any prior soil environment, groundwater, as well as leafy
tillage or seedbed preparation, and it is and root vegetables. Consumption of diets
possible when all the weeds are controlled by having high nitrate contents has contributed
the use of herbicides. Tillage is helpful in water to endogenous nitrosation, which could
conservation as they roughen the surface and lead to a thyroid condition, various kinds

344 Agriculture-Explanation
unacademy.com | Download the Unacademy app
Give your feedback here: Link
of human cancers, neural tube defects
(during foetus development), and diabetes. extreme, so it can be eliminated. Thus,
Overuse of Nitrogen fertilisers can increase Option (b) is correct.
the acidity of the soil. (Statement 2 is
correct)
17. Answer: (b)
y Leaching of nitrate into the groundwater
Biofertilizers are the living microbes that
can occur. (Statement 3 is correct)
inhabit the root zone or the interior plant
parts. They are a good source for enhancing
16. Answer: (b) nutrient availability in soil and plants. These
are categorised on the basis of the specific
The Sustainable Sugarcane Initiative (SSI) is
nutrient availability concern.
an innovative set of agronomic practices that
involve using fewer seeds, raising seeds in a y Biofertilizers are low-cost, effective,
nursery, and following new planting methods, environmentally friendly, and renewable
with wider seed spacing, and better water and sources of plant nutrients to supplement
nutrient management to increase the cane fertilisers. Integration of chemical, organic
yields significantly. The initiative was launched and biological sources of plant nutrients
jointly by International Crops Research and their management is necessary for
Institute for the Semi-Arid Tropics (ICRISAT) maintaining soil health for sustainable
and Worldwide Fund for Nature (WWF). agriculture. The bacterial organisms present
in the biofertilizer either fix atmospheric
Statement 1 is correct: By practicing SSI, the
nitrogen or solubilise insoluble forms of
seed cost can be reduced up to 75% and there
soil nutrients.
will be a reduction in the plant mortality rate.
While in conventional methods, the cost of y Nitrogen-fixing Biofertilizers include
seeds (seed material used are the stem cuttings Clostridium, Azotobacter, Nostoc,
known as “setts”) occupies a major part of the Anabaena, Rhizobium, Azollae, Anabaena
cost of cultivation. and Azospirillum. [Option (b) is correct]

Statement 2 is correct: Drip irrigation can y Phosphorus solubilizing Biofertilizer:


be practiced effectively in SSI due to wider Bacillus subtilis, Pseudomonas striata,
spacing and raising of single seedlings. Bacillus circulans Aspergillus awamori,
Penicillium sp, etc.
Statement 3 is not correct: The SSI method
encourages the application of organic manure
as it enhances the macro and micronutrient 18. Answer: (d)
content in the soil in an eco-friendly way and
Biopesticides are certain types of pesticides
helps in the optimum utilisation of some of the
derived from such natural materials as
chemical fertilisers.
animals, plants, bacteria, and certain minerals.
Statement 4 is correct: SSI supports Biopesticides are usually inherently less toxic
intercropping in sugarcane with crops like than conventional pesticides. Biopesticides
wheat, potato, cowpea, watermelon, brinjal generally affect only the target pest and closely
etc. In addition to effective utilisation of land, related organisms, in contrast to broad-
this practice will reduce weed growth up to spectrum, conventional pesticides that may
60% and give extra income to farmers. affect organisms as different as birds, insects
and mammals.
Elimination Technique: There is no
application of chemical/inorganic Option (d) is correct: Microbial pesticides are
fertilisers at all in this. Statement 3 is too one of the major classes of biopesticides which

Agriculture-Explanation 345
unacademy.com | Download the Unacademy app
Give your feedback here: Link
consist of a microorganism (e.g., a bacterium, small quantities and often decompose
fungus, virus or protozoan) as the active quickly, resulting in lower exposures and
ingredient. Microbial pesticides can control largely avoiding the pollution problems
many different kinds of pests, although each caused by conventional pesticides.
separate active ingredient is relatively specific y When used as a component of Integrated
for its target pest. For example, there are fungi Pest Management (IPM) programs,
that control certain weeds and other fungi that biopesticides can greatly reduce the use of
kill specific insects. conventional pesticides, while crop yields
Plant-Incorporated-Protectants (PIPs) are remain high.
pesticidal substances that plants produce
from genetic material that has been added to
the plant. 19. Answer: (b)

y Bacteria: Bacteria are microscopic, single- Carbon sequestration is the process of capturing
celled organisms that thrive in diverse and storing atmospheric carbon dioxide. It is
environments. These organisms can live in soil, one method of reducing the amount of carbon
the ocean and inside the human gut. Humans’ dioxide in the atmosphere with the goal of
relationship with bacteria is complex. reducing global climate change.

y Fungi: Fungi are eukaryotic, non-vascular, Types of Carbon Sequestration


non-motile and heterotrophic organisms. y Biological Carbon Sequestration
They can be either single-celled or y Geological Carbon Sequestration
multicellular organisms. Fungi consist
y Technological Carbon Sequestration
of long thread-like structures known as
hyphae. These hyphae together form a y Industrial Carbon Sequestration
mesh-like structure called mycelium. Fungi y Zero Tillage
possess a cell wall that is made up of chitin
Zero tillage refers to the arable land on
and polysaccharides. The cell wall comprises
which no tillage is applied between harvest
a protoplast which is differentiated into
and sowing. Zero tillage is a minimum tillage
other cell parts such as the cell membrane,
practice in which the crop is sown directly into
cytoplasm, cell organelles and nuclei.
soil not tilled since the harvest of the previous
y Biopesticides often are effective in very crop. [Option (b) is correct]

346 Agriculture-Explanation
unacademy.com | Download the Unacademy app
Give your feedback here: Link
8 INDIAN POLITY
AND GOVERNANCE
Historical Background & Making of
1 Indian Constitution
1. Consider the following statements in (a) 1 only
respect of the Constitution Day: (2023) (b) 2 only
Statement-I: (c) Both 1 and 2
The Constitution Day is celebrated on (d) Neither 1 nor 2
26th November every year to promote
constitutional values among citizens.
Statement-II: 3. Which one of the following factors
constitutes the best safeguard of liberty in
On 26th November, 1949, the Constituent a liberal democracy? (2021)
Assembly of India set up a Drafting
Committee under the Chairmanship of (a) A committed judiciary
Dr. B.R. Ambedkar to prepare a Draft (b) Centralization of powers
Constitution of India. (c) Elected government
Which one of the following is correct in (d) Separation of powers
respect of the above statements?
(a) Both Statement-I and Statement-II are
4. What was the exact constitutional status
correct and Statement-II is the correct
of India on 26th January, 1950? (2021)
explanation for Statement-I
(a) A Democratic Republic
(b)
Both Statement-I and Statement-II
are correct and Statement-II is not the (b) A Sovereign Democratic Republic
correct explanation for Statement-I (c)
A Sovereign Secular Democratic
(c) Statement-I is correct but Statement-II Republic
is incorrect (d) A Sovereign Socialist Secular Democratic
(d) Statement-I is incorrect but Statement- Republic
II is correct

5. Constitutional government means (2021)


2. We adopted parliamentary democracy (a) a representative government of a nation
based on the British model, but how does with federal structure
our model differ from that model? (2021)
(b)
a government whose Head enjoys
1. As regards legislation, the British nominal powers
Parliament is supreme or sovereign but
(c) a government whose Head enjoys real
in India, the power of the Parliament to
powers
legislate is limited.
(d) a government limited by the terms of
2. In India, matters related to the
the Constitution
constitutionality of the Amendment of
an Act of the Parliament are referred to
the Constitution Bench by the Supreme 6. In the context of polity, which one of the
Court. following would you accept as the most
Select the correct answer using the code appropriate definition of liberty? (2019)
given below. (a)
Protection against the tyranny of

348 Historical Background & Making of Indian Constitution


unacademy.com | Download the Unacademy app
Give your feedback here: Link
political rulers 9. Democracy's superior virtue lies in the fact
(b) Absence of restraints that it calls into activity (2017)

(c) Opportunity to do whatever one likes (a)


the intelligence and character of
ordinary men and women.
(d) Opportunity to develop oneself fully
(b) the methods for strengthening executive
leadership.
7. Which one of the following reflects the (c) a superior individual with dynamism
most appropriate relationship between and vision.
law and liberty? (2018)
(d) a band of dedicated party workers
(a) If there are more laws, there is less
liberty.
(b) If there are no laws, there is no liberty. 10. The mind of the makers of the Constitution
of India is reflected in which of the
(c) If there is liberty, laws have to be made following? (2017)
by the people.
(a) The Preamble
(d) If laws are changed too often, liberty is
in danger. (b) The Fundamental Rights
(c) The Directive Principles of State Policy

8. Which of the following are regarded as the (d) The Fundamental Duties
main feature of “Rule of Law”? (2018)
1. Limitation of power 11.
The distribution of powers between
2. Equality before the law the Centre and the States in the Indian
Constitution is based on the scheme
3.
People’s responsibility to the provided in the (2012)
Government
(a) Morley-Minto Reforms, 1909
4. Liberty and civil rights
(b) Montagu-Chelmsford Act, 1919
Select the correct answer using the codes
given below: (c) Government of India Act, 1935

(a) 1 and 3 only (d) Indian Independence Act, 1947

(b) 2 and 4 only


(c) 1, 2 and 4 only
(d) 1, 2, 3 and 4

Historical Background & Making of Indian Constitution 349


unacademy.com | Download the Unacademy app
Give your feedback here: Link
Historical Background & Making of
1 Indian Constitution-Explanation
1. Answer: (c) Court having five or more judges on it. These
Statement 1 is correct: The Constitution Day benches are not a routine phenomenon. Most
of India, also known as National Law Day, is cases before the Supreme Court are heard
celebrated on 26th November every year. It is and decided by a bench of two judges (called
observed to commemorate the adoption of a Division Bench) and sometimes by three.
the Constitution of India by the Constituent Constitution Benches are set up when the case
Assembly in 1949, which came into effect on involves a substantial question of law about
26th January 1950, marking the establishment the interpretation of the Constitution (Article
of India as a democratic republic. 145(3) of the Constitution, which mandates that
a bench hear such matters of not less than five
Statement 2 is incorrect: On 29th August 1947, judges). Presently, Constitution Benches are
the Constituent Assembly established a Drafting set up ad hoc as and when the need arises. The
Committee under the Chairmanship of Dr. B.R. idea behind a Constitution Bench is clear: it is
Ambedkar. This committee was responsible for constituted in rare cases to decide important
the task of drafting the Constitution of India. questions of fact or legal and constitutional
The drafting process took approximately two interpretation.
years, and the final draft was adopted by the
Constituent Assembly on 26th November 1949.
3. Answer: (d)

2. Answer: (c) Option (d) is correct: The Separation of powers


between the legislature, the executive and the
Statement 1 is correct: Britain follows judiciary constitutes an important safeguard
Parliamentary sovereignty as a principle of its of liberty in a liberal democracy. The doctrine
constitution. It makes Parliament the supreme of Separation of powers entails the division of
legal authority in the UK, which can create or end the legislative, executive, and judicial functions
any law. Generally, the courts cannot overrule of government among different organs. This
its legislation, and no Parliament can pass separation minimizes the possibility of arbitrary
laws that future Parliaments cannot change. excesses by the government since all three
Indian Parliament is not a sovereign body like organs act as checks and balances on the
the British Parliament. The Indian Parliament powers of each other. Therefore, none of the
may, in the exercise of its constituent power, three organs can usurp the essential functions
amend by way of addition, variation or repeal of other organs.
any provision of the Constitution following
the procedure laid down for the purpose. This demarcation prevents the concentration
However, the Parliament cannot amend those of excessive power by any branch of the
provisions which form the ‘basic structure’ of Government. It thus helps to safeguard the
the Constitution. The Supreme Court ruled this liberty and rights of the people in a democracy.
in the Kesavananda Bharati case (1973).
Statement 2 is correct: In India, matters related 4. Answer: (b)
to the constitutionality of an amendment of Option (b) is correct: The original Preamble,
an act of the Parliament are referred to the adopted by the Constituent Assembly on 26th
Constitution Bench by the Supreme Court. A November 1949 and came into force on 26th
Constitution Bench is a bench of the Supreme January 1950, declared India as a “Sovereign

350 Historical Background & Making of Indian Constitution-Explanation


unacademy.com | Download the Unacademy app
Give your feedback here: Link
Democratic Republic”. restriction on liberty of individuals so as to
By the 42nd Amendment of 1976, enacted allow equal liberty to all. Law is precisely
during the Emergency, the words “Socialist” the means to impose such restrictions. Law
and “Secular” were inserted. The Preamble now restricts liberty in requiring us to do things
reads “Sovereign Socialist Secular Democratic that we might otherwise want to do, and
Republic”. in requiring us to refrain from actions that
we might otherwise want to do. Therefore,
restrictions upon liberty imposed by the law
5. Answer: (d) intend to protect the liberty of others which
Option (d) is correct: A constitutional government would be impeded by the behaviour that the
seeks to limit and regulate the exercise of law restricts. Thus, if there are no laws, there
political power by the government. Constitutional is no liberty.
government is, by definition, limited government.
It means the government is conducted according
8. Answer: (c)
to rules and principles, which are binding on all
political actors. The Constitution lays down limits The Rule of Law is the foundational feature
on the powers of the government and gives the of all modern democratic nations. A.V. Dicey,
rights to the citizens. the British jurist, has given a few elements or
aspects of Rule of Law.
Option (c) is correct: Equality before the law,
6. Answer: (d) that is, equal subjection of all citizens (rich or
Option (d) is correct: The term ‘liberty’ means poor, high, or low, official, or non-official) to
the absence of restraints on activities of the ordinary law of the land administered by
individuals and at the same time, providing the ordinary law courts.
opportunities for the development of individual Absence of arbitrary power/ limitation of power,
personalities. However, it does not mean that is, no man can be punished except for a
‘licence’ to do what one likes and has to be breach of law. It implies the government has
enjoyed within the limitations mentioned in the limited powers that it derived from the law.
Constitution itself. In brief, liberty conceived
by the Preamble or Fundamental Rights is not The primacy of the rights, liberty and civil rights,
absolute but qualified. of the individual, that is, the Constitution is
the result of rights of individuals as defined
Though, liberty in narrow sense is the absence and enforced by the court of law rather than
of restraints. But a wider interpretation of the Constitution being the source of individual
liberty is positive and that is facilitating the rights.
overall development of an individual.
The first and second elements are applicable
to the Indian system, the Constitution is the
7. Answer: (b) source of individual rights. Rule of law along
Liberty or freedom is formally defined as with its three basic elements results in the
absence of restraint. Law, on the other hand, formation of a government that is responsible
imposes restraints on certain activities of to the people, not the other way around.
individuals. When liberty is accepted as a
universal principle, it is imperative that liberty 9. Answer: (a)
of one individual does not endanger the liberty
of another individual. Option (a) is correct: The Indian Constitution
provides for representative democracy under
Option (b) is correct: This necessitates which the executive is responsible to the

Historical Background & Making of Indian Constitution-Explanation 351


unacademy.com | Download the Unacademy app
Give your feedback here: Link
legislature for all its activities. The take-off India Federation consisting of provinces
point for democracy is the idea of consent, and princely states as units. The Act
i.e., the desire, approval and participation of divided the powers between the Centre
people. It is the decision of the people that and units in terms of three lists—Federal
creates a democratic government and decides List (for Centre, with 59 items), Provincial
its functioning. Since democracy requires List (for provinces, with 54 items) and the
voters’ decision making, hence intelligence Concurrent List (for both, with 36 items).
and character of ordinary men and women are Residuary powers were given to the Viceroy.
called in. However, the federation never came into
being as the princely states did not join it.

10. Answer: (a) y It abolished dyarchy in the provinces and


introduced ‘provincial autonomy’ in its
Option (a) is correct: The Preamble refers to
place. The provinces were allowed to act
the introduction or preface to the Constitution.
as autonomous units of administration
It contains the summary or essence of the
in their defined spheres. Moreover, the
Constitution. The Preamble reveals four
Act introduced responsible governments
ingredients or components:
in provinces, that is, the governor was
y Source of the authority of the Constitution required to act with the advice of ministers
y Nature of Indian polity responsible to the provincial legislature.
This came into effect in 1937 and was
y Objectives of the Constitution
discontinued in 1939.
y Date of adoption of the Constitution
y It provided for the adoption of dyarchy
Preamble embodies the basic philosophy at the Centre. Consequently, the federal
and fundamental values (political, moral, and subjects were divided into reserved
religious) on which the Constitution is based subjects and transferred subjects. However,
upon. It contains the grand and noble vision this provision of the Act did not come into
of the Constituent Assembly and reflects the operation at all.
dreams and aspirations of the founding fathers
y It introduced bicameralism in six out of
of the Constitution.
eleven provinces. Thus, the legislatures of
Sir Alladi Krishnaswami Iyer, a member of the Bengal, Bombay, Madras, Bihar, Assam and
Constituent Assembly, said, “The Preamble the United Provinces were made bicameral
to our Constitution expresses what we had consisting of a legislative council (upper
thought or dreamt for so long.” house) and a legislative assembly (lower
house). However, many restrictions were
placed on them.
11. Answer: (c)
y It further extended the principle of
Option (c) is correct: The Constitution of India
communal representation by providing
provides for a federal system of government
separate electorates for depressed classes
in the country. Since the structural part of
(scheduled castes), women and labour
the Constitution is, to a large extent, derived
(workers).
from the Government of India Act of 1935,
the distribution of power between the Center y It abolished the Council of India, established
and the States is also largely taken from the by the Government of India Act of 1858. The
Government of India Act of 1935. secretary of state for India was provided
with a team of advisors.
The main features of the act are as follows:
y It extended the franchise. About 10 percent
y It provided for the establishment of an All-

352 Historical Background & Making of Indian Constitution-Explanation


unacademy.com | Download the Unacademy app
Give your feedback here: Link
of the total population got the voting right. y It provided for the establishment of not only
y It provided for the establishment of a Federal Public Service Commission but
a Reserve Bank of India to control the also a Provincial Public Service Commission
currency and credit of the country. and Joint Public Service Commission for
two or more provinces.

Historical Background & Making of Indian Constitution-Explanation 353


unacademy.com | Download the Unacademy app
Give your feedback here: Link
Features of the Indian
2 Constitution
1. In essence, what does 'Due Process of Law' 4. If a particular area is brought under the
mean? (2023) Fifth Schedule of the Constitution of India,
(a) The principle of natural justice which one of the following statements best
reflects the consequence of it ? (2022)
(b) The procedure established by law
(a) This would prevent the transfer of land
(c) Fair application of law of tribal people to non-tribal people.
(d) Equality before law (b) This would create a local self-governing
body in that area.
2. Which one of the following statements (c) This would convert that area into a
best reflects the Chief purpose of the Union Territory.
'Constitution' of a country? (2023) (d) The State having such areas would be
(a) It determines the objective for the declared a Special Category State.
making of necessary laws.
(b) It enables the creation of political 5.
Under the Indian Constitution,
offices and a government. concentration of wealth violates (2021)
(c) It defines and limits the powers of (a) the Right to Equality
government.
(b) the Directive Principles of State Policy
(d) It secures social justice, social equality
and social security (c) the Right to Freedom
(d) the Concept of Welfare

3. Consider the following statements: (2023)


1. According to the Constitution of 6. What is the position of the Right to Property
India, the Central Government has a in India? (2021)
duty to protect States from internal (a) Legal right available to citizens only
disturbances. (b) Legal right available to any person
2. The Constitution of India exempts the (c) Fundamental Right available to citizens
States from providing legal counsel only
to person being held for preventive
detention (d) Neither Fundamental Right nor legal
right
3. According to the Prevention of Terrorism
Act, 2002, confession of the accused
before the police cannot be used as 7. ‘Right to Privacy’ is protected under which
evidence. Article of the Constitution of India? (2021)
How many of the above statements are (a) Article 15
correct? (b) Article 19
(a) Only one (c) Article 21
(b) Only two (d) Article 29
(c) All three
(d) None

354 Features of the Indian Constitution


unacademy.com | Download the Unacademy app
Give your feedback here: Link
8. With reference to India, consider the (b)
The Union Legislature has elected
following statements : (2021) representatives from constituent units.
1. There is only one citizenship and one (c) The Union Cabinet can have elected
domicile. representatives from regional parties.
2. A citizen by birth only can become the (d) The Fundamental Rights are enforceable
Head of State. by Courts of Law.
3. A foreigner once granted citizenship
cannot be deprived of it under any 11. Consider the following statements : (2021)
circumstances.
1. ‘Right to the City’ is an agreed human
Which of the statements given above is / right and the UN-Habitat monitors the
are correct? commitments made by each country in
(a) 1 only this regard.
(b) 2 only 2. ‘Right to the City’ gives every occupant
(c) 1 and 3 of the city the right to reclaim public
spaces and public participation in the
(d) 2 and 3
city.
3. ‘Right to the City’ means that the State
9. Which one of the following best defines cannot deny any public service or facility
the term ‘State’? (2021) to the unauthorized colonies in the city.
(a) A community of persons permanently Which of the statements given above is /
occupying a definite territory are correct?
independent of external control and
(a) 1 only
possessing an organized government
(b) 3 only
(b) A politically organized people of a
definite territory and possessing an (c) 1 and 2
authority to govern them, maintain (d) 2 and 3
law and order, protect their natural
rights and safeguard their means of
sustenance 12. A legislation which confers on the executive
or administrative authority an unguided
(c) A number of persons who have been and uncontrolled discretionary power in
living in a definite territory for a very long the matter of application of law violates
time with their own culture, tradition which one of the following Articles of the
and government Constitution of India?
(d) A society permanently living in a definite (a) Article 14
territory with a central authority, an
executive responsible to the central (b) Article 28
authority and an independent judiciary (c) Article 32
(d) Article 44
10. Which one of the following in Indian polity
is an essential feature that indicates that it 13. A Constitutional government by definition
is federal in character? (2021) is a (2020)
(a) The independence of the judiciary is (a) government by legislature
safeguarded.

Features of the Indian Constitution 355


unacademy.com | Download the Unacademy app
Give your feedback here: Link
(b) popular government secularism, fundamental rights and
(c) multi-party government democracy.

(d) limited government 2. The Constitution of India provides


for ‘judicial review’ to safeguard the
‘citizens’ liberties and to preserve the
14. Other than the Fundamental Rights, which ideals on which the Constitution is
of the following parts of the Constitution based.
of India reflect/reflects the principles and Which of the statements given above is/are
provisions of the Universal Declaration of correct?
Human Rights (1948)? (2020)
(a) 1 only
1. Preamble
(b) 2 only
2. Directive Principles of State Policy
(c) Both 1 and 2
3. Fundamental Duties
(d) Neither 1 nor 2
Select the correct answer using the code
given below:
(a) 1 and 2 only 18.
One common agreement between
Gandhism and Marxism is (2020)
(b) 2 only
(a) the final goal of a stateless society
(c) 1 and 3 only
(b) class struggle
(d) 1, 2 and 3
(c) abolition of private property
(d) economic determinism
15. Which one of the following categories
of Fundamental Rights incorporates
protection against untouchability as a form 19. The Preamble to the Constitution of India
of discrimination? (2020) is (2020)
(a) Right against Exploitation (a) a part of the Constitution but has no
(b) Right to Freedom legal effect

(c) Right to Constitutional Remedies (b) not a part of the Constitution and has
no legal effect either
(d) Right to Equality
(c) a part of the Constitution and has the
same legal effect as any other part
16. Which part of the Constitution of India (d) a part of the Constitution but has no
declares the ideal of a Welfare State? legal effect independently of other parts
(2020)
(a) Directive Principles of State Policy
20. With reference to the provisions contained
(b) Fundamental Rights in Part IV of the Constitution of India, which
(c) Preamble of the following statements is/are correct?
(d) Seventh Schedule (2020)
1. They shall be enforceable by courts.

17. Consider the following statements: (2020) 2. They shall not be enforceable by any
court.
1. The Constitution of India defines its
‘basic structure’ in terms of federalism, 3. The principles laid down in this part are

356 Features of the Indian Constitution


unacademy.com | Download the Unacademy app
Give your feedback here: Link
to influence the making of laws by the (c) Constitutional Right
state. (d) Legal Right
Select the correct answer using the code
given below:
25.
Which of the following are envisaged
(a) 1 only by the Right against Exploitation in the
(b) 2 only Constitution of India? (2017)
(c) 1 and 3 only 1. Prohibition of traffic in human beings
(d) 2 and 3 only and forced labour
2. Abolition of untouchability

21. Under which Schedule of the Constitution 3. Protection of the interests of minorities
of India can the transfer of tribal land to 4. Prohibition of employment of children
private parties for mining be declared null in factories and mines
and void? (2019) Select the correct answer using the code
(a) Third Schedule given below:
(b) Fifth Schedule (a) 1, 2 and 4 only
(c) Ninth Schedule (b) 2, 3 and 4 only
(d) Twelfth Schedule (c) 1 and 4 only
(d) 1, 2, 3 and 4
22.
The Ninth Schedule was introduced in
the Constitution of India during the prime 26. Which one of the following is not a feature
ministership of: (2019) of Indian federalism? (2017)
(a) Jawaharlal Nehru (a) There is an independent judiciary in
(b) Lal Bahadur Shastri India.
(c) Indira Gandhi (b)
Powers have been clearly divided
(d) Morarji Desai between the Centre and the States.
(c) The federating units have been given
unequal representation in the Rajya
23. Which Article of the Constitution of India Sabha.
safeguards one’s right to marry the person
of one’s choice? (2019) (d) It is the result of an agreement among
the federating units.
(a) Article 19
(b) Article 21
27. Which of the following statements is/are
(c) Article 25 true of the Fundamental Duties of an Indian
(d) Article 29 citizen? (2017)
24. Right to vote and to be elected in India is a 1. A legislative process has been provided
(2017) to enforce these duties.
(a) Fundamental Right 2. They are correlative to legal duties.
(b) Natural Right Select the correct answer using the code
given below:
(a) 1 only

Features of the Indian Constitution 357


unacademy.com | Download the Unacademy app
Give your feedback here: Link
(b) 2 only (c) Right to work, education and public
(c) Both 1 and 2 assistance

(d) Neither 1 nor 2 (d)


Securing living wage and human
conditions of work to workers

28.
Which one of the following objectives
is not embodied in the Preamble to the 32. Consider the following statements: (2017)
Constitution of India? (2017) With reference to the Constitution of India,
(a) Liberty of thought the Directive Principles of State Policy
constitute limitations upon
(b) Economic liberty
1. legislative function.
(c) Liberty of expression
2. executive function.
(d) Liberty of belief
Which of the above statements is/are
correct?
29. In the context of India, which one of the (a) 1 only
following is the correct relationship
between Rights and Duties? (2017) (b) 2 only

(a) Rights are correlative with Duties. (c) Both 1 and 2

(b)
Rights are personal and hence (d) Neither 1 nor 2
independent of society and Duties.
(c) Rights, not Duties, are important for the 33. Which one of the following statements is
advancement of the personality of the correct? (2017)
citizen. (a) Rights are claims of the State against
(d) Duties, not Rights, are important for the the citizens.
stability of the State. (b)
Rights are privileges that are
incorporated in the Constitution of a
30. One of the implications of equality in State.
society is the absence of (2017) (c) Rights are claims of the citizens against
(a) Privileges the State.

(b) Restraints (d) Rights are privileges of a few citizens


against the many.
(c) Competition
(d) Ideology
34. The ideal of “Welfare State” in the Indian
Constitution is enshrined in its (2015)
31. Which principle among the following was (a) Preamble
added to the Directive Principles of State
Policy by the 42nd Amendment to the (b) Directive Principles of State Policy
Constitution? (2017) (c) Fundamental Rights
(a) Equal pay for equal work for both men (d) Seventh Schedule
and women
(b)
Participation of workers in the 35. “To uphold and protect the Sovereignty
management of industries Unity and Integrity of India” is a provision

358 Features of the Indian Constitution


unacademy.com | Download the Unacademy app
Give your feedback here: Link
made in the (2015) 2. Places effective restrictions on the
(a) Preamble of the Constitute Authority of the State in the interest of
individual liberty.
(b) Directive Principles of State Policy
Which of the statements given above is/are
(c) Fundamental Rights correct?
(d) Fundamental Duties (a) 1 only
(b) 2 only
36. The provisions in the Fifth Schedule and (c) Both 1 and 2
Sixth Schedule in the Constitution of India
are made in order to (2015) (d) Neither 1 nor 2

(a) protect the interests of Scheduled


Tribes 39. In the Constitution of India, promotion of
(b)
determine the boundaries between international peace and security is included
states in the: (2014)

(c) determine the powers, authorities, and (a) Preamble of the Constitution
responsibilities of Panchayats (b) Directive Principles of State Policy
(d) protect the interests of all the border (c) Fundamental Duties
States (d) Ninth Schedule

37.
Consider the following statements 40. ‘Economic Justice’ as one of the objectives
regarding the Directive Principles of State of Indian Constitution has been provided in
Policy: (2015) (2013)
1. The Principles spell out the socio- (a) The Preamble and Fundamental Rights
economic democracy in the country.
(b)
The Preamble and the Directive
2. The provisions contained in these Principles of State Policy
Principles are not enforceable by any
court. (c)
The Fundamental Rights and the
Directive Principles of State Policy
Which of the statements given above is/are
correct? (d) None of the above

(a) 1 only
(b) 2 only 41. According to the Constitution of India,
which of the following are fundamental for
(c) Both 1 and 2 the governance of the country? (2013)
(d) Neither 1 nor 2 (a) Fundamental Rights
(b) Fundamental Duties
38.
Consider the following statements: A (c) Directive Principles of State Policy
Constitutional Government is one which
(2014) (d) Fundamental Rights and Fundamental
Duties
1. A Constitutional Government is one
which places the effective restrictions 42. Consider the following provisions under
on individual liberty in the interest of the Directive Principles of State Policy
State Authority. as enshrined in the Constitution of India:
(2012)

Features of the Indian Constitution 359


unacademy.com | Download the Unacademy app
Give your feedback here: Link
1. Securing for citizens of India a uniform 2. To protect the weaker sections from
civil code social injustice
2. Organising village Panchayats 3. To develop the scientific temper and
3. Promoting cottage industries in rural spirit of inquiry
areas 4. To strive towards excellence in all
4. Securing for all the workers reasonable spheres of individual and collective
leisure and cultural opportunities activity

Which of the above are the Gandhian Select the correct answer using the code
Principles that are reflected in the Directive given below:
Principles of State Policy? (a) 1 and 2 only
(a) 1, 2 and 4 only (b) 2 only
(b) 2 and 3 only (c) 1, 3 and 4
(c) 1, 3 and 4 only (d) 1, 2, 3 and 4
(d) 1, 2, 3 and 4
45. Consider the following: (2011)
43. Which of the following provisions of the 1. Right to education
Constitution of India have a bearing on
2. Right to equal access to public service
Education? (2012)
3. Right to food
1. Directive Principles of State Policy
Which of the above is/are Human Right/
2. Rural and Urban Local Bodies
Human Rights under “Universal Declaration
3. Fifth Schedule of Human Rights”?
4. Sixth Schedule (a) 1 only
5. Seventh Schedule (b) 1 and 2 only
Select the correct answer using the code (c) 3 only
given below:
(d) 1, 2 and 3
(a) 1 and 2 only
(b) 3, 4 and 5 only
46.
Under the Constitution of India, which
(c) 1, 2 and 5 only one of the following is not a Fundamental
(d) 1, 2, 3, 4 and 5 Duty? (2011)
(a) To vote in public elections

44. Which of the following is/are among the (b) To develop the scientific temper
Fundamental Duties of citizens laid down (c) To safeguard public property
in the Indian Constitution? (2012)
(d) To abide by the Constitution and respect
1. To preserve the rich heritage of our its ideals
composite culture

360 Features of the Indian Constitution


unacademy.com | Download the Unacademy app
Give your feedback here: Link
Features of the Indian
2 Constitution-Explanation
1. Answer: (c) afford him the earliest opportunity of making a
Due process of law refers to questioning the representation against the order, which means,
legitimacy of a legislative or an executive action legal options to question the detention have
on the grounds of it not being fundamentally been provided in the constitution.
fair, reasonable or just. It was identified as an Statement 3 is incorrect: Under the UAPA, the
implied interpretation in the landmark Maneka confession of an accused made to a police
Gandhi case while procedure established by officer is not admissible as evidence in court.
law is explicitly mentioned under article 21. Section 25 of the Indian Evidence Act, 1872,
specifies that a confession made to a police
2. Answer: (c ) officer is considered to be involuntary and
The purpose of the constitution can be derived hence cannot be used as evidence against the
from the larger principle of constitutionalism accused.
which refers to limiting the powers of the
government by a single or a set of laws
4. Answer: (a)
determining power structure and organization
of the government. Option (a) is correct: This would prevent the
shifting of the land of tribal people to non-
tribal people. This aspect of the law relating
3. Answer: (a) to special constitutional protections for
Statement 1 is correct: According to the Scheduled Tribes and Scheduled Areas has
Constitution of India, the responsibility for also seen some important developments. A
maintaining law and order and protecting states leading decision on the subject was passed by
from internal disturbances primarily lies with the Supreme Court in Samatha vs. the State of
the respective state governments.However, in Andhra Pradesh. The Court was asked to rule
certain exceptional circumstances, the central on whether the grant of a mining lease, in a
government has the power to intervene and Scheduled Area to a non-tribal, was in violation
provide assistance to a state government in of laws preventing alienation of Adivasi lands.
handling internal disturbances. This is outlined The specific context for the case was the
in Article 355 of the Constitution, which states Andhra Pradesh Scheduled Areas Land
that it is the duty of the central government to Transfer Regulation 1 of 1970, which explicitly
protect every state against external aggression prohibits any person in a Scheduled Area from
and internal disturbances and to ensure that transferring lands to anyone other than a
the government of each state is carried out Scheduled Tribe. The premise of the regulation
in accordance with the provisions of the is that all land in Scheduled Area is presumed
Constitution. to have been Adivasi land; hence, not only
Statement 2 is incorrect: : Article 22 (5) of should no land now pass into the hands of
the Indian Constitution does provide for non-Adivasis, but any land presently owned by
the following when any person is detained non-tribal should, if being transferred, come
in pursuance of an order made under any back to the hands of Scheduled Tribes. The
law providing for preventive detention. The question before the Court was whether the
authority making the order shall, as soon as may grant of a mining lease on government land to
be, communicate to such person the grounds a non-tribal violated this principle.
on which the order has been made and shall

Features of the Indian Constitution-Explanation 361


unacademy.com | Download the Unacademy app
Give your feedback here: Link
The Court did not rely purely on the specific of their life.
clauses of the Regulation and instead held
that the Constitution itself requires that land
in Scheduled Areas should remain with the 8. Answer: (a)
Adivasis to preserve their autonomy, culture Statement 1 is correct: In India, people have
and society. The Regulation, hence, should be single citizenship, i.e., a person can have only
interpreted ‘expansively’ in order to fulfill this one citizenship. Also, at a particular time, a
mandate. person can have only one domicile.
Statement 2 is not correct: Any citizen of India
5. Answer: (b) (Citizen by birth or Naturalised Citizen) can
become the Head of State, i.e., the President,
Option (b) is correct: The concentration of in India.
wealth violates the Directive principles of state
policy. Under Article 39 of the Constitution of Statement 3 is not correct: A foreigner,
India, the State shall direct its policy towards once granted citizenship, can be deprived
ensuring that the operation of the economic of citizenship under the circumstances like
system does not result in the concentration registration is done by fraud, a person is
of wealth and means of production to the disloyal towards the constitution, voluntarily
common detriment. acquiring citizenship of another country, etc.

6. Answer: (b) 9. Answer: (a)

Option (b) is correct: The Supreme Court has Option (a) is correct: A state is a form of human
recently held that a citizen’s right to own association distinguished from other social
private property is a human right. The Right to groups based on its purpose of formation. The
Property is a legal right available to any person. state is a political organization of society. The
Article 300-A provides that no person shall be elements of a state are:
deprived of his property save by authority of y People, i.e., the community of persons
law. “ Since the article says that no person permanently living there.
shall be deprived, the statement, “legal rights y Government
are available to citizens only”, is not correct.
y Territory: Demarcated boundary that
separates one state from the other.
7. Answer: (c) y Sovereignty: It has an independent say in
Option (c) is correct: In K. S. Puttaswamy Case all domestic and external decision-making.
(2017), the Supreme Court of India has held
that the right to privacy is a Fundamental
10. Answer: (a)
Right and is protected under Article 21
of the Constitution of India. Privacy is a Option (a) is correct: The establishment of an
constitutionally protected right that deals with independent judiciary necessarily indicates
a guarantee of life and liberty in Article 21 of the federal character of Indian polity.
the Constitution. It includes the preservation Following are the federal features of the Indian
of personal intimacies, sanctity of family life, Constitution:
marriage, procreation, the home and sexual
y Dual polity consists of the Union at the
orientation. It connotes a right to be left
Centre and the states at the periphery.
alone. It safeguards individual autonomy and
recognises one’s ability to control vital aspects y Written Constitution

362 Features of the Indian Constitution-Explanation


unacademy.com | Download the Unacademy app
Give your feedback here: Link
y Division of powers between the Centre and a basic human right for all, including migrants,
the states (by Schedule seven) slum dwellers and the homeless. RTC gives
y Supremacy of the Constitution every occupant, irrespective of their legality, a
claim over the city. It requires the State to act
y The rigidity of the constitution
on the principles of social justice and provide
y Bicameral legislature consisting of Rajya equal services to each and every resident. So,
Sabha and Lok Sabha ‘Right to the City’ means that the State cannot
y Independence of judiciary deny any public service of the facility to the
unauthorized colonies in the City.

11. Answer: (d)


12. Answer: (a)
Statement 1 is not correct: Though, the right
to the city was at the heart of the New Urban Option (a) is correct: Article 14 of the Indian
Agenda agreed upon at Habitat III. But ‘Right Constitution says that the State shall not
to the City’ is not an agreed human right. In deny to any person equality before the law
the run-up to Habitat III, India opposed the or the equal protection of the laws within
inclusion of the Right to the City in the draft the territory of India. The concept of ‘equality
New Urban Agenda in 2016 that was to define before the law’ is an element of the concept
the way cities worldwide are shaped over the of ‘Rule of Law’. This concept has the following
next two decades. three elements or aspects:

Statement 2 is correct. The Right to the City y Absence of arbitrary power


is the right of all inhabitants (present and y Equality before the law that is, equal
future, permanent and temporary) to inhabit, subjection of all citizens to the ordinary
use, occupy, produce, transform, govern and law of the land
enjoy cities, towns and human settlements
y The constitution is the result of the rights
that are just, inclusive, safe, sustainable and
of the individual
democratic, defined as common goods for
enjoying life with dignity and peace. The right The first and the second elements are
to the city further implies responsibilities on applicable to India. This means ‘equality before
governments and people to claim, defend, and the law’ under Article 14 connotes the absence
promote this right. This right claims for: of arbitrary power with the authorities. Thus,
when a legislation confers uncontrolled
y the social function of the city;
discretionary powers on any authority, it tends
y uality public spaces; to violate article 14 of the constitution. In
y sustainable and inclusive rural-urban the Jayantilal Kalidas Mehta vs the State of
linkages; Maharashtra case (1970), some sections were
contended that gave unguided, uncontrolled
y inclusive economies;
and arbitrary powers to the collector, which
y inclusive citizenship; were violating Article 14 of the constitution.
y enhanced political participation; Option (b) is not correct: Article 28 is related to
y non-discrimination; the right to freedom of religion. It provides that
no religious instruction shall be provided in any
y gender equality; and
educational institution wholly maintained out
y cultural diversity of State funds. Further, no person attending
Statement 3 is correct: The Right to the City any educational institution recognised by the
(RTC) recognises equal access to urban life as State or receiving aid out of State funds shall
be required to attend any religious instruction

Features of the Indian Constitution-Explanation 363


unacademy.com | Download the Unacademy app
Give your feedback here: Link
or worship in that institution without his UDHR, everyone, as a member of society, has
consent. the right to social security and is entitled
Option (c) is not correct: Article 32 confers to realisation, through national effort and
the right to remedies for the enforcement of international cooperation and in accordance
the fundamental rights of an aggrieved citizen. with the organisation and resources of each
Article 32 affirms the right to move the Supreme State, of the economic, social, and cultural
Court if a fundamental right is violated. Under rights indispensable for his dignity and the
this article, the Supreme Court can issue writs free development of his personality. The
for the enforcement of any of the fundamental Directive Principles of State Policy of the Indian
rights of the citizens. Constitution bear a close resemblance to the
Universal Declaration of Human Rights.
Option (d) is not correct: Article 44 provides for
one of the Directive Principles of State Policy. Statement 3 is correct: The duties of the
Article 44 says that the “State shall endeavour individual to the society are both a part
to secure for the citizens a uniform civil code of Fundamental Duties mentioned in the
throughout the territory of India”. Constitution of India and the principles and
provisions of the Universal Declaration of
Human Rights.
13. Answer: (d)
Elimination Technique: principles and
Option (d) is correct: By the existence of a
provisions of the Universal Declaration of
Constitution, a Constitutional government
Human Rights reflect in all the parts of the
is defined with a legal instrument or merely
Constitution of India given above.
a set of fixed norms or principles generally
accepted as the fundamental law of the polity
that effectively controls or limits the exercise 15. Answer: (d)
of political power. Hence, it is often termed as Option (d) is correct: Article 17 of the
‘limited government’ as well. Constitution of India abolishes the age-old
The essence of Constitutionalism is the control practice of Untouchability and forbids its
or limit of power by its distribution among practice in any form. Untouchability refers to
several state organs. Constitutional government a social practice that looks down upon certain
also refers to a written Constitution guiding oppressed classes solely on account of their
the country with mechanisms called checks birth and makes any discrimination against
and balances. them on this ground. This comes under the
Right to Equality (Article 14- 18).

14. Answer: (d) The Parliament of India passed the


Untouchability (offences) Act in 1955 which
Statement 1 is correct: As per Article 3 of the
came into force 1st June 1955 to make
Universal Declaration of Human Rights (UDHR),
untouchability law further strong. In 1976, this
everyone has the right to life, liberty, and
Act was further amended and renamed as
security of person. The Preamble mentions
Protection of Civil Rights Act, 1955.
the term ‘democratic’ which denotes that
the Government gets its authority from the
will of the people. It mentioned all are equal 16. Answer: (a)
“irrespective of the race, religion, language, sex Option (a) is correct: Part IV (Article 36-51)
and culture” in this modern era democracy is of the Constitution of India, deals with the
considered as a human right. Directive Principles of State Policy (DPSP)
Statement 2 is correct: As per Article 22 of which declares the ideal of a Welfare State.

364 Features of the Indian Constitution-Explanation


unacademy.com | Download the Unacademy app
Give your feedback here: Link
DPSP aims to create a socio-economic establishment of a stateless and classless
condition where the citizens of the State can society, their means for achieving this aim
lead a good life and establish socio-economic are different. For Mahatma Gandhi, in an ideal
democracy through a welfare state. Article 38 state, there is no State which he wanted to
of the Constitution of India mentions that: achieve through non-violent means but Marx,
who saw State is an instrument of oppression
y The State shall strive to promote the welfare
and an organ of the bourgeoisie that only works
of the people by securing and protecting as
for maintaining the class dominance, wanted a
effectively as it may a social order in which
stateless society by striving for class struggle
justice, social, economic, and political, shall
and revolution.
inform all the institutions of national life.
y The State shall, in particular, strive to
minimise the inequalities in income, and 19. Answer: (d)
endeavour to eliminate inequalities in Option (d) is correct: The Constitution of India
status, facilities and opportunities, not begins with a Preamble. The Preamble contains
only amongst individuals but also amongst the ideals, objectives, and basic principles of
groups of people residing in different areas the Constitution. Preamble is a part of Indian
or engaged in different vocations. Constitution, and it certainly does not have a
legal effect independently since it cannot be
enforced in a court of law. It acts as an aid
17. Answer: (d)
during the interpretation of other Articles when
Statement 1 is not correct: The Indian the language is found ambiguous. Therefore, it
Constitution does not define the term ‘basic begets some kind of legal effect when read along
structure’. It is a judicial tool that was coined by with the Constitution, not independent from it.
the Supreme Court of India in the Kesavananda
In the Berubari Union case 1960, the Supreme
Bharati vs State of Kerala (1973) landmark
Court held that, “Preamble is not a part of the
judgement.
Constitution”. Later, in the Kesavananda Bharati
Statement 2 is incorrect: The judicial review Vs. State of Kerala 1973 case, the Supreme
doctrine is based on the principle of rule of Court ruled that “Preamble is a part of the
law and separation of powers. Judicial review Constitution and can be amended” as any
is the process for testing and balancing the other provisions of the Constitution, provided
separation of powers. The Concept of Judicial the basic structure of the Constitution is not
Review is the basic principle of the Constitution destroyed.
in India. Although there is no explicit provision
In the Union Government Vs. LIC of India
in the Indian Constitution for judicial review, it
case 1995, the Supreme Court has once again
is an integral part of our Constitution.
held that Preamble is an integral part of the
Elimination Technique: Indian Constitution Constitution but is not directly enforceable in
does not define ‘basic structure’. It a court of justice in India.
was coined by the Supreme Court in
the Kesavananda Bharati case. Hence,
20. Answer: (d)
statement 1 is not correct.
Statement 1 is not correct and Statement 2 is
correct: Article 37 deals with the application
18. Answer: (a) of the principles contained in Part-IV (Directive
Option (a) is correct: There is a great similarity Principles of States Policy) of the Indian
between Mahatma Gandhi and Karl Marx. Constitution. The provisions contained in this
However, while the final aim of both is the Part shall not be enforced by any court.

Features of the Indian Constitution-Explanation 365


unacademy.com | Download the Unacademy app
Give your feedback here: Link
Statement 3 is correct: The provisions the Ninth Schedule are also open to judicial
contained in this Part shall not be enforced scrutiny.
by any court, but the principles therein laid Option (a) is correct: The Ninth Schedule
down are nevertheless fundamental in the was introduced by the 1st Constitutional
governance of the country and it shall be the Amendment Act, 1951 during the Prime
duty of the State to apply these principles in Ministership of Jawaharlal Nehru.
making laws.
Additional Information:
Elimination Technique: Directive Principles Most of the laws protected under the Schedule
are not enforced by any courts. Hence, concern agriculture/land issues, the list includes
statement 1 is not correct and 2 is correct. other subjects, such as reservation, for example,
a Tamil Nadu law that provides 69 per cent
reservation in the state is part of this Schedule.
21. Answer: (b)
Option (b) is correct: The Fifth Schedule
contains provisions in relation to the 23. Answer: (b)
administration and control of scheduled Option (b) is correct: In 2018, the Supreme Court
areas and scheduled tribes. The purpose of in Hadiya Marriage case has held that the Right
Scheduled Areas, as also recognised in several to Marry a person of one’s choice is integral
judgments, is to preserve the tribal autonomy, to Article 21 (Right to Life and Liberty) of the
their culture and economic empowerment. Constitution. Article 21 declares that no person
It was further fortified by the Samatha Vs. State shall be deprived of his life or personal liberty
of Andhra Pradesh & Ors (1997) judgement where except according to procedure established by
the Supreme Court declared that the transfer law. This right is available to both citizens and
of tribal land to private parties for mining was non-citizens. The protection under Article 21 is
null and void under the Fifth Schedule. available not only against arbitrary executive
action but also against arbitrary legislative
Additional Information: action.
Only the President of India has the power to
declare an area as Scheduled Area by order
and can also declare a scheduled area to cease 24. Answer: (c)
to be known as scheduled area. Further, the All Indian citizens who are eligible to vote are
President can also by order alter the boundaries given a chance to exercise their franchise and
of the scheduled areas. However, to change the participate in the electoral process. The Indian
boundary of a scheduled area, the President is Constitution has granted the right to vote to
required to consult the Governor of the state all Indian citizens of sound mind above the
in which the area is located. No separate law / age of 18, irrespective of an individual’s caste,
act is needed to establish, change boundaries religion, social or economic status.
or discontinue a scheduled area. Option (c) is correct: Justice Chelameswar and
Justice Abhay Manohar Sapre in the Rajbala
22. Answer: (a) case for Haryana PRI elections (2015) held
that the right to vote and right to contest at
The Ninth Schedule of the Constitution contains an election to a Panchayat are Constitutional
a list of Central and State laws which cannot Rights. Also, the provisions in the Constitution
be challenged in courts. Currently, 284 laws of India shed light on this matter:
are protected from judicial review. However,
the Supreme Court in 2007 held that laws in y Article 326: Elections to the House of the
People and to the Legislative Assemblies of

366 Features of the Indian Constitution-Explanation


unacademy.com | Download the Unacademy app
Give your feedback here: Link
States to be on the basis of adult suffrage. pay.
The elections to the House of the People and Article 24 prohibits the employment of children
to the Legislative Assembly of every State below the age of 14 years in any factory, mine
shall be on the basis of adult suffrage; but is or other hazardous activities like construction
to say, every person who is a citizen of India work or railways. However, it does not prohibit
and who is not less than twenty one years their employment in any harmless or innocent
of age on such date as may be fixed in that work.
behalf by or under any law made by the
appropriate legislature and is not otherwise Additional Information:
disqualified under this constitution or any y Abolition of Untouchability (Article 17) is
law made by the appropriate Legislature on under the Right to Equality.
the ground of non-residence, unsoundness y Protection of interests of Minorities (Article
of mind, crime or corrupt or illegal practice, 29) is under Cultural and Educational Rights.
shall be entitled to be registered as a voter
at any such election. Elimination Technique: Fundamental Rights
provides for Rights Against Exploitation in
y Article 327: Power of Parliament to make
Article 23 to 24. Hence, statement 2 and 3
provision with respect to elections to
(Abolition of untouchability and Protection
Legislatures Subject to the provisions of
of the interests of minorities) are not
the Constitution, Parliament may from
correct.
time to time by law made provision with
respect to all matters relating to, or
in connection with, elections to either 26. Answer: (d)
House of Parliament or to the House or
The Constitution of India provides for a federal
either House of the Legislature of a State
system of government in the country. However,
including the preparation of electoral rolls,
the term ‘Federation’ has nowhere been
the delimitation of constituencies and all
mentioned in the Constitution.
other matters necessary for securing the
due constitution of such House or Houses. Article 1 of the Constitution describes India as
a ‘Union of States’. According to B.R. Ambedkar,
the phrase ‘Union of States’ has been preferred
25. Answer: (c) to ‘Federation of States’ to indicate two things:
The Fundamental Rights are enshrined in Part y The Indian Federation is not the result of
III of the Constitution from Articles 12 to 35. It an agreement among the states like the
also provides for Rights Against Exploitation in American Federation. (Option (d) is correct)
Article 23 to 24.
y The states have no right to secede from the
Option (c) is correct: Article 23 prohibits federation.
trafficking in human beings, beggars (forced
Federal Features of the Constitution:
labour) and other forms of forced labour.
y Independent Judiciary: Constitution
y This right is available to both citizens and
establishes an independent judiciary
non-citizens.
headed by the Supreme Court. It is
y It protects individuals not only against the maintained through measures like the
state but also against private persons. security of tenure of judges, fixed service
y Exception to Article 23: It permits the state conditions.
to impose compulsory service for public y Division of Powers: The Constitution clearly
purposes, for example, military service or divided the powers between the centre
social service, for which it is not bound to

Features of the Indian Constitution-Explanation 367


unacademy.com | Download the Unacademy app
Give your feedback here: Link
and the states in terms of the Union List, promote among them all
the State List and the Concurrent List in FRATERNITY assuring the dignity of the
the Seventh Schedule. The subjects not individual and the unity and integrity of the
mentioned in any of the lists are a part of Nation
the Residuary list, controlled by the centre.
IN OUR CONSTITUENT ASSEMBLY this twenty-
y Bicameralism: Two houses have been sixth day of November 1949, do HEREBY
provided, the Lok Sabha and the Rajya ADOPT, ENACT AND GIVE TO OURSELVES THIS
Sabha. States are given representation CONSTITUTION
in the Rajya Sabha on the basis of their
population.
29 Answer: (a)

27. Answer: (d) Option (a) is correct: The Rights and Duties of
the citizens are correlative and inseparable.
Fundamental Duties were incorporated in the Rights are what we want others to do for us,
Constitution at the suggestion of the Swaran whereas duties are those acts which we should
Singh Committee in the year 1976 through the perform for others. Thus, a right comes with
42nd amendment. Later in 2002, one more an obligation to show respect for the rights of
fundamental duty was added. There are, in others. The obligations that accompany rights
total, 11 Fundamental Duties enshrined in are in the form of duties.
Article 51 A of the Constitution.
Our duties and the consequences we bear for
Statement 1 is not correct: Though failing to keep them therefore exist as a self-
recommended by the Committee, no legislative contained whole. They follow a simple logic:
process has been provided to enforce these that peaceful co-existence requires a degree
duties. They are non-justiciable in nature, like of self-sacrifice, and that, if necessary, this
the Directive Principles. must be enforced through the set of sanctions.
Statement 2 is not correct: One cannot move Rights, on the other hand, follow a different
to the court if fundamental duties are violated; principle entirely. The twin principles of anti-
hence there is no legal sanction against their dehumanisation and anti-hierarchy reveal the
violation and are not correlative to legal duties. transformative purpose of the fundamental
rights chapter: the recognition that true
democracy could not exist without ensuring
28. Answer: (b)
that at a basic level, the dignity and equality of
Option (b) is correct: Economic Liberty is individuals was protected, both from the state
not embodied in the Preamble of the Indian as well as from social majorities. It was only
Constitution. It provides for the liberty of with these guarantees could an individual rise
thought, expression, belief, faith, and worship. from the status of subject to that of citizen.
The Preamble of Indian Constitution:
WE, THE PEOPLE OF INDIA, having solemnly 30. Answer:(a)
resolved to constitute India into a SOVEREIGN
Option (a) is correct: The term ‘equality’
SOCIALIST SECULAR DEMOCRATIC REPUBLIC
means the absence of special privileges to
and to secure to all its citizens:
any section of the society and the provision
JUSTICE, social, economic, and political of adequate opportunities for all individuals
LIBERTY of thought, expression, belief, faith, without any discrimination. The Preamble of
and worship the Indian Constitution secures all citizens
of India equality of status and opportunity.
EQUALITY of status and of opportunity and to

368 Features of the Indian Constitution-Explanation


unacademy.com | Download the Unacademy app
Give your feedback here: Link
This provision embraces three dimensions of (Article 48 A).
equality: civic, political, and economic.
The very idea can be found in Article 18 of the 32. Answer: (d)
Indian Constitution, which states, “Abolition of
titles No title, not being a military or academic Statement 1 is not correct: The Directive
distinction, shall be conferred by the State No Principles of State Policy confer no legal
citizen of India shall accept any title from any rights, create no legal remedies and are non-
foreign State No person who is not a citizen of justiciable in nature. Therefore, the government
India shall, while he holds any office of profit (centre, state, local) cannot be compelled to
or trust under the State, accept without the implement them. Hence, it does not limit the
consent of the President any title from any legislative function.
foreign State No person holding any office of Statement 2 is not correct: Directive Principles
profit or trust under the State shall, without do not constitute limitations on the government
the consent of the President, accept any because they are not enforceable. However,
present, emolument, or office of any kind from Fundamental Rights operate as limitations on
or under any foreign State Right to Freedom”. the tyranny of the executive and the legislature.

31. Answer: (b) 33. Answer: (c)


Directive Principles of State Policy (DPSP) Option (c) is correct: Rights are legal and moral
are enumerated in Part IV of the Constitution entitlements or claims of any citizen over
from Articles 36 to 51. This idea of DPSP was other fellow beings, over society and over the
borrowed from the Irish Constitution. government (State). In most cases, the claimed
Dr B.R. Ambedkar described them as ‘Novel rights are directed towards the state, i.e.,
features’ of the Indian Constitution. The Directive through these rights, people make demands
Principles, along with the Fundamental Rights, upon the state.
contain the philosophy of the Constitution and A democracy must ensure that individuals
is the soul of the Constitution. have certain rights, and that the government
The Directive Principles are non-justiciable in will always recognise these rights. Therefore,
nature; that is, they are not legally enforceable. it is often a practice in most democratic
Therefore, the government cannot be compelled countries to list the rights of the citizens in
to implement them, but they help the courts in the Constitution itself. Such a list of rights
examining the Constitutional validity of a law mentioned and protected by the Constitution is
called the ‘bill of rights’. A bill of rights prohibits
Option (b) is correct: The 42nd Amendment Act the government from thus acting against the
of 1976 included four new Directive Principles rights of the individuals and ensures a remedy
to the original list. They require the State: in case there is violation of these rights.
y To secure opportunities for the healthy
development of children (Article 39).
34. Answer: (b)
y To promote equal justice and to provide
free legal aid to the poor (Article 39 A) Option (b) is correct: The Directive Principles of
State Policy constitute a very comprehensive
y To take steps to secure the participation of economic, social and political program for a
workers in the management of industries modern democratic State. They embody the
(Article 43 A). concept of a ‘Welfare State’. In brief, they seek
y To protect and improve the environment to establish economic and social democracy in
and to safeguard forests and wildlife the country.

Features of the Indian Constitution-Explanation 369


unacademy.com | Download the Unacademy app
Give your feedback here: Link
According to Dr B.R. Ambedkar, the Directive made in the Fundamental Duties (Part IV-A of
Principles of State Policy is a ‘novel feature’ of the Indian Constitution).
the Indian Constitution. They are enumerated In 1976, the Sardar Swaran Singh Committee
in Part IV of the Indian Constitution. was set up to make recommendations about
Principles seeking a Welfare State: Fundamental Duties. In the same year, the
y Article 38: To promote the welfare of the Fundamental Duties of citizens were added
people by securing a social order permeated in the Constitution. Later, in 2002, one more
by justice–social, economic and political Fundamental Duty was added.
and to minimise inequalities in income, According to Article 51A, it shall be the duty of
status, facilities and opportunities every citizen of India:
y Article 39: To secure the right to adequate y To abide by the Constitution and respect
means of livelihood for all citizens; the its ideals and institutions, the National Flag
equitable distribution of material resources and the National Anthem.
of the community for the common good; y To cherish and follow the noble ideals that
prevention of concentration of wealth and inspired the national struggle for freedom.
means of production; equal pay for equal
work for men and women; preservation y To uphold and protect the Sovereignty,
of the health and strength of workers Unity and Integrity of India. [Option (d) is
and children against forcible abuse; and correct]
opportunities for the healthy development y To defend the country and render national
of children. service when called upon to do so.
y Article 39A: To promote equal justice and y To promote harmony and the spirit of
to provide free legal aid to the poor. common brotherhood amongst all the
y Article 41: To secure the right to work, to people of India transcending religious,
education and to public assistance in cases linguistic and regional or sectional
of unemployment, old age, sickness and diversities and to renounce practices
disablement. derogatory to the dignity of women.

y Article 42: To make provision for just and y To value and preserve the rich heritage of
humane conditions of work and maternity the country’s composite culture.
relief. y To protect and improve the natural
y Article 43: To secure a living wage, a decent environment, including forests, lakes, rivers
standard of life and social and cultural and wildlife and to have compassion for
opportunities for all workers. living creatures.

y Article 43A: To take steps to secure the y To develop scientific temper, humanism
participation of workers in the management and the spirit of inquiry and reform.
of industries. y To safeguard public property and to abjure
y Article 47: To raise the level of nutrition violence.
and the standard of living of people and to y To strive towards excellence in all spheres
improve public health. of individual and collective activity so that
the nation constantly rises to higher levels
of endeavour and achievement.
35. Answer: (d)
y To provide opportunities for education to
The provisions to uphold and protect the his child or ward, between the age of six
Sovereignty, Unity and Integrity of India are and fourteen years. This duty was added by

370 Features of the Indian Constitution-Explanation


unacademy.com | Download the Unacademy app
Give your feedback here: Link
the 86th Constitutional Amendment Act, violation. Therefore, the government (Central,
2002. State and local) cannot be compelled to
implement them.

36. Answer: (a)


Option (a) is correct: The provisions in the 38. Answer: (b)
Fifth Schedule and Sixth Schedule in the Statement 1 is not correct: A Constitutional
Constitution of India are made in order to Government can place restrictions on individual
protect the interests of Scheduled Tribes. liberty but not in the interest of State Authority
y Article 244 in Part X of the Constitution but for greater good of greater numbers. In the
envisages a special system of administration case of India, the Fundamental Rights are not
for certain areas designated as Scheduled absolute but qualified. The State can impose
Areas and Tribal Areas. The Fifth Schedule reasonable restrictions on them. However,
of the Constitution deals with the whether such restrictions are reasonable or
administration and control of Scheduled not is to be decided by the Courts. Thus, they
Areas and Scheduled Tribes in any state strike a balance between the rights of the
except the four states of Assam, Meghalaya, individual and those of the society as a whole,
Tripura, and Mizoram. between individual liberty and social control.

y The Sixth Schedule of the Constitution, on Statement 2 is correct: A Constitutional


the other hand, deals with the administration Government is one that derives its power,
of the Tribal Areas in the four northeastern function and authority from a written
states of Assam, Meghalaya, Tripura, and document i.e., Constitution. Such a
Mizoram. Government is characterised by limitations
of power. A Constitutional Government does
not exercise unrestricted power rather its
37. Answer: (c) authority is subjected to effective restrictions.
The phrase ‘Directive Principles of State Policy’ The Constitution imposes restrictions on
denotes the ideals that the State should keep the Authority of the State in the interest of
in mind while formulating policies and enacting individual liberty. In India, Fundamental Rights
laws. The Directive Principles resemble the are available to citizens against the arbitrary
‘Instrument of Instructions’ enumerated in the action of the State.
Government of India Act of 1935.
Statement 1 is correct: The Directive Principles 39. Answer: (b)
constitute a very comprehensive economic,
Option (b) is correct: Promotion of international
social and political program for a modern
peace and security is included in Article 51 of
democratic State. They aim at realising the
the Directive Principles of States Policy (DPSP),
high ideals of justice, liberty, equality and
Part IV of the Indian Constitution.
fraternity as outlined in the Preamble to the
Constitution. They embody the concept of a y DPSP asks the government to promote
‘welfare state’ and not that of a ‘police state’, international peace and security and
which existed during the colonial era. In brief, maintain just and honourable relations
they seek to establish economic and social between nations, to foster respect for
democracy in the country. international law and treaty obligations,
and to encourage the settlement of
Statement 2 is correct: The Directive Principles
international disputes by arbitration.
are non-justiciable in nature; that is, they are
not legally enforceable by the courts for their y DPSP denotes the ideals that the State

Features of the Indian Constitution-Explanation 371


unacademy.com | Download the Unacademy app
Give your feedback here: Link
should keep in mind while formulating are fundamental in the governance of the
policies and enacting laws. These country, and it shall be the duty of the State
are the Constitutional instructions or to apply these principles in making laws. Thus,
recommendations to the State in legislative, they impose a moral obligation on the State
executive, and administrative matters. Dr B Authorities for their application.
R Ambedkar described these principles as In the Minerva Mills case (1980), the Supreme
‘novel features’ of the Indian Constitution. Court held that the Indian Constitution is
founded on the bedrock of the balance
40. Answer: (b) between the Fundamental Rights and the
Directive Principles.
Economic justice denotes the non-
discrimination between people on the basis of
economic factors. It involves the elimination 42. Answer: (b)
of glaring inequalities in wealth, income, and ‘Directive Principles of State Policy’ denotes
property. A combination of Social Justice and the ideals that the State should keep in mind
Economic Justice denotes what is known as while formulating policies and enacting laws.
‘distributive justice’. Gandhian principles and ideals have a huge
Option (b) is correct: ‘Economic Justice’ as one influence on Constituent Assembly members.
of the objectives of the Indian Constitution Directive Principles of States Policy that
that has been provided in the Preamble and reflects Gandhian principles are as follows:
the Directive Principles of State Policy. y To organise village panchayats and endow
y The Preamble of the Indian Constitution them with necessary powers and authority
talks about Social, Economic, and Political to enable them to function as units of self-
Justice. The Directive Principles of State government (Article 40). (Statement 2 is
Policy of the Indian Constitution lay down correct)
the framework of a democratic socialist y To promote cottage industries on an
state, aim at providing social and Economic individual or co-operation basis in rural
Justice, and set the path towards welfare areas (Article 43). (Statement 3 is correct)
state.
y To promote voluntary formation,
y Article 38 states that the State shall strive autonomous functioning, democratic
to promote the welfare of the people by control, and professional management of
securing and protecting as effectively as it co-operative societies 8a (Article 43B).
may a social order in which justice, social,
economic and political, shall inform all the y To promote the educational and economic
institutions of social life. interests of SCs, STs, and other weaker
sections of the society and to protect
y Similarly, Article 39 states that the Right them from social injustice and exploitation
to adequate means of livelihood for all (Article 46).
citizens, equal pay for equal work for both
men and women. y To prohibit the consumption of intoxicating
drinks and drugs which are injurious to
health (Article 47).
41. Answer: (c) y To prohibit the slaughter of cows, calves
Option (c) is correct: Though the Directive and other milch and draught cattle and to
Principles are non-justiciable, the Constitution improve their breeds (Article 48).
(Article 37) makes it clear that ‘these principles

372 Features of the Indian Constitution-Explanation


unacademy.com | Download the Unacademy app
Give your feedback here: Link
43. Answer: (d) List from State List.
Option (d) is correct: Provisions of the
Constitution of India have a bearing on 44. Answer: (c)
Education are:
The Fundamental Duties in the Indian
y Directive Principles of State Policy: Constitution are inspired by the Constitution
Directive Principles of State Policy denotes of erstwhile USSR. Notably, none of the
the ideals that the State should keep Constitutions of major democratic countries
in mind while formulating policies and like the USA, Canada, France, Germany,
enacting laws. Article 45 of DPSP says to Australia and so on specifically contain a list
provide early childhood care and education of duties of citizens.
for all children until they complete the age
of six years (Article 45). The Fundamental Duties were incorporated in
the Constitution of India with the enactment
y Rural and Urban Local Bodies: Eleventh of the 42nd Constitutional Amendment Act
Schedule contains a comprehensive list in 1976. Such incorporation is based on the
of items in regard to which a state is recommendation of the Sardar Swaran Singh
required to endow powers, authorities and Committee, created in 1976 by the Congress
responsibilities to Panchayat in order to Party. This amendment added a new part,
enable them to function as institutions of namely, Part IVA to the Constitution. This new
self-governance. Item 17 of the Eleventh part consists of only one Article, that is, Article
Schedule talks of education including 51A which for the first time specified a code of
primary and secondary schools. ten fundamental duties of the citizens.
y Fifth Schedule: The Fifth Schedule of the Option (c) is correct: According to Article 51 A,
Constitution deals with the administration it shall be the duty of every citizen of India:
and control of scheduled areas and
scheduled tribes in any state except the y To abide by the Constitution and respect
four states of Assam, Meghalaya, Tripura its ideals and institutions, the National Flag
and Mizoram. and the National Anthem.

y Sixth Schedule: The Constitution, under y To cherish and follow the noble ideals that
Sixth Schedule, contains special provisions inspired the national struggle for freedom.
for the administration of tribal areas in y To uphold and protect the sovereignty,
the four north-eastern states of Assam, unity and integrity of India.
Meghalaya, Tripura and Mizoram. The y To defend the country and render national
District Council in these four states can service called upon to do so.
establish, construct or manage primary
y To promote harmony and the spirit of
schools, dispensaries, markets, ferries,
common brotherhood amongst all the
fisheries, roads and so on in the district.
people of India transcending religious,
y Seventh Schedule: The Constitution linguistic, and regional or sectional
provides for a three-fold distribution of diversities and to renounce practices
legislative subjects between the Centre derogatory to the dignity of women.
and the states, viz., List-I (the Union List),
y To value and preserve the rich heritage of
List-II (the State List) and List-III (the
the country’s composite culture.
Concurrent List) in the Seventh Schedule.
Earlier education as an item was present in y To protect and improve the natural
the state list. The 42nd Amendment Act of environment including forests, lakes, rivers
1976 transferred education to Concurrent and wildlife and to have compassion for

Features of the Indian Constitution-Explanation 373


unacademy.com | Download the Unacademy app
Give your feedback here: Link
living creatures. Duties of citizens were added in the
y To develop scientific temper, humanism Constitution. Later, in 2002, one more
and the spirit of inquiry and reform. Fundamental Duty was added.
y To safeguard public property and to abjure According to Article 51A, it shall be the duty of
violence. every citizen of India:
y To strive towards excellence in all spheres y to abide by the Constitution and respect
of individual and collective activity so that its ideals and institutions, the National Flag
the nation constantly rises to higher levels and the National Anthem
of endeavour and achievement. y to cherish and follow the noble ideals that
y To provide opportunities for education to inspired the national struggle for freedom
his child or ward between the age of six y to uphold and protect the sovereignty,
and fourteen years. This duty was added by unity and integrity of India
the 86th Constitutional Amendment Act,
2002. y to defend the country and render national
service when called upon to do so
y to promote harmony and the spirit of
45. Answer: (d) common brotherhood amongst all the
On 10 December 1948, the General Assembly people of India transcending religious,
of the United Nations announced the Universal linguistic and regional or sectional
Declaration of Human Rights (UDHR) - 30 rights diversities and to renounce practices
and freedoms of all human beings. derogatory to the dignity of women
Statement 1 is correct: Article 26 of UDHR y to value and preserve the rich heritage of
states that everyone has the right to education. the country’s composite culture
Education shall be free, at least in the y to protect and improve the natural
elementary and fundamental stages. environment including forests, lakes, rivers
Statement 2 is Correct: Article 21 of UDHR and wildlife and to have compassion for
states that everyone has the right of equal living creatures
access to public service in the country. y to develop scientific temper, humanism
Statement 3 is correct: Article 25 of UDHR and the spirit of inquiry and reform
states that everyone has the right to a y to safeguard public property and to abjure
standard of living adequate for the health violence
and well-being of himself and of his family,
including food, clothing, housing and medical y to strive towards excellence in all spheres
care and necessary social services, and the of individual and collective activity so that
right to security in the event of unemployment, the nation constantly rises to higher levels
sickness, disability, widowhood, old age or of endeavour and achievement
other lack of livelihood in circumstances y to provide opportunities for education to
beyond his control. his child or ward, between the age of six
and fourteen years. This Duty was added
by the 86th Constitutional Amendment Act,
46. Answer: (a) 2002.
In 1976, the Sardar Swaran Singh Committee
was set up to make recommendations about Elimination Technique: To vote in public
Fundamental Duties. In 1976, the Fundamental elections is not a Fundamental Duty.

374 Features of the Indian Constitution-Explanation


unacademy.com | Download the Unacademy app
Give your feedback here: Link
3 Legislature

1. Consider the following statements : (2023) How many of the above statements are
Statement-I : correct?

In India, prisons are managed by State (a) Only one


Governments with their own rules (b) Only two
and regulations for the day-to-day (c) All three
administration of prisons.
(a) None
Statement-II:
In India, prisons are governed by the Prisons
Act, 1894 which expressly kept the subject 3. With reference to 'Scheduled Areas' in
of prisons in the control of Provincial India, consider the following statements :
Governments. (2023)

Which one of the following is correct in 1. Within a State, the notification of an


respect of the above statements? area as Scheduled Area takes place
through an Order of the President.
(a) Both Statement-I and Statement-II are
correct and Statement-II is the correct 2. The largest administrative unit forming
explanation for Statement-I the Scheduled Area is the District and
the lowest is the cluster of villages in
(b)
Both Statement-I and Statement-II the Block.
are correct and Statement-II is not the
correct explanation for Statement-I 3. The Chief Ministers of the concerned
States are required to submit annual
(c) Statement-I is correct but Statement-II reports to the Union Home Ministry on
is incorrect the administration of Scheduled Areas
(d) Statement-I is incorrect but Statement- in the States.
II is correct How many of the above statements are
correct?
2. With reference to Finance Bill and Money (a) Only one
Bill in the Indian Parliament, consider the (b) Only two
following statements : (2023)
(c) All three
1. When the Lok Sabha transmits Finance
Bill to the Rajya Sabha, it can amend or (d) None
reject the Bill.
2. When the Lok Sabha transmits Money 4. With reference to Deputy Speaker of Lok
Bill to the Rajya Sabha, it cannot amend Sabha, consider the following statements :
or reject the Bill, it can only make (2022)
recommendations. 1. As per the Rules of Procedure and
3. In the case of disagreement between Conduct of Business in Lok Sabha, the
the Lok Sabha and the Rajya Sabha, election of Deputy Speaker shall be
there is no joint sitting for Money Bill, held on such date as the Speaker may
but a joint sitting becomes necessary fix.
for Finance Bill. 2. There is a mandatory provision that

Legislature 375
unacademy.com | Download the Unacademy app
Give your feedback here: Link
the election of a candidate as Deputy against the Council of Ministers
Speaker of Lok Sabha shall be from 3. To impeach the President of India
either the principal opposition party or
the ruling party. Select the correct answer using the code
given below :
3. The Deputy Speaker has the same power
as of the Speaker when presiding over (a) 1 and 2
the sitting of the House and no appeal (b) 2 only
lies against his rulings. (c) 1 and 3
4. The well established parliamentary (d) 3 only
practice regarding the appointment
of Deputy Speaker is that the motion
is moved by the Speaker and duly 7. Consider the following statements : (2022)
seconded by the Prime Minister. 1. A bill amending the Constitution
Which of the statements given above are requires a prior recommendation of the
correct ? President of India.
(a) 1 and 3 only 2. When a Constitution Amendment Bill is
presented to the President of India, it is
(b) 1, 2 and 3
obligatory for the President of India to
(c) 3 and 4 only give his/her assent.
(d) 2 and 4 only 3. A Constitution Amendment Bill must be
passed by both the Lok Sabha and the
5. With reference to anti-defection law in Rajya Sabha by a special majority and
India, consider the following statements : there is no provision for joint sitting.
(2022) Which of the statements given above are
1. The law specifies that a nominated correct ?
legislator cannot join any political party (a) 1 and 2 only
within six months of being appointed to (b) 2 and 3 only
the House.
(c) 1 and 3 only
2. The law does not provide any time-
(d) 1,2 and 3
frame within which the presiding officer
has to decide a defection case.
Which of the statements given above is/are 8. Along with the Budget, the Finance Minister
correct ? also places other documents before the
Parliament which include ‘The Macro
(a) 1 only
Economic Framework Statement’. The
(b) 2 only aforesaid document is presented because
(c) Both 1 and 2 this is mandated by (2020)
(d) Neither 1 nor 2 (a) Long standing parliamentary convention
(b) Article 112 and Article 110 (1) of the
6. Which of the following is/are the exclusive Constitution of India
power(s) of Lok Sabha ? (2022) (c) Article 113 of the Constitution of India
1. To ratify the declaration of Emergency (d) Provisions of the Fiscal Responsibility
2. To pass a motion of no-confidence and Budget Management Act, 2003

376 Legislature
unacademy.com | Download the Unacademy app
Give your feedback here: Link
9. A Parliamentary System of Government is 12. Consider the following statements: (2019)
one in which (2020) 1. The Parliament (Prevention of
(a) all political parties in the Parliament are Disqualification) Act, 1959 exempts
represented in the Government several posts from disqualification on
(b) the Government is responsible to the the grounds of ‘Office of Profit’.
Parliament and can be removed by it 2. The above-mentioned Act was amended
(c) the Government is elected by the five times.
people and can be removed by them. 3. The term ‘Office of Profit’ is well-defined
(d)
the Government is chosen by the in the Constitution of India.
Parliament but cannot be removed by it Which of the statements given above is/are
before completion of a fixed-term correct?
(a) 1 and 2 only
10. Consider the following statements: (2020) (b) 3 only
1. According to the Constitution of India, (c) 2 and 3 only
a person who is eligible to vote can (d) 1, 2 and 3
be made a minister in a state for six
months even if he/she is not a member
of the Legislature of that state. 13. Consider the following statements: (2018)
2. According to the Representation of 1. In the first Lok Sabha, the single
People Act, 1951, a person convicted largest party in the opposition was the
of a criminal offence and sentenced Swatantra Party.
to imprisonment for five years 2. In the Lok Sabha, a “Leader of the
is permanently disqualified from Opposition” was recognised for the first
contesting an election even after his time in 1969.
release from prison.
3. In the Lok Sabha, if the party does not
Which of the statements given above is/are have a minimum of 75 members, its
correct? leader cannot be recognised as the
(a) 1 only leader of opposition.
(b) 2 only Which of the following statements given is/
(c) Both 1 and 2 are correct?

(d) Neither 1 nor 2 (a) 1 and 3 only


(b) 2 only

11. Rajya Sabha has equal powers with Lok (c) 2 and 3 only
Sabha in (2020) (d) 1, 2 and 3
(a) the matter of creating new All India
Services 14. Consider the following statements: (2018)
(b) amending the Constitution 1. Parliament of India can place a
(c) the removal of the government particular law in the Ninth Schedule of
(d) making cut motions the Constitution of India.
2. The validity of law placed in the Ninth
Schedule cannot be examined by any

Legislature 377
unacademy.com | Download the Unacademy app
Give your feedback here: Link
court and no Judgement can be made to imposition, abolition, remission,
on it. alteration or regulation of any tax.
Which of the statements given above is/are (b) A Money Bill has the provisions for the
correct? custody of the Consolidated Fund of
(a) 1 only India or the Contingency Fund of India.

(b) 2 only (c) A Money Bill is concerned with the


appropriation of money out of the
(c) 1 and 2 only
Contingency Fund of India.
(d) Neither 1 nor 2
(d) A Money Bill deals with the regulations
of borrowings of money or giving of any
15. With reference to the Parliament of India, guarantee by the Government of India.
which of the following Parliamentary
Committees scrutinises and reports to
18. Consider the following statements: (2018)
the House whether the powers to make
regulations, rules, sub-rules, by-laws, etc. 1. The Speaker of the Legislative Assembly
conferred by the Constitution or delegated shall vacate his/her office if he/she
by the Parliament are being properly ceases to be a member of the Assembly.
exercised by the Executives within the 2. Whenever the Legislative Assembly is
scope of such delegation? (2018) dissolved, the Speaker shall vacate his/
(a) Committee on Government Assurances her office immediately.

(b) Committee on Subordinate Legislation Which of the statements given above is/are
correct?
(c) Committee on rules
(a) 1 only
(d) Business Advisory Committee
(b) 2 only
(c) Both 1 and 2
16. Right to Privacy is protected as an intrinsic
part of Right to Life and Personal Liberty. (d) Neither 1 nor 2
Which of the following in the Constitution
of India correctly and appropriately imply 19. Consider the following statements: (2017)
the above statement? (2018)
1. In the election for Lok Sabha or State
(a) Article 14 and the provisions under the Assembly, the winning candidate must
42nd amendment to the Constitution. get at least 50 per cent of the votes
(b) Article 17 and the Directive Principle of polled to be declared elected.
the State Policy in Part IV 2. According to the provisions laid down in
(c) Article 21 and the freedoms guaranteed the Constitution of India, in Lok Sabha,
in Part III the Speaker's post goes to the majority
(d) Article 24 and provisions under the 44th party and the Deputy Speaker's to the
Amendments to the Constitution Opposition.
Which of the statements given above is/are
correct?
17.
Regarding Money Bill, which of the
following statements is not correct? (2018) (a) 1 only

(a) A bill shall be deemed to be a Money (b) 2 only


Bill if it contains only provisions related (c) Both 1 and 2

378 Legislature
unacademy.com | Download the Unacademy app
Give your feedback here: Link
(d) Neither 1 nor 2 which the election is to be contested.
(c) any citizen of India whose name appears
20. Out of the following statements, choose the in the electoral roll of a constituency.
one that brings out the principle underlying (d) any citizen of India.
the Cabinet form of Government: (2017)
(a) An arrangement for minimising criticism 23. Which of the following statements is/are
against the Government whose correct? (2016)
responsibilities are complex and hard
to carry out to the satisfaction of all. 1. A bill pending in the Lok Sabha lapses
on its prorogation.
(b) A mechanism for speeding up the
activities of the Government whose 2. A bill pending in the Rajya Sabha, which
responsibilities are increasing day by has not been passed by the Lok Sabha,
day. shall not lapse on the dissolution of the
Lok Sabha.
(c)
A mechanism of parliamentary
democracy for ensuring collective Select the correct answer using the code
responsibility of the Government to the given below.
people. (a) 1 only
(d) A device for strengthening the hands of (b) 2 only
the head of the Government whose hold (c) Both 1 and 2
over the people is in a state of decline.
(d) Neither 1 nor 2

21. With reference to the Parliament of India,


24. The Parliament of India acquires the powers
consider the following statements: (2017)
to legislate on any items in the State List
1. A private member's bill is a bill in the national interest if the resolution to
presented by a Member of Parliament that effect has been passed by the (2016)
who is not elected but only nominated
(a) Lok Sabha by a simple majority of its
by the President of India.
total membership
2. Recently, a private member's bill has
(b) Lok Sabha by a majority of not less than
been passed in the Parliament of India
two-thirds of its total membership
for the first time in its history.
(c) Rajya Sabha by a simple majority of its
Which of the statements given above is/are
total membership
correct?
(d) Rajya Sabha by a majority of not less
(a) 1 only
than two- thirds of its members present
(b) 2 only and voting
(c) Both 1 and 2
(d) Neither 1 nor 2 25.
There is a Parliamentary System of
Government in India because the (2015)
22. For election to the Lok Sabha, a nomination (a) Lok Sabha is elected directly by the
paper can be filed by (2017) people
(a) anyone residing in India. (b) Parliament can amend the Constitution
(b) a resident of the constituency from (c) Rajya Sabha cannot be dissolved

Legislature 379
unacademy.com | Download the Unacademy app
Give your feedback here: Link
(d) Council of Ministers is responsible to (d) 1, 2 and 3
the Lok Sabha

29.
Which one of the Schedules of the
26. Consider the following statements: (2015) Constitution of India contains provisions
1. The Legislative Council of a State in India regarding the anti-defection? (2014)
can be larger in size than half of the (a) Second Schedule
Legislative Assembly of that particular (b) Fifth Schedule
State.
(c) Eighth Schedule
2. The Governor of a State nominates the
Chairman of the Legislative Council of (d) Tenth Schedule
that particular State.
Which of the statements given above is/are 30. Which of the following committees is the
correct? largest Committee of the Parliament?
(a) 1 only (2014)

(b) 2 only (a) The Committee on Public Accounts

(c) Both 1 and 2 (b) The Committee on Estimates

(d) Neither 1 nor 2 (c) The Committee on Public Undertakings


(d) The Committee on Petitions

27. When a bill is referred to a joint sitting both 31.


Consider the following statements
the Houses of the Parliament, has to be regarding a No-Confidence Motion in India:
passed by (2015) (2014)

(a) a simple majority of member present 1. There is no mention of No-Confidence


and voting Motion in the Constitution of India.

(b)
three-fourths majority of member 2. A Motion of No-Confidence can be
present and voting introduced in the Lok Sabha only.

(c) two-thirds majority of the House Which of the statements given above is/are
correct?
(d) absolute majority of the House
(a) 1 only
(b) 2 only
28. Consider the following statements: (2015)
(c) Both 1 and 2
1. The Rajya Sabha has no power either to
reject or to amend a Money Bill. (d) Neither 1 nor 2

2. The Rajya Sabha cannot vote on the


Demands for Grants. 32. Consider the following statements: (2013)
3. The Rajya Sabha cannot discuss the The Parliamentary Committee on Public
Annual Financial Statement. Accounts (PAC)
Which of the statements given above is/are 1. Consists of not more than 25 Members
correct? of the Lok Sabha.
(a) 1 only 2. Scrutinises appropriation and finance
(b) 1 and 2 only accounts of Government.

(c) 2 and 3 only 3. Examine the report of CAG.

380 Legislature
unacademy.com | Download the Unacademy app
Give your feedback here: Link
Which of the statements given above is/are 35.
What will follow if a Money Bill is
correct? substantially amended by the Rajya Sabha?
(a) 1 only (2013)

(b) 2 and 3 only (a) The Lok Sabha may still proceed with
the Bill, accepting or not accepting the
(c) 3 only
recommendations of the Rajya Sabha
(d) 1, 2 and 3
(b) The Lok Sabha cannot consider the Bill
further
33. In the context of India, which of the following (c) The Lok Sabha may send the Bill to the
principles is/are implied institutionally in Rajya Sabha for reconsideration
the parliamentary government? (2013)
(d) The President may call a joint sitting for
1. Members of the Cabinet are Members passing the Bill
of the Parliament.
2. Ministers hold the office till they enjoy
36. Consider the following statements: (2013)
confidence in the Parliament.
1. An amendment to the Constitution of
3. The Cabinet is headed by the Head of
India can be initiated by an introduction
the State.
of a Bill in the Lok Sabha only.
Select the correct answer using the code
2. If such an amendment seeks to make
given below:
changes in the federal character of
(a) 1 and 2 only the Constitution, the amendment also
(b) 3 only requires to be ratified by the Legislature
of all the States of India.
(c) 2 and 3 only
Which of the statements given above is/are
(d) 1, 2 and 3
correct?
(a) 1 only
34. Consider the following statements: (2013)
(b) 2 only
1. The Chairman and the Deputy Chairman
(c) Both 1 and 2
of the Rajya Sabha are not the Members
of that House. (d) Neither 1 nor 2

2. While the nominated members of the


two Houses of the Parliament have no 37. The Parliament can make any law for
voting right in the Presidential election, whole or any part of India for implementing
they have the right to vote in the election International Treaties (2013)
of the Vice President.
(a) with the consent of all the States
Which of the statements given above is/are
(b) with the consent of the majority of
correct?
States
(a) 1 only
(c)
with the consent of the States
(b) 2 only concerned
(c) Both 1 and 2 (d) without the consent of any State
(d) Neither 1 nor 2
38. Regarding the Office of the Lok Sabha
Speaker, consider the following statements:

Legislature 381
unacademy.com | Download the Unacademy app
Give your feedback here: Link
(2012) (c) to allow a reduction of specific amount
1. He/She holds the office during the in demand for grant
pleasure of the President. (d) to postpone the proceedings to check
2. He/She need not be a member of the the inappropriate or violent behaviour
House at the time of his/her election on the part of some members
but has to become a member of the
House within six months from the date 41. Which of the following are the methods of
of his/her election. Parliamentary control over public finance
3. If he/she intends to resign, the letter of in India? (2012)
his/her resignation has to be addressed 1. Placing Annual Financial Statement
to the Deputy Speaker. before the Parliament.
Which of the statements given above is/are 2.
Withdrawal of moneys from
correct? Consolidated Fund of India only after
(a) 1 and 2 only passing the Appropriation Bill.
(b) 3 only 3. Provisions of supplementary grants and
(c) 1, 2 and 3 vote-on-account.

(d) None 4. A periodic or at least a mid-year review


of programmes of the Government
against macroeconomic forecasts and
39. Consider the following statements: (2012) expenditure by a Parliamentary Budget
1. Union Territories are not represented in Office.
the Rajya Sabha. 5. Introducing Finance Bill in the
2. It is within the purview of the Chief Parliament.
Election Commissioner to adjudicate Select the correct answer using the code
the election disputes. given below:
3. According to the Constitution of India, (a) 1, 2, 3 and 5 only
the Parliament consists of the Lok
(b) 1, 2 and 4 only
Sabha and the Rajya Sabha only.
(c) 3, 4 and 5 only
Which of the statements given above is/are
correct? (d) 1, 2, 3, 4 and 5

(a) 1 only
(b) 2 and 3 42. Which of the following special powers have
been conferred on the Rajya Sabha by the
(c) 1 and 3
Constitution of India? (2012)
(d) None
(a) To change the existing territory of
a State and to change the name of a
40. In the Parliament of India, the purpose of State.
an Adjournment Motion is: (2012) (b) To pass a resolution empowering the
(a) to allow a discussion on a definite Parliament to make laws in the State
matter of urgent public importance List and to create one or more All India
Services.
(b)
to let opposition members, collect
information from the ministers (c) To amend the election procedure of
the President and to determine the

382 Legislature
unacademy.com | Download the Unacademy app
Give your feedback here: Link
pension of the President after his/her (b) Public Account
retirement. (c) Consolidated Fund of India
(d) To determine the functions of the (d) Deposits and Advances Fund
Election Commission and to determine
the number of Election Commissioners.
46.
When the annual Union Budget is not
passed by the Lok Sabha, (2011)
43. A deadlock between the Lok Sabha and the
Rajya Sabha calls for a joint sitting of the (a) the Budget is modified and presented
Parliament during the passage of (2012) again

1. Ordinary Legislation (b) the Budget is referred to the Rajya


Sabha for suggestions
2. Money Bill
(c) the Union Finance Minister is asked to
3. Constitution Amendment Bill resign
Select the correct answer using the code (d)
the Prime Minister submits the
given below: resignation of Council of Ministers
(a) 1 only
(b) 2 and 3 only 47. What is the difference between “vote-on-
(c) 1 and 3 only account” and “interim budget”? (2011)
(d) 1, 2 and 3 1. The provision of a “vote-on-account” is
44. The authorization for the withdrawal of used by a regular Government, while an
funds from the Consolidated Fund of India “interim budget” is a provision used by
must come from (2011) a caretaker Government.

(a) The President of India 2. A “vote-on-account” only deals with


the expenditure in the Government’s
(b) The Parliament of India Budget, while an “interim budget”
(c) The Prime Minister of India includes both expenditure and receipts.
(d) The Union Finance Minister Which of the statements given above is/are
correct?

45.
All revenues received by the Union (a) 1 only
Government by way of taxes and other (b) 2 only
receipts for the conduct of Government (c) Both 1 and 2
business are credited to the (2011)
(d) Neither 1 nor 2
(a) Contingency Fund of India

Legislature 383
unacademy.com | Download the Unacademy app
Give your feedback here: Link
3 Legislature-Explanation

1. Answer: (c ) administrative unit is the gram sabha and not


Statement 1 is correct: Entry no 4 of the state the cluster of villages at the block level.
list under the seventh schedule identifies Statement 3 is incorrect: The Governor of
Prisons to be an exclusive subject competent each State having Scheduled Areas therein
only for the state legislatures to enact laws on. shall annually, or whenever so required by
Statement 2 is incorrect: The prison act of the President, make a report to the President
1984 does provide an outline which can be regarding the administration of the Scheduled
customised as per the policies of the state Areas in that State and the executive power
government. of the Union shall extend to the giving of
directions to the State as to the administration
**It is in the context of the recent ruling of the of the said areas.
Bihar government allowing for an early release
of an ex legislator convicted for murdering
an Ias officer on duty. While the first half of 4. Answer: (a)
the second statement is partially correct, the Statement 1 is correct
second half is incorrect simply because of the
RULES OF PROCEDURE AND CONDUCT OF
seventh schedule and the fact that provincial
BUSINESS IN LOK SABHA: The election of a
governments did not even exist in 1894.
Deputy Speaker shall be held on such date as
the Speaker may fix, and the Secretary-General
2. Answer: (a) shall send to every member notice of this date.
Statement 1 is incorrect: A finance bill is a part Statement 2 is incorrect
of the budgetary law making process which RULES OF PROCEDURE AND CONDUCT OF
equates the finance bill as that to a money bill. BUSINESS IN LOK SABHA: At any time before
Statement 2 is correct: Rajya sabha can only noon on the day preceding the date so fixed, any
make recommendations and not amendments member may give notice in writing, addressed
or rejections to a money bill. to the Secretary-General, of a motion that
Statement 3 is incorrect: There is no another member be chosen as the Deputy
constitutional compulsion for a joint sitting Speaker of the House and the notice shall
under any circumstances. Since a money be seconded by a third member and shall be
bill and a finance bill are the same type of accompanied by a statement by the member
legislation and both houses don’t have equal whose name is proposed in the notice that the
powers with respect to a money bill, there is member proposed is willing to serve as Deputy
no question of a joint sitting. Speaker.
Statement 3 is correct

3. Answer: (a) RULES OF PROCEDURE AND CONDUCT OF


BUSINESS IN LOK SABHA: The Deputy Speaker
Statement 1 is correct: The fifth schedule under or any other member competent to preside over
article 244(1) of constitution defines “scheduled a sitting of the House under the Constitution or
areas” as such areas as the President may by these rules shall, when so presiding, have the
order declare to be Scheduled areas after same powers as the Speaker when so presiding
consultation with the Governor of the state. and all references to the Speaker in these rules
Statement 2 is incorrect: the lowest

384 Legislature-Explanation
unacademy.com | Download the Unacademy app
Give your feedback here: Link
shall in these circumstances be deemed to be President is to be impeached for violation of
references to any such person so presiding. the Constitution, the charge shall be preferred
Statement 4 is incorrect by either House of Parliament. Impeachment
resolution must be passed by both the houses
RULES OF PROCEDURE AND CONDUCT OF
separately 2/3rd votes of total membership.
BUSINESS IN LOK SABHA: The motions which
have been moved and duly seconded shall
be put one by one in the order in which they 7. Answer: (b)
have been moved, and decided, if necessary, Statement 1 is not correct: Constitution
by division. If any motion is carried, the person Amendment Bills are not treated as Money Bills
presiding shall, without putting later motions, or Financial Bills. Accordingly, the President’s
declare that the member proposed in the recommendation under Articles 117 and 274 of
motion which has been carried, has been the Constitution with regard to these Bills is
chosen as the Deputy Speaker of the House. not asked for.
Statement 2 is correct: Article 368:
5. Answer: (b) Notwithstanding anything in this Constitution,
Statement 1 is not correct: Schedule 10: Parliament may in exercise of its constituent
A nominated member of a House shall be power amend by way of addition, variation or
disqualified for being a member of the House repeal any provision of this Constitution in
if he joins any political party after the expiry of accordance with the procedure laid down in
six months from the date on which he takes this Article.
his seat after complying with the requirements An amendment of this Constitution may be
of Article 99 or, as the case may be, Article 188. initiated only by the introduction of a Bill for
Statement 2 is correct: Schedule 10: If any the purpose in either House of Parliament, and
question arises as to whether a member of a when the Bill is passed in each House by a
House has become subject to disqualification majority of the total membership of that House
under this Schedule, the question shall be and by a majority of not less than two-thirds of
referred for the decision of the Chairman or, as the members of that House present and voting,
the case may be, the Speaker of such House it shall be presented to the President who
and his decision shall be final. shall give his assent to the Bill and thereupon
the Constitution shall stand amended in
accordance with the terms of the Bill.
6. Answer: (b)
Statement 3 is correct: In case of any
Statement 1 is incorrect: Article 352: Every
disagreement between the two Houses of
Proclamation issued under this article shall be
Parliament on a Constitution Amendment Bill,
laid before each House of Parliament
there cannot be a joint sitting of the Houses
Article 356: Every Proclamation under this of Parliament on the Bill as article 368 of the
article shall be laid before each House of Constitution requires each House to pass the
Parliament. Bill by the prescribed special majority.
Article 360: A Proclamation issued under this
shall be laid before each House of Parliament.
8. Answer: (d)
Statement 2 is correct: Article 75(3): The
Option (d) is correct: The Macro Economic
Council of Ministers shall be collectively
Framework Statement is presented to
responsible to the House of the People.
the Parliament as mandated by the Fiscal
Statement 3 is incorrect: Article 61: When a Responsibility and Budget Management Act,

Legislature-Explanation 385
unacademy.com | Download the Unacademy app
Give your feedback here: Link
2003. The FRBM Rules came into force from be a member of the Legislature of a state can
July 5, 2004. be made a Minister even when they are not
Every year, a macro-economic framework the members of the state Legislature (Article
statement is presented before Parliament at 164). Voter eligibility is not mentioned in
the time of the Union Budget presentation. This the Constitution. This question is related to
is under Section 3(5) of the Fiscal Responsibility checking the concept rather than fact.
and Budget Management (FRBM) Act, 2003. Statement 2 is not correct: Section 8 of RPA,
The FRBM Act instructs the government to 1951 states the grounds for disqualification
assess growth prospects for the economy with on conviction of certain offences pursuant to
regards to specific underlying assumptions. Section 8 of RPA, 1951, in case the candidate
The statement contains an overview of is convicted of certain offences specified and
the economy. This includes an assessment sentenced for imprisonment for not less than
regarding the GDP growth rate, fiscal balance two years then the person will be disqualified
of the central government and the external to contest the election from the date of the
sector balance of the economy. conviction and will continue to be disqualified
for further six years even after the release.

9. Answer: (b)
11. Answer: (b)
Option (b) is correct: In a Parliamentary System
of Government, the Government is responsible Option (b) is correct: Both Rajya Sabha and
to the Parliament and can be removed by it. Lok Sabha have the powers to amend the
The Government is responsible collectively to Constitution. To amend the Constitution a bill
the Lok Sabha and it stays in power as long may originate either in the Rajya Sabha or in
as it enjoys the confidence of the House. The the Lok Sabha. The amendment cannot be
Government can be removed by the House if a effective unless it is passed by both the Houses
No-Confidence Motion is passed. with the required majority. The procedure of
amendment in the Constitution is laid down
There are two forms of Government,
in Part XX (Article 368) of the Constitution of
Parliamentary and Presidential. In the
India.
Parliamentary System, the political party
winning the majority seats in the Parliament Additional Information:
makes the Government and elects a person y With regards to the creation of All India
from among themselves as the Prime Minister Services, Rajya Sabha has special powers
who is the Head of the Government. In this (Article 312) such that if Rajya Sabha
system, the legislative and executive body is passing a resolution by a majority of
of Government are closely related, while the 2/3rd of the members present and voting,
judiciary is independent of the other two bodies Parliament may, by law, create one or more
of Government. The executive is accountable All India Services in national interest.
to the legislature. y As per Article 75, the Government (Council
of Ministers) is collectively responsible to
10. Answer: (a) the Lok Sabha. Therefore, Lok Sabha has
greater power as compared to Rajya Sabha.
Statement 1 is correct: One who is eligible to
vote is not necessarily eligible to be a minister y Cut Motions, which are meant to reduce
(for e.g., on grounds of age). Question used the amount allocated to the ministries in
the term “a voter can” and not “every voter”, the demand for grants, can be made only
here many voters who meet the eligibility to in Lok Sabha. Rajya Sabha can only make
recommendations in this regard.

386 Legislature-Explanation
unacademy.com | Download the Unacademy app
Give your feedback here: Link
12. Answer: (a) 13. Answer: (b)
Statement 1 is correct: The Parliament Statement 1 is not correct: The first Lok Sabha
(Prevention of Disqualification) Act, 1959 elections in India were held in 1951-52. Indian
exempts several posts from disqualification on National Congress won with the sweeping
the grounds of ‘Office of Profit’. The posts are: majority scoring 364 out of 489 seats. The
y Ministers in the Government of India and Communist Party of India emerged as the
States. largest party in the opposition winning a total
of 16 seats. The Swatantra Party on the other
y Whips in the Parliament.
hand was formed in 1959. It was founded by C.
y Parliamentary Secretary and Under Rajagopalachari who was dissatisfied with the
Secretaries. Indian National Congress’s socialist agenda.
y Officers of the National Cadet Corps and Statement 2 is correct: In parliamentary form
the Territorial Army. government the Leader of the Opposition has
y Chairman and Members of Advisory a significant role to play. His function involves
Committees set up by the Government. providing constructive criticism of government
policies. In 1969, when Indira Gandhi was the
Statement 2 is correct: This act has been
Prime Minister, the Congress split to form the
amended five times 1960, 1992, 1993, 2006 and
Indian National Congress (Requisitionists) and
in 2013.
the Indian National Congress (Organisation). The
Statement 3 is not correct: The word ‘office’ Leader of INC(O), Ram Subhag Singh, became
has not been defined in the Constitution the first person to be formally recognised as
or the Representation of the People Act of LoP in the Lok Sabha. Leaders of Opposition in
1951. But the definition has evolved over the the Lok Sabha and Rajya Sabha were accorded
years with interpretations made in various statutory recognition in 1977.
court judgments. An office of profit has been
Statement 3 is not correct: The leader of
interpreted to be a position that brings to the
the largest opposition party having not less
office-holder some financial gain, or advantage,
than one-tenth seats of the total strength of
or benefit. The amount of such profit is
the house is recognised as the Leader of the
immaterial.
Opposition in that house.
In 1964, the Supreme Court ruled that the test
for determining whether a person holds an office Elimination Technique: The Swatantra
of profit is the test of appointment. Several Party was formed in 1959 and first Lok
factors are considered in this determination Sabha elections were held in 1951-52.
which are as follows: The leader of the largest opposition party
having not less than one-tenth seats of the
y whether the government is the appointing
total strength of the house is recognised
authority,
as the Leader of the Opposition in that
y whether the government has the power to house.
terminate the appointment,
y whether the government determines the
14. Answer: (a)
remuneration,
Statement 1 is correct: The Ninth Schedule
y what is the source of remuneration, and
contains a list of central and state laws. The
y the power that comes with the position. Schedule became a part of the Constitution in
1951 when the document was amended for the
first time. It was created by the new article
31B, which along with 31A were inserted in the

Legislature-Explanation 387
unacademy.com | Download the Unacademy app
Give your feedback here: Link
Constitution to protect laws related to agrarian recognized that the Right to Privacy includes
reforms and for abolishing the Zamindari autonomy over personal decisions, as in beef
system. At present there are around 284 laws consumption; bodily integrity, as in reproductive
in the Ninth Schedule. Thus, the Parliament can rights; and protection of personal information,
place a particular law in the Ninth Schedule of as in the privacy of health records.
the Constitution of India.
Statement 2 is not correct: In a significant 17. Answer: (c)
judgement delivered in I.R. Coelho case (2007),
the Supreme Court ruled that there could not Option (c) is correct: Article 110 of the
be any blanket immunity from judicial review of Constitution deals with the definition of Money
laws included in the Ninth Schedule. The court Bills. It states that a bill is deemed to be a
held that judicial review is a ‘basic feature’ of Money Bill if it contains ‘only’ provisions dealing
the Constitution, and it could not be taken with all or any of the following matters:
away by putting a law in the Ninth Schedule. y The imposition, abolition, remission,
alteration, or regulation of any tax.

15. Answer: (b) y The regulation of the borrowing of money


by the Union government.
Parliament performs varied, complex, and
voluminous functions. But it neither has the y The custody of the Consolidated Fund of
adequate time nor necessary expertise to make India or the Contingency Fund of India, the
a detailed scrutiny of all legislative measures payment of money into or the withdrawal
and other matters. Therefore, it is assisted by of money from any such fund.
a number of Parliamentary Committees in the y The appropriation of money out of the
discharge of its duties. Consolidated Fund of India.
Option (b) is correct: The Committee on y Declaration of any expenditure charged on
Subordinate Legislation examines and reports the Consolidated Fund of India or increasing
to the House whether the powers to make the amount of any such expenditure.
regulations, rules, sub-rules delegated by the y The receipt of money on account of the
Parliament or conferred by the Constitution to Consolidated Fund of India or the public
the Executive are being properly exercised by account of India or the custody or issue of
it. In both the Houses, the Committee consists such money, or the audit of the accounts of
of 15 members. It was constituted in 1953. the Union or of a state.
y Any matter incidental to any of the matters
16. Answer: (c) specified above.
Option (c) is correct: The Supreme Court in the
case of Puttaswamy vs Union of India (2017) 18. Answer: (a)
delivered the landmark judgement where it
Statement 1 is correct: The Speaker is elected
upheld the Right to Privacy as a Fundamental
by the assembly itself from amongst its
Right protected under Part III of the Indian
members. Usually, the Speaker remains in office
Constitution. The Supreme Court held that
during the life of the Assembly. However, he/
“Right to Privacy” is an integral part of Right to
she vacates his/her office earlier if he ceases
Life and Personal Liberty guaranteed in Article
to be a member of the Assembly, if he resigns
21 of the Constitution.
by writing to the deputy speaker, or if he is
The Right to Privacy was reinforced by removed by a resolution passed by a majority
the concurring opinions of the judges who of all the then members of the assembly. Such

388 Legislature-Explanation
unacademy.com | Download the Unacademy app
Give your feedback here: Link
a resolution can be moved only after giving 14 the voice of the people, where the members
days advance notice. are elected directly by people of India.
Statement 2 is not correct: Whenever the
Legislative Assembly is dissolved, the Speaker 21. Answer: (d)
does not vacate his office and continues till
the newly elected Legislative Assembly meets. Statement 1 is not correct: A Private member’s
bill is a bill introduced by any Member of
Parliament in either House of Parliament other
19. Answer: (d) than a minister. Its introduction requires one
Statement 1 is not correct: India has the month’s notice. Its rejection by the House has
first-past-the-post (FPTP) system (simple no implication on the parliamentary confidence
majority system). In this voting method, the in the government. A bill introduced by a
candidate with the highest number of votes Minister is known as a public bill.
in a constituency is declared the winner. This Statement 2 is not correct: Around 14 private
system is used in India in the direct elections members bills have been passed since 1952.
to the Lok Sabha and State Legislative The last private member bill to become a
Assemblies. It does not always allow for a truly law was way back in 1970. Most recently (as
representative mandate, as the candidate could per 2018 records), the Rights of Transgender
win despite securing less than half the votes in Persons Bill, 2014, was passed by the Rajya
an election. In 2014, the National Democratic Sabha in 2015, but remains stuck in the Lok
Alliance won 336 seats with only 38.5% of the Sabha. It was brought in by DMK MP Tiruchi
popular vote. The winning candidate does not Siva, the bill has not been converted into a law.
need to get at least 50 per cent of the votes
polled to be declared elected.
22. Answer: (c)
Statement 2 is not correct: Up to the 10th
Lok Sabha, both the Speaker and the Deputy The Parliament has laid down the following
Speaker were usually from the ruling party. After additional qualifications in the Representation
the 11th Lok Sabha, there was a consensus (no of People Act (1951).
provisions) that the Speaker comes from the y He must be registered as an elector for
ruling party (or ruling alliance), and the post of a parliamentary constituency. This is the
Deputy Speaker goes to the main opposition same in the case of both the Rajya Sabha
party. Currently, in the 17th Lok Sabha, there and the Lok Sabha. The requirement that
is no Deputy Speaker. Hence, it is just a a candidate contesting an election to the
convention and not a provision. Rajya Sabha from a particular state should
be an elector in that state was dispensed
with in 2003. In 2006, the Supreme Court
20. Answer: (c) upheld the Constitutional validity of this
Article 74 states that there shall be a Council change. (Option (c) is correct)
of Ministers with the Prime Minister to aid and y He must be a member of a scheduled
advise the President. caste or scheduled tribe in any state or
Option (c) is correct: The fundamental principle union territory if he wants to contest a seat
underlying the working of the parliamentary reserved for them. However, a member of
system of government is a principle of collective scheduled castes or scheduled tribes can
responsibility. Article 75 clearly states that the also contest a seat not reserved for them.
Council of Ministers is collectively responsible
to the Lok Sabha, whereas the Lok Sabha is

Legislature-Explanation 389
unacademy.com | Download the Unacademy app
Give your feedback here: Link
23. Answer: (b) supported by two-thirds of the members
Statement 1 is not correct: Prorogation not only present and voting.
terminates a sitting but also a session of the y The Parliament acquires the power to
House. Prorogation does not affect the bills or legislate with respect to matters in the
any other business pending before the House. State List, while a proclamation of national
However, all pending notices (other than those emergency is in operation.
for introducing bills) lapse on prorogation and y When the legislatures of two or more states
fresh notices have to be given for the next pass resolutions requesting the Parliament
session. to enact laws on a matter in the State List,
Statement 2 is correct: In case of dissolution then the Parliament can make laws for
of Lok Sabha (Rajya Sabha being the permanent regulating that matter. A law so enacted
house is not subjected to dissolution) the applies only to those states which have
position with respect to lapsing of bills is as passed the resolutions.
follows: y The Parliament can make laws on any
y A bill pending in the Lok Sabha lapses matter in the State List for implementing
(whether originating in the Lok Sabha or the international treaties, agreements, or
transmitted to it by the Rajya Sabha). conventions.
y A bill passed by the Lok Sabha but pending y When the President’s rule is imposed in a
in the Rajya Sabha lapses. state, the Parliament becomes empowered
y A bill not passed by the two Houses due to make laws with respect to any matter in
to disagreement and if the president has the State List in relation to that state.
notified the holding of a joint sitting before
the dissolution of Lok Sabha, does not 25. Answer: (d)
lapse.
India has a Parliamentary System of
y A bill pending in the Rajya Sabha but not Government. Article 74 and Article 75 deal with
passed by the Lok Sabha does not lapse. the Parliamentary System at the Centre and
y A bill passed by both Houses but pending Article 163 and Article 164 deal with the States.
assent of the president does not lapse. The features of Parliamentary Government in
y A bill passed by both Houses but returned India are:
by the president for reconsideration of y Presence of nominal and real executives.
Houses does not lapse. y Majority party rule.
y Collective responsibility of the executive
24. Answer: (d) to the legislature: This is the bedrock
Option (d) is correct: Constitution empowers principle of parliamentary government. The
the Parliament to make laws on any matter ministers are collectively responsible to the
enumerated in the State List under the Parliament in general and to the Lok Sabha
following five extraordinary circumstances: in particular (Article 75). The principle of
collective responsibility implies that the
y If the Rajya Sabha declares that it is
Lok Sabha can remove the ministry (i.e.,
necessary in the national interest that
Council of Ministers headed by the Prime
Parliament should make laws on a matter
Minister) from office by passing a vote of no
in the State List, then the Parliament
confidence. [Option (d) is correct]
becomes competent to make laws on
that matter. Such a resolution must be y Membership of the Ministers in the
Legislature.

390 Legislature-Explanation
unacademy.com | Download the Unacademy app
Give your feedback here: Link
y Leadership of the Prime Minister or the the other House:
Chief Minister. y if the bill is rejected by the other House.
y Dissolution of the Lower House (Lok Sabha y if the Houses have finally disagreed as to
or Assembly). the Amendments to be made in the Bill.
y if more than six months have elapsed from
26. Answer: (d) the date of the receipt of the Bill by the
Statement 1 is not correct: The maximum other House without the Bill being passed
strength of the Council is fixed at one-third by it.
of the total strength of the Assembly, and the Option (a) is correct: The President can
minimum strength is fixed at 40. The size of the summon both the Houses to meet in a Joint
Council depends on the size of the Assembly Sitting for the purpose of deliberating and
of the concerned state. voting on the Bill. The provision of Joint Sitting
There is no uniformity in the organisation of is applicable to ordinary Bills or Financial Bills
State Legislatures. At present (2021), only six only and not to Money Bills or Constitutional
states have two Houses (bicameral). These Amendment Bills. In the case of a Money Bill,
are Andhra Pradesh, Telangana, Uttar Pradesh, the Lok Sabha has overriding powers, while
Bihar, Maharashtra and Karnataka. The Jammu a Constitutional Amendment Bill must be
and Kashmir Legislative Council was abolished passed by each House separately. The Bill in a
by the Jammu and Kashmir Reorganisation Act, Joint Sitting is passed by a simple majority of
2019. members present and voting.

Statement 2 is not correct: The Chairman is Additional Information:


elected by the Council itself from amongst its y The Speaker of Lok Sabha presides over
members. The Chairman vacates his office in a Joint Sitting of the two Houses and the
any of the following three cases: Deputy Speaker, in his absence. If the
y if he ceases to be a member of the Council Deputy Speaker is also absent, the Deputy
Chairman of Rajya Sabha presides.
y if he resigns by writing to the Deputy
Chairman y The quorum to constitute a Joint Sitting is
one-tenth of the total number of members
y if he is removed by a resolution passed by
of both the Houses.
a majority of all the then members of the
Council. Such a resolution can be moved
only after giving 14 days advance notice. 28. Answer: (b)
As a Presiding Officer, the powers and functions The Constitution has defined the relative roles
of the Chairman in the Council are similar to or position of both the Houses of Parliament
those of the Speaker in the Assembly. about the enactment of the Budget in the
following way:

27. Answer: (a) y A money bill or finance bill dealing with


taxation cannot be introduced in the Rajya
Joint Sitting is an extraordinary machinery
Sabha, it must be introduced only in the
provided by the Constitution (Article 108) to
Lok Sabha.
resolve a deadlock between the two Houses
over the passage of a Bill. A deadlock is y The Rajya Sabha has no power either to
deemed to have taken place under any one of reject or to amend a Money Bill. It should
the following three situations after a bill has return the Money Bill (or Finance Bill) to
been passed by one House and transmitted to the Lok Sabha within fourteen days. The

Legislature-Explanation 391
unacademy.com | Download the Unacademy app
Give your feedback here: Link
Lok Sabha can either accept or reject the 30. Answer: (b)
recommendations made by Rajya Sabha in Option (b) is correct: The Committee of
this regard. (Statement 1 is correct) Estimates is the largest committee of the
y The Rajya Sabha has no power to vote on Parliament. All members of the Committee are
the Demand for Grants; it is the exclusive from Lok Sabha only. The Rajya Sabha has no
privilege of the Lok Sabha. (Statement 2 is representation in this Committee.
correct) y The members of the Committee are elected
Statement 3 is not correct: Article 112 states by the Lok Sabha every year from amongst
that the President shall, in respect of every its own members, according to the
financial year, cause to be laid before both the principles of proportional representation by
Houses of Parliament a statement of estimated means of a single transferable vote. Thus,
receipts and expenditure of the Government all parties get due representation in it.
of India for that year, which implies that y The term of office is one year.
Rajya Sabha can discuss the Annual Financial
Statement. y A Minister cannot be elected as a Member
of the Committee.
y The Chairman of the Committee is appointed
29. Answer: (d) by the Speaker from amongst its Members
Option (d) is correct: The Anti-Defection Law and he is invariably from the Ruling Party.
sought to prevent such political defections y The function of the Committee is to
which may be due to reward of office or other examine the estimates included in the
similar considerations. The Tenth Schedule Budget and suggest ‘economies’ in public
was inserted in the Constitution in 1985. It lays expenditure. Hence, it has been described
down the process by which legislators may be as a ‘Continuous Economy Committee’.
disqualified on grounds of defection by the
Presiding Officer of a legislature based on a Additional Information:
petition by any other Member of the House. y Public Accounts Committee: At present,
y A legislator is deemed to have defected if he the Committee consists of 22 members (15
either voluntarily gives up the membership from the Lok Sabha and 7 from the Rajya
of his party or disobeys the directives of Sabha). The function of the Committee is
the party leadership on a vote and has not to examine the annual audit reports of the
received pardon from the political party Comptroller and Auditor General of India
within fifteen days (CAG), which are laid before the Parliament
by the President.
y The Law applies to both Parliament and
State Assemblies. y Committee on Public Undertaking: It has
22 members (15 from the Lok Sabha and
Additional Information: 7 from the Rajya Sabha). Function of the
y Second Schedule contains provisions Committee is to examine the reports and
related to allowances, privileges, and accounts of public undertakings.
emoluments of some constitutional bodies. y Committee on Petitions: It examines
y Fifth Schedule contains provision in relation petitions on Bills and on matters of general
to administration and control of Scheduled public importance. It also entertains
Areas and Scheduled Tribes. representations from individuals and
y Eighth Schedule contains 22 languages associations on matters pertaining to Union
recognised in the Constitution. Subjects.

392 Legislature-Explanation
unacademy.com | Download the Unacademy app
Give your feedback here: Link
31. Answer: (c) the audit report of CAG on it. (Statement 3
Statement 1 is correct: The Indian Constitution is Correct)
does not mention No-Confidence Motion. y To examine the accounts of state
However, Article 75 specifies that the Council corporations, trading concerns and
of Ministers shall be collectively responsible manufacturing projects and the audit
to the House of the People. The implication of report of CAG on them (except those public
this Article is that the majority of Lok Sabha undertakings which are allotted to the
MPs must not be against the Prime Minister Committee on Public Undertakings).
and his Cabinet. y To examine the accounts of autonomous
Statement 2 is correct: Article 75 of the and semi-autonomous bodies, the audit of
Constitution says that the Council of Ministers which is conducted by the CAG.
shall be collectively responsible to the Lok y To consider the report of the CAG relating
Sabha. It means that the ministry stays in to the audit of any receipt or to examine
office so long as it enjoys the confidence of the accounts of stores and stocks.
the majority of the members of the Lok Sabha.
In other words, the Lok Sabha can remove the y To examine the money spent on any service
ministry from office by passing a No-Confidence during a financial year in excess of the
Motion. Thus, it can only be introduced in the amount granted by the Lok Sabha for that
Lok Sabha. The Motion needs the support of at purpose.
least 50 members to be admitted.
y Article 118 of the Constitution permits both 33. Answer: (a)
Houses of Parliament to make its own rules Statement 1 is correct: Membership of the
for its functioning. Lok Sabha’s Rule 198 Ministers in the legislature is such that they
mentions the procedure for a Motion of No- are members of both the legislature and the
Confidence. executive. This means that a person cannot
be a Minister without being a member of the
32. Answer: (b) Parliament. The Constitution stipulates that a
Minister who is not a Member of the Parliament
Statement 1 is not correct: Public Accounts for a period of six consecutive months ceases
Committee was set up first in 1921 under the to be a Minister.
provisions of the Government of India Act
of 1919 and has since been in existence. At Statement 2 is correct: Collective responsibility
present, it consists of 22 members (15 from of the executive to the legislature is the bedrock
the Lok Sabha and 7 from the Rajya Sabha). The principle of parliamentary government. The
members are elected by the Parliament every Ministers are collectively responsible to the
year from amongst its members according to Parliament in general and to the Lok Sabha
the principle of proportional representation by in particular (Article 75). They act as a team,
means of the single transferable vote. and swim and sink together. The principle of
collective responsibility implies that the Lok
Functions of the Committee are: Sabha can remove the Ministry (i.eCouncil of
y To examine the appropriation accounts Ministers headed by the Prime Minister) from
and the finance accounts of the Union office by passing a vote of ‘No Confidence’.
government and any other accounts laid Statement 3 is not correct: The Cabinet is
before the Lok Sabha. (Statement 2 is headed by the Prime Minister and not by the
Correct) Head of the State (the President).
y Scrutinise the appropriation accounts and

Legislature-Explanation 393
unacademy.com | Download the Unacademy app
Give your feedback here: Link
34. Answer: (b) y The procedure to pass a Money Bill in
Unlike the Speaker (who is a member of the Parliament is a key provision limiting the
House), the Chairman is not a member of the powers of Rajya Sabha compared to Lok
House. The Chairman, however, can cast a vote Sabha. Any Bill other than a Money Bill
in the case of an equality of votes. cannot become law unless both Houses
agree to it with or without amendments.
Statement 1 is not correct: The Deputy
Chairman is elected by the Rajya Sabha itself
from amongst its members. Whenever the 36. Answer: (d)
office of the Deputy Chairman falls vacant, the Statement 1 is not correct: Rajya Sabha shares
Rajya Sabha elects another member to fill the equal status with Lok Sabha in matters of
vacancy. introduction and passage of Constitutional
Statement 2 is correct: The Electoral College amendment Bills, hence a Bill initiating
for the election of the Vice President consists amendment to the Constitution can be
of both elected and nominated members of introduced in Rajya Sabha also.
the Parliament (in the case of the President, Statement 2 is not correct: Those provisions
only elected members). of the Constitution which are related to the
The President is elected by members of federal structure of the polity can be amended
electoral college consisting of: by a special majority of the Parliament and
y The elected members of both the Houses also with the consent of half of the State
of Parliament. Legislatures by a simple majority. If one or some
or all the remaining States take no action on
y The elected members of the Legislative
the Bill, it does not matter; the moment half of
Assemblies of the States.
the States give their consent, the formality is
y The elected members of the Legislative completed. There is no time limit within which
Assemblies of the Union Territories of Delhi the States should give their consent to the Bill.
and Puducherry.
Thus, the nominated members of the Houses
37. Answer: (d)
have no vote in Presidential elections.
Option (d) is correct: The Constitution
empowers the Parliament to make laws for the
35. Answer: (a) whole or any part of India for implementing
Option (a) is correct: Under Article 109 (1), a International Treaties. It can make laws on
Money Bill cannot be introduced in Rajya Sabha. the subjects enumerated in the State List
Once passed by Lok Sabha, it is sent to Rajya (which at present has 59 subjects, originally
Sabha along with the Speaker’s certificate that 66 subjects) under the following five abnormal
it is a Money Bill for its recommendations. circumstances:
However, Rajya Sabha can neither reject nor y When Rajya Sabha passes a resolution to
amend the Bill, and must return it within 14 days, that effect.
after which Lok Sabha may choose to accept
y When a proclamation of National Emergency
or reject all or any of its recommendations. In
is in operation.
either case, the Bill is deemed to have been
passed by both Houses. Under Article 109(5), y When two or more states make a joint
if Rajya Sabha fails to return the Bill to Lok request to the Parliament.
Sabha within 14 days, it is deemed to have y When necessary to give effect to
been passed anyway. International Agreements, Treaties and
Conventions.

394 Legislature-Explanation
unacademy.com | Download the Unacademy app
Give your feedback here: Link
y When President’s Rule is in operation in the 39. Answer: (d)
state. Statement 1 is not correct: Out of the seven
All the Ordinances issued by the President Union Territories, two (Delhi and Puducherry)
(during the recess of the Parliament) must be have representation in Rajya Sabha. The
approved by the Parliament within six weeks populations of other five Union Territories
after its reassembly and consent of the states are too small to have any representative
is not required. in the Rajya Sabha. The representatives of
each Union Territory in the Rajya Sabha are
indirectly elected by members of an electoral
38. Answer: (b) college specially constituted for the purpose.
Statement 1 is not correct: The Speaker can This election is also held in accordance with
be removed from Office only on a resolution the system of proportional representation by
of the House passed by a majority of all the means of the single transferable vote.
then members of the House. Such a resolution Statement 2 is not correct: Representative
has to satisfy some conditions like, it should of the People’s Act, 1951 vested the power to
be specific with respect to the charges, and adjudicate the disputes regarding election to
it should not contain arguments, inferences, the Parliament or any State Assembly with
ironical expressions, imputations or defamatory the High Court of the concerned state. Also,
statements, etc. Not only these, but discussions all doubts and disputes in connection with
should also be confined to charges referred to election of the President are inquired into and
in the resolution. It is also mandatory to give a decided by the Supreme Court whose decision
minimum of 14 days’ notice of the intention to is final.
move the resolution.
Statement 3 is not correct: Under the
Statement 2 is not correct: The Speaker is Constitution, the Parliament of India consists
elected by the Lok Sabha from amongst its of three parts viz, the President, the Council of
members, therefore the Speaker has to be a States, and the House of the People. Though
member of Parliament to become eligible to the President of India is not a member of
contest election for the post of Speaker. either House of Parliament or does not sit in
Statement 3 is correct: The Speaker holds the Parliament to attend its meetings, he is an
Office from the date of his/her election till integral part of the Parliament. This is because
immediately before the first meeting of the Lok a Bill passed by both the Houses of Parliament
Sabha after the dissolution of the one to which cannot become law without the President’s
he/she was elected. He/She is eligible for re- assent.
election. On the dissolution of the Lok Sabha,
although the Speaker ceases to be a member
of the House, he/she does not vacate his/her 40. Answer: (a)
Office. The Speaker may, at any time, resign Option (a) is correct: Adjournment Motion
from Office by writing under his/her hand to is introduced in the Parliament to draw the
the Deputy Speaker. attention of the House to a definite matter
of urgent public importance and needs the
Elimination Technique: The Speaker does support of 50 members to be admitted. As it
not hold the office during the pleasure of interrupts the normal business of the House,
the President. Hence, statement 1 is not it is regarded as an extraordinary device. It
correct. He is elected by the Lok Sabha involves an element of censure against the
from amongst its members so 2 is also government and hence Rajya Sabha is not
incorrect. permitted to make use of this device. The

Legislature-Explanation 395
unacademy.com | Download the Unacademy app
Give your feedback here: Link
discussion on an Adjournment Motion should is made by the Parliament under extraordinary
last for not less than two hours and thirty or special circumstances. It is granted when
minutes. the amount authorised by the Parliament
The right to move a motion for an adjournment through the Appropriation Act for a particular
of the business of the House is subject to the service for the current financial year is found
following restrictions: to be insufficient for that year.

y It should raise a matter which is definite, Statement 5 is correct: The Finance Bill is
factual, urgent and of public importance. introduced to give effect to the financial
proposals of the Government of India for
y It should not cover more than one matter.
the following year. It is subjected to all the
y It should be restricted to a specific matter conditions applicable to a Money Bill. Unlike the
of recent occurrence and should not be Appropriation Bill, the amendments (seeking
framed in general terms. to reject or reduce a tax) can be moved in the
y It should not raise a question of privilege. case of the Finance Bill.

y It should not revive discussion on a matter


that has been discussed in the same 42. Answer: (b)
session.
Option (b) is correct: The Rajya Sabha is the
y It should not deal with any matter that is Upper House (Second Chamber or House of
under adjudication by court. Elders). It represents the states and union
y It should not raise any question that can be territories of the Indian union. Due to its
raised on a distinct motion. federal character, the Rajya Sabha has been
given two exclusive or special powers that are
not enjoyed by the Lok Sabha:
41. Answer: (a)
y It can authorise the Parliament to make a
Statement 1 is correct: The Constitution law on a subject enumerated in the State
refers to the budget as the ‘annual financial List (Article 249).
statement’. In other words, the term ‘budget’
y It can authorise the Parliament to create
has nowhere been used in the Constitution.
new All-India Services common to both the
The President shall in respect of every financial
Centre and states (Article 312).
year cause to be laid before both the Houses of
Parliament a statement of estimated receipts
and expenditure of the Government of India for 43. Answer: (a)
that year.
Joint sitting is an extraordinary machinery
Statement 2 is correct: The Constitution provided by the Constitution to resolve a
states that ‘no money shall be withdrawn from deadlock between the two Houses over the
the Consolidated Fund of India except under passage of a bill. A deadlock is deemed to have
appropriation made by law’. Accordingly, an taken place under any one of the following
Appropriation Bill is introduced to provide for three situations after a bill has been passed by
the appropriation, out of the Consolidated one House and transmitted to the other House:
Fund of India, all money required to meet:
y if the bill is rejected by the other House.
y The grants voted by the Lok Sabha.
y if the Houses have finally disagreed as to
y The expenditure charged on the the amendments to be made in the bill; or
Consolidated Fund of India.
y if more than six months have elapsed from
Statement 3 is correct: Supplementary Grant the date of the receipt of the bill by the

396 Legislature-Explanation
unacademy.com | Download the Unacademy app
Give your feedback here: Link
other House without the bill being passed credited to the Consolidated Fund of India.
by it. All the legally authorised payments on behalf
Option (a) is correct: In the above three of the Government of India are made out of
situations, the President can summon both the this fund.
Houses to meet in a joint sitting for the purpose Additional Information:
of deliberating and voting on the bill. It must
be noted here that the provision of joint sitting y Contingency Fund of India: The Contingency
is applicable to ordinary bills or financial bills Fund of India is the emergency fund for the
only and not to money bills or Constitutional Nation. Constituted under Article 267(1) of
amendment bills. In the case of a money bill, the Indian Constitution, the Contingency
the Lok Sabha has overriding powers, while a Fund of India is used at a time when there
Constitutional amendment bill must be passed is a crisis in the nation — a natural calamity,
by each House separately. for instance — and money is required to deal
with it. The Union government has its own
contingency fund with a corpus of Rs 500
44. Answer: (b) crore. States can also opt to have their own
Consolidated Fund of India is a fund to which Contingency Funds. The Contingency Fund
all Government receipts are credited, and all of the Union Government is at the disposal
payments are debited. In other words, of the President of India, who releases the
funds on request of the Union Cabinet,
y All revenues received by the Government which later gets approval from Parliament.
of India, A Parliament approval is mandatory.
y All loans raised by the Government by the y Public Account: Public Account of India
issue of treasury bills, loans or ways and accounts for flows for those transactions
means of advances; and where the Government is merely acting
y All money received by the Government in as a banker. This fund was constituted
repayment of loans forms the Consolidated under Article 266 (2) of the Constitution. It
Fund of India. accounts for flows for those transactions
Option (b) is correct: All the legally authorised where the government is merely acting as
payments on behalf of the Government of India a banker. Examples of those are provident
are made out of this fund. No money out of this funds, small savings and so on. These funds
fund can be appropriated (issued or drawn) do not belong to the Government. They
except in accordance with a parliamentary law. have to be paid back at some time to their
rightful owners. Because of this nature
Elimination Technique: The authorization of the fund, expenditures from it are not
for the withdrawal of funds from the required to be approved by the Parliament.
Consolidated Fund of India comes from
Parliament of India hence option (b) is
correct. 46. Answer: (d)
Option (d) is correct: When the annual Union
Budget is not passed by the Lok Sabha, the
45. Answer: (c)
Prime Minister submits the resignation of the
Option (c) is correct: Article 266 of the Indian Council of Ministers. If the Budget is not passed
Constitution provides for the Consolidated under any condition in Parliament, then it can
Fund of India. All revenues received by the Union be understood that the ruling party is in the
Government by way of taxes and other receipts minority. Technically it means the Government
for the conduct of Government business are has lost the confidence vote in the Lok Sabha

Legislature-Explanation 397
unacademy.com | Download the Unacademy app
Give your feedback here: Link
and it has to resign. In Indian history, this has estimation. Vote on account is passed every
not happened so far. year, used by both the regular and caretaker
Government.

47. Answer: (b) Statement 2 is correct: A “Vote-on-account”


deals only with expenditure while interim
Statement 1 is not correct: The Constitution budget both revenue and expenditure. An
has authorised the Lok Sabha to make any Interim Budget is similar to a Union Budget.
grant in advance in respect to the estimated In the Interim Budget, the ruling Government
expenditure for a part of the financial year, tables estimates of its expenditure, revenue,
pending the completion of the voting of the fiscal deficit and financial performance and
demands for grants and the enactment of the projections for the upcoming financial year.
Appropriation Bill. This provision is known as The ruling Government at the end of its tenure,
the ‘vote on account’. It is passed (or granted) presents an Interim Budget for three to four
after the general discussion on the Budget is months so as to keep the country running
over. It is generally granted for two months for unhampered.
an amount equivalent to one-sixth of the total

398 Legislature-Explanation
unacademy.com | Download the Unacademy app
Give your feedback here: Link
4 Executive

1. Consider the following statements : (2023) as compared to Arunachal Pradesh.


1. If the election of the President of India How many of the above statements are
is declared void by the Supreme Court correct?
of India, all acts done by him/her in the (a) Only one
performance of duties of his/her office
of President before the date of decision (b) Only two
become invalid. (c) Only three
2. Elections for the post of the President of (d) All four
India can be postponed on the grounds
that some Legislative Assemblies have
3. Consider the following statements : (2022)
been dissolved and elections are yet to
take place. 1. The Constitution of India classifies
the ministers into four ranks viz.
3. When a Bill is presented to the President
Cabinet Minister, Minister of State with
of India, the Constitution prescribes
Independent Charge, Minister of State
time limits within which he/she has to
and Deputy Minister.
declare his/ her assent.
2. The total number of ministers in the
How many of the above statements are
Union Government, including the Prime
correct?
Minister, shall not exceed 15 percent of
(a) Only one the total number of members in the Lok
(b) Only two Sabha.
(c) All three Which of the statements given above is/are
(d) None correct ?

2. Consider the following statements in (a) 1 only


respect of election to the President of (b) 2 only
India: (2023) (c) Both 1 and 2
1. The members nominated to either House (d) Neither 1 nor 2
of the Parliament or the Legislative
Assemblies of States are also eligible to
be included in the Electoral College. 4. In the context of India, which one of the
following is the characteristic appropriate
2. Higher the number of elective Assembly
for bureaucracy? (2020)
seats, higher is the value of vote of each
MLA of that State. (a) An agency for widening the scope of
parliamentary democracy
3. The value of vote of each MLA of Madhya
Pradesh is greater than that of Kerala. (b)
An agency for strengthening the
structure of federalism
4. The value of vote of each MLA of
Puducherry is higher than that of (c)
Political An agency for facilitating
Arunachal Pradesh because the ratio stability and economic growth
of total population to total number of (d) An agency for the implementation of
elective seats in Puducherry is greater public policy

Executive 399
unacademy.com | Download the Unacademy app
Give your feedback here: Link
5. Consider the following statements: (2020) (a) First Administrative Reforms
1. The President of India can summon a Commission (1966)
session of Parliament at such a place as (b) Rajamannar Committee (1969)
he/she thinks fit. (c) Sarkaria Commission (1983)
2. The Constitution of India provides for (d) National Commission to Review the
three sessions of the Parliament in a Working of the Constitution (2000)
year, but it is not mandatory to conduct
all three sessions.
3. There is no minimum number of days 8. With reference to the election of the
that the Parliament is required to meet President of India, consider the following
in a year. statements: (2018)

Which of the statements given above is/are 1. The value of the vote of each MLA varies
correct? from State to State.

(a) 1 only 2. The value of vote of MPs in Lok Sabha is


more than the value of the vote of MPs
(b) 2 only of the Rajya Sabha.
(c) 1 and 3 only Which of the statements given above is/are
(d) 2 and 3 only correct?
(a) 1 only
6. With reference to the Legislative Assembly (b) 2 only
of a State in India, consider the following (c) Both 1 and 2
statements: (2019)
(d) Neither 1 nor 2
1. The Governor makes a customary
address to Members of the House at the
commencement of the first session of 9. Consider the following statements: (2018)
the year. 1. No criminal proceedings shall be
2. When a State Legislature does not have instituted against the Governor of a
a rule on a particular matter, it follows State in any court during his term of
the Lok Sabha rule on that matter. office.
Which of the statements given above is/are 2. The emoluments and allowances of
correct? the Governor of a state shall not be
(a) 1 only diminished during his term of office.

(b) 2 only Which of the statements given above is/are


correct?
(c) Both 1 and 2
(a) 1 only
(d) Neither 1 nor 2
(b) 2 only
(c) Both 1 and 2
7. Which one of the following suggested that
the Governor should be an eminent person (d) Neither 1 nor 2
from outside the State and should be a
detached figure without intense political 10. If the president of India exercises his
links or should not have taken part in power as provided under Article 356 of
politics in the recent past? (2019) the constitution in respect of a particular

400 Executive
unacademy.com | Download the Unacademy app
Give your feedback here: Link
state, then (2018) 1. The Chief Secretary in a State is
(a) the assembly of a state is automatically appointed by the Governor of that State.
dissolved. 2. The Chief Secretary in a state has a
(b) the power of the Legislature of that fixed tenure.
State shall be exercisable by or under Which of the statements given above is/are
the authority of the Parliament. correct?
(c) Article 19 is suspended in that State. (a) 1 only
(d) the President can make laws relating to (b) 2 only
that State. (c) Both 1 and 2
(d) Neither 1 nor 2
11. Which of the following are not necessarily
the consequences of the proclamation of
the President's rule in a State? (2017) 14. Consider the following statements: (2015)

1. Dissolution of the State Legislative 1. The Executive Power of the Union of


Assembly India is vested in the Prime Minister.

2. Removal of the Council of Ministers in 2. The Prime Minister is the ex officio


the State Chairman of the Civil Services Board.

3. Dissolution of the local bodies Which of the statements given above is/are
correct?
Select the correct answer using the code
given below: (a) 1 only

(a) 1 and 2 only (b) 2 only

(b) 1 and 3 only (c) Both 1 and 2

(c) 2 and 3 only (d) Neither 1 nor 2

(d) 1, 2 and 3
15. Which of the following is/are the functions/
functions of the Cabinet Secretariat?
12. The Parliament of India exercises control (2014)
over the functions of the Council of
Ministers through (2017) 1. Preparations of agenda for Cabinet
Meetings
1. Adjournment motion
2. Secretarial assistance to Cabinet
2. Question hour Meetings
3. Supplementary questions 3. Allocations of financial resources to the
Select the correct answer using the code Ministers
given below: Select the correct answer using the code
(a) 1 only given below:
(b) 2 and 3 only (a) 1 only
(c) 1 and 3 only (b) 2 and 3 only
(d) 1, 2 and 3 (c) 1 and 2 only
(d) 1, 2 and 3
13. Consider the following statements: (2016)

Executive 401
unacademy.com | Download the Unacademy app
Give your feedback here: Link
16. Which of the following are the discretionary 3. The Prime Minister shall communicate
powers of the Governor of a State? (2014) to the President about the proposals
1. Sending a report to the President of for legislation.
India for imposing the President’s rule. Select the correct answer using the given
2. Appointing the Ministers. code below:

3. Reserving certain bills passed by the (a) 1 only


State Legislature for consideration of (b) 2 and 3 only
the President of India. (c) 1 and 3 only
4. Making the rules to conduct the (d) 1, 2 and 3
business of the State Government.
Select the answers using the codes given
below: 19. Which one of the following statements is
correct? (2013)
(a) 1 and 2 only
(a) In India, the same person cannot be
(b) 1 and 3 only appointed as Governor for two or more
(c) 2, 3 and 4 only States at the same time.
(d) 1, 2, 3 and 4 (b) The Judges of the High Court of the
States in India are appointed by the
Governor of the State just as the Judges
17. Consider the following statements: (2014)
of the Supreme Court are appointed by
1. The President shall make rules for the the President.
more convenient transaction of the
(c) No procedure has been laid down in the
business of the government of India,
Constitution of India for the removal of
and for the allocation among Ministers
a Governor from his/her post.
of the said business.
(d) In the case of a Union Territory having
2. All executive actions of the government
a legislative setup, the Chief Minister is
of India shall be expressed to be taken
appointed by the Lt. Governor on the
in the name of the Prime Minister.
basis of majority support.
Which of the statements given above is/are
correct?
20. The Prime Minister of India, at the time of
(a) 1 only
his/her appointment: (2012)
(b) 2 only
(a) Need not necessarily be a member of
(c) Both 1 and 2 one of the Houses of the Parliament but
(d) Neither 1 nor 2 must become a member of one of the
Houses within six months
(b) Need not necessarily be a member of
18. Consider the following statements: (2013)
one of the Houses of the Parliament
1. The Council of Ministers in the Centre but must become a member of the Lok
shall be collectively responsible to the Sabha within six months
Parliament.
(c) Must be a member of one of the Houses
2. The Union Ministers shall hold the office of the Parliament
during the pleasure of the President of
(d) Must be a member of the Lok Sabha
India.

402 Executive
unacademy.com | Download the Unacademy app
Give your feedback here: Link
21. According to the Constitution of India, it 4. The Report of the National Commission
is the duty of the President of India to be for Scheduled Castes
laid before the Parliament which of the Select the correct answer using the code
following? (2012) given below:
1. The Recommendations of the Union (a) 1 only
Finance Commission
(b) 2 and 4
2. The Report of the Public Accounts
Committee (c) 1, 3 and 4

3. The Report of the Comptroller and (d) 1, 2, 3 and 4


Auditor General

Executive 403
unacademy.com | Download the Unacademy app
Give your feedback here: Link
4 Executive-Explanation

1. Answer: (d) States is currently calculated using data from


Statement 1 is incorrect: Article 71 of the the 1971 Census.
Constitution explicitly states that any acts Statement 3 is not correct: The vote value of
done by the president or vice president in their each MLA of Madhya Pradesh is less than that
official capacity shall not be invalidated just of the vote value of each MLA of Kerala as the
because the Supreme court has declared their ratio of total population to total elective seats
election to be invalid. in Kerala is greater relative to that in Madhya
Statement 2 is incorrect: The Presidential and Pradesh.
Vice presidential election Act and rules of 1952 Statement 4 is correct: The vote value of
empower the election commission of India each MLA of Puducherry is higher than that
to conduct elections to these offices without of Arunachal Pradesh as the ratio of total
any provision allowing for changes to election population to total elective seats in Puducherry
cycles due to non functional legislative is greater relative to that in Arunachal Pradesh.
assemblies. In 2022 Presidential election the vote value of
Statement 3 is incorrect: The Constitution each MLA from Puducherry was 16 whereas
does not provide for any time limit to either the vote value of each MLA from Arunachal
the President or Governor with respect to Pradesh came out to be 8.
delivering assent https://www.dnaindia.com/india/report-
presidential-elections-2022-from-up-to-
sikkim-know-state-wise-vote-value-of-
2. Answer: (a) mlas-2969061
Statement 1 is not correct: The elected
members of the Upper and Lower Houses of
Parliament that is the Rajya Sabha and the Lok 3. Answer: (b)
Sabha as well as the elected Members of the Statement 1 is not correct: Article 74: There
Legislative Assemblies of States and Union shall be a Council of Ministers with the Prime
Territories (MLAs) comprise the electoral Minister at the head to aid and advise the
college for the Presidential Election in India. President who shall, in the exercise of his
Statement 2 is not correct: The value of vote functions, act in accordance with such advice:
of each MLA depends on the ratio of the Provided that the President may require
State’s population and the number of MLAs in the council of Ministers to reconsider such
its legislative Assembly. By dividing the State’s advice, either generally or otherwise, and the
population by the number of MLAs in its President shall act in accordance with the
legislative Assembly, and then further dividing advice tendered after such reconsideration.
the quotient achieved by 1000, one may The question whether any, and if so what, advice
calculate the value of each MLA’s vote. Based was tendered by Ministers to the President
on a calculation that takes into account each shall not be inquired into in any court.
State’s population in relation to the number Statement 2 is correct: Article 75 (1A): (91st
of members in its legislative Assembly, each Amendment)
MLA’s vote value varies from State to State.
The total number of Ministers, including the
According to the Constitution (Eighty-fourth
Prime Minister, in the Council of Ministers shall
Amendment) Act of 2001, the population of the

404 Executive-Explanation
unacademy.com | Download the Unacademy app
Give your feedback here: Link
not exceed fifteen per cent. of the total number 6. Answer: (a)
of members of the House of the People. Statement 1 is correct: Article 176(1) of the
Constitution of India enjoins that the Governor
4. Answer: (d) shall address both the Houses assembled
together at the commencement of the first
Option (d) is correct: Bureaucracy refers to Session after each general election to the
a permanent executive body of non-elected Assembly and at the commencement of the
government officials and an administrative first session of each year.
policy-making group. The public administration
in many countries is an example of a Statement 2 is not correct: When the State
bureaucracy. Providing policy inputs to the legislature does not have a rule on a particular
Government and to implement the policies matter, the rules made by the Speaker of
once they become law or directives are the the Assembly or Chairman of the Council
two basic functions of Bureaucracy. So, are applicable. Article 208(1) states that, the
bureaucracy can be considered as an agency Legislature of a State may make rules for
for the implementation of public policy. regulating subject to the provisions of the
Constitution, its procedure and the conduct
of its business. Article 208(2) states that
4. Answer: (c) until such rules are made for the conduct of
Statement 1 is correct: Under Article 85(1) of business, the adaptations and modifications
the Constitution the President can summon made by the Speaker of Legislative Assembly
each House of Parliament to meet at such or the Chairman of the Legislative Council shall
time and place as he thinks fit, but six months be applicable.
duration shall not exceed between its last
sitting in one Session and the date appointed 7. Answer: (c)
for its first sitting in the next Session.
Option (c) is correct: Sarkaria Commission was
Statement 2 is not correct: The Indian set up in June 1983 to examine the relationship
Constitution does not provide for three and balance of power between state and central
sessions of the Parliament. It is by convention governments and suggest changes within the
that Parliament meets for three sessions in a framework of the Constitution. The Commission
year. suggested various recommendations for the
Statement 3 is correct: To meet in a year appointment of Governor.
there is no minimum number of days that the It recommended that the Governor:
Parliament is required in fact, the number of
days that Parliament meets has reduced over y Should be an eminent person.
the years. During the first two decades of y Must be a person from outside the State.
Parliament, Lok Sabha met for an average of y Should not have participated in active
a little more than 120 days a year. In the last politics at least for some time before his
decade this has come down to approximately appointment.
70 days.
y Should not be connected with local politics
Elimination Technique: The Constitution of the state.
asks House of Parliament to meet at such
time and place as president thinks fit
and six months duration shall not exceed 8. Answer: (a)
between its sitting hence statement 1 is Statement 1 is correct: The President is elected
correct and 2 is incorrect. by an electoral college, which comprises

Executive-Explanation 405
unacademy.com | Download the Unacademy app
Give your feedback here: Link
only of elected representatives (Parliament, by or under the authority of the Parliament.
Legislative Assemblies of States and UTs of The Parliament can also delegate the power
Delhi and Puducherry). Nominated members of to make laws for the State to the President or
Parliament are not allowed to vote. The value to any other Authority specified by him in this
of a vote of an MLA varies from State to State, regard.
mainly to reflect the population of each state.
For example: the value of one MLA’s vote in
Delhi is 58, in Uttar Pradesh it is 208 and in 11. Answer: (b)
Sikkim it is just 7. Article 355 imposes a duty upon the Centre to
Statement 2 is not correct: The value of vote of ensure that the government of every state is
each Member of Parliament is equal whether carried on in accordance with the Constitution.
he/she is from Lok Sabha or Rajya Sabha. It is this duty in the performance of which the
centre takes over the State government under
Article 356 in case of failure of Constitutional
9. Answer: (c) machinery in the state. This is popularly known
Statement 1 is correct: The Governor is entitled as ‘President’s Rule’, also known as ‘State
to a number of privileges and immunities. He Emergency’ or ‘Constitutional Emergency’.
enjoys personal immunity from legal liability Option (b) is correct: Consequences of
for his official acts. During his term of office, he President’s Rule are mentioned below.
is immune from any criminal proceedings, even y The President can take up the functions of
in respect of his personal acts and cannot be the state government and powers vested in
arrested or imprisoned. However, after giving the governor.
two months’ notice, civil proceedings can be
instituted against him during his term of office y The President can declare that the powers
in respect of his personal acts. of the state legislature are to be exercised
by the Parliament.
Statement 2 is correct: A Governor holds office
for a term of five years from the date on which y The President either suspends or dissolves
he enters upon his office. He is entitled to such the State Legislative Assembly.
emoluments, allowances and privileges as may y The President dismisses the State Council
be determined by Parliament. His emoluments of Ministers headed by the Chief Minister.
and allowances cannot be diminished during y There is no provision for dissolution of
his term of office. the local bodies when President’s rule is
imposed in the state.
10. Answer: (b)
The President’s Rule can be proclaimed under 12. Answer: (d)
Article 356 on two grounds, one mentioned The Parliament exercises control over the
in Article 356 itself and another in Article ministers through various devices like question
365. When the President’s rule is imposed, hour, discussions, adjournment motion, no-
the President dismisses the State Council of confidence motion, etc.
Ministers headed by the Chief Minister. Further,
Option (d) is correct: Adjournment Motion
the President either suspended or dissolved
is introduced in the Parliament to draw the
the State Legislative Assembly.
attention of the House to a definite matter
Option (b) is correct: When the State Legislature of urgent importance and needs the support
is suspended or dissolved, the power of the of 50 members to be admitted. It involves an
Legislature of that State shall be exercisable element of censure against the government,

406 Executive-Explanation
unacademy.com | Download the Unacademy app
Give your feedback here: Link
and hence Rajya Sabha is not permitted to the Vice-President, the Prime Minister, the
make use of this device. The discussion on an Council of Ministers and the Attorney General
adjournment motion should last for not less of India. The President is the Head of the Indian
than two hours and thirty minutes. Hence, it State. He is the First Citizen of India and acts
is a tool of the Parliament to exercise control as the symbol of unity, integrity and solidarity
over the functions of the Council of Ministers. of the nation.
uestion Hour is also used to exercise control The executive powers and functions of the
over the functions of the Council of Ministers. President are:
The first hour of every parliament sitting is y All executive actions of the Government
slotted for this. During this time, the members of India are formally taken in his name.
can ask questions, and ministers usually give (Statement 1 is not correct)
answers. The questions are of three kinds,
namely, starred, unstarred and short notices. y He can make rules specifying the manner
in which the orders and other instruments
y Starred Question (distinguished by an made and executed in his name shall be
asterisk) requires an oral answer, and hence authenticated.
Supplementary questions can follow.
y He appoints the Prime Minister and the
y Unstarred questions require a written other ministers. They hold office during his
answer and hence supplementary questions pleasure.
cannot follow.
The Civil Services Board is responsible for
y Short notice questions are asked by giving recruitment and promotions below the rank
a notice of less than ten days, answered of Joint Secretary. It is headed by the Chief
orally. Secretary of the state with Personnel Secretary
and either Financial Commissioner or Home
13. Answer: (d) Secretary as its members. (Statement 2 is not
correct)
Statement 1 is not correct: Chief Secretary is
the topmost bureaucratic position in a State,
generally held by an Indian Administrative 15. Answer: (c)
Officer. The Chief Secretary is ‘a linchpin’ in the The Cabinet Secretariat is responsible for the
administration. The Chief Secretary is chosen administration of the Government of India
by the Chief Minister from among the senior (Transaction of Business) Rules, 1961 and the
most Indian Administrative Officers. He is then Government of India (Allocation of Business)
appointed by the Chief Minister in the name Rules, 1961, facilitating smooth transaction
of the Governor. The Governor is the Chief of business in Ministries/Departments of
Executive Head of the State and all executive the Government. This Secretariat provides
actions in a State are taken by and in the name Secretarial assistance to the Cabinet and
of the Governor. its Committees. The secretarial assistance,
Statement 2 is not correct: Chief Secretary of provided by Cabinet Secretariat to the Cabinet
a State has not been guaranteed any security and Cabinet Committees, includes:
of tenure. He assumes his office as per the y Preparation and circulation of the agenda.
pleasure of the Governor and can be removed (Statement 1 is correct)
anytime by the Governor.
y Convening of the meetings of the Cabinet
and its Committees on the orders of the
14. Answer: (d) Prime Minister. (Statement 2 is correct)
The Union executive consists of the President, y Circulation of the record of discussions

Executive-Explanation 407
unacademy.com | Download the Unacademy app
Give your feedback here: Link
after obtaining the approval of the Prime Government is not under the discretionary
Minister. powers of the Governor of a State.
y Monitoring implementation of decisions Elimination Technique: Appointing the
taken by the Cabinet and its Committees. Ministers and making the rules to conduct
Statement 3 is not correct: Allocations of the business of the State Government is
financial resources to the Ministers is not a not under the discretionary powers of the
function of the Cabinet Secretariat. Financial Governor hence statement 2 and 4 are
resources are allocated to Ministries, as per incorrect.
the provisions in the Budget prepared by the
Finance Ministry.
17. Answer: (a)
Statement 1 is correct: The President is
16. Answer: (b) the Head of the Indian State. He is the First
The Constitution of India provides for a Citizen of India and acts as the symbol of
Parliamentary form of government in the States unity, integrity, and solidarity of the Nation.
as in the Centre. Consequently, the Governor He performs a wide spectrum of functions
has been made only a nominal executive, ranging from executive, legislative, judicial,
the real executive constitutes the Council military, financial to diplomatic. Under the
of Ministers headed by the Chief Minister. In executive powers of the President granted by
other words, the Governor has to exercise his the Constitution, he/she can make rules for
powers and functions with the aid and advice more convenient transactions of business of
of the Council of Ministers headed by the the Union Government, and for allocation of
Chief Minister, except in matters in which he is the said business among the Ministers.
required to act at his discretion. The Governor Statement 2 is not correct: All executive actions
has Constitutional discretion in the following of the Government of India are formally taken
cases: in the President’s name and not in the name of
y Recommendation for the imposition of the the Prime Minister (head of the government or
President’s Rule in the state. (Statement 1 real head).
is correct) Other executive functions of the President are:
y Reservation of a Bill for the consideration y The President can make rules specifying
of the President. (Statement 3 is correct) the manner in which the orders and other
y While exercising his functions as the instruments made and executed in his
administrator of an adjoining Union Territory name shall be authenticated. He appoints
(in case of additional charge). the Prime Minister and the other Ministers.
y Determining the amount payable by the They hold office during his (President’s)
Government of Assam, Meghalaya, Tripura pleasure.
and Mizoram to an autonomous Tribal y He appoints the Attorney General of India
District Council as royalty accruing from and determines his remuneration. The
licences for mineral exploration. Attorney General holds office during the
y Seeking information from the Chief Minister pleasure of the President.
with regard to the administrative and y He appoints the Comptroller and Auditor
legislative matters of the State. General of India, the Chief Election
Statement 2 and statement 4 are not correct: Commissioner and other Election
Appointing the Ministers and making the Commissioners, the Chairman and Members
rules to conduct the business of the State of the Union Public Service Commission,

408 Executive-Explanation
unacademy.com | Download the Unacademy app
Give your feedback here: Link
the Governors of States, the Chairman and Option (d) is not correct: Section 45 in the
Members of Finance Commission, and so Government of Union Territories Act, 1963,
on. states that the Chief Minister shall be appointed
by the President and the other Ministers shall
be appointed by the President on the advice of
18. Answer: (b)
the Chief Ministers.
Article 75 of the Indian Constitution states
that:
20. Answer: (a)
y The Prime Minister shall be appointed by
the President and the other Ministers shall The Constitution does not contain any specific
be appointed by the President on the advice procedure for the selection and appointment
of the Prime Minister. of the Prime Minister. Article 75 says only that
the Prime Minister shall be appointed by the
y The Council of Ministers shall be collectively
president. However, this does not imply that
responsible to the House of the People.
the president is free to appoint anyone as
(Statement 1 is not correct)
the Prime Minister. In accordance with the
y The Ministers shall hold office during the conventions of the parliamentary system of
pleasure of the President. (Statement 2 is government, the President has to appoint the
correct) leader of the majority party in the Lok Sabha as
According to Article 78 of the Indian the Prime Minister.
Constitution, it shall be the duty of the Prime But, when no party has a clear majority in the
Minister to communicate to the President all Lok Sabha, then the President may exercise
the decisions of the Council of Ministers relating his personal discretion in the selection and
to the administration of the affairs of the Union appointment of the Prime Minister. In such a
and proposals for legislation. (Statement 3 is situation, the President usually appoints the
correct) leader of the largest party or coalition in the
Lok Sabha as the Prime Minister and asks him
to seek a vote of confidence in the House
19. Answer: (c)
within a month. This discretion was exercised
Option (a) is not correct: There is a Governor by the President, for the first time in 1979, when
for each state, but the 7th Constitutional Neelam Sanjiva Reddy (the then President)
Amendment Act of 1956 facilitated the appointed Charan Singh (the coalition leader)
appointment of the same person as a Governor as the Prime Minister after the fall of the Janata
for two or more states. Party government headed by Morarji Desai.
Option (b) is not correct: The Judges of a High Option (a) is correct: In 1997, the Supreme
Court and the Supreme Court are appointed by Court held that a person who is not a member
the President. of either House of Parliament can be appointed
Option (c) is correct: The Constitution does not as Prime Minister for six months, within which,
lay down any grounds upon which a Governor he should become a member of either House
may be removed by the President. A Governor of Parliament; otherwise, he ceases to be the
holds office for a term of five years from Prime Minister.
the date on which he enters upon his office. In the scheme of parliamentary system of
However, this term of five years is subject to government provided by the constitution, the
the pleasure of the President. Further, he can President is the nominal executive authority
resign at any time by addressing a resignation (de jure executive) and Prime Minister is the
letter to the President. real executive authority (de facto executive). In

Executive-Explanation 409
unacademy.com | Download the Unacademy app
Give your feedback here: Link
other words, the President is the head of the each general election and the first session
State while the Prime Minister is the head of of each year.
the government. Constitutionally, the Prime y He can send messages to the Houses of
Minister may be a member of any of the two Parliament, whether with respect to a bill
Houses of Parliament. pending in the Parliament or otherwise.
y He lays the reports of the Comptroller
21. Answer: (c) and Auditor General, Union Public Service
Option (c) is correct: According to the Commission, Finance Commission, National
Constitution of India, it is the duty of the Commission of Scheduled Caste, National
President of India to be laid before the Commission of Scheduled Tribe, before the
Parliament -the recommendations of the Parliament.
Union Finance Commission, the report of the y He can appoint any member of the Lok
Comptroller and Auditor General of India and Sabha to preside over its proceedings when
the report of the National Commission for the offices of both the Speaker and the
Scheduled Castes. Deputy Speaker fall vacant. Similarly, he
Under the Constitution, the Parliament of can also appoint any member of the Rajya
India consists of three parts viz, the President, Sabha to preside over its proceedings when
the Council of States, and the House of the the offices of both the Chairman and the
People. The President is an integral part of the Deputy Chairman fall vacant.
Parliament of India and enjoys the following y He nominates 12 members of the Rajya
legislative powers: Sabha from amongst persons having
y He can summon or prorogue the Parliament special knowledge or practical experience
and dissolve the Lok Sabha. He can also in literature, science, art and social service.
summon a joint sitting of both the Houses y He can nominate two members to the Lok
of Parliament, which is presided over by the Sabha from the Anglo-Indian Community.
Speaker of the Lok Sabha. y He decides on questions as to
y He can address the Parliament at the disqualifications of members of the
commencement of the first session after Parliament, in consultation with the
Election Commission.

410 Executive-Explanation
unacademy.com | Download the Unacademy app
Give your feedback here: Link
5 Judiciary

1. In India, which one of the following 1. Mandamus will not lie against a private .
Constitutional Amendments was widely organisation unless it is entrusted with
believed to be enacted to overcome the a public duty.
judicial interpretations of Fundamental
Rights? (2023) 2. Mandamus will not lie against a Company
even though it may be a Government
(a) 1st Amendment Company.
(b) 42nd Amendment 3. Any public minded person can be a
(c) 44th Amendment petitioner to move the Court to obtain
(d) 86th Amendment the writ of Quo Warranto.
Which of the statements given above are
correct ?
2. Consider the following statements : (2023)
(a) 1 and 2 only
Statement-I:
(b) 2 and 3 only
The Supreme Court of India has held in
some judgments that the reservation (c) 1 and 3 only
policies made under Article 16(4) of the (d) 1, 2 and 3
Constitution of India would be limited by
Article 335 for maintenance of efficiency of
administration. 4. With reference to India, consider the
following statements: (2022)
Statement-II :
1. Government law officers and legal
Article 335 of the Constitution of firms are recognised as advocates, but
India defines the term 'efficiency of corporate lawyers and patent attorneys
administration'. are excluded from recognition as
Which one of the following is correct in advocates.
respect of the above statements? 2. Bar Councils have the power to lay down
(a) Both Statement-I and Statement-II are the rules relating to legal education and
correct and Statement-II is the correct recognition of law colleges.
explanation for Statement-I Which of the statements given above is/are
(b)
Both Statement-I and Statement-II correct ?
are correct and Statement-II is not the (a) 1 only
correct explanation for Statement-I
(b) 2 only
(c) Statement-I is correct but Statement-II
is incorrect (c) Both 1 and 2

(d) Statement-I is incorrect but Statement- (d) Neither 1 nor 2


II is correct
5. Consider the following statements: (2022)
3. With reference to the writs issued by the 1. Pursuant to the report of H.N. Sanyal
Courts in India, consider the following Committee, the Contempt of Courts
statements : (2022) Act, 1971 was passed.

Judiciary 411
unacademy.com | Download the Unacademy app
Give your feedback here: Link
2. The Constitution of India empowers the Which of the statements given above is /
Supreme Court and the High Courts to are correct?
punish for contempt of themselves. (a) 1 only
3. The Constitution of India defines Civil (b) 2 only
Contempt and Criminal Contempt.
(c) Both 1 and 2
4. In India, the Parliament is vested with
the powers to make laws on Contempt (d) Neither 1 nor 2
of Court.
Which of the statements given above is/are 8. With reference to Indian judiciary, consider
correct ? the following statements : (2021)
(a) 1 and 2 only 1. Any retired judge of the Supreme Court
(b) 1, 2 and 4 of India can be called back to sit and act
as a Supreme Court judge by the Chief
(c) 3 and 4 only Justice of India with prior permission of
(d) 3 only the President of India.
2. A High Court in India has the power
6. With reference to India, consider the to review its own judgement as the
following statements : (2021) Supreme Court does.

1. Judicial custody means an accused Which of the statements given above is /


is in the custody of the concerned are correct?
magistrate and such accused is locked (a) 1 only
up in police station, not in jail. (b) 2 only
2. During judicial custody, the police officer (c) Both 1 and 2
in charge of the case is not allowed to
interrogate the suspect without the (d) Neither 1 nor 2
approval of the court.
Which of the statements given above is/are 9. In India, separation of judiciary from the
correct? executive is enjoined by (2020)
(a) 1 only (a) the Preamble of the Constitution
(b) 2 only (b) a Directive Principle of State Policy
(c) Both 1 and 2 (c) the Seventh Schedule
(d) Neither 1 nor 2 (d) the conventional practice

7. With reference to India, consider the 10. Consider the following statements: (2019)
following statements : (2021) 1. The motion to impeach a Judge of
1. When a prisoner makes out a sufficient the Supreme Court of India cannot
case, parole cannot be denied to such be rejected by the Speaker of the Lok
prisoner because it becomes a matter Sabha as per the Judges (Inquiry) Act
of his/her right. 1968.
2. State Governments have their own 2. The Constitution of India defines
Prisoners Release on Parole Rules. and gives details or what constitutes
‘incapacity and proved misbehaviour’

412 Judiciary
unacademy.com | Download the Unacademy app
Give your feedback here: Link
of the Judges of the Supreme Court of constitutional powers under Article 142.
India. It could mean which one of the following?
3. The details of the process of (2019)
impeachment of the Judges of the (a) The decisions taken by the Election
Supreme Court of India are given in the Commission of India while discharging
Judges (Inquiry) Act, 1968. its duties cannot be challenged in any
4. If the motion for the impeachment court of law.
of a Judge is taken up for voting, the (b) The Supreme Court of India is not
law requires the motion to be backed constrained in the exercise of its powers
by each House of the Parliament by the laws made by Parliament.
and supported by a majority of total (c) In the event of a grave financial crisis in
membership of that House and by not the country, the President of India can
less than two-thirds of total members declare a Financial Emergency without
of that House present and voting. the counsel from the Cabinet.
Which of the statements given above is/are (d) State Legislatures cannot make laws on
correct? certain matters without the concurrence
(a) 1 and 2 only of the Union Legislature.
(b) 3 only
(c) 3 and 4 only 13. With reference to the Constitution of India,
(d) 1, 3 and 4 only consider the following statements: (2019)
1. No High Court shall have the jurisdiction
to declare any central law to be
11. Consider the following statements: (2019)
constitutionally invalid.
1. The 44th Amendment to the Constitution
2. An amendment to the Constitution of
of India introduced an Article placing the
India cannot be called into question by
election of the Prime Minister beyond
the Supreme Court of India.
judicial review.
Which of the statements given above is/are
2. The Supreme Court of India struck
correct?
down the 99th Amendment to the
Constitution of India as being violative (a) 1 only
of the independence of judiciary. (b) 2 only
Which of the statements given above is/are (c) Both 1 and 2
correct? (d) Neither 1 nor 2
(a) 1 only
(b) 2 only 14. In India, Judicial Review implies (2017)
(c) Both 1 and 2 (a) The power of the Judiciary to pronounce
(d) Neither 1 nor 2 upon the Constitutionality of laws and
executive orders.

12. With reference to the Constitution of India, (b) The power of the Judiciary to question
prohibitions or limitations or provisions the wisdom of the laws enacted by the
contained in ordinary laws cannot act Legislatures.
as prohibitions or limitations on the (c) The power of the Judiciary to review all
the legislative enactments before they

Judiciary 413
unacademy.com | Download the Unacademy app
Give your feedback here: Link
are assented to by the President. House of the Parliament or that of
(d) The power of the Judiciary to review its Legislature of a State
own judgements given earlier in similar 3. A dispute between the Government of
or different cases. India and a Union Territory
4. A dispute between two or more States
15. Who/which of the following is the Custodian Select the correct answer using the code
of the Constitution of India? (2015) given below:
(a) The President of India (a) 1 and 2 only
(b) The Prime Minister of India (b) 2 and 3 only
(c) The Lok Sabha Secretariat (c) 1 and 4 only
(d) The Supreme Court of India (d) 3 and 4 only

16. The power to increase the number of 19. What is the provision to safeguard the
Judges in the Supreme Court of India is autonomy of the Supreme Court of India?
vested in (2014) (2012)
(a) The President of India 1. While appointing the Supreme Court
(b) The Parliament Judges, the President of India has to
consult the Chief Justice of India.
(c) The Chief Justice of India
2. The Supreme Court Judges can be
(d) The Law Commission
removed by the Chief Justice of India
only.
17. The power of Supreme Court of India to 3. The salaries of the Judges are charged
decide dispute between the centre and on the Consolidated Fund of India to
states fall under its: (2014) which the legislature does not have to
(a) Advisory jurisdiction vote.
(b) Appellate jurisdiction 4. All appointments of officers and staff of
the Supreme Court of India are made by
(c) Original jurisdiction
the Government only after consulting
(d) Writ jurisdiction the Chief Justice of India.
Which of the statements given above is/are
18. Which of the following are included in the correct?
original jurisdiction of the Supreme Court? (a) 1 and 3 only
(2012)
(b) 3 and 4 only
1. A dispute between the Government of
(c) 4 only
India and one or more States
(d) 1, 2, 3 and 4
2. A dispute regarding elections to either

414 Judiciary
unacademy.com | Download the Unacademy app
Give your feedback here: Link
5 Judiciary-Explanation

1. Answer: (a) 3. Answer: (d)


Option (a) is correct: The first Constitutional Statement 1 is correct and Statement 2 is
Amendment Act of 1951 was widely believed correct
to be enacted to overcome the judicial Mandamus literally means “Command”. It is
interpretations of the Fundamental Rights. The thus an order of a superior court commanding
Act added the fourth clause to Article 15 that a person holding a public office or a public
empowered the government to make any law authority- (including the Government) to do or
for the upliftment of socially and educationally not to do something, in the nature of public
backward classes of citizens or for the duty. it does not lie against a company.
Scheduled Castes and Scheduled Tribes.
Statement 3 is correct
The added clause elucidates that in case such
There are five Types of Writs. These are – Habeas
special provisions are introduced, they cannot
Corpus, Mandamus, Prohibition, Certiorari and
be said to be breaching Article 15 and Article
Quo Warranto. A citizen has a right to move the
29(2) of the Constitution.
Courts for securing the Fundamental Rights and
The need to insert this clause was felt after legal rights enforced by filling a Writ Petition.
the decision of the Supreme Court in the
State of Madras v. Srimathi Champakam (1951).
According to the facts of this case, the Madras 4. Answer: (b)
government issued an Order that provided Statement 1 is not correct: Advocates are
reservation on the grounds of religion, race, the only recognized class of persons entitled
and caste. This Order was contended to be in to Practice the profession of Law. With
breach of Article 15(1) of the Indian Constitution. the development of the society the legal
The Court also gave a literal interpretation profession underwent a metamorphosis and
to the constitutional provisions and held several attempts have been made for proper
that reserving seats in public institutions for dispensation of justice and to cater the legal
backward classes violates Articles 15(1) and needs of the society. They have been excluded
29(2). Therefore, in order to nullify the effect from recognition as advocates.
of similar judicial pronouncements, Article 15
Statement 2 is correct: The Bar Council of India
was amended.
visits and inspects Universities/Law colleges in
the country as part of its statutory function
2. Answer: (c) of promoting legal education and laying down
standards in consultation with the Universities
Statement 1 is correct: The Supreme court
in India and the State Bar Councils.
in the Nagraj case of 2006, and the Jarnail
singh case of 2018, laid down guidelines for
reservation in promotions . It said reservation 5. Answer: (b)
in promotion may be implemented if it does not
Statement 1 is correct: The Sanyal Committee
hamper the overall administrative efficiency of
examined the law relating to contempt of
the government.
courts in general, and the law relating to the
Statement 2 is incorrect: The Constitution procedure for contempt proceedings including
states the term ‘efficiency of administration’ the punishment thereof in particular. The
but does not define it. Committee submitted its report in 1963, which

Judiciary-Explanation 415
unacademy.com | Download the Unacademy app
Give your feedback here: Link
inter alia defined and limited the powers of (iii) interferes or tends to interfere with,
certain courts in punishing for contempt of or obstructs or tends to obstruct, the
courts and provided to regulate the procedure administration of justice in any other
in relation thereto. The recommendations of manner;
the Committee were generally accepted by the (d) “High Court” means the High Court for a
Government after having wide consultation State or a Union territory, and includes the
with the State Governments, Union Territory court of the Judicial Commissioner in any
Administrations, and all other stakeholders. Union territory.
After the aforesaid deliberations the Contempt
of Courts Act, 1971 (70 of 1971) came to be Statement 4 in correct
enacted (hereinafter referred to as the “Act SEVENTH SCHEDULE (Article 246) List I—
1971”), which repealed and replaced the Act Union List: Entry 77. Constitution, organisation,
1952. jurisdiction and powers of the Supreme Court
Statement 2 is correct: Article 129: Supreme (including contempt of such Court), and the
Court to be a court of record The Supreme fees taken therein; persons entitled to practise
Court shall be a court of record and shall have before the Supreme Court.
all the powers of such a court including the
power to punish for contempt of itself. 6. Answer: (b)
Article 215: High Courts to be courts of record Statement 1 is not correct: The provisions for
Every High Court shall be a court of record custody in India are governed by Section 167 of
and shall have all the powers of such a court the Code of Criminal Procedure. Police Custody
including the power to punish for contempt of means that police have the physical custody
itself. of the accused, while Judicial Custody means
Statement 3 is not correct: an accused is in the custody of the concerned
THE CONTEMPT OF COURTS ACT, 1971 Magistrate. In Police custody, the accused
is lodged in a police station lockup, while in
Definitions.—In this Act, unless the context Judicial Custody, he is locked up in jail.
otherwise requires,—
Statement 2 is correct: During Judicial Custody,
(a) “contempt of court” means civil contempt the police officer in charge of the case is not
or criminal contempt; allowed to interrogate the suspect. However,
(b) “civil contempt” means wilful disobedience the court may allow the interrogations to be
to any judgement, decree, direction, order, conducted if it opines the interrogation is
writ or other process of a court or wilful necessary under the facts produced before
breach of an undertaking given to a court; the court. However, during police custody,
(c) “criminal contempt” means the publication the police officer in charge of the case may
(whether by words, spoken or written, or interrogate the suspect.
by signs, or by visible representations, or
otherwise) of any matter or the doing of any 7. Answer: (b)
other act whatsoever which—
Statement 1 is not correct: The granting
(i) scandalises or tends to scandalise, or authority for parole is the deputy home
lowers or tends to lower the authority secretary of the state government. Parole
of, any court; or is a system of releasing a prisoner with the
(ii) prejudices, or interferes or tends to suspension of the sentence. Parole cannot be
interfere with, the due course of any claimed as a matter of right. Parole may be
judicial proceeding; or denied to a prisoner even when he makes out

416 Judiciary-Explanation
unacademy.com | Download the Unacademy app
Give your feedback here: Link
sufficient cause for release on parole. Parole 9. Answer: (b)
is to be granted only on a sufficient cause, Option (b) is correct: Article 50 of the Indian
such as cases of severe illness or death of Constitution, as enshrined in form of Directly
any member of the prisoner’s family. Parole Principles of State Policy in Part IV, provides
and Furlough can be denied if the competent that “the State shall take steps to separate
authority is of the opinion that releasing the the Judiciary from the Executive in the public
inmate would not be in the interest of society. services of the State”.
Statement 2 is correct: In India, parole (as The Criminal Procedure Code 1973 also
well as furlough) is covered under the Prisons mandates the separation of judiciary from the
Act of 1894. Prisoners convicted of multiple executive, which enables the state to work
murders or under the anti-terror Unlawful independently and impartially without the
Activities Prevention Act (UAPA) are not eligible interference of any other organs of the State.
for parole. Since prisons are a State subject
to the Constitution, the Prisons Act of each The Directive Principles of State Policies
state government defines the rules under (Article 36 - 51) act as a guideline for the state
which parole is granted in that state. The and are needed to be taken into consideration
management and administration of prisons fall while coming up with any new law but a citizen
exclusively in the domain of state governments cannot compel the state to follow DPSPs as
and are governed by the Prisons Act, 1894 and they are non-justiciable. DPSPs are positive
the Prison Manuals of the respective state obligations on the state.
governments. DPSPs were not made justiciable because at
the time of the drafting of the Constitution,
India was a newly born independent state and
8. Answer: (a) was lacking in financial resources and also
Statement 1 is correct: The Chief Justice struggling with various issues and making
of India may at any time, with the previous DPSPs justiciable would have put India in great
consent of the President, request a retired difficulty.
judge of the Supreme Court or a retired judge
of a high court (duly qualified for appointment
as a Supreme Court judge) to act as a judge 10. Answer: (c)
of the Supreme Court for a temporary period. Statement 1 is not correct: The Judges (Inquiry)
He can do so only with the previous consent Act,1968 contains the detailed procedures
of the President and also of the person to be relating to the removal of Judges of the
so appointed. Such a judge is entitled to such Supreme Court or the High Court. Under this
allowances as the President may determine. Act, the Chairman or the Speaker may or may
He will also enjoy all the jurisdiction, powers not admit the motion of impeachment.
and privileges of a judge of the Supreme Court. Statement 2 is not correct: The term
Statement 2 is not correct: Article 137 of the “incapacity and proved misbehaviour” is not
Indian Constitution deals with the review of mentioned anywhere in the Constitution. The
judgments or orders by the Supreme Court. It term ‘misconduct’ of a judge is to be proved
says that subject to the provisions of any law outside Parliament before a non-parliamentary
made by Parliament or any rules made under committee. However, the Judges (Inquiry) Bill,
article 145; the Supreme Court shall have the 2006, defines ‘proved misbehaviour’:
power to review any judgment pronounced or y wilful or persistent conduct bringing
order made by it. dishonour or disrepute to the judiciary;
y wilful or persistent failure to perform the

Judiciary-Explanation 417
unacademy.com | Download the Unacademy app
Give your feedback here: Link
duties of a judge; Commission (NJAC) Act were brought by the
y wilful abuse of judicial office, corruption, government to replace the 1993 collegium
lack of integrity or committing an offence system for the appointment of judges to the
involving moral turpitude. Supreme Court and the High courts. But the
Supreme Court of India struck down the 99th
Moreover, the Judicial Standards and
amendment to the Constitution as it impinged
Accountability Bill, 2010, also proposed to
upon the principles of “independence of the
widen the definition of misconduct.
judiciary”, as well as “separation of powers”.
Statement 3 is correct: The Judges (Inquiry)
Additional Information:
Act, 1968 states “the procedure for the
investigation and proof of the misbehaviour or The 44th Constitution Amendment Act, 1978
incapacity of a judge of the Supreme Court or removed the 39th Constitution Amendment
of a High Court”. Act, 1975. It was enacted by the Janata Party
to “restore the Constitution to the condition it
Statement 4 is correct: The motion for removal
was before the Emergency”. Some provisions
of a Judge is to be signed by 50 members of
of 44th Constitutional Amendment Act, 1978
Rajya Sabha or 100 members of Lok Sabha and
are as follows:
it has to be passed by a majority of the House
and 2/3rd of those present and voting in the y Right to Property was removed as a
same session. Fundamental Right (Article 31) and was
made Legal Right (Article 300A)
Additional Information:
y Proclamation of Emergency could be done
If the motion is admitted by the Chairman or
on the basis of war, external aggression
the Speaker, he/she must constitute a three-
or armed rebellion. The word “Internal
member committee to investigate the charges
Disturbance” was replaced with “armed
against the judge. Such a committee consists
rebellion.”
of the Chief Justice or a Judge of the Supreme
Court of India, Chief Justice of a High Court y Written recommendation of the cabinet
and a distinguished Jurist. to the President was made necessary to
proclaim National Emergency.

11. Answer: (b)


12. Answer: (b)
Judicial review is the power of the Judiciary
to examine the constitutionality of any act or Option (b) is correct: Article 142 of the Indian
any orders of both the Central as well as State Constitution states that the Supreme Court
government. Judicial review is considered as a of India may pass an order which it finds
basic structure of the Constitution. necessary for doing complete justice in a case
or matter pending before it. Such an order
Statement 1 is not correct: It was the 39th
would be applicable throughout the territory
Constitutional Amendment Act (not 44th
of India.
Amendment Act) which removed the authority
of the Supreme Court to adjudicate petitions One of the important instances of application
regarding elections of the President, Vice by the Supreme Court of Article 142 was in the
President, Prime Minister and Speaker of the Union Carbide case - relating to the victims of
Lok Sabha. Instead, a body constituted by the Bhopal gas tragedy — where the Court felt
Parliament would be vested with the power to a need to deviate from existing law to bring
resolve such election disputes. relief to the thousands of persons affected by
the gas leak. In this judgement, the Supreme
Statement 2 is correct: The 99th Constitutional
Court, while awarding compensation of $470
Amendment and National Judicial Appointments

418 Judiciary-Explanation
unacademy.com | Download the Unacademy app
Give your feedback here: Link
million to the victims, went to the extent of the Supreme Court and the High Courts to
saying that to do complete justice, it could review the laws, which means an amendment
even override the laws made by Parliament to the Constitution of India can be called into
by holding that, “prohibitions or limitations or question by the Supreme Court of India.
provisions contained in ordinary laws cannot,
ipso facto, act as prohibitions or limitations
on the constitutional powers under Article 14. Answer: (a)
142.” By this statement the Supreme Court of Option (a) is correct: Judicial Review, power of
India placed itself above the laws made by the Judiciary of a country to examine the actions
Parliament or the legislatures of the States. of the legislative, executive, and administrative
arms of the government and to determine
whether such actions are consistent with the
13. Answer: (d) Constitution. Actions judged inconsistent are
The Constitution of India does not give a declared unconstitutional and, therefore, null
detailed description of the original jurisdiction and void. The institution of Judicial Review in
of the High Court. It is accepted that the this sense depends upon the presence of a
original jurisdiction of a High Court is exercised written Constitution.
by issue of Writs to any person or authority Though the phrase ‘Judicial Review’ has
including Government. nowhere been mentioned in the Constitution,
Article 226 of the Constitution vests in the the provisions of several articles (Article 13)
High Court the power to issue writs for the explicitly confer the power of judicial review
restoration of fundamental rights. However, on the Supreme Court.
this power of the High Court does not derogate The Constitutional Validity can be challenged
the similar power conferred on the Supreme on three following grounds:
Court in Article 32 of the Constitution.
y It infringes on Fundamental Rights.
Statement 1 is not correct: The original
jurisdiction of the High Courts also extends to y It is outside the competence of the
the matters of admiralty, probate, matrimonial authority which has framed it.
and contempt of Court cases. The High Courts y It is repugnant to the Constitutional
also have full powers to make rules to regulate provisions.
their business in relation to the administration
of justice.
15. Answer: (d)
Recently, a petitioner challenged the validity
Option (d) is correct: The Supreme Court is a
of the Epidemic Diseases Act, a central law
federal court, the highest court of appeal, the
in the Supreme Court. The Supreme court
guarantor of the Fundamental Rights of the
bench led by Justice DY Chandrachud refused
citizens and the guardian of the Constitution.
to entertain the plea and asked the petitioner
that the High Courts also have jurisdiction in y Article 32 is one of the Fundamental
respect of central Acts. Rights listed in the Constitution that
each citizen is entitled to. It deals with
Statement 2 is not correct: The 42nd
the ‘Right to Constitutional Remedies’ or
Constitution Amendment Act, 1976 curtailed
affirms the right to move the Supreme
the jurisdiction of the Supreme Court and the
Court by appropriate proceedings for the
High Courts to review the constitutionality
enforcement of the rights conferred in Part
of laws. However, the 43rd Constitution
III of the Constitution.
Amendment Act,1977 restored the power of

Judiciary-Explanation 419
unacademy.com | Download the Unacademy app
Give your feedback here: Link
16. Answer: (b) treaty, agreement, covenant, engagements,
Option (b) is correct: The Constitution has and or other similar instrument which,
vested the power to increase the number of having been entered into or executed
Judges of the Supreme Court of India with the before the commencement of this
Parliament. Parliament accordingly enacted Constitution, continues in operation after
the Supreme Court (Number of Judges) Act, such commencement, or which provides
1956. Originally, the strength of the Supreme that the said jurisdiction shall not extend
Court was fixed at eight (one Chief Justice and to such a dispute.
seven other Judges). In addition, Article 32 of the Constitution
y The Parliament has increased this number gives an extensive Original jurisdiction to the
of other Judges progressively to ten in 1956, Supreme Court regarding enforcement of
to thirteen in 1960, to seventeen in 1977, to Fundamental Rights. It is empowered to issue
twenty-five in 1986, to thirty in 2009 and directions, orders or writs, including writs in
to thirty-three in 2019 by amending the the nature of Habeas Corpus, Mandamus,
Supreme Court (Number of Judges) Act. Prohibition, Quo Warranto and Certiorari to
Therefore, at present the Supreme Court enforce them.
consists of thirty-four Judges (one Chief
justice and thirty-three other Judges). 18. Answer: (c)
Option (c) is correct: Article 131 of the Indian
17. Answer: (c) Constitution deals with the original jurisdiction
Option (c) is correct: The power of the Supreme of the Supreme Court. The Supreme Court
Court of India to decide disputes between shall, to the exclusion of any other court, have
the centre and states fall under its Original original jurisdiction in any dispute
jurisdiction. The Supreme Court has exclusive y between the Government of India and one
Original jurisdiction in the federal dispute. or more States; or
Federal disputes refer to disputes between y between the Government of India and any
different units of the Indian Federation. State or States on one side and one or
Article 131 of the Indian Constitution deals with more other States on the other; or
the Original jurisdiction of the Supreme Court. y between two or more States, if and in so
Original jurisdiction, subject to the provisions far as the dispute involves any question
of this Constitution, the Supreme Court shall, (whether of law or fact) on which the
to the exclusion of any other court, have existence or extent of a legal right depends:
Original jurisdiction in any dispute: Provided that the said jurisdiction shall
y Between the Government of India and one not extend to a dispute arising out of any
or more States; or treaty, agreement, covenant, engagements,
y Between the Government of India and any and or other similar instrument which,
State or States on one side and one or having been entered into or executed
more other States on the other; or before the commencement of this
Constitution, continues in operation after
y Between two or more States, if and in so
such commencement, or which provides
far as the dispute involves any question
that the said jurisdiction shall not extend
(whether of law or fact) on which the
to such a dispute
existence or extent of a legal right depends:
Provided that the said jurisdiction shall Additional Information:
not extend to a dispute arising out of any y The Supreme Court has original, appellate,

420 Judiciary-Explanation
unacademy.com | Download the Unacademy app
Give your feedback here: Link
and advisory jurisdiction. In addition, Article consultation with the chief justice and such
32 of the Constitution gives an extensive other judges of the Supreme Court and the
original jurisdiction to the Supreme Court high courts as he deems necessary.
in regard to enforcement of Fundamental Statement 2 is not correct: A judge of the
Rights. It is empowered to issue directions, Supreme Court can be removed from his Office
orders or writs, including writs in the nature by an order of the president. The President can
of habeas corpus, mandamus, prohibition, issue the removal order only after an address
quo warranto and certiorari to enforce by Parliament has been presented to him in the
them. same session for such removal. 5 The address
must be supported by a special majority of
19. Answer: (a) each House of Parliament (ie, a majority of the
total membership of that House and a majority
The Supreme Court has been assigned a of not less than two-thirds of the members of
very significant role in the Indian democratic that House present and voting).
political system. It is a federal court, the
highest court of appeal, the guarantor of Statement 3 is correct: The salaries, allowances
the fundamental rights of the citizens and and pensions of the judges and the staff as
guardian of the Constitution. Therefore, its well as all the administrative expenses of the
independence becomes very essential for the Supreme Court are charged on the Consolidated
effective discharge of the duties assigned to it. Fund of India

Statement 1 is correct: The chief justice is Statement 4 is not correct: The Chief Justice
appointed by the president after consultation of India can appoint officers and servants of
with such judges of the Supreme Court and the Supreme Court without any interference
high courts as he deems necessary. The other from the executive. He can also prescribe their
judges are appointed by the president after conditions of service.

Judiciary-Explanation 421
unacademy.com | Download the Unacademy app
Give your feedback here: Link
6 Local Self Government

1. Local self-government can be best 4. The fundamental object of the Panchayati


explained as an exercise in (2017) Raj system is to ensure which among the
(a) Federalism following? (2015)

(b) Democratic decentralisation 1. People’s participation in development

(c) Administrative delegation 2. Political accountability

(d) Direct democracy 3. Democratic decentralisation


4. Financial mobilisation

2. Consider the following statements: (2016) Select the correct answer using the code
given below:
1. The minimum age prescribed for any
person to be a member of a panchayat (a) 1, 2 and 3 only
is 25 years. (b) 2 and 4 only
2. A panchayat reconstituted after (c) 1 and 3 only
premature dissolution continues only (d) 1, 2, 3 and 4
for the remainder period.
Which of the statements given above is/are
correct? 5. The Government enacted the Panchayat
Extension to Scheduled Areas (PESA) Act
(a) 1 only in 1996. Which one of the following is not
(b) 2 only identified as its objective? (2013)
(c) Both 1 and 2 (a) To provide self-governance
(d) Neither 1 nor 2 (b) To recognize traditional rights
(c) To create autonomous regions in tribal
3. With reference to “Gram Nyayalayas Acts”, areas
which of the following statements is/are (d) To free tribal people from exploitation
correct? (2016)
1. As per the Act, Gram Nyayalayas can 6. Under the Scheduled Tribes and Other
hear only civil cases and not criminal Traditional Forest Dwellers (Recognition of
cases. Rights) Act, 2006, who shall be the authority
2. The Act allows local social activists as to initiate the process of determining
mediators/conciliators. the nature and extent of individual or
Select the correct answer using the code community forest rights or both? (2013)
given below. (a) State Forest Department
(a) 1 only (b) District Collector/ Deputy Commissioner
(b) 2 only (c) Tahsildar/ Block Development Officer/
(c) Both 1 and 2 Mandal Revenue Officer

(d) Neither 1 nor 2 (d) Gram Sabha

422 Local Self Government


unacademy.com | Download the Unacademy app
Give your feedback here: Link
7. In the areas covered under the Panchayat 2. State Election Commissions to conduct
(Extension to the Scheduled Areas) Act, all Panchayat elections.
1996, what is the role/power of Gram 3. Establishment of State Finance
Sabha? (2012) Commissions.
1. Gram Sabha has the power to prevent Select the correct answer using the code
alienation of land in the Scheduled given below:
Areas.
(a) 1 only
2. Gram Sabha has the ownership of minor
forest produce. (b) 1 and 2 only

3. Recommendation of Gram Sabha (c) 2 and 3 only


is required for granting prospecting (d) 1, 2 and 3
licence or mining lease for any mineral
in the Scheduled Areas.
9. Consider the following statements: (2011)
Which of the statements given above is/are
In India, a Metropolitan Planning Committee
correct?
1. is constituted under the provisions of
(a) 1 only
the Constitution of India.
(b) 1 and 2 only
2. prepares the draft development plans
(c) 2 and 3 only for the metropolitan area.
(d) 1, 2 and 3 3. has the sole responsibility for
implementing Government sponsored
8.
The Constitution (Seventy-Third schemes in the metropolitan area.
Amendment) Act, 1992, which aims at Which of the statements given above is/are
promoting the Panchayati Raj Institutions correct?
in the country, provides for which of the (a) 1 and 2 only
following? (2011)
(b) 2 only
1. Constitution of District Planning
(c) 1 and 3 only
Committees.
(d) 1, 2 and 3

Local Self Government 423


unacademy.com | Download the Unacademy app
Give your feedback here: Link
Local Self Government-
6 Explanation
1. Answer: (b) access to justice to the citizens at their
Option (b) is correct: Local self-government doorsteps and to ensure that opportunities for
has been entrenched in all the states of securing justice are not denied to any citizen
India by the acts of the State Legislatures due to social, economic, or other disabilities.
to build democracy at the grass-root level. The Gram Nyayalaya shall be a mobile court
It is entrusted with rural development. It and shall exercise the powers of both Criminal
was Constitutionalised through the 73rd and Civil Courts.
Constitutional Amendment Act of 1992. Statement 2 is correct: The Gram Nyayalayas
Balwant Rai G Mehta committee submitted its try to settle the disputes as far as possible
report in November 1957 and suggested the by bringing about conciliation between the
establishment of the scheme of ‘democratic parties and for this purpose, it makes use of
decentralisation’, which finally came to be the mediators or conciliators to be appointed
known as Panchayati Raj. for this purpose. Social activists and people
involved with Non-Governmental Organisations
provide their service as a mediator or conciliator
2. Answer: (b) for quick and easy resolution of disputes.
Statement 1 is not correct: The 73rd Constitution
amendment Act of 1992 provided for a three- 4. Answer: (c)
tier system of Panchayati raj in every state, that
is, panchayats at the village, intermediate, and Option (c) is correct: Panchayati Raj functions
district levels. All the members of panchayats as a system of governance, with people’s
at the village, intermediate and district levels participation in development, in which gram
shall be elected directly by the people. 21 panchayats are the basic units of local
years to be the minimum age for contesting administration. The system has three levels:
elections to panchayats. Gram Panchayat (village level), Mandal Parishad
or Block Samiti of Panchayat Samiti (block
Statement 2 is correct: A panchayat constituted level), and Zila Parishad (district level).
upon the dissolution of a panchayat before the
expiration of its duration shall continue only Balwant Rai G Mehta Committee submitted its
for the remainder of the period for which the report in November 1957 and recommended the
dissolved panchayat would have continued establishment of the scheme of ‘democratic
had it not been so dissolved. In other words, decentralisation’, which ultimately came to be
a panchayat reconstituted after premature known as Panchayati Raj.
dissolution does not enjoy the full period of Elimination Technique
five years but remains in office only for the
remainder of the period. The fundamental object of the Panchayati
Raj system is to ensure People’s
participation in development and
3. Answer: (b) Democratic decentralisation.
Statement 1 is not correct: The Gram Nyayalayas
Act, 2008 has been enacted to provide for the 5. Answer: (c)
establishment of the Gram Nyayalayas at the
grass roots level for the purposes of providing PESA is a law enacted by the Government of
India to cover the “Scheduled Areas”, which

424 Local Self Government-Explanation


unacademy.com | Download the Unacademy app
Give your feedback here: Link
are not covered in the 73rd Constitutional thereafter forward a copy of the same to the
Amendment. This particular Act extends the Sub-Divisional Level Committee.
provisions of Part IX to the Scheduled Areas of
the country.
7. Answer: (b)
y PESA brought powers further down to the
The Panchayat (Extension to the Scheduled
Gram Sabha level. The Gram Sabha in the
Areas) Act was enacted by the Government
Panchayat Act was entrusted with wide-
of India to cover the “Scheduled Areas”, which
ranging powers starting from consultation
are not covered in the 73rd Constitutional
on land acquisition to that of ownership
amendment.
over minor forest produces and leasing of
minor minerals. This particular Act extends the provisions of
Part IX to the Scheduled Areas of the country.
y The PESA Act enables Tribal Self Rule in their
PESA brought powers further down to the
areas. The Act extended the provisions of
Gram Sabha level. Under PESA Act, the Gram
Panchayats to the tribal areas of nine states
Sabha are endowed specifically with:
that have Fifth Schedule Areas. Most of the
Northeastern states under Sixth Schedule y The power to enforce prohibition or
Areas (where autonomous councils exist) to regulate or restrict the sale and
are not covered by PESA, as these states consumption of any intoxicant.
have their own Autonomous councils for y The power to prevent alienation of land in
governance. (Option (c) is correct) the Scheduled Areas and to take appropriate
action to restore any unlawfully alienated
land of a Scheduled Tribe. (Statement 1 is
6. Answer: (d)
correct)
Scheduled Tribes and Other Traditional Forest
y The ownership of minor forest produce.
Dwellers (Recognition of Forest Rights) Act,
(Statement 2 is correct)
2006, has been enacted to recognize and vest
the forest rights and occupation of forest land y The power to manage village markets.
in forest-dwelling Scheduled Tribes and other y The recommendations of the Gram Sabha
traditional forest dwellers, who have been or the Panchayats at the appropriate level
residing in such forests for generations, but shall be made mandatory prior to grant
whose rights could not be recorded. of prospecting licence or mining lease for
Option (d) is correct: Under Section 6(1) of the minor minerals in the Scheduled Areas.
Scheduled Tribes and Other Traditional Forest (Statement 3 is not correct).
Dwellers (Recognition of Rights) Act, 2006, y Every Gram Sabha shall be competent
the Gram Sabha is the authority to initiate the to safeguard and preserve the traditions
process for determining the nature and extent and customs of the people, their cultural
of individual or community forest rights or identity, community resources and the
both that may be given to the forest-dwelling customary mode of dispute resolution.
Scheduled Tribes and other traditional forest
dwellers within the local limits of its jurisdiction
under the Act by receiving claims, consolidating 8. Answer: (d)
and verifying them and preparing a map Option 1 is correct: The 73rd and the 74th
delineating the area of each recommended Constitutional Amendment Acts, 1992 enjoin
claim in such manner as may be prescribed for upon the states to establish a three-tier system
the exercise of such rights and the Gram Sabha of Panchayats at the village, intermediate and
shall, then, pass a resolution to that effect and district levels, and Municipalities in the urban

Local Self Government-Explanation 425


unacademy.com | Download the Unacademy app
Give your feedback here: Link
areas, respectively. States are expected to 9. Answer: (a)
devolve adequate powers, responsibilities, The 74th Amendment Act,1992 has added a new
and finances upon these bodies so as to Part IX-A to the Constitution of India. This Part
enable them to prepare plans and implement is entitled as ‘The Municipalities’ and consists
schemes for economic development and social of provisions from Articles 243-P to 243-ZG. In
justice. This Act mandated the establishment addition, the Act has also added a new Twelfth
of the District Planning Committee (DPC) for Schedule to the Constitution. This Schedule
consolidating plans prepared by Panchayats contains 18 functional items of Municipalities.
and Municipalities in the district into the Draft The Act gave constitutional status to the
District Plan. Municipalities.
Option 2 is correct: The Constitution (Seventy- Statement 1 is correct: Metropolitan Planning
Third Amendment) Act, 1992, provided for the Committee was formed through 74th
formation of the State Election Commission Amendment Act, 1992.
for the superintendence, direction, and control
of the preparation of electoral rolls and the Statement 2 is correct: Every metropolitan area
conduct of all elections to the Panchayats. shall have a Metropolitan Planning Committee
to prepare a draft development plan.
Option 3 is correct: The Constitution (Seventy-
Third Amendment) Act, 1992 mandated that the Statement 3 is not correct: Metropolitan
Governor of a state shall, after every five years, Planning Committee does not have the sole
constitute a State Finance Commission to responsibility for implementing Government
review the financial position of the Panchayats. sponsored schemes in the metropolitan area.

426 Local Self Government-Explanation


unacademy.com | Download the Unacademy app
Give your feedback here: Link
7 Governance

1. With reference to India, consider the 3. With reference to the “Tea Board” in India,
following pairs: (2023) consider the following statements: (2022)
Action: The Act under which it is covered 1. The Tea Board is a statutory body.
1. Unauthorized wearing of police or 2. It is a regulatory body attached to the
military uniforms : The Official Secrets Ministry of Agriculture and Farmers
Act, 1923 Welfare.
2. Knowingly misleading or otherwise 3. The Tea Board’s Head Office is situated
interfering with a police officer or in Bengaluru.
military officer when engaged in their 4. The Board has overseas offices at Dubai
duties : The Indian Evidence Act, 1872 and Moscow.
3. Celebratory gunfire which can endanger Which of the statements given above are
the personal safety of others : The Arms correct ?
(Amendment) Act, 2019
(a) 1 and 3
How many of the above pairs are correctly
matched? (b) 2 and 4

(a) Only one (c) 3 and 4

(b) Only two (d) 1 and 4

(c) All three


(d) None 4. With reference to the Union Government,
consider the following statements: (2021)
1. N. Gopalaswamy Iyengar Committee
2. Consider the following statements: (2022) suggested that a minister and a
1. The India Sanitation Coalition is a secretary be designated solely for
platform to promote sustainable pursuing the subject of administrative
sanitation and is funded by the reform and promoting it.
Government of India and the World 2. In 1970, the Department of Personnel
Health Organization. was constituted on the Administrative
2. The National Institute of Urban Affairs is Reforms Commission, 1966, and this
an apex body of the Ministry of Housing was placed under the Prime Minister’s
and Urban Affairs in Government of charge.
India and provides innovative solutions Which of the statements given above is/are
to address the challenges of Urban correct?
India.
(a) 1 only
Which of the statements given above is/are
correct? (b) 2 only

(a) 1 only (c) Both 1 and 2

(b) 2 only (d) Neither 1 nor 2

(c) Both 1 and 2


(d) Neither 1 nor 2

Governance 427
unacademy.com | Download the Unacademy app
Give your feedback here: Link
5. At the national level, which ministry is create durable assets like physical
the nodal agency to ensure effective infrastructure for health, education,
implementation of the Scheduled Tribes etc.
and Other Traditional Forest Dwellers 2. A specified portion of each MP's fund
(Recognition of Forest Rights) Act, 2006? must benefit SC/ST populations.
(2021)
3. MPLADS funds are sanctioned on a
(a) Ministry of Environment, Forest and yearly basis and the unused funds
Climate Change cannot be carried forward to the next
(b) Ministry of Panchayati Raj year.
(c) Ministry of Rural Development 4. The district authority must inspect
(d) Ministry of Tribal Affairs at least 10% of all works under
implementation every year.
Select the correct answer using the code
6. Consider the following statements in given below:
respect of Bharat Ratna and Padma Awards:
(2021) (a) 1 and 2 only

1. Bharat Ratna and Padma Awards are (b) 3 and 4 only


titles under the Article 18 (1) of the (c) 1, 2 and 3 only
Constitution of India. (d) 1, 2 and 4 only
2. Padma Awards, which were instituted
in the year 1954, were suspended only
once. 8. Consider the following statements: (2020)

3. The number of Bharat Ratna Awards 1. Aadhaar metadata cannot be stored for
is restricted to a maximum of five in a more than three months.
particular year. 2. State cannot enter into any contract
Which of the above statements are not with private corporations for sharing of
correct? Aadhar data.

(a) 1 and 2 only 3. Aadhaar is mandatory for obtaining


insurance products.
(b) 2 and 3 only
4. Aadhar is mandatory for getting benefits
(c) 1 and 3 only funded out of the Consolidated fund of
(d) 1, 2 and 3 India.
Which of the statements given above is/are
7. With reference to the funds under Members correct?
of Parliament Local Area Development (a) 1 and 4 only
Scheme (MPLADS), which of the following (b) 2 and 4 only
statements are correct? (2020)
(c) 3 only
1. MPLADS funds must be used to
(d) 1, 2 and 3 only

428 Governance
unacademy.com | Download the Unacademy app
Give your feedback here: Link
7 Governance-Explanation

1. Answer: (b) partnership model.


Statement 1 is correct: THE OFFICIAL SECRETS Statement 2 is correct: The National Institute
ACT, 1923 - Section 6 in The Official Secrets of Urban Affairs (NIUA) is India’s leading national
Act, 1923 6. Unauthorised use of uniforms, think tank on urban planning and development.
falsification of reports, forgery, personation As a hub for generation and dissemination of
and false documents.— (1) If any person for the cutting-edge research in the urban sector,
purpose of gaining admission or of assisting any NIUA seeks to provide innovative solutions to
other person to gain admission to a prohibited address the challenges of a fast-urbanising
place or for any other purpose prejudicial to India. It has worked closely with the Ministry
the safety of the State— (a) uses or wears, of Housing and Urban Affairs, alongside other
without lawful authority, any naval, military, air government and civil sectors, to identify key
force, police or other official uniform, or any areas of research, and address the lacunae in
uniform so nearly resembling the same as to urban policy and planning.
be calculated to deceive, or falsely represents
himself to be a person who is or has been
3. Answer: (d)
entitled to use or wear any such uniform.
Statements 1 and 4 are correct: The Tea Board
Statement 2 is incorrect: THE OFFICIAL
was established on 1st April 1954 as a statutory
SECRETS ACT, 1923- No person in the vicinity of
body in accordance with Section (4) of the Tea
any prohibited place shall obstruct, knowingly
Act 1953. As the apex body, it takes care of the
mislead or otherwise interfere with or impede,
overall development of the tea industry. The
any police officer, or any member of 3 [the
Board is headed by a Chairman and consists
Armed Forces of the Union] engaged on guard,
of 30 members appointed by the Government
Sentry, patrol, or other similar duty in relation
of India representing different sections of the
to the prohibited place.
tea industry. For the purpose of tea promotion,
Statement 3 is correct: ARMS (AMENDMENT) three overseas offices are located in London,
ACT, 2019- Whoever uses firearm in a rash Dubai and Moscow.
or negligent manner or in celebratory gunfire
Statements 2 and 3 are not correct: It is a
so as to endanger human life or personal
regulatory body attached to the Ministry of
safety of others shall be punishable with an
Commerce and Industry. The Head Office of
imprisonment for a term which may extend to
the Board is located at Kolkata and has two
two years, or with fine which may extend to
Regional Offices in the North-East Zone at
rupees one lakh, or with both.
Jorhat in Assam and Coonoor in Tamil Nadu
in the South Zone. In addition, there are 18
2. Answer: (b) regional offices spread across all major tea
Statement 1 is not correct: India Sanitation producing states and four metropolitan cities.
Coalition (ISC) is a multi-stakeholder platform
that brings together the private sector, 4. Answer: (b)
government, financial institutions, civil society
Statement 1 is not correct: Sh. N. Gopalaswamy
groups, media, donors/bi-lateral/multilateral,
Ayyangar, in his Report, ‘ Reorganization of the
experts etc. to work in the sanitation space
Machinery of Central Government’ in 1950,
to drive sustainable sanitation through a
recommended the grouping of ministries,

Governance-Explanation 429
unacademy.com | Download the Unacademy app
Give your feedback here: Link
improvement in the personnel’s capabilities, instituted in 1954 to be awarded to citizens
and the working of the O&Ms Division. It nowhere of India in recognition of their distinguished
mentions that a minister and a secretary be contribution in various spheres of activity.
designated solely for pursuing the subject of The government suspended the practice of
administrative reform and promoting it. granting the Padma awards for two years in
Statement 2 is correct: In 1970, based on 1977. It was again suspended in mid-1992
the recommendations of the Administrative when 2 PILs were filed in the High Courts of
Reforms Commission, the Department of India. They are given in three categories: Padma
Personnel was set up in the Cabinet Secretariat. Vibhushan (for exceptional and distinguished
It was placed under the Prime Minister’s Office. service), Padma Bhushan (distinguished service
The Administrative Reforms Commission had in of higher-order) and Padma Shri (distinguished
its report on Machinery of the Government of service).
India, and its procedure of work recommended Statement 3 is not correct: A maximum of 3
inter-alia setting up of a separate Department people can be awarded the Bharat Ratna. The
of Personnel directly under the Prime Minister. total number of Padma awards to be conferred
each year is limited to 120. But the count
excludes posthumous awards and any non-
5. Answer: (d)
resident Indian or Overseas Citizen of India or
Option (d) is correct: The Ministry of Tribal foreign-based winners.
Affairs is the nodal agency at the national level
to ensure the effective implementation of the
Scheduled Tribes and Other Traditional Forest 7. Answer: (d)
Dwellers (Recognition of Forest Rights) Act Statement 1 is correct: The funds under
2006. The preamble to the FRA clearly states Members of Parliament Local Area Development
it be ‘An Act to recognize and vest the forest Scheme have objectives to create durable
rights and occupation in forest land in forest- community assets and for the provision of
dwelling Scheduled Tribes and other traditional basic facilities including physical infrastructure
forest dwellers who have been residing in such for health, education, etc., based on locally felt
forests for generations but whose rights could needs.
not be recorded. It provides a framework for Statement 2 is correct: There is a special focus
recording the forest rights so vested and the on areas inhabited by Scheduled Caste and
nature of evidence required for such recognition Scheduled Tribe populations by earmarking 15%
and vesting in respect of forest land. and 7.5 % of the MPLADS funds, respectively.
Out of an amount of Rs.5 Crores, the M.P. shall
6. Answer: (d) recommend for areas inhabited by the SC
population, Rs.75 lakhs, and Rs.37.5 lakhs for
Statement 1 is not correct: National awards
areas inhabited by ST population.
such as Bharat Ratna, Padma Vibhushan, Padma
Bhushan and Padma Shri do not amount to Statement 3 is not correct: The annual
titles within the meaning of Article 18(1) of the entitlement of Rs 5 crore shall be released,
Constitution. Article 18(1) abolishes all titles. It in two equal installments of Rs 2.5 crore
prohibits the State to confer titles on anybody, each, by Government of India directly to the
whether a citizen or a non-citizen. Military and District Authority based on unspent amount
academic distinctions are, however, exempted and utilisation certificate. The funds are non-
from the prohibition. Thus, a university can lapsable and can be carried forward.
give a title of honour to a man of merit. Statement 4 is correct: The District Authority
Statement 2 is not correct: Padma Awards were would be responsible for overall coordination

430 Governance-Explanation
unacademy.com | Download the Unacademy app
Give your feedback here: Link
and supervision of the works under the scheme the Aadhaar Act, enabling body corporate and
at the district level and inspect at least 10% individuals to seek authentication, has been
of the works under implementation every year. held unconstitutional by the Supreme Court.
The District Authority should involve the MPs Statement 3 is not correct: In the 2018
in the inspections of projects to the extent judgement, the Supreme Court held that Aadhar
feasible. is not mandatory to purchase the insurance
products, however, it can be voluntarily used
8. Answer: (b) for KYC of the insurance products. Even IRDA
has made similar rules.
Statement 1 is not correct: A Constitution
Bench of the Supreme Court ruled that the Statement 4 is correct: The Supreme Court
Aadhaar metadata cannot be stored beyond ruled that any welfare scheme, driving funds
six months. out of the Consolidated Fund of India, would
require Aadhar to deliver the benefits to the
Statement 2 is correct: Part of Section 57 of intended beneficiaries.

Governance-Explanation 431
unacademy.com | Download the Unacademy app
Give your feedback here: Link
Constitutional and Non-
8 constitutional Bodies
1. Consider the following organizations/ decides the election schedule for the
bodies in India: (2023) conduct of both general elections and
1. The National Commission for Backward by-elections.
Classes 3. Election Commission resolves the
2. The National Human Rights Commission disputes relating to splits/mergers of
recognized political parties.
3. The National Law Commission
Which of the statements given above is/are
4. The National Consumer Disputes correct?
Redressal Commission
(a) 1 and 2 only
How many of the above are constitutional
bodies? (b) 2 only

(a) Only one (c) 2 and 3 only

(b) Only two (d) 3 only

(c) Only three


(d) All four 4. Consider the following statements: (2013)
Attorney General of India can

2. Consider the following statements : (2022) 1. take part in the proceedings of the Lok
Sabha
1. Attorney General of India and Solicitor
General of India are the only officers 2. be a member of a Committee of the Lok
of the Government who are allowed Sabha
to participate in the meetings of the 3. speak in the Lok Sabha
Parliament of India. 4. vote in the Lok Sabha
2. According to the Constitution of India, Which of the statements given above is/are
the Attorney General of India submits correct?
his resignation when the Government
which appointed him resigns. (a) 1 only

Which of the statements given above is/are (b) 2 and 4 only


correct ? (c) 1, 2 and 3 only
(a) 1 only (d) 1 and 3 only
(b) 2 only
(c) Both 1 and 2 5. Which of the following bodies does not/
(d) Neither 1 nor 2 do not find mention in the Constitution?
(2013)
1. National Development Council
3. Consider the following statements: (2017)
2. Planning Commission
1. The Election Commission of India is a
five-member body. 3. Zonal Councils

2. The Union Ministry of Home Affairs Select the correct answer using the code
given below:

432 Constitutional and Non-constitutional Bodies


unacademy.com | Download the Unacademy app
Give your feedback here: Link
(a) 1 and 2 only emergency/financial emergency.
(b) 2 only 2. CAG reports on the execution of projects
(c) 1 and 3 only or programmes by the ministries are
discussed by the Public Accounts
(d) 1, 2 and 3
Committee.
3. Information from CAG reports can be
6. Which of the following is /are among the used by investigating agencies to press
noticeable features of the recommendations charges against those who have violated
of the Thirteenth Finance Commission? the law while managing public finances.
(2012)
4. While dealing with the audit and
1. A design for the Goods and Services Tax, accounting of government companies,
and a compensation package linked to CAG has certain judicial powers for
adherence to the proposed design. prosecuting those who violate the law.
2. A design for the creation of lakhs of jobs Which of the statements given above is/are
in the next ten years in consonance correct?
with India’s demographic dividend.
(a) 1, 3 and 4 only
3. Devolution of a specified share of
(b) 2 only
central taxes to local bodies as grants.
(c) 2 and 3 only
Select the correct answer using the code
given below: (d) 1, 2, 3 and 4

(a) 1 only
(b) 2 and 3 only 8. With reference to the Finance Commission
of India, which of the following statements
(c) 1 and 3 only
is correct? (2011)
(d) 1, 2 and 3
(a) It encourages the inflow of foreign
capital for infrastructure development
7. In India, other than ensuring that public (b) It facilitates the proper distribution
funds are used efficiently and for intended of finances among the Public Sector
purpose, what is the importance of the Undertakings
office of the Comptroller and Auditor
(c) It ensures transparency in financial
General (CAG)? (2012)
administration
1. CAG exercises exchequer control on
(d) None of the statements (a), (b) and (c)
behalf of the Parliament when the
given above is correct in this context
President of India declares national

Constitutional and Non-constitutional Bodies 433


unacademy.com | Download the Unacademy app
Give your feedback here: Link
Constitutional and Non-
8 constitutional Bodies-Explanation
1. Answer: (a) 2. Answer: (d)
1. The National Commission for Backward Statement 1 is incorrect
Classes: It is a statutory body rather Article 88: Rights of Ministers and Attorney
than a constitutional body. It was General in respects Houses Every Minister and
established under the National the Attorney General of India shall have the right
Commission for Backward Classes Act, to speak in, and otherwise to take part in the
1993, to investigate and recommend proceedings of either House, any joint sitting of
measures for the advancement of the Houses, and any committee of Parliament
socially and educationally backward of which he may be named a member, but shall
classes. not by virtue of this article be entitled to vote
2. The National Human Rights Commission: Officers of Parliament.
It is a constitutional body in India. It No mention of Solicitor General of India.
was established under the Protection of
Human Rights Act, 1993, in accordance Statement 2 is incorrect
with the provisions of the Constitution. Article 76: The Attorney General shall hold office
The NHRC is responsible for protecting during the pleasure of the President, and shall
and promoting human rights and has receive such remuneration as the President may
the power to inquire into violations determine Conduct of Government Business.
of human rights and recommend
Elimination Technique
necessary actions.
The Solicitor General of India is not
3. The National Law Commission: The
allowed to participate in the meetings of
National Law Commission is not a
the Parliament of India. Hence statement
constitutional body. It is a statutory
1 is incorrect.
body that was established in 1955 by
an executive order of the Government
of India. The commission’s primary role 3. Answer: (d)
is to study and review existing laws,
The Election Commission is a permanent
suggest reforms, and propose new
and independent body established by the
legislations to promote justice and legal
Constitution of India directly to ensure free
reforms in the country.
and fair elections in the country.
4. The National Consumer Disputes
Statement 1 is not correct: According to Article
Redressal Commission: The National
324, the Election Commission shall consist
Consumer Disputes Redressal
of the Chief Election Commissioner and such
Commission (NCDRC) is not a
number of other Election Commissioners, if
constitutional body. It is a statutory
any, as the President may form a time-to-time
body established under the Consumer
fix. At present, there are two members apart
Protection Act, 1986. The NCDRC is
from the Chief Election Commissioner.
responsible for adjudicating consumer
disputes at the national level and Statement 2 is not correct: Article 324 of
providing redressal mechanisms for the Constitution provides that the power of
consumers. superintendence, direction, and control of
elections to Parliament, State Legislatures, the

434 Constitutional and Non-constitutional Bodies-Explanation


unacademy.com | Download the Unacademy app
Give your feedback here: Link
office of President of India and the office of Commission and Zonal Councils, do not find
vice-President of India shall be vested in the mention in the Indian Constitution.
Election Commission. y National Development Council: It was
Statement 3 is correct: The Commission also set up on 6th August 1952 to strengthen
resolves the disputes relating to splits/mergers and mobilise the effort and resources
of recognized political parties. However, the of the nation in support of the plan, to
decision of the ECI can be challenged in the promote common economic policy in all
Courts. vital spheres, and to ensure the balanced
and rapid development of all parts of the
Elimination Technique country. It is not a Constitutional Body.
y The Election Commission of India is a y Planning Commission: The Planning
three-member body. Commission was set up in pursuance of
y The Election Commission decides the declared objectives of the government to
election schedule for the conduct promote a rapid rise in the standard of
of both general elections and by- living of the people by efficient exploitation
elections. of the resources of the country, increasing
y Hence 1 and 2 are incorrect. production and offering opportunities to
all for employment in the service of the
community. It is neither a Constitutional
4. Answer: (c) nor a Statutory Body.
The Attorney General for India is the Central y Zonal Councils: They are Advisory Councils
government’s Chief Legal Advisor, and its and are made up of the states of India that
primary lawyer in the Supreme Court of India. have been grouped into five zones to foster
He is a part of the Union Executive. cooperation among them. These were set
y The Attorney General (AG) is appointed by up vide Part-III of the States Reorganisation
the President under Article 76 (1) of the Act, 1956.
Indian Constitution and is the highest law
officer of the country. He must be a person
6. Answer: (c)
who is qualified to be appointed a Judge
of the Supreme Court. The term of office The Finance Commission is constituted by the
of the AG is not fixed by the Constitution. President under Article 280 of the Constitution.
Further, the Constitution does not contain Its main work is to give recommendations on
the procedure and grounds for his removal. distribution of central tax revenues between
He holds office during the pleasure of the the Union and the States.
President. Option (c) is correct: The Thirteenth Finance
y The Attorney General of India has the right Commission was constituted under the
to speak and take part in the proceedings chairmanship of Dr. Vijay Kelkar. The Commission
of either House, any joint sitting of both the submitted its report in Parliament on February
Houses and any Committee of Parliament, 25, 2010. The major recommendations of the
without being entitled to vote. (Option (c) Commission were:
is correct) y The share of states in the net proceeds of
the shareable central taxes should be 32%.
This is 1.5 percentage-points higher than
5. Answer: (d)
the recommendation of the 12th finance
Option (d) is correct: All the three bodies, commission.
National Development Council, Planning

Constitutional and Non-constitutional Bodies-Explanation 435


unacademy.com | Download the Unacademy app
Give your feedback here: Link
y 2.5% share of the divisible pool for local of the country at both the levels–the Centre
bodies in the form of grants, as the and the state. His duty is to uphold the
Constitution does not allow the sharing of Constitution of India and laws of Parliament in
tax revenues with them. While 1.5% will the field of financial administration.
be constant, 1% will be on the basis of Statement 1 is not correct: The role of the
performance. Comptroller and Auditor General of India
y Revenue deficit to be progressively reduced (CAG) is to uphold the Constitution of India
and eliminated, followed by revenue surplus and the laws of Parliament in the field of
by 2013–2014. financial administration. The accountability
y Fiscal deficit to be reduced to 3% of the of the executive (i.e., council of ministers)
gross domestic product (GDP) by 2014– to the Parliament in the sphere of financial
2015. administration is secured through audit
reports of the CAG. He is responsible only to
y A target of 68% of GDP for the combined the Parliament.
debt of centre and states.
Statement 2 is correct: The CAG submits three
y Fiscal Responsibility and Budget audit reports to the President—audit report
Management Act, 2003 needs to be on appropriation accounts, audit report on
amended to mention the nature of shocks finance accounts, and audit report on public
which shall require targets relaxation. undertakings. The President lays these reports
y Both Centre and States should conclude before both the Houses of Parliament. After
‘Grand Bargain’ to implement the model this, the Public Accounts Committee examines
Goods and Services Act (GST). To incentivize them and reports its findings to the Parliament.
the states, the commission recommended Statement 3 is correct: Information from
a sanction of the grant of Rs 500 billion. CAG reports can be used by investigating
y Initiatives to reduce the number of Central agencies to press charges against those who
Sponsored Schemes (CSS) and to restore have violated the law while managing public
the predominance of formula-based plan finances. Later, the High Court of Karnataka
grants. in Karnataka Power Corporation vs Emta Coal
y States need to address the problem of Limited on 12 April 2016 has held that the
losses in the power sector in a time bound report of the Comptroller and Auditor General
manner. of India (CAG) cannot be the sole basis for any
liability being caused or for prosecution to be
Elimination Technique launched.
A design for the creation of lakhs of Statement 4 is not correct: While dealing
jobs in the next ten years was not the with the audit and accounting of government
recommendations of the Thirteenth companies, CAG does not have judicial powers
Finance Commission. Hence 2 is incorrect. for prosecuting those who violate the law.

7. Answer: (c) 8. Answer: (d)


The Constitution of India (Article 148) provides Article 280 of the Constitution of India provides
for an independent office of the Comptroller for a Finance Commission as a Quasi-Judicial
and Auditor General of India (CAG). He is Body. The Finance Commission consists of
the head of the Indian Audit and Accounts a Chairman and four other members to be
Department. He is the guardian of the public appointed by the President. They hold office for
purse and controls the entire financial system such a period as specified by the President in

436 Constitutional and Non-constitutional Bodies-Explanation


unacademy.com | Download the Unacademy app
Give your feedback here: Link
his order. They are eligible for reappointment. y The measures needed to augment the
The Finance Commission is required to make Consolidated Fund of a state to supplement
recommendations to the President of India on the resources of the panchayats and the
the following matters: municipalities in the state on the basis of
the recommendations made by the State
y The distribution of the net proceeds of Finance Commission.
taxes to be shared between the centre
and the states, and the allocation between y Any other matter referred to it by the
the states of the respective shares of such President in the interests of sound finance.
proceeds. Option (d) is correct: None of the statements
y The principles that should govern the (a), (b) and (c) given above is correct in this
grants-in-aid to the states by the centre context.
(i.e., out of the Consolidated Fund of India).

Constitutional and Non-constitutional Bodies-Explanation 437


unacademy.com | Download the Unacademy app
Give your feedback here: Link
9 Judicial & Quasi-Judicial Bodies

1. In India, Legal Services Authorities provide 3. With reference to National Legal Services
free legal services to which of the following Authority, consider the following
types of citizens? (2020) statements: (2013)
1. Person with an annual income of less 1. Its objective is to provide free and
than Rs. 1,00,000 competent legal services to the weaker
2. Transgender with an annual income of sections of the society on the basis of
less than Rs. 2,00,000 equal opportunity.

3. Member of Other Backward Classes 2. It issues guidelines for the State Legal
(OBC) with an annual income of less Services Authorities to implement the
than Rs. 3,00,000 legal programs and schemes throughout
the country.
4. All Senior Citizens
Which of the statements given above is/are
Select the correct answer using the code
correct?
given below:
(a) 1 only
(a) 1 and 2 only
(b) 2 only
(b) 3 and 4 only
(c) Both 1 and 2
(c) 2 and 3 only
(d) Neither 1 nor 2
(d) 1 and 4 only

4. With reference to the Delimitation


2. In India, which of the following review
Commission, consider the following
the independent regulators in sectors like
statements: (2012)
telecommunications, insurance, electricity,
etc.? (2019) 1. The orders of the Delimitation
Commission cannot be challenged in a
1. Ad Hoc Committees set up by the
Court of Law.
Parliament
2. When the orders of the Delimitation
2. Parliamentary Department Related
Commission are laid before the Lok
Standing Committees
Sabha or State Legislative Assembly,
3. Finance Commission they cannot effect any modifications in
4. Financial Sector Legislative Reforms the orders.
Commission Which of the statements given above is/are
5. NITI Aayog correct?

Select the correct answer using the code (a) 1 only


given below: (b) 2 only
(a) 1 and 2 only (c) Both 1 and 2
(b) 1, 3 and 4 only (d) Neither 1 nor 2
(c) 3, 4 and 5 only
(d) 2 and 5 only

438 Judicial & Quasi-Judicial Bodies


unacademy.com | Download the Unacademy app
Give your feedback here: Link
Judicial & Quasi-Judicial Bodies-
9 Explanation
1. Answer: (a) specific purpose and they cease to exist
Under the Legal Services Authorities Act, when they finish the task assigned to them
1987, the National Legal Services Authority, is and submit a report. They are appointed by
constituted to provide free legal services to the the House or the Speaker or the Presiding
weaker section of the society and to provide Officer of both the Houses in consultation
Lok Adalats to settle disputes amicably. with each other as and when necessary for
a particular purpose. They have also been
Statement 1 and statement 2 are correct: set up to investigate serious issues which
The sections of the society as enlisted under involve frauds or corruption on a large
Section 12 of the Legal Services Authorities Act scale.
are entitled for free legal services, they are:
y Parliamentary Department Related Standing
y A member of a Scheduled Caste or Committees: There are 24 departmentally
Scheduled Tribe. related Standing Committees in India.
y A victim of trafficking in human beings or They cover under their jurisdiction Central
beggar as referred to in Article 23 of the Ministries/ Departments and while
Constitution. reviewing their work they also look into the
y A woman or a child. working of the regulators in their specific
departments.
y A mentally ill or otherwise disabled person.
y A person under circumstances of
undeserved want such as being a victim 3. Answer: (c)
of a mass disaster, ethnic violence, caste Statement 1 is correct: The National Legal
atrocity, flood, drought, earthquake, or Services Authority (NALSA) has been constituted
industrial disaster; or under the Legal Services Authorities Act, 1987
y An industrial worker. to provide free Legal Services to the weaker
sections of the society and to organise Lok
y Income level: Low income (Annual income
Adalats for amicable settlement of disputes.
less than Rs 1,00,000), Senior citizen (Annual
income less than Rs 2,00,000), Transgender Statement 2 is correct: In every State, the
(Annual income less than Rs 2,00,000). State Legal Services Authority has been
constituted to give effect to the policies and
directions of the NALSA and to give free legal
2. Answer: (a) services to the people and conduct Lok Adalats
Option (a) is correct: In India, the Ad Hoc in the State. The NALSA lays down policies,
Committees set up by the Parliament and principles, guidelines and frames effective and
the Parliamentary Department related economic schemes for the State Legal Services
Standing Committees review the independent Authorities to implement the Legal Services
regulators in various sectors. Finance Programmes throughout the country.
Commission, Financial Sector Legislative y The State Legal Services Authority is
Reforms Commission, NITI Aayog have no role headed by the Hon’ble Chief Justice of the
in reviewing the independent regulators. respective High Court who is the Patron-in-
y Ad-Hoc Committees set up by the Chief of the State Legal Services Authority.
Parliament: They are appointed for a y

Judicial & Quasi-Judicial Bodies-Explanation 439


unacademy.com | Download the Unacademy app
Give your feedback here: Link
y In every District, District Legal Services Delimitation involves fixing the boundaries of
Authority has been constituted to constituencies. Article 82 of the Constitution
implement Legal Services Programmes authorises Parliament to enact a Delimitation
in the District. The District Legal Services Act after every Census which establishes a
Authority is situated in the District Courts delimitation commission. The main task of the
Complex in every District and chaired by commission is to redraw the boundaries of the
the District Judge of the respective district various assembly and Lok Sabha constituencies
Additional Information: to ensure an equitable population distribution.
Delimitation commissions have been set up by
y Article 39A of the Constitution of India
the parliament four times in the past under
provides for free legal aid to the poor and
‘Delimitation Commission Acts’ of 1952, 1962,
weaker sections of the society and ensures
1972 and 2002. Delimitation commission is a
justice for all.
high-powered body appointed by the president
y Articles 14 and 22(1) of the Constitution also of India once parliament enact the delimitation
make it obligatory for the State to ensure law. Its orders have a tender of law. Its orders
equality before law. cannot be questioned in a court of law.
Statement 2 is correct: When the orders of the
4. Answer: (c) Delimitation Commission are laid before the
Lok Sabha or State Legislative Assembly, they
The Delimitation Act, 2002, was enacted to set
cannot affect any modifications in the orders.
up a Delimitation Commission for the purpose
This means, the copies of the orders are laid
of effecting delimitation on the basis of the 2001
before the Lok Sabha and the respective
census so as to correct the aforesaid distortion
legislative assemblies, but no change in the
in the sizes of electoral constituencies. The
orders is permitted.
proposed Delimitation Commission would also
re-fix the number of seats for the Scheduled Elimination Technique
Castes and the Scheduled Tribes on the basis
The orders of the Delimitation Commission
of the 2001 census, without affecting the total
cannot be challenged in a Court. Hence 1
number of seats based on the 1971 census.
is correct.
Statement 1 is correct: The process of

440 Judicial & Quasi-Judicial Bodies-Explanation


unacademy.com | Download the Unacademy app
Give your feedback here: Link
9 INTERNATIONAL
RELATIONS
1 India's Foreign Policy

1. Consider the following countries: (2015) 2. With reference to "Look East Policy" of
1. China India, consider the following statements:
(2011)
2. France
1. India wants to establish itself as an
3. India important regional player in East Asian
4. Israel affairs.
5. Pakistan 2. India wants to plug the vacuum created
Which among the above are Nuclear by the termination of the Cold War.
Weapons States as recognized by the 3. India wants to restore the historical
Treaty on the Nonproliferation of Nuclear and cultural ties with its neighbours in
Weapons, commonly known as Nuclear Southeast and East Asia.
Non-Proliferation Treaty (NPT)? Which of the statements given above is/are
(a) 1 and 2 only correct?
(b) 1, 3, 4 and 5 only (a) 1 only
(c) 2, 4 and 5 only (b) 1 and 3 only
(d) 1, 2, 3, 4 and 5 (c) 3 only
(d) 1, 2 and 3

442 India's Foreign Policy


unacademy.com | Download the Unacademy app
Give your feedback here: Link
1 India's Foreign Policy-Explanation

1. Answer: (a) 2. Answer: (b)


Option (a) is correct: The Treaty on the Non- Option (b) is correct: The Look East policy
Proliferation of Nuclear Weapons (NPT), which emerged as an important foreign policy
entered into force in March 1970, seeks to initiative of India in the post-Cold War period.
inhibit the spread of nuclear weapons. Its It was launched in 1991 by the Narasimha Rao
190 (191 with North Korea*) states-parties are government to develop political contacts,
classified in two categories: nuclear-weapon increasing economic integration, and forging
states (NWS)- consisting of the United States, security cooperation with countries of
Russia, China, France, and the United Kingdom Southeast Asia. The policy marked a shift
and non-nuclear-weapon states (NNWS)- in India’s perspective of the world, with
remaining all other states. Under the treaty, the strategic and economic importance of
the five NWS commit to pursuing general Southeast Asia to India’s national interests
and complete disarmament, while the NNWS being recognised. The second phase, which
agree to forgo developing or acquiring nuclear began in 2003, extends the coverage of the
weapons. Look East policy from Australia to East Asia,
with the Association of Southeast Asian
Elimination Technique Nations (ASEAN) as its core. The new phase
India did not join or sign the Nuclear Non- thus marks a shift in focus from trade to wider
Proliferation Treaty (NPT). So, options economic and security cooperation, political
(b), (c), and (d) can be easily eliminated. partnerships, and physical connectivity through
Therefore, we can mark option (a) as the road and rail links.
correct answer.

India's Foreign Policy-Explanation 443


unacademy.com | Download the Unacademy app
Give your feedback here: Link
2 India & Its Neighbors

1. Recently, India signed a deal known countries will be strengthened.


as 'Action Plan for Prioritisation and (c) India will not depend on Pakistan for
Implementation of Cooperation Areas in the access to Afghanistan and Central Asia.
Nuclear Field' with which of the following
countries? (2019) (d) Pakistan will facilitate and protect the
installation of a gas pipeline between
(a) Japan Iraq and India.
(b) Russia
(c) The United Kingdom 3. ‘Belt and Road Initiative’ is sometimes
(d) The United States of America mentioned in the news in the context of
the affairs of: (2016)

2. What is the importance of developing (a) African Union


Chabahar Port by India? (2017) (b) Brazil
(a) India’s trade with African countries will (c) European Union
enormously increase. (d) China
(b) India’s relations with oil-producing Arab

444 India & Its Neighbors


unacademy.com | Download the Unacademy app
Give your feedback here: Link
2 India & Its Neighbors-Explanation

1. Answer: (b) Option (c) is correct: The first and foremost


Option (b) is correct: An ‘Action Plan for significance of the Chabahar port is the fact
Prioritization and Implementation of Co- that India can bypass Pakistan in transporting
operation Areas in the Nuclear Field’ was goods to Afghanistan and Central Asia.
jointly signed by India and Russia in October Chabahar port will boost India’s access to Iran,
2018 in New Delhi. The document was signed the key gateway to the International North-
by the Director General of Rosatom State South Transport Corridor that has sea, rail
Atomic Energy Corporation and Secretary of and road routes between India, Russia, Iran,
the Department of Atomic Energy and the Europe, and Central Asia. Chabahar port will
Chairman of the Atomic Energy Commission of be beneficial to India in countering Chinese
India. presence in the Arabian Sea, which China is
trying to ensure by helping Pakistan develop
With the signing of the document, the two the Gwadar port. Gwadar port is less than 400
countries plan to implement the project of six km from Chabahar by road and 100 km by sea.
nuclear power units of Russian design at a new
site in India, as well as further cooperation
in third countries in new promising areas of 3. Answer: (d)
nuclear technology, apart from the construction Option (d) is correct: The Belt and Road
of nuclear power plants. At present India and Initiative (BRI) is a transcontinental long-term
Russia are jointly involved in the Rooppur policy and investment program which aims at
Nuclear Power Plant project in Bangladesh. infrastructure development and acceleration
of the economic integration of countries along
2. Answer: (c) the route of the historic Silk Road. The Initiative
was unveiled by China in 2013.
BRI aims to promote the connectivity of
Asian, European, and African continents and
their adjacent seas, establish and strengthen
partnerships among the countries along the
Belt and Road, set up all-dimensional, multi-
tiered and composite connectivity networks,
and realise diversified, independent, balanced
and sustainable development in these
countries.

India & Its Neighbors-Explanation 445


unacademy.com | Download the Unacademy app
Give your feedback here: Link
International Groups and Political
3 Organizations
1. Consider the following statements about 3. With reference to the “United Nations
G-20: (2023) Credentials Committee”, consider the
1. The G-20 group was originally established following statements: (2022)
as a platform for the Finance Ministers 1. It is a committee set up by the UN
and Central Bank Governors to discuss Security Council and works under its
international economic and financial supervision.
issues. 2. It traditionally meets in March, June
2. Digital public infrastructure is one of and September every year.
India’s G-20 priorities. 3. It assesses the credentials of all UN
Which of the statements given above is/are members before submitting a report to
correct? the General Assembly for approval.
(a) 1 only Which of the statements given above is/are
(b) 2 only correct?

(c) Both 1 and 2 (a) 3 only

(d) Neither 1 nor 2 (b) 1 and 3


(c) 2 and 3

2. With reference to the United Nations (d) 1 and 2


Convention on the Law of Sea, consider the
following statements: (2022) 4. With reference to the United Nations
1. A coastal state has the right to establish General Assembly, consider the following
the breadth of its territorial sea up to statements :
a limit not exceeding 12 nautical miles, 1. The UN General Assembly can grant
measured from baseline determined in observer status to the non-member
accordance with the convention. States.
2. Ships of all states, whether coastal or 2. Inter-governmental organisations can
land-locked, enjoy the right of innocent seek observer status in the UN General
passage through the territorial sea. Assembly.
3. The Exclusive Economic Zone shall not 3. Permanent Observers in the UN General
extend beyond 200 nautical miles from Assembly can maintain missions at the
the baseline from which the breadth of UN headquarters.
the territorial sea is measured.
Which of the statements given above are
Which of the statements given above are correct?
correct ?
(a) 1 and 2 only
(a) 1 and 2 only
(b) 2 and 3 only
(b) 2 and 3 only
(c) 1 and 3 only
(c) 1 and 3 only
(d) 1, 2 and 3
(d) 1, 2 and 3

446 International Groups and Political Organizations


unacademy.com | Download the Unacademy app
Give your feedback here: Link
5. Consider the following : (2022) 2. The UNCAC is the ever-first legally
1. Asian Infrastructure Investment Bank binding global anti-corruption
instrument.
2. Missile Technology Control Regime
3. A highlight of the United Nations
3. Shanghai Cooperation Organisation
Convention against Transnational
India is a member of which of the above? Organised Crime (UNTOC) is the
(a) 1 and 2 only inclusion of a specific chapter aimed at
returning assets to their rightful owners
(b) 3 only
from whom they had been taken illicitly.
(c) 2 and 3 only
4. The United Nations Office on Drugs
(d) 1, 2 and 3 and Crime (UNODC) is mandated by
its member States to assist in the
implementation of both UNCAC and
6. With reference to the “G20 Common
UNTOC.
Framework”, consider the following
statements: (2022) Which of the statements given above are
correct?
1. It is an initiative endorsed by the G20
together with the Paris Club. (a) 1 and 3 only

2. It is an initiative to support Low Income (b) 2, 3 and 4 only


Countries with unsustainable debt. (c) 2 and 4 only
Which of the statements given above is/are (d) 1, 2, 3 and 4
correct?
(a) 1 only
9. With reference to Asian Infrastructure
(b) 2 only Investment Bank (AIIB), consider the
(c) Both 1 and 2 following statements: (2019)

(d) Neither 1 nor 2 1. AIIB has more than 80 member nations.


2. India is the largest shareholder in AIIB.

7. In which one of the following groups are all 3. AIIB does not have any members from
the four countries members of G20? (2020) outside Asia.

(a) Argentina, Mexico, South Africa, and Which of the statements given above is/are
Turkey correct?

(b) Australia, Canada, Malaysia, and New (a) 1 only


Zealand (b) 2 and 3 only
(c) Brazil, Saudi Arabia, and Vietnam (c) 1 and 3 only
(d) Indonesia, Japan, Singapore, and South (d) 1, 2 and 3
Korea

10. Which is /are the consequence/


8. Consider the following statements: (2019) consequences of becoming a member of
1. The United Nations Convention against the ‘Nuclear Supplier Group’? (2018)
Corruption (UNCAC) has a 'Protocol 1. It will have access to the latest and
against the Smuggling of Migrants by most effective nuclear technologies.
Land, Sea and Air'.
2. It automatically becomes a member

International Groups and Political Organizations 447


unacademy.com | Download the Unacademy app
Give your feedback here: Link
of “The Treaty of non-proliferation of 6. USA
Nuclear weapons (NPT)”. Which of the above are among the 'free-
Which of the statements given below is/ trade partners' of ASEAN?
are correct? (a) 1, 2, 4 and 5 only
(a) 1 only (b) 3, 4, 5 and 6 only
(b) 2 only (c) 1, 3, 4 and 5 only
(c) Both 1 and 2 (d) 2, 3, 4 and 6 only
(d) Neither 1 nor 2

14. With reference to ‘Asia Pacific Ministerial


11.
International Labour Organization's Conference on Housing and Urban
Conventions 138 and 182 are related to: Development (APMCHUD)’, consider the
(2018) following statements: (2017)
(a) Child Labour 1. The first APMCHUD was held in India
(b) Adaptation of agricultural practices to in 2006 on the theme ‘Emerging
global climate change Urban Forms — Policy Responses and
Governance Structure’.
(c) Regulation of food prices and food
security 2. India hosts all the Annual Ministerial
Conferences in partnership with ADB,
(d) Gender parity at the workplace APEC and ASEAN.
Which of the statements given above is/are
12. In the Indian context, what is the implication correct?
of ratifying the ‘Additional Protocol’ with (a) 1 only
the ‘International Atomic Energy Agency
(IAEA)’? (2018) (b) 2 only

(a) The civilian nuclear reactors come (c) Both 1 and 2


under IAEA safeguards. (d) Neither 1 nor 2
(b) The military nuclear installations come
under the inspection of IAEA. 15. Consider the following statements: (2017)
(c) The country will have the privilege to 1. India has ratified the Trade Facilitation
buy uranium from the Nuclear Suppliers Agreement (TFA) of WTO.
Group (NSG).
2. TFA is a part of WTO's Bali Ministerial
(d) The country automatically becomes a Package of 2013.
member of the NSG.
3. TFA came into force in January 2016.
Which of the statements given above are
13. Consider the following countries: (2018) correct?
1. Australia (a) 1 and 2 only
2. Canada (b) 1 and 3 only
3. China (c) 2 and 3 only
4. India (d) 1, 2 and 3
5. Japan

448 International Groups and Political Organizations


unacademy.com | Download the Unacademy app
Give your feedback here: Link
16. Consider the following in respect of the (c) 1, 2 and 3
(IONS): (2017) (d) India is a member of none of them
1. Inaugural IONS was held in India in 2015
under the chairmanship of the Indian
Navy. 19. With reference to a grouping of countries
known as BRICS, consider the following
2. IONS is a voluntary initiative that seeks statements: (2014)
to increase maritime cooperation
among navies of the littoral states of 1. The First Summit of BRICS was held in
the Indian Ocean Region. Rio de Janeiro in 2009.

Which of the above statements is/are 2. South Africa was the last to join the
correct? BRICS grouping.

(a) 1 only Which of the statements given above is/are


correct?
(b) 2 only
(a) 1 only
(c) Both 1 and 2
(b) 2 only
(d) Neither 1 nor 2
(c) Both 1 and 2
17. With reference to ‘Indian Ocean Rim
Association for Regional Cooperation (IOR- (d) Neither 1 nor 2
ARC)’, Consider the following statements:
(2015) 20. Consider the following countries: (2014)
1. It was established very recently in 1. Denmark
response to incidents of piracy and
2. Japan
accidents of oil spills.
3. Russian Federation
2. It is an alliance meant for maritime
security only. 4. United Kingdom
Which of the following statements given 5. United States of America
above is/are correct? Which of the above are the members of the
(a) 1 only 'Arctic Council'?
(b) 2 only (a) 1, 2 and 3
(c) Both 1 and 2 (b) 2, 3 and 4
(d) Neither 1 nor 2 (c) 1, 4 and 5
(d) 1, 3 and 5
18. India is a member of which of the following?
(2015) 21. Recently, the USA decided to support India's
1. Asia-Pacific Economic Cooperation membership in multilateral export control
regimes called the "Australia Group" and
2. Association of South-East Asian Nations
the "Wassenaar Arrangement". What is the
3. East Asia Summit difference between them? (2011)
Select the correct answer using the code 1. The Australia Group is an informal
given below: arrangement which aims to allow
(a) 1 and 2 only exporting countries to minimize the
(b) 3 only risk of assisting chemical and biological

International Groups and Political Organizations 449


unacademy.com | Download the Unacademy app
Give your feedback here: Link
weapons proliferation, whereas the the European Union and American
Wassenaar- Arrangement is a formal continents.
group under the OECD holding identical Which of the statements given above is/are
objectives. correct?
2.
The Australia Group comprises (a) 1 only
predominantly of Asian, African, and
North American countries, whereas (b) 2 only
the member countries of Wassenaar (c) Both 1 and 2
Arrangement are predominantly from (d) Neither 1 nor 2

450 International Groups and Political Organizations


unacademy.com | Download the Unacademy app
Give your feedback here: Link
International Groups and Political
3 Organizations-Explanation
1. Answer: (c) nautical miles from the baselines from which
Statement 1 is correct: The G20 (Group of the breadth of the territorial sea is measured,
Twenty) was indeed initially established as a by straight lines not exceeding 60 nautical miles
forum for Finance Ministers and Central Bank in length, connecting fixed points, defined by
Governors to discuss international economic the coordinates of latitude and longitude.
and financial matters. It was created in 1999
in response to the financial crises of the late 3. Answer: (a)
1990s and aimed to enhance cooperation and
Option (a) is correct: In most cases, accreditation
policy coordination among major economies.
at the UN is a formality whereby the General
Statement 2 is correct: India has placed a focus Assembly approves specific individuals to
on digital public infrastructure as one of its represent a Member State. According to this
priorities within the G20. India has recognized procedure, the Head of State or Government or
the potential of digital technologies to drive Minister for Foreign Affairs of a Member State
economic growth, enhance governance, and submits documentation to the UN Secretary-
improve public service delivery.Under the G20 General stating that the named individuals are
framework, India has advocated for initiatives entitled to represent that Member State. The
and policies that promote digital inclusion, documentation is referred to a Credentials
digital infrastructure development, and digital Committee appointed by the General Assembly
innovation. This includes efforts to expand at the beginning of each regular session (month
broadband connectivity, improve digital of September).
skills and literacy, promote digital payments,
Thus, the review of the credentials of
and harness emerging technologies such as
UN Member States is an annual process.
artificial intelligence and blockchain.
The Credentials Committee examines the
credentials and determines whether they are
2. Answer: (d) complete and have been issued by the proper
Statement 1 is correct: The coastal States authority, but it does not generally inquire
exercise sovereignty over their territorial sea into the legitimacy of the issuing authority.
which they have the right to establish its The Committee then submits a report to
breadth up to a limit not to exceed 12 nautical the General Assembly recommending either
miles; foreign vessels are allowed “innocent rejection or approval of the credentials of the
passage” through those waters. representatives of all Member States. Typically,
the General Assembly adopts the Committee’s
Statement 2 is correct: Under Article 17 of recommendations without discussion.
the UNCLOS III Right of Innocent Passage is
defined as “Subject to this Convention, ships
of all States, whether coastal or land-locked, 4. Answer: (d)
enjoy the right of innocent passage through According to the Charter of the United Nations,
the territorial sea”, and the Passage is defined the General Assembly may:
under Article 18.
Elect the non-permanent members of the
Statement 3 is correct: The coastal State shall Security Council and the members of other
delineate the outer limits of its continental United Nations councils and organs and, on
shelf, where that shelf extends beyond 200 the recommendation of the Security Council,

International Groups and Political Organizations-Explanation 451


unacademy.com | Download the Unacademy app
Give your feedback here: Link
appoint the Secretary-General. missiles and missile technology. India became
Make recommendations on maintaining a member of the MTCR in 2016.
international peace and security, including The Shanghai Cooperation Organisation (SCO)
disarmament. is a permanent intergovernmental international
Discuss any question relating to international organization, created on 15 June 2001 in
peace and security (except where a dispute or Shanghai (China). India became a member in
situation is currently being discussed by the 2017.
Security Council)
Make recommendations to promote 6. Answer: (c)
international political cooperation, the Statement 1 and Statement 2 are correct
development and codification of international
law, the realization of human rights and The Common Framework for Debt Treatments
fundamental freedoms, and international beyond the DSSI is an agreement of the G20
collaboration in the economic, social, and Paris Club countries to coordinate and
humanitarian, cultural, educational and health cooperate on debt treatments for up to 73
fields. low-income countries that are eligible for the
Debt Service Suspension Initiative (DSSI).
Make recommendations for the peaceful
settlement of any situation that might impair For countries with sustainable debt but liquidity
friendly relations among countries issues, it can provide a deferral of a portion of
debt service payments for a number of years
The Assembly may also take action in cases of that can ease financing pressures. This type of
a threat to the peace, breach of peace or act treatment is often referred to as rescheduling
of aggression, when the Security Council has or reprofiling. Such a debt treatment can also
failed to act owing to the negative vote of a benefit countries where high debt service
permanent member. payments are a source of debt vulnerability.
Statement 1 is correct: The UN General
Assembly can grant observer status to the
non-member States. 7. Answer: (a)

Statement 2 is correct: Inter-governmental The G20 is an informal group of 19 countries


organisations can seek observer status in the and the European Union, with representatives
UN General Assembly. of the International Monetary Fund and the
World Bank. The G20 membership comprises
Statement 3 is correct: Permanent Observers a mix of the world’s largest advanced and
in the UN General Assembly can maintain emerging economies, representing about
missions at the UN headquarters. two-thirds of the world’s population, 85% of
global gross domestic product, 80% of global
5. Answer: (d) investment and over 75% of global trade. The
G20 Presidency rotates annually according to
The Asian Infrastructure Investment Bank
a system that ensures a regional balance over
(AIIB) is a multilateral development bank that
time.
aims to improve economic and social outcomes
in Asia. India became a member in 2016. Option (a) is correct: The members of the G20
include Argentina, Australia, Brazil, Canada,
The Missile Technology Control Regime
China, France, Germany, India, Indonesia, Italy,
(MTCR) is a multilateral export control regime
Japan, Republic of Korea, Mexico, Russia,
whose members have an informal political
Saudi Arabia, South Africa, Turkey, the United
understanding to limit the proliferation of
Kingdom, the United States, and the European

452 International Groups and Political Organizations-Explanation


unacademy.com | Download the Unacademy app
Give your feedback here: Link
Union.
Crime and two of its supplementary Protocols
namely: The Protocol to Prevent, Suppress
and Punish Trafficking in Persons, Especially
Women and Children and the Protocol against
the Smuggling of Migrants by Land, Air and Sea.
This convention does not cover the aspect of
asset return to rightful owners.
Statement 4 is correct: United Nations Office
on Drugs and Crime (UNODC) is a global leader
in the fight against illicit drugs and international
crime. Established in 1997 through a merger
between the United Nations Drug Control
Programme and the Centre for International
Crime Prevention, UNODC operates in all
regions of the world through an extensive
network of field offices.

9. Answer: (a)
Asian Infrastructure Investment Bank (AIIB)
is a multilateral development bank that aims
to support the building of infrastructure in
the Asia-Pacific region. It is headquartered
in Beijing, China. It offers sovereign and non-
sovereign financing for sound and sustainable
8. Answer: (c) projects in energy and power, transportation
Statement 1 is not correct: The United Nations and telecommunications, rural infrastructure
Convention against Corruption (UNCAC) only and agricultural development, water supply
covers five main areas: preventive measures, and sanitation, environmental protection, and
criminalization and law enforcement, urban development and logistics.
international cooperation, asset recovery, and Statement 1 is correct: The bank currently has
technical assistance and information exchange. 103 member nations.
This convention does not cover the aspect of
Statement 2 is not correct: China is the largest
Smuggling of Migrants by Land, Sea and Air’.
shareholder (shareholding 30.77%) in AIIB
Statement 2 is correct: The United Nations followed by India (shareholding 8.64%), Russia
Convention against Corruption is the only (6.75%) and Germany (4.63%).
legally binding universal anti-corruption
Statement 3 is not correct: AIIB has 39 non-
instrument. The Convention’s far-reaching
regional members (members from outside Asia)
approach and the mandatory character of
as well. Some of them are Austria, Canada,
many of its provisions make it a unique tool
Finland, etc.
for developing a comprehensive response to a
global problem. y Elimination Technique: AIIB does not
Statement 3 is not correct: The General have any members from outside Asia.
Assembly adopted the United Nations Statement 3 is not correct. Thus,
Convention against Transnational Organised Option (a) is correct.

International Groups and Political Organizations-Explanation 453


unacademy.com | Download the Unacademy app
Give your feedback here: Link
10. Answer: (a) ILO Convention No. 182 is the first ILO
The Nuclear Suppliers Group (NSG) is a group Convention to achieve universal ratification. It
of nuclear supplier countries that seeks to was also the most rapidly ratified Convention
contribute to the non-proliferation of nuclear in the history of the ILO, with the majority of
weapons through the implementation of two ratifications occurring within the first 3 years
sets of guidelines for nuclear exports and after it was adopted in 1999. ILO Convention
nuclear-related exports. No. 138 has also been widely ratified by ILO
member States.
Statement 1 is correct: All nuclear-based
programmes of India are being run on indigenous
technology. By becoming a member of the 12. Answer: (a)
Nuclear Supplier Group (NSG), India will have The International Atomic Energy Agency (IAEA)
access to the latest and most effective nuclear is headquartered in Vienna, Austria. It is the
technologies. If India becomes a member, it world’s centre for cooperation in the nuclear
will also have a better international market for field and seeks to promote the safe, secure,
export as well as for import of nuclear-related and peaceful use of nuclear technologies. It was
materials. established as an autonomous organisation
Statement 2 is not correct: NPT (Non- on 29 July 1957. The IAEA reports to both
proliferation Treaty) is an international treaty, the United Nations General Assembly and the
which came into force in 1970. The main Security Council.
objective was to prevent the spread of nuclear Option (a) is correct: The Additional Protocol
weapons and weapons technology. The nuclear is not a stand-alone agreement, but rather
powers were convinced that the Nuclear Non- a protocol to a safeguards agreement that
Proliferation Treaty (NPT) alone would not halt provides additional tools for verification.
the spread of nuclear weapons. Also, joining the In particular, it significantly increases the
NSG will not automatically lead to becoming a IAEA’s ability to verify the peaceful use of all
member of “The Treaty of non-proliferation of nuclear material in States with comprehensive
Nuclear weapons (NPT). In a similar case in safeguards agreements. Hence, civilian nuclear
2008, India got a ‘clean waiver’ from NSG to do reactors come under IAEA safeguards
nuclear trade with member countries, where
in, it is forbidden to trade with a country who An Additional Protocol to the Safeguards
has not signed the NPT. Agreement between the Government of India
and the IAEA for the Application of Safeguards
to Civilian Nuclear Facilities entered into force
11. Answer: (a) on 25 July 2014.
Option (a) is correct: The two International
Labour Organization’s (ILO) Conventions on 13. Answer: (c)
child labour are Convention No. 138 on Minimum
Age and Convention No. 182 on the Worst The Association of Southeast Asian Nations
Forms of Child Labour. These Conventions are (ASEAN) was established in August 1967 in
“fundamental” Conventions. This means that, Bangkok, Thailand, with the signing of the
under the ILO Declaration on Fundamental ASEAN Declaration (Bangkok Declaration).
Principles and Rights at Work, all ILO member Currently, it has 10 member nations which are:
States have an obligation to respect, promote Brunei, Cambodia, Indonesia, Laos, Malaysia,
and realize the abolition of child labour, even Myanmar, the Philippines, Singapore, Thailand,
if they have not ratified the Conventions in and Vietnam.
question. Option (c) is correct: ASEAN has a total of 6 free

454 International Groups and Political Organizations-Explanation


unacademy.com | Download the Unacademy app
Give your feedback here: Link
trade partners namely: Australia, New Zealand, the best practices from around the world. It
South Korea, Japan, India, and China. ASEAN attempts to create an international framework
and its 5 free trade partners (excluding India) for reducing trade costs. It has provisions that
have entered into a regional trading agreement expedite the movement of goods, clearance
called the Regional Comprehensive Economic of goods and release of goods by simplifying
Partnership (RCEP). The purpose of RCEP was customs rules and regulations among the
to make it easier for products and services of countries.
each of these countries to be available across Statement 1 is correct: India ratified the Trade
this region. India has however chosen to stay Facilitation Agreement of WTO in 2016. India is
out of RCEP. the 76th WTO member to accept the TFA.
Statement 2 is correct: TFA was an outcome
14. Answer: (d) of the WTO’s 9th Bali (Indonesia) Ministerial
The Asia Pacific Ministerial Conference on package of 2013. The agreement includes
Housing and Urban Development (APMCHUD) provisions for:
is an inter-governmental mechanism for y Lowering import tariffs and agricultural
collaboration and cooperation in the field of subsidies: It makes it easier for developing
housing and urban development among the countries to trade with the developed
Asia Pacific countries. world in global markets.
APMCHUD is composed of the Biennial y Abolish hard import quotas: Developed
Ministerial Conference, the Bureau, and the countries would abolish hard import
Secretariat. It is represented by the Hon’ble quotas on agricultural products from the
Ministers of Housing and Urban Development developing world and instead would only
of the Asia Pacific countries. be allowed to charge tariffs on the amount
The Bureau of APMCHUD elected by the Biennial of agricultural imports exceeding specific
Conference of Ministers responsible for Housing limits.
and Urban Development holds office until the y Reduction in red tape at international
next Conference. The permanent Secretariat of borders: It aims to reduce red-tapism
APMCHUD is hosted by India in New Delhi. to facilitate trade by reforming customs
Statement 1 and statement 2 are not correct: bureaucracies and formalities.
The first APMCHUD was held in India in Statement 3 is not correct: It came into force
2006 on the theme, “A vision for sustainable in February 2017 and not in 2016.
urbanisation in the Asia-Pacific by 2020”, was
aimed at galvanising government action and
political commitment at the regional level to 16. Answer: (b)
improve the lives of 581 million slum dwellers. Statement 1 is not correct: The inaugural
Subsequent conferences were held in Tehran, IONS Seminar was held by the Indian Navy
Iran (2008); Solo, Central Java, Indonesia in 2008. Subsequent seminars and meetings
(2010); Amman, Jordan (2012); Seoul, Republic of the ‘Conclave of Chiefs’ have been held at
of Korea (2014); New Delhi, India (2016). the commencement of each two-year IONS
Chairmanship: the United Arab Emirates in
2010, South Africa in 2012, Australia in 2014,
15. Answer: (a) Bangladesh in 2016 and Iran in 2018.
The Trade Facilitation Agreement (TFA) sets In 2020 the IONS Chairmanship rotates to
forth a series of measures for expeditiously France and in 2022 to Thailand. IONS includes
moving goods across borders inspired by 24 nations that permanently hold territory that

International Groups and Political Organizations-Explanation 455


unacademy.com | Download the Unacademy app
Give your feedback here: Link
abuts or lies within the Indian Ocean and eight Asia-Pacific. APEC’s 21 members are Australia,
observer nations. Brunei Darussalam, Canada, Chile, People’s
Statement 2 is correct: The ‘Indian Ocean Naval Republic of China, Hong Kong, Indonesia, Japan,
Symposium’ (IONS) is a voluntary initiative that Republic of Korea, Malaysia, Mexico, New
seeks to increase maritime cooperation among Zealand, Papua New Guinea, Peru, Philippines,
navies of the littoral states of the Indian Ocean the Russian Federation, Singapore, Chinese
Region by providing an open and inclusive Taipei, Thailand, United States of America and
forum for discussion of regionally relevant Vietnam. India is not a member of the Asia-
maritime issues. In the process, it endeavours Pacific economic cooperation.
to generate a flow of information between Option 2 is not correct: The Association
naval professionals that would lead to common of Southeast Asian Nations (ASEAN), was
understanding and possibly cooperative established in August 1967 in Bangkok,
solutions on the way ahead. Thailand, with the signing of the ASEAN
Declaration (Bangkok Declaration) by the
Founding Fathers of ASEAN, namely Indonesia,
17. Answer: (d)
Malaysia, Philippines, Singapore and Thailand.
Statement 1 is not correct: Indian Ocean Rim Later, Brunei Darussalam, Viet Nam, Lao PDR,
Association for Regional Cooperation (IOR- Myanmar and Cambodia joined ASEAN, making
ARC), a regional cooperation initiative of the up what is today the ten Member States of
Indian Ocean Rim countries, was established ASEAN. India is not a member of ASEAN.
in Mauritius in March 1997 with the aim of
Option 3 is correct: The East Asia Summit
promoting economic and technical cooperation.
(EAS) is the Indo-Pacific’s premier forum for
It was not established in response to incidents
strategic dialogue. The EAS has 18 members
of piracy and accidents of oil spills.
- the ten ASEAN countries (Brunei, Cambodia,
Statement 2 is not correct: IOR-ARC is the only Indonesia, Laos, Malaysia, Myanmar, the
pan-Indian ocean grouping. It brings together Philippines, Singapore, Thailand, Vietnam)
countries from three continents having different along with Australia, China, India, Japan, New
sizes, economic strengths, and a wide diversity Zealand, the Republic of Korea, Russia and the
of languages, and cultures. It aims to create United States.
a platform for trade, and socio-economic and
cultural cooperation in the Indian Ocean rim Elimination Technique
area, which constitutes a population of about India is not a member of the Asia-Pacific
two billion people. Economic Cooperation (APEC). So, options
The Indian Ocean Rim Association (IORA) is (a) and (c) can be eliminated. Option (b) is
a dynamic inter-governmental organization most likely to be the answer.
aimed at strengthening regional cooperation
and sustainable development within the Indian 19. Answer: (b)
Ocean region through its 23 Member States
and 9 Dialogue Partners. BRICS is the acronym for an association of five
major emerging national economies: Brazil,
Russia, India, China, and South Africa. BRICS
18. Answer: (b) is an important grouping bringing together the
Option 1 is not correct: The Asia-Pacific major emerging economies from the world,
Economic Cooperation (APEC) is a regional comprising 41% of the world population, having
economic forum established in 1989 to 24% of the world GDP and over 16% share in
leverage the growing interdependence of the world trade.

456 International Groups and Political Organizations-Explanation


unacademy.com | Download the Unacademy app
Give your feedback here: Link
y The first BRIC (Brazil, Russia, India, and 2019, thirteen non-Arctic states that have
China) formal summit commenced on 16th Observer status are: Germany, Netherlands,
June 2009 in Yekaterinburg (Russia), when Poland, the United Kingdom, France, Spain,
South Africa was not a member of the BRIC China, India, Italy, Japan, South Korea,
group. (Statement 1 is not correct) Singapore, and Switzerland.
y The BRIC group was renamed as BRICS Elimination Technique
(Brazil, Russia, India, China, South Africa)
after South Africa was accepted as a full The Arctic Council is a council of eight
member at the BRIC Foreign Ministers’ Arctic states. Japan and the UK obviously
meeting in New York in September 2010. have none of its regions in the Arctic.
So, we can easily mark option (d) as the
y Accordingly, South Africa attended the 3rd correct answer.
BRICS Summit in Sanya, China on 14 April
2011 and this is the first Summit of South
Africa after becoming a member of the 21. Answer: (d)
BRIC group. Statement 1 is not correct: The Australia Group
y The grouping was originally known as BRIC (AG), on the other hand, is an informal forum of
(Brazil, Russia, India, and China) before the countries which, through the harmonisation of
inclusion of South Africa. South Africa was export controls, seeks to ensure that exports
the last to join the BRICS grouping in 2010, do not contribute to the development of
after it was accepted as a full member at chemical or biological weapons.
the BRIC Foreign Ministers’ meeting in New y The Wassenaar Arrangement, formally
York. (Statement 2 is correct) established in July 1996, is a voluntary
export control regime whose 42 members
20. Answer: (d) exchange information on transfers of
conventional weapons and dual-use goods
Option (d) is correct: The Arctic Council is a and technologies. Through such exchanges,
high-level intergovernmental body set up in Wassenaar aims to promote “greater
1996 by the Ottawa declaration to promote responsibility” among its members in
cooperation, coordination and interaction exports of weapons and dual-use goods and
among the Arctic States together with the to prevent “destabilising accumulations.” It
indigenous communities and other Arctic is not formal group under the OECD
inhabitants on common Arctic issues, in
particular on issues of sustainable development y Unlike its predecessor, the Cold War-era
and environmental protection in the Arctic. Coordinating Committee for Multilateral
Export Controls (COCOM), which was
y The Arctic Council works as a consensus- created to restrict exports to the former
based body to deal with issues such as Soviet Union and Eastern bloc, Wassenaar
the change in biodiversity, melting sea ice, is not targeted at any region or group of
plastic pollution, and black carbon. The states, but rather at “states of concern”
Council has members, ad hoc observer to members. Wassenaar members also
countries and “permanent participants”. lack veto authority over other member’s
y The Ottawa Declaration declares Canada, proposed exports, a power that COCOM
the Kingdom of Denmark, Finland, Iceland, members exercised.
Norway, the Russian Federation, Sweden and Statement 2 is not correct: In the Australia
the United States of America as a member Group, there are not many Asian and African
of the Arctic Council. Denmark represents countries. Only India (joined in 2018) and Japan
Greenland and the Faroe Islands. As of May

International Groups and Political Organizations-Explanation 457


unacademy.com | Download the Unacademy app
Give your feedback here: Link
(Joined in 1985) are the Asian countries in
the Australia Group and there is not a single
African country in that group. The Wassenaar
agreement has South Africa, Japan, from
Asia, but yes, the countries from the EU and
Americas are there.

458 International Groups and Political Organizations-Explanation


unacademy.com | Download the Unacademy app
Give your feedback here: Link
4 Places in news (2022)

1. Consider the following pairs: (2023) (a) Discovery of rich deposits of rare earth
elements
Regions often Reason for being in
mentioned in news (b) Establishment of Chinese military bases
news : (c) Southward expansion of Sahara Desert
1. North Kivu War between (d) Successful coups
and Ituri : Armenia and
Azerbaijan
4. Consider the following countries: (2022)
2. Nagorno- Insurgency in
Karabakh : Mozambique 1. Armenia
3. Kherson and Dispute between 2. Azerbaijan
Zaporizhzhia : Israel and Lebanon 3. Croatia
How many of the above pairs are correctly 4. Romania
matched?
5. Uzbekistan
(a) Only one
Which of the above are members of the
(b) Only two Organization of Turkic States ?
(c) All three (a) 1, 2 and 4 only
(d) None (b) 1 and 3 only
(c) 2 and 5 only
2. Consider the following pairs: (2023) (d) 3, 4 and 5 only
Area of conflict Country where
mentioned in news : it is located 5. Which one of the following statements best
1. Donbas : Syria reflects the issue with Senkaku Islands,
2. Kachin : Ethiopia sometimes mentioned in the news? (2022)

3. Tigray : North Yemen (a) It is generally believed that they are


How many of the above pairs are correctly artificial islands made by a country
matched? around the South China Sea.

(a) Only one (b) China and Japan engage in maritime


disputes over these islands in the East
(b) Only two
China Sea. -
(c) All three
(c) A permanent American military base
(d) None has been set up there to help Taiwan to
increase its defence capabilities.

3. In the recent years Chad, Guinea, Mali (d)


Though the International Court of
and Sudan caught international attention Justice declared them as no man’s land,
for which one of the following reasons is some South-East Asian countries claim
common to all of them? (2023) them.

Places in news (2022) 459


unacademy.com | Download the Unacademy app
Give your feedback here: Link
6. Which one of the following statements 3. Lebanon Severe and prolonged
best describes the ‘Polar Code’ ? (2022) economic depression
(a) It is the international code of safety for 4. Tunisia Suspension of Parliament
ships operating in polar waters. by President
(b) It is the agreement of the countries How many pairs given above are correctly
around the North Pole regarding the matched?
demarcation of their territories in the
(a) Only one pair
polar region.
(b) Only two pairs
(c) It is a set of norms to be followed by the
countries whose scientists undertake (c) Only three pairs
research studies in the North Pole and (d) All four pairs
South Pole.
(d) It is a trade and security agreement
8. Consider the following pairs: (2014)
of the member countries of the Arctic
Council. Region often in news Country
1. Chechnya Russian
7. Consider the following pairs : (2022) Federation
2. Darfur Mali
Country Important reason for being
3. Swat Valley Iraq
in the news recently
1. Chad Setting up of permanent Which of the above pairs is/are correctly
military base by China matched?

2. Guinea Suspension of Constitution (a) 1 only


and Government by (b) 2 and 3 only
military
(c) 1 and 3 only
(d) 1, 2 and 3

460 Places in news (2022)


unacademy.com | Download the Unacademy app
Give your feedback here: Link
Places in news
4 (2022)-Explanation
1. Answer: (d) 2. Kachin: Kachin is not located in
1. North Kivu and Ituri: These regions are Ethiopia. It refers to a state in northern
located in the Democratic Republic Myanmar (formerly known as Burma).
of the Congo (DRC) and have been in The Kachin conflict involves the Kachin
the news primarily due to conflicts Independence Army (KIA) and the
and violence involving various armed Myanmar military. The conflict in Kachin
groups. The region has experienced State has been ongoing for several
long-standing conflicts related to decades.
ethnic tensions, competition over land 3. Tigray: Tigray is not located in North
and resources, and power struggles. Yemen. It refers to a region in northern
2. Nagorno-Karabakh: Nagorno-Karabakh Ethiopia. The Tigray conflict began
is a disputed territory located in the in November 2020 when tensions
South Caucasus region between Armenia escalated between the Tigray People’s
and Azerbaijan. It has been the subject Liberation Front (TPLF) and the
of a long-standing conflict between the Ethiopian government. The conflict has
two countries. The most recent major resulted in significant humanitarian
escalation of hostilities in Nagorno- concerns and has drawn international
Karabakh occurred in 2020 when a attention.
war broke out between Armenia and
Azerbaijan over control of the region. 3. Answer: (d)
3. Kherson and Zaporizhzhia: These In the recent years Chad, Guinea, Mali and
regions are located in Ukraine and Sudan have caught international attention for
have not been directly involved in a Successful coups.
dispute between Israel and Lebanon.
1. Chad: In April 2021, President Idriss
The ongoing conflict and tensions
Déby, who had been in power for three
between Israel and Lebanon primarily
decades, passed away due to injuries
revolve around issues such as territorial
sustained in a military operation.
disputes, border security, and regional
Following his death, a military council
geopolitical dynamics. Kherson and
led by his son, General Mahamat Idriss
Zaporizhzhia have not been central to
Déby, took control of the country. This
these specific issues.
transition was met with both domestic
and international attention.
2. Answer: (d) 2. Guinea: In September 2021, Guinea
1. Donbas: Donbas is not located in Syria. witnessed a military coup led by the
It refers to a region in eastern Ukraine, Special Forces under the leadership of
primarily consisting of Donetsk and Colonel Mamady Doumbouya. President
Luhansk oblasts. The conflict in Donbas Alpha Condé was detained, and the
began in 2014, following Russia’s military declared the dissolution of
annexation of Crimea, and involves the government and the suspension
pro-Russian separatists and Ukrainian of the constitution. The coup received
government forces. widespread international attention and
condemnation.

Places in news (2022)-Explanation 461


unacademy.com | Download the Unacademy app
Give your feedback here: Link
3. Mali: Mali experienced two coups in reached between the military and
recent years. In 2020, President Ibrahim civilian leaders. This transition in Sudan
Boubacar Keïta was overthrown by a garnered international attention and
military coup led by Colonel Assimi support.
Goïta. This led to a transitional period
and the appointment of a civilian-led
government. However, in May 2021, 4. Answer: (c)
Colonel Goïta staged another coup, Option (c) is correct: The organization of Turkic
dissolving the transitional government States (then called the Cooperation Council of
and assuming interim presidency. These Turkic Speaking States - Turkic Council) was
events attracted international attention established in 2009 as an intergovernmental
and raised concerns about stability and organization to promote comprehensive
governance in Mali. cooperation among the Turkic States. Its
4. Sudan: In April 2019, Sudan underwent four founding member states are Azerbaijan,
a significant political transition. Kazakhstan, Kyrgyzstan, and Türkiye. During
Protests against President Omar al- the 7th Summit held in Baku in October 2019,
Bashir’s long rule eventually led to his Uzbekistan joined as a full member. Hungary
removal by the military. A transitional received observer status at the Organization
military council was established, during its 6th Summit in Cholpon-Ata, Kyrgyz
followed by negotiations between the Republic in September 2018. And later in
military and civilian groups. Eventually, the 8th Summit held in November 2021,
a transitional government was formed, Turkmenistan joined as an Observer Member
and a power-sharing agreement was Of the Organization.

5. Answer: (b)
Option (b) is correct: The 50-year-old controversy between Japan, the People’s Republic of China
(PRC), and Taiwan over the sovereignty of a group of tiny, uninhabited islets and rocks in the East
China Sea, administered by Japan and referred to as the Senkaku Islands in Japan.

462 Places in news (2022)-Explanation


unacademy.com | Download the Unacademy app
Give your feedback here: Link
10, possibly top 3, most severe crisis episodes
globally since the mid-nineteenth century.
Option 4 is correct: President Kais Saied
invoked an emergency act, Article 80, from
Tunisia’s constitution on Sunday (July 25),
removed the Prime Minister, and suspended the
Parliament the next day. Saied took charge of
the executive powers in the country promising
the appointment of a new leader chosen by
him. Saied has relieved cabinet ministers of
the country and appointed acting ministers.

8. Answer: (a)
Pair 1 is correctly matched: Chechnya is in
southwestern Russia, situated on the northern
flank of the Greater Caucasus range. The region
6. Answer: (a) is rich in oil. After a decade of unsuccessfully
fighting for independence, the autonomous
Option (a) is correct: The International Maritime
region is now firmly under the control of
Organization has adopted a mandatory Polar
Russia. In addition, jihadist groups, including
Code to provide for safe ship operation and
those aligned with Islamic State terrorist
environmental protection in the polar regions.
organizations, exist in the region.
If you operate a SOLAS or MARPOL ship in
Arctic or Antarctic waters, then your ship will
soon need to comply with all or part of this
Code.

7. Answer: (c)
Option 1 is not correct: China established its
first, and so far only, the overseas military base
in the eastern African nation of Djibouti in 2017.
The facility is strategically located near the
Bab el-Mandeb Strait linking the Indian Ocean
and the Red Sea. and A permanent Chinese
military installation in Equatorial Guinea is the
culmination of nearly a decade’s investment in
Pair 2 is not correctly matched: The Darfur
Africa – and will not be the last of such bases
region lies in the western part of Sudan,
on the continent’s Atlantic coast.
which borders Libya, Chad, and the Central
Option 2 is correct: In Guinea, military leaders African Republic. The population of Darfur was
detain the president, declaring the dissolution estimated in 2002 at about six million, eighty
of government and suspension of the percent of whom live in rural areas. Since
constitution. 2003, the Darfur region of western Sudan has
Option 3 is correct: Lebanon’s severe and been the site of terrible violence, death, and
prolonged economic depression is, according displacement. The UN described it as “the
to the World Bank, ‘likely to rank in the top world’s worst humanitarian crisis”.

Places in news (2022)-Explanation 463


unacademy.com | Download the Unacademy app
Give your feedback here: Link
Pair 3 is not correctly matched: The Swat Valley
is located in northern Khyber Pakhtunkhwa and
is enclosed by the Himalayas and the Hindu
Kush. It has both mountain ranges and plains.
The valley has tourism potential but since 2008
there has been a humanitarian crisis owing to
the presence of the Taliban.

Elimination Technique
Swat valley is in Pakistan, a famous tourist
attraction. So, Ooptions (b), (c), and (d)can
be eliminated easily. Therefore, we can
mark option (a) as the correct answer.

464 Places in news (2022)-Explanation


unacademy.com | Download the Unacademy app
Give your feedback here: Link
10 ECONOMY
1 1A1: Money: Barter to Bitcoins

1. With reference to Non-Fungible Tokens (a) Indian Banks' Association


(NFTs), consider the following statements (b) National Securities Depository Limited
: (2022)
1. They enable the digital representation (c) National Payments Corporation of India
of physical assets. (d) Reserve Bank of India
2. They are unique cryptographic tokens
that exist on a blockchain. 4. Which one of the following statements
3. They can be traded or exchanged at correctly describes the meaning of legal
equivalency and therefore can be tender money? (2018)
used as a medium of commercial (a) The money which is tendered in courts
transactions. of law to defray the fee of legal cases.
Which of the statements given above are (b) The money which a creditor is under
correct ? compulsion to accept in settlement of
(a) 1 and 2 only his claims.

(b) 2 and 3 only (c) The bank money in the form of cheques,
drafts, bills of exchange, etc.
(c) 1 and 3 only '
(d) The metallic money in circulation in a
(d) 1, 2 and 3 country.

2. With reference to “Blockchain Technology” 5. With reference to digital payments,


consider the following statements: [2020] consider the following statements: (2018)
1. It is a public ledger that everyone 1. BHIM app allows the user to transfer
can inspect, but which no single user money to anyone with a UPI-enabled
controls. bank account.
2. The structure and design of blockchain 2. While a chip-pin debit card has four
is such that all the data in it are about factors of authentication, BHIM app has
cryptocurrency only only two factors of authentication.
3. Applications that depend on basic Which of the statements given above is/are
features of blockchain can be developed correct?
without anybody’s permission.
(a) 1 only
Which of the statements given above is/are
correct? (b) 2 only

(a) 1 only (c) Both 1 and 2

(b) 1 and 2 only (d) Neither 1 nor 2

(c) 2 only
(d) 1 and 3 only 6. Which one of the following best describes
the term "Merchant Discount Rate"
sometimes seen in news? [2018 - I]
3. Which one of the following links all the (a) The incentive given by a bank to a
ATMs in India? (2018) merchant for accepting payments

466 1A1: Money: Barter to Bitcoins


unacademy.com | Download the Unacademy app
Give your feedback here: Link
through debit cards pertaining to that bank on its network regardless of where
bank. they open their accounts.
(b) The amount paid back by banks to their 2. It is an effort to increase RBI’s control
customers when they use debit cards over commercial banks through
for financial transactions for purchasing computerization.
goods or services. 3. It is a detailed procedure by which a
(c) The charge to a merchant by a bank for bank with huge non-performing assets
accepting payments from his customers is taken over by another bank.
through the bank's debit cards. Select the correct answer using the code
(d) The incentive given by the Government, given below:
to merchants for promoting digital (a) 1 only
payments by their customers through
Point of Sale (PoS) machines and debit (b) 2 and 3 only
cards. (c) 1 and 3 only
(d) 1, 2 and 3
7. Consider the following statements: [asked
in UPSC- Prelims-2017] 9. With reference to ‘Bitcoins’, sometimes
1. National Payments Corporation of India seen in the news, which of the following
(NPCI) helps in promoting the financial statements is/are correct? (2016)
inclusion in the country. 1. Bitcoins are tracked by the Central
2. NPCI has launched RuPay, a card Banks of the countries.
payment scheme. 2. Anyone with a Bitcoin address can send
Which of the statements given above is/are and receive Bitcoins from anyone else
correct? with a Bitcoin address.
(a) 1 only 3. Online payments can be sent without
(b) 2 only either side knowing the identity of the
other.
(c) Both 1 and 2
Select the correct answer using the code
(d) Neither 1 nor 2 given below.
(a) 1 and 2 only
8. The Term ‘Core Banking Solutions’ is (b) 2 and 3 only
sometimes seen in the news. Which of
the following statements best describes/ (c) 3 only
describe this term? (2016) (d) 1, 2 and 3
1. It is the networking of a bank’s branches
which enable customers to operate
their accounts from any branch of the

1A1: Money: Barter to Bitcoins 467


unacademy.com | Download the Unacademy app
Give your feedback here: Link
1A1: Money: Barter to Bitcoins_
1 Explanation
1. Answer: (a) creating a “chain” of blocks. Stating which
Statement 3 is not correct: since the word block came previously is commonly referred
Non-fungible itself insinuate that it cannot be to as referencing. It is not necessary that the
used as a medium of exchange because it will structure and design of blockchain is about
not be divisible easily and its individual parts cryptocurrency only.
value will not equal to each other. So mutual Statement 3 is correct: The applications that
substitution will be difficult therefore number3 depend on the basic features of the blockchain
is wrong and by elimination the answer should can be developed without asking anybody for
be “A”. permission or paying anyone. Since blockchain
Statement 1 is correct: The distinct operates through a decentralized platform
construction of each NFT has the potential requiring no central supervision, it is used
for several use cases. For example, they are in voting, banking, messaging apps, internet
an ideal vehicle to digitally represent physical advertising, etc.
assets like real estate and artwork.
Statement 2 is correct: Non-fungible tokens 3. Answer: (c)
(NFTs) are cryptographic assets on a blockchain
The Reserve Bank of India encouraged the
with unique identification codes and metadata
setting up of National Payments Corporation of
that distinguish them from each other.
India (NPCI) to act as an umbrella organization
Elimination Technique for operating various Retail Payment Systems
y Non-fungible tokens cannot be traded. (RPS) in India. NPCI became functional in
So, statement 3 or options (d), (c), and early 2009. NPCI has taken over the National
(b) can be easily eliminated. Therefore, Financial Switch (NFS) from the Institute
we can mark option (a) as the correct for Development and Research in Banking
answer. Technology (IDRBT).
Option (c) is correct: National Payments
Corporation of India links all the ATMs in India.
It is expected to bring greater efficiency by
2. Answer: (d) way of uniformity and standardization in retail
Explanation: payments and expanding and extending the
Statement 1 is correct: Blockchain is a shared, reach of both existing and innovative payment
trusted, public ledger of transactions, that products for greater customer convenience.
everyone can inspect but which no single Additional Information:
user controls. The participants in a blockchain National Payments Corporation of India (NPCI),
system collectively keep the ledger up to date: an umbrella organization for operating retail
it can be amended only according to strict payments and settlement systems in India,
rules and by general agreement. is an initiative of Reserve Bank of India (RBI)
Statement 2 is not correct: A blockchain is a and Indian Banks’ Association (IBA) under the
data structure. The term blockchain comes provisions of the Payment and Settlement
from the structure that stores your data. All Systems Act, 2007, for creating a robust
data becomes separated into blocks. Every Payment & Settlement Infrastructure in India.
block states which block came before it It has been incorporated as a “Not for Profit”

468 1A1: Money: Barter to Bitcoins_Explanation


unacademy.com | Download the Unacademy app
Give your feedback here: Link
Company under the provisions of Section card, the second element is your phone. The
25 of Companies Act 1956 (now Section 8 of BHIM app has three levels of authentication,
Companies Act 2013). First the app binds with a device’s ID and
mobile number, second a user needs to sync
whichever bank account (UPI or non-UPI
4. Answer: (b)
enabled) in order to conduct the transaction.
Currency notes and coins (fiat money) do not Third, when a user sets up the app, they are
have intrinsic value like gold or silver coins, asked to create a pin which is needed to log
but they cannot be refused by any citizen into the app.
of the country for settlement of any kind of
transaction, hence called legal tenders.
6. Answer: (c)
Option (b) is correct: Legal tender money is the
money which a creditor is under compulsion to Explanation:
accept in settlement of his claims. Legal tenders Option (c) is correct: Merchant Discount
are the mediums to settle financial obligations Rate (MDR) is a fee charged from a merchant
and transactions which are recognised by the by a bank for accepting payments from
law of the land. customers through credit and debit cards in
Cheques drawn on savings or current accounts, their establishments. MDR compensates the
however, can be refused by anyone as a mode card-issuing bank, the lender which puts the
of payment. Hence, demand deposits are not PoS terminal and payment gateways such as
legal tenders. MasterCard or Visa for their services. MDR
charges are usually shared in pre-agreed
proportions between the bank and a merchant
5. Answer: (a) and are expressed in percentage of transaction
Digital payment is a transaction that takes amount.
place via digital or online modes using Mobile
or Computer, without physical exchange of
money involved. Here the payer and the payee, 7. Answer: (c)
use electronic mediums to exchange money. Explanation:
Statement 1 is correct: Bharat Interface for Statement 1 is correct: National Payment
Money (BHIM) is mobile Payment app developed Corporation of India (NPCI) is an umbrella
by the National Payments Corporation of India organization for operating retail payments
(NPCI) based upon the system of Unified settlement systems in India. NPCI was
Payments Interface (UPI). BHIM app integrates established jointly by the Reserve Bank of India
the Payments platform as the official app of (RBI) and Indian Bank’s Association (IBA) under
the government of India. It allows the user to the Payment and Settlement Systems Act of
transfer money to anyone with a UPI-enabled 2007. NPCI) is an umbrella organization for
bank account. operating retail payments settlement systems
Statement 2 is not correct: Debit cards do in India. It aims to facilitate affordable
not have four factor authorization and BHIM payment mechanisms to benefit the common
app has three factor authorization. A system people across the country and help financial
that uses two-factor authentication is a bank inclusion.
account with a debit card, where you need to Statement 2 is correct: NPCI facilitates all the
know a PIN and have the physical debit card to retail payments in India. NPCI operates the
withdraw money. A two-factor authentication RuPay card network, which is India’s domestic
app is a similar idea, but instead of a physical card network. NPCI also operates the Unified

1A1: Money: Barter to Bitcoins_Explanation 469


unacademy.com | Download the Unacademy app
Give your feedback here: Link
Payments Interface (PUI), which is a real- Gold.
time payments system that allows for instant y Bitcoins are not under the regulation of the
payments between bank accounts. Central Monetary authorities of nations.
They work outside the framework of
8. Answer: (a) regulated financial markets. (Statement 1
is not correct)
Option (a) is correct: ‘Core Banking Solutions
(CBS)’ is the process of networking bank y Bitcoins can be traded and transferred
branches via computer networks due to which amongst users having bitcoin addresses
the customers can operate their accounts in any that are unique codes identifying coins
branch of the bank. The data and transaction and wallets in which they are held. Anyone
systems across branches are linked to one with a Bitcoin address can send and
another to create a central system. receive Bitcoins from anyone else with a
Bitcoin address. These transactions are
CBS essentially helps in the integration of the authenticated by mathematical operations
range of services that can be offered by all the performed by various computers attached
bank’s branches from centralized data centres. to the network. The records of these
It also helps the banks, apart from providing transactions are then stored in a ledger
better customer service, in generating MIS called the blockchain. (Statement 2 is
reports for the top management and in the correct)
submission of various reports to the regulators
and the Government. y While the ledger of the transactions can be
viewed by anyone, however, the identity of
Elimination Technique the parties involved in the transactions need
y Core Banking Solution is not related to not be known. These coins are transferred
Non-Performing Assets. It is somewhat only to virtual addresses. (Statement 3 is
related to the IT Infrastructure of correct)
banks. So, options (b), (c), and (d) can Elimination Technique
be easily eliminated and we can mark
Option (a) as the correct answer. y Bitcoins are out of the regulatory
purview of the central banks. So, option
(a) and (d) can be easily eliminated.
9. Answer: (b)
y Statement 2 is more likely to be true.
Bitcoin is a cryptocurrency. These currencies Therefore, we can mark option (b) as
are not held in physical form and mostly not the correct answer.
backed by any sort of acceptable assets like

470 1A1: Money: Barter to Bitcoins_Explanation


unacademy.com | Download the Unacademy app
Give your feedback here: Link
2 1B1: Bank Classification

1. Which one of the following activities of the (c) Both 1 and 2


Reserve Bank of India is considered to be (d) Neither 1 nor 2
part of 'sterilization? (2023)
(a) Conducting "Open Market Operations' 4. Consider the following statements: (2022)
(b) Oversight of settlement and payment 1. In India, credit rating agencies are
systems regulated by the Reserve Bank of India.
(c) Debt and cash management for the 2. The rating agency popularly known as
Central and State Governments ICRA is a public limited company.
(d) Regulating the functions of Non-banking 3. Brickwork Ratings is an Indian credit
Financial Institutions rating agency.
Which of the statements given above are
2. Consider the following markets : (2023) correct?

1. Government Bond Market (a) 1 and 2 only

2. Call Money Market (b) 2 and 3 only

3. Treasury Bill Market (c) 1 and 3 only

4. Stock Market (d) 1, 2 and 3

How many of the above are included in


capital markets? 5. With reference to the Indian economy,
consider the following statements: (2022)
(a) Only one
1. If the inflation is too high, the Reserve
(b) Only two Bank of India (RBI) is likely to buy
(c) Only three government securities.

(d) All four 2. If the rupee is rapidly depreciating, RBI


is likely to sell dollars in the market.
3. If interest rates in the USA or European
3. With reference to the Central Bank
Union were to fall, that is likely to induce
digital currencies, consider the following
RBI to buy dollars.
statements: (2023)
Which of the statements given above are
1. It is possible to make payments in a
correct ?
digital currency without using US dollar
or SWIFT system. (a) 1 and 2 only

2. A digital currency can be distributed (b) 2 and 3 only


with a condition programmed into it (c) 1 and 3 only
such as time-frame for spending it.
(d) 1, 2 and 3
Which of the statements given above is/are
correct?
6. In India, which one of the following is
(a) 1 only
responsible for maintaining price stability
(b) 2 only by controlling inflation ? (2022)

1B1: Bank Classification 471


unacademy.com | Download the Unacademy app
Give your feedback here: Link
(a) Department of Consumer Affairs Select the correct answer using the code
(b) Expenditure Management Commission given below:

(c) Financial Stability and Development (a) 1 and 2 only


Council (b) 2 only
(d) Reserve Bank of India (c) 3 only
(d) 1, 2 and 3
7. In India, the central bank’s function as
the ‘lender of last resort’ usually refers to
which of the following? (2021) 10.
With reference to ‘Urban Cooperative
1. Lending to trade and industry bodies Banks’ in India, consider the following
when they fail to borrow from other statements: (2021)
sources 1. They are supervised and regulated
by local boards set up by the State
2. Providing liquidity to the banks having a Governments.
temporary crisis
2. They can issue equity shares and
3. Lending to governments to finance preference shares.
budgetary deficits
3. They were brought under the purview
Select the correct answer using the code of the Banking Regulation Act, 1949
given below: through an Amendment in 1966.
(a) 1 and 2 Which of the statements given above is/are
(b) 2 only correct?
(c) 2 and 3 (a) 1 only
(d) 3 only (b) 2 and 3 only
(c) 1 and 3 only
8. The money multiplier in an economy (d) 1, 2 and 3
increases with which one of the following?
(2021)
(a) Increase in the Cash Reserve Ratio in 11. Consider the following statements: (2020)
the banks 1. In terms of short-term credit delivery to
the agriculture sector, District Central
(b) Increase in the Statutory Liquidity Ratio Cooperative Banks (DCCBs) deliver
in the banks more credit in comparison to Scheduled
(c) Increase in the banking habit of the Commercial Banks and Regional Rural
people Banks.
(d) Increase in the population of the country 2. One of the most important functions
of DCCBs is to provide funds to the
9. Indian Government Bond Yields are Primary Agricultural Credit Societies.
influenced by which of the following? Which of the statements given above is/are
(2021) correct?
1. Actions of the United States Federal (a) 1 only
Reserve
(b) 2 only
2. Actions of the Reserve Bank of India
(c) Both 1 and 2
3. Inflation and short-term interest rates
(d) Neither 1 nor 2

472 1B1: Bank Classification


unacademy.com | Download the Unacademy app
Give your feedback here: Link
12. If you withdraw Rs. 1,00,000 in cash from (d) Increase in the population of the country
your Demand Deposit Account at your bank,
the immediate effect on aggregate money
supply in the economy will be (2020) 16. Consider the following events: [2018]
(a) to reduce it by Rs. 1,00,000 1. The first democratically elected
communist party government formed
(b) to increase it by Rs. 1,00,000 in a State in Indi(a)
(c) to increase it by more than Rs. 1,00,000 2. India's then largest bank, 'Imperial Bank
(d) to leave it unchanged of India', was renamed 'State Bank of
India'.

13. If the RBI decides to adopt an expansionist 3. Air India was nationalised and became
monetary policy, which of the following it the national carrier.
would not do? (2020) 4. Goa became a part of independent
1. Cut and optimize the Statutory Liquidity Indi(a)
Ratio Which of the following is the correct
2. Increase the Marginal Standing Facility chronological sequence of the above
Rate events?
3. Cut the Bank Rate and Repo Rate (a) 4 - 1 - 2 - 3
Select the correct answer using the code (b) 3 - 2 - 1 - 4
given below: (c) 4 - 2 - 1 - 3
(a) 1 and 2 only (d) 3 - 1 - 2 - 4
(b) 2 only
(c) 1 and 3 only 17. Which of the following statements is/are
(d) 1, 2 and 3 correct regarding the 'Monetary Policy
Committee (MPC)? (2017)
1. It decides the RBI's benchmark interest
rates.
14. Which of the following is not included in
2. It is a 12-member body including the
the assets of a commercial bank in India?
Governor of RBI and is reconstituted
(2019)
every year.
(a) Advances
3. It functions under the chairmanship of
(b) Deposits
the Union Finance Minister.
(c) Investments
Select the correct answer using the code
(d) Money at call and short notice given below:
(a) 1 only
15.
The money multiplier in an economy (b) 1 and 2 only
increases with which one of the following?
(c) 3 only
(2019)
(a) Increase in the cash reserve ratio (d) 2 and 3 only

(b) Increase in the banking habit of the


population 18. What is the purpose of setting up of Small
(c) Increase in the statutory liquidity ratio Finance Banks (SFBs) in India? [2017]
1. To supply credit to small business units

1B1: Bank Classification 473


unacademy.com | Download the Unacademy app
Give your feedback here: Link
2. To supply credit to small and marginal 3. Payment Banks cannot undertake
farmers lending activities.
3. To encourage young entrepreneurs to Select the correct answer using the code
set up business particularly in rural given below.
areas. (a) 1 and 2 only
Select the correct answer using the code (b) 1 and 3 only
given below:
(c) 2 only
(a) 1 and 2 only
(d) 1, 2 and 3
(b) 2 and 3 only
(c) 1 and 3 only
21. When the Reserve Bank of India reduces
(d) 1, 2 and 3 the Statutory Liquidity by 50 basis points,
which of the following is likely to happen?
19.
What is/are the purpose/purposes of (2015)
‘Marginal Cost of Funds based Lending (a)
India’s GDP growth rate increases
Rate (MCLR)’ announced by the RBI? (2016) drastically
1. These guidelines help improve the (b) Foreign Institutional Investors may bring
transparency in the methodology more capital into our country
followed by the banks for determining (c) Scheduled Commercial Banks may cut
the interest rates on advances. their lending rates
2. These guidelines help ensure the (d) It may drastically reduce the liquidity to
availability of bank credit at interest the banking system.
rates which are fair to the borrowers as
well as the banks.
Select the correct answers using the code 22. With reference to India economy, consider
given below: the following: (2015)
1. Bank rate
(a) 1 only
2. Open market operations
(b) 2 only
3. Public debt
(c) Both 1 and 2
4. Public revenue
(d) Neither 1 nor 2
Which of the above is/are component/
components of Monetary Policy?
20.
The establishment of ‘Payment Banks’ (a) 1 only
is being allowed in India to promote
financial inclusion. Which of the following (b) 2, 3 and 4 only
statements is/are correct in this context? (c) 1 and 2 only
(2016) (d) 1, 3 and 4 only
1. Mobile telephone companies and
supermarket chains that are owned and
23. Which reference to inflation in India, which
controlled by residents are eligible to
of the following statements is correct?
be promoters of Payment Banks.
(2015)
2. Payment Banks can issue both credit (a) Controlling the inflation in India is the
cards and debit cards. responsibility of the Government of
India only

474 1B1: Bank Classification


unacademy.com | Download the Unacademy app
Give your feedback here: Link
(b) The Reserve Bank of India has no role in Select the correct answer using the code
controlling the inflation given below:
(c) Decreased money circulation helps in (a) 1 only
controlling the inflation (b) 1 and 2 only
(d) Increased money circulation helps in (c) 2 and 3 only
controlling the inflation
(d) 1, 2, 3 and 4

24. The terms ‘Marginal Standing Facility Rate’


and ‘Net Demand and Time Liabilities’ 27.
What is/are the facility/facilities the
sometimes appearing in news, are used in beneficiaries can get from the services of
relation to: (2014) Business Correspondent (Bank Saathi) in
(a) Banking operations branchless areas? (2014)
(b) Communication networking 1. It enables the beneficiaries to draw their
subsidies and social security benefits in
(c) Military strategies their villages.
(d)
Supply and demand of agricultural 2. It enables the beneficiaries in the rural
products areas to make deposits and withdrawals.
Select the correct answer using the code
25. If the interest rate is decreased in an given below:
economy, it will (2014)
(a) decrease the consumption expenditure (a) 1 only
in the economy. (b) 2 only
(b)
increase the tax collection of the (c) Both 1 and 2
Government (d) Neither 1 nor 2
(c) increase the investment expenditure in
the economy 28. Supply of money remaining the same when
(d)
increase the total savings in the there is an increase in demand for money,
economy there will be (2013)
(a) a fall in the level of prices
26.
In the context of the Indian economy, (b) an increase in the rate of interest
which of the following is/are the (c) a decrease in the rate of interest
purpose/purposes of ‘Statutory Reserve
(d) an increase in the level of income and
Requirements’? (2014)
employment
1. To enable the Central Bank to control
the amount of advances the banks can
create. 29. Consider the following liquid assets: (2013)
1. Demand deposits with the banks
2. To make the people’s deposits with
banks safe and liquid. 2. Time deposits with the banks
3. To prevent the commercial banks from 3. Savings deposits with the banks
making excessive profits. 4. Currency
4. To force the banks to have sufficient The correct sequence of these assets in
vault cash to meet their day-to-day the decreasing order of liquidity is
requirements.
(a) 1-4-3-2

1B1: Bank Classification 475


unacademy.com | Download the Unacademy app
Give your feedback here: Link
(b) 4-3-2-1 (b) 2, 3 and 4 only
(c) 2-3-1-4 (c) 1, 2 and 3 only
(d) 4-1-3-2 (d) 1, 2, 3 and 4

30. Priority Sector Lending by banks in India 34. Which of the following grants/grant direct
constitutes the lending to: (2013) credit assistance to rural households?
(a) Agriculture (2013)
(b) micro and small enterprises 1. Regional Rural Banks
(c) weaker sections 2. National Bank for Agriculture and Rural
(d) All of the above Development
3. Land Development Banks.

31. In the context of Indian economy, ‘Open Select the correct answer using the code
Market Operations’ refers to: (2013) given below:
(a) borrowing by scheduled banks from the (a) 1 and 2 only
RBI (b) 2 only
(b) lending by commercial banks to industry (c) 1 and 3 only
and trade
(d) 1, 2 and 3
(c)
purchase and sale of government
securities by the RBI
35. Which of the following measures would
(d) None of the above result in an increase in the money supply in
the economy? (2012)
32. An increase in the Bank Rate generally 1. Purchase of government securities from
indicates that the (2013) the public by the Central Bank
(a) Market rate of interest is likely to fall 2. Deposit of currency in commercial
(b) Central Bank is no longer making loans banks by the public
to commercial banks 3. Borrowing by the government from the
(c) Central Bank is following an easy money Central Bank
policy 4. Sale of government securities to the
(d) Central Bank is following a tight money public by the Central Bank
policy Select the correct answer using the code
given below:
33. The Reserve Bank of India regulates the (a) 1 only
commercial banks in matters of: (2013) (b) 2 and 4 only
1. liquidity of assets
(c) 1 and 3 only
2. branch expansion
(d) 2, 3 and 4 only
3. merger of banks
4. winding-up of banks 36. The Reserve Bank of India (RBI) acts as a
Select the correct answer using the code bankers' bank. This would imply which of
given below: the following? (2012)
(a) 1 and 4 only 1. Other banks retain their deposits with
the RBI.

476 1B1: Bank Classification


unacademy.com | Download the Unacademy app
Give your feedback here: Link
2. The RBI lends funds to the commercial (c) 2 and 3 only
banks in times of need. (d) 1, 2, 3 and 4
3. The RBI advises the commercial banks
on monetary matters.
40. In India, which of the following have the
Select the correct answer using the code highest share in disbursement of credit to
given below: agriculture and allied activities? (2011)
(a) 2 and 3 only (a) Commercial Banks
(b) 1 and 2 only (b) Cooperative Banks
(c) 1 and 3 only (c) Regional Rural Banks
(d) 1, 2 and 3 (d) Microfinance Institutions

37. Which of the following is not an instrument 41.


With reference to the Non-banking
of Selective Credit Control? (Pre-1995) Financial Companies (NBFCs) in India,
(a) Regulation of consumer credit consider the following statements: [2010]
(b) Rationing of credit 1. They cannot engage in the acquisition
(c) Margin requirements of securities issued by the government.

(d) Cash reserve ratio 2. They cannot accept demand deposits


like Savings Account.
Which of the statements given above is/
38. The lowering of Bank Rate by the Reserve are correct?
Bank of India leads to (2011) (a) 1 only
(a) More liquidity in the market (b) 2 only
(b) Less liquidity in the market (c) Both 1 and 2
(c) No change in the liquidity in the market (d) Neither 1 nor 2
(d)
Mobilization of more deposits by
commercial banks
42.
In the context of independent India’s
economy, which one of the following was
39. Microfinance is the provision of financial the earliest event to take place? [2009]
services to people of low-income groups. (a) Nationalization of Insurance companies
This includes both the consumers and
the self-employed. The service/services (b) Nationalization of State Bank of India
rendered under microfinance is/are: (2011) (c) Enactment of Banking Regulation Act
1. Credit facilities (d) Introduction of First Five-Year Plan
2. Savings facilities
3. Insurance facilities 43. Consider the following statements: [2004]
4. Fund Transfer facilities 1. The National Housing Bank the apex
Select the correct answer using the code institution of housing finance in India,
given below the lists: was set up as a wholly-owned subsidiary
of the Reserve Bank of India
(a) 1 only
2. The Small Industries Development
(b) 1 and 4 only Bank of India was established as a

1B1: Bank Classification 477


unacademy.com | Download the Unacademy app
Give your feedback here: Link
whollyowned subsidiary of the
Industrial Development Bank of India
Which of the statements given above is/are
correct?
(a) 1 only
(b) 2 only
(c) Both 1 and 2
(d) Neither 1 nor 2

478 1B1: Bank Classification


unacademy.com | Download the Unacademy app
Give your feedback here: Link
1B1: Bank Classification_
2 Explanation
1. Answer: (a) and regulated by the central bank. They are
Sterilization in economics refers to the designed to function as a digital representation
measures taken by a country’s central bank of physical cash and operate within a country’s
to counteract the effects of capital inflows legal and regulatory framework.
and outflows on the domestic money supply. Statement 2 is correct: digital currency can
This is done to keep inflation in check and be distributed with programmed conditions,
stabilize the country’s economy. Open Market including a specific time frame for spending
Operations (OMO) are a key instrument used it. Smart contracts, which are self-executing
for this purpose. These are operations by contracts with the terms of the agreement
which the central bank (like the Reserve Bank directly written into lines of code, can be
of India) buys or sells government bonds in the utilised to impose conditions and automate
open market. certain actions related to digital currencies.
By incorporating time-based conditions into
the programming code of a digital currency
2. Answer: (b) or a smart contract, it is possible to enforce
The call money market deals with very restrictions on when and how the currency can
short-term funds. Treasury bills are short- be spent.
term securities issued by a government with
maturity periods typically less than a year. The
money market is part of the financial market 4. Answer: (b)
where short-term borrowing and lending takes Statement 1 is not correct: All the credit rating
place, typically for a period of less than a agencies in India are regulated by SEBI (Credit
year. Thus these two form a part of the Money Rating Agencies) Regulations, 1999 of the
market. The capital market, on the other hand, Securities and Exchange Board of India Act,
is concerned with long-term borrowing and 1992.
lending, usually over a period of more than Statement 2 is correct: ICRA Limited is a public
one year. Government bonds are long-term limited company that was set up in 1991 in
debt securities issued by the government to Gurugram. The company was formerly known
raise capital, and they mature over a period as Investment Information and Credit Rating
greater than one year. Similarly stocks in the Agency of India Limited.
stock market are traded for the long term.
Thus government bonds and the stock market Statement 3 is correct: Brickwork Ratings is
form the part of the Capital market. recognised as an external credit assessment
agency (ECAI) by the Reserve Bank of India
(RBI) to carry out credit ratings in India.
3. Answer: (c)
Elimination Technique
Statement 1 is correct: Central bank digital
currencies (CBDCs) have the potential to enable y The Reserve Bank of India (RBI)
payments in digital currency without relying on only regulates the money market
the US dollar or traditional payment systems (basically the banking sector). The
like SWIFT (Society for Worldwide Interbank Credit Rating Agencies (CRAs) rate
Financial Telecommunication).CBDCs are the credit worthiness of companies
digital forms of a country’s fiat currency issued (financial or non-financial) or any

1B1: Bank Classification_Explanation 479


unacademy.com | Download the Unacademy app
Give your feedback here: Link
foreign exchange, and reserves management,
sovereign governments. From that, we government debt management, financial
can easily guess that RBI has nothing regulation, and supervision, apart from currency
to do with non-financial companies management and acting as a banker to the
(e.g., Reliance Industries, Tata Motors, banks and to the Government. One of its chief
etc.) in the dimensions of regulation functions is to keep a check on inflation by use
or supervision. So, Statement 1 can be of monetary policy in the form of qualitative
easily eliminated. and quantitative measures.
y If we eliminate Statement 1, Option (a),
(c), and (d) gets easily eliminated and
7. Answer: (b)
we can mark Option (b) as the correct
answer. Specific activities of the Reserve Bank of India
(RBI) are as follows:
5. Answer: (b) y Role as a banker:
Statement 1 is not correct: The Reserve Bank ○ RBI pays and receives money on behalf
of India (RBI) buys government bonds from of the government.
the market and pays back money to the ○ It floats loans on behalf of the
bondholders, thus injecting more money into government.
the economy.
○ Issue Advances to government (both
Statement 2 is correct: To soften the rupee’s interest and non-interest bearing).
fall, the RBI would sell in the market some of
y Portfolio Manager- Invests surplus cash of
the dollars it has in its forex reserves.
the government.
Statement 3 is correct: If the interest rate in US
y Advisor- Acts as one on monetary and
and EU falls, there will be an inflow of dollars
banking-related subjects.
in the Indian market, leading to appreciation of
the rupee. To reduce the supply of dollars in Option (b) is correct: A lender of last resort
the economy, RBI will like to buy dollars from (LoR) is a financial institution, usually a
the market. country’s central bank, that provides loans to
banks and other qualifying entities that are
Elimination Technique in financial distress or are regarded as highly
y If the inflation is too high the RBI is hazardous or on the verge of collapse.
likely to sell government securities to As a Banker to Banks, the Reserve Bank (Central
suck excess money from the market. So, bank of India) in India also acts as the ‘lender
Statement 1 is not correct and Options of the last resort’. It can come to the rescue
(a), (c), and (d) can be eliminated easily. of a bank that is solvent but faces temporary
Therefore, we can mark option (b) as liquidity problems by supplying it with much-
the correct answer. needed liquidity when no one else is willing to
extend credit to that bank. The Reserve Bank
extends this facility to protect the interest
6. Answer: (d) of the depositors of the bank and to prevent
Option (d) is correct: The Reserve Bank of possible failure of the bank, which in turn
India, the nation’s central bank, is entrusted may also affect other banks and institutions
with the responsibility of ensuring monetary and can have an adverse impact on financial
stability and operating the currency and credit stability and thus on the economy.
system of the country to its advantage. Its
functions comprise monetary management,

480 1B1: Bank Classification_Explanation


unacademy.com | Download the Unacademy app
Give your feedback here: Link
8. Answer: (c) including interest rates, inflation, and
The money multiplier reflects the amplified economic growth. The price of a G-Sec, like
change in the money supply that ultimately other financial instruments, keeps fluctuating in
results from the injection of additional reserves the secondary market. The price is determined
into the banking system. by the demand and supply of the securities.
Specifically, the prices of G-Secs are influenced
Option (c) is correct: The monetary base, often
by the level and changes in interest rates in the
known as high-powered money, is the money
economy and other macroeconomic factors,
created by the Federal Reserve. Banks make
such as the expected rate of inflation, liquidity
money by lending it out. To generate more
in the market, etc.
income, a bank lends or invests its excess
reserves. The money supply grows by multiple Statement 3 is correct: The Reserve Bank
dollars for every dollar added to the monetary of India’s monetary policy, particularly the
base. Hence, an increase in the banking habits direction of interest rates, the government’s
of the people increases the money supply. budgetary position, global markets, the state
of the economy, and inflation, are all key
An unbanked large population will not increase
influences on the yield. Policy actions by
the money multiplier.
RBI (e.g., announcements regarding changes
The money multiplier is inversely proportional in policy interest rates like Repo Rate, Cash
to the Cash Reserve Ratio (CRR). So, it will not Reserve Ratio, Open Market Operations, etc.)
increase with the rise in CRR. also affect the prices of G-Secs.
An increase in the SLR will decrease the Additional Information:
loanable funds with the bank, and so it will
A Government Security (G-Sec) is a tradable
not help improve the money multiplier.
instrument issued by the Central Government
or the State Government. It acknowledges the
9. Answer: (d) Government’s debt obligation. Such securities
are short-term (usually called treasury bills,
A bond’s yield refers to the expected earnings
with original maturities of less than one year) or
generated and realized on a fixed-income
long-term (usually called Government bonds
investment over a particular period, expressed
or dated securities with an original maturity of
as a percentage or interest rate. There are
one year or more).
numerous methods for arriving at a bond’s
yield, and each of these methods can shed
light on a different aspect of its potential risk 10. Answer: (b)
and return.
Urban cooperative banks (UCBs) are largely
Statement 1 is correct: The developments in localised financial service providers and have
international bond markets, specifically the been in existence for over a century.
US Treasuries, affect the prices of G-Secs in
Statement 1 is not correct: The recent Banking
India. Bond yields in India are influenced by the
Regulation (Amendment) Act 2020 enables
actions of the United States Federal Reserve.
the RBI to get all the powers, including those
The foreign investors will accordingly buy/sell
hitherto exclusively with the registrar of
Indian government securities in the secondary
cooperative societies.
market depending on the returns offered by
the US feds on US Treasury Bonds & this will Statement 2 is correct: In 2021, the Reserve
influence the Indian Government’s Bond yields. Bank of India came out with draft guidelines
allowing primary urban cooperative banks
Statement 2 is correct: A bond’s yields are
(UCBs) to augment capital through the
influenced by a variety of economic factors,
issuance of equity shares, preference shares

1B1: Bank Classification_Explanation 481


unacademy.com | Download the Unacademy app
Give your feedback here: Link
and debt instruments. do not deliver more short-term credit to the
Statement 3 is correct: The Banking Regulation agricultural sector in comparison to SCBs and
Act 1949 was amended in 1965 to bring RRBs.
cooperative banks under RBI regulation by Statement 2 is correct: The role of the DCCBs
insertion of a special section – Sec 56 (as is to give financial assistance to the primary
applicable to cooperative societies). This co-operative societies affiliated to it in the
enabled some select sections to be made district. One of these societies is Primary
applicable as they are to cooperative banks, Agricultural Credit Societies. Hence, one of
while some others were made applicable with the most important functions of DCCBs is to
certain modifications, and a large number provide funds to the Primary Agricultural Credit
of sections were not made applicable at all. Societies.
Since March 1, 1966, when banking laws
became applicable to cooperative societies,
the Registrar of Cooperative Societies and the 12. Answer: (d)
Reserve Bank of India have had a ‘duality of The aggregate money supply is the total stock
authority’ over UCBs. of all types of money, such as currency and
demand deposits held with the public and
Elimination Technique banks. The stock of money kept with the
y Cooperative banks are regulated by government and RBI, etc., is not considered
the Reserve Bank of India (RBI). So, in money supply as these are not in actual
statement 1 is not correct or Options circulation in the economy.
(a), (c), and (d) can be easily eliminated. Option (d) is correct: The most common
We can mark Option (b) as the correct measure of liquidity to estimate aggregate
answer. money supply includes currency and demand
y Note- If one is not aware of the fact deposits both, hence changing the form of
that who regulates the Cooperative money from demand deposit to currency will
banks, one can recall a recent news change the form, in which the money is held
where the RBI supersedes the board of but not the aggregate money supply and hence
Punjab and Maharashtra Cooperative leave it unchanged.
Bank. From this news, one can easily Additional Information:
eliminate Statement 1.
There are three main sources of money supply
in the economy which are:
11. Answer: (b)
y The Reserve Bank of India: It has the sole
District Central Cooperative Banks are authority to issue paper currency of all
cooperative banks operating at the district level, denominations.
generally in the rural areas of India. Scheduled
Commercial Banks (SCBs) alone account for y Commercial Banks: Commercial banks
over 79% of institutional credit being provided create credit as per the demand deposits.
to the agricultural sector. Regional Rural Banks y Government: It produces or mints coins
(RRBs) account for another 5%, making a total of all denominations. However, these are
of 84% of agricultural institutional credit from issued for circulation only through the
these two agencies. Reserve Bank in terms of the RBI Act.
Statement 1 is not correct: Also, in short term
credit delivery to the agriculture sector, the 13. Answer: (b)
contribution is around 60% for SCBs alone and
Under an expansionary monetary policy, the
30% for the Cooperative Banks. Hence, DCCBs

482 1B1: Bank Classification_Explanation


unacademy.com | Download the Unacademy app
Give your feedback here: Link
RBI seeks to increase the money supply in the RBI requires banks to hold and not lend.
economy. It can also be defined as the ratio of the stock
Statement 1 and statement 3 are not correct: of money to the stock of high-powered money
Cutting and optimizing the Statutory Liquidity in an economy. Through the process of Money
Ratio will enable more lendable funds in the Multiplier, initial deposits are cycled through
hands of banks, and thus, increase the lending by the banks in the economy, by each time
and the money supply. Similarly, cutting the keeping a stipulated number of reserves with
Bank Rate and Repo Rate will make borrowing them and further lending them. When this lent
by the banks cheaper, the benefit of which amount comes back to the banks in the form
would then be passed on to the borrowers of deposits again a certain amount is kept as
taking loans from banks thus providing cheaper reserves and the balance is lent, this process
credit which will result in more borrowing and goes on and is called the Money Multiplier.
thus increased money supply. Option (b) is correct: In the above mentioned
Statement 2 is correct: Increasing the Marginal four situations, only the increase in banking
Standing Facility Rate will make borrowings habits of the population will increase the
by the banks more expensive and thus, the money multiplier as only when the deposits
higher interest rates would be passed on to come into the banking system, they can be
the borrowers of the banks making credit more used by the banks to create more Money.
expensive in the economy and discouraging Additional Information:
loan activities and reducing the money supply.
Hence, the RBI will not take this step under an Increase in the cash reserve ratio and the
expansionist monetary policy. statutory liquidity ratio will decrease the
money multiplier as now banks would be
able to further lend only a smaller proportion
14. Answer: (b) of deposits due to the increased reserve
A liability is something that someone or some requirements as compared to earlier.
institution owes to another. Simply, liabilities Increase in population of the country would
are the obligations that are rising out of not have much impact until this increased
previous transactions, which is payable by population participates in the cyclical flow of
the enterprise or the government through the the economy and engages with the banking
assets possessed by the enterprise. system.
Option (b) is correct: The money deposited
by the depositors in the bank, is a liability for 16. Answer: (b)
the bank as on demand of the depositor the
money so deposited has to be returned by the Explanation:
bank to the depositor. While assets are things y The first democratically elected communist
that a firm owns or what a firm can claim from party government formed in a state in India
others. For banks, Advances, Investments and in 1957.
Money at call and short notice serve as its y Imperial Bank of India (currently, the
Assets. State Bank of India) was nationalized by
the Government of India in 1955 with the
15. Answer: (b) Reserve Bank of India taking a 60% stake.

Money multiplier is the amount of money that y Air India was nationalized in 1953. The
banks generate with each rupee of reserves. government of India passed the Air
Reserves are the amount of deposits that the Corporations Act in 1953 and purchased a
majority stake in the carrier from Tata Sons.

1B1: Bank Classification_Explanation 483


unacademy.com | Download the Unacademy app
Give your feedback here: Link
y Goa became an independent state of India institution intended to further the objective
on 18 December 1961 and Goa was freed of financial inclusion by primarily undertaking
from the Portuguese. basic banking activities of acceptance of
deposits and lending to un-served and
underserved sections including small business
17. Answer: (a) units, small and marginal farmers, micro
Statement 1 is correct: The primary purpose and small industries and unorganized sector
of the Monetary Policy Committee (MPC) is entities, but without any restriction in the area
to maintain price stability while also keeping of operations, unlike Regional Rural Banks or
in mind the objective of economic growth. Local Area Banks.
The committee determines RBI’s benchmark Statement 3 is not correct: SFB are not
interest rates, primarily the Repo Rate, so as mandated to encourage young entrepreneurs
to keep inflation within the acceptable limits. to set up business particularly in rural areas.
Statement 2 is not correct: Section 45ZB
of the RBI Act provides for an empowered
Six-member Committee (not 12 members), 19. Answer: (c)
including the RBI Governor. Three members The Marginal Cost of Funds based Lending
of the Committee come from within the RBI, Rate (MCLR) was implemented by the Reserve
and the other three members are appointed by Bank of India (RBI) in 2016. It has replaced the
the Government. Members of the MPC will be earlier base rate-based lending system. Under
appointed for a period of four years and shall this system, the banks take into account the
not be eligible for reappointment. cost to lend an additional amount of one rupee
Statement 3 is not correct: It functions under to determine their final lending rate.
the Chairmanship of the Governor of the RBI y Earlier in the base rate system, the
and not under the Chairmanship of the Union revision of rates in accordance with the
Finance Minister. changes in Repo Rate by the RBI used
to happen very slowly and in different
Elimination Technique manners. Now under the MCLR system,
y As we know that the MPC of the RBI the banks compulsorily have to adjust
decides the benchmark interest their lending rates in accordance with the
rate. Option (c) and (d) can be easily RBI repo rate in a specified manner, thus
eliminated. bringing transparency to the entire system.
y We can recall the term- “Bi-monthly (Statement 1 is correct)
monetary policy meeting” which is often y Earlier, some unfair practices used to take
in the news. From there we can have place where discrimination in charging
a guess that the MPC is constituted 6 the lending rates used to happen based
times a year. So, Statement 2 is not on the unequitable premise. Under the
correct and we can eliminate Option MCLR system, the borrowers, as well as
(b) as well. Therefore, the correct the lenders, will deal in a transparent and
answer is Option (a). fair lending rate ecosystem. (Statement 2
is correct)

18. Answer: (a)


Explanation: 20. Answer: (b)

Statement 1 and Statement 2 are correct: The Payments Banks are registered as Public limited
Small Finance Bank (SFB) is a private financial companies under the Companies Act, 2013 and

484 1B1: Bank Classification_Explanation


unacademy.com | Download the Unacademy app
Give your feedback here: Link
hold a license under the Banking Regulation
Act, 1949. The objectives of setting up of y The Payment Bank cannot issue credit
payments banks will be to further financial cards (as per the RBI guidelines). So,
inclusion by providing small savings accounts option (a), (c), and (d) can be easily
and payments/remittance services to migrant eliminated. We can mark option (b) as
labour workforce, low-income households, the correct answer.
small businesses, other unorganised sector
entities and other users.
Statement 1 is correct: The eligible promoters 21. Answer: (c)
of Payment Banks are the existing non-bank
Statutory Liquidity Ratio (SLR) is that ratio
Pre-paid Payment Instrument (PPI) issuers; and
of the Net Demand and Time Liabilities that
other entities such as individuals/professionals;
the banks must keep in Liquid Assets and
Non-Banking Finance Companies (NBFCs),
Securities as specified by the Reserve Bank of
corporate Business Correspondents (BCs),
India (RBI).
mobile telephone companies, supermarket
chains, companies, real sector cooperatives; Option (c) is correct: A reduction in SLR will
that are owned and controlled by residents; leave more funds in the hands of banks, which
and public sector entities may apply to set up can be used for lending purposes. As the
payments banks. Some prominent examples of interest earned on these additional funds will
Payment Banks include Airtel Payments Bank, bring in greater profits for the banks, the banks
Paytm Payments Bank etc. may choose to cut their lending rates to pass
on the benefits to the borrowers. This is the
As per the Reserve Bank of India (RBI) guidelines,
mechanism through which the Monetary Policy
y Payment Banks can only issue ATM/ debit formulated by the Reserve Bank of India (RBI)
cards and they are not allowed to issue is transmitted onwards to the end economic
credit cards. (Statement 2 is not correct) participants.
y Payment Banks can not undertake lending
activities. This means they cannot deploy
their funds in loans. (Statement 3 is correct) 22. Answer: (c)
Additional Information: Monetary Policy refers to the macroeconomic
y Setting up of Payment Banks was policy laid down by the Reserve Bank of India
recommended by the Nachiket Mor (RBI). The objectives of Monetary Policy are to
Committee. keep inflation in control while leaving sufficient
room for economic growth. There are various
y Payment Banks can only accept Demand
tools employed by the RBI to give effect to the
Deposits up to Rs. 1,00,000 per individual.
Monetary Policy.
They cannot accept time deposits like Fixed
Deposits. Option (c) is correct: Bank rate and Open
market operations are two such instruments
y They cannot accept Non-Resident Indian
which are used for Monetary Policy operation.
(NRI) deposits.
y Bank Rate: It is the rate at which the RBI
y Eligible deposits mobilised by the Payment
lends to the banks without a collateral.
Bank are covered under Deposit Insurance
Bank rates influence the lending rates of
and Credit Guarantee Corporation.
commercial banks. Higher bank rate will
Elimination Technique translate to higher lending rates by the
banks. In order to curb liquidity, the central
bank can resort to raising the bank rate and

1B1: Bank Classification_Explanation 485


unacademy.com | Download the Unacademy app
Give your feedback here: Link
vice versa. recreation, transport, consumer staples, etc.
y Open Market Operations: The Open Market Inflation measures the average price change
Operations (OMO) is a mechanism through in a basket of commodities and services over
which the RBI sale and purchase of time.
Government Securities (G-Secs) to and from Option (c) is correct: Controlling inflation is not
the market with an objective to adjust the only the responsibility of the Government of
rupee liquidity conditions in the market on India. It is the prime responsibility of the RBI.
a durable basis. The objective of OMO is to RBI uses various monetary policy tools such as
regulate the money supply in the economy. repo rate, CRR, SLR, etc., to control inflation
When the RBI wants to increase the money in the economy. By using these tools, the RBI
supply in the economy, it purchases the influences aggregate demand in the market
government securities from the market, by influencing money circulation. Decreased
and it sells government securities to suck money circulation pulls down the aggregate
out liquidity from the system. demand in the economy, due to which, given
Additional Information: a constant supply, the prices of goods are
reduced.
y Public Debt: Public debt includes the total
liabilities of the Union government that
have to be paid from the Consolidated 24. Answer: (a)
Fund of India. Sometimes, the term is also Option (a) is correct: Marginal Standing Facility
used to refer to the overall liabilities of the Rate and Net Demand and Time Liabilities are
central and state governments. However, terms used in reference to the operations of
the Union government clearly distinguishes the banking sector in India.
its debt liabilities from those of the states.
It calls overall liabilities of both the y Marginal Standing Facility Rate is the rate
Union government and states as General at which the Reserve Bank of India lends
Government Debt (GGD) or Consolidated overnight funds to Commercial Banks
General Government Debt. against their holdings of government
securities. Marginal Standing Facility (MSF)
y Public Revenue: Public revenue is the is a very short-term borrowing scheme for
revenue obtained from various government scheduled commercial banks. Banks may
sources to meet public spending borrow funds through MSF during severe
requirements. Taxes, fines, contributions, cash shortages or acute shortages of
etc., are significant sources of public liquidity.
revenue.
y Net Demand and Time Liabilities (NDTL) is
Elimination Technique the difference between the sum of demand
y Public debt and public revenue are and time liabilities (deposits) of a bank
part of the government’s fiscal policy. (with the public or the other bank) and the
So, we can easily mark option (c) as deposits in the form of assets held by the
the correct answer. other bank.

25. Answer: (c)

23. Answer: (c) Option (c) is correct: Interest rate is a variable


that affects households, business houses,
Inflation refers to the persistent rise in the corporations, manufacturers, investors,
prices of most goods and services of daily or borrowers, etc. And even if anyone does not
common use, such as food, clothing, housing,

486 1B1: Bank Classification_Explanation


unacademy.com | Download the Unacademy app
Give your feedback here: Link
have debts or assets, the influence of the can create.
interest rate on the economy will affect us y Cash Reserve Ratio (CRR) refers to this
indirectly. liquid cash that banks have to maintain
y The interest rate is simply the price of with the Reserve Bank of India (RBI) as a
money. It is the term at which money or certain percentage of their demand and
goods today may be traded off for money or time liabilities.
goods at a future date. y Statutory Liquidity Ratio (SLR) is a minimum
y The interest rate is also used as an percentage of deposits that a commercial
instrument in economic policy. Setting the bank has to maintain in the form of liquid
interest rate to achieve a Monetary Policy cash, gold, or other securities. It is basically
objective, often price stability or low and the reserve requirement that banks are
stable inflation, is usually the responsibility expected to keep before offering credit to
of the Central bank (the Reserve Bank of customers. These are not reserved with the
India). Reserve Bank of India (RBI) but with banks
y Interest rates are a big factor when it themselves.
comes to debt. Reduced interest rates in
the economy lowers the cost of borrowings
and this makes available debt at cheaper Elimination Technique
rates to the firms operating in the economy. y Statement 3 is meaningless. No entity
y Due to cheaper debt firms would be able can put a limit on banks profit or
to finance their investment decisions at earnings. Profit maximization is the
lower interest rates hence promoting them prime objective of commercial banks.
to borrow and invest more. It will increase So, Option (c) and (d) can be easily
the aggregate investment expenditure in eliminated.
the economy. y Statement 2 is more likely to be
y A fall in interest rates also reduces the incorrect. We can market Option (a) as
monthly cost of mortgage repayments. the correct answer.
This will leave householders with more
disposable income and should cause a rise 27. Answer: (c)
in consumer spending.
Business Correspondents are retail agents
engaged by banks for providing banking
26. Answer: (a) services at locations other than a bank branch/
Statutory Reserve Requirements are the ATM.
legally mandated reserve requirements that y Through banking correspondents, the
the banks have to maintain in specified ways beneficiaries can access their subsidies
as mandated by the Government/ the Reserve and social security benefits from their
Bank of India (RBI). accounts. (Statement 1 is correct)
Option (a) is correct: Monetary Policy y Deposits and withdrawals can be made
instruments are used by the Central Bank (RBI) by people residing in the rural areas easily
to give effect to its monetary policies. Statutory through the Bank Saathi. Especially where
Reserve Requirements like Cash Reserve Ratio, the access to bank branches is limited, the
Statutory Liquidity Ratio are employed by the Banking Correspondents come as a great
RBI as Monetary Policy instruments to have help in conducting day to day banking
control over the amount of advances the banks transactions. (Statement 2 is correct)

1B1: Bank Classification_Explanation 487


unacademy.com | Download the Unacademy app
Give your feedback here: Link
28. Answer: (b) provide institutional credit to certain sectors
Market Equilibrium for any commodity is defined and segments of the economy that are facing
at the point where market demand equals to difficulty in getting credit. It allows adequate
the market Supply of that commodity. A change flow of funds to the key economic sectors and
in market demand or market supply will disrupt serves a social purpose of inclusive growth.
the equilibrium and cause movement in the Option (d) is correct: Some segments/ areas
prices of that commodity. are starved of availability of loans due to
Option (b) is correct: If supply of money which they fall behind vis a vis the national
remains the same when there is an increase in development agenda. The Reserve Bank of
demand for money, there will be an increase in India (RBI) had mandated a certain proportion
the rate of interest as the availability of funds of loans that financial institutions have to
will be comparative to scarce in comparison to mandatorily provide to these priority sectors.
the demand for them. Priority Sector constitutes the lending to the
following categories:
y Agriculture
29. Answer: (d)
y Micro, Small and Medium Enterprises
Option (d) is correct: Liquidity is the ease with
which an asset or security can be converted y Export Credit
into cash, without impacting its market price. y Education
Various kinds of assets are understood to y Housing
possess different liquidity situations.
y Social Infrastructure
y Currency is the most liquid form of asset as
the ease of convertibility to currency/ cash y Renewable Energy and
is the hallmark for judging the liquidity of y Others (the others category includes
other assets. personal loans to weaker sections, loans
y Demand deposits come next in terms of to distressed persons, loans to state
liquidity as they can be accessed by the sponsored organisations for SC/ST)
depositors as and when they demand it. It Weaker Sections Under PSL: Small and
is the ordinary current account balances Marginal Farmers, Scheduled Castes and
held by depositors. Scheduled Tribes, Beneficiaries of Differential
y Savings account balances can also be Rate of Interest (DRI) scheme, Self Help
accessed by borrowers with comparative Groups, Distressed farmers indebted to non-
ease; however, they are less liquid than institutional lenders, Persons with disabilities,
demand deposits as there are some limits etc.
like minimum balance to be maintained,
etc., in case of Savings deposits. 31. Answer: (c)
y Time deposits are the deposits of a fixed Option (c) is correct: Open Market Operations
nature, generally consisting of a time (OMOs) is a monetary policy tool or instrument
obligation for example: Fixed deposits, they employed by the Reserve Bank of India (RBI)
come the last out of the above-mentioned to manage the liquidity in the banking system.
assets in terms of liquidity.
y It is the sale and purchase of government
Securities and Treasury Bills by the RBI or
30. Answer: (d) the central bank of the country.

Priority Sector Lending (PSL) is aimed to y The objective of OMO is to regulate the

488 1B1: Bank Classification_Explanation


unacademy.com | Download the Unacademy app
Give your feedback here: Link
money supply in the economy. When the y It regulates the commercial banks of the
RBI wants to increase the money supply in country both at policy and operational
the economy, it purchases the government levels. Matters coming under the Reserve
Securities from the market, and it sells Bank’s regulatory umbrella include liquidity
government Securities to suck out liquidity of assets, branch expansion, merger of
from the system. banks, winding-up of banks.
y The RBI carries out the OMO through
commercial banks and does not directly
deal with the public. OMO is one of the tools 34. Answer: (c)
that RBI uses to smoothen the liquidity Option 1 is correct: Regional Rural Banks were
conditions through the year and minimize formed under the Regional Rural Banks Act,
its impact on the interest rate and inflation 1976 on the basis of the Narasimhan Working
rate levels. Group (1975). They were primarily set up to
provide direct credit to the rural and agricultural
sectors of the country. The ownership equity in
32. Answer: (d) the Regional Rural Banks is held by the Central
Bank Rate is the rate at which the RBI lends to Government, respective State Governments,
the banks without any security or collateral. An and a sponsor bank in the ratio of 50:15:35
increase in the Bank Rate will make borrowing respectively.
from the RBI expensive for the Commercial Option 2 is not correct: National Bank for
Banks. This will make further lending by these Agriculture and Rural Development (NABARD)
banks to borrowers expensive, which will came into existence on 12th July 1982 by
reduce the overall lending activity and thus the transferring the agricultural credit functions
money supply in the economy. of RBI and refinance functions of the then
Option (d) is correct: The RBI employs a tight Agricultural Refinance and Development
money policy wherein it seeks to reduce the Corporation (ARDC). Its’ mission is to promote
money supply in the economy to generally bring sustainable and equitable agriculture and rural
inflation under control. Hence, an increase in development through participative financial
bank rate indicates that the RBI is following and non-financial interventions, innovations,
a tight money policy because as soon as the technology, and institutional development for
bank rate increases, cost of borrowings of the securing prosperity. NABARD does not grant
commercial banks from the RBI also increase direct credit assistance to rural households,
and this will force the commercial banks to instead it provides refinancing facilities to
increase their lending rate. High lending rate such segments.
directly affects the customers, discouraging Option 3 is correct: Land Development Banks
them from taking loans, which results in less (LDBs) are registered as societies under the
money supply in the market. Co-operative societies acts of states. LDBs
provide long-term loans to the agriculturists
33. Answer: (d) for permanent improvements on land.

Option (d) is correct: The Reserve Bank of Elimination Technique


India (RBI) is the Central Bank of the country y The National Bank for Agriculture and
regulating the Indian banking system. The RBI Rural Development (NABARD) does not
was established on April 1, 1935, in accordance grant direct credit to rural households.
with the provisions of the Reserve Bank of Option (a), (b), and (d) can be easily
India Act, 1934. eliminated and we can mark option (c)

1B1: Bank Classification_Explanation 489


unacademy.com | Download the Unacademy app
Give your feedback here: Link
as the correct answer. we can mark option (c) as the correct
answer.

35. Answer: (c)


Money supply is the total stock of all types 36. Answer: (d)
of money (currency, deposit, etc.) held by the The Reserve Bank of India (RBI) acts as a
public at any specific point of time. It is very banker ‘s bank in order to facilitate a smooth
significant to note that the term public includes inter-bank transfer of funds, or to make
all the economic entities except government payments and to receive funds on their behalf,
and banking systems (this money is not in smooth functioning of the banking sector. The
actual circulation in the economy hence not RBI provides a multitude of facilities to banks
part of the money supply). such as opening accounts for banks, managing
Statement 1 is correct: Purchase of government Payment and Settlement System, lending
securities from the public by the Central banks in crisis situations, etc.
Bank will lead to the Central Bank paying Statement 1 is correct: The Reserve Bank of
public money in exchange of the government India (RBI) holds the legally mandated reserve
securities, which will lead to an increase in the requirements of the banks with itself as
money supply. deposits. Banks are required to maintain a
Statement 2 is not correct: However, Deposit portion of their demand and time liabilities
of currency in commercial banks by the public as cash reserves with the Reserve Bank. For
and sale of government securities to the public this purpose, they need to maintain accounts
by the central bank will reduce the money with the Reserve Bank. They also need to keep
supply in the economy. accounts with the Reserve Bank for settling
inter-bank obligations, such as, clearing
Statement 3 is correct: Also borrowing by the transactions of individual bank customers who
government from the central bank will lead to have their accounts with different banks or
an increased money supply as the borrowings clearing money market transactions between
made by the government would be spent upon two banks, buying and selling securities and
development or social welfare expenses, which foreign currencies.
in effect will transfer this additional money to
the market and into the hands of the public. Statement 2 is correct: Through the Marginal
Standing Facility and Repo Operations the RBI
Statement 4 is not correct: By selling provides funds to the commercial banks in
Government Securities to the public, the times of need.
central bank sucks money held with the public
as the public purchases those securities. This Statement 3 is correct: Moral suasion is a step
does not result in the increase in money supply taken by the central bank for convincing and
in the economy rather it decreases money advising the commercial banks to adhere to
supply. the policies and act in the desired manner
through general interaction.
Elimination Technique
y Sale of government securities would 37. Answer: (d)
decrease money supply in the economy.
Therefore, option (b) and (d) can be Explanation:
easily eliminated. Statement 1 and Option (d) is correct: Selective credit control
statement 3 are likely to be correct. So, refers to a qualitative method of credit control

490 1B1: Bank Classification_Explanation


unacademy.com | Download the Unacademy app
Give your feedback here: Link
by the central bank. The method aims, unlike y Women especially are often considered as
general or quantitative methods, at the not credit-worthy by banks. Microfinance
regulation of credit taken for specific purposes came as a boon for the financially vulnerable
or branches of economic activity. It aims at and left out populations of the society.
encouraging good credit, i.e., development
credit while at the same time discouraging
bad credit, i.e., speculative credit, etc. The 40. Answer: (a)
instruments of selective credit control are Option (a) is correct: Commercial Banks have
minimum margins, regulation of consumer had the highest share in disbursement of credit
credit for durable goods, rationing of credit, etc. to agriculture and allied activities.
Cash Reserve Ratio (CRR) is not an instrument y The Indian agriculture sector receives the
of selective credit control. highest amount of credit from Commercial
Banks. Initially it was limited but after
38. Answer: (a) 2007 this limit was increased and now the
Commercial Bank’s share in credit given
Option (a) is correct: Bank Rate is the rate at agriculture is around 60%.
which the RBI lends to the commercial banks
without keeping a collateral. Lowering of
the bank rate will make access to borrowed 41. Answer: (a)
funds cheaper for banks, this benefit accruing Explanation:
to them is passed on to the borrower in the
y A Non-Banking Financial Company (NBFC) is
economy in the form of lower interest rates on
a company registered under the Companies
further lending by the commercial banks. This
Act, engaged in the business of loans and
increases the credit uptake in the economy
advances, acquisition of shares/stocks/
and thus the money supply and liquidity.
bonds/debentures/securities issued by
Government or local authority or other
39. Answer: (d) marketable securities. The workings and
Option (d) is correct: Microfinance generally operations of NBFCs are regulated by the
refers to the provision of basic financial Reserve Bank of India (RBI). (Statement 1
services such as loans, saving accounts, fund is correct)
transfers and insurances for low-income but y NBFCs are allowed to accept/renew public
economically active people. deposits for a minimum period of 12 months
y In most instances the term microfinance and a maximum period of 60 months.
refers to the provision of small loans y They cannot accept deposits which are
(microcredits) for micro-entrepreneurs. The repayable on demand. (Statement 2 is not
idea of microfinance, however, is not new correct)
but can be traced back to the principle of y They cannot offer gifts/incentives or any
self-help and solidarity which was devised other additional benefits to the depositors.
by savings banks and cooperative banking
y They should have minimum investment-
groups.
grade credit ratings.
y Around two thirds of the world population
y The repayment of deposits by NBFCs is not
is cut off from the conventional financial
guaranteed by the Reserve bank of India.
market. Low-income people typically have
no collateral and therefore no chance to
take out a loan, to save money or to invest
for the future.

1B1: Bank Classification_Explanation 491


unacademy.com | Download the Unacademy app
Give your feedback here: Link
42. Answer: (c) and is mainly focused on the development
Explanation: of the primary sector.

Option (c) is correct:


y The nationalization of insurance companies 43. Answer: (c)
began in the year 1956 after the passing of Explanation:
the Life Insurance Corporation of India Act
1956. Statement 1 is correct: The National Housing
Bank (NHB) is the apex institution in India
y The erstwhile name of the State Bank of for housing. It was set up as a wholly owned
India (SBI) is the Imperial bank of India. The subsidiary of the Reserve Bank of India (RBI) in
State Bank of India (SBI) was nationalized 1988 under the National Housing Bank Act 1987.
in 1955 after the Government of India
acquired the majority stake in Imperial Statement 2 is correct: The Small Industries
Bank of India. Development Bank of India (SIDBI) was
established in 1990 as a wholly owned
y The Banking Regulation Act was passed subsidiary of the Industrial Development Bank
in 1949. It outlines a framework for of India (IDBI) with the aim to aid in the growth
supervision and regulation of Commercial and development of micro, small and medium-
banks in India. Initially, the Act was known scale enterprises (MSME). Currently, the
as the Banking Companies Act. ownership is held by 34 Government of India
y The first Five Year Plan was launched in 1951 owned/controlled institutions.

492 1B1: Bank Classification_Explanation


unacademy.com | Download the Unacademy app
Give your feedback here: Link
3 1B2: NPA, Bad-Loans, BASEL

1. With reference to the ‘Banks Board Bureau loan to.


(BBB)’, which of the following statements 2. It helps in evaluating the emerging risk
are correct ? (2022) of a firm that a bank is going to give
1. The Governor of RBI is the Chairman of loan to.
BBB.
3. The higher a borrowing firm’s level of
2. BBB recommends for the selection of Interest Coverage Ratio, the worse is its
heads for Public Sector Banks. ability to service its debt.
3. BBB helps the Public Sector Banks in Select the correct answer using the code
developing strategies and capital raising given below:
plans.
(a) 1 and 2 only
Select the correct answer using the code
given below : (b) 2 only

(a) 1 and 2 only (c) 1 and 3 only

(b) 2 and 3 only (d) 1, 2 and 3

(c) 1 and 3 only


(d) 1, 2 and 3
4. What was the purpose of the Inter-Creditor
Agreement signed by Indian banks and
2. Consider the following statements: (2021) financial institutions recently?[2019]
1. The Governor of the Reserve Bank of (a) To lessen the Government of India's
India (RBI) is appointed by the Central perennial burden of fiscal deficit and
Government. current account deficit
2. Certain provisions in the Constitution of (b) To support the infrastructure projects
India give the Central Government the of Central and State Governments
right to issue directions to the RBI in
public interest. (c) To act as independent regulator in case
of applications for loans of Rs 50 crore
3. The Governor of the RBI draws his power or more
from the RBI Act.
(d) To aim at faster resolution of stressed
Which of the above statements are correct? assets of Rs 50 crore or more which are
(a) 1 and 2 only under consortium lending.
(b) 2 and 3 only
(c) 1 and 3 only 5. The Chairman of public sector banks are
(d) 1, 2 and 3 selected by the: (2019)
(a) Banks Board Bureau

3. What is the importance of the term (b) Reserve Bank of India


“Interest Coverage Ratio” of a firm in India? (c) Union Ministry of Finance
(2020) (d) Management of concerned bank
1. It helps in understanding the present
risk of a firm that a bank is going to give

1B2: NPA, Bad-Loans, BASEL 493


unacademy.com | Download the Unacademy app
Give your feedback here: Link
6. With reference to the governance of public (c) It is a disinvestment plan of the
sector banking in India, consider the Government regarding Central Public
following statements: (2018) Sector Undertakings.
1. Capital infusion into public sector banks (d)
It is an important provision in
by the Government of India has steadily ‘The Insolvency and Bankruptcy
increased in the last decade. Code’ recently implemented by the
2. To put the public sector banks in order, Government.
the merger of associate banks with the
parent State Bank of India has been
affected. 9. ‘Basel III Accord’ or simply ‘Basel III’, often
seen in the news, seeks to: (2015)
Which of the statements given above is/are (a) develop national strategies for the
correct? conservation and sustainable use of
(a) 1 only biological diversity
(b) 2 only (b) improve banking sector’s ability to deal
(c) Both 1 and 2 with financial and economic stress and
improve risk management
(d) Neither 1 nor 2
(c) reduce the greenhouse gas emissions
but places a heavier burden on
7. Consider the following statements: (2018) developed countries
1. Capital Adequacy Ratio (CAR) is the
(d) transfer technology from developed
amount that banks have to maintain in
Countries to poor countries to
the form of their own funds to offset
enable them to replace the use of
any loss that banks incur if the account
chlorofluorocarbons in refrigeration
holders fail to repay dues.
with harmless chemicals
2. CAR is decided by each individual bank.
Which of the statements given above is/are
10. Why is the offering of “teaser loans” by
correct?
commercial banks a cause of economic
(a) 1 only concern? (2011)
(b) 2 only 1. The teaser loans are considered to be
an aspect of subprime lending and
(c) Both 1 and 2
banks may be exposed to the risk of
(d) Neither 1 nor 2 defaulters in future.
2. In India, the teaser loans are mostly
8. Which of the following statements best given to inexperienced entrepreneurs to
describes the term ‘Scheme for Sustainable set up manufacturing or export units.
Structuring of Stressed Assets (S4A)’, Which of the statements given above is/are
recently seen in the news? (2017) correct?
(a)
It is a procedure for considering
(a) 1 only
ecological costs of developmental
schemes formulated by the Government. (b) 2 only

(b) It is a scheme of RBI for reworking the (c) Both 1 and 2


financial structure of big corporate (d) Neither 1 nor 2
entities facing genuine difficulties.

494 1B2: NPA, Bad-Loans, BASEL


unacademy.com | Download the Unacademy app
Give your feedback here: Link
1B2: NPA, Bad-Loans, BASEL _
3 Explanation
1. Answer: (b) the Bank, consider it necessary in the public
Statement 1 is not correct: Banks Board Bureau interest.
comprises the Chairman, three ex-officio Statement 3 is correct: According to the RBI
members i.e. Secretary, Department of Public Act, 1934, the Governor have the powers related
Enterprises, Secretary of the Department of to the general supervision and direction of the
Financial Services, and Deputy Governor of Bank’s affairs and business. Section 7 of the
the Reserve Bank of India, and five expert RBI Act also says:
members, two of which are from the private Save as otherwise provided in regulations made
sector. The Chairman is selected by the central by the Central Board, the Governor and in his
government and the RBI governor does not absence, the Deputy Governor nominated by
head it. him on his behalf, shall also have powers of
Statements 2 and 3 are correct: Banks Board general superintendence and direction of the
Bureau recommends the selection of head for affairs and the business of the Bank, and may
Public Sector Banks and other key personnel exercise all powers and do all acts and things
if required. As it was based on the P J Nayak which may be exercised or done by the Bank.
Committee recommendations it developed
strategies for raising capital and improving the Elimination Technique
performance of PSBs. y As we know that the Reserve Bank of
India (RBI) is not a constitutional body
Elimination Technique and there are no such direct provisions
y The Governor of RBI is not the chairman related to RBI as mentioned in the
of the Bank Board Bureau (BBB). Indian Constitution. So, statement
Therefore, Options (a), (c), and (d) can 2 or Options (a), (b), and (d) can be
be easily eliminated and we can mark eliminated and we can mark option (c)
Option (b) as the correct answer. as the correct answer.

2. Answer: (c)
Statement 1 is correct: According to Section 3. Answer: (a)
8 of the RBI Act 1934, the Central Government Interest Coverage Ratio is an indicator of a
(Union Government) appoints the Governor company’s ability to pay interest out of its
and Deputy Governors of the Reserve Bank of current earnings. It is expressed as the number
India (RBI). of times that the interest can be covered from
Statement 2 is not correct: The RBI is an entity its current earnings.
independent of the government as it takes its Importance of the term Interest Coverage
own decisions. However, in certain instances, it Ratio:
has to listen to the government. This provision y It can help in understanding the present
in the RBI Act is contained in Section 7 of RBI risk, as a firm with a low-interest coverage
Act, 1934, which says: ratio just may not have enough earnings
(1) The Central Government may, from time in order to meet the annual interest
to time, give such directions to the Bank as it obligations. (Statement 1 is correct)
may, after consultation with the Governor of y Emerging risk can also be assessed by

1B2: NPA, Bad-Loans, BASEL _Explanation 495


unacademy.com | Download the Unacademy app
Give your feedback here: Link
taking a holistic look at the ratio for several Board of Directors.
years to predict trends in the movement of y To help banks in terms of developing
earnings vis a vis the interest obligations in business strategies and capital raising
the future. (Statement 2 is correct) plans etc.
y A higher Interest Coverage Ratio reflects y To build a data bank containing data relating
the higher potential of the company’s to the performance of PSBs/FIs and its
earnings to service its outstanding debt officers.
and interest. (Statement 3 is not correct)
y To advise the Government on the desired
structure at the Board level, and, for senior
4. Answer: (d) management personnel, for each PSB and
Explanation: Financial Institution (FI).

An intercreditor agreement (ICA) is an


agreement between one or more creditors 6. Answer: (b)
(lenders) who have shared interests in a The purpose of capital infusion into public
particular borrower. sector banks (PSBs) is to strengthen the
Option (d) is correct: The agreement is part lending capacity and improve credit growth
of project ‘Sashakt’, which was approved among PSBs that are saddled with a heavy,
under the recommendations of Sunil Mehta non-performing assets (NPAs) as well as are
Committee to address the bad loan problem of struggling to meet capital adequacy ratio.
India. The objective is to use this ICA for faster Statement 1 is not correct: Capital infusion into
facilitation of resolution of stressed assets. public sector banks by the Government of India
It is aimed at the resolution of loan accounts has not steadily increased in the last decade.
with a size of Rs. 50 crore and above under As per the Department of Financial Services
consortium lending (an arrangement where data, Rs. 25,000 crores were infused in 2016-
two or more lenders join together to finance a 2017 Financial Year while only Rs. 10,000 crores
single borrower). were infused in the FY of 2017-18.
Statement 2 is correct: In 2017, State Bank of
5. Answer: (a) India merged with five of its associate banks,
Option (a) is correct: The Chairman of public which are: State Bank of Bikaner and Jaipur,
sector banks are selected by the Banks State Bank of Mysore, State Bank of Travancore,
Board Bureau, which was set up on the State Bank of Hyderabad, and State Bank of
recommendations of the P J Nayak Committee. Patiala along with Bharatiya Mahila Bank. To put
the public sector banks in order, the merger of
Additional Information:
these associate banks with the parent State
Banks Board Bureau is an autonomous body, Bank of India has been affected. This step has
created with an objective to empower the aided in the banks rationalising its costs and
boards of the public sector banks. Its main enhancing its depositor base.
functions are:
y Selection and appointment of Board of
Directors in PSBs and FIs (Whole-time 7. Answer: (a)
Directors and Non-Executive Chairman). Banks need to maintain a stipulated amount
y Advise the Government on matters relating of capital as a percentage of its total credit
to appointments, confirmation or extension exposure in order to offset any losses that the
of tenure and termination of services of the banks may incur if its credit turns bad.

496 1B2: NPA, Bad-Loans, BASEL _Explanation


unacademy.com | Download the Unacademy app
Give your feedback here: Link
Statement 1 is correct: Capital Adequacy Ratio the financial sector to the real economy.
(CAR) is the ratio of a bank’s capital in relation to y Basel Committee on Banking Supervision is
its risk-weighted assets and current liabilities. a consortium of Central Banks from various
It is the amount that banks have to maintain in countries. In the aftermath of the 2008
the form of their own funds to offset any loss subprime crisis, it rolled out the Basel
that banks incur if the account holders fail to III accord, which enlists certain capital
repay dues. adequacy norms to be followed by the
Statement 2 is not correct: CAR is decided by banking system internationally so that in
the Reserve Bank of India (RBI) and not by each future financial and economic situations
individual bank. It is decided by the Central can be better handled. The Basel III accord
Bank to prevent commercial banks from taking prescribes the following for banks:
excess leverage and becoming insolvent in the ○ Banks need to maintain a Minimum
process. Total Capital Ratio of 12.9% of their total
As per RBI norms, Indian Scheduled Commercial risk-weighted assets.
Banks are required to maintain a CAR of 9% ○ Under it, 10.5% should come from Tier 1
while Indian Public Sector Banks maintain a capital and 2% from Tier 2 capital.
CAR of 12%.
○ Banks also need to maintain a Counter-
Cyclical Capital Buffer.
8. Answer: (b)
Explanation: 10. Answer: (a)
Option (b) is correct: Scheme for Sustainable Teaser loans are debts which are provided at
Structuring of Stressed Assets (S4A) aims very low interest rates at the starting but are
at deep financial restructuring of big debted subject to subsequent future interest rate
projects by allowing lenders (bank) to acquire revision. They are provided in order to attract
equity of the stressed project. The scheme maximum takers for the loan products.
makes financial restructuring of large projects
at the same time helping the lender’s ability to y They are considered an aspect of subprime
deal with such stressed assets. It is intended lending (likely prone to default) as borrowers
to restore the flow of credit to critical sectors gets attracted to lower interest rates,
including infrastructure. provided as attraction tactics. However,
in future with rate revision, they may find
it difficult to cope up with the interest
9. Answer: (b) payments and default on it. (Statement 1
Option (b) is correct: ‘Basel III Accord’, a global is correct)
regulatory framework for more resilient banks y Teaser loans are generally provided in the
and banking systems” adopted in December home and automobile segments. They
2010, with the objective to improve the banking are not mostly given to inexperienced
sector’s ability to absorb shocks arising from entrepreneurs to set up manufacturing or
financial and economic stress, whatever the export units. (Statement 2 is not correct)
source, thus reducing the risk of spill over from

1B2: NPA, Bad-Loans, BASEL _Explanation 497


unacademy.com | Download the Unacademy app
Give your feedback here: Link
4 1C: Sharemarket, Companies Act

1. In the context of finance, the term ‘beta’ correct ?


refers to (2023) (a) 1 only
(a) the process of simultaneous buying (b) 2 only
and selling of an asset from different
platforms (c) Both 1 and 2

(b) an investment strategy of a portfolio (d) Neither 1 nor 2


manager to balance risk versus reward
(c) a type of systemic risk that arises where 4. With reference to India, consider the
perfect hedging is not possible following statements : (2021)
1. Retail investors through demat
(d) a numeric value that measures the accounts can invest in ‘Treasury Bills’
fluctuations of a stock to changes in and ‘Government of India Debt Bonds’
the overall stock market in the primary market.
2. The ‘Negotiated Dealing System- Order
2. With reference to the Indian economy, Matching’ is a government securities
what are the advantages of “Inflation- trading platform of the Reserve Bank of
Indexed Bonds (IIBs)” ? (2022) India.
1. Government can reduce the coupon
rates on its borrowing by way of IIBs. 3. The ‘Central Depository Services Ltd.’ is
jointly promoted by the Reserve Bank of
2. IIBs provide protection to the investors India and the Bombay Stock Exchange.
from uncertainty regarding inflation.
Which of the statements given above is/are
3. The interest received as well as capital correct?
gains on IIBs are not taxable.
(a) 1 only
Which of the statements given above are
correct ? (b) 1 and 2

(a) 1 and 2 only (c) 3 only

(b) 2 and 3 only (d) 2 and 3

(c) 1 and 3 only


5. In the context of the Indian economy,
(d) 1, 2 and 3 non-financial debt includes which of the
following? (2020)
3. With reference to Convertible Bonds, 1. Housing loans owned by household
consider the following statements: (2022) 2. Amounts outstanding on credit cards
1. As there is an option to exchange the 3. Treasury Bills
bond for equity, Convertible Bonds pay
a lower rate of interest. Select the correct answer using the code
given below:
2. The option to convert to equity affords
the bondholder a degree of indexation (a) 1 only
to rising consumer prices. (b) 1 and 2 only
Which of the statements given above is/are (c) 3 only

498 1C: Sharemarket, Companies Act


unacademy.com | Download the Unacademy app
Give your feedback here: Link
(d) 1, 2 and 3 (c) Promissory Note
(d) Participatory Note
6. Which of the following phrases defines the
nature of the ‘Hundi’ generally referred to 9. Consider the following statements: (2018)
in the sources of the post-Harsha period? 1. The Reserve Bank of India manages and
[2020] services Government of India Securities
(a) An advisory issued by the king to his but not any State Government
subordinates Securities.
(b) A diary to be maintained for daily 2. Treasury bills are issued by the
accounts Government of India and there are
(c) A bill of exchange no treasury bills issued by the State
(d) An order from the feudal lord to his Governments.
subordinates 3. Treasury bills are issued at a discount
from the par value.

7. With reference to the Indian Economy, Which of the statements given above is/are
consider the following statements: (2020) correct?
1. ’Commercial Paper’ is a short term (a) 1 and 2 only
unsecured promissory note. (b) 3 only
2. ‘Certificate of Deposit’ is a long-term (c) 2 and 3 only
instrument issued by the Reserve Bank
of India to a corporation. (d) 1, 2 and 3

3. ‘Call Money’ is a short-term finance


used for interbank transactions. 10. With reference to ‘IFC Masala Bonds’,
4. ‘Zero-Coupon Bonds’ are the interest- sometimes seen in the news, which of the
bearing short-term bonds issued by statements given below is/are correct?
the Scheduled Commercial Banks to (2016)
corporations. 1. The International Finance Corporation,
which offers these bonds, is an arm of
Which of the statements given above is/are the World Bank.
correct?
2. They are the rupee-denominated bonds
(a) 1 and 2 only and are a source of debt financing for
(b) 4 only the public and private sectors.
(c) 1 and 3 only Select the correct answer using the code
(d) 2, 3 and 4 only given below.
(a) 1 only

8. Which of the following is issued by (b) 2 only


registered foreign portfolio investors to (c) Both 1 and 2
overseas investors who want to be part of (d) Neither 1 nor 2
the Indian stock market without registering
themselves directly? (2019)
(a) Certificate of Deposit 11. With reference to ‘Financial Stability
(b) Commercial Paper and Development Council’, consider the
following statements: (2016)

1C: Sharemarket, Companies Act 499


unacademy.com | Download the Unacademy app
Give your feedback here: Link
1. It is an organ of NITI Aayog. (d) 1, 2 and 3
2. It is headed by the Union Finance
Minister. 12. What does venture capital mean? (2014)
3. It monitors macroprudential supervision (a)
A short-term capital provided to
of the economy. industries
Which of the statements given above is/are (b) A long-term start-up capital provided
correct? to new entrepreneurs
(a) 1 and 2 only (c) Funds provided to industries at times of
(b) 3 only incurring losses

(c) 2 and 3 only (d) Funds provided for replacement and


renovation of industries

500 1C: Sharemarket, Companies Act


unacademy.com | Download the Unacademy app
Give your feedback here: Link
1C: Sharemarket, Companies Act
4 _Explanation
1. Answer: (d) of the option to convert the bond into common
In finance, the beta (β or market beta or beta stock.
coefficient) is a measure of how an individual The option to convert to equity affords the
asset moves (on average) when the overall bondholder a degree of indexation to rising
stock market increases or decreases. Thus, consumer prices.
beta is a useful measure of the contribution
of an individual asset to the risk of the market
4. Answer: (b)
portfolio when it is added in small quantities.
Beta is a numeric value that measures the Statement 1 is correct: Retail investors through
fluctuations of a stock to changes in the overall the Demat account can invest in ‘Treasury Bills’
stock market. and ‘Government of India Debt Bonds’ in the
primary market. RBI Retail Direct Scheme allows
retail investors to buy and sell government
2. Answer: (a) securities (G-sec) online both in the primary
Statement 1 is correct: Inflation Indexed Bonds and secondary markets. According to details
(IIBs) - IIBs are bonds wherein both coupon provided by RBI, these small investors can now
flows and Principal amounts are protected invest in G-Secs by opening a gilt securities
against inflation. The inflation index used in account with the RBI. The account opened will
IIBs may be Wholesale Price Index (WPI) or be called Retail Direct Gilt (RDG) Account.
Consumer Price Index (CPI). Globally, IIBs were Statement 2 is correct: The ‘Negotiated Dealing
first issued in 1981 in the UK. In India, the System-Order Matching’ is a government
Government of India through the RBI issued securities trading platform of the Reserve
IIBs (linked to WPI) in June 2013. Government Bank of India. It is a screen-based electronic
can reduce the coupon rates on its borrowing anonymous order matching system for
by way of IIBs. secondary market trading in Government
Statement 2 is correct: Inflation indexed bonds securities owned by RBI.
(IIBs) provide insurance to investors from Statement 3 is not correct: Central Depository
inflation and cost savings for the Government Services Limited (CDSL) was founded in 1999
on account of reduction in coupon payments for convenient, dependable, and secured
with lowering inflation rate, elimination of depository services. CDSL is promoted by
uncertainty risk premium, and containing BSE, which later divested its stakes among
inflationary expectations. nationalized banks.
Statement 3 is not correct: Extant tax provisions
will be applicable on interest payment and
capital gains on IIBs. There will be no special 5. Answer: (d)
tax treatment for these bonds. Debts are contractual obligations to repay
monetary loans, often with related interest
expenses.
3. Answer: (c)
Option (d) is correct: Non-financial debt is the
Both the statements are correct:
debt held by the households, governments, and
convertible bonds tend to offer a lower coupon firms, not in the financial sector. It includes
rate or rate of return in exchange for the value housing loans owned by households, amounts

1C: Sharemarket, Companies Act 501


unacademy.com | Download the Unacademy app
Give your feedback here: Link
outstanding on credit cards, treasury bills. 8. Answer: (d)
Additional Information: Option (d) is correct: The registered portfolio
Treasury bills: It is a short-term money market investors issue Participatory Notes to overseas
instrument, issued by the RBI on behalf of investors who want to be a part of the Indian
the government to curb temporary liquidity stock market without registering themselves
shortfalls. directly.
Participatory Note is a derivative instrument
issued in foreign jurisdictions, by a SEBI
6. Answer: (c)
registered Foreign Institutional Investor (FII)
Explanation: against underlying Indian securities. The
Option (c) is correct: The movement of goods underlying Indian security instrument may be
during the Medieval period was facilitated equity, debt, derivatives or may even be an
by the growth of a financial system which index.
permitted easy transfer of money from one part Additional Information:
of the country to another. This was done using
y Certificate of Deposits: It is a negotiable
Hundis, which were instruments of credit or
money market instrument and issued
bills of exchange. The Hundis was a letter of
in dematerialised form or as a usance
credit payable after a period at a discount.
Promissory Note, for funds deposited at a
bank or other eligible financial institution
7. Answer: (c) for a specified time period.

Statement 1 is correct: Commercial papers y Commercial Paper: It is an unsecured


are debt instruments issued by corporations money market instrument issued in the
to generally meet their short-term obligations. form of a promissory note.
They are also unsecured, that is they are not y Promissory Note: It is a financial instrument
backed or secured by a particular asset. in which one party (the issuer) promises
Statement 2 is not correct: Certificate of to pay the second party (the payee) a
Deposits are fixed income financial instruments determined sum of money at a fixed or
generally issued by commercial banks to future date on demand of the payee.
individuals/ other lenders.
Statement 3 is correct: Call Money is a short- 9. Answer: (c)
term debt generally used by banks to lend and
Government Securities are tradable
borrow within themselves generally to meet
instruments issued by the Central Government
short term obligations or shortfalls in reserve
or the State Governments. They acknowledge
requirements mandated by law.
the government’s debt obligation.
Statement 4 is not correct: Zero-Coupon
Statement 1 is not correct: The Reserve
Bond (Also known as Pure Discount Bond or
Bank of India (RBI) manages and services the
Accrual Bond) refers to those bonds which are
government securities and debt for the centre
issued at a discount to its par value and makes
as well as State Government Securities in
no periodic interest payment (non-interest
terms of the agreements that it enters in with
bearing), unlike a normal coupon-bearing bond.
other State Governments.
Zero Coupon Bonds are not interest bearing
instead they are issued at deep discounts and Statement 2 is correct: Treasury bills or T-bills,
redeemable at par on a future date. which are money market instruments and
are short term debt instruments issued by
the Government of India. Presently, they are

502 1C: Sharemarket, Companies Act


unacademy.com | Download the Unacademy app
Give your feedback here: Link
issued in three tenors, namely, 91 day, 182 day exchange rate fluctuation is shifted to the
and 364 day. They are not issued by the State investor. Both Public and Private Sector
Governments. entities can use them to raise funds
Statement 3 is correct: Treasury bills are zero for purposes such as Debt Financing.
coupon securities and pay no interest. Instead, (Statement 2 is correct)
they are issued at a discount and redeemed at
the face value at maturity. 11. Answer: (c)
Currently, the RBI acts as banker to all the State The Financial Stability and Development Council
Governments in India (including Union Territory (FSDC) was set up in 2016. It was primarily
of Puducherry), except Sikkim. For Sikkim, it established to coordinate the activities of
has limited agreement for management of its various financial regulatory authorities.
public debt.
y FSDC was constituted by an Executive
Elimination Technique Order of the Union Government as a non-
y The Reserve Bank of India is a banker statutory apex body under the Ministry
of the central as well as the state of Finance. It is not an organ of the NITI
government and itt deals with G-Secs (National Institute of Transforming India)
of central and state government as Aayog. (Statement 1 is not correct)
well. So, Option (a) and (d) can be y The Union Finance Minister is the Chairman
easily eliminated. of the Financial Stability and Development
y Statements 2 and 3 are well facts. So, Council. This means the FSDC is headed by
we can mark Option (c) as the correct the Union Finance Minister. (Statement 2 is
answer. correct)
y The four primary functions of the Council
consist of Monitoring the macroprudential
supervision of the economy, focus
10. Answer: (c) on financial literacy, strengthening
Masala Bonds are the bonds which are issued mechanisms of financial development
outside India by Indian Entities to raise capital. and stability, addressing the coordination
These are usually used by corporations to raise issues of various regulators. (Statement 3
funds from foreign investors. is correct)
y The first Masala Bonds was issued by the Elimination Technique
International Finance Corporation (IFC) in
2014. The IFC is an arm of the World Bank; y NITI Aayog is a policy think-tank body
other arms along it include International of the Government of India. FSDC is
Bank for Reconstruction and Development not its organ. Option (a) and (d) can be
(IBRD), International Development Agency eliminated. Statement 2 is more likely
(IDA), Multilateral Investment Guarantee to be true. Therefore, we can mark (c)
Agency (MIGA), and International Centre for as the correct answer.
Settlement of Investment Disputes (ICSID).
(Statement 1 is correct)
y These bonds, although issued in foreign 12. Answer: (b)
markets, are denominated in Rupees. That Option (b) is correct: Start-up companies with
is, their value is stated in Rupees and their the potential to grow need a certain amount
repayment obligation is also in terms of of investment. Wealthy investors like to invest
Rupees. Due to this the risk of currency

1C: Sharemarket, Companies Act 503


unacademy.com | Download the Unacademy app
Give your feedback here: Link
their capital in such businesses with a long- investments are risky as they are illiquid
term growth perspective. This capital is known but are capable of giving impressive returns
as venture capital and the investors are called if invested in the right venture. The returns
venture capitalists. to the venture capitalists depend upon the
y Venture capital is the capital invested in growth of the company. Venture capitalists
a project in which there is a substantial have the power to influence major decisions
element of risk, typically in a new or of the companies they are investing in as it
expanding business (entrepreneurs). Such is their money at stake.

504 1C: Sharemarket, Companies Act


unacademy.com | Download the Unacademy app
Give your feedback here: Link
1D1: Insurance, Pension, Financial
5 inclusion
1. Consider the following statements: (2023) 3. In India, under cyber insurance for
1. The Self-Help Group (SHG) Programme individuals, which of the following benefits
was originally initiated by the State are generally covered, in addition to
Bank of India by providing microcredit payment for the loss of funds and other
to the financial deprived. benefits ? [2020]
1. Cost of restoration of the computer
2. In an SHG, all members of a group system in case of malware disrupting
take responsibility for a loan that an access to one’s computer
individual member takes.
2. Cost of a new computer if some
3. The Regional Rural Banks and Scheduled miscreant wilfully damages it, if proved
Commercial Banks support SHGs. so
How many of the above statements are 3. Cost of hiring a specialized consultant
correct? to minimize the loss in case of cyber
(a) Only one extortion
(b) Only two 4. Cost of defence in the Court of Law if
(c) All three any third party files a suit

(d) None Select the correct answer using the code


given below :

2. Under the Kisan Credit Card Scheme, short- (a) 1, 2 and 4 only
term credit support is given to farmers for (b) 1, 3 and 4 only
which of the following purposes? (2020) (c) 2 and 3 only
1. Working capital for maintenance of farm
assets (d) 1, 2, 3 and 4

2. Purchase of combine harvesters,


tractors and mini trucks 4.
The Service Area Approach was
3. Consumption requirements of farm implemented under the purview of: (2019)
households (a)
Integrated Rural Development
Programme
4. Post-harvest expenses
(b) Lead Bank Scheme
5. Construction of family house and setting
up of village cold storage facility (c)
Mahatma Gandhi National Rural
Employment Guarantee Scheme
Select the correct answer using the code
given below: (d) National Skill Development Mission

(a) 1, 2 and 5 only


(b) 1, 3 and 4 only 5. Regarding ‘Atal Pension Yojana’, which of
the following statements is/are correct?
(c) 2, 3, 4 and 5 only
[2016]
(d) 1, 2, 3 and 4 1. It is a minimum guaranteed pension
scheme mainly targeted at unorganized
sector workers.

1D1: Insurance, Pension, Financial inclusion 505


unacademy.com | Download the Unacademy app
Give your feedback here: Link
2. Only one member of a family can join 8. Pradhan Mantri Jan-Dhan Yojana has been
the scheme. launched for (2015)
3. Same amount of pension is guaranteed (a) providing housing loan to poor people
for the spouse for life after subscriber’s at cheaper interest rates
death. (b) promoting women’s Self-Help Groups in
Select the correct answer using the code backward areas
given below. (c) promoting financial inclusion in the
(a) 1 only country

(b) 2 and 3 only (d) providing financial help to marginalized


communities
(c) 1 and 3 only
(d) 1, 2 and 3
9. The basic aim of Lead Bank Scheme is that
(2012)
6. Pradhan Mantri MUDRA Yojana is aimed at
(a) big banks should try to open offices in
[2016]
each district
(a) bringing the small entrepreneurs into
formal financial system (b) there should be stiff competition among
the various nationalized banks
(b) providing loans to poor farmers for
cultivating particular crops (c) individual banks should adopt particular
districts for intensive development
(c) providing pensions to old and destitute
persons (d) all the banks should make intensive
efforts to mobilize deposits
(d) funding the voluntary organizations
involved in the promotion of skill
development and employment 10. Consider the following: (2012)
generation 1. Hotels and restaurants
2. Motor transport undertakings
7. With reference to ‘Pradhan Mantri Fasal 3. Newspaper establishments
Bima Yojana’, consider the following 4. Private medical institutions
statements: [2016]
1. Under this scheme, farmers will have to The employees of which of the above can
pay a uniform premium of two percent have the 'Social Security' coverage under
for any crop they cultivate in any season Employees' State Insurance Scheme?
of the year. (a) 1, 2 and 3 only
2. This scheme covers post-harvest losses (b) 4 only
arising out of cyclones and unseasonal (c) 1, 3 and 4 only
rains.
(d) 1, 2, 3 and 4
Which of the statements given above is/are
correct?
11. With reference to “Aam Admi Bima Yojana”.
(a) 1 only
Consider the following statements: (2011)
(b) 2 only 1. The member insured under the scheme
(c) Both 1 and 2 must be the head of the family or
(d) Neither 1 nor 2 earning member of the family in a rural
landless household.

506 1D1: Insurance, Pension, Financial inclusion


unacademy.com | Download the Unacademy app
Give your feedback here: Link
2. The member insured must be in the age (a) 1 only
group of 30 to 65 years. (b) 2 and 3 only
3. There is a provision for free scholarship (c) 1 and 3 only
for up to two children of the insured
(d) 1, 2 and 3
who are studying between classes 9
and 12.
Which of the statements given above is/are
correct?

1D1: Insurance, Pension, Financial inclusion 507


unacademy.com | Download the Unacademy app
Give your feedback here: Link
1D1: Insurance, Pension, Financial
5 inclusion_Explanation
1. Answer: (b) working capital required for floriculture,
Statement 1 is incorrect: While the Self-Help horticulture, etc. (Statement 1 is correct)
Group (SHG) program has made significant y Consumption requirements of farmer
contributions to microcredit and financial households. (Statement 3 is correct)
inclusion in India, it was not initiated by the y To meet the short-term credit requirements
State Bank of India (SBI). The SHG movement for cultivation of crops.
in India started as a grassroots initiative
y Post-harvest expenses. (Statement 4 is
and gained momentum through various
correct)
organizations, NGOs, and government agencies.
y Produce Marketing loan.
Statement 2 is correct: In a Self-Help Group
(SHG), it is common for all members of the y Investment credit requirement for
group to take joint responsibility for the agriculture and allied activities like pump
loans that individual members acquire. This sets, sprayers, dairy animals, floriculture,
collective responsibility is a core principle of horticulture, etc.
the SHG model and serves as a form of peer y Short term credit requirements of rearing
support and social collateral. of animals, birds, fish, shrimp, other aquatic
Statement 3 is correct: Both Regional Rural organisms, capture of fish.
Banks (RRBs) and Scheduled Commercial
Banks (SCBs) provide support to Self-Help
3. Answer: (b)
Groups (SHGs) in India. These banks play a
crucial role in extending financial services, Explanation:
including loans, savings accounts, and other Option (b) is correct: All above benefits except
financial products, to SHGs. “Cost of a new computer if some miscreant
wilfully damages it, if proved so” are generally
covered under cyber insurance policies for
2. Answer: (b)
individuals. For availing the benefit of Cost of
The Kisan Credit Card scheme is a Government new computer under wilful damage by some
of India scheme which provides farmers with miscreant a hardware specific insurance would
timely access to credit. The scheme was be required.
launched with the aim of providing short-term
formal credit to farmers and was created by
NABARD (National Bank for Agriculture and 4. Answer: (b)
Rural Development). Option (b) is correct: The Service Area Approach
The KCC scheme was introduced to ensure (SAA) was introduced in April 1989 as a part of
that the credit requirements for farmers in the the Lead Bank Scheme for planned and orderly
agriculture, fisheries and animal husbandry development of rural and semi-urban areas. It
sector were being met. was applicable to all scheduled commercial
banks including Regional Rural Banks. Under
The aim of Kisan Credit card:
SAA, each bank branch in a rural or semi-urban
y Working capital for maintenance of farm area was designated to serve an area of 15 to
assets, activities allied to agriculture, 25 villages and the branch was responsible for
like dairy animals, inland fishery and also meeting the needs of bank credit in its service

508 1D1: Insurance, Pension, Financial inclusion_Explanation


unacademy.com | Download the Unacademy app
Give your feedback here: Link
area. The primary objective of SAA was to 7. Answer: (b)
increase productive lending and forge effective Explanation:
linkages between bank credit, production,
The Pradhan Mantri Fasal Bima Yojana is a
productivity and increase in income levels.
Central government sponsored crop insurance
scheme, which has been implemented since
5. Answer: (c) 2018.
Explanation: Statement 1 is not correct: The premium rate
for Kharif Crops is 2%, for Rabi Crops it is 1.5%
Statement 1 is correct: Atal Pension Yojana
and for annual commercial and horticulture
is a pension scheme that primarily aims at
crops it is 5%.
providing financial security to the unorganized
sector workers. The scheme provides for Statement 2 is correct: The scheme under
minimum fixed pension for the beneficiaries its ambit covers Yield Losses, prevention of
ranging from Rupees 1000 per month to Rupees sowing due to bad weather, post-harvest
5000 per month. losses and localized problems like landslide,
hailstorm etc.
Statement 2 is not correct: The scheme is open
to all adults of a family. Earlier under the Pradhan Mantri Fasal Bima
Yojana, it was mandatory for loanee farmers
Statement 3 is correct: The spouse of a
to obtain crop insurance under the scheme.
subscriber after his/her death is entitled to
However, in 2020 obtaining crop insurance
the same amount of pension, as was being
under the scheme has been made optional.
provided to the subscriber.

8. Answer: (c)
6. Answer: (a)
Option (c) is correct: Pradhan Mantri Jan-Dhan
Explanation:
Yojana (PMJDY) was launched in August 2014
Option (a) is correct: MUDRA stands for Micro (in two phases) under the National Mission for
Units Development and Refinance Agency. Financial Inclusion. The aim of the PMJDY is to
Under the Pradhan Mantri MUDRA Yojana non- promote financial inclusion in the country by
corporate, non-farm small/micro enterprises providing universal access to banking facilities
can avail loans of up to Rs 10 lakh. It aimed at with at least one basic banking account for
bringing the small entrepreneurs into a formal every household, financial literacy, access to
financial system. credit, insurance and pension.
MUDRA is a refinance scheme where the Under this scheme, people not having a bank
authority formed under the MUDRA Yojana does account can open a simple Savings Bank A/c.
not directly lend to the borrowers, instead These accounts have certain benefits for the
the loans are provided by various financial account holders:
institutions like Commercial Banks, NBFCs etc.
y No requirement for maintaining a minimum
Loans provided under the MUDRA Yojana are
balance in the account.
classified into three types, based upon the size
of the loans granted. y Provision of interest on the deposited
amounts.
y Shishu Loan: Loans of up to Rs 50000
y Rupay Debit Card is provided to the account
y Kishore Loan: Loans ranging from Rs 50000
holders.
to 5 lakh
y Overdraft facility of up to Rupees 10000.
y Tarun Loan: Loans ranging from Rs 5 lakh
to Rupees 10 lakh. y Benefits of other government schemes

1D1: Insurance, Pension, Financial inclusion_Explanation 509


unacademy.com | Download the Unacademy app
Give your feedback here: Link
like Pradhan Mantri Suraksha Bima Yojana, provisions of the Employee State Insurance
Direct Benefit Transfer, etc., are available (ESI) Act to various classes of establishments,
through these accounts. industrial, commercial, or agricultural or
PMJDY has provided a platform for the three otherwise. Under these enabling provisions
social security schemes viz. Pradhan Mantri most of the State Govts have extended the ESI
Jeevan Jyoti Bima Yojana (PMJJBY), Pradhan Act to certain specific class of establishments,
Mantri Suraksha Bima Yojana (PMSBY), Atal such as, shops, hotels, restaurants,
Pension Yojana (APY) and Pradhan Mantri Mudra cinemas, preview theatres, motors transport
Yojana (PMMY). undertakings and newspaper establishments
etc., employing 20 or more persons. The ESI
Scheme is mainly financed by contributions
9. Answer: (c) raised from employees covered under the
Option (c) is correct: Introduced in 1969 the Lead scheme and their employers, as a fixed
Bank Scheme purposed to provide services in percentage of wages. The employees of Private
rural areas, through the service area approach Medical institutions are also eligible under the
wherein a particular bank was designated as scheme.
the main/lead bank for a particular area and
was expected to provide banking services to
11. Answer: (c)
the people residing there.
Aam Admi Bima Yojana is a social security
y It aimed that individual bank should
scheme launched in 2007 in order to provide
adopt particular districts for intensive
insurance coverage to certain specified
development, as due to concentrated focus
sections.
banking activities could be enhanced in
hitherto un-serviced/ lowly serviced rural y Under this scheme only one member from
areas. the household can be insured. Either
it is the head of the family as chosen by
the family in question or it is the earning
10. Answer: (d) member in case of the rural landless
Employees’ State Insurance Scheme of India is household. (Statement 1 is correct)
an integrated social security scheme tailored y The member insured must be in the age
to provide social protection to workers and group of 18 to 59 years. (Statement 2 is not
their dependants, in the organised sector, in correct)
contingencies, such as, sickness, maternity and
y Along with other insurance benefits the
death or disablement due to an employment
scheme also includes Scholarship benefits
injury or occupational hazard.
available for up to two children of the
Option (d) is correct: The State or Central insured member studying between class 9
government is empowered to extend the and 12. (Statement 3 is correct)

510 1D1: Insurance, Pension, Financial inclusion_Explanation


unacademy.com | Download the Unacademy app
Give your feedback here: Link
6 2A1: Budget Direct Taxes

1. With reference to India's decision to Parliament.


levy an equalization tax of 6% on online 2. No amount can be withdrawn from the
advertisement services offered by non- Consolidated Fund of India without the
resident entities, which of the following authorization of the Parliament of India.
statements is/are correct? (2018)
1. It is introduced as a part of the Income 3. All the disbursements made from Public
Tax Act. Account also need Authorization from
the Parliament of India.
2. Non-resident entities that offer
advertisement services in India can Which of the following statements given
claim a tax credit in their home country above is/are correct?
under the "Double Taxation Avoidance (a) 1 and 2 only
Agreements". (b) 2 and 3 only
Select the correct answer using the code (c) 2 only
given below:
(d) 1, 2 and 3
(a) 1 only
(b) 2 only 4. What will follow if a Money Bill is
(c) Both 1 and 2 substantially amended by the Rajya Sabha?
(d) Neither 1 nor 2 [2013 - I]
(a) The Lok Sabha may still proceed with
the Bill, accepting or not accepting the
2. Consider the following statements: [2015] recommendations of the Rajya Sabha
1. The Rajya Sabha has no power either to
reject or to amend a Money Bill. (b) The Lok Sabha cannot consider the bill
further
2. The Rajya Sabha cannot vote on the
Demands for Grants. (c) The Lok Sabha may send the Bill to the
Rajya Sabha for reconsideration
3. The Rajya Sabha cannot discuss the
Annual Financial Statement. (d) The President may call a joint sitting for
passing the Bill
Which of the statements given above is /
are correct?
5. Under which of the following circumstances
(a) 1 only may 'capital gains' arise? (2012)
(b) 1 and 2 only 1. When there is an increase in the sales
(c) 2 and 3 only of a product

(d) 1, 2 and 3 2. When there is a natural increase in the


value of the property owned
3. When you purchase a painting and there
3. With reference to the Union Government,
is a growth in its value due to increase
consider the following statements. (2015)
in its popularity
1. The Department of Revenue is
responsible for the preparation of the Select the correct answer using the code
Union Budget that is presented to the given below:

2A1: Budget Direct Taxes 511


unacademy.com | Download the Unacademy app
Give your feedback here: Link
(a) 1 only (c) The Prime Minister of India
(b) 2 and 3 only (d) The Union Finance Minister
(c) 2 only
(d) 1, 2 and 3 7. Which one of the following is responsible
for the preparation and presentation of
Union Budget to the Parliament? [2010]
6. The authorization for the withdrawal of (a) Department of Revenue
funds from the Consolidated Fund of India (b) Department of Economic Affairs
must come from [2011 - I]
(a) The President of India (c) Department of Financial Services

(b) The Parliament of India (d) Department of Expenditure

512 2A1: Budget Direct Taxes


unacademy.com | Download the Unacademy app
Give your feedback here: Link
2A1: Budget Direct Taxes
6 _Explanation
1. Answer: (d) recommendations made by Rajya Sabha in
Equalization levy of 6% was introduced in this regard. (Statement 1 is Correct)
2016 as a direct tax on non-resident service y The Rajya Sabha has no power to vote on
providers for online advertisement as existing the demand for grants; it is the exclusive
Double Tax Avoidance Agreements and other privilege of the Lok Sabha. (Statement 2 is
taxation provisions could not rightfully tax the Correct)
overseas digital transactions, due to which y Article 112 states that, the President shall in
these transaction by non-resident service respect of every financial year cause to be
providers for online advertisement largely went laid before both the Houses of Parliament
untaxed. Such a levy could ensure the country a statement of estimated receipts and
gets its rightful share of taxes. expenditure of the Government of India
Statement 1 is not correct: Equalization tax of for that year, which implies that Rajya
6% was introduced as a separate item under Sabha can discuss the Annual Financial
the Finance Bill 2016 and not as a part of Statement. (Statement 3 is not Correct)
Income Tax Act.
Statement 2 is not correct: There was a major 3. Answer: (c)
criticism of the decision that the tax credit
Statement 1 is not correct: Under the Finance
was not available to the taxpayers under the
Ministry, the Budget Division of the Department
“Double Taxation Avoidance Agreements”
of Economic Affairs is the nodal body which is
which could result in double taxation for the
directly responsible for the formulation of the
non-residents.
Union Budget.
Recent amendments to the Finance Act 2020
Statement 2 is correct: All the legally authorised
have expanded the net of equalization levy to
payments on behalf of the Government of India
non-resident e-commerce players also.
are made out of the Consolidated Fund of India.
No money out of this fund can be appropriated
2. Answer: (b) (issued or drawn) except in accordance with a
Explanation: Parliamentary law.

The Constitution has defined the relative roles Consolidated Fund of India, a fund to which
or position of both the Houses of Parliament all receipts are credited, and all payments are
about the enactment of the budget in the debited. This consists of:
following way: y All revenues received by the Government
y A money bill or finance bill dealing with of India.
taxation cannot be introduced in the Rajya y All loans raised by the Government by the
Sabha, it must be introduced only in the issue of treasury bills, loans or ways and
Lok Sabha. means of advances.
y The Rajya Sabha has no power either to y All money received by the government in
reject or to amend a Money Bill. It should repayment of loans forms the Consolidated
return the Money bill (or Finance bill) to Fund of India.
the Lok Sabha within fourteen days. The Statement 3 is not correct: All public money
Lok Sabha can either accept or reject the (other than those which are credited to the

2A1: Budget Direct Taxes 513


unacademy.com | Download the Unacademy app
Give your feedback here: Link
Consolidated Fund of India) received by or and cost price (lower) of the asset. Capital loss
on behalf of the Government of India shall be arises when the cost price is higher than the
credited to the Public Account of India. This selling price.
includes provident fund deposits, judicial y Increase in sales of a product increases the
deposits, savings bank deposits, departmental turnover of that firm. It does not result in
deposits, remittances and so on. This account capital gains. (Statement 1 is not correct)
is operated by executive action; that is, the
y The natural increase in the value of the
payments from this account can be made
property owned results in capital gains as
without Parliamentary appropriation.
property like real estate are capital assets.
Additional Information: (Statement 2 is correct)
y Apart from the Consolidated Fund of India and y Increase in a painting’s popularity causes
Public Account of India, Constitution envisaged the growth in the value of the painting which
another fund for the Central Government, that is an asset of capital nature. (Statement 3
is, The Contingency Fund of India. is correct)
y The Contingency Fund of India, a fund into
which amounts determined by law are paid
6. Answer: (b)
from time to time. This fund is placed at
the disposal of the President, and he can Explanation:
make advances out of it to meet unforeseen Consolidated Fund of India is a fund to which
expenditure pending its authorization by the all government receipts are credited, and all
Parliament. The fund is held by the finance payments are debited.
secretary on behalf of the President. It is
Option (b) is correct: All the legally authorized
also operated by executive action.
payments on behalf of the Government of India
are made out of the Consolidated Fund of India.
4. Answer: (a) No money out of this fund can be appropriated
(issued or drawn) except in accordance with a
Explanation:
parliamentary law.
Option (a) is correct: A Money Bill can be
introduced only in the Lok Sabha and not
in the Rajya Sabha. Rajya Sabha cannot 7. Answer: (b)
amend or reject a Money Bill. It should Explanation:
return the bill to the Lok Sabha within
Union Budget is an annual financial statement
14 days, either with recommendations or
of income (receipts) and spending (expenditure)
without recommendations. The Lok Sabha
of the government for a particular financial
can either accept or reject all or any of the
year. Article 112 of the Indian Constitution
recommendations of the Rajya Sabha. In both
requires the annual financial statement to be
the cases, the money bill is deemed to have
laid before the Parliament.
been passed by the two Houses.
Option (b) is correct: The Union Budget
is made through a consultative process
5. Answer: (b) involving the Ministry of Finance, NITI Aayog,
Capital gain is the profit one earns on the sale and other ministries. The Budget Division of
of an asset like stocks, bonds, or real estate. the Department of Economic affairs under
It results in capital gain when the selling price the Ministry of Finance is the nodal body for
of an asset exceeds its purchase price. It is the preparing the budget.
difference between the selling price (higher)

514 2A1: Budget Direct Taxes


unacademy.com | Download the Unacademy app
Give your feedback here: Link
7 2A2: Budget Indirect Taxes GST

1. Consider the following items: (2018) (a) 1 only


1. Cereal grains hulled (b) 2 and 3 only
2. Chicken eggs cooked (c) 1 and 3 only
3. Fish processed and canned (d) 1, 2 and 3
4. Newspapers containing advertising
material 3. The sales tax you pay while purchasing a
Which of the above items is/are exempted toothpaste is: (2014)
under GST (Good and Services Tax)? (a) tax imposed by the Central Government
(a) 1 only (b) tax imposed by the Central Government
(b) 2 and 3 only but collected by the State Government

(c) 1, 2 and 4 only (c) tax imposed by the State Government


but collected by Central Government
(d) 1, 2, 3 and 4
(d) tax imposed and collected by State
Government
2. What is/are the most likely advantages
of implementing 'Goods and Services Tax
4. Which one of the following is not a feature
(GST)'? (2017)
of “Value Added Tax”? (2011)
1. It will replace multiple taxes collected
(a) It is a multi-point destination-based
by multiple authorities and will thus
system of tax.
create a single market in India.
(b) It is a tax levied on value addition at each
2. It will drastically reduce the 'Current
stage of transaction in the production
Account Deficit' of India and will enable
distribution chain.
it to increase its foreign exchange
reserves. (c) It is a tax on the final consumption of
goods or services and must ultimately
3. It will enormously increase the growth
be borne by the consumer.
and size of the economy of India and
will enable it to overtake China in the (d) It is basically a subject of the Central
near future. Government, and the State Governments
are only a facilitator for its successful
Select the correct answer using the code
implementation.
given below:

2A2: Budget Indirect Taxes GST 515


unacademy.com | Download the Unacademy app
Give your feedback here: Link
2A2: Budget Indirect Taxes GST
7 _Explanation
1. Answer: (a) 3. Answer: (d)
Goods and Services Tax was introduced in Sales tax is always a percentage of a product’s
2017 as a destination based indirect tax. The value which is charged at the point of exchange
Constitution was amended under the 101st or purchase. The different kinds of sales tax
Constitutional Amendment Act to insert Article are retail, manufacturers, wholesale, use, and
246A and Article 269A among other changes to value added tax (VAT).
facilitate the GST. Option (d) is correct: Sales tax was an indirect
Option (a) is correct: Cereal grains hulled, tax imposed on the trade of goods and services.
chicken eggs, and fish were exempted under Sales tax was imposed and collected by the
the GST. Since, chicken eggs cooked can be respective State Government. However, for
made available in the restaurants, these interstate trade there used to be the Central
are not exempt from the GST. Also, GST is Sales Tax, which was imposed by the Central
chargeable at 5% and 18% rates in different Government.
cases on provision of advertising services in y The sales tax rates used to be different in
the newspapers. different states, going against the concept
of a National Market. The Goods and Service
2. Answer: (a) Tax introduced on 1st July 2017 replaced
the sales tax regime and has now created
Goods and Services Tax (GST) was introduced a common national market based on the
in 2017 as a destination-based indirect tax. principles of one nation one market.
The Constitution was amended under the 101st
Constitutional Amendment Act to insert Article
246A and Article 269A, among other changes 4. Answer: (d)
to facilitate the GST. Value added Tax or VAT was an indirect tax
Statement 1 is correct: There used to be a wide which was levied on goods and services on each
disparity in various taxes and tax rates across point of value addition in the production cycle
states; GST has subsumed the majority of starting from the raw material to the end stage
such indirect taxes and now acts as a unified of retail sale. It is a multipoint destination-
indirect tax with common tax rates across the based system of tax, borne by the consumer
country, allowing for seamless movement of (not by the seller) on consuming final goods
goods and factors of production across states and services.
for the country, effectively creating a single Statement (d) is not correct: VAT is entirely
national market. a subject of the State Government. The VAT
Statement 2 and statement 3 are not correct: system is enforced by the State governments
GST is a domestic indirect tax. There is no (not the Central Government) and the rates of
such stated advantage that GST drastically taxation also vary from state to state.
reduces the ‘Current Account Deficit’ of However, The GST subsumed all previous
India and will enable it to increase its foreign taxes that were levied on the sale of goods or
exchange reserves. There is also no such stated provision of services by either Central or State
advantage which says that it will enable India governments.
to take over China in the near future.

516 2A2: Budget Indirect Taxes GST


unacademy.com | Download the Unacademy app
Give your feedback here: Link
2BC: Finance Commission,
8 BlackMoney, Subsidies
1. Consider the following : (2023) (a) Diversion of resources to the purchase
1. Demographic performance of real estate and investment in luxury
housing
2. Forest and ecology
(b) Investment in unproductive activities
3. Governance reforms
and purchase of precious stones,
4. Stable government jewelry, gold, etc.
5. Tax and fiscal efforts (c) Large donations to political parties and
For the horizontal tax devolution, the growth of regionalism
Fifteenth Finance Commission used how (d) Loss of revenue to the State Exchequer
many of the above as criteria other than due to tax evasion
population area and income distance?
(a) Only two 4. In India, which of the following review
(b) Only three the independent regulators in sectors like
telecommunications, insurance, electricity
(c) Only four
etc.? [2019]
(d) All five 1. Ad Hoc Committees set up by the
Parliament.
2. Which one of the following situations best 2. Parliamentary Department Related
reflects “Indirect Transfers” often talked Standing Committees
about in the media recently with reference
3. Finance Commission
to India ? (2022)
(a) An Indian company investing in a foreign 4. Financial Sector Legislative Reforms
enterprise and paying taxes to the Commission
foreign country on the profits arising 5. NITI Aayog
out of its investment
Select the correct answer using the code
(b) A foreign company investing in India given below.
and paying taxes to the country of its
(a) 1 and 2
base on the profits arising out of its
investment (b) 1 , 3 and 4

(c) An Indian company purchases tangible (c) 3, 4 and 5


assets in a foreign country and sells (d) 2 and 5
such assets after their value increases
and transfers the proceeds to India
5. With reference to the 'Prohibition of Benami
(d) A foreign company transfers shares and Property Transactions Act, 1988 (PBPT
such shares derive their substantial Act)', consider the following statements:
value from assets located in India [2017]
1. A property transaction is not treated as
3. Which one of the following effects of a benami transaction if the owner of the
creation of black money in India has been property is not aware of the transaction.
the main cause of worry to the Government 2. Properties held benami are liable for
of India? (2021)

2BC: Finance Commission, BlackMoney, Subsidies 517


unacademy.com | Download the Unacademy app
Give your feedback here: Link
confiscation by the Government. (a) 1 only
3. The Act provides for three authorities (b) 2 only
for investigations but does not provide (c) 1 and 3 only
for any appellate mechanism.
(d) 2 and 3 only
Which of the statements given above is/are
correct?

518 2BC: Finance Commission, BlackMoney, Subsidies


unacademy.com | Download the Unacademy app
Give your feedback here: Link
2BC: Finance Commission,
8 BlackMoney, Subsidies_Explanation
1. Answer: (b) public procurement, non-profit organizations,
Tax devolution refers to the distribution of tax external trade, international transactions
revenues between the central government and involving tax havens, and the informal service
the state governments. It is a constitutional sector. So, the Diversion of resources to the
mechanism established to allocate the purchase of real estate and investment in
proceeds of certain taxes among the Union and luxury housing, Investment in unproductive
the states in a fair and equitable manner. For activities and the purchase of precious stones,
horizontal devolution, it has suggested 12.5% jewellery, gold, etc., are a source of generation
weightage to demographic performance, 45% of Black money. Large donations to political
to income, 15% each to population and area, parties are other ways to create black money.
10% to forest and ecology and 2.5% to tax and Option (d) is correct: Loss of revenue to the
fiscal efforts. state exchequer due to tax evasion is the
main cause of worry for the Government of
India. The estimates of the black money in the
2. Answer: (d) system vary from 7% of GDP to 120% of GDP,
Option (d) is correct: highlighting the wide variance in the methods
Indirect transfers refer to situations where of estimation.
foreign entities own shares or assets in
India, the shares of such foreign entities are 4. Answer: (a)
transferred instead of a direct transfer of the
underlying assets in India. Only those indirect Explanation:
transfer transactions wherein more than 50% NITI, FC don’t look after it so #4 and #5 is
of undelying assets are in India will be subject wrong, by elimination the answer is A.
to levy of cpaital gains tax Option (a) is correct: In India, the Ad Hoc
Committees set up by the Parliament and
3. Answer: (d) the Parliamentary Department related
Standing Committees review the independent
Black money includes all funds earned through regulators in various sectors. Finance
illegal activity and otherwise legal income Commission, Financial Sector Legislative
that is not recorded for tax purposes. Higher Reforms Commission, NITI Aayog have no role
amounts of black money in the economy often in reviewing the independent regulators.
lead to increased corruption. On the other
hand, black money can reduce the negative y Ad-Hoc Committees set up by the
impact of oppressive laws. Black money can be Parliament: They are appointed for a
illegally disguised as legitimate money through specific purpose and they cease to exist
money laundering. when they finish the task assigned to them
and submit a report. They are appointed by
While the source of the generation of black the house or the Speaker or the Presiding
money may lie in any sphere of economic Officer of both the houses in consultation
activity, there are certain sectors of the economy with each other as and when necessary for
or activities which are more vulnerable to this a particular purpose. They have also been
menace. These include real estate, the bullion set up to investigate serious issues which
and jewellery market, financial markets, have greatly agitated the public mind, and

2BC: Finance Commission, BlackMoney, Subsidies_Explanation 519


unacademy.com | Download the Unacademy app
Give your feedback here: Link
which involve frauds or corruption on a Statement 1 is not correct: A property
large scale. transaction is treated as a Benami transaction
even if the owner of the property is not aware
y Parliamentary Department Related
of the transaction as the basic principle in the
Standing Committees: There are
Indian legal framework is “Ignorance of the law
24 departmentally related Standing
can never be an excuse for the violation of law”.
Committees in India. They cover under their
jurisdiction Central Ministries/ Departments Statement 2 is correct: Any property held
and while reviewing their work they also by benami is liable for confiscation by the
look into the working of the regulators in government and also liable for confiscation
their specific departments. by the government without the payment of
compensation.
Statement 3 is not correct: An appellate
5. Answer: (b)
mechanism has been provided in the form of
Explanation: Adjudicating Authority and Appellate Tribunal.

520 2BC: Finance Commission, BlackMoney, Subsidies_Explanation


unacademy.com | Download the Unacademy app
Give your feedback here: Link
9 3A: BoP, CAD Currency Exchange

1. Consider the investments in the following Select the correct answer using the code
assets: (2023) given below :
1. Brand recognition (a) 1 only
2. Inventory (b) 2 only
3. Intellectual property (c) Both 1 and 2
4. Mailing list of clients (d) Neither 1 nor 2
How many of the above are considered
intangible investments? 4. With reference to the Indian economy,
(a) Only one consider the following statements: (2022)
(b) Only two 1. An increase in Nominal Effective
Exchange Rate (NEER) indicates the
(c) Only three
appreciation of rupee.
(d) All four
2. An increase in the Real Effective
Exchange Rate (REER) indicates an
2. With reference to the Indian economy, improvement in trade competitiveness.
consider the following statements : (2022)
3. An increasing trend in domestic inflation
1. A share of the household financial
relative to inflation in other countries is
savings goes towards government
likely to cause an increasing divergence
borrowings.
between NEER and REER.
2. Dated securities issued at market-
Which of the above statements are correct
related rates in auctions form a large
?
component of internal debt.
(a) 1 and 2 only
Which of the above statements is/are
correct ? (b) 2 and 3 only

(a) 1 only (c) 1 and 3 only

(b) 2 only (d) 1, 2 and 3

(c) Both 1 and 2


(d) Neither 1 nor 2 5. Consider the following statements: (2022)
1. Tight monetary policy of the US Federal
Reserve could lead to capital flight.
3. With reference to the expenditure made
2. Capital flight may increase the interest
by an organization or a company, which of
cost of firms with existing External
the following statements is/are correct ?
Commercial Borrowings (ECBs).
(2022)
1. Acquiring new technology is capital 3. Devaluation of domestic currency
expenditure. decreases the currency risk associated
with ECBs.
2. Debt financing is considered capital
expenditure, while equity financing is Which of the statements given above are
considered revenue expenditure. correct ?

3A: BoP, CAD Currency Exchange 521


unacademy.com | Download the Unacademy app
Give your feedback here: Link
(a) 1 and 2 only (b) Article 112 and Article 110(1) of the
(b) 2 and 3 only Constitution of India

(c) 1 and 3 only (c) Article 113 of the Constitution of India

(d) 1, 2 and 3 (d) Provisions of the Fiscal Responsibility


and Budget Management Act, 2003

6. Consider the following statements : (2021)


9. Consider the following statements: [2020]
The effect of devaluation of a currency
1. The value of Indo-Sri Lanka trade has
is that it necessarily
consistently increased in the last
1. improves the competitiveness of the decade.
domestic exports in the foreign markets
2. “Textile and textile articles” constitute
2. increases the foreign value of domestic an important item of the trade between
currency India and Bangladesh.
3. improves the trade balance 3. In the last five years, Nepal has been
Which of the above statements is/are the largest trading partner of India in
correct? South Asia.

(a) 1 only Which of the statements given above is/are


correct?
(b) 1 and 2
(a) 1 and 2 only
(c) 3 only
(b) 2 only
(d) 2 and 3
(c) 3 only

7. Consider the following : (2021) (d) 1, 2 and 3


1. Foreign currency convertible bonds
2. Foreign institutional investment with 10. Term ‘West Texas Intermediate’, sometimes
certain conditions found in news, refers to a grade of_ _
(Prelims-2020)
3. Global depository receipts
(a) Crude oil
4. Non-resident external deposits
(b) Bullion
Which of the above can be included in
(c) Rare earth elements
Foreign Direct Investments?
(d) Uranium
(a) 1, 2 and 3
(b) 3 only
11. With reference to Foreign Direct Investment
(c) 2 and 4
in India, which one of the following is
(d) 1 and 4 considered its major characteristic? (2020)
(a) It is the investment through capital
8. Along with the Budget, the Finance Minister instruments essentially in a listed
also places other documents before the company.
Parliament which include ‘The Macro (b) It is largely non-debt, creating capital
Economic Framework Statement’. The flow.
aforesaid document is presented because
(c) It is an investment which involves debt-
this is mandated by [2020]
servicing.
(a) Long standing parliamentary convention

522 3A: BoP, CAD Currency Exchange


unacademy.com | Download the Unacademy app
Give your feedback here: Link
(d) It is the instrument made by foreign (a) Spices
institutional investors in the Government (b) Fresh fruits
securities.
(c) Pulses

12. “Gold Tranche” (Reserve Tranche) refers to (d) Vegetable oils


(2020)
(a) a loan system of the World Bank 16. Consider the following statements: (2019)
(b) one of the operations of a Central Bank 1. Most of India’s external debt is owed by
governmental entities.
(c) a credit system granted by WTO to its
members 2. All of India’s external debt is denominated
in US dollars.
(d) a credit system granted by IMF to its
members Which of the statements given above is/are
correct?

13. If another global financial crisis happens (a) 1 only


in the near future, which of the following (b) 2 only
actions/policies are most likely to give (c) Both 1 and 2
some immunity to India? (2020)
(d) Neither 1 nor 2
1. Not depending on the short-term
foreign borrowings
17. In the context of India, which of the
2. Opening up to more foreign banks
following factors is/are contributor/
3. Maintaining full capital account contributors to reducing the risk of a
convertibility currency crisis? (2019)
Select the correct answer using the code 1. The foreign currency earnings of India’s
given below: IT sector.
(a) 1 only 2. Increasing government expenditure.
(b) 1 and 2 only 3. Remittances from Indians abroad.
(c) 3 only Select the correct answer using the code
(d) 1, 2 and 3 given below:
(a) 1 only
14. Among the following, which one is the (b) 1 and 3 only
largest exporter of rice in the world in the (c) 2 only
last five years? (2019)
(d) 1, 2 and 3
(a) China
(b) India
18. Which one of the following is not the most
(c) Myanmar likely measure the Government/RBI takes
(d) Vietnam to stop the slide of the Indian rupee? (2019)
(a) Curbing imports of non-essential goods
15.
Among the agricultural commodities and promoting exports.
imported by India, which one of the (b) Encouraging Indian borrowers to issue
following accounts for the highest imports rupee denominated Masala Bonds.
in terms of value in the last five years? (c) Easing conditions relating to external
(2019) commercial borrowing.

3A: BoP, CAD Currency Exchange 523


unacademy.com | Download the Unacademy app
Give your feedback here: Link
(d) Following an expansionary monetary Governments.
policy. 3. As per the Constitution of India, it is
mandatory for a State to take the Central
19. Consider the following statements: (2019) Government's consent for raising any
1. Purchasing Power Parity (PPP) exchange loan if the former owes any outstanding
rates are calculated by comparing the liabilities to the latter.
prices of the same basket of goods and Which of the statements given above is/are
services in different countries. correct?
2. In terms of PPP dollars, India is the sixth (a) 1 only
largest economy in the world.
(b) 2 and 3 only
Which of the statements given above is/are
correct? (c) 1 and 3 only

(a) 1 only (d) 1, 2 and 3

(b) 2 only
22. Consider the following statements: (2017)
(c) Both 1 and 2 1. Tax revenue as a per cent of GDP of
(d) Neither 1 nor 2 India has steadily increased in the last
decade.
20. Consider the following statements: (2018) 2. The fiscal deficit as a per cent of GDP of
1. The quantity of imported edible oils is India has steadily increased in the last
more than the domestic production of decade.
edible oils in the last five years. Which of the statements given above is/are
2. The Government does not impose any correct?
customs duty on all the imported edible (a) 1 only
oils as a special case.
(b) 2 only
Which of the statements given above is/are
correct? (c) Both 1 and 2

(a) 1 only (d) Neither 1 nor 2

(b) 2 only
23. Which of the following is/are included in
(c) Both 1 and 2 the capital budget of the Government of
(d) Neither 1 nor 2 India? (2016)
1. Expenditure on acquisition of assets
21. Consider the following statements: (2018) like roads, buildings, machinery, etc.
1. The Fiscal Responsibility and Budget 2.
Loans received from foreign
Management (FRBM) Review Committee governments.
Report has recommended a debt to GDP 3. Loans and advances granted to the
ratio of 60% for the general (combined) States and Union Territories.
government by 2023, comprising 40%
for the Central Government and 20% for Select the correct answer using the code
the State Governments. given below.

2. The Central Government has domestic (a) 1 only


liabilities of 21% of GDP as compared (b) 2 and 3 only
to that of 49% of GDP of the State (c) 1 and 3 only

524 3A: BoP, CAD Currency Exchange


unacademy.com | Download the Unacademy app
Give your feedback here: Link
(d) 1, 2 and 3 (d) Renminbi

24. There has been a persistent deficit budget 27. Which of the following best describes the
year after year. Which action/actions of the term ‘import cover’, sometimes seen in the
following can be taken by the government news? (2016)
to reduce the deficit? (2016) (a) It is the ratio of value of imports to the
1. Reducing revenue expenditure Gross Domestic Product of a country.
2. Introducing new welfare scheme (b) It is the total value of imports of a
3. Rationalizing subsidies country in a year.

4. Reducing import duty (c) It is the ratio between the value of


exports and that of imports between
Select the correct answer using the code
two countries.
given below.
(d) It is the number of months of imports
(a) 1 only
that could be paid for by a country’s
(b) 2 and 3 only international reserves.
(c) 1 and 3 only
(d) 1, 2, 3 and 4 28. The problem of international liquidity is
related to the non-availability of: (2015)
(a) goods and services
25.
What is/are the purpose/purposes of
the Government’s ‘Sovereign Gold Bond (b) gold and silver
Scheme’ and ‘Gold Monetization Scheme’? (c) dollars and other hard currencies
(2016)
(d) exportable surplus
1. To bring the idle gold lying with Indian
households into the economy
29. Convertibility of rupee implies: (2015)
2. To promote FDI in the gold and jewellery
(a) being able to convert rupee notes into
sector
gold
3. To reduce Indian’s dependence on gold
(b) allowing the value of rupee to be fixed
imports
by market forces
Select the correct answer using the code
(c) freely permitting the conversion of
given below.
rupee to other currencies and vice versa
(a) 1 only
(d) developing an international market for
(b) 2 and 3 only currencies in India
(c) 1 and 3 only
(d) 1, 2 and 3 30. There has been a persistent deficit budget
year after year. Which of the following
actions can be taken by the government to
26.
Recently, which one of the following reduce the deficit? (2015)
currencies has been proposed to be added 1. Reducing revenue expenditure
to the basket of IMF’s SDR? (2016)
2. Introducing new welfare schemes
(a) Rouble
3. Rationalizing subsidies
(b) Rand
4. Expanding industries
(c) Indian Rupee
Select the correct answer using the code

3A: BoP, CAD Currency Exchange 525


unacademy.com | Download the Unacademy app
Give your feedback here: Link
given below. 3. Balance of Invisibles
(a) 1 and 3 only 4. Special Drawing Rights
(b) 2 and 3 only Select the correct answer using the code
(c) 1 only given below:

(d) 1, 2, 3 and 4 (a) 1 only


(b) 2 and 3 only

31. With reference to the Fourteenth Finance (c) 1 and 3 only


Commission, which of the following (d) 1, 2 and 4
statements is/are correct? (2015)
1. It has increased the share of States
in the central divisible pool from 32 34. The Balance of Payments of a country is a
percent to 42 percent. systematic record of (2013)
(a) all import and export transactions of a
2.
It has made recommendations country during a given period of time,
concerning sector-specific grants. normally a year
Select the correct answer using the code (b) goods exported from a country during a
given below: year
(a) 1 only (c) economic transaction between the
(b) 2 only government of one country to another
(c) Both 1 and 2 (d) capital movements from one country to
(d) Neither 1 nor 2 another

32. With reference to the Union Budget, which 35. Which of the following constitute Capital
of the following is/are covered under Non- Account? (2013)
Plan Expenditure? (2014) 1. Foreign Loans
1. Defence Expenditure 2. Foreign Direct Investment
2. Interest Payments 3. Private Remittances
3. Salaries and pensions 4. Portfolio Investment
4. Subsidies Select the correct answer using the code
Select the correct answer using the code given below:
given below: (a) 1, 2 and 3 only
(a) 1 only (b) 1, 2 and 4 only
(b) 2 and 3 only (c) 2, 3 and 4 only
(c) 1, 2, 3 and 4 (d) 1, 3 and 4 only
(d) None
36. Which one of the following groups of items
33. With reference to Balance of Payments, is included in India’s foreign – exchange
which of the following constitutes/ reserves? (2013)
constitute the Current Account? (2014) (a) Foreign – currency assets, Special
1. Balance of Trade Drawing Rights (SDRs) and loans from
foreign countries.
2. Foreign Assets

526 3A: BoP, CAD Currency Exchange


unacademy.com | Download the Unacademy app
Give your feedback here: Link
(b) Foreign – currency assets, gold holdings 39. In terms of economy, the visit by foreign
of the RBI and SDRs. nationals to witness the XIX Commonwealth
(c) Foreign – currency assets, loans from Games in India amounted to (2011)
the World Bank and SDRs. (a) Export

(d) Foreign – currency assets, gold holdings (b) Import


of the RBI and loans from the World (c) Production
Bank. (d) Consumption

37. Consider the following statements: (2012) 40. Why is the Government of India disinvesting
The price of any currency in international its equity in the Central Public Sector
market is decided by the Enterprises (CPSEs)? (2011)
1. World Bank 1. The Government intends to use the
2. demand for goods/services provided by revenue earned from the disinvestment
the country concerned mainly to pay back the external debt.

3. stability of the government of the 2. The Government no longer intends to


concerned country retain the management control of the
CPSEs.
4. economic potential of the country in
question Which of the statements given above is/are
correct?
Which of the statements given above are
correct? (a) 1 only

(a) 1, 2, 3 and 4 (b) 2 only

(b) 2 and 3 only (c) Both 1 and 2

(c) 3 and 4 only (d) Neither 1 nor 2

(d) 1 and 4 only


41. Which one of the following statements
appropriately describes the “fiscal
38.
Which of the following would include stimulus”? (2011)
Foreign Direct Investment in India? (2012) (a) It is a massive investment by the
1. Subsidiaries of companies in India Government in the manufacturing
2. Majority foreign equity holding in Indian sector to ensure the supply of goods
companies to meet demand surge caused by rapid
economic growth.
3. Companies exclusively financed by
foreign companies (b) It is an intense affirmative action of the
Government to boost economic activity
4. Portfolio investment
in the country.
Select the correct answer using the code
(c) It is the Government’s intensive action
given below:
on financial institutions to ensure
(a) 1, 2, 3 and 4 disbursement of loans to agriculture
(b) 2 and 4 only and allied sectors to promote greater
food production and contain food
(c) 1 and 3 only
inflation.
(d) 1, 2 and 3 only
(d) It is an extreme affirmative action by

3A: BoP, CAD Currency Exchange 527


unacademy.com | Download the Unacademy app
Give your feedback here: Link
the government to pursue its policy of Which of the following action/actions
financial inclusion. can help in reducing the Current Account
Deficit?
42. Both Foreign Direct Investment (FDI) and (a) 1 and 2 only
Foreign Institutional Investor (FII) are (b) 2 and 3 only
related to investment in a country. Which of
the following statements best represents (c) 3 only
an important difference between the two? (d) 1 and 3 only
(2011)
(a) FII helps bring better management skills 44. With reference to the "Tea Board" in India,
and technology, while FDI only brings in consider the following statements:
capital. 1. The Tea Board is a statutory body.
(b) FII helps in increasing capital availability 2. It is a regulatory body attached to the
in general, while FDI only targets specific Minister of Agriculture and Farmers
sectors. Welfare.
(c) FDI flows only into the secondary 3. The Tea Board's Head Office is situated
market, while FII targets the primary in Bengaluru.
market.
4. The Board has overseas offices at Dubai
(d) FII is considered to be more stable than and Moscow.
FDI.
Which of the statements given above are
correct?
43. Consider the following action which the
(a) 1 and 3
Government can take: (2011)
1. Devaluing the domestic currency. (b) 2 and 4
2. Reduction in export subsidy. (c) 3 and 4
3. Adopting suitable policies which attract (d) 1 and 4
greater FDI and more funds from FIIs.

528 3A: BoP, CAD Currency Exchange


unacademy.com | Download the Unacademy app
Give your feedback here: Link
3A: BoP, CAD Currency Exchange
9 _Explanation
1. Answer: (c) Statement and statement 2 are correct :
1. Brand recognition: Brand recognition Acquiring new technology is considered as
refers to the awareness and familiarity that capital expenditure as it will generate profit
customers have with a particular brand. It is an in the future time and helps in creation of
intangible asset as it represents the value and new assets. Debt Financing being long term in
reputation associated with the brand. It is built nature is considered as capital expenditure and
over time through marketing efforts, customer equity financing being short term is considered
experiences, and the overall perception of the under revenue expenditure.
brand in the market.
2. Inventory: In the context of investments, 3. Answer: (a)
typically refers to the physical goods or
Statement 1 is correct: Capital expenditures
products held by a business for sale. Unlike
are incurred when a business uses collateral or
the other assets listed, inventory is considered
takes on debt to buy a new asset or add value
a tangible asset because it has a physical
to an existing asset. Capital expenses include
presence and can be physically counted or
the cost of fixed assets and the acquisition of
measured.
intangible assets. CapEx spending is important
3. Intellectual property: Itrefers to creations for companies to maintain existing property
of the mind, such as inventions, literary and and equipment, and invest in new technology
artistic works, designs, symbols, names, and and other assets for growth.
images used in commerce. Examples include
Capital expenditures (CapEx) are used by a
patents, copyrights, trademarks, and trade
company to acquire, upgrade, and maintain
secrets. Intellectual property is intangible
physical assets such as property, plants,
in nature and can be a valuable asset for
buildings, technology, or equipment.
businesses, as it grants exclusive rights and
protection for their creations or innovations. Statement 2 is not correct: A revenue
expenditure occurs when a company spends
4. Mailing list of clients: It is typically
money on a short-term benefit (i.e., less than
considered an intangible asset. It represents
one year). Typically, these expenditures are
a database or collection of customer contact
used to fund ongoing operations – which, when
information, such as names, addresses, and
they are expensed, are known as operating
email addresses. This asset is valuable to
expenses.
businesses as it enables targeted marketing
and communication with existing or potential Debt financing means borrowing money from
customers. While the mailing list may be stored an outside source and promising to pay it back
electronically or in a physical format, its value with interest by a set date in the future. Equity
lies in the intangible information it contains. financing means someone is putting money or
assets into the business in exchange for some
To summarise, brand recognition, intellectual
percentage of ownership.
property, and the mailing list of clients are
considered intangible investments, while
inventory is a tangible investment. 4. Answer: (c)
Statement 1 is correct: In economics, the NEER
2. Answer: (c) is an indicator of a country’s international

3A: BoP, CAD Currency Exchange 529


unacademy.com | Download the Unacademy app
Give your feedback here: Link
competitiveness in terms of the foreign imports. This may assist the country to grow
exchange (forex) market. exports while decreasing imports. So, it
improves the competitiveness of domestic
An increase in NEER indicates an appreciation
exports in foreign markets.
of the local currency against the weighted
basket of currencies of its trading partners. Statement 3 is not correct: Trade Balance is
the difference between the value of a country’s
Statement 2 is not correct: The real effective
exports and the value of its imports. A country
exchange rate (REER) is the weighted average
that imports more goods and services than it
of a country’s currency in relation to an index or
exports in terms of value has a trade deficit,
basket of other major currencies. An increase
while a country that exports more goods and
in a nation’s REER is an indication that its
services than it imports has a trade surplus.
exports are becoming more expensive and its
Devaluation improves exports, but the imports
imports are becoming cheaper. It is losing its
suffer. So, it is not necessary that the trade
trade competitiveness.
balance will improve.
Statement 3 is correct: An increasing trend
in domestic inflation relative to inflation in
other countries is likely to cause an increasing 7. Answer: (a)
divergence between NEER and REER. A Foreign Direct Investment (FDI) is an
investment made by a firm or individual in
Elimination Technique
one country into business interests located in
y An increase in REER indicates the another country. With FDI, foreign companies
deterioration of a country’s trade are directly involved with day-to-day operations
competitiveness. So, option (a), (b), in the other country. This means they aren’t just
and (d) can be directly eliminated as bringing money with them but also knowledge,
we can mark option (c) as the correct skills and technology.
answer.
Option (a) is correct: Foreign investment
includes Foreign currency convertible bonds,
5. Answer: X Foreign institutional investment with certain
UPSC has dropped this question. conditions. In India, Global depository receipts
are considered FDI.

6.Answer: (a) Foreign investment in Indian securities has also


been made possible through the purchase of
Devaluation is the deliberate downward Global Depository Receipts, Foreign Currency
adjustment of a country’s currency value. Convertible Bonds and Foreign Currency Bonds
The government issuing the currency decides issued by Indian issuers, which are listed,
to devalue a currency. Devaluing a currency traded and settled overseas.
reduces the cost of a country’s exports and
can help shrink trade deficits. The Non-Resident external deposits act as a
‘debt creating’ flow in the balance of payments
Statement 2 is not correct: Devaluation has the accounts and are not part of FDI.
effect of making the home currency cheaper in
comparison to other currencies.
Statement 1 is correct: A currency devaluation 8. Answer: (d)
has two consequences. The devaluation lowers Explanation:
the cost of the country’s goods for foreigners. Option (d) is correct: The Macro-Economic
The devaluation raises the cost of imported Framework statement is presented to
products for domestic customers, discouraging the Parliament as mandated by the Fiscal

530 3A: BoP, CAD Currency Exchange


unacademy.com | Download the Unacademy app
Give your feedback here: Link
Responsibility and Budget Management Act, do not create any debt and are non-volatile.
2003. The FRBM Rules came into force from Hence, these are largely non-debt creating
July 5, 2004. Every year, a macro-economic capital flow and their returns depend on the
framework statement is presented before performance of the projects financed by the
Parliament at the time of the Union Budget foreign investors.
presentation. This is under Section 3(5) of the Additional Information:
Fiscal Responsibility and Budget Management
(FRBM) Act, 2003. Routes through which India gets FDI:
y Automatic route: The non-resident or
Indian company does not require prior nod
9. Answer: (b) of the RBI or government of India for FDI.
Explanation: y Government route or Approval route:
Statement 1 is not correct: The value of India- In this route, government’s approval is
Sri Lanka trade has not considerably increased mandatory. The company will have to file
in the last decade. It registered a downfall post an application through Foreign Investment
2014-15. Facilitation Portal, which facilitates single-
Statement 2 is correct: Imports of readymade window clearance. The application is then
garments from Bangladesh and export of forwarded to the respective ministry,
fabrics from India constitutes an important which will approve/reject the application
link item in India-Bangladesh trade. in consultation with the Department for
Promotion of Industry and Internal Trade
Statement 3 is not correct: In South Asia,
(DPIIT), Ministry of Commerce.
Bangladesh has been India’s largest trading
partner.
12. Answer: (d)

10. Answer: (a) Option (d) is correct: Reserve Tranche is the


component of a member country’s quota with
Explanation:
the International Monetary Fund (IMF), that
Option (a) is correct: West Texas Intermediate is, in the form of gold or foreign currency. It
is a particular grade of crude oil. This oil is is a credit system granted by the IMF to its
primarily sourced from Texas and serves as a members.
global benchmark for setting oil prices.
For any member country, out of the total quota,
25% should be paid in the form of foreign
11. Answer: (b) currency or gold. Hence, this is called a Reserve
Tranche or Gold Tranche. The remaining 75%
Foreign Direct Investment (FDI) is a type of
can be paid in domestic currencies, and it
investment where an investor based in one
is called Credit Tranche. This quota can be
country invests in another country. Generally,
accessed by a country without agreeing to IMF
FDI takes place when an investor establishes
conditionalities.
foreign business operations or acquires
foreign business assets, including establishing
ownership or controlling interest in a foreign 13. Answer: (a)
company.
Global financial crisis refers to extreme
Option (b) is correct: The Foreign Direct stress in global financial markets and banking
Investment (FDI) is a critical driver of economic systems. A fast-changing global environment
growth and is often favoured over other means demands organizations to have both financial
of external finance because FDI equity inflows stability and liquidity.

3A: BoP, CAD Currency Exchange 531


unacademy.com | Download the Unacademy app
Give your feedback here: Link
Statement 1 is correct: Financial crises concern, as the country depends on imports for
can induce significant hardships on both meeting roughly 60% of its domestic demand.
international creditors and the debtor nations, Out of these imports Palm Oil, particularly
which are forced into default. Both economic constitutes the largest amount of imports
and political considerations leave it difficult to which is primarily sourced from Malaysia
resist coming to the aid of a distressed nation and Indonesia. Despite a growth in domestic
or region. So, not depending on short-term production of vegetable oils there still remains
foreign borrowings will provide immunity to a wide gap in domestic demand and supply of
India. the same.
Statement 2 is not correct: At the time of the Various initiatives like Oil Palm Area expansion
global financial crisis, there is a high risk of under Rastriya Krishi Vikas Yojana, increasing
banks becoming inefficient in NPA recovery the MSP for oilseeds etc. are being implemented
and declaring themselves insolvent in the near by the government to boost domestic supply
future. Hence, opening up to more foreign of vegetable oils.
banks in India would be a huge risk.
Statement 3 is not correct: Capital account 16. Answer: (d)
convertibility is a feature of a nation’s financial
regime that centers on the ability to conduct External debt refers to the money borrowed
transactions of local financial assets into from a source outside the country which has
foreign financial assets freely or at market- to be paid back in the currency in which it
determined exchange rates. With the global is borrowed. It can be obtained from foreign
financial crisis, this step will prove to be a commercial banks, international financial
huge failure. institutions like IMF, World Bank, ADB etc and
from the government of foreign nations.
As per the Reserve Bank of India, the following
14. Answer: (b) are the observations about India’s external
Option (b) is correct: India is popular for its debt at the end of June 2020.
rice cultivation all over the world. India is the y Commercial borrowings remained the
largest rice exporter with a significant market largest component of external debt, with
share. In the last few years, India topped in a share of 38.1 per cent, followed by non-
the list of rice exporting countries, followed by resident deposits (23.9 per cent) and short-
Thailand, which stood second in the list. term trade credit (18.2 per cent). (Statement
1 is not correct)
15. Answer: (d) y US dollar denominated debt remained the
The most frequent imported agricultural largest component of India’s external debt,
products in India are vegetable oils, pulses, with a share of 53.9 percent at end-June
fresh fruits, cashew nuts, raw sugar, raw 2020, followed by the Indian rupee (31.6 per
cotton and spices. Together, the value of these cent), yen (5.7 per cent), SDR (4.5 per cent)
goods accounts for almost 84 percent of all and the euro (3.5 per cent). (Statement 2 is
agricultural imports. not correct)

Option (d) is correct: Amongst the mentioned Additional Information:


agricultural commodities, Vegetable Oils Normally these types of debts are in the form
constitute the highest imports in terms of of tied loans, meaning that these have to be
value in the last five years. used for a predefined purpose as determined
Vegetable Oil imports have been a cause for by a consensus of the borrower and the lender.

532 3A: BoP, CAD Currency Exchange


unacademy.com | Download the Unacademy app
Give your feedback here: Link
Government is also eligible to raise loans from
abroad. The interest rate on foreign loans related to the currency crisis. So,
is linked to LIBOR (London Interbank Offer option (c) and (d) can be eliminated.
Rate) and the actual rate will be LIBOR plus The correct option is more likely to be
applicable spread, depending upon the credit (b).
rating of the borrower.

18. Answer: (d)


17. Answer: (b) Option (d) is correct: The sliding of Indian
Currency crisis is a situation in which the value rupee or any other currency is termed as
of a domestic currency falls drastically relative Depreciation, which is the fall in the value of a
to other currencies. It may occur due to various currency in terms of its exchange rate relative
reasons such as inflation, fluctuations in to other currencies. Currency depreciation can
the financial market, increasing demand for occur due to increasing demand for imports,
imports, deficit in balance of payment account, high current account deficit, insufficient inflow
etc. of foreign direct investment, outflow of foreign
investment, poor economic growth, etc.
Statement 1 is correct: Foreign currency
earnings of India’s IT sector will lead to the In India, the exchange rate is governed by the
increase in supply of foreign currencies, and Reserve Bank of India (RBI). India follows a
this further increases the forex reserves. managed floating rate system, which means
Therefore, more supply and less demand for the exchange rate of rupee is based on the
foreign currency relative to Indian rupee helps demand and supply of rupee in the market but
in reducing the risk of currency crisis. when the rupee value becomes more volatile,
the RBI intervenes in the market to manage
Statement 2 is not correct: Currency crisis
the volatility. There are various ways to control
can be reduced by manipulating the demand
the rupee value from sliding which are curbing
and supply of currency in the foreign exchange
imports, encouraging Indian borrowers to issue
market. Government expenditure is not
rupee denominated bonds, easing norms for
likely to reduce currency risks as increase in
external commercial borrowings, etc.
government expenditure does not directly
affect the demand and supply of rupee. It may However, the expansionary monetary policy is
likely cause a rise in aggregate demand (AD). not the most likely measure to stop the sliding
This can further lead to higher growth in the of Indian rupee, as it can cause the rupee to
short-term. slide more. An expansionary Monetary Policy
will increase the money supply in the economy,
Statement 3 is correct: Remittance is the sum
which will reduce the interest rates. The fall
of money (in foreign currency), sent from abroad
in interest rates makes the domestic interest
or overseas to the home country. Remittance
rates lower than what are available in the
is the major source of cash inflow into any
foreign markets, the investors pull out their
country. With the inflow of foreign currency,
investment from the nation which can lead to
forex reserves again increased which further
the flight of foreign exchange from the country
reduced the currency crisis. India is the largest
in turn depreciating the Rupee further.
remittance receiving country in the world.

Elimination Technique
19. Answer: (a)
y Currency crisis is mostly related to
external economic events. Increasing The basis for Purchasing Power Parity (PPP) is
government expenditure is not directly the “law of one price”. It aims to determine

3A: BoP, CAD Currency Exchange 533


unacademy.com | Download the Unacademy app
Give your feedback here: Link
the adjustments needed to be made in the 21. Answer: (c)
exchange rates of two currencies to make The Fiscal Responsibility and Budget
them at par with the purchasing power of each Management (FRBM) Act was enacted in 2003
other. In other words, the expenditure on a with an aim to introduce transparency in India’s
similar commodity must be the same in both Fiscal Management System. It sets particular
currencies when accounted for the exchange Debt-to-GDP, Revenue Deficit and Fiscal Deficit
rate. targets for the Government. Subsequently,
Statement 1 is correct: PPP exchange rate is these targets were revised by the FRBM Review
the rate at which the currency of one country Committee set under N.K Singh.
needs to be converted into the currency of Statement 1 is correct: The FRBM review
another country (with which the PPP exchange committee in its 2017 report has recommended
rate is to be calculated) in order to buy a similar bringing down the debt to GDP ratio for both
amount or basket of goods and services as in Centre and States combined to 60 percent
the first country. (which comprised 40 percent on account of
Statement 2 is not correct: In terms of PPP centre and 20 percent on account of states)
dollars, India is the third largest economy in the by 2023.
world. (Fourth largest if the European Union is Statement 2 is not correct: At the time of the
also considered as a common economic space report submission domestic liabilities of the
for ranking purposes). Central Government accounted for 46.5% of
the GDP and that of States accounted for 24%
20. Answer: (a) of the GDP.

Statement 1 is correct: Edible oil imports are Statement 3 is correct: Article 293 (3) of the
a huge cause of concern due to their rising Constitution of India states that, “A State may
import bill. A high domestic demand for edible not without the consent of the Government of
oils in the country, makes it dependent upon India raise any loan if there is still outstanding
imports for meeting around 70 percent of its any part of a loan which has been made to
domestic demand. Hence, the quantity of the State by the Government of India or by
imported edible oils is more than the domestic its predecessor Government, or in respect
production of edible oils in the last five years. of which a guarantee has been given by the
Government of India or by its predecessor
Statement 2 is not correct: The government
Government”.
levies 35% and 45% import duty on all crude
and refined edible oils respectively. While in The Act allows for an “escape clause” under
2018 the import duty on Olive oil was 40%. which in special circumstances like calamities
the government can flexibly follow the Fiscal
Additional Information:
Deficit targets, allowing it for more room and
Around 62 percent of edible oil imports space to tackle the situation. This terminology
are accounted for by Palm Oil due to a high of escape clause was innovated by the N.K
domestic demand. With effect from January Singh Panel. For the Fiscal Year 2019-20, the
2020, import policy of refined Palm Oil is total Central Government debt stood at 48.6%
amended from ‘free’ to ‘Restricted’ category, of the GDP.
meaning that the government has imposed
certain trade barriers which the importer has
to overcome before importing the product into 22. Answer: (d)
the country. Statement 1 is not correct: Tax revenue as a
percentage of GDP has not steadily increased
in the last decade. There has been a rise and

534 3A: BoP, CAD Currency Exchange


unacademy.com | Download the Unacademy app
Give your feedback here: Link
fall in it without following a fixed pattern.

Statement 2 is not correct: Fiscal deficit as a percentage of GDP has not steadily increased in the
last decade. There has been a rise and fall in it with no steady pattern of rising or falling.

Additional Information: government. It is an indication of the total


Tax Revenue: Tax Revenue forms part of the borrowings needed by the government.
Receipt Budget, which in turn is a part of
the Annual Financial Statement of the Union 23. Answer: (d)
Budget. It gives a detailed report on revenue
collected from different items like corporation The Union Budget is divided into two parts,
tax, income tax, wealth tax, customs, union namely the Revenue Budget and the Capital
excise, service, taxes on Union Territories like Budget. Wherein the Revenue Budget consists
land revenue, stamp, etc. of incomes and expenditures related to the day
to day functioning of the government, Capital
Fiscal Deficit: It is the difference between Budget consists of incomes and expenditures
total revenue and total expenditure of the

3A: BoP, CAD Currency Exchange 535


unacademy.com | Download the Unacademy app
Give your feedback here: Link
which are made on account of changes in the curtail the Government expenditure and
Assets or Liabilities of the government. help bridging its deficit gap. (Statement 1
y Expenditure made on the acquisition of and statement 3 are correct)
assets like roads, buildings, machinery, y Revenue expenditure is that part of
etc., form a part of the Capital Expenditure Government expenditure that does not
of the Government of India. (Statement 1 is result in the creation of assets. Payment
correct) of salaries, wages, pensions, subsidies,
y Loans received from foreign governments and interest fall in the category of revenue
are a part of Capital Receipts of the expenditure.
Government of India as they come with a y While introducing new welfare schemes
future repayment liability. (Statement 2 is will increase the Government expenditure
correct) and reducing import duties will reduce
y Loans and Advances granted by the Government revenue, which in effect these
Government of India to the States/the UTs steps will lead to a widening budget deficit.
form a part of its capital expenditure as it (Statement 2 and statement 4 are not
adds up to the Assets of the Government of correct)
India, which are expected to provide future Additional Information:
benefits when these loans are repaid to it. There are three kinds of deficits in the Budget
(Statement 3 is correct) commonly measured in India:
y Fiscal Deficit: It is the excess of Government
24. Answer: (c) expenditure over the non-debt creating
A deficit budget or budgetary deficit is the receipts of the Government.
difference between all receipts and expenses y Revenue Deficit: It is the excess of
in both revenue and capital account of the Government revenue expenditure over its
Government. Budgetary deficit is the sum of revenue receipts.
revenue account deficit and capital account y Primary Deficit: It is the difference between
deficit. If revenue expenses of the Government the Fiscal Deficit and Interest payments on
exceed revenue receipts, it results in a account of previous borrowings.
revenue account deficit. Similarly, if the capital
disbursements of the Government exceed
capital receipts, it leads to a capital account 25. Answer: (c)
deficit. Budgetary deficit is usually expressed The Sovereign Gold Bond Scheme and the Gold
as a percentage of GDP. Monetization Scheme was launched in 2015 to
The Government of India has been experiencing reduce India’s dependence upon the imports
a persistent deficit budget year after year due of gold.
to sluggish economic growth, higher interest y Through the Gold Monetization Scheme,
payment on loans, lower tax collections, the gold held by households can be
increased investment, etc. The effects of the deposited in an account with banks, just
budget deficit are manifold, and it may lead like a normal Savings Bank A/c is operated
to the increased national debt, higher interest to bring them into the fold of the Indian
rates, inflation, etc. economy. The difference being that the
y One of the most suitable ways which the amount credited in the account would
Government can employ to tackle this equal the value of the gold brought in. On
situation is by reducing revenue expenditure this stated amount of the gold, households
and rationalizing subsidies. This will help will earn an interest and the physical gold

536 3A: BoP, CAD Currency Exchange


unacademy.com | Download the Unacademy app
Give your feedback here: Link
thus accumulated can be sold to the Gems 27. Answer: (d)
and Jewellery sector of the country as their Option (d) is correct: ‘Import Cover’ is a term
raw material, so, import dependence can used to describe the number of months over
be reduced and the gold laying idle can be which a country can pay for its’ imports from
used for generating value in the economy. the International Reserves of the country.
(Statement 1 is correct) Import cover of reserves is a traditional trade-
y These schemes are not concerned with based indicator of reserve adequacy. It is
increasing the FDI in the gold and jewellery defined in terms of the number of months of
sector; instead, their primary motive is to import equivalent to reserves. The excess of
reduce the country’s dependence on gold inflow of Foreign Exchange over the outflow of
imports. (Statement 2 is not correct) foreign exchange in a country gets credited to
y They seek to reduce India’s dependence on its’ international reserves account. This reserve
gold imports as through gold monetisation can be used as a buffer to cushion against the
scheme the demand of the industry can be uncertainties of future inflows.
met by the idle gold in the country and the Foreign exchange reserves of India surged to
speculative demand for investment in gold, $605 billion in the week to June 4, 2021, as the
can be met by an investment by people RBI mopped up dollars flowing into the nation’s
into the Sovereign Gold Bonds instead of booming stock market as well as via foreign direct
keeping physical gold with themselves investments. The pile is the world’s fifth-biggest
thus reducing the demand for the same. after China, Japan, Switzerland, and Russia, and
(Statement 3 is correct) is enough to cover 15 months of imports.

26. Answer: (d) 28. Answer: (c)


Option (d) is correct: In 2016, the International International Liquidity primarily connotes the
Monetary Fund (IMF) added the Chinese resources which are at the disposal of the
Renminbi (RMB) to the basket of currencies countries in order to tackle their Balance of
that make up the Special Drawing Right (SDR). Payments deficits.
y The SDR was created as a supplementary Option (c) is correct: Modern global trading
international reserve asset in the context systems primarily depend upon dollars and
of the Bretton Woods fixed exchange other global currencies (hard currencies) like
rate system. The collapse of the Bretton Yen, Euro, Pound, etc. These currencies account
Woods system in 1973 and the shift of for the major proportions of every country’s
major currencies to floating exchange rate Balance of Payments today. Hence, the non-
regimes lessened the reliance on the SDR availability of them will lead to a problem of
as a global reserve asset. Nonetheless, international liquidity.
SDR allocations can play a role in providing Additional Information:
liquidity and supplementing member
countries’ official reserves, as was the case y A hard currency refers to a currency that
amid the global financial crisis. is generally issued by developed countries,
globally traded, and seen as politically
y The SDR is neither a currency nor a claim and economically stable. International
on the IMF. It serves as the unit of account investors put their confidence and trust
of the IMF and some other international in hard currencies because they will not
organizations. dramatically depreciate or appreciate
(fluctuate in relative value to other
currencies).

3A: BoP, CAD Currency Exchange 537


unacademy.com | Download the Unacademy app
Give your feedback here: Link
29. Answer: (c) expenditure of the government and
Option (c) is correct: Convertibility of rupee contribute to increasing the deficit. (Option
means that rupee can be converted into 2 and 4 are not correct)
other currencies in a free fashion for various Additional Information:
purposes and vice versa. y Fiscal Deficit: It is the excess of Government
Convertibility of rupee can be understood Expenditure over the Non-Debt Creating
from two broad aspects, that is Current receipts of the Government.
Account Convertibility and Capital Account y Revenue Deficit: It is the excess of
Convertibility. Government Revenue Expenditure over its’
y Current Account Convertibility implies that Revenue Receipts.
rupee can be converted freely into other y Primary Deficit: It is the difference between
currencies for trade purposes, basically for the Fiscal Deficit and Interest payments on
import and export purposes. India allows account of previous borrowings.
for free Current Account Convertibility.
y Capital Account Convertibility implies that
rupee can be converted freely into other 31. Answer: (a)
currencies for capital transaction purposes. The Finance Commission is a Constitutional
India does not freely allow Capital Account body formed every five years to give
Convertibility. suggestions on center-state financial relations.
Each Finance Commission is required to make
recommendations on:
30. Answer: (a)
y Sharing of central taxes with states,
Budget deficit is a situation in which the distribution of central grants to states,
expenditures of the government exceed the measures to improve the finances of states
receipts. It is a situation when the government to supplement the resources of panchayats
spends more than it earns. and municipalities, and any other matter
A Budget deficit leads to debts, if the deficit referred to it.
is not managed properly, which further leads y The 14th Finance Commission considerably
to more deficits and higher interest rates. It increased the devolution of taxes from the
could also lead to high levels of inflation, which Centre to states from 32 percent to 42
further leads to recession and inflationary percent. The Commission had recommended
monetary policies. that tax devolution should be the primary
y By reducing the revenue expenditure, that source of transfer of funds to states. This
is, the expense incurred in the day-to-day would increase the flow of unconditional
function of the government like salaries of transfers and give states more flexibility in
employees etc.; revenue expenditure can their spending. (Statement 1 is correct)
be reduced. (Option 1 is correct) y It has not made any recommendations
y Rationalizing subsidies that are reassessing concerning sector-specific grants.
and controlling the outflow of government (Statement 2 is not correct)
expenditure on account of various subsidies
like Fertilizer, LPG, etc. The government can
reduce its expenditure and help in bridging 32. Answer: (c)
the deficit. (Option 3 is correct) Option (c) is correct: The government’s
y Launching new welfare schemes and expenditure can be divided into two parts —
Industrial Expansion will need additional plan and non-plan. Plan expenditure is that

538 3A: BoP, CAD Currency Exchange


unacademy.com | Download the Unacademy app
Give your feedback here: Link
component of government expenses which of Payments, the Current Account constitutes
helps increase the productive capacity in Balance of Trade and Balance of Invisibles.
the economy. It includes outlays for different y Balance of Trade is the difference between
sectors, such as rural development and a country’s Imports and Exports. The
education. Unlike non-plan expenditure, which excess of exports over Imports is referred
is mostly fixed and obligatory in nature. to as Net Exports.
y The practice of classifying expenditure y Balance of Invisibles is the difference
budget as Plan and Non-Plan in the Budget between a country’s Service Imports and
documents was introduced from the First Service Exports.
Five Year Plan in 1951, with the Planning
Commission deciding the estimates of plan
allocation. 34. Answer: (a)
y From 2017-18 onwards, the distinction Option (a) is correct: The Balance of Payments
between Plan and Non-Plan expenditure is a record of all transactions conducted
has been done away with. Earlier, Non- between a country and the rest of the world.
plan expenditure constituted the dominant It keeps a systematic record of all imports and
portion of budget expenditure. These exports of goods/services/capital and other
expenses are spread across defence, factors and transfer payments generally during
interest payments for debt, subsidies the span of a year.
(especially for food and fertilizers) and The Balance of Payments in India consists of
salaries and pensions of employees in three sub-components:
various government services.
y Current Account: It covers all transactions
(other than those in financial items) that
33. Answer: (c) involve economic values and occur between
The Balance of Payments (BoP) is the record of resident and non-resident entities. It
all the transactions between a country and the classifies transactions in goods and
rest of the world. As per the Reserve Bank of services, income, and current transfers.
India (RBI), BoP shows: y Capital Account: The major components of
y The transaction in goods, services and the capital account are capital transfers
income between an economy and the rest and acquisition/disposal of non-produced,
of the world, nonfinancial assets.

y Changes of ownership and other changes y Financial Account: The financial account
in that economy’s monetary gold, special records an economy’s transaction in
drawing rights (SDRs), and financial claims external financial assets and liabilities.
on and liabilities to the rest of the world,
and 35. Answer: (b)
y Unrequited transfers (a remittance that an Capital Account is a sub-component under
immigrant makes to his family or friends in the Balance of Payments. It keeps a record
the home country). of transactions of international nature which
Option (c) is correct: The Current Account are concerned with altering either an Asset or
keeps the record of transactions of goods and Liability for the domestic country.
services between the country and the rest of Option (b) is correct: Foreign Loans, Foreign
the world and is a subcomponent under the Direct Investment and Portfolio Investment
Balance of Payments. With reference to Balance form a component of the Capital Account as

3A: BoP, CAD Currency Exchange 539


unacademy.com | Download the Unacademy app
Give your feedback here: Link
they are adding up to the existing liabilities of euro, or yen). To maintain its exchange rate, the
the country as they warrant a future repayment government will buy and sell its own currency
liability. against the currency to which it is pegged.

Elimination Technique y The World Bank is a vital source of financial


and technical assistance to developing
y Remittance or private remittance is a countries around the world. It has no
part of the current account. So, option relation with the determination of the price
(a), (c), and (d) can be easily eliminated of currency in the international market.
and we can mark option (b) as the
correct answer. Statement 2 is correct: The price of any
currency in the international market can also
be determined at floating rate. It is the market
36. Answer: (b) determined rate, i.e., the value of currency is
Option (b) is correct: Foreign exchange reserves determined based on demand and supply. If the
are the reserves held by a country that consist demand for the currency is high, the value will
of primarily foreign currencies and other increase. If demand is low, this will drive that
reserve assets which can be used to balance currency price lower. The currencies of most
the international payments of a country. of the world’s major economies were allowed
to float freely following the collapse of the
y Forex reserves in the Indian context consist
Bretton Woods system between 1968 and 1973.
of foreign currencies, holdings of gold by
Therefore, most exchange rates are not set but
the RBI and Special Drawing Rights with
are determined by on-going trading activity
the International Monetary Fund which are
in the world’s currency markets. Demand
foreign exchange reserve assets created
for goods/ services provided by the country
and maintained by the RBI. India’s Foreign
concerned directly impacts the demand or
reserves also constitute a Reverse Tranche
supply for that country’s currency impacting
Position with the IMF. This is that required
the exchange rates.
quota of currency that each IMF member
has to provide. These funds can be accessed Statement 3 is correct: Similarly, stability of the
at any time by that member country. government of the concerned country provides
a greater trust and acceptability of that nation’s
currency which is not the case with the currency
37. Answer: (b) of a country with unstable government. Hence
In a globalized world interconnectedness the stability of the government leads to greater
between countries has increased to global acceptability and thus demand for that
unprecedented levels. Greater exchange of currency.
goods and services between nations requires Statement 4 is not correct: Countries’ economic
a robust exchange rate determination system. potential has no role in deciding the price of its
Today the exchange rates of currencies are currency in the international market.
market determined and are largely determined
by the forces of demand and exchange amongst
other factors. 38. Answer: (d)

Statement 1 is not correct: A fixed exchange Foreign direct investment (FDI) is when a
rate is one of the systems that determines the company takes controlling ownership in a
price or the value of currency. A fixed or pegged business entity in another country. With FDI,
rate is determined by the government through foreign companies are directly involved with
its central bank. The rate is set against another day-to-day operations in the other country. It
major world currency (such as the U.S. dollar,

540 3A: BoP, CAD Currency Exchange


unacademy.com | Download the Unacademy app
Give your feedback here: Link
visit by foreign nationals to witness the XIX
means that companies not just bring money Commonwealth Games in India amounted to
but also bring technology, new skill sets, etc. exports.
Option (d) is correct: Foreign direct investments y The hosting of global events brings many
can be made in a variety of ways, including foreign nationals into the country. They
the opening of a subsidiary or associate consume varying amounts of goods and
company in a foreign country (or opening of services in the domestic territory of India.
subsidiaries by any foreign company in India),
y To pay for these goods and services in
holding majority equity in foreign companies
Rupees which they exchange an equivalent
(or majority foreign equity holding in Indian
amount of foreign currency in the country.
companies), acquiring a controlling interest in
an existing foreign company, or by means of a y This amounts to the export of goods and
merger or joint venture with a foreign company. services from India to foreign nationals,
Moreover, companies exclusively financed by however in all possibilities the consumption
foreign companies would also include foreign of these goods and services happens within
direct investment in India. the territory of India.
Additional Information:
y A subsidiary is a company that belongs to 40. Answer: (d)
another company, which is usually referred Disinvestment is the process wherein the
to as the holding company. Opening a government sells its stake in the Public Sector
subsidiary of a domestic firm in a foreign Undertakings.
country, leads to FDI inflows.
Statement 1 is not correct: The revenue earned
y With the emergence of the service sector from disinvestment is not intended to be used
at the helm of receiving FDI the distinction for paying back external debt, instead it is used
between FDI and FPI has become difficult as for creating other social value creating assets
FDI in the service sector is generally made like highways etc.
through the route of acquiring securities
Statement 2 is not correct: Disinvestment may
in a particular concern. To differentiate
not necessarily mean that the government is
between FDI and FPI, a rule has been
letting go the management control in CPSEs,
devised wherein if a foreign fund buys
instead the Government may choose to sell
less than 10 percent stake in a company
only a minority stake and keep managerial
it is termed as FPI and if it is more than 10
control in its hands.
percent it would be considered as FDI.
Additional Information:
Elimination Technique
y The government undertakes disinvestment
y Portfolio investment is related to to reduce the fiscal burden on the
ownership of stock, or bond of any exchequer or to raise money for meeting
entity. It is altogether not a part of specific needs, such as to bridge the
foreign direct investment. Option (a), revenue shortfall from other regular
and (b) can be eliminated. Option (d) is sources. In some cases, disinvestment may
more likely to be the answer. be done to privatise assets. However, not
all disinvestment is privatisation. Some of
the benefits of disinvestment are that it
39. Answer: (a)
can be helpful in the long-term growth of
Option (a) is correct: In terms of economy, the the country; it allows the government and

3A: BoP, CAD Currency Exchange 541


unacademy.com | Download the Unacademy app
Give your feedback here: Link
even the company to reduce debt. with the intent of establishing long lasting
y Disinvestment allows a larger share of PSU business interests in that nation with the
ownership in the open market, which in help of the targeted business.
turn allows for the development of a strong y Foreign Institutional Investors are foreign
capital market in India. investors which invest in another country
y The Department of Disinvestment under the through various income generating assets
Union Ministry of Finance which handles like Stocks, Bonds etc. Their primary
all disinvestment-related works for the interests are short term to earn higher
government. The department was later returns on their investments as compared
renamed the Department of Investment to their own domestic territories. They are
and Public Asset Management. more volatile in nature than FDI due to their
short term and limited intentions.
y Disinvestment targets are set under each
Union Budget, and every year the targets Option (b) is correct: FII helps in increasing
change. the general capital availability in the country
as they invest large sums of money into varied
asset classes across the domestic markets. On
41. Answer: (b) the contrary, FDI is targeted only at a particular
Option (b) is correct: Fiscal Stimulus is the sector and at times only to a particular business
process of infusing money into an economy entity.
that is going through a crisis. It is an intense
affirmative action of the government to boost
43. Answer: (d)
economic activity in the country.
Current Account Deficit (CAD) is the shortfall
y During the economic downturn, the
between the money received by selling products
government stepped in with a set of
to other countries and the money spent to buy
measures to reinvigorate the economy. Most
goods and services from other nations. If the
often, these involve increased spending by
value of goods and services imported exceeds
the government to increase employment
the value of those exported, the country is said
and general demand in the economy, which
to be in a deficit, and the difference in the two
acts as a signal for companies to increase
values is CAD.
production, which in turn will lead to more
hiring and so on through the virtuous Statement 1 is correct: Devaluing the domestic
economic cycle. currency will make the domestic currency
cheaper in comparison to foreign currencies
y Moreover, the government also reduces
which will make Indian goods cheaper in
various types of taxes to increase the
comparison to foreign goods, hence increasing
money supply in the market. It is generally
the exports of goods and services from the
not limited to or specific to a particular
country.
sector.
Statement 2 is not correct: Reduction in
export subsidy will make exports from India
42. Answer: (b) more expensive thus reducing exports and it
Foreign Direct Investment (FDI) and Foreign will lead to an increase in the Current Account
Institutional Investment (FII) are both channels deficit of the country.
through which foreign capital flows into a country. Statement 3 is correct: Attracting greater
y Foreign Direct Investment is an investment Foreign Direct Investment (FDI) and Foreign
by a foreign entity into a domestic business Institutional Investment (FII) will increase
the inflow of foreign exchange in the country.

542 3A: BoP, CAD Currency Exchange


unacademy.com | Download the Unacademy app
Give your feedback here: Link
Both these steps will help in bridging Current Chairman appointed by the Government of
Account Deficit in the country. India representing different sections of the Tea
industry.

44. Answer: (d) Statement 3 is not correct: The Tea Board’s


Head Office is situated in Kolkata and there
Explanation: are two Zonal offices-one each in the North
Statement 1 is correct and Statement 2 is not Eastern Region at Jorhat in Assam and in the
correct: Tea Board of India is a statutory body Southern Region at Coonoor in Tamil Nadu.
functioning under the Ministry of Commerce Besides, there are 18 regional offices spread
and Industry (Government of India). It was set over in all the major tea growing states and
up under the Tea Act of 1953. It is an apex body four metros.
which looks after the overall development Statement 4 is correct: The Tea Board of
of the tea industry. The Board consists of 32 India has overseas offices in Moscow, Dubai,
Members, including Chairman and Deputy Hamburg, London and New York.

3A: BoP, CAD Currency Exchange 543


unacademy.com | Download the Unacademy app
Give your feedback here: Link
3B: WTO, IMF & other Intl. Orgs &
10 Agreeements
1. “Rapid Financing Instrument” and “Rapid (c) 1 and 3 only
Credit Facility” are related to the provisions (d) 1, 2 and 3
of lending by which one of the following?
(2022)
(a) Asian Development Bank 4. In which one of the following groups are
(b) International Monetary Fund all the four countries members of G20 ?
[2020]
(c) United Nations Environment Programme (a) Argentina, Mexico, South Africa and
Finance Initiative Turkey
(d) World Bank (b) Australia, Canada, Malaysia and New
Zealand
2. Find correct statement(s) about AIIB: (c) Brazil, Saudi Arabia and Vietnam
(UPSC-Prelims-2020)
1. AIIB has more than 80 member nations. (d) Indonesia, Japan, Singapore and South
Korea
2. India is the largest shareholder in AIIB.
3. AIIB does not have any members from
outside Asia. 5. Consider the following countries : [2018]
1. Australia
Codes:
2. Canada
(a) 1 only
3. China
(b) 2 and 3 only
4. India
(c) 1 and 3 only
5. Japan
(d) 1, 2 and 3 only
6. USA
Which of the above are among the 'free-
3. With reference to Trade-Related trade partners' of ASEAN ?
Investment Measures (TRIMS), which of
the following statements is/are correct? (a) 1, 2, 4 and 5
(2020) (b) 3, 4, 5 and 6
1. Quantitative restrictions on imports by (c) 1, 3, 4 and 5
foreign investors are prohibited.
(d) 2, 3, 4 and 6
2. They apply to investment measures
related to trade in both goods and
6. India enacted the Geographical Indications
services.
of Goods (Registration and Protection) Act,
3. They are not concerned with the 1999 in order to comply with the obligations
regulation of foreign investment. to (2018)
Select the correct answer using the code (a) ILO
given below: (b) IMF
(a) 1 and 2 only (c) UNCTAD
(b) 2 only (d) WTO

544 3B: WTO, IMF & other Intl. Orgs & Agreeements
unacademy.com | Download the Unacademy app
Give your feedback here: Link
7. The term 'Digital Single Market Strategy' Which of the statements given above is/are
seen in the news refers to [2017] correct?
(a) ASEAN (a) 1 only
(b) BRICS (b) 2 only
(c) EU (c) Both 1 and 2
(d) G20 (d) Neither 1 nor 2

8. Consider the following statements: [2017] 11.


‘Global Financial Stability Report’ is
1. India has ratified the Trade Facilitation prepared by the [2016]
Agreement (TFA) of WTO. (a) European Central Bank
2. TFA is a part of WTO's Bali Ministerial (b) International Monetary Fund
Package of 2013.
(c) International Bank for Reconstruction
3. TFA came into force in January 2016. and Development
Which of the statements given above is/are (d) Organization for Economic Cooperation
correct? and Development
(a) 1 and 2 only
(b) 1 and 3 only 12. In the context of which of the following do
(c) 2 and 3 only you sometimes find the terms amber box,
blue box, and green box in the news? (2016)
(d) 1, 2 and 3
(a) WTO affairs
(b) SAARC affairs
9. Find correct statement(s): (Asked in UPSC-
Pre-2016) (c) UNFCCC affairs
1. New Development Bank has been set (d) India-EU negotiations on FTA
up by APEC.
2. The headquarters of New Development 13.
The terms 'Agreement on Agriculture',
Bank is in Shanghai. 'Agreement on the Application of Sanitary
Answer Codes: and Phytosanitary Measures' and 'Peace
Clause' appear in the news frequently in
(a) 1 only
the context of the affairs of the [2015]
(b) 2 only (a) Food and Agriculture Organization
(c) Both 1 and 2 (b) United Nations Framework Conference
(d) Neither 1 nor 2 on Climate Change
(c) World Trade Organisation
10.
With reference to the International (d) United Nations Environment Programme
Monetary and Financial Committee (IMFC),
consider the following statements: (2016)
14.
Which of the following organizations
1. IMFC discusses matters of concern
brings out the publication known as ‘World
affecting the global economy and
Economic Outlook’? (2014)
advises the International Monetary Fund
(a) The International Monetary Fund
(IMF) on the direction of its work.
(b)
The United Nations Development
2. The World Bank participates as an
Programme
observer in IMFC’s meetings.

3B: WTO, IMF & other Intl. Orgs & Agreeements 545
unacademy.com | Download the Unacademy app
Give your feedback here: Link
(c) The World Economic Forum (d) Neither 1 nor 2
(d) The World Bank
18.
The Global Competitiveness Report is
15. Regarding the International Monetary Fund, published by the
which one of the following statements is (a) International Monetary Fund
correct? (2011) (b) United Nations Conference on Trade
(a) It can grant loans to any country. and Development
(b) It can grant loans to only developed (c) World Economic Forum
countries. (d) World Bank
(c) It grants loans to only member countries.
(d) It can grant loans to the central bank of 19. Consider the following countries:
a country. 1. Armenia
2. Azerbaijan
16.
‘Broad-based Trade and Investment 3. Croatia
Agreement (BTIA)’ is sometimes seen in
the news in the context of negotiations 4. Romania
held between India and 5. Uzbekistan
(a) European Union Which of the above are members of the
(b) Gulf Cooperation Council Organization of Turkic States?
(c) Organization for Economic Cooperation (a) 1, 2 and 4
and Development (b) 1 and 3
(d) Shanghai Cooperation Organization (c) 2 and 5
(d) 3, 4 and 5

17. With reference to the "G20 Common


20. Consider the following:
Framework", consider the following
1. Asian Infrastructure Investment Bank
statements:
1. It is an initiative endorsed by the G20 2. Missile Technology Control Regime
together with the Paris Club. 3. Shanghai Cooperation Organisation
2. It is an initiative to support Low Income India is a member of which of the above?
Countries with unsustainable debt.
(a) 1 and 2 only
Which of the statements given above is/are
(b) 3 only
correct ?
(c) 2 and 3 only
(a) 1 only
(d) 1, 2 and 3
(b) 2 only
(c) Both 1 and 2

546 3B: WTO, IMF & other Intl. Orgs & Agreeements
unacademy.com | Download the Unacademy app
Give your feedback here: Link
3B: WTO, IMF & other Intl. Orgs &
10 Agreeements_Explanation
1. Answer: (b) related to trade in goods that restrict or distort
Option (b) is correct: The Rapid Financing trade. The TRIMS Agreement prohibits certain
Instrument (RFI) provides rapid financial measures that violate the national treatment
assistance, which is available to all member and quantitative restrictions requirements of
countries facing an urgent balance of payments the General Agreement on Tariffs and Trade
need. The RFI was created as part of a broader (GATT).
reform to make the IMF’s financial support y It prohibits the imposition of Quantitative
more flexible to address the diverse needs of restrictions on imports by foreign investors.
member countries. (Statement 1 is correct)
The Rapid Credit Facility (RCF) provides y TRIMS is concerned with trade in goods.
rapid concessional financial assistance to It does not apply to trade in services.
low-income countries (LICs) facing an urgent (Statement 2 is not correct)
balance of payments (BoP) need with no ex post y TRIMS is largely concerned with the
conditionality where a full-fledged economic treatment aspect of foreign entities in
program is neither necessary nor feasible. a domestic territory. It does not concern
the regulation of foreign investment.
2. Answer: (a) (Statement 3 is correct)

Explanation: Elimination Technique


Statement 1 is correct: Asian Infrastructure y TRIPS is concerned with goods only
Investment Bank (AIIB) is a multilateral and not services. Therefore, option (a),
development bank focused on developing Asia. (b), and (d) can be eliminated. Option
AIIB with members from all over the world, focus (c) is the correct answer.
on investments in infrastructure and other
productive sectors seek to foster sustainable
economic development, create wealth and
improve infrastructure connectivity. Currently, 4. Answer: (a)
it has 103 approved member countries. Explanation:
Statement 2 is not correct: China is the largest Option (a) is correct: The G20 or Group
shareholder of AIIB followed by India, Russia, of Twenty is an intergovernmental forum
and Germany. comprising 19 countries and the European
Statement 3 is not correct: There are many Union (EU). It works to address major issues
non-regional members in the AIIB. They are related to the global economy, such as
Australia,New Zealand, Qatar, Canada, etc. international financial stability, climate change
mitigation, and sustainable development. The
member countries of G20 are - Germany, Saudi
3. Answer: (c) Arabia, Argentina, Australia, Brazil, Canada,
Under the Agreement on Trade-Related China, South Korea, United States, France,
Investment Measures of the World Trade India, Indonesia, Italy, Japan, Mexico, United
Organization (WTO), commonly known as the Kingdom, Russia, South Africa, Turkey, and the
TRIMS Agreement, WTO members have agreed European Union.
not to apply certain investment measures

3B: WTO, IMF & other Intl. Orgs & Agreeements_Explanation 547
unacademy.com | Download the Unacademy app
Give your feedback here: Link
5. Answer: (c) 7. Answer: (c)
Explanation: Explanation:
The Association of Southeast Asian Nations Option (c) is correct: The Digital Single Market
(ASEAN) was established in August 1967 in Strategy was adopted by the European Union
Bangkok, Thailand, with the signing of the in 2015. This strategy is based on three broad
ASEAN Declaration (Bangkok Declaration). pillars namely: Improving access to digital
goods and services, an environment where
Currently, it has 10 member nations which are:
digital networks and services can prosper and
Brunei, Cambodia, Indonesia, Laos, Malaysia,
Digital as a driver for growth.
Myanmar, the Philippines, Singapore, Thailand,
and Vietnam.
ASEAN has a total of 6 free trade partners 8. Answer: (a)
namely: Australia, New Zealand, South Korea, Explanation:
Japan, India, China. Therefore, from the above-
The Trade Facilitation Agreement (TFA) of WTO
mentioned countries Australia, China, India and
attempts to create an international framework
Japan are the free-trade partners of ASEAN.
for reducing trade costs. It has provisions
(Option c is correct)
which expedite movement of goods, clearance
Additional Information: of goods and release of goods by simplifying
Recently ASEAN and its 5 free trade partners customs rules and regulations amongst the
(excluding India) have entered into a regional countries.
trading agreement called the Regional Statement 1 is correct: India ratified the Trade
Comprehensive Economic Partnership (RCEP). Facilitation Agreement of WTO in 2016.
The purpose of RCEP was to make it easier
Statement 2 is correct: TFA was an outcome of
for products and services of each of these
the WTO’s Bali Ministerial.
countries to be available across this region.
India has however chosen to stay out of RCEP. Statement 3 is not correct: It came into force
in February 2017

6. Answer: (d)
9. Answer: (b)
Option (d) is correct: A geographical indication
(GI) is a sign used on products that have a specific Explanation:
geographical origin and possess qualities or a Statement 1 is not correct: During the sixth
reputation that are due to that origin. India BRICS Summit in Fortaleza (2014), the leaders
enacted the Geographical Indications of Goods signed the Agreement establishing the New
(Registration and Protection) Act, 1999 in order Development Bank (NDB). In the Fortaleza
to comply with the obligations to the World Declaration, the leaders stressed that the NDB
Trade Organization. will strengthen cooperation among BRICS and
Trade-Related Aspects of Intellectual Property will supplement the efforts of multilateral
Rights (TRIPS) Agreement was signed in and regional financial institutions for global
1994, forming a part of the larger Marrakesh development, thus contributing to collective
Agreement forming the WTO. Geographical commitments for achieving the goal of strong,
Indications have been understood to be sustainable and balanced growth.
Intellectual Property Rights under the TRIPS Statement 2 is correct: The headquarters of
Agreement. the New Development Bank shall be located in
Saffron of Jammu & Kashmir is the recent item Shanghai, China.
which has received GI tag in India.

548 3B: WTO, IMF & other Intl. Orgs & Agreeements_Explanation
unacademy.com | Download the Unacademy app
Give your feedback here: Link
10. Answer: (c) governments to their Agricultural Sector under
three heads, namely Amber Box, Blue Box and
The International Monetary and Financial
Green Box Subsidies.
Committee (IMFC) is a committee of the
International Monetary Fund (IMF). y Amber Box: These are the subsidies which
are trade-distorting. These subsidies need
y The IMFC advises and reports to the IMF
to be eliminated subject to a de-minimis
Board of Governors on the supervision
level, which is the amount of support that
and management of the international
is permitted under the amber box. This
monetary and financial system, including
level is at 5% of agricultural production
on responses to unfolding events that may
for the developed countries and 10% of
disrupt the system. In effect, IMFC is the
agricultural production for the developing
key to providing advice and direction to the
countries.
work of the IMF. (Statement 1 is correct)
y Blue Box: These are the production
y The IMFC has 24 members amongst
limitation subsidies. There are no limits on
central bank governors, Ministers, or
the amount of subsidy that can be provided
other comparable ranks. The World Bank
under this head.
participates as an observer in the IMFC’s
Meetings. (Statement 2 is correct) y Green Box: These are the non-trade
distorting subsidies. Programmes under
this head are not specifically targeted
11. Answer: (b) at a particular product; instead they are
Explanation: general protection and developmental
Option (b) is correct: The Global Financial programmes. These subsidies are allowed
Stability report is prepared by the International without any limit.
Monetary Fund (IMF). The report provides an
assessment of the global financial system 13. Answer: (c)
and markets and addresses emerging market
Explanation:
financing in a global context. It focuses on
current market conditions, highlighting Option (c) is correct: The Agreement on the
systemic issues that could pose a risk to Application of Sanitary and Phytosanitary
financial stability and sustained market access Measures (the “SPS Agreement”) entered into
by emerging market borrowers. force with the establishment of the World Trade
Organization on 1 January 1995. It concerns the
The IMF also publishes other report namely,
application of food safety and animal and plant
the ‘World Economic Outlook’ report and
health regulations.
Fiscal Monitor.
The WTO Agreement on Agriculture, which
came into force in 1995, represents a significant
12. Answer: (a) step towards reforming agricultural trade
Option (a) is correct: Amber Box, Blue Box, and making it fairer and more competitive.
etc., are agricultural subsidies of the World Article 13 (“due restraint”) of the Agriculture
Trade Organization (WTO). In WTO terminology, Agreement protects countries using subsidies
subsidies, in general, are identified by “Boxes”. which comply with the agreement from being
WTO’s Agreement on Agriculture, which seeks challenged under other WTO agreements.
to eliminate the trade-distorting subsidies Without this “peace clause”, countries would
in the context of global trade in agricultural have greater freedom to take action against
products, divides the subsidies provided by each others’ subsidies, under the Subsidies
and Countervailing Measures Agreement and

3B: WTO, IMF & other Intl. Orgs & Agreeements_Explanation 549
unacademy.com | Download the Unacademy app
Give your feedback here: Link
related provisions. Competitiveness Report is an annual report
published by the World Economic Forum.

14. Answer: (a)


Explanation: 17. Answer: (c)

Option (a) is correct: Negotiations started Explanation:


between India and the European Union on a Option (c) is correct: Organization of Turkic
Broad-based Trade and Investment Agreement States (then called the Cooperation Council of
in 2007. India and the EU expect to promote Turkic Speaking States – Turkic Council) was
bilateral trade by removing barriers to trade established in 2009 as an intergovernmental
in goods and services and investment across organization, with the overarching aim of
all sectors of the economy. Both parties promoting comprehensive cooperation among
believe that a comprehensive and ambitious Turkic States.
agreement that is consistent with WTO rules Its four founding member states are Azerbaijan,
and principles would open new markets and Kazakhstan, Kyrgyzstan and Turkey. During
would expand opportunities for Indian and EU the 7thSummit held in Baku in October 2019,
businesses. Uzbekistan joined as a full member.

15. Answer: (c) 18. xAnswer: (d)


Explanation: Explanation:
To address the problem of unsustainable Statement 1 is correct: The Asian Infrastructure
debt levels, the G20 reached agreement in Investment Bank (AIIB) is a multilateral
November 2020 on a Common Framework for development bank which began its operation
Debt Treatments. in Beijing in in2016. India is one of the founding
Statement 1 is correct: The Common Framework members of the AIIB and is the second largest
for debt treatment beyond the DSSI (Common shareholder of AIIB.
Framework) is an initiative endorsed by the Statement 2 is correct: The Missile Technology
G20, together with the Paris Club. Control Regime (MTCR) is a multilateral export
Statement 2 is correct: The  Common Framework control regime whose members have an
for debt treatment beyond the DSSI (Common informal political understanding to limit the
Framework) is an initiative to support, in a proliferation of missiles and missile technology.
structural manner, Low Income Countries with India became a member of the MTCR in 2016.
unsustainable debt. The Common Framework Statement 3 is correct: The Shanghai
considers debt treatment, on a case-by-case Cooperation Organization (SCO) is a permanent
basis, driven by requests from eligible debtor intergovernmental international organization,
countries. In response to a request for debt established in 2001. SCO has nine member
treatment, a Creditor Committee is convened. countries, they are - China, India, Kazakhstan,
Negotiations are supported by the IMF and Kyrgyzstan, Russia, Pakistan, Tajikistan,
the World Bank, including through their Debt Uzbekistan, and Iran. India joined SCO in 2017.
Sustainability Analysis.

19. Answer: (a)


16. Answer: (c)
Option (a) is correct: The ‘World Economic
Explanation: Outlook’ report is published by the International
Option (c) is correct: The Global Monetary Fund (IMF). It is a bi-annual report,

550 3B: WTO, IMF & other Intl. Orgs & Agreeements_Explanation
unacademy.com | Download the Unacademy app
Give your feedback here: Link
published twice a year. accountable to its 189 member countries.
y The report analyses global economic y The IMF’s primary purpose is to ensure
developments during the near and the stability of the international monetary
medium-term. It gives an overview as system such as the system of exchange
well as more detailed analysis of the rates and international payments that
world economy; consider issues affecting enables countries (and their citizens) to
industrial countries, developing countries, transact with each other.
and economies in transition to market; and y IMF provides loans to member countries
address topics of pressing current interest. that are experiencing actual or potential
balance-of-payments problems. It also
20. Answer: (c) provides financial assistance to its member
countries through allocation of Special
Option (c) is correct: The International Monetary Drawing Rights (SDRs) and Quota based on
Fund (IMF) is a Bretton Woods Organization, economic size and other parameters.
created in 1945 and it is governed by, and is

3B: WTO, IMF & other Intl. Orgs & Agreeements_Explanation 551
unacademy.com | Download the Unacademy app
Give your feedback here: Link
11 4A: Sectors of Economy- Agri

1. Which one of the following best describes 3. With reference to WaterCredit’, consider
the concept of 'Small Farmer Large Field? the following statements : (2021)
(2023) 1. It puts microfinance tools to work in
(a) Resettlement of a large number of the water and sanitation sector.
people, uprooted from their countries 2. It is a global initiative launched under the
due to war, by giving them a large aegis of the World Health Organization
cultivable land which they cultivate and the World Bank.
collectively and share the produce 3. It aims to enable the poor people
(b)
Many marginal farmers in an area to meet their water needs without
organize themselves into groups and depending on subsidies.
synchronize and harmonize selected Which of the statements given above are
agricultural operations correct?
(c) Many marginal farmers in an area together (a) 1 and 2 only
make a contract with a corporate body
and surrender their land to the corporate (b) 2 and 3 only
body for a fixed term for which the (c) 1 and 3 only
corporate body makes a payment of (d) 1, 2 and 3
agreed amount to the farmers
(d) A company extends loans, technical 4.
Find correct statement(s): (UPSC-
knowledge and material inputs to a Prelims-2020)
number of small farmers in an area 1. 36% of India’s districts are classified
so that they produce the agricultural as “overexploited” or “critical” by the
commodity required by the company CGWA.
for its manufacturing process and
2. Central Ground Water Authority (CGWA)
commercial production
was formed under the Environment
(Protection) Act.
2. Consider the following statements : (2023) 3. India has the largest area under
1. The Government of India provides groundwater irrigation in the world.
Minimum Support Price for niger Codes:
(Guizotia abyssinica) seeds.
(a) 1 only
2. Niger is cultivated as a Kharif crop.
(b) 2 and 3 Only
3. Some tribal people in India use niger
(c) 2 only
seed oil for cooking.
(d) 1 and 3 Only
How many of the above statements are
correct?
5. With reference to chemical fertilisers in
(a) Only one
India, consider the following statements:
(b) Only two (2020)
(c) All three 1. At present, the retail price of chemical
(d) None fertilisers is market-driven and not

552 4A: Sectors of Economy- Agri


unacademy.com | Download the Unacademy app
Give your feedback here: Link
administered by the Government. (d) 1, 2, 3 and 4
2. Ammonia, which is an input of urea, is
produced from natural gas. 8. In India, which of the following can
3. Sulphur, which is a raw material for be considered as public investment in
phosphoric acid fertiliser, is a by- agriculture? (2020)
product of oil refineries. 1. Fixing Minimum Support Price for
agricultural produce of all crops.
Which of the statements given above is/are
correct? 2. Computerization of Primary Agricultural
Credit Societies.
(a) 1 only
3. Social Capital development.
(b) 2 and 3 only
4. Free Electricity supply to farmers.
(c) 2 only
5. Waiver of agricultural loans by the
(d) 1, 2 and 3 banking system.
6. Setting up of cold storage facilities by
6. Consider the following statements (2020) the government.
1. In the case of all cereals, pulses and
oilseeds, the procurement at Minimum Select the correct answer using the code
Support Price (MSP) is unlimited in any given below:
State/UT of India. (a) 1, 2 and 5 only
2. In the case of cereals and pulses, the (b) 1, 3, 4 and 5 only
MSP is fixed in any State/UT at a level to (c) 2, 3 and 6 only
which the market price will never rise.
(d) 1, 2 ,3, 4, 5 and 6
Which of the statements given above is/are
correct?
9. The economic cost of food grains to the
(a) 1 only Food Corporation of India is Minimum
(b) 2 only Support Price and bonus (if any) paid to the
(c) Both 1 and 2 farmers plus: (2019)
(a) Transportation cost only
(d) Neither 1 nor 2
(b) Interest cost only

7. Which of the following factors/policies (c)


Procurement incidentals and
were affecting the price of rice in India in distribution cost
the recent past? (2020) (d) Procurement incidentals and charges
1. Minimum Support Price for godowns
2. Government’s trading
3. Government’s stockpiling 10. With reference to organic farming in India,
consider the following statements: (2018)
4. Consumer subsidies
1. 'The National Programme for Organic
Select the correct answer using the code Production' (NPOP) is operated under
given below: the guidelines and directions of the
(a) 1, 2 and 4 only Union Ministry of Rural Development.

(b) 1, 3 and 4 only 2. 'The Agricultural and Processed Food


Products Export Development Authority
(c) 2 and 3 only
(APEDA) functions as the Secretariat for

4A: Sectors of Economy- Agri 553


unacademy.com | Download the Unacademy app
Give your feedback here: Link
the implementation of NPOP. The Cabinet Committee on Economic Affairs
3. Sikkim has become India's first fully has announced the Minimum Support Price
organic State. for which of the above?

Which of the statements given above is/are (a) 1, 2, 3 and 7 only


correct? (b) 2, 4, 5 and 6 only
(a) 1 and 2 only (c) 1, 3, 4, 5 and 6 only
(b) 2 and 3 only (d) 1, 2, 3, 4, 5 and 7
(c) 3 only
(d) 1, 2 and 3 13. What is/are the advantages/advantages
of implementing the 'National Agriculture
Market' scheme? (2017)
11. As per the NSSO 70th Round "Situation
1. It is a pan-India electronic trading portal
Assessment Survey of Agricultural
for agricultural commodities.
Households", consider the following
statements: (2018) 2. It provides the farmers' access to
1. Rajasthan has the highest percentage a nationwide market, with prices
share of agricultural households among commensurate with the quality of their
its rural households. produce.

2. Out of the total agricultural households Select the correct answer using the code
in the country, a little over 60 percent given below:
belong to OBCs. (a) 1 only
3. In Kerala, a little over 60 percent (b) 2 only
of agricultural households reported (c) Both 1 and 2
to have received maximum income
from sources other than agricultural (d) Neither 1 nor 2
activities.
Which of the statements given above is/are 14.
Why does the Govt promote the use
correct? of Neem-coated Urea’ in agriculture?
(Prelims-2016)
(a) 2 and 3 only (a)
Release of Neem oil in the soil
(b) 2 only increases nitrogen fixation by the soil
(c) 1 and 3 only microorganisms.

(d) 1, 2 and 3 (b) Neem coating slows down the rate of


dissolution of urea in the soil.

12. Consider the following: (2018) (c) Nitrous oxide, a greenhouse gas, is not
1. Areca nut at all released into atmosphere by crop
fields.
2. Barley
(d) It is a combination of a weedicide and a
3. Coffee
fertilizer for particular crops.
4. Finger millet
5. Groundnut 15. Which of the following is/are the advantage/
6. Sesamum advantages of practising drip irrigation?
(2016)
7. Turmeric
1. Reduction in weed

554 4A: Sectors of Economy- Agri


unacademy.com | Download the Unacademy app
Give your feedback here: Link
2. Reduction in soil salinity 18.
The substitution of steel for wooden
3. Reduction in soil erosion ploughs in agricultural production is an
example of (2015)
Select the correct answer using the code
(a) labour-augmenting technological
given below.
progress
(a) 1 and 2 only
(b) capital-augmenting technological
(b) 3 only progress
(c) 1 and 3 only (c) capital-reducing technological progress
(d) None of the above is an advantage of (d) None of the above
practising drip irrigation
19. In India, markets in agricultural products
16. Consider the following statements: (2015) are regulated under the (2015)
1. The Accelerated Irrigation Benefits (a) Essential Commodities Act, 1955
Program was launched during 1996-
(b) Agricultural Produce Market Committee
97 to provide loan assistance to poor
Act enacted by States
farmers.
(c)
Agricultural Produce (Grading and
2. The Command Area Development
Marking) Act, 1937
Program was launched in 1974-75 for the
development of water-use efficiency. (d) Food Products Order, 1956 and Meat
and Food Products Order, 1973
Which of the statements given above is/are
correct?
20.
The Fair and Remunerative Price of
(a) 1 only
Sugarcane is approved by the (2015)
(b) 2 only (a) Cabinet Committee on Economic Affairs
(c) Both 1 and 2 (b) Commission for Agricultural Costs and
(d) Neither 1 nor 2 Prices
(c) Directorate of Marketing and Inspection,
17. Which one of the following best describes Ministry of Agriculture
the main objective of ‘Seed Village (d)
Agricultural Produce Marketing
Concept? (2015) Committee
(a) Encouraging the farmers to use their
own farm seeds and discouraging them
21. India has experienced persistent and high
to buy the seeds from others.
food inflation in the recent past. What
(b) Involving the farmers for training in could be the reasons? (2011)
quality seed production and thereby to 1. Due to gradual switch over to the
make available quality seeds to others cultivation of commercial crops, the
at appropriate time and affordable cost. area under the cultivation of food grains
(c) Earmarking some villages exclusively has steadily decreased in the last five
for the production of certified seeds. years by about 30%.

(d) Identifying the entrepreneurs in the 2. As a consequence of increasing incomes,


village and providing them technology the consumption patterns of people
and finance to set up seed companies. have undergone a significant change.
3. The food supply chain has structural
constraints.

4A: Sectors of Economy- Agri 555


unacademy.com | Download the Unacademy app
Give your feedback here: Link
Which of the statements given above are 2. To increase the processing of perishable
correct? items and reduce wastage.
(a) 1 and 2 only 3. To provide emerging and eco-friendly
(b) 2 and 3 only food processing technologies to
entrepreneurs.
(c) 1 and 3 only
Select the correct answer using the code
(d) 1, 2 and 3 given below:
(a) 1 only
22. With what purpose is the Government of
India promoting the concept of "Mega Food (b) 1 and 2 only
Parks"? (2011) (c) 2 and 3 only
1. To provide good infrastructure facilities (d) 1, 2 and 3
for the food processing industry.

556 4A: Sectors of Economy- Agri


unacademy.com | Download the Unacademy app
Give your feedback here: Link
4A: Sectors of Economy- Agri_
11 Explanation
1. Answer: (b) communities.
The concept of “Small Farmer Large Field” refers
to a collaborative approach where marginal 3. Answer: (c)
farmers in a particular area come together and
Statement 1 is correct: WaterCredit is a powerful
pool their resources to collectively undertake
solution and the first to put microfinance tools
agricultural operations on a larger scale. It
to work in the water and sanitation sector.
involves organising themselves into groups or
WaterCredit helps bring small loans to those
cooperatives to synchronize and harmonize
who need access to affordable financing and
selected farming activities.
expert resources to make household water and
In many regions, small-scale farmers face toilet solutions a reality.
numerous challenges such as limited access to
Statement 2 is not correct: It is an initiative
resources, including land, capital, machinery,
of water.org. Water.org is a global non-profit
and technology. These constraints often hinder
organization working to bring water and
their productivity and profitability. By adopting
sanitation to the world. It wants to make it
the “Small Farmer Large Field” approach,
safe, accessible, and cost-effective.
farmers aim to overcome these limitations
by leveraging their collective strength and Statement 3 is correct: WaterCredit helps
resources. bring small loans to those who need access
to affordable financing and expert resources
to make household water and toilet solutions
2. Answer: (c) a reality. It aims to enable poor people to
Statement 1 is correct: The Government meet their water needs without depending on
of India provides a Minimum Support Price subsidies.
(MSP) for niger seeds (Guizotia abyssinica).
The MSP for niger seeds is determined by the
4. Answer: (b)
government’s agricultural price policy, which
takes into account various factors such as Explanation:
production costs, market conditions, and the Statement 2 is correct: Central Ground Water
overall welfare of farmers. Authority (CGWA) was consfor the purposes
Statement 2 is correct:The cultivation of niger of regulation and control of ground water
as a Kharif crop is influenced by its growth development and management in the country,
requirements and the prevailing weather under sub-section (3) of Section 3 of the
conditions during the rainy season. It is one Environment (Protection) Act, 1986
of the Kharif crops cultivated in various states Statement 3 is correct: At 39 million hectares
of India, including Rajasthan, Gujarat, Uttar (67% of its total irrigation), India has the world’s
Pradesh, Madhya Pradesh, and Maharashtra. largest groundwater well equipped irrigation
Statement 3 is correct: Some tribal communities system. India, Pakistan, and Bangladesh are,
in India use niger seed oil for cooking purposes. respectively, the first, fourth and sixth largest
Niger seeds (Guizotia abyssinica) are known users of groundwater globally. India pumps
for their oil-rich content, and the oil extracted more than the US and China combined - the
from these seeds has culinary applications second and third-largest users, respectively.
in certain regions, including among tribal There is only 1 option where both 2 and 3 are

4A: Sectors of Economy- Agri_Explanation 557


unacademy.com | Download the Unacademy app
Give your feedback here: Link
together. So answer is B: 2 and 3 only. Agricultural Costs and Prices (CACP). The Centre
currently fixes MSPs for 23 farm commodities
based on the Commission for Agricultural
5. Answer: (b) Costs and Prices (CACP) recommendations.
Statement 1 is not correct: The Government Statement 1 is not correct: Under the new
of India subsidises fertilisers to ensure that PM-AASHA scheme, the existing Price Support
fertilisers are easily available to farmers Scheme (PSS) will continue for pulses, with
and the county remains self -sufficient in Central agencies including the NAFED and the
agriculture. This has been achieved largely Food Corporation of India, physically procuring
by controlling the price of fertiliser and the the produce whenever the market rates fall
amount of production. For instance, as per the below MSP, up to a maximum limit of 25% of
New Urea Policy of 2015, the government fixes the total harvest.
the market price of urea along with a fixed
Statement 2 is not correct: CACP considers
subsidy component. Similarly, for Phosphorus
various factors such as the cost of cultivation
and Potassium, as per the Nutrient Based
and production, productivity of crops, and
Subsidy Scheme of 2010, subsidy is provided
market prices for the determination of MSPs.
based on nutrient content per kg of fertiliser.
The MSP fixed for each crop is uniform for the
Statement 2 is correct: Fertiliser production entire country. However, there is no instance
uses 1.2% of the world’s total energy out of of MSP being fixed in any State/UT at a level to
which 90% is used for ammonia production, which the market price will never rise.
which is a key ingredient in the production of
nitrogen fertilisers. Ammonia can be produced
from natural gas. 7. Answer: (d)
Statement 3 is correct: Sulphur is a major by- Several factors impact the demand and supply
product of oil refining and gas processing. Most of rice in the market and thus have a direct/
crude oil grades contain some sulphur, most indirect impact on the prices of rice. Some of
of which must be removed during the refining them are:
process to meet strict sulphur content limits y Minimum Support Price (MSP) is the
in refined products. Industries, for instance, price at which the government purchases
the Mathura oil refinery, have been responsible crops from the farmers. MSP increases
for producing pollutants like sulphur dioxide the overall price of rice in the market as
and nitrogen dioxide. Also, Sulphur is used in the traders have to buy near MSP value
phosphoric acid fertiliser (There is a process otherwise farmers have the option to sell
known as ‘The Wet Process’ for producing the to government appointed agencies at
same). MSP. MSP helps to incentivize the farmers
and thus ensures adequate food grains
production in the country. (Statement 1 is
6. Answer: (d)
correct)
The Minimum Support Price (MSP) is the rate
y Government’s trading also affects the price
at which the government buys grains from
of rice as the government carries out an
farmers. The reason behind the idea of MSP
auction of the excess stockpile or intervenes
is to counter price volatility of agricultural
when market prices soar to stabilize them.
commodities due to the factors like variation
(Statement 2 is correct)
in their supply, lack of market integration and
information asymmetry. The MSP is fixed on y Government’s stockpiling is done to offload
the recommendations of the Commission for the stock when market prices soar, providing
cheap rice to consumers and stocking when

558 4A: Sectors of Economy- Agri_Explanation


unacademy.com | Download the Unacademy app
Give your feedback here: Link
prices fall ensuring remunerative prices to 10. Answer: (b)
farmers. (Statement 3 is correct) Statement 1 is not correct: The Ministry of
y Government also gives subsidized food to Commerce and Industry has implemented the
poor consumers at Fair price Shops, which National Programme for Organic Production
reduces the overall demand in the open (NPOP) since 2001.
market and prices fluctuate. (Statement 4 The objectives of NPOP are to provide the means
is correct) of evaluation of certification programmes for
organic agriculture and products (including
8. Answer: (c) wild harvest, aquaculture, livestock products)
as per the approved criteria, to accredit
Public investment is the investment made by
certification programmes of Certification
the government in sectors and assets that are
Bodies seeking accreditation, to facilitate
expected to provide facilities to the general
certification of organic products in conformity
public.
with the prescribed standards, to facilitate
Option (c) is correct: Public investment in certification of organic products in conformity
agriculture includes Computerization of with the importing countries organic standards
Primary Agricultural Credit Societies, Social as per equivalence agreement between the
Capital Development and Setting up of cold two countries or as per importing country
storage facilities by the government. They requirements, to encourage the development
are considered as public investment in the of organic farming and organic processing.
agricultural sector as they provide an overall
Statement 2 is correct: The Agricultural and
benefit of development to the agriculture
Processed Food Products Export Development
sector.
Authority (APEDA) shall function as the
However, Fixing Minimum Support Price for Secretariat for the implementation of the NPOP.
agricultural produce of all crops, Free Electricity APEDA was established by the Government
supply to farmers and Waiver of agricultural of India under the Agricultural and Processed
loans by the banking system will not be Food Products Export Development Authority
considered as public investment, as they will Act passed by the Parliament in December
only benefit certain sections of farmers and 1985.
that too only in the short term. They may prove
Statement 3 is correct: Sikkim has become
to be detrimental to public interest in the long
India’s first fully organic state by implementing
run.
organic practices on around 75,000 hectares of
agricultural land.
9. Answer: (c)
Option (c) is correct: Economic Cost is the 11. Answer: (c)
total cost to the Food Corporation of India. It
The situation assessment survey of agricultural
consists of Acquisition Cost and Distribution
households is conducted by the National
Cost. Acquisition cost consists of Minimum
Sample Survey Office (NSSO). It is a detailed
Support Price (MSP) plus procurement
socio-economic survey of farmer households
incidental cost.
and is the only large-scale survey that focuses
Procurement incidentals are expenses incurred specifically on estimating household crop
during procurement till the food grains reach income in India.
the first point of godown.
Statement 1 is correct: As per the survey,
Rajasthan had the highest percentage of
agricultural households (78.4 percent) among

4A: Sectors of Economy- Agri_Explanation 559


unacademy.com | Download the Unacademy app
Give your feedback here: Link
its rural households and the least percentage 13. Answer: (c)
stood for Kerala (27.3 percent). Statement 1 is correct: National Agriculture
Statement 2 is not correct: Out of the total Market is a pan-India electronic trading
estimated agricultural households in the portal that networks the existing APMC
country, about 45 percent belonged to Other mandis to create a unified national market
Backward Classes (OBCs). for agricultural commodities. Its vision is to
Statement 3 is correct: In Kerala, about promote uniformity in agriculture marketing by
61 percent of the agricultural households streamlining procedures across the integrated
reported to have received maximum income markets, removing information asymmetry
from sources other than agricultural activities. between buyers and sellers and promoting
real-time price discovery based on actual
Recently, the government has cleared the
demand and supply.
formation of an overarching body- National
Statistical Office (NSO) through the merger Statement 2 is correct: Its mission is to facilitate
of the NSSO and Central Statics Office (CSO) the integration of APMCs across the country
under the Ministry of Statistics and Programme through a common online market platform
Implementation (MoSPI). to facilitate pan-India trade in agriculture
commodities, providing better price discovery
through a transparent auction process based
12. Answer: (b) on the quality of products along with timely
Minimum Support Price (MSP) is a form of online payment.
market intervention by the Government of Small Farmers Agribusiness Consortium (SFAC)
India to insure agricultural producers against is the lead agency for implementing the
any sharp fall in farm prices. The minimum National Agricultural Market under the aegis
support prices are announced by the The of the Ministry of Agriculture and Farmers’
Cabinet Committee on Economic Affairs, Welfare, Government of India.
Government of India at the beginning of the
sowing season for certain crops on the basis of
the recommendations of the Commission for 14. Answer: (b)
Agricultural Costs and Prices (CACP). Explanation:
Option (b) is correct: Government announces Option (b) is correct: The use of Neem Coated
MSP for 22 mandated crops and fair and Urea has been found to improve the uptake
remunerative price (FRP) for sugarcane. The of Nitrogen, Phosphorous and potassium
mandated crops are 14 crops of the kharif significantly. Neem-coating makes more
season, 6 rabi crops and two other commercial difficult for black marketers to divert urea
crops. The list of crops are as follows: to industrial consumers. Neem-coating also
Cereals: Paddy, Wheat, Barley, Jowar, Bajra, benefits farmers by reducing nitrogen losses
Maize and Ragi (finger millet). from the soil by providing greater nutrient to
the crop. As a result, farmers need less urea to
Pulses: Gram, Arhar/tur, Moong, Urad and
achieve the same effect. In Neem-coated Urea,
Lentil.
every granule of urea has a coating of neem
Oilseeds: Groundnut, Rapeseed/mustard, oil which slows down the rate of dissolution
Toria, Soyabean, Sunflower seed, Sesamum, of urea in the soil and thus increase the
Safflower seed and Niger seed. availability of nitrogen for crops.
Others: Raw cotton, Raw jute, Copra, De- Benefits of use of Neem-coated Urea:
husked coconut, Sugarcane and Virginia flu
y Increase in the crop yield.
cured (VFC) tobacco.

560 4A: Sectors of Economy- Agri_Explanation


unacademy.com | Download the Unacademy app
Give your feedback here: Link
y Improvement in soil fertility. Statement 2 is correct: Centrally Sponsored
y Optimum utilization of limited water Command Area Development Program (CADP)
resources. was launched in December 1974 to improve
potential irrigation utilization and optimize
y Decrease the cost of cultivation.
agricultural production from irrigated land
y Better nutrition for family. through an integrated and coordinated
y Slow release of nitrate into the soil. approach of efficient water management.

y Improves ground water quality.


y Lesser quantity of urea is required, saves 17. Answer: (b)
precious foreign exchange. Option (b) is correct: ‘Seed Village concept’ is
the term given to a practice wherein a group of
likeminded (in their profession) farmers brought
15.Answer: (c)
together into a ‘Self Help Group’ are trained to
Drip irrigation is a type of micro-irrigation that work together in their effort to produce seeds
can save water and nutrients by allowing the of crops of their choice and cater to the needs
water to drip slowly into the roots of plants, of their own and that of fellow farmers, in
vegetables, trees, and grass. The irrigation appropriate time and at an affordable cost. The
comes from above the soil surface or buried main feature of this program is:
below.
y Seed is available at the doorsteps of the
Option (c) is correct: The goal is to water farmers at appropriate times.
directly into the root zone and minimise as
y Seed is available at an affordable cost
much evaporation as possible. Drip irrigation
(lesser than the market price) due to
systems distribute water through a network of
decreased overheads.
valves, pipes, tubing, and emitters and it can
be more efficient than other types of irrigation y Increased confidence among the secondary
systems such as sprinkler or surface irrigation. seed producers about the quality of the
Some of the advantages of using drip irrigation product because of known sources of seed
are: It saves water, improves plant growth, production.
controls weed (systems place water to the y Producer and consumer are mutually
area directly around the plant. As a result, benefited (Win-Win situation).
weed seeds are water starved and germination
y Facilitates fast spread of new cultivars
is limited), reducing soil erosion. It does not
and region-specific varieties that are not
reduce soil salinity.
covered by the seed agencies.

16. Answer: (b)


18. Answer: (b)
Statement 1 is not correct: The Central
Option (b) is correct: Capital augmenting
Government launched the Accelerated
technological progress in economic parlance
Irrigation Benefits Program (AIBP) in the
results in more productive use of existing
year 1996-97 to provide financial assistance
capital goods Capital in an economic context
(loans) to major/medium irrigation projects
means machinery or capital goods which can
in the country (not to poor farmers), with the
be employed to produce other goods.
objective to accelerate implementation of
such projects which were beyond resource y Substitution of steel for wooden ploughs
capability of the States or were in advanced can be regarded as the substitution of a
stage of completion. lesser machine by a better machine. This

4A: Sectors of Economy- Agri_Explanation 561


unacademy.com | Download the Unacademy app
Give your feedback here: Link
encourages more steel production. Hence 21. Answer: (b)
it is considered as capital-augmenting Food inflation is referred to as the persistent
technological progress. On the other hand, rise in the price of agricultural produce for
labour-augmenting technological progress a certain period of time. The causes of food
is technical progress that increases the inflation are manifold. It may be due to less
effective labour input. agricultural production, soaring demand for
agricultural produce, supply shortages, etc. In
19. Answer: (b) the previous decade high food inflation was a
problem for the country.
Option (b) is correct: Agriculture falls under
the State List of the Constitution. Agriculture Statement 1 is not correct: With respect to
marketing in most states is regulated by 2011, India has experienced persistent and high
the Agricultural Produce Market Committee food inflation and it is due to the fact that the
(APMCs) established by the state governments area under food grain cultivation did decrease
under the respective APMC Acts. The APMCs in the last five years. The area under food
provide infrastructure for marketing of grain cultivation decreased around 4-5% of the
agricultural produce, regulate sale of such gross cropped area of the country. This caused
produce and collect market fees from such a demand-supply mismatch, which resulted in
sale, and regulate competition in agricultural high food inflation.
marketing. Statement 2 is correct: Due to the liberalization
of the economy, the overall economic activity
got pushed up bringing economic growth to
20. Answer: (a)
the country in effect increasing the incomes
Option (a) is correct: The pricing of sugarcane of people. Due to these increased incomes,
is governed by the statutory provisions of the consumption of food grains has also increased,
Sugarcane (Control) Order, 1966 issued under thus increasing the demand for the same.
the Essential Commodities Act (ECA), 1955. The production did not increase at the same
Prior to the 2009-10 sugar season, the Central pace, due to which food inflation did happen
Government was fixing the Statutory Minimum frequently.
Price (SMP) of sugarcane and farmers were
Statement 3 is correct: Food supply chain from
entitled to share profits of a sugar mill on a
farm to the plate of the consumers, has got
50:50 basis. As this sharing of profits remained
structural constraints like lack of warehousing,
virtually unimplemented, the Sugarcane
low farm sorting and grading, lack of cold
(Control) Order, 1966 was amended in October
chain facilities in the country etc. These were
2009 and the concept of SMP was replaced
the major supply side constraints which led to
by the Fair and Remunerative Price (FRP) of
food inflation.
sugarcane.
y Cabinet Committee on Economic Affairs
on behalf of the government approves 22. Answer: (b)
minimum support price (MSP) for 22 crops Option (b) is correct: The Mega Food Park
and fair and remunerative price (FRP) for Scheme is based on the “Cluster” approach
sugarcane. and envisages creation of state of art
y The MSP and FRP are fixed in line with support infrastructure in a well-defined agri/
the recommendation of the Commission horticultural zone for setting up of modern food
of Agricultural Costs and Prices (CACP), a processing units in the industrial plots provided
statutory body that advises the government in the park with a well-established supply
on the pricing policy for major farm produce. chain. The Scheme of Mega Food Park aims

562 4A: Sectors of Economy- Agri_Explanation


unacademy.com | Download the Unacademy app
Give your feedback here: Link
at providing a mechanism to link agricultural
production to the market by bringing together
farmers, processors, and retailers so as to
ensure maximizing value addition, minimizing
wastage, increasing farmers income and
creating employment opportunities particularly
in the rural sector.

4A: Sectors of Economy- Agri_Explanation 563


unacademy.com | Download the Unacademy app
Give your feedback here: Link
4B: Sectors- MFG, Services, Ease of
12 Doing Biz, IPR, Startup, MSME
1. Consider the following statements with Select the correct answer using the code
reference to India: (2023) given below :
1. According to the ‘Micro, Small and (a) 1 only
Medium Enterprises Development (b) 2 only
(MSMED) Act, 2006’, the ‘medium
enterprises’ are those with investments (c) Both 1 and 2
in plant and machinery between Rs. 15 (d) Neither 1 nor 2
crore and Rs. 25 crore.
2. All bank loans to the Micro, Small and 4. Consider the following statements: [2019]
Medium Enterprises qualify under the 1. According to the Indian Patents Act, a
priority sector. biological process to create a seed can
Which of the statements given above is/are be patented in India.
correct? 2. In India, there is no Intellectual Property
(a) 1 only Appellate Board.
(b) 2 only 3. Plant Varieties are not eligible to the
(c) Both 1 and 2 patented in India.

(d) Neither 1 nor 2 Which of the statements given above is/


are correct?
(a) 1 and 3 only
2. In India, which one of the following
compiles information on industrial (b) 2 and 3 only
disputes, closures, retrenchments and (c) 3 only
lay-offs in factories employing workers? (d) 1,2, and 3
(2022)
(a) Central Statistics Office
5. Atal innovation mission is set up under the
(b) Department for Promotion of Industry [2019]
and Internal Trade (a) Department of science of technology
(c) Labour Bureau (b) Ministry of labour and employment
(d) National Technical Manpower (c) NITI Ayog
Information System
(d)
Ministry of skill development and
entrepreneurship
3.
With reference to foreign-owned
e-commerce firms operating in India, 6. Which of the following statements is/are
which of the following statements is/are correct regarding the Maternity Benefit
correct ? (2022) (Amendment) Act, 2017? [2019]
1. They can sell their own goods in addition 1. Pregnant women are entitled for three
to offering their platforms as market- months predelivery and three months
places. post-delivery paid leave
2. The degree to which they can own big 2. Enterprises with creches must allow
sellers on their platforms is limited.

564 4B: Sectors- MFG, Services, Ease of Doing Biz, IPR, Startup, MSME
unacademy.com | Download the Unacademy app
Give your feedback here: Link
the mother minimum six creche visits (a) 1 only
daily (b) 2 only
3. Women with two children get reduced (c) Both 1 and 2
entitlements.
(d) Neither 1 nor 2
Select the correct answer using the code
given below
9. With reference to the 'National Intellectual
(a) 1 and 2 only Property Rights Policy', consider the
(b) 2 only following statements: (2017)
(c) 3 only 1. It reiterates India's commitment to the
Doha Development Agenda and the
(d) 1,2 and 3
TRIPS Agreement.
2. Department of Industrial Policy and
7. As per the Industrial Employment (Standing
Promotion is the nodal agency for
Orders) Central (Amendment) Rules, 2018:
regulating intellectual property rights in
[2019]
India.
1. If rules for fixed-term employment are
implemented, it becomes easier for the Which of the above statements is/are
firms/companies to lay off workers correct?

2. No notice of termination of employment (a) 1 only


shall be necessary in the case of (b) 2 only
temporary workman (c) Both 1 and 2
Which of the following statements given (d) Neither 1 nor 2
above is/are correct
(a) 1 only 10. What is/are the recent policy initiative(s)
(b) 2 only of Government of India to promote the
(c) Both 1 and 2 growth of manufacturing sector? (2012)
1. Setting up of National Investment and
(d) Neither 1 nor 2
Manufacturing Zones
2. Providing the benefit of 'single window
8. Consider the following statements [2018]
clearance'
1. The Food Safety and Standards Act,
2006 replaced the Prevention of Food 3. Establishing the Technology Acquisition
Adulteration Act, 1954. and Development Fund

2. The Food Safety and Standards Select the correct answer using the code
Authority of India (FSSAI) is under the given below:
charge of Director General of (a) 1 only
Health Services in the Union Ministry of (b) 2 and 3 only
Health and Family Welfare. (c) 1 and 3 only
Which of the statements given above is/are (d) 1, 2 and 3
correct ?

4B: Sectors- MFG, Services, Ease of Doing Biz, IPR, Startup, MSME 565
unacademy.com | Download the Unacademy app
Give your feedback here: Link
4B: Sectors- MFG, Services, Ease of Doing
12 Biz, IPR, Startup, MSME_Explanation

1. Answer: (d) a lack of mutual understanding, a difference


Statement 1 is incorrect: According to the of opinion and a failure to settle the disputes
MSMED Act, the classification of micro, small, between the management and the workers/
and medium enterprises in India is based on labour trade unions. The conflicts between
their investment in plant and machinery or the management and workers are inevitable
equipment for manufacturing enterprises, as both rationalize their gains and losses in
and investment in equipment for service economic activities.
enterprises. Here are the investment limits
specified in the act: 3. Answer: (d)
1. Micro Enterprises: Investment up to Rs. 1 Statement 1 is not correct: FDI in the inventory
crore. model is prohibited.
2. Small Enterprises: Investment between Rs. Statement 2 is not correct: Any big seller can
1 crore and Rs. 10 crore. not have more than 25% of the total sales on an
3. Medium Enterprises: Investment between e-commerce platform. However, the number
Rs. 10 crore and Rs. 50 crore. of sellers that can be owned by the platform is
Statement 2 is not correct: All bank loans not prohibited. Thus to overcome this limit of
to MSMEs that meet the criteria outlined in 25%, platforms open multiple companies.
the RBI’s Master Direction on “Priority Sector
Lending (PSL) – Targets and Classification” 4. Answer: (c)
dated September 4, 2020 are eligible to be
Explanation:
classified as priority sector lending.
The Indian Patent Act, 1970 strikes a balance
between the rights of the applicant and his
2. Answer: (c) obligation to the society granting the rights.
Option (c) is correct: The present edition on Some salient features of the Act include,
Statistics of “Industrial Disputes, Closures, product and process patent, term of patent
Retrenchments and Lay-offs in India during as 20 years, patent examination conducted
2006” is based on the voluntary returns on request, fast track mechanism for quick
received every month from the Labour disposal of appeals, pre-grant and post-grant
Departments of the States and the Regional opposition allowed, protection of biodiversity
Labour Commissioner (Central). The statistics and traditional knowledge, and, publication of
of Industrial Disputes resulted in the temporary applications after 18 months of date of filing of
stoppage of work either by the employees patent application.
of an establishment who resorted to Strikes
Statement 1 is not correct: Under Section 3(j)
to express a grievance, to enforce a demand
of India’s Patents Act, a seed or a plant, or a
or by an employer (or a group of employers)
biological process to create a seed or plant
who withhold work from the employees in
cannot be patented.
an establishment in connection with matters
relating to employment or non-employment Statement 2 is not correct: Intellectual
or terms and conditions of employment. Thus, Property Appellate Board has been constituted
both types of work stoppages known as Strikes by a Gazette notification of the Central
and Lockouts take place as and when there is Government in the Ministry of Commerce

566 4B: Sectors- MFG, Services, Ease of Doing Biz, IPR, Startup, MSME_Explanation
unacademy.com | Download the Unacademy app
Give your feedback here: Link
and Industry on 15th September 2003 to hear more employees, including the right of
appeals against the decisions of the Registrar mothers to visit the crèche four times per
under the Trade Marks Act, 1999 and the day. (Statement 2 is not correct) Thus,
Geographical Indications of Goods (Registration Statement 2 is wrong and by elimination
and Protection) Act, 1999. of options containing this statement, we
arrive at the correct answer [C].
Statement 3 is correct: In India, there are no
laws that allow for patenting patented in India.
7. Answer: (c)
5. Answer: (c) Explanation:
Explanation: Recently, the Ministry of Labour and
Employment notified fixed term employment
Option (c) is correct: Atal Innovation Mission is
for all sectors through an amendment to
a flagship scheme by the Government of India,
the Industrial Employment (Standing Orders)
set up under NITI Aayog (National Institute
Central Rules, 1946. Under the rules, a fixed
for Transforming India), erstwhile Planning
term employment workman is engaged on the
Commission, to create and promote a culture
basis of a written contract of employment for
of innovation and entrepreneurship across the
a fixed period.
country.
Statement 1 is correct: The changes introduced
Its objective is to develop new programmes and
in Industrial Employment (Standing Orders)
policies for fostering innovation in different
Act, 1946 have made it easy for the firms to
sectors of the economy, provide platforms
hire and fire employees by introducing fixed
and collaboration opportunities for different
term employment in all sectors.
stakeholders, and create an umbrella structure
to oversee the innovation & entrepreneurship Statement 2 is correct: As the Amendment
ecosystem of the country. provides that subject to the provisions of
the Industrial Disputes Act, 1947, no notice of
termination of employment shall be necessary
6. Answer: (c) in the case of temporary workman whether
Explanation: monthly rated, weekly rated or piece rated and
The 2017 landmark amendment to the probationers or badli workmen.
Maternity Benefit Act of 1961 (MBA), The key
changes include: 8. Answer: (a)
y Increased paid maternity leave from 12 Explanation:
weeks to 26 weeks for women employees,
The Food Safety and Standards Act was
unless they have two or more surviving
enacted in 2006 as an Act to consolidate the
children; (Statement 3 is correct)
laws relating to food and to establish the
y Recognition of the rights of an adopting Food Safety and Standards Authority of India
mother and of a commissioning mother for laying down science-based standards
(using a surrogate to bear a child) for the for articles of food and to regulate their
first time, who may claim paid maternity manufacture, storage, distribution, sale and
leave for 12 weeks. import, to ensure availability of safe and
y A “work from home” option that may be of wholesome food for human consumption and
benefit after the maternity leave expires. for matters connected therewith or incidental
y Mandatory crèche (day care) facilities thereto.
for every establishment employing 50 or Statement 1 is correct: The Food Safety and

4B: Sectors- MFG, Services, Ease of Doing Biz, IPR, Startup, MSME_Explanation 567
unacademy.com | Download the Unacademy app
Give your feedback here: Link
Standard Act repealed various Acts and Orders under the Ministry of Commerce and Industry
like the Edible Oils Packaging (Regulation) is the nodal agency for regulating intellectual
Order 1998, Fruit Products Order (FPO) 1955, property rights in India.
Meat Food Products Order (MFPO), Prevention
of Food Adulteration Act,1954 etc.
10. Answer: (d)
Statement 2 is not correct: Ministry of Health
& Family Welfare, Government of India is the Statement 1 is correct: Setting up of National
Administrative Ministry for the implementation Investment and Manufacturing Zones (NIMZs)
of FSSAI. The Chairperson and Chief Executive are one of the important instruments of
Officer of Food Safety and Standards Authority National Manufacturing Policy, 2011. NIMZs are
of India (FSSAI) is appointed by Government envisaged as large areas of developed land
of India, who holds the rank of Secretary to with the requisite eco-system for promoting
Government of India. The authority acts in world class manufacturing activity. NIMZs are
independent capacity. based on the principle of industrial growth
in partnership with States and focuses on
manufacturing growth and employment
9. Answer: (c) generation.
The National Intellectual Property Rights (IPR) Statement 2 is correct: Through a single
Policy is a vision document that encompasses window clearance approach the new
and brings to a single platform all IPRs. It views manufacturing units can obtain all clearances
IPRs holistically, taking into account all inter- from separate agencies, ministries, and various
linkages and thus aims to create and exploit licenses etc. by filing all documents with a
synergies between all forms of intellectual single authority instead of having to deal with
property (IP), concerned statutes and agencies. multiple regulators and bodies. The single
It sets in place an institutional mechanism for window clearance mechanism reduces delay
implementation, monitoring and review. It aims and thus helps in faster clearance and setting
to incorporate and adapt global best practices up manufacturing industries.
to the Indian scenario. Statement 3 is correct: Establishing the
Statement 1 is correct: It reiterates India’s Technology Acquisition and Development Fund
commitment to Doha Development Agenda (TADF), provides funding support to Micro,
and TRIPS Agreement as the policy will enable Small and Medium Enterprises (MSMEs) for
smooth facilitation of trade across nations by the acquisition and development of clean and
the smooth transition of Intellectual Property green technology. The Scheme is applicable to
Rights across borders. all existing and new Micro, Small and Medium
Statement 2 is correct: Department of Industrial Enterprises (MSMEs) including those in the
Policy and Promotion (now, Department for National Investment and Manufacturing Zones
Promotion of Industry and Internal Trade) (NIMZs) in respect of their investments made
after notification of the Scheme.

568 4B: Sectors- MFG, Services, Ease of Doing Biz, IPR, Startup, MSME_Explanation
unacademy.com | Download the Unacademy app
Give your feedback here: Link
4C: NITI, Planning Commission, FYP,
13 Unemployment
1. Consider the following statements: (UPSC- (c) 1 and 3 only
2022) (d) 1, 2 and 3
1. Vietnam has been one of the fastest
growing economies in the world in the
recent years. 3. With reference to the Indian Economy after
the 1991 economic liberalization, consider
2. Vietnam is led by a multi-party political the following statements: (2020)
system. 1. Worker productivity (rupees per worker
3. Vietnam's economic growth is linked to at 2004-05 prices) increased in urban
its integration with global supply chains areas while it decreased in rural areas.
and focus on exports. 2. The percentage share of rural areas in
4. For a long time Vietnam's low labour the workforce steadily increased.
costs and stable exchange rates have 3. In rural areas, the growth in the non-
attracted global manufacturers. farm economy increased.
5. Vietnam has the most productive 4. The growth rate in rural employment
eservice sector in the Indo-Pacific decreased.
region.
Which of the statements given above is/are
Which of the statements given above are correct?
correct?
(a) 1 and 2 only
(a) 2 and 4
(b) 3 and 4 only
(b) 3 and 5
(c) 3 only
(c) 1, 3 and 4
(d) 1, 2 and 4
(d) 1 and 2

4. With reference to land reforms in


2. With reference to casual workers employed independent India, which one of the
in India, consider the following statements: following statements is correct? (2019)
(2021) (a) The ceiling laws were aimed at family
1. All casual workers are entitled for holdings and not individual holdings.
Employees Provident Fund coverage.
(b) The major aim of land reforms was
2. All casual workers are entitled for providing agricultural land to all the
regular working hours and overtime landless.
payment.
(c) It resulted in cultivation of cash crops
3. The government can by a notification as a predominant form of cultivation.
specify that an establishment or
industry shall pay wages only through (d) Land reforms permitted no exemptions
its bank account. to the ceiling limits.

Which of the above statements are correct?


5. With reference to India’s Five-Year Plans,
(a) 1 and 2 only which of the following statements is/are
(b) 2 and 3 only correct? (2019)
1. From the Second Five-Year Plan, there

4C: NITI, Planning Commission, FYP, Unemployment 569


unacademy.com | Download the Unacademy app
Give your feedback here: Link
was a determined thrust towards (b) inclusive and sustainable growth
substitution of basic and capital goods (c) sustainable and inclusive growth to
industries. reduce unemployment
2. The Fourth Five-Year Plan adopted the (d) faster, sustainable, and more inclusive
objective of correcting the earlier trend growth
of increased concentration of wealth
and economic power.
8. Disguised unemployment generally means:
3. In the Fifth Five-Year Plan, for the first (2013)
time, the financial sector was included (a)
large number of people remain
as an integral part of the Plan. unemployed
Select the correct answer using the code (b) alternative employment is not available
given below:
(c) marginal productivity of labour is zero
(a) 1 and 2 only
(d) productivity of workers is low
(b) 2 only
(c) 3 only 9. How do District Rural Development
(d) 1, 2 and 3 Agencies (DRDAs) help in the reduction of
rural poverty in India? (2012)
6. Which of the following have occurred in 1. DRDAs act as Panchayati Raj Institutions
India after its liberalization of economic in certain specified backward regions of
policies in 1991? (2017) the country.
1. Share of agriculture in GDP increased 2. DRDAs undertake area-specific
enormously. scientific study of the causes of poverty
2. Share of India's exports in world trade and malnutrition and prepare detailed
increased. remedial measures.

3. FDI inflows increased. 3. DRDAs secure inter-sectoral and


inter-departmental coordination and
4. India's foreign exchange reserves
cooperation for effective implementation
increased enormously.
of anti-poverty programmes.
Select the correct answer using the codes
4. DRDAs watch over and ensure effective
given below:
utilization of the funds intended for
(a) 1 and 4 only anti-poverty programmes.
(b) 2, 3 and 4 only Which of the statements given above is/are
(c) 2 and 3 only correct?

(d) 1, 2, 3 and 4 (a) 1, 2 and 3 only


(b) 3 and 4 only
7. The main objective of the 12th Five-Year (c) 4 only
Plan is: (2014)
(d) 1, 2, 3 and 4
(a) inclusive growth and poverty reduction

570 4C: NITI, Planning Commission, FYP, Unemployment


unacademy.com | Download the Unacademy app
Give your feedback here: Link
4C: NITI, Planning Commission, FYP,
13 Unemployment_Explanation
1. Answer: (c) from 28.12.2016) to enable the employers to
Explanation: pay wages to their employees by (a) cash or (b)
cheque or (c) crediting to their bank account.
Statement 1 is correct: According to the World The amendment in the Act also enables
Bank, Vietnam has been one of the fastest the appropriate Government to specify the
growing economies in the world in recent industrial or another establishment, by
years. notification in the Official Gazette, which shall
Statement 2 is WRONG: Vietnam is a socialist pay to every person employed in such industrial
republic with a one-party system led by the or another establishment, the wages only by
Communist Party of Vietnam (CPV). cheque or by crediting in his bank account.
Statement 3 is correct: Vietnam’s economic
growth is linked to its integration with global 3. Answer: (b)
supply chains and focus on exports.
Rules and laws which were aimed at
Statement 4 is correct: In Vietnam, Sub-sectors regulating economic activities became major
of the digital economy such as e-commerce, hindrances in growth and development, hence,
digital financial services, are on the rise. It has liberalization was introduced to put an end to
the most productive e-service sector in the these restrictions and open various sectors
Indo-Pacific region. of the economy for its revival in 1991. Post
liberalization there have been many impacts
2. Answer: (b) on the Indian economy.

Statement 1 is not correct: Casual workers are y Worker productivity has comparatively
engaged by various Ministries/ Departments increased both in the urban as well as the
and their attached and subordinate offices rural areas. As per the 2017 report of NITI
for work of casual or seasonal, or intermittent Aayog, in rural areas worker productivity
nature. They are engaged according to was Rs. 37,273 per worker in 2004-05 and it
the requirements of different Ministries/ increased to Rs. 1,01,755 per worker in 2011-
Departments/ attached/subordinate offices. All 12. While, in urban areas it increased from
casual workers are not entitled to Employees Rs. 1,20,419 per worker in 2004-05 to Rs.
Provident Fund coverage. There are certain 2,82,515 per worker in 2011-12. (Statement
conditions which need to be filled. 1 is not correct)

Statement 2 is correct: Holding that an y The rural share in the total workforce
employer cannot differentiate between declined steadily from 76.1 per cent in
contractual and permanent employees, the 1999-00 to 70.9 per cent in 2011-12. Due
Supreme Court ruled in 2020 that casual to a faster reduction in the rural share in
workers are also entitled to social security total employment than in national NDP (Net
benefits under the Employees’ Provident Fund. Domestic Product), the difference between
So, all casual workers are entitled to regular the rural share in output and employment
working hours and overtime payment. narrowed down to 24 per cent by the year
2011-12. (Statement 2 is not correct)
Statement 3 is correct: The Payment of Wages
Act, 1936 has been amended by the Payment y The share of the manufacturing and service
of Wages (Amendment) Act, 2017 (effective sector has increased in the Rural NDP,

4C: NITI, Planning Commission, FYP, Unemployment_Explanation 571


unacademy.com | Download the Unacademy app
Give your feedback here: Link
taking the place of the decline in the share unknowingly developed an urban bias, which
of contribution of the agricultural sector in to an extent promoted concentration of wealth
the Rural NDP. (Statement 3 is correct) in the hands of industrialists due to the heavy
focus on rapid industrialisation. The Fourth
y The growth rate in rural employment has
Five-Year Plan however, getting its inspiration
decreased, even going negative in the 2005-
from the Directive Principles of State Policy
12 time period. (Statement 4 is correct)
sought to move further on setting up the
society on Socialistic Patterns in which it
4. Answer: (b) adopted means to prevent concentration of
Option (b) is correct: Land reforms in India wealth and economic power.
were one of the most crucial steps taken Statement 3 is not correct: For the first time,
after independence to overcome the problems the financial sector was included as an integral
associated with the land revenue systems part of the plan in the Ninth Five-Year Plan.
introduced by the British government. Land
reforms consisted of Abolition of intermediaries,
6. Answer: (b)
Land Ceiling, Protection to tenant farmers and
consolidation of land holdings. India enacted its New Economic Policy in 1991,
which ushered into a new era of Liberalisation,
y Land reforms introduced ceiling laws that
Privatisation and Globalization.
were aimed at individual land holdings and
not at family holdings. Statement 1 is not correct: The share of
agriculture in the GDP of India has actually
y The major aim of the land reforms included
decreased since 1991, from about 31 per cent
redistribution of agricultural land to the
of GDP in 1993-94 to around 17 per cent in 2018
landless poor by putting a ceiling on the
and 20 per cent in 2020-21.
surplus land. This helped both in increasing
agricultural productivity in the country Statement 2 is correct: Liberalization and
along with aiding poverty alleviation and Globalization of the Indian economy made way
promoting social equity. smoother for the flow of foreign capital into the
country and interlinked India to a greater extent
y Land reforms were not connected to the
to the world. This also led to the development
cultivation of cash crops in any way directly
of domestic industry, the produce of which
as a predominant form of cultivation.
found acceptance throughout the world. The
y Land reforms permitted many exemptions total value of exports of India was $17,900
to the ceiling limits such as for garden million in 1991. In the fiscal year (FY)2018-19,
lands, forest lands and barren lands, etc. India’s total merchandise exports hit a new
high of $330 billion, surpassing the earlier peak
5. Answer: (a) of $314 billion clocked in 2013-14 FY. India’s
overall exports in April-September 2020-21 are
Statement 1 is correct: The Second Five-Year estimated to be USD 221.86 billion. Thus the
Plan sought to rapidly industrialise India, for share of exports of goods and services from
which a basic and capital goods industry was India to the world has increased.
needed to set the base for the coming up of
subsequent consumer goods industry. There Statement 3 is correct: FDI Inflows in the year
was a major emphasis on basic and capital following the reforms stood at $ 133 Million,
goods industries as they were the major which has today increased to $ 81.72 billion
determinants of rate of growth of the economy. during the financial year 2020-21.

Statement 2 is correct: Earlier plans had Statement 4 is correct: As of March-end 1991,


Indian Foreign Exchange reserves stood at

572 4C: NITI, Planning Commission, FYP, Unemployment_Explanation


unacademy.com | Download the Unacademy app
Give your feedback here: Link
$ 5.8 billion, which were barely enough for whereas sustainability includes ensuring
covering around three weeks of the country’s environmental sustainability, development
import bills; this forex crisis led the country of human capital through improved
through a series of challenges and eventually health, education, skill development,
ushering into the reform’s era. The subsequent nutrition, information technology,
economic reforms and policies have today etc., and development of institutional
enabled India to hold Foreign Exchange capabilities, infrastructure like power
Reserves worth nearly $493.48 billion (around telecommunication, roads, transport etc.
Rs 37.30 lakh crore) in 2020. Thus, it increased
enormously.
8. Answer: (c)
Option (c) is correct: Disguised unemployment
7. Answer: (d) occurs when surplus labour is employed, out
Option (d) is correct: Five-Year Plans (FYPs) are of which some employees have zero or almost
centralized and integrated national economic zero marginal productivity. As such, this type of
programs. Joseph Stalin implemented the unemployment does not affect the aggregate
first FYP in the Soviet Union in the late 1920s. output.
Most communist states and several capitalist y Disguised unemployment is a hidden form
countries subsequently have adopted them. of unemployment where the work requiring
China and India both continue to use FYPs. say 10 people, is being done by 15 people.
After independence, India launched its First Here, the marginal productivity that is the
FYP in 1951, under the socialist influence of additional output obtained as a result of
first Prime Minister Jawaharlal Nehru. adding another person to employment is
y The 12th Five-Year Plan commenced at zero as the labour is not adding anything
a time when the global economy was to the output.
going through a second financial crisis, y It is prevalent in the Agricultural sector of
precipitated by the sovereign debt problems India.
of the Eurozone which erupted in the last
year of the Eleventh Plan.
9. Answer: (b)
y The crisis affected all countries, including
India which resulted in a slowdown of growth Statement 1 is not correct: District Rural
to 6.2% in 2011-12. This posed an immediate Development Agency does not act as Panchayati
challenge of bringing the economy back to a Raj Institutions in certain specified backward
higher growth path. Short term downturns regions of the country. It is the principal
occur in all economies. organ at the district level to manage and
oversee the implementation of different anti-
y The 12th, therefore, focused on bringing
poverty programmes of the Ministry of Rural
the economy back to rapid growth while
Development. It is a supporting and facilitating
ensuring that the growth is both inclusive
organization which plays a very effective role
and sustainable. The broad vision and
as a catalyst in the development process.
aspirations which the Twelfth Plan seeks to
fulfill are reflected in the subtitle: ‘Faster, Statement 2 is not correct: DRDAs do not
Sustainable, and more Inclusive Growth’. undertake any area-specific scientific study
Inclusiveness is to be achieved through of the causes of poverty and malnutrition
poverty reduction, promoting group and prepare detailed remedial measures. The
equality and regional balance, reducing main objective of the agency is to identify the
inequality, empowering people etc., families living below the poverty line through a

4C: NITI, Planning Commission, FYP, Unemployment_Explanation 573


unacademy.com | Download the Unacademy app
Give your feedback here: Link
survey conducted by the government servants such as Employment Assurance Scheme.
working at field level. Statement 4 is correct: DRDAs watch over
Statement 3 is correct: DRDAs secure inter- and ensure effective utilization of the funds
sectoral and inter-departmental coordination intended for anti-poverty programmes. DRDAs
and cooperation for effective implementation also give financial assistance to these B.P.L.
of anti-poverty programmes. The agencies draw families through banks and government
up model plans through technical departments, sponsored schemes.
for executing the labour generating schemes

574 4C: NITI, Planning Commission, FYP, Unemployment_Explanation


unacademy.com | Download the Unacademy app
Give your feedback here: Link
14 4D: GDP, GNP

1. Which of the following activities constitute 4. Increase in absolute and per capita real GNP
a real sector in the economy ? (2022) do not connote a higher level of economic
1. Farmers harvesting their crops development, if (2018)
2. Textile mills converting raw cotton into (a) industrial output fails to keep pace with
fabrics agricultural output.

3. A commercial bank lending money to a (b) agricultural output fails to keep pace
trading company with industrial output.

4. A corporate body issuing Rupee (c) poverty and unemployment increase.


Denominated Bonds overseas (d) imports grow faster than exports.
Select the correct answer using the code
given below : 5. A decrease in the tax to GDP ratio of a
(a) 1 and 2 only country indicates which of the following?
(2015)
(b) 2, 3 and 4 only
1. Slowing economic growth rates
(c) 1, 3 and 4 only
2. Less equitable distribution of national
(d) 1, 2, 3 and 4 income
Select the correct answer using the code
2. Which among the following steps is most given below.
likely to be taken at the time of an economic
(a) 1 only
recession? (2021)
(a) Cut in tax rates accompanied by (b) 2 only
increase in interest rate (c) Both 1 and 2
(b)
Increase in expenditure on public (d) Neither 1 nor 2
projects
(c) Increase in tax rates accompanied by 6. With reference to India economy, consider
reduction of interest rate the following statements: (2015)
(d) Reduction of expenditure on public 1. The rate of growth of real Gross Domestic
projects Product has steadily increased in the
last decade.

3. Despite being a high saving economy, 2. The Gross Domestic Product at market
capital formation may not result in prices (in rupees) has steadily increased
significant increase in output due to (2018) in the last decade.
(a) weak administrative machinery Which of the statements given above is/are
(b) Illiteracy correct?

(c) high population density (a) 1 only

(d) high capital-output ratio (b) 2 only


(c) Both 1 and 2
(d) Neither 1 nor 2

4D: GDP, GNP 575


unacademy.com | Download the Unacademy app
Give your feedback here: Link
7. Economic growth in country X will (d) Money value of final goods and services
necessarily have to occur if (2013) produced.
(a) there is technical progress in the world
economy 9. In the context of Indian economy, consider
(b) there is population growth in X the following statements: (2011)
(c) there is capital formation in X 1. The growth rate of GDP has steadily
increased in the last five years.
(d) the volume of trade grows in the world
economy 2. The growth rate in per capita income
has steadily increased in the last five
years.
8. The National Income of a country for a
given period is equal to the: (2013) Which of the statements given above is/are
(a) Total value of goods and services correct?
produced by the nationals. (a) 1 only
(b)
Sum of total consumption and (b) 2 only
investment expenditure. (c) Both 1 and 2
(c) Sum of personal income of all individuals. (d) Neither 1 nor 2

576 4D: GDP, GNP


unacademy.com | Download the Unacademy app
Give your feedback here: Link
14 4D: GDP, GNP_Explanation

1. Answer: (a) which make available capital goods to the


The real sector of the economy deals with the producers like machines, tools etc. This aids
production side, while the nominal economy in further production of goods translating into
deals with the financial side. Financial activities higher output in the economy.
majorly support real(production) activity, Option (d) is correct: Capital-output ratio
(Hence Statement 1 and 2 are correct) but is the amount of capital needed to produce
does not contribute itself too much except the one unit of output. A higher ratio means that
factor income it generates (Hence statements more capital is needed to produce one unit
3 and 4 are not correct). of output due to which the increased capital
formation may not result in significant increase
in output. A higher ratio generally indicates
2. Answer: (b) inefficient deployment of the capital resources
A recession is a slowdown or a massive of a nation.
contraction in economic activities. A significant
fall in spending generally leads to a recession.
Such a slowdown in economic activities may 4. Answer: (c)
last for some quarters, thereby completely Per capita real GNP accurately captures the
hampering the growth of an economy. In such economic growth of a nation by adjusting for
a situation, economic indicators such as GDP, price level changes so that the actual growth
corporate profits, employment, etc., fall. in the production of goods and services can be
Option (b) is correct: This creates a mess in understood. By dividing real GNP on a per capita
the entire economy. To tackle the menace, basis, a clearer comparison can be made with
economies generally react by loosening their other nations having different populations.
monetary policies by infusing more money But, what these measures cannot accurately
into the system, i.e., by increasing the money capture is development. Development
supply. During a recession, the government happens when the fruits of economic growth
can: are translated into increased prosperity and
y Lower interest rates to increase borrowing better standards of living for all the residents
capacity. of a nation.

y Increase government spending. Option (c) is correct: Increasing poverty and


unemployment reflect that the fruits of growth
y Cut tax rates to enhance people’s spending in Real GNP have not adequately spread to
capacity. all. While economic growth is happening for
So, from the given options in the question, the nation in general a section of people is
option (b) is correct. left behind hence a higher level of economic
development is not taking place in the country.

3. Answer: (d)
Under the concept of circular flow of income, 5. Answer: (a)
it is illustrated that the savings from the The tax to GDP ratio is defined as the amount
household sector are subsequently moved of taxes collected as a proportion of the
to the producers in an economy who in turn country’s Gross Domestic Product (GDP).
invest it and enhance the capital formation

4D: GDP, GNP_Explanation 577


unacademy.com | Download the Unacademy app
Give your feedback here: Link
The higher the tax to GDP ratio, the better decade. However, the Financial Year 2020-
the financial position the country will be in. 2021 may be an exception due to the fall
The ratio represents that the government is of GDP caused because of the COVID-19
able to finance its expenditure. It reduces a pandemic. (Statement 2 is correct)
government’s dependence on borrowings.
Statement 1 is correct: A decrease in the tax 7. Answer: (c)
to GDP ratio of a country indicates the slowing
Economic Growth is the increase in the
economic growth rates. As economic activities
production of goods and services in an
slow down, the economic growth rates also
economy over a specified period of time.
slows down, and this results in fall in income,
employment and sales, which directly impacts y Attaining economic growth is a primary
the direct and indirect tax collection, leading purpose behind all economic planning. This
to a lower tax to GDP ratio. can be attained by various means. Capital
formation, that is the acquisition of new or
Statement 2 is not correct: Tax to GDP ratio
existing fixed assets that are intended for
does not uncover the distribution of the income
use in the production of other goods and
across the country. It presents a sum total
services for a period of over one year will
picture of all the tax collected from domestic
lead to an increase in the production of
residents to the GDP added by all the domestic
goods and services in the economy hence
residents.
necessarily leading to economic growth.
Additional Information: Technical progress is nothing but the
y A higher tax to GDP ratio means that an discovery of new and improved methods to
economy’s tax buoyancy is strong as the produce goods. It also helps the intensive
share of tax revenue rises in sync with the utilization of resources. Technological
rise in the country’s GDP. India, despite progress takes place through inventions
seeing higher growth rates, has struggled and innovations, but it does not play any
to widen the tax base. Lower tax-to-GDP direct role to economic growth.
ratio constrains the government to spend y Growth in the volume of global trade, will
on infrastructure and puts pressure on induce economic growth in the world.
the government to meet its fiscal deficit However, this growth may not necessarily
targets. lead to economic growth in country X as it
may not participate in the global economic
systems to that great an extent.
6. Answer: (b)
y Population growth will lead to economic
Gross Domestic Product is the value of all final
growth in country X but only if this
goods and services produced in the domestic
population is productive and adds up to
territory of a country in a year.
the production of goods and services in the
y Real GDP is the GDP of a country at constant economy of country X.
prices. That is, the growth rate of the GDP
has been adjusted to remove the impact of
change in price levels in comparison to the 8. Answer: (d)
base year. In India, the rate of growth in real Option (d) is correct: National Income means
GDP has not steadily increased. (Statement the sum total of Income that everyone earns
1 is not correct) in a nation. As the earning of everyone is
y The Gross Domestic Product at Market dependent or directly related to the production
Prices has steadily increased in the past of some sorts of goods or services in the

578 4D: GDP, GNP_Explanation


unacademy.com | Download the Unacademy app
Give your feedback here: Link
economy, hence the money value of final goods measures of all sources of income in aggregate
and services produced in an economy is the (like GDP) and dividing it by the total population.
most relevant measure of the National Income The per capita income is often calculated to
of a country. measure standard of living in a country. This
way it helps to locate the development status
of the nations.
9. Answer: (d)
y Although the GDP has grown steadily in
Gross Domestic Product (GDP) is the final value the last five years, the growth rate has
of the goods and services produced within the not steadily increased. It has gone up and
geographic boundaries of a country during a down with no definite unidirectional trend.
specified period of time, normally a year. GDP (Statement 1 is not correct)
growth rate is an important indicator of the
economic performance of a country. y The growth rate in per capita incomes
has experienced fluctuation with no
Per capita income: It is the amount of income unidirectional trend. In fact, the per capita
being earned by a person in a certain region. In incomes actually fell on a year-on-year
other words, it is defined as the mean income basis from 2007 to 2008. (Statement 2 is
of people in an economic unit such as a city not correct)
or a country. It can be calculated by taking

4D: GDP, GNP_Explanation 579


unacademy.com | Download the Unacademy app
Give your feedback here: Link
15 4E: Inflation

1. Consider the following statements : (2023) 3. Inflation-indexing wages


Statement-I: 4. Higher purchasing power
In the post-pandemic recent past, many 5. Rising interest rates
Central Banks worldwide had carried out Select the correct answer using the code
interest rate hikes. given below:
Statement-II : (a) 1, 2 and 4 only
Central Banks generally assume that they (b) 3, 4 and 5 only
have the ability to counteract the rising
consumer prices via monetary policy (c) 1, 2, 3 and 5 only
means. (d) 1, 2, 3, 4 and 5
Which one of the following is correct in
respect of the above statements? 4. Consider the following statements: (2020)
(a) Both Statement-I and Statement-II are 1. The weightage of food in Consumer
correct and Statement-II is the correct Price Index (CPI) is higher than that in
explanation for Statement-I Wholesale Price Index (WPI).
(b)
Both Statement-I and Statement-II 2. The WPI does not capture changes in
are correct and Statement-II is not the the prices of services, which CPI does.
correct explanation for Statement-I 3. The Reserve Bank of India has now
(c) Statement-I is correct but Statement-II adopted WPI as its key measure of
is incorrect inflation and to decide on changing the
(d) Statement-I is incorrect but Statement- key policy rates.
II is correct Which of the statements given above is/are
correct?

2. Which one of the following is likely to be (a) 1 and 2 only


the most inflationary in its effects? (2021) (b) 2 only
(a) Repayment of public debt (c) 3 only
(b) Borrowing from the public to finance a (d) 1, 2 and 3
budget deficit
(c) Borrowing from the banks to finance a 5. Which of the following brings out the
budget deficit 'Consumer Price Index Number for
(d) Creation of new money to finance a Industrial Workers'? [2015]
budget deficit (a) The Reserve Bank of India
(b) The Department of Economic Affairs
3. With reference to the Indian economy, (c) The Labour Bureau
demand-pull inflation can be caused /
(d)
The Department of Personnel and
increased by which of the following? (2021)
Training
1. Expansionary policies
2. Fiscal stimulus

580 4E: Inflation


unacademy.com | Download the Unacademy app
Give your feedback here: Link
6. In the Index of Eight Core Industries, which 10. In India, deficit financing is used for raising
one of the following is given the highest resources for: (2013)
weight? (2015) (a) economic development
(a) Coal Production (b) redemption of public debt
(b) Electricity generation (c) adjusting the balance of payments
(c) Fertilizer Production (d) reducing the foreign debt
(d) Steel Production

11. In India the overall Index of Industrial


7. A rise in general level of prices may be Production, the Indices of Eight Core
caused by (2013) Industries have a combined weight of
1. an increase in the money supply 37.90%. Which of the following are among
2. a decrease in the aggregate level of those Eight Core Industries? (2012)
output 1. Cement
3. an increase in the effective demand 2. Fertilizers
Select the correct answer using the code 3. Natural Gas
given below: 4. Refinery products
(a) 1 only 5. Textiles
(b) 1 and 2 only Select the correct answer using the code
(c) 2 and 3 only given below:
(d) 1, 2 and 3 (a) 1 and 5 only
(b) 2, 3 and 4 only
8. Which one of the following is likely to be (c) 1, 2, 3 and 4 only
the most inflationary in its effect? (2013)
(a) Repayment of public debt (d) 1, 2, 3, 4 and 5

(b) Borrowing from the public to finance a


budget deficit 12. Economic growth is usually coupled with
(c) Borrowing from banks to finance a (2011)
budget deficit (a) Deflation

(d) Creating new money to finance a budget (b) Inflation


deficit (c) Stagflation
(d) Hyperinflation
9. Consider the following statements: (2013)
1. Inflation benefits the debtors. 13. A rapid increase in the rate of inflation is
2. Inflation benefits the bondholders. sometimes attributed to the “base effect”.
Which of the statements given above is/are What is “base effect”? (2011)
correct? (a) It is the impact of drastic deficiency in
supply due to failure of crops.
(a) 1 only
(b) It is the impact of the surge in demand
(b) 2 only due to rapid economic growth.
(c) Both 1 and 2 (c) It is the impact of the price levels of
(d) Neither 1 nor 2 previous year on the calculation of

4E: Inflation 581


unacademy.com | Download the Unacademy app
Give your feedback here: Link
inflation rate.
(d) None of the statements (a), (b) and (c)
given above is correct in this context.

582 4E: Inflation


unacademy.com | Download the Unacademy app
Give your feedback here: Link
15 4E: Inflation_Explanation

1. Answer: (a) increase the demand for goods and services,


Statement 1 is correct: Interest Rates Likely it may lead to a sharp rise in inflation if the
to Return Toward Pre-Pandemic Levels When economic output fails to support demand. In
Inflation is Tamed. Real interest rates have turn, there will be a sharp increase in prices
rapidly increased recently as monetary policy of existing goods and services as the demand
has tightened in response to higher inflation. will rise, but supply won’t. Simply put, the
Whether this uptick is temporary or partly problem with printing money for emerging and
reflects structural factors is an important poorer economies is a sharp rise in inflation —
question for policymakers. something that could cause more harm than
good. Another problem with printing more
Statement 2 is correct: Central banks money is a decline in currency value due to
use monetary policy to manage economic higher inflation.
fluctuations and achieve price stability, which
means that inflation is low and stable. Central
banks in many advanced economies set explicit 3. Answer: (a)
inflation targets. Many developing countries Demand-pull inflation is the upward pressure
also are moving to inflation targeting. on prices that follows a shortage in supply, a
condition that economists describe as “too
2. Answer: (d) many dollars chasing too few goods.”

Inflation is the rate at which the value of Option (a) is correct: Demand-pull inflation
a currency is falling and, consequently, the can be caused or increased by:
general level of prices for goods and services y Expansionary policies
is rising. Inflation is sometimes classified into y Fiscal stimulus
three types: Demand-Pull inflation, Cost-Push
y Higher purchasing power
inflation, and Built-In inflation.
Interest rates tend to move in the same
The financing of deficits is fundamentally
direction as inflation but with lags because
inflationary. The threat of inflation looms
interest rates are the primary tool used by
big since deficit financing raises aggregate
central banks to manage inflation.
expenditure and, as a result, increases
aggregate demand. Rising interest rates will discourage consumers
from borrowing money, preventing demand-
Option (d) is correct: The flow of money in
pull inflation. The standard tool of central banks
the economy is increased by printing new
to address “demand-pull” inflation has been
currency notes. As a result, inflationary
to raise interest rates. By making it costlier
pressures develop, resulting in higher costs for
for firms and households to borrow, demand
products and services in the country. It is most
for investment and consumption goods along
inflationary in nature.
with workers comes down, thereby cooling an
Printing new currency notes increases the overheated economy.
flow of money in the economy. This leads to an
Inflation-indexing wages mean that wages in
increase in inflationary pressures, which leads
the economy are dependent on inflation. So,
to the rise in the prices of goods and services
wages in an economy change w.r.t. to change
in the country.
in inflation changes. So effectively, it will not
While additional money printing is likely to

4E: Inflation_Explanation 583


unacademy.com | Download the Unacademy app
Give your feedback here: Link
cause any change to demand-pull inflation. Option (c) is correct: The Consumer Price
index (CPI) for Industrial Workers is released
by the Labour Bureau, Ministry of Labour
4. Answer: (a) and Employment. CPI for Industrial Workers
Consumer Price Index (CPI) is a measure measures the changes in prices over a period
of change in the retail prices of goods and of time for a specified basket of commodities
services consumed by a defined population consumed by the Industrial Workers.
group in a given area. This index is an important
economic indicator and is widely considered as
a barometer of inflation, a tool for monitoring 6. Answer: (b)
price stability and as a deflator in national The Index of Eight Core Industries is released by
accounts. the Office of the Economic Adviser, Department
Wholesale Price Index (WPI) represents the for Promotion of Industry and Internal Trade
price of goods at a wholesale stage i.e., goods under the Ministry of Commerce and Industry.
that are sold in bulk and traded between This is a monthly index that reflects the
organizations instead of consumers. The index individual and cumulative performance of eight
basket of the WPI covers commodities falling selected sectors of the economy.
under the three major groups, namely Primary Option (b) is correct: In the Index of Eight Core
Articles, Fuel and Power and Manufactured Industries, refinery products hold the highest
products. weight (28.04%). But in the given option,
Statement 1 is correct: The weightage of food electricity generation holds the highest weight
in Consumer Price Index (CPI) is nearly 50% (19.85%) as compared with coal production,
which is more than in Wholesale Price Index fertilizer production and steel production in
(WPI). this index.
Statement 2 is correct: CPI captures the The weights of the Eight Core industries are:
changes in price levels of both goods and Coal (10.33%), Crude Oil production (8.98%),
services however WPI only captures changes in Natural Gas (6.88%), Refinery Products
goods and not in services. (28.04%), Fertilizers (2.63%), Steel (17.92%)
and Cement production (5.37%) and Electricity
Statement 3 is not correct: The RBI seeks to
Generation (19.85%).
control inflation at the consumer level and
hence, takes CPI for making changes in key Since these eight industries are the essential
policy rates. ingredient in the functioning of the broader
economy, mapping their health provides a
Elimination Technique fundamental understanding of the state of the
y The RBI uses CPI to measure inflation. economy.
So, Options (c) and (d) can be easily
eliminated.
7. Answer: (d)
y As we know, manufactured goods have
A rise in the general price levels is termed as
more weightage in WPI basket. So, it is
inflation. Prices can be pushed up due to many
obvious that food has more weightage
factors, amongst others some factors are as
in CPI basket. Therefore, we can mark
follows:
Option (a) as the correct answer.
y An increase in the money supply puts more
money in the hands of the consumers as
5. Answer: (c)
a whole, which increases the aggregate
Explanation: demand in the economy without much

584 4E: Inflation_Explanation


unacademy.com | Download the Unacademy app
Give your feedback here: Link
change in the aggregate supply of goods 9. Answer: (a)
and services. Due to which these goods and Inflation refers to the rise in the prices of most
services become comparatively scarce, and goods and services of daily or common use,
their prices rise. (Statement 1 is correct) such as food, clothing, housing, recreation,
y A decrease in the aggregate level of output, transport, consumer staples, etc. Inflation
again pushes the supply below what it was measures the average price change in a basket
earlier, given that the aggregate demand of commodities and services over time. The
remains constant. In this case the price rise opposite and rare fall in the price index of this
is due to the excess of aggregate demand basket of items is called ‘deflation’. Inflation
over the aggregate supply. (Statement 2 is is indicative of the decrease in the purchasing
correct) power of a unit of a country’s currency. This is
y A rise in the general level of prices may measured in percentage.
also be caused by an increase in effective y During inflation debtors gain and creditors
demand. Effective demand refers to the lose. When prices rise, the value of money
willingness and ability of consumers to falls. Though debtors return the same
purchase goods at different prices. It shows amount of money, they pay less in terms of
the amount of goods that consumers are value of money. Thus, inflation brings about
actually buying – supported by their ability a redistribution of real wealth in favour of
to pay. With the increase in potential debtors at the cost of creditors. (Statement
buying capacity, effective consumer 1 is correct)
demand increases, which push the general y Bondholders are a form of lenders, and
price levels of goods and services to go up. hence they tend to lose because of inflation.
(Statement 3 is correct) Inflation makes interest rates go up, in turn
making bond values go down. (Statement 2
8. Answer: (d) is not correct)

Option (d) is correct: Inflation is a sustained


increase in the general price levels of goods 10. Answer: (a)
and services in the economy over a period. Deficit financing means generating funds to
It severely impacts the purchasing power of finance the deficit which results from excess
money. expenditure over revenue. The gap is being
y Creating or printing new money to finance a covered by borrowing from the public by the
budget deficit directly increases the money sale of bonds or by printing new money.
supply in the market. Due to the availability Option (a) is correct: Deficit financing can be
of more money in the market, the aggregate justified to the extent that it helps to stimulate
demand picks up, which may further lead rapid economic development. For developing
to the rise in general price levels of goods countries like India, higher economic growth is
and services. a priority. A higher economic growth requires
y Borrowing from the public to finance a finances. With the private sector being shy of
budget deficit and borrowing from banks making huge expenditures, the responsibility
to finance a budget deficit will decrease of drawing financial resources rests on the
the money supply in the economy and government. Often both the tax and non-tax
control the rise in general price levels of revenues fail to mobilize enough resources
the economy. just through taxes. The deficit is often funded
through borrowings or printing new currency
notes. However, deficit spending without any

4E: Inflation_Explanation 585


unacademy.com | Download the Unacademy app
Give your feedback here: Link
limit is dangerous to the economy as a whole. to higher market prices for their produce.
Additional Information: This stimulates them to produce more and, in
the process, causes economic growth due to
y Deficit financing by printing new currency
more production of goods and services in the
notes increases the flow of money in
economy.
the economy. This leads to increase in
inflationary pressures which leads to a Additional Information:
rise in prices of goods and services in y Deflation: When the overall price level
the country. Deficit financing is inherently decreases so that inflation rate becomes
inflationary. Since deficit financing negative, it is called deflation. A reduction
raises aggregate expenditure and, hence, in money supply or credit availability is the
increases aggregate demand, the danger of reason for deflation in most cases. Reduced
inflation looms large. investment spending by the government or
y Deficit financing affects investment individuals may also lead to this situation.
adversely. When there is inflation in the Deflation leads to a problem of increased
economy employees demand higher wages unemployment due to slack in demand.
to survive. If their demands are accepted, it y Stagflation: Stagflation is a combination
increases the cost of production which de- of persistent high inflation with high
motivates the investors unemployment and stagnant demand in a
country’s economy.

11. Answer: (c) y Hyperinflation: It describes rapid, excessive,


and out-of-control general price increases
The Index of eight core industries is a monthly
in an economy. Hyperinflation is when the
production volume index compiled and
prices of goods and services rise more than
released by Office of the Economic Adviser
50% per month.
(OEA), Department of Industrial Policy &
Promotion (DIPP), Ministry of Commerce &
Industry, Government of India. 13. Answer: (c)
Option (c) is correct: The objective of the Option (c) is correct: The base effect refers to
index is to provide an advance indication on the impact of the rise in price level (i.e., last
production performance of industries of ‘core’ year’s inflation) in the previous year over the
nature before the release of Index of Industrial corresponding rise in price levels in the current
Production (IIP) by the Central Statistics Office. year (i.e., current inflation).
These industries are likely to impact on general y If the price index had risen at a high rate
economic activities as well as industrial in the corresponding period of the previous
activities. The index measures collective year leading to a high inflation rate, some
and individual performance of production in of the potential rises is already factored in,
selected eight core industries viz. Coal, Crude therefore a similar absolute increase in the
Oil, Natural Gas, Petroleum Refinery Products, Price index in the current year will lead to a
Fertilizers, Steel, Cement and Electricity. relatively lower inflation rate.
y On the other hand, if the inflation rate was
12. Answer: (b) too low in the corresponding period of the
Option (b) is correct: Economic growth is previous year, even a relatively smaller rise
usually coupled with Inflation, that is the in the Price Index will arithmetically give a
general rise in price levels, as inflation makes high rate of current inflation.
production more attractive for suppliers, due

586 4E: Inflation_Explanation


unacademy.com | Download the Unacademy app
Give your feedback here: Link
16 5A: Infra: Energy

1. With reference to India's projects on (d) Statement-I is incorrect but Statement-


connectivity, consider the following II is correct
statements : (2023)
1. East-West Corridor under Golden 3. In India, what is the role of the Coal
Quadrilateral Project connects Controller's Organization (CCO)? (2022)
Dibrugarh and Surat. 1. CCO is the major source of Coal
2. Trilateral Highway connects Moreh in Statistics in Government of India.
Manipur and Chiang Mai in Thailand via 2. It monitors progress of development of
Myanmar. Captive Coal/Lignite blocks.
3. Bangladesh-China -India -Myanmar 3. It hears any objection to the
Economic Corridor connects Varanasi in Government's notification relating to
Uttar Pradesh with Kunming in China. acquisition of coal - bearing areas.
How many of the above statements are 4. It ensures that coal mining companies
correct? deliver the coal to end users in the
(a) Only one prescribed time.

(b) Only two Select the correct answer using the code
given below:
(c) All three
(a) 1, 2 and 3
(d) None
(b) 3 and 4 only
(c) 1 and 2 only
2. Consider the following statements : (2023)
(d) 1, 2 and 4
Statement-I :
India, despite having uranium deposits,
4. Consider the following minerals: [2020]
depends on coal for most of its electricity
1. Bentonite
production.
2. Chromite
Statement-II :
3. Kyanite
Uranium, enriched to the extent of at least
60%, is required for the production of 4. Sillimanite
electricity. In India, which of the above is/are officially
Which one of the following is correct in designated as major minerals?
respect of the above statements? (a) 1 and 2 only
(a) Both Statement-I and Statement-II are (b) 4 only
correct and Statement-II is the correct (c) 1 and 3 only
explanation for Statement-I
(d) 2, 3 and 4 only
(b)
Both Statement-I and Statement-II
are correct and Statement-II is not the
correct explanation for Statement-1 5. Consider the following statements: (2019)
1. Coal sector was nationalized by the
(c) Statement-I is correct but Statement-II Government of India under Indira
is incorrect

5A: Infra: Energy 587


unacademy.com | Download the Unacademy app
Give your feedback here: Link
Gandhi. go before the Appellate Tribunals of
2. Now, coal blocks are allocated on lottery Electricity.
basis. Which of the statements given above is/are
3. Till recently, India imported coal to correct?
meet the shortages of domestic supply, (a) 1 and 2 only
but now India is self-sufficient in coal (b) 2 and 3 only
production.
(c) 1 and 3 only
Which of the statements given above is/are
correct? (d) 1,2 and 3

(a) 1 only
8. With reference to solar power production
(b) 2 and 3 only in India, consider the following statements:
(c) 3 only (2018)
(d) 1, 2 and 3 1. India is the third largest in the world in
the manufacture of silicon wafers used
in photovoltaic units.
6. With reference to the management of minor
minerals in India, consider the following 2. The solar power tariffs are determined
statements : [2019] by the Solar Energy Corporation of India.
1. Sand is a 'minor mineral' according to Which of the statements given above is/are
the prevailing law in the country. correct?
2. State Governments have the power to (a) 1 only
grant mining leases of minor minerals, (b) 2 only
but the powers regarding the formation
of rules related to the grant of minor (c) Both 1 and 2
minerals lie with the Centre Government. (d) Neither 1 nor 2
3. State Governments have the power to
frame rules to prevent illegal mining of 9. Which one of the following is the purpose
minor minerals. of ‘UDAY’, a scheme of the Government?
Which of the statements given above is/are (2016)
correct? (a) Providing technical and financial
assistance to start-up entrepreneurs in
(a) 1 and 3 only the field of renewable sources of energy
(b) 2 and 3 only (b) Providing electricity to every household
(c) 3 only in the country by 2018
(d) 1, 2 and 3 only (c) Replacing the coal-based power plants
with natural gas, nuclear, solar, wind
7. Consider the following statements : (2019) and tidal power plants over a period of
1. Petroleum and Natural Gas Regulatory time
Board (PNGRB) is the first regulatory (d) Providing for financial turnaround and
body set up by the Government of India. revival of power distribution companies
2. One of the tasks of PNGRB is to ensure
competitive markets for gas. 10.
What is/are the purpose/purposes of
3. Appeals against the decisions of PNGRB District Mineral Foundations in India?
(2016)

588 5A: Infra: Energy


unacademy.com | Download the Unacademy app
Give your feedback here: Link
1. Promoting mineral exploration activities (b) 2 and 3 only
in mineral-rich districts (c) 1 and 3 only
2. Protecting the interests of the persons (d) 1, 2 and 3
affected by mining operations
3. Authorising State Governments to issue 12. If the National Water Mission is properly
licences for mineral exploration and completely implemented, how will it
Select the correct answer using the code impact the country? (2012)
given below. 1. Part of the water needs of urban areas
(a) 1 and 2 only will be met through recycling of waste
water.
(b) 2 only
2. The water requirements of coastal cities
(c) 1 and 3 only
with inadequate alternative sources
(d) 1, 2 and 3 of water will be met by adopting
appropriate technologies that allow for
11. Despite having large reserves of coal, why the use of ocean water.
does India import millions of tonnes of 3. All the rivers of Himalayan origin will be
coal? (2012) linked to the rivers of peninsular India
1. It is the policy of India to save its own
4. The expenses incurred by farmers for
coal reserves for future and import it
digging bore wells and for installing
from other countries for the present
motors and pump sets to draw
use.
groundwater will be completely
2. most of the power plants in India are reimbursed by the Government.
coal based and they are not able to get
Select the correct answer using the code
sufficient supplies of coal from within
given below:
the country?
(a) 1 only
3. Steel companies need large quantity of
coking coal which has to be imported (b) 1 and 2 only

Which of the statements given above is/are (c) 3 and 4 only


correct? (d) 1, 2, 3 and 4
(a) 1 only

5A: Infra: Energy 589


unacademy.com | Download the Unacademy app
Give your feedback here: Link
16 5A: Infra: Energy_Explanation

1. Answer: (d) made the statistical authority with respect to


Statement 1 is not correct: East–West corridor coal and lignite statistics. Entrusted with the
under Golden Quadrilateral Project connects responsibility of carrying out Annual Coal &
Silchar and Porbandar. Lignite survey and publishing of Provisional
Coal Statistics and Coal Directory of India.
Statement 2 is not correct: India–Myanmar–
Thailand Trilateral Highway (IMT Highway), is a Statement 2 is correct: It is entrusted with
highway under upgrade under India’s Look East the task of monitoring captive mines. Work
policy that will connect Moreh, India with Mae such as permission for opening and reopening
Sot, Thailand via Myanmar. of coal mines has also been entrusted to CCO.

Statement 3 is not correct: The Bangladesh, Statement 3 is correct: Under Coal Bearing
China, India and Myanmar Economic Corridor Area (Acquisition and Development) Act, 1957-
(BCIM) is a proposed corridor connecting India Coal Controller is the competent authority
and China through Myanmar and Bangladesh under this act to hear any objection to the
as a corridor. Central Government’s Notification relating to
acquisition of coal bearing land and to furnish
The corridor would run from Kunming to his reports to Central Govt.
Kolkata, linking Mandalay in Myanmar as well
as Dhaka and Chittagong in Bangladesh. Statement 4 is not correct: Ensuring that coal
mining companies deliver the coal to end users
in the prescribed time is not the function of
2. Answer: (c) the Coal Controller’s Organization (CCO).
Statement 1 is correct:Both conventional
(thermal, nuclear, and hydro) and renewable 4. Answer: (d)
(wind, solar, biomass, etc.) energy sources are
used to produce electricity in India. However, Explanation:
the majority of electricity is produced by coal- Bentonite is essentially a highly plastic clay
fired thermal power plants, which provide containing not less than 85% clay mineral,
about 58% of the total electricity. In the year montmorillonite. It gets its name from Fort
2021–2022, approximately 3.15 per cent of Benton, America, the place where its presence
the nation’s electricity was generated using and usages were first discovered. The
nuclear energy. Government of India declared Bentonite as
Statement 2 is not correct: Commercially, the ‘Minor Mineral’.
U 235 isotope is enriched to 3 to 5% (from Option (d) is correct: Chromite (Cr) is the single
the natural state of 0.7%) and is then further commercially viable ore of chromium which is
processed to create nuclear fuel. chemically known as iron chromium oxide (Fe
Cr2O4). Kyanite, Sillimanite and andalusite are
anhydrous aluminosilicate minerals that have
3. Answer: (a) the same chemical formula Al2O3 but differ
Explanation: in crystal structure and physical properties.
Statement 1 is correct: The Coal Controller’s Chromite, kyanite, and Sillimanite are
Organisation (CCO) is an attached office under designated as major minerals.
the Ministry of Coal. Under Collection of
Statistics Act, 2008 Coal Controller has been

590 5A: Infra: Energy_Explanation


unacademy.com | Download the Unacademy app
Give your feedback here: Link
5. Answer: (a) State Governments have indeed prescribed
rules for the grant of mineral concessions in
Coal sector was in private hands in the British
respect of minerals classified as minor minerals
Era but after independence the sector went
under the MMDR Act, 1957. Minor Minerals get
under government control. Nationalization of
specified in the schedule appended in Minor
the coal industry in India in the early seventies
Mineral concession Rules issued by States.
was a fall out of two related events. In the first
instance, it was the oil price shock which led Statement 3 is correct: As opposed to major
the country to take up a close scrutiny of its minerals, the regulatory and administrative
energy options. Secondly, the much-needed jurisdiction of minor minerals falls under the
investment for the growth of this sector. purview of State governments. These include
the powers to frame rules, prescribe rates
Statement 1 is correct: Coal sector
of royalty, contribution to District Mineral
nationalization was done in two phases from
Foundation, the procedure for grant of mineral
1971- 1973. The Coal Mines (Nationalization)
concessions, regulation of their mining, control
Act, 1973 was enacted during the Prime
of illegal mining etc.
Ministership of Indira Gandhi.
Statement 2 is not correct: Presently, coal
blocks are allocated through auctions and not 7. Answer: (b)
on lottery basis. Explanation:
Statement 3 is not correct: India imports coal Statement 1 is not correct: The Petroleum
of both coking and non-coking variety to meet and Natural Gas Regulatory Board (PNGRB)
the shortages in domestic supply. However, was established under the Petroleum and
India is not yet self-sufficient in coal production Natural Gas Regulatory Board Act, 2006 of the
despite having the world’s fifth largest coal Parliament. It is not the first regulatory body
reserves and continues to import it. of the Government of India.
Additional Information: Statement 2 is correct: The Act provides
Recently, the Government of India allowed for the establishment of Petroleum and
Commercial extraction of coal by the private Natural Gas Regulatory Board to protect the
sector to boost investment in the sector and interests of consumers and entities engaged
to make India self-reliant for meeting its coal in specified activities relating to petroleum,
demand. petroleum products and natural gas and to
promote competitive markets and for matters
connected therewith or incidental thereto.
6. Answer: (a)
Statement 3 is correct: Subject to the
Explanation: provisions of the Act, the Appellate Tribunal
Statement 1 is correct: As per the Mines and established under section 110 of the Electricity
Minerals (Development and Regulation) Act, Act, 2003 shall be the Appellate Tribunal for
1957 “Minor Minerals” means building stones, the purposes of this Act and the said Appellate
gravel, ordinary clay, ordinary sand other than Tribunal shall exercise the jurisdiction, powers
sand used for prescribed purposes. and authority conferred on it by or under this
Statement 2 is not correct: The power to frame Act.
policy and legislation relating to minor minerals
is entirely delegated to the State Governments 8. Answer: (d)
while policy and legislation relating to the
Silicon is a chemical element and a
major minerals are dealt by the Ministry of
semiconductor that is economically important
Mines under the Central Government. Various

5A: Infra: Energy_Explanation 591


unacademy.com | Download the Unacademy app
Give your feedback here: Link
worldwide due to its industrial uses in 10. Answer: (b)
steel refining, aluminium-casting, chemical District Mineral Foundations are statutory
processes, and perhaps as it is most widely bodies in India established by the State
known, in semiconductor electronics. Silicon Governments by notification as a trust or non-
wafers are thin slices of semiconductor that profit body in the mining operation affected
are used for the fabrication of integrated districts. They derive their legal status from
circuits, and to manufacture solar cells. section 9B of Mines and Minerals (Development
Statement 1 is not correct: China is the world’s and Regulation) Act, 1957 as amended in March
largest silicon producer, with a production 2015 as Mines and Minerals (Development and
volume estimated at 5.4 million metric tons Regulation) Amendment Act, 2015.
in 2020. The second largest producer of this Option (b) is correct: The purpose of District
metalloid in the world is Russia, Norway being Mineral Foundation is to work for the interest
third followed by the United States and brazil. of the benefit of the persons and areas affected
Every solar panel which is made in India is mining related operations in such manner as
assembled while all the material comes from may be prescribed by the State Government.
China, Europe and some other countries
Additional Information:
Statement 2 is not correct: Central Electricity
Regulatory Commission determines power y Composition and Functions of the DMF
tariffs including solar power tariffs in India and is prescribed by the State Governments
regulates the tariff of generating companies taking guidelines from Article 244 of Indian
owned or controlled by the Central Government. Constitution, fifth and sixth schedules,
Solar Energy Corporation of India has a power- Provisions of the Panchayats (Extension
trading licence, but it does not set solar power to the Scheduled Areas) Act, 1996 and the
tariffs. Scheduled Tribes and Other Traditional
Forest Dwellers (Recognition of Forest
Rights) Act, 2006.
9. Answer: (d) y Funds every mining lease holder of will pay
Option (d) is correct: Ujwal DISCOM Assurance a fraction of royalty, not exceeding one-
Yojana (UDAY) was launched by the Ministry third of the royalty, to the DMF as per rates
of Power (Government of India) in 2015. The prescribed by Central Government. This
scheme envisages: fund will be used for welfare of the people
y Financial Turnaround affected in the mining affected areas.

y Operational Improvement
y Reduction of Cost of Generation of Power 11. Answer: (b)

y Development of Renewable Energy Statement 1 is not correct: India is the second


largest coal importer, despite having the
y Energy Efficiency and Conservation
world’s fourth largest coal reserves and being
Under the scheme, the state government took the second largest producer. India still imports
over 75% of the outstanding debt of their coal because the coal sector in India is state
distribution companies and issued low-interest owned and runs a monopoly business. The
bonds to finance the rest of the outstanding state-owned Coal India limited alone is not
debt. The government had also announced capable of producing the volume of coal that
UDAY 2.0 to minimize DISCOM losses and revive is required for domestic consumption, which
the sector further. creates a huge demand-supply mismatch.
Therefore, to meet the timely domestic demand
India imports coal. It is not the policy of India

592 5A: Infra: Energy_Explanation


unacademy.com | Download the Unacademy app
Give your feedback here: Link
to save its own coal reserves for future and
import it from other countries for the present and (d) can be eliminated. We can mark
use option (b) as the correct answer.

Statement 2 is correct: Despite private sector


participation in a restricted manner, state 12. Answer: (b)
run Coal India Ltd continued to dominate The National Water Mission was launched in
the market (but with inefficiency). During the 2011 with the purpose of water conservation,
high growth years of 2000s, the increasing minimization of wastage and managing water
demand of Coal could not be fulfilled by the resources in such a way that there is an equal
state-run Coal India Ltd., leading to higher distribution of water across the nation.
demand-supply gap. Most of the power plants Option (b) is correct: The objectives of the
in India are coal based and they are not able mission are:
to get sufficient supplies of coal from within
y Increase water use efficiency by 20% through
the country. This is one of the reasons India
regulations, differential entitlements and
imports coal.
pricing.
Statement 3 is correct: The country’s coal
y A considerable share of water needs of
production has increased from 431 MT in
urban areas is to be met through recycling
2006-07 to 554 MT in 2011-12 (an increase of
of wastewater.
28.5%). On the other hand, the demand for
coal has grown at a CAGR of more than 7% y Water requirements of coastal cities are
in the last decade and has reached around to be met through the adoption of low-
600 MT. This demand mainly arises from the temperature desalination technologies.
side of Steel companies. The prevailing supply y Consult with states to ensure that basin-
from Coal India was not sufficient to meet the level management strategies are made to
demand, therefore there was huge demand for deal with variability in rainfall and river
coal imports. flows due to climate change.
Elimination Technique y Enhance storage above and below ground,
implement rainwater harvesting.
y There is no such policy in India to save
its coal reserves. India imports coal y Adopt large scale irrigation programs which
only because of its shortage of coal rely on sprinklers, drip irrigation and ridge
extraction capacity. So, option (a), (c), and furrow irrigation.

5A: Infra: Energy_Explanation 593


unacademy.com | Download the Unacademy app
Give your feedback here: Link
5B: Infra: Transport, Urban Rural,
17 Communication, Investment, PPP
1. Consider the following statements : (2023) (c) Only three
Statement-I : (d) All four
Interest income from the deposits in
Infrastructure Investment Trusts 3. Which one of the following statements
(InvITs) distributed to their investors are best describes the 'Polar Code'? (UPSC-
exempted from tax, but the dividend is 2022)
taxable. (a) It is the international code of safety for
ships operating in polar waters.
Statement-II :
(b) It is the agreement of the countries
InvITs are recognized as borrowers under
around the North Pole regarding the
the 'Securitization and Reconstruction
demarcation of their territories in the
of Financial Assets and Enforcement of
polar region.
Security Interest Act, 2002'.
(c) It is a set of norms to be followed by the
Which one of the following is correct in
countries whose scientists undertake
respect of the above statements?
research studies in the North Pole and
(a) Both Statement-I and Statement-II are South Pole.
correct and Statement-II is the correct
(d) It is a trade and security agreement
explanation for Statement-I
of the member countries of the Arctic
(b)
Both Statement-I and Statement-II Council.
are correct and Statement-II is not the
correct explanation for Statement-I
4. Consider the following statements: (UPSC-
(c) Statement-I is correct but Statement-II 2022)
is incorrect 1. The India Sanitation Coalition is a
(d) Statement-I is incorrect but Statement- platform to promote sustainable
II is correct sanitation and is funded by the
Government of India and the World
Health Organization.
2. Consider the following infrastructure
2. The National Institute of Urban Affairs is
sectors : (2023)
an apex body of the Minister of Housing
1. Affordable housing and Urban Affairs in Government of
2. Mass rapid transport India and provides innovative solutions
to address the challenges of Urban
3. Health care
India.
4. Renewable energy
Which of the statements given above is/are
How many of the above does UNOPS correct?
Sustainable Investments in Infrastructure
(a) 1 only
and Innovation (S3i) initiative focus on its
investments? (b) 2 only

(a) Only one (c) Both 1 and 2

(b) Only two (d) Neither 1 nor 2

594 5B: Infra: Transport, Urban Rural, Communication, Investment, PPP


unacademy.com | Download the Unacademy app
Give your feedback here: Link
5. With reference to the funds under Members (b) 3 only
of Parliament Local Area Development (c) 2 and 3 only
Scheme(MPLADS), which of the following
statements are correct? [2020] (d) 1, 2 and 3
1. MPLADS funds must be used to
create durable assets like physical 7. With reference to 'National Investment and
infrastructure for health, education etc., Infrastructure Fund', which of the following
2. A specified portion of each MP’s fund statements is/are correct? [2017]
must benefit SC/ST populations 1. It is an organ of NITI Aayog.

3. MPLADS funds are sanctioned on yearly 2. It has a corpus of ` 4, 00,000 crore at


basis and the unused funds cannot be present.
carried forward to the next year. Select the correct answer using the code
4. The district authority must inspect given below:
at least 10% of all works under (a) 1 only
implementation every year (b) 2 only
Select the correct answer using the code (c) Both 1 and 2
given below:
(d) Neither 1 nor 2
(a) 1 and 2 only
(b) 3 and 4 only 8. The Global Infrastructure Facility is a/an
(c) 1, 2 and 3 only [2017]
(d) 1, 2 and 4 only (a)
ASEAN initiative to upgrade
infrastructure in Asia and financed by
credit from the Asian Development
5. Which of the following is/are the aim/aims Bank.
of “Digital India” Plan of the Government of
India? (Pre18 Set-D) (b) World Bank collaboration that facilitates
1. Formation of India’s own Internet the preparation and structuring
companies like China did. of complex infrastructure Public-
Private Partnerships (PPPs) to enable
2. Establish a policy framework to mobilization of private sector and
encourage overseas multinational institutional investor capital.
corporations that collect Big Data to
build their large data centres within our (c) Collaboration among the major banks
national geographical boundaries. of the world working with the OECD
and focused on expanding the set of
3. Connect many of our villages to the infrastructure projects that have the
Internet and bring Wi-Fi to many of our potential to mobilize private investment.
school, public places and major tourist
centres. (d) UNCTAD funded initiative that seeks
to finance and facilitate infrastructure
Select the correct answer using the code development in the world.
given below
(a) 1 and 2 only

5B: Infra: Transport, Urban Rural, Communication, Investment, PPP 595


unacademy.com | Download the Unacademy app
Give your feedback here: Link
5B: Infra: Transport, Urban Rural,
17 Communication, Investment, PPP_Explanation

1. Answer: (d) in the specified countries by providing support


Statement 1 incorrect: REITs and InvITs make for healthcare infrastructure projects. These
distributions to their unitholders in the form of projects aim to address the needs of both
interest, dividend income, and rental income private and public healthcare systems with a
all of which have pass-through status and are particular emphasis on primary care clinics,
taxed in the hands of unitholders. hospitals, diagnostic centers, laboratories, and
the potential implementation of established
Statement 2 correct: With InvITs and REITs healthcare technologies. The goal is to bring
now recognised as borrowers under the about effective and sustainable solutions to
SARFAESI Act, lenders to these trusts, shall meet healthcare demands.
have adequate statutory enforcement options,
absence of which was earlier becoming a Statement 4 is correct: UNOPS S3i initiative
constraint for bankers to lend directly at trust prioritize renewable energy. Within the realm
level. of renewable energy, the primary emphasis lies
in investing in infrastructure projects related
to renewable energy sources, including but
2. Answer: (c) not limited to solar, hydro, wind, biomass,
Established in 1973 as a component of the and hydrogen, particularly in eligible emerging
United Nations Development Programme markets and developing nations. The main
(UNDP), UNOPS gained autonomy and focus is on the generation of renewable
financial independence in 1995. UNOPS’ energy and the accompanying distribution and
overarching vision is to promote sustainable storage infrastructure. However, investments
implementation practices in the realms in innovative technologies and solutions
of development, humanitarian aid, and throughout the renewable energy value chain
peacebuilding.The development of climate- may also be taken into consideration.
resilient and sustainable infrastructure is
crucial across various sectors that play a 3. Answer: (a)
significant role in sustainable development.
Explanation:
Statement 1 is correct: UNOPS S3i initiative
prioritize affordable housing. The objective in Option (a) is correct: Polar code is International
the field of affordable housing is to provide Maritime Organizations (IMOs) Code for Ships
significant support for affordable housing Operating in Polar Waters. It is mandatory
initiatives targeting the middle-income and under both the International Convention for
lower-middle-income groups of the population. the Safety of Life at Sea (SOLAS) and the
The focus is on implementing these projects International Convention for the Prevention of
on a large scale. Pollution from Ships (MARPOL).

Statement 2 is not correct: Mass Rapid The Polar Code covers the full range of
Transport is not included in the UNOPS S3i design, construction, equipment, operational,
initiative. training, search and rescue and environmental
protection matters relevant to ships operating
Statement 3 is correct: UNOPS S3i initiative in the inhospitable waters surrounding the two
prioritize Healthcare. Regarding healthcare, the poles.
objective is to enhance the quality of healthcare

596 5B: Infra: Transport, Urban Rural, Communication, Investment, PPP_Explanation


unacademy.com | Download the Unacademy app
Give your feedback here: Link
4. Answer: (d) and utilization certificate. The funds are non-
lapsable and can be carried forward.
Explanation:
Statement 4 is correct: The District Authority
Statement 1 is not correct: India Sanitation
would be responsible for overall coordination
Coalition (ISC), launched in June 2015, at the
and supervision of the works under the scheme
Federation of Indian Chamber of Commerce
at the district level and inspect at least 10%
and Industry (FICCI), enables and supports safe
of the works under implementation every year.
and sustainable sanitation by bringing multiple
The District Authority should involve the MPs
organizations on a common platform through
in the inspections of projects to the extent
a range of catalytic actions. These include
feasible.
supporting the unlocking of WASH financing
with a focus on the private sector, forging
partnerships with allied organizations for 4. Answer: (b)
leading the discourse on sustainable sanitation;
Explanation:
convening, curating, and disseminating best
practices in the sanitation advocacy — space, Statement 1 is not correct: Digital India is a
and providing inputs into the policy aspects flagship programme of the Government of
of sanitation through participation at allied India with a vision to transform India into a
forums. It is not funded by WHO. digitally empowered society and knowledge
economy. It aims to make government
Statement 2 is not correct: The National
services available to citizens electronically by
Institute of Urban Affairs is an institute
online infrastructure improvement and also by
for research, training, and information
enhancing internet connectivity. It also aims to
dissemination in urban development
empower the country digitally in the domain
of technology.
5. Answer: (d) Statement 2 is not correct: There is no provision
Explanation: of establishing own internet companies and
a policy framework to encourage overseas
Statement 1 is correct: The funds under Members
multinational corporations that collect Big Data
of Parliament Local Area Development Scheme
to build large data centres within our national
have objectives to create durable community
geographical boundaries.
assets and for the provision of basic facilities
including community infrastructure based on Statement 3 is correct: The major objectives of
locally felt needs. this initiative are:
Statement 2 is correct: There is a special focus y To provide high-speed internet in all gram
on areas inhabited by Scheduled Caste and panchayats.
Scheduled Tribe populations by earmarking 15% y To provide easy access to the Common
and 7.5 % of the MPLADS funds respectively. Service Centre (CSC) in all the localities.
Out of an amount of Rs.5 Crores, the M.P. shall
y Restructuring many existing schemes that
recommend for areas inhabited by the S.C.
can be implemented in a synchronized
population, Rs.75 lakhs, and Rs.37.5 lakhs for
manner
areas inhabited by S.T. population.
Statement 3 is not correct: The annual
entitlement of Rs 5 crore shall be released, 5. Answer: (d)
in two equal installments of Rs 2.5 crore Explanation:
each, by Government of India directly to the
National Investment and Infrastructure Fund
District Authority based on unspent amount
Limited (NIIFL) is a collaborative investment

5B: Infra: Transport, Urban Rural, Communication, Investment, PPP_Explanation 597


unacademy.com | Download the Unacademy app
Give your feedback here: Link
platform for international and Indian 6. Answer: (b)
investors, anchored by the Government of Explanation:
India. NIIFL invests across asset classes such
as infrastructure, private equity and other Option (b) is correct: The Global Infrastructure
diversified sectors in India, with the objective Facility (GIF) is a global, open platform that
to generate attractive risk-adjusted returns for facilitates the preparation and structuring
its investors. of complex infrastructure public private
partnerships (PPPs) to enable mobilization of
Statement 1 is not correct: NIIF is not an organ private sector and institutional investor capital.
of NITI Aayog. It has its own governing council The GIF platform coordinates and integrates
chaired by the Finance Minister of India. the efforts of Multilateral Development Banks
Statement 2 is not correct: NIIF has a planned (MDBs), private sector investors and financiers,
corpus of Rs. 40,000 Crore. and governments interested in infrastructure
investment in Emerging Markets and Developing
Economies (EMDEs). The GIF is co-chaired by
the World Bank Group.

598 5B: Infra: Transport, Urban Rural, Communication, Investment, PPP_Explanation


unacademy.com | Download the Unacademy app
Give your feedback here: Link
18 6A: HRD: Census, Health Hunger

1. Consider the following specific stages of are entitled to a take-home ration' of


demographic transition associated with 1600 calories per day during pregnancy
economic development : [2012 - I] and for six months thereafter.
1. Low birthrate with low death rate Which of the statements given above is/are
2. High birthrate with high death rate correct?
3. High birthrate with low death rate (a) 1 and 2
Select the correct order of the above stages (b) 2 only
using the codes given below : (c) 1 and 3
(a) 1, 2, 3 (d) 3 only
(b) 2, 1, 3
(c) 2, 3, 1 4. Which of the following are the objectives
(d) 3, 2, 1 of 'National Nutrition Mission'? [2017]
1. To create awareness relating to
malnutrition among pregnant women
2. The endeavour of ‘Janani Suraksha Yojana’
and lactating mothers.
Programme is [2012 - I]
1. to promote institutional deliveries 2. To reduce the incidence of anemia
among young children, adolescent girls
2. to provide monetary assistance to the
and women.
mother to meet the cost of delivery
3. To promote the consumption of millets,
3. to provide for wage loss due to
coarse cereals and unpolished rice.
pregnancy and confinement
4. To promote the consumption of poultry
Which of the statements given above is/are
eggs.
correct?
Select the correct answer using the code
(a) 1 and 2 only
given below:
(b) 2 only
(a) 1 and 2 only
(c) 3 only
(b) 1, 2 and 3 only
(d) 1, 2 and 3
(c) 1, 2 and 4 only
(d) 3 and 4 only
3. With reference to the provisions made
under the National Food Security Act, 2013
consider the following statements: [2018] 5. Which of the following is/are the indicator/
1. The families coming under the category indicators used by IFPRI to compute the
of 'below poverty line (BPL)' only are Global Hunger Index Report? [2016]
eligible to receive subsidised grains. 1. Under nourishment

2. The eldest woman in a household, of 2. Child stunting


age 18 years or above, shall be the head 3. Child mortality
of the household for the purpose of Select the correct answer using the code
issuance of a ration card given below.
3. Pregnant women and lactating mothers

6A: HRD: Census, Health Hunger 599


unacademy.com | Download the Unacademy app
Give your feedback here: Link
(a) 1 only India’s public sector health care system
(b) 2 and 3 only largely focuses on curative care with limited
preventive, promotive and rehabilitative
(c) 1, 2 and 3 care.
(d) 1 and 3 only Statement-II:
Under India’s decentralised approach to
6. India is regarded as a country with a health care delivery, the States are primarily
“Demographic Dividend”. This is due to responsible for organizing health services.
(2011)
(a) Its high population in the age group Which one of the following is correct in
below 15 years. respect of the above statements?

(b) Its high population in the age group of (a) Both Statement-I and Statement-II are
15-64 years. correct and Statement-II is the correct
explanation for Statement-I
(c) Its high population in the age group
above 65 years. (b)
Both Statement-I and Statement-II
are correct and Statement-II is not the
(d) Its high total population. correct explanation for Statement-I
(c) Statement-I is correct but Statement-II
7. Consider the following statements: (2023) is incorrect
Statement-I: (d) Statement-I is incorrect but Statement-
II is correct

600 6A: HRD: Census, Health Hunger


unacademy.com | Download the Unacademy app
Give your feedback here: Link
6A: HRD: Census, Health Hunger_
18 Explanation
1.Answer: (c) access to adequate quantities of quality food
Explanation: at affordable prices for people to live a life
with dignity.
Demographic transition is a long-term trend
of declining birth and death rates, resulting in Statement 1 is not correct: Act covers both
substantive change in the age distribution of BPL and APL.
a population. There are three basic phass of Statement 2 is correct: As a step towards
demographic transition. women empowerment, the eldest woman
y The first stage is low population growth of the household of age 18 years or above is
in a society that is underdeveloped and mandated to be the head of the household for
technologically backward. the purpose of issuing ration cards under the
Act.
Statement 3 is not correct: It provides 600
2.Answer: (a) calorie and not 1600 calorie.
Explanation:
y Janani Suraksha Yojana (JSY) is a safe 4. Answer: (a)
motherhood intervention under the
National Rural Health Mission (NRHM) Explanation:
being implemented with the objective of Statement 1 is correct: To address the problems
reducing maternal and neo-natal mortality of malnutrition in the country, the government
by promoting institutional delivery among has launched the National Nutrition Mission
the poor pregnant women. (Statement 1 is in 2017. It was to create awareness relating
correct) to malnutrition among pregnant women and
y Janani Suraksha Yojana (JSY) is a 100% lactating mothers.
centrally sponsored scheme and it Statement 2 is correct: The National Nutrition
integrates cash assistance with delivery Mission has been set up under the oversight of
and post-delivery care. (Statement 2 is the Ministry of Women and Child Development
correct) with the aim to reduce anaemia among young
y It also provides Rs.1000. It is an incentive children, adolescent girls and women of
for coming to hospital. The amount is reproductive age (15–49 years).
not large enough to be termed “wage Statement 3 and Statement 4 are not relevant.
compensation”. (Statement 3 is wrong)

5. Answer: (c)
3. Answer: (b) Explanation:
Explanation: The Global Hunger Index report is released
National Food Security Act, 2013 provides a by the International Food Policy Research
legal right to persons belonging to “eligible Institute (IFPRI) annually, to measure hunger
households” to receive food grains at a at the global, regional and country level and
subsidised price. It has the objective to the progress towards tackling hunger.
provide for food and nutritional security in Option (c) is correct: There are four indicators
the human life cycle approach, by ensuring used to compute the Global Hunger Index,

6A: HRD: Census, Health Hunger_Explanation 601


unacademy.com | Download the Unacademy app
Give your feedback here: Link
which are Undernourishment, Child Wasting, 7. Answer: (b)
Child Stunting and Child Mortality. Statement 1 is correct: India’s public sector
health care system does indeed have a
significant focus on curative care, and there
6. Answer: (b)
have been some limitations in terms of
Demographic dividend is understood as the preventive, promotive, and rehabilitative
prospective economic growth that can be care.Historically, India’s public health care
achieved as a result of the changes of the age system has been primarily geared towards
structure of the population of a country, that providing curative services, with an emphasis
is, generally when the working population in on addressing immediate health issues and
a nation becomes more than the dependent providing medical treatment.
population.
Statement 2 is correct: India’s decentralization
Option (b) is correct: India is regarded as a approach to healthcare delivery places primary
country with high Demographic Dividend as it responsibility for organising health services on
has a high population in the working age group the individual states within the country. The
of 15-64 years. Indian healthcare system follows a federal
Additional Information: structure, where the central government and
y Since 2018, India’s working-age population state governments share responsibilities and
(people between 15 and 64 years of age) work together to provide healthcare services
has grown larger than the dependent to the population.Under India’s Constitution,
population — children aged 14 or below healthcare is considered a state subject,
as well as people above 65 years of age. meaning that the state governments have
This bulge in the working-age population is the primary authority and responsibility for
going to last till 2055, or 37 years from its organising and delivering health services within
beginning. their respective states.

602 6A: HRD: Census, Health Hunger_Explanation


unacademy.com | Download the Unacademy app
Give your feedback here: Link
19 6B: HRD: Education and Skill

1. With reference to Pradhan Mantri Kaushal institutions to open their campuses in


Vikas Yojana, consider the following India.
statements [2018] 2. To increase the quality of education
1. It is the flagship scheme of the Ministry provided in government schools by
of Labour and Employment. taking help from the private sector and
2. It, among other things, will also impart the community.
training in soft skills, entrepreneurship, 3. To encourage voluntary monetary
financial and digital literacy. contributions from private individuals
3. It aims to align the competencies of the and organizations so as to improve the
unregulated workforce of the country infrastructure facilities for primary and
to the National Skill Qualification secondary schools.
Framework. Select the correct answer using the code
Which of the statements given above is/are given below:
correct ? (a) 2 only
(a) 1 and 3 only (b) 3 only
(b) 2 only (c) 1 and 2 only
(c) 2 and 3 only (d) 2 and 3 only
(d) 1, 2 and 3
4. What is the aim of the programme 'Unnat
2. Consider the following in respect of Bharat Abhiyan'? [2017]
'National Career Service': [2017] (a) Achieving 100% literacy by promoting
1. National Career Service is an initiative collaboration between voluntary
of the Department of Personnel and organizations and government's
Training, Government of India. education system and local
2. National Career Service has been communities.
launched in a Mission Mode to improve (b)
Connecting institutions of higher
the employment opportunities to education with local communities
uneducated youth of the country. to address development challenges
Which of the above statements is/are through appropriate technologies.
correct? (c) Strengthening India's scientific research
(a) 1 only institutions in order to make India a
scientific and technological power.
(b) 2 only
(d) Developing human capital by allocating
(c) Both 1 and 2
special funds for health care and
(d) Neither 1 nor 2 education of rural and urban poor,
and organizing skill development
3. What is the purpose of Vidyanjali Yojana'? programmes and vocational training for
[2017] them.
1. To enable the famous foreign educational

6B: HRD: Education and Skill 603


unacademy.com | Download the Unacademy app
Give your feedback here: Link
5. ‘SWAYAM’, an initiative of the Government education to the citizens for free
of India, aims at [2016]
(a) promoting the Self Help Groups in rural 6. To obtain the full benefits of demographic
areas dividend, what should India do? (2013)
(b) providing financial and technical (a) Promoting skill development
assistance to young start-up (b)
Introducing more social security
entrepreneurs schemes
(c) promoting the education and health of (c) Reducing infant mortality rate
adolescent girls
(d) Privatisation of higher education
(d)
providing affordable and quality

604 6B: HRD: Education and Skill


unacademy.com | Download the Unacademy app
Give your feedback here: Link
6B: HRD: Education and Skill_
19 Explanation
1. Answer: (c) private sector involvement in schools across
Explanation: the country. This initiative would connect
schools with varied volunteers from the Indian
Statement 1 is not correct: Pradhan Mantri Diaspora namely, young professionals, retired
Kaushal Vikas Yojana is the flagship scheme teachers, retired Government officials, retired
of the Ministry of Skill Development & professionals, NGOs, Private Sector and Public
Entrepreneurship (MSDE) implemented by Sector Companies, Corporate Institutions and
National Skill Development Corporation. The many others.
objective of this Skill Certification Scheme is
to enable a large number of Indian youths to Statement 2 and Statement 3 are not relevant.
take up industry-relevant skill training that will
help them in securing a better livelihood. 4. Answer: (b)
Statement 2 is correct: The training programme Explanation:
under the scheme comes under the National
Option (b) is correct: Unnat Bharat Abhiyan
Skill Qualification Framework (NSQF)
was launched by the Ministry of Human
wherein various training centres provide
Resource Development (MHRD) in 2014. The
skills and training in areas such as soft skills,
Mission (Abhiyan) was inspired by the vision of
entrepreneurship, financial and digital literacy.
transformational change in rural development
Statement 3 is correct: Recognition of Prior processes by leveraging knowledge institutions
Learning is a subcomponent under the to help build the architecture of an Inclusive
scheme wherein the existing competencies India. It intends to link Higher Educational
of the individuals through the way of their Institutions with at least 5 villages to uplift
knowledge/ skill set and experience which their social and economic conditions. Llater,
have been gained either through formal or Unnat Bharat Abhiyan 2.0 was launched in
informal means is assessed and recognised. April, 2018.
As these individuals are generally found to be
Additional Information
working in the informal sector and are largely
unregulated this scheme has helped them to The technological interventions under the UBA
recognise their competencies to the NSQF. cover different subjects broadly categorized as
follows:
y Sustainable agriculture
2. Answer: (d)
y Water resource management
Explanation: it is labour ministry portal so #1
is wrong. It allows educated persons to upload y Artisans, industries and livelihood
biodata so #2 is wrong. So neither1 nor 2 is y Basic amenities (infrastructure & services)
right. and rural energy system

3. Answer: (a) 5. Answer: (d)


Explanation: Explanation:
Vidyanjali is an initiative taken by the Ministry SWAYAM stands for “Study Webs of Active
of Education, Government of India with the aim Learning for Young Aspiring Minds” is basically
to strengthen Schools through community and an integrated MOOCs (massive open online

6B: HRD: Education and Skill_Explanation 605


unacademy.com | Download the Unacademy app
Give your feedback here: Link
course) platform for distance education that is y The ‘demographic dividend’ provides India
aimed at offering all the courses from school with strong advantages for achieving high
(Class 9) to post-graduation level for free of rates of growth powered by an enlarging
cost for resident of India. The platform has working age population. This represents
been developed collaboratively by the erstwhile a huge opportunity for the country to
Ministry of Human Resource Development increase productivity by investing in the
(MHRD) and All India Council for Technical skill development of the working age
Education (AICTE) with the help of Microsoft. population.
Note: In order to obtain a certificate for a y A vast majority of those who have limited
particular course students need to pay a access to education and training; are either
nominal fee. inactive or engaged in low-skilled work in
the informal economy. To capitalise on the
demographic bulge, the Government has
6. Answer: (a) undertaken a number of policy initiatives,
Option (a) is correct: Demographic dividend programmatic and systemic interventions.
refers to the growth in an economy that is y Efforts are being made to equip youth with
the result of a change in the age structure the skills required by today’s industries and
of a country’s population. The change in age address the demand & supply gaps,
structure is typically brought on by a decline in
fertility and mortality rates.

606 6B: HRD: Education and Skill_Explanation


unacademy.com | Download the Unacademy app
Give your feedback here: Link
20 6C: HRD: POVERTY

1. In a given year in India, official poverty lines 3. How does the National Rural Livelihood
are higher in some States than in others Mission seek to improve livelihood options
because: (2019) of rural poor? (2012)
(a) poverty rates vary from State to State. 1. By setting up a large number of
(b) price levels vary from State to State. new manufacturing industries and
agribusiness centres in rural areas
(c) Gross State Product varies from State
to State. 2. By strengthening 'Self-Help Groups' and
providing skill development
(d) quality of public distribution varies from
State. 3. By supplying seeds, fertilizers, diesel
pump-sets and micro-irrigation
equipment free of cost to farmers
2. The Multi-dimensional Poverty Index
developed by Oxford Poverty and Human Select the correct answer using the code
Development Initiative with UNDP support given below:
covers which of the following? (2012) (a) 1 and 2 only
1. Deprivation of education, health, assets (b) 2 only
and services at household level
(c) 1 and 3 only
2. Purchasing power parity at national
level (d) 1, 2 and 3

3. Extent of budget deficit and GDP


growth rate at national level 4. Among the following who are eligible to
Select the correct answer using the code benefit from the “Mahatma Gandhi National
given below: Rural Employment Guarantee Act”? (2011)
(a) Adult members of only the scheduled
(a) 1 only caste and scheduled tribe households
(b) 2 and 3 only (b) Adult members of below poverty line
(c) 1 and 3 only (BPL) households
(d) 1, 2 and 3 (c) Adult members of households of all
backward communities
(d) Adult members of any household

6C: HRD: POVERTY 607


unacademy.com | Download the Unacademy app
Give your feedback here: Link
20 6C: HRD: POVERTY_Explanation

1. Answer: (b) 3. Answer: (b)


Option (b) is correct: In India, the Poverty line is The National Rural Livelihoods Mission
the cut off value in Rupees that a person must (NRLM) was launched by the Ministry of Rural
earn in a month to be classified as being above Development (MoRD), Government of India
the poverty line. The poverty line estimates are in June 2011. The objective of the Mission is
last based upon the recommendations of the to promote sustainable livelihoods for the
Suresh Tendulkar Committee 2011. A standard poor such that they come out of poverty.
basket consisting of the minimum decent The institutions of the poor are intended to
requirement of Food, Education, Health, facilitate access to formal credit; support for
Electricity and Transport is converted in the diversification and strengthening of livelihoods;
terms of expenditure based on the prevailing and access to entitlements and public services.
price levels. As the commodity basket remains The Mission consists of four components, viz.,
same throughout but the prices of this basket social mobilization, community institution and
vary from state to state, hence the differences capacity building; financial inclusion; livelihood
in the official poverty line amongst states. promotion; and convergence.
Option (b) is correct: One of the features of
2. Answer: (a) the National Rural Livelihood Mission is-
The United Nations Development Programme one member (preferably a woman) from
(UNDP) and Oxford Poverty and Human each rural poor household would be brought
Development Initiative release the Multi- under the Self-Help Group (SHG) network.
dimensional Poverty Index (MPI). Started in Women SHG groups would have bank-linkage
2010, it is released in July every year. arrangements. SHGs would be federated at
village level and higher levels to provide space,
Option (a) is correct: The Global voice, and resources and to reduce dependence
Multidimensional Poverty Index is computed on external agencies.
by scoring each surveyed household on ten
parameters based on nutrition, child mortality, Other Major Features are:
years of schooling, school attendance, cooking y Participatory social assessment would
fuel, sanitation, drinking water, electricity, be organised to identify and rank all
housing, and household assets. It defines households according to vulnerability. The
poverty in a broader sense than just on ranking would be with reference to poorest
economic terms. of the poor, single woman and woman-
y In order to be termed poor a person has headed households, disabled, landless,
to be deprived of three or more indicators and migrant labour and they would receive
out of the ten mentioned parameters. special focus.
Only deprivation of education, health, y Training and capacity building of the poor,
assets and services at household level is particularly in relation to managing the
covered under the index out of the above- institutions, livelihoods, credit absorption
mentioned parameters. and creditworthiness.
y The Mission also supports development of
skills for rural youth and their placement,
training, and self-employment through

608 6C: HRD: POVERTY_Explanation


unacademy.com | Download the Unacademy app
Give your feedback here: Link
rural self-employment institutes (RSETIs), institutions and coverage from loss of life,
innovations, infrastructure creation and health etc.
market support.
y Provision of Revolving Fund as a support to 4. Answer: (d)
SHGs to strengthen their institutional and
Option (d) is correct: Mahatma Gandhi National
financial management capacity and build a
Rural Employment Guarantee Act (MGNREGA)
good credit history.
provides a minimum guaranteed 100 days of
y Provision of Community Investment unskilled work annually to adult members of
Support Fund (CIF) in the intensive blocks any rural household who have agreed to do so.
to the SHGs through the Federations to
MGNREGA is an Indian labour law and social
advance loans and/or undertake common/
security measure that aims to guarantee the
collective socio-economic activities.
‘right to work’. It has generated more rural
y Introduction of financial inclusion model, employment than any other government
loaning from banks, association and scheme or private initiative in the history of
coordination with banking/financial independent India.

6C: HRD: POVERTY_Explanation 609


unacademy.com | Download the Unacademy app
Give your feedback here: Link
21 6D: HRD: Weaker Section, HDI, SDG

1. Consider the following statements about 3. accumulation of tangible wealth.


particularly Vulnerable Tribal Groups 4. accumulation of intangible wealth
(PVTGs) in India: [2019 - I]
1. PVTGs reside in 18 states and one union Which of the statements given above is/are
Territory. correct?

2. A stagnant or declining population is (a) 1 and 2 only


one of the criteria for determining PVTG (b) 2 only
status. (c) 2 and 4 only
3. There are 92 PVTGs officially notified in (d) 1, 3 and 4 only
the country so far.
4. Irular and Konda Reddi tribes are 4. Which of the following can be said to
included in the list of PVTGs. be essentially the parts of ‘Inclusive
Which of the statements given above are Governance’? (2012)
correct? 1. Permitting the Non-Banking Financial
(a) 1,2 and 3 Companies to do banking

(b) 2,3 and 4 2. Establishing effective District Planning


Committees in all the districts
(c) 1, 2 and 4
3. Increasing the government spending on
(d) 1,3 and 4 public health
4. Strengthening the Mid-day Meal Scheme
2. In the context of any country which one of
the following would be considered as part Select the correct answer using the code
of its social capital? [2019] given below:
(a) The proportion of literates in the (a) 1 and 2 only
population. (b) 3 and 4 only
(b)
The stock of its buildings, other (c) 2, 3 and 4 only
infrastructure and machines.
(d) 1, 2, 3 and 4
(c) The size of the population in the working
age group.
5. Which of the following can aid in furthering
(d) The level of mutual trust and harmony the Government’s objective of inclusive
in the society. growth? (2011)
1. Promoting Self-Help Groups
3. Human capital formation as a concept 2. Promoting Micro, Small and Medium
is better explained in terms of a process Enterprises
which enables (2018)
3. Implementing the Right to Education
1. individuals of a country to accumulate
Act
more capital.
Select the correct answer using the code
2. increasing the knowledge, skill levels
given below:
and capacities of the people of the
country.

610 6D: HRD: Weaker Section, HDI, SDG


unacademy.com | Download the Unacademy app
Give your feedback here: Link
(a) 1 only
(b) 1 and 2 only
(c) 2 and 3 only
(d) 1, 2 and 3

6D: HRD: Weaker Section, HDI, SDG 611


unacademy.com | Download the Unacademy app
Give your feedback here: Link
6D: HRD: Weaker Section, HDI,
21 SDG_Explanation
1. Answer: (c) etc. However, these form the part of economic
Explanation: #3 is wrong. There are of total 75 capital and not human capital.
such groups of tribals found in 18 states and in Statement 2 is correct: By increasing the
1 union territory. And NOT 92 groups. So Thus, knowledge, skill levels and capacities of the
Statement #3 is wrong and by elimination of people of the country their ability to translate
options containing this statement, we arrive at their capabilities into economic capital is
the correct answer [C] enhanced thus aiding the process of Human
PVTGs constitute the most vulnerable section Capital Formation in the country.
among tribals and inhabit isolated, remote and Statement 4 is correct: Through the process
difficult areas in small and scattered hamlets/ of Human Capital Formation the people can
habitats. accumulate knowledge and skill levels which
are attributed to intangible wealth as they
cannot be touched and seen.
2. Answer: (d)
Explanation:
4. Answer: (c)
Option (d) is correct: Social capital is defined
as the networks along with shared values, Inclusive governance is a participative way
norms, and understanding that facilitates of ‘Governance’ wherein citizens and other
mutual cooperation within or among groups. stakeholders have a say in the decision-making
These networks and cooperation are also process of the Government. It is an integrated
determined by the level of mutual trust and approach that equally manages to improve two
harmony in the society. parts of the service delivery-efficient delivery
of service, and empowering marginalized
Social Capital along with economic capital and communities to demand services. Inclusive
cultural capital form the three components of governance is not only about structural change
Social Resources. The access to these Social and policy, but it also includes how the actors
Resources is a hallmark of a society’s well being in governance act and interact with others.
and often the three forms of capital within the
Social Resources are overlapping and can be Statement 1 is not correct: NBFCs (Non-Banking
converted into one another. Financial Companies) play an important role
in promoting inclusive growth in the country,
by catering to the diverse financial needs of
3. Answer: (c) bank excluded customers. Permitting the Non-
Human Capital can be defined as the economic Banking Financial Companies to do banking is
value of an individual’s skills, abilities, not an essential part of inclusive governance.
knowledge, or experience, etc. Statement 2 is correct: District Planning
Statement 1 and statement 3 are not correct: Committee (DPC) is the committee created as
Human Capital Formation relates to the per article 243ZD of the Constitution of India
enhancement in the abilities of humans as at the district level for planning at the district
a resource in himself/herself and not the and below. The Committee in each district
accumulation of tangible wealth like land, should consolidate the plans prepared by
money, etc. Tangible wealth is related to assets the Panchayats and the Municipalities in the
which can be seen and touched like gold, land district and prepare a draft development plan

612 6D: HRD: Weaker Section, HDI, SDG_Explanation


unacademy.com | Download the Unacademy app
Give your feedback here: Link
for the district. them with social platforms to voice their
concerns. Such groups provide greater
Statement 3 is correct: Increasing the
economic bargaining and social mobility
government spending on public health will help
power in the hands of the members.
ensure universal access to healthcare facilities
(Statement 1 is correct)
thus ensuring inclusive governance
y Promoting Micro, Small and Medium
Statement 4 is correct: Strengthening the Mid-
Enterprises will enable small grassroot
day Meal Scheme will help ensure nutritional
level entrepreneurs to set up business
security for the poor children and thus amounts
enterprises and also provide for jobs in the
to an inclusive governance measure.
informally serviced geographical areas of
the country. (Statement 2 is correct)
5. Answer: (d) y By implementing the Right to Education Act,
Inclusive growth is the concept where the the economically disadvantaged sections of
benefits of economic and social development our society can have access to education.
are shared by all the sections of the society. This access will help them in enhancing
y By promoting Self-Help Groups the their capabilities and human capital thus
members which are mostly women can allowing for greater future economic
have access to establishing sustainable and social growth for all. (Statement 3 is
economic enterprises and also provide correct)

6D: HRD: Weaker Section, HDI, SDG_Explanation 613


unacademy.com | Download the Unacademy app
Give your feedback here: Link
22 7: MicroEconomics

1. Consider the following statements: (2021) (c) 1, 3 and 4


Other things remaining unchanged, (d) 1, 2 and 3
market demand for a good might
increase if
2. If a commodity is provided free to the
1. price of its substitute increases public by the Government, then (2018)
2. price of its complement increases (a) The opportunity cost is zero.
3. the good is an inferior good and income (b) The opportunity cost is ignored.
of the consumers increases (c) The opportunity cost is transferred
4. its price falls from the consumers of the product to
Which of the above statements are the tax-paying public.
correct? (d) The opportunity cost is transferred
(a) 1 and 4 only from the consumers of the product to
the Government.
(b) 2, 3 and 4

614 7: MicroEconomics
unacademy.com | Download the Unacademy app
Give your feedback here: Link
22 7: MicroEconomics_Explanation

1. Answer: (a) the price of a complement increases, then the


The amount of a good, service or resource that other products’ demand will decrease.
people are willing and able to buy during a
specified period at a specified price is known 2. Answer: (c)
as the quantity demanded.
Opportunity cost is the loss or gain of making
The following factors have the greatest impact a decision. It is the value of what one loses
on purchasing power and thus demand: when choosing between two or more options
y Income or alternatives. Due to the limited number of
y Expectations resources available to any individual/society/
country etc. choices have to be made in respect
y Number of buyers of the benefits/uses that have to be availed
y Prices of related goods out of the available resources. While choosing
y Preferences one use for the available resources we forgo
the other alternative uses which could have
Statements 1 and 4 are correct: The Demand
also been chosen instead of the currently
for a good, service or resource rises if the cost
chosen use. The value that could have been
of one of its substitutes rises or if its price
obtained out of other alternative uses and is
falls.
now foregone is referred to as the opportunity
Statement 3 is not correct: An inferior good is a cost of using one particular alternative.
term used in economics to describe a product
Option (c) is correct: If a commodity is provided
whose demand falls as people’s (consumers’)
free to the public by the Government, then
earnings rise.
the opportunity cost is transferred from the
Statement 2 is not correct: An object used in consumers of the product to the tax-paying
combination with another product or service public.
is a complementary good or service. Usually,
The government while providing free
when consumed alone, the complementary
commodities to certain consumers uses the
good has little or no value. Still, when paired
tax revenue that it has obtained. The taxpayers
with another good or service, it adds value to
could have also availed other benefits out of
the overall value of the bid. A product may be
their paid taxes, hence the opportunity cost
considered to be complementary if it shares a
of providing a free commodity is in the sense
beneficial relationship with another product.
of other prospective benefits that could have
An ideal example could be a sharpener and
been availed, foregone by the tax-paying public.
eraser given away with a box of pencils. So, if

7: MicroEconomics_Explanation 615
unacademy.com | Download the Unacademy app
Give your feedback here: Link
11 SCIENCE & TECH
AND BASIC SCIENCE
1 Biotechnology

1. ‘Aerial metagenomics’ best refers to which (c) 1 and 3 only


one of the following situations? (2023) (d) 1, 2 and 3
(a) Collecting DNA samples from air in a
habitat at one go
4. Which one of the following statements
(b) Understanding the genetic makeup of best describes the role of B cells and T
avian species of a habitat cells in the human body? (2022)
(c) Using air-borne devices to collect blood (a)
They protect the body from
samples from moving animals environmental allergens.
(d) Sending drones to inaccessible areas to (b) They alleviate the body’s pain and
collect plant and animal samples from inflammation.
land surfaces and water bodies
(c) They act as immunosuppressants in the
body.
2. ‘Microsatellite DNA’ is used in the case of (d) They protect the body from diseases
which one of the following? (2023) caused by pathogens.
(a) Studying the evolutionary relationships
among various species of fauna
5. Consider the following statements : (2022)
(b) Stimulating ‘stem cells’ to transform
into diverse functional tissues DNA Barcoding can be a tool to :

(c)
Promoting clonal propagation of 1. assess the age of a plant or animal.
horticultural plants 2. distinguish among species that look
(d) Assessing the efficacy of drugs by alike.
conducting series of drug trials in a 3. identify undesirable animal or plant
population materials in processed foods.
Which of the statements given above is/are
3. In the context of vaccines manufactured to correct ?
prevent COVID-19 pandemic, consider the (a) 1 only
following statements : (2022) (b) 3 only
1. The Serum Institute of India produced (c) 1 and 2
COVID-19 vaccine named Covishield
(d) 2 and 3
using mRNA platform.
2. Sputnik V vaccine is manufactured using
a vector based platform. 6. Consider the following statements : (2022)
3. COVAXIN is an inactivated pathogen 1. Biofilms can form on medical implants
based vaccine. within human tissues.
Which of the statements given above are 2. Biofilms can form on food and food
correct? processing surfaces.
(a) 1 and 2 only 3. Biofilms can exhibit antibiotic
resistance.
(b) 2 and 3 only

617 Biotechnology
unacademy.com | Download the Unacademy app
Give your feedback here: Link
Which of the statements given above are 2. Fungi
correct ?
3. Virus
(a) 1 and 2 only
Which of the above can be cultured in
(b) 2 and 3 only artificial/synthetic medium?
(c) 1 and 3 only (a) 1 and 2 only
(d) 1, 2 and 3 (b) 2 and 3 only
(c) 1 and 3 only
7. With reference to recent developments (d) 1, 2 and 3
regarding ‘Recombinant Vector Vaccines’,
consider the following statements: (2021)
10. The term ‘ACE2’ is talked about in the
1. Genetic engineering is applied in the
context of (2021)
development of these vaccines.
(a) genes introduced in the genetically
2. Bacteria and viruses are used as vectors.
modified plants
Which of the statements given above is/are
(b) development of India’s own satellite
correct?
navigation system
(a) 1 only
(c) radio collars for wildlife tracking
(b) 2 only
(d) spread of viral diseases
(c) Both 1 and 2
(d) Neither 1 nor 2
11. Consider the following statements : (2021)
1. Adenoviruses have single-stranded DNA
8. In the context of hereditary diseases, genomes whereas retroviruses have
consider the following statements: (2021) double-stranded DNA genomes.
1. Passing on mitochondrial diseases 2. Common cold is sometimes caused by
from parent to child can be prevented an adenovirus whereas AIDS is caused
by mitochondrial replacement therapy by a retrovirus.
either before or after in vitro fertilization
Which of the statements given above is/are
of the egg.
correct?
2. A child inherits mitochondrial diseases
(a) 1 only
entirely from mother and not from
father. (b) 2 only

Which of the statements given above is/ (c) Both 1 and 2


are correct? (d) Neither 1 nor 2
(a) 1 only
(b) 2 only 12. Bollgard I and Bollgard II technologies are
(c) Both 1 and 2 mentioned in the context of (2021)

(d) Neither 1 nor 2 (a) clonal propagation of crop plants


(b) developing genetically modified crop
plants
9. Consider the following: (2021)
(c) production of plant growth substances
1. Bacteria

Biotechnology 618
unacademy.com | Download the Unacademy app
Give your feedback here: Link
(d) production of biofertilizers sperm
(b) Genetic modification of sperm producing
13.
What is the importance of using cells
Pneumococcal Conjugate Vaccines in (c)
Development of stem cells into
India? (2020) functional embryos
1. These vaccines are effective against (d) Prevention of mitochondrial diseases in
pneumonia as well as meningitis and offspring
sepsis.
2. Dependence on antibiotics that are not 16. What is the Cas9 protein that is often
effective against drug-resistant bacteria mentioned in the news? (2019)
can be reduced.
(a) A molecular scissors used in targeted
3. These vaccines have no side effects and gene editing.
cause no allergic reactions.
(b)
A biosensor used in the accurate
Select the correct answer using the code detection of pathogens in patients.
given below:
(c) A gene that makes plants pest-resistant.
(a) 1 only
(d) A herbicidal substance synthesized in
(b) 1 and 2 only generally modified crops.
(c) 3 only
(d) 1, 2 and 3 17. RNA interference (RNAi) technology has
gained popularity in the last few years.
14. Consider the following statements: (2020) Why? (2019)

1. Genetic changes can be introduced in 1. It is used in developing gene silencing


the cells that produce eggs or sperms therapies.
of a prospective parent. 2. It can be used in developing therapies
2. A person’s genome can be edited before for the treatment of cancer.
birth at the early embryonic stage. 3. It can be used to develop hormone
3. Human induced pluripotent stem cells replacement therapies.
can be injected into the embryo of a 4. It can be used to produce crop plants
pig. that are resistant to virtual pathogens.
Which of the statements given above is/are Select the correct answer using the code
correct? given below:
(a) 1 only (a) 1, 2 and 4 only
(b) 2 and 3 only (b) 2 and 3 only
(c) 2 only (c) 1 and 3 only
(d) 1, 2 and 3 (d) 1 and 4 only

15. In the context of recent advances in human 18. With reference to the recent developments
reproductive technology, ‘Pronuclear in science, which one of the following
Transfer” is used for (2020) statements is not correct? (2019)
(a) Fertilization of egg in vitro by the donor (a)
Functional chromosomes can be

619 Biotechnology
unacademy.com | Download the Unacademy app
Give your feedback here: Link
created by joining segments of DNA often seen in the news, be used in the
taken from cells of different species. immediate future? (2017)
(b) Pieces of artificial functional DNA can 1. Genome sequencing can be used to
be created in laboratories. identify genetic markers for disease
(c) A piece of DNA taken out from an animal resistance and drought tolerance in
cell can be made to replicate outside a various crop plants.
living cell in a laboratory. 2. This technique helps in reducing the
(d) Cells taken out from plants and animals time required to develop new varieties
can be made to undergo cell division in of crop plants.
laboratory petri dishes. 3. It can be used to decipher the host-
pathogen relationships in crops.

19. With reference to the Genetically Modified Select the correct answer using the code
mustard (GM mustard) developed in India, given below:
consider the following statements: (2018) (a) 1 only
1. GM mustard has the genes of a soil (b) 2 and 3 only
bacterium that give the plant the (c) 1 and 3 only
property of pest-resistance to a wide
variety of pests. (d) 1, 2 and 3

2. GM mustard has the genes that


allow the plant cross-pollination and 22. In the context of the developments in
hybridization. Bioinformatics, the term ‘transcriptome’,
3. GM mustard has been developed jointly sometimes seen in the news, refers to
by the IARI and Punjab Agricultural (2016)
University. (a) A range of enzymes used in genome
Which of the statements given above is/are editing
correct? (b) The full range of mRNA molecules
(a) 1 and 3 only expressed by an organism

(b) 2 only (c) The description of the mechanism of


gene expression
(c) 2 and 3 only
(d) A mechanism of genetic mutations
(d) 1, 2 and 3 taking place in cells

20. What is the application of Somatic Cell 23. Consider the following techniques/
Nuclear Transfer Technology? (2017) phenomena: (2014)
(a) Production of bio larvicides 1. Budding and grafting in fruit plants
(b) Manufacture of biodegradable plastics 2. Cytoplasmic male sterility
(c) Reproductive cloning of animals 3. Gene silencing
(d) Production of organisms free of diseases Which of the above is/are used to create
transgenic crops?
21. With reference to agriculture in India, how (a) 1 only
can the technique of ‘genome sequencing', (b) 2 and 3 only

Biotechnology 620
unacademy.com | Download the Unacademy app
Give your feedback here: Link
(c) 1 and 3 only stations
(d) None 4. To increase their shelf life.
Select the correct answer using the code
24. Mycorrhizal biotechnology has been used given below:
in rehabilitating degraded sites because (a) 1 and 2 only
mycorrhiza enables the plants to (2013) (b) 3 and 4 only
1. Resist drought and increase the (c) 1, 2 and 4 only
absorptive area
(d) 1, 2, 3 and 4
2. Tolerate extremes of pH
3. Resist disease infestation
27. With reference to ‘stem cells’, frequently
Select the correct answer using the code in the news, which of the following
given below: statements is/are correct? (2012)
(a) 1 only 1. Stem cells can be derived from
(b) 2 and 3 only mammals only.
(c) 1 and 3 only 2. Stem cells can be used for screening
(d) 1, 2 and 3 new drugs.
3. Stem cells can be used for medical
therapies.
25.
Recombinant DNA technology (Genetic
Engineering) allows genes to be transferred Select the correct answer using the code
(2013) given below:

1. across different species of plants (a) 1 and 2 only

2. from animals to plants (b) 2 and 3 only

3.
from microorganisms to higher (c) 3 only
organisms (d) 1, 2 and 3
Select the correct answer using the code
given below: 28.
What are the reasons for the people's
(a) 1 only resistance to the introduction of BT brinjal
(b) 2 and 3 only in India? (2012)

(c) 1 and 3 only 1. Bt brinjal has been created by inserting a


gene from a soil fungus into its genome.
(d) 1, 2 and 3
2. The seeds of Bt brinjal are terminator
seeds and therefore, the farmers have
26. Other than resistance to pests, what are the to buy the seeds before every season
prospects for which genetically engineered from the seed companies.
plants have been created? (2012) 3. There is an apprehension that the
1. To enable them to withstand drought consumption of Bt brinjal may have
2. To increase the nutritive value of the adverse impact on health.
produce 4. There is some concern that the
3. To enable them to grow and do introduction of BT brinjal may have
photosynthesis in spaceships and space adverse effect on the biodiversity.

621 Biotechnology
unacademy.com | Download the Unacademy app
Give your feedback here: Link
Select the correct answer using the code Which of the statements given above is/are
given below: correct?
(a) 1, 2 and 3 only (a) 1 and 2 only
(b) 2 and 3 only (b) 2 only
(c) 3 and 4 only (c) 1 and 3 only
(d) 1, 2, 3 and 4 (d) 1, 2 and 3

29. At present, scientists can determine the 30. A genetically engineered form of brinjal,
arrangement or relative positions of genes known as the Bt-Brinjal, has been
or DNA sequences on a chromosome. How developed. The objective of this is (2011)
does this knowledge benefit us? (2011) (a) To make it pest-resistant
1. It is possible to know the pedigree of (b)
To improve its taste and nutritive
livestock. qualities
2. It is possible to understand the causes (c) To make it drought resistant
of all human diseases.
(d) To make its shelf-life longer
3. It is possible to develop disease-
resistant animal breeds.

Biotechnology 622
unacademy.com | Download the Unacademy app
Give your feedback here: Link
1 Biotechnology-Explanation

1. Answer: (a) relied on the viral vector vaccine Covishield for


‘Aerial metagenomics’ refers to the collection its vaccination drive. Over 88% of the doses
and analysis of DNA samples from the air in a given out so far have been of Covishield, the
particular habitat all at once. It is a method Oxford-AstraZeneca vaccine manufactured
used to study the genetic composition and locally under licence by the Serum Institute of
diversity of microbial communities present in India. Unlike mRNA vaccines, Serum Institutes
the atmosphere. Covishield uses the viral vector platform.

During aerial metagenomics, air samples are Statement 2 is correct: The Sputnik V vaccine
collected using specialised equipment such is based on a proven well-studied human
as air samplers or filters. These samples adenovirus vector platform.
contain a wide range of genetic material, Statement 3 is correct: Covaxin is an inactivated
including DNA from bacteria, fungi, viruses, vaccine. This destroys the pathogen’s ability to
and other microorganisms present in the air. replicate but keeps it intact so that the immune
The collected samples are then subjected to system can still recognise it and produce an
DNA extraction and sequencing techniques to immune response.
obtain genetic information.
Elimination Technique: We know that
the Serum Institute of India produced a
2. Answer: (a) COVID-19 vaccine named Covishield using
“Microsatellite DNA” is used in studying the the viral vector platform. Statement 1 is
evolutionary relationships among various eliminated. So, Option(b) is correct.
species of fauna. Microsatellites, also known
as simple sequence repeats (SSRs), are short 4. Answer: (d)
segments of DNA consisting of repetitive
nucleotide sequences. These repetitive Option (d) is correct: T cells and B cells are
sequences vary in length and are highly part of a specialized network of immune cells
polymorphic, meaning they exhibit variations that specifically respond to pathogens and
in the number of repeats among individuals fight off infections. B cells and T cells are the
within a species. white blood cells of the immune system that
are responsible for adaptive immune response
Microsatellite DNA analysis involves examining in an organism. Both the cells are made in
the variation in microsatellite loci across the bone marrow. B cells mature in the bone
different species or populations. By comparing marrow while the T cells travel to the thymus
the presence, absence, or frequency of and mature there. These cells are structurally
specific microsatellite alleles among different similar and are involved in adaptive immune
organisms, scientists can assess the genetic response in an organism
diversity and relatedness between species.
This information is valuable for understanding
evolutionary relationships, population genetics, 5. Answer: (d)
and phylogenetic studies. DNA barcoding is a method of identifying
biological specimens and distinguishing among
3. Answer: (b) species that look alike. It involves sequencing
of stretches of the genomic region which
Statement 1 is incorrect: India has majorly

623 Biotechnology-Explanation
unacademy.com | Download the Unacademy app
Give your feedback here: Link
evolves fast enough to allow species-level to generate immunity. These vaccine genomes
discrimination. may evolve to lose the extra genes during
Statement 1 is incorrect: To measure the age of the process of manufacture of the vaccine or
plant and animal remains from the more recent replication within an individual, and there is a
past, scientists use a radioactive isotope of concern that this evolution might severely limit
carbon, called carbon-14, as their clock. the vaccine’s efficacy.

Statements 2 and 3 are correct: DNA Barcoding Statement 1 is correct: As a result of advances
can be used to distinguish among species that in the fields of molecular biology and genetic
look alike and identify the undesirable animal engineering, it is now possible to create live
or plant materials in processed foods. DNA recombinant vectors capable of delivering
barcoding in plants not only speeds up writing heterologous antigens by the introduction of
the encyclopedia of life, but also opens up antigen-encoding genes.
the possibility of establishing a Digital Plant
Identification System (DPIS) which works 8. Answer: (c)
independently of type, age, or developmental
stage of the sample under study. Hence, if used Mitochondrial disease, or mitochondrial
properly, DNA barcoding can be an effective disorder, refers to a group of disorders that
and efficient tool for exploring and protecting affect the mitochondria, which are tiny
biodiversity, expediting bioprospecting, and compartments that are present in almost every
defending against bio-piracy. cell of the body.
Statement 2 is correct: An individual’s
mitochondrial genome is entirely derived from
6. Answer: (d) the mother because sperm contain relatively
Biofilm is a complex structure of microbiome few mitochondria, and these are degraded
having different bacterial colonies or single after fertilization. It follows that mitochondrial
types of cells in a group; that adhere to the inheritance is essentially maternal inheritance.
surface. It is observed that a child inherits mitochondrial
Statements 1 & 2 are correct: Biofilms can diseases entirely from the mother and not
be formed on medical implants with human from the father.
tissues, food, and food processing surfaces. Statement 1 is correct: Mitochondrial
Hence, they pose grave health issues, especially replacement therapy (MRT) is a new form of
in the form of infections. reproductive in vitro fertilization (IVF) which
Statement 3 is correct: Bacterial biofilms are a works on the principle of replacing a woman’s
serious global health concern due to their ability abnormal mitochondrial DNA (mt-DNA) with the
to tolerate antibiotics (exhibiting antibiotic donor’s healthy one. Passing on mitochondrial
resistance), host defence systems and other diseases from parent to child can be prevented
external stresses; therefore it contributes to by mitochondrial replacement therapy either
persistent chronic infections. before or after in vitro fertilization of an egg.

7. Answer: (c) 9. Answer: (a)

Statement 2 is correct: Recombinant vector Option (a) is correct:


vaccines are live replicating viruses that are Bacteria can be formed in simple, synthetic or
engineered to carry extra genes derived from a complex media, where they vary in nutritional
pathogen like Bacteria and viruses. These extra make-up.
genes produce proteins against which we want y Simple media facilitates the growth of non-

Biotechnology-Explanation 624
unacademy.com | Download the Unacademy app
Give your feedback here: Link
fastidious bacteria, and the exact chemical developed or researched will look to inhibit
compositions of simple media are known. these activities at one stage or the other.
y Synthetic media is composed of minimal
ingredients needed for the growth of 11. Answer: (b)
microorganisms, for example, Davis and
Mingioli Medium. Option (b) is correct: Adenovirus is a type
of virus that has no envelope, whereas
y In complex media, the exact chemical retroviruses are characterized as enveloped
composition is not known, for example, in viruses. Adenoviruses belong to the virus group
Tryptic Soy Broth. that is composed of non–enveloped viruses.
Fungal Media containing high carbohydrate They are very common human pathogens, and
sources and nitrogen sources are required some may also infect animals. Adenoviruses
for the growth of fungi at a pH range of 5 to contain a double-stranded DNA genome.
6 and a temperature range from 15 to 37˚C. The adenovirus family is divided into two
There are two general types of fungal culture main genera, namely, Mastadenoviruses and
media: natural and synthetic. Natural media Aviadenoviruses.
are composed of natural substrates, such as Retroviruses are a family of viruses that
herbaceous or woody stems, seeds, leaves, are categorized as enveloped viruses. One
corn meal, wheat germ, oatmeal, etc. of the most common retroviruses that
Cultivation of Viruses: Viruses cannot be infect humans worldwide is the Human
cultured in an artificial/synthetic medium. Immunodeficiency Virus (HIV) which causes
Viruses require a living host cell for replication. Acquired Immunodeficiency Syndrome (AIDS).
Retroviruses contain RNA genome.
Elimination Technique: A virus is an
infectious agent that can only replicate Adenoviruses are common viruses that
within a host living organism. Virus is typically cause the common cold - or flu-like
eliminated. Thus, Option (a) is correct. illness.

10. Answer: (d) 12. Answer: (b)

Option (d) is correct: A coronavirus is surrounded Option (b) is correct: Bollgard I Bt cotton (single-
by a fatty outer layer (“envelope”) and on the gene technology) is India’s first biotech crop
surface of this layer is the “corona” (crown) technology approved for commercialization in
of spikes made of protein. On the surface of India in 2002, followed by Bollgard II – double-
human cells is an enzyme called angiotensin- gene technology in mid-2006 by Genetic
converting enzyme 2 (ACE2), which acts as the Engineering Approval Committee (GEAC), the
receptor that enables SARS-CoV2 to launch its Indian regulatory body for biotech crops.
attack. The virus’s spike protein binds to the Bollgard cotton provides in-built protection for
receptor, then fuses with the cell surface, and cotton against destructive American Bollworm
releases its genetic material (RNA in the case of Heliothis Armigera infestations and contains an
SARS-CoV2) into the cell. The coronavirus that insecticidal protein from a naturally occurring
causes SARS, called SARS-CoV, uses the same soil microorganism, Bacillus thuringiensis (Bt).
ACE2 receptor to invade a cell. Once inside,
the virus replicates itself by using the cell’s
molecular mechanism. All these stages involve 13. Answer: (b)
various interactions between virus proteins Pneumococcal Conjugate Vaccine (PCV)
and human proteins. Any treatment being prevents us from pneumococcal disease. In

625 Biotechnology-Explanation
unacademy.com | Download the Unacademy app
Give your feedback here: Link
the 1980s the first pneumococcal vaccine was sperm cell becomes a pronucleus only after the
developed. Under the Universal Immunization sperm enters the ovum, but before the fusion
Programme (UIP) the government is planning of genetic material of the sperm and egg.
for the nationwide rollout of the Pneumococcal Option (d) is correct: The technique of pronuclear
Conjugate Vaccine (PCV). transfer involves the transfer of pronuclei from
Statement 1 is correct: Pneumonia, Sepsis and one zygote to another. Pronuclear Transfer
Meningitis are the symptoms of pneumococcal has the potential to prevent mitochondrial
disease. Pneumococcal Conjugate Vaccines disease in the offspring as Mitochondrial DNA
are effective against pneumonia as well as is transmitted maternally.
meningitis and sepsis.
Statement 2 is correct: The usage and
dependence of antibiotics that are drug
resistant can be reduced as the vaccine
prevents the very occurrence of the infection.
Statement 3 is not correct: PCV Pneumococcal
Conjugate Vaccines have side effects like fever,
loss of appetite, fussiness, and other side
effects.

14. Answer: (d)


Statement 1 is correct: Genetic changes
can be introduced in the cells that produce
eggs or sperm of a prospective parent by 16. Answer: (a)
Genetic Engineering. Using scientific methods
CRISPR (Clustered Regularly Interspaced Short
Genetically modified organisms (GMOs) are
Palindromic Repeats)-Cas9 is a technology that
produced that include recombinant DNA
allows scientists to essentially cut-and-paste
technology and reproductive cloning.
DNA. CRISPR is a dynamic, versatile tool that
Statement 2 is correct: Using the Genetically allows us to target nearly any genomic location
Modified Organisms technique, various genetic and potentially repair broken genes. It can
diseases of the off-springs can be cured, i.e., remove, add or alter specific DNA sequences
by editing their DNA. A nucleus is extracted in the genome of higher organisms.
from a cell of the individual to be cloned and
Option (a) is correct: CRISPR-Cas9 is a gene-
is inserted into the enucleated cytoplasm of
editing tool. It has two components:
a host egg (an enucleated egg is an egg cell
that has had its own nucleus removed) in y A short RNA sequence that can bind to a
reproductive cloning. specific target of the DNA and
Statement 3 is correct: Researchers have y The Cas9 enzyme which acts as molecular
successfully used human-induced pluripotent scissors to cut the DNA.
stem cells to create embryos that are part- To edit a gene of interest, the short RNA
human, part-pig. sequence that perfectly matches with the DNA
sequence that has to be edited is introduced.
Once it binds to the DNA, the Cas9 enzyme
15. Answer: (d)
cuts the DNA at the targeted location where
Pronucleus is the nucleus of a sperm or an the RNA sequence is bound. Once the DNA
egg cell during the process of fertilization. The

Biotechnology-Explanation 626
unacademy.com | Download the Unacademy app
Give your feedback here: Link
is cut, the natural DNA repair mechanism is 18. Answer: (a)
utilized to add or remove genetic material or DNA stands for Deoxyribonucleic acid. It is
make changes to the DNA. made up of units of biological building blocks
called nucleotides. DNA is a vitally important
17. Answer: (a) molecule for not only humans but for most
other organisms as well. DNA contains our
RNA interface or Post-Transcriptional Gene
hereditary material and our genes, which
Silencing is a genetic regulatory system that
makes every individual unique.
functions to silence the activity of specific
genes. RNAi occurs naturally, through the In the nucleus of each cell, the DNA molecule
production of nuclear-encoded pre-micro is packaged into thread-like structures called
RNA (pre-miRNA), and can be induced chromosomes. Each chromosome is made up of
experimentally, using short segments of DNA tightly coiled many times around proteins
synthetic double-stranded RNA (dsRNA). called histones that support its structure.

Statement 1 is correct: Gene silencing is Option (a) is not correct: It is not possible to
a modern gene-editing technique. Using create a functional chromosome by joining
techniques like RNA interference, CRISPR-CAS9 segments of DNA taken from different species.
and antisense RNA technique, a gene of our Chromosomes are very long and code for so
interest can be suppressed, or its expression many genes. Within a species, translocation
is controlled. It is used in developing gene- can take place and different parts of two
silencing therapies. chromosomes may join to produce functional
chromosomes. But this often gives rise to
Statement 2 is correct: RNAi is used in
abnormalities like Philadelphia Chromosome.
functional genomics (systematic analysis
of loss-of-function phenotypes induced by
RNAi triggers) and developing therapies for 19. Answer: (b)
the treatment of viral infection, dominant Genetically modified crops are those types
disorders, neurological disorders, and many of crops that have been modified genetically,
types of cancers (in vivo inactivation of gene which might include increasing nutrients,
products linked to human disease progression reducing the maturation time of the plant,
and pathology). yields, and stress tolerance, and creating a
Statement 3 is not correct: Hormone plant that can withstand diseases, and heavier
replacement therapy is a treatment for women applications of pesticides and herbicides.
who have low hormone levels, like a woman Statement 1 is not correct: DMH-11 is a
going through menopause (menstrual periods Genetically Modified (GM) mustard, and it is
stop permanently, and they are no longer able also an herbicide-tolerant mustard. DMH -11
to bear children). It is also called estrogen contains three genes viz. Bar gene, Barnase
replacement therapy. The RNA interface cannot and Barstar sourced from a soil bacterium.
be used to develop hormone replacement The genes of a soil bacterium do not give the
therapies. plant the property of pest resistance to a wide
Statement 4 is correct: RNAi has resulted in variety of pests.
the invention of novel crops such as nicotine- Statement 2 is correct: GM Mustard is a perfect
free tobacco, decaffeinated coffee, nutrient flower so it ‘self-pollinates’. GM mustard has
fortified vegetation. It can be used to produce the genes that allow plant cross-pollination
crop plants that are resistant to virtual and hybridization.
pathogens.
Statement 3 is not correct: GM Mustard has

627 Biotechnology-Explanation
unacademy.com | Download the Unacademy app
Give your feedback here: Link
been developed by a team of scientists at Delhi or better treated by early detection and
University (not IARI and Punjab University) led intervention. Genome sequencing is also now
by former vice-chancellor Deepak Pental with used in farming adding speed and precision
the aim of reducing India’s demand for edible oil to selective breeding methods. It is also being
imports. They developed Dhara Mustard Hybrid used in farming various crop plants.
(DMH) -11, a genetically modified hybrid variety Statement 3 is correct: Genome sequence
of the mustard species (Brassica juncea). reveals a compact genome, a dependency of
Elimination Technique: The GM mustard the pathogen on its host for some essential
has been developed by a team of nutrients and a potential role in the regulation
scientists at Delhi University. Statement 3 of host plant cytokinin and auxin.
is eliminated. Thus, Option (b) is correct.
22. Answer: (b)
20. Answer: (c) Option (b) is correct: Transcriptome is the
Option (c) is correct: Somatic Cell Nuclear sum total of all the messenger RNA molecules
Transfer (SCNT) is a technology applied in expressed from the genes of an organism. It
cloning, stem cell research, and regenerative is the full range of messenger RNA, or mRNA,
medicine. Somatic cells are cells that have molecules expressed by an organism. The term
gone through the differentiation process and “transcriptome” can also be used to describe
are not germ cells. Somatic cells donate their the array of mRNA transcripts produced in a
nuclei, which scientists transplant into eggs particular cell or tissue type. In contrast with
after removing their nuclei (enucleated eggs). the genome, which is characterised by its
stability, the transcriptome actively changes.
The successful cloning of monkeys, the first
non-human primate species, by Somatic Cell In fact, an organism’s transcriptome varies
Nuclear Transfer (SCNT) attracted world-wide depending on many factors, including the stage
attention. Remarkably, it has taken more than of development and environmental conditions.
20 years since the cloning of Dolly the Sheep
in 1997 to achieve this feat. This success 23. Answer: (b)
was largely due to a recent understanding of
epigenetic barriers that impede SCNT-mediated Option (b) is correct: Transgenic plants are
reprogramming and the establishment of key the ones whose DNA is modified using genetic
methods to overcome these barriers, which engineering techniques. The aim is to introduce
also allowed efficient derivation of human a new trait to the plant which does not occur
pluripotent stem cells for cell therapy. naturally in the species. Cytoplasmic male
sterility and gene silencing technique is used
to create transgenic crops.
21. Answer: (d) y Cytoplasmic male sterility is total or
Genome sequencing is the process of partial male sterility in plants. These
determining the complete DNA sequence of systems are widely exploited in crop
an organism’s genome. It involves figuring out plants for hybrid breeding due to the
the order of DNA nucleotides, or bases, in a convenience of controlling sterility.
genome—the order of As, Cs, Gs, and Ts that Gene silencing using RNA interference
make up an organism’s DNA. technology, transcriptional gene silencing,
Statement 1 and statement 2 are correct: and virus-induced gene silencing, is
Genomes for plant diseases allow the detection commonly used in horticulture crops. The
of rare disorders that can be prevented presence of double-stranded RNA inside

Biotechnology-Explanation 628
unacademy.com | Download the Unacademy app
Give your feedback here: Link
the cell leads to the production of small varying pH from as low as 3 to as high as 9.5
interfering RNAs, short hairpin RNAs and (extremes of pH). However, at the extremes of
microRNAs. this range, the occurrence of plants is much
y Grafting and budding are horticulture more limited. Most plants grow in soils of pH
techniques used to join parts from two or 4.0 to 8.0.
more plans so that they appear to grow as Statement 3 is correct: Mycorrhizal fungi
a single plant. In grafting the upper part provide an effective alternative method of
of one plant grows on the root system disease control, especially for those pathogens
of another plant. In the budding process, which affect below-ground plant organs. In
a bud is taken from one plant and grown mycorrhizal fungi lies the enormous potential
on another. It does not lead to any genetic for use as biocontrol agents for soil- and root-
modification. borne diseases.

24. Answer: (d) 25. Answer: (d)


Mycorrhizal is a fungus which grows in Statement 1 is correct: Recombinant DNA
association with the roots of a plant in a technology combines DNA from different
symbiotic or mildly pathogenic relationship. sources to create a different sequence of DNA.
These specialised fungi colonise plant roots Recombinant DNA technology is the joining of
and extend far into the soil. Mycorrhizal fungal DNA molecules from two different species.
filaments in the soil are truly extensions of root To produce new genetic combinations the
systems and are more effective in nutrient and recombinant DNA molecule is inserted into a
water absorption than the roots themselves. host organism
Approximately 95 percent of plant species on Statement 2 is correct: Recombinant DNA
the planet form a symbiotic relationship with technology has been used in Vaccines Creation
the beneficial mycorrhizal fungi. where viral proteins are produced by bacteria
Statement 1 is correct: Mycorrhizal root or yeast from recombined viral genes. They are
systems increase the absorbing area of roots considered safer than those created by more
10 to 1000 times, thereby greatly improving the traditional methods containing viral particles.
ability of the plants to utilise the soil resource. Statement 3 is correct: Recombinant DNA
Mycorrhizal fungi are able to absorb and transfer technology has been used to produce both
all major macro and micronutrients necessary insect and herbicide-resistant crops. The
for plant growth. Not all mycorrhizal fungi have most common herbicide-resistant crops are
the same capacities and tolerances. Some are resistant to the application of glyphosate, a
better at imparting drought resistance, while common weed killer.
others may be more effective in protecting
against pathogens or have more tolerance to
soil temperature extremes. Because of the wide 26. Answer: (c)
variety of soil, climatic, and biotic conditions Genetically engineered plant (GE plant) means
characterising man-made environments, it is a plant in which the genetic material has been
improbable that a single mycorrhizal fungus changed through in vitronucleic acid techniques,
could benefit all host species and adapt to all including recombinant deoxyribonucleic acid
conditions. (DNA) and direct injection of nucleic acid
Statement 2 is correct: Mycorrhizal plants into cells or organelles. Also referred to as a
occur in all terrestrial environments where genetically modified (GM) or recombinant DNA
their host plants grow. This includes soils with (rDNA) or transgenic plant.

629 Biotechnology-Explanation
unacademy.com | Download the Unacademy app
Give your feedback here: Link
Option (c) is correct: Genetically modified 28. Answer: (c)
(GM) or transgenic plants, have been and are The possible reasons for the people’s resistance
being developed with the aim of enhancing to the introduction of Bt brinjal in India is too
productivity; decreasing dependence on the due to an apprehension that the consumption
use of agricultural chemicals; and improving of Bt brinjal may have an adverse impact on
the nutritional value of foods and livestock health and there was also some concern that
feeds, longer shelf life, increase resistance to the introduction of BT brinjal may have an
droughts, increase resistance to insects, pests, adverse effect on the biodiversity.
increased resistance to herbicides.
Statement 1 is not correct: Bt Brinjal is a
transgenic brinjal created by inserting a
27. Answer: (b) gene cry1Ac from the soil bacterium Bacillus
Stem cells are special human cells that are able thuringiensis into Brinjal.
to develop into many different cell types. This Statement 2 is not correct: The seeds of Bt
can range from muscle cells to brain cells. In brinjal are not terminator seeds. Although
some cases, they can also fix damaged tissues. proposed nearly 20 years ago, so-called
Types of Stem cells are: Embryonic Stem Cells, ‘terminator’ seeds have never been developed
Adult Stem Cells, Induced Pluripotent Stem and do not currently exist. Seeds from Bt
Cells, Mesenchymal stem cells brinjal open-pollinated varieties will be free for
Statement 1 is not correct: Stem cells can farmers to save and replant as they wish.
be derived from two main sources: adult Statement 3 is correct: An independent inquiry
body tissues and embryos. Scientists are has revealed that the cultivation of genetically
also working on techniques to develop engineered (GE, also called genetically
stem cells from other cells, using genetic modified, or GM) Bt brinjal poses risks to the
“reprogramming” techniques. environment and possibly to human health. The
Statement 2 is correct: Scientists use stem occurrence of wild, weedy and also cultivated
cells in laboratories to test new drugs for relatives presents a likelihood that the GE Bt
safety and effectiveness before they are tested gene will spread to these relatives but, so far,
on humans. Scientists can also test how drugs this has largely been overlooked in the risk
might affect different organs such as the heart assessments for GE Bt brinjal.
or the liver by programming stem cells to Statement 4 is correct: Genetically engineered
acquire properties of the organ the drugs are Bt brinjal has several implications on
targeting. plant biodiversity. An independent study
Statement 3 is correct: It can also be used to commissioned by Greenpeace International
understand how disease occurs by studying finds that brinjal relatives do occur in the
how stem cells turn into specific tissue such regions where cultivation of GE Bt brinjal is
as heart muscles or nerves. proposed and that GE Bt brinjal may mate
with these relatives to spread the GE Bt gene.
Application of Stem cells
The spread of the GE Bt gene would have
y treating patients with currently untreatable considerable ecological implications, as well
conditions as implications for future crop contamination
y growing organs for transplants and farmers’ rights.

y medical research There are many concerns with GE brinjal,


which has been engineered to be resistant to
certain insect pests using Bt genes from the
soil bacterium Bacillus thuringiensis. These

Biotechnology-Explanation 630
unacademy.com | Download the Unacademy app
Give your feedback here: Link
concerns include food safety and possible sequencing of complex communities of
effects on organisms other than the pest insect microorganisms, metagenome sequencing
(non-target organisms), such as beneficial of environmental or human microbiomes,
insects and butterflies. and environmental profiling.

Elimination Technique: The seeds of Bt y DNA sequencing in Agriculture: Sequencing


brinjal are not terminator seeds. Statement microorganisms to engineer resistant genes
2 is not eliminated. Thus, Option (c) is in crops also helps in developing disease-
correct. resistant animal breeds, mapping and
whole-genome sequencing of food plants
to increase productivity and nutritional
29. Answer: (c) contents as well as environmental
DNA stands for deoxyribonucleic acid. It is tolerance. (Statement 3 is correct)
made up of units of biological building blocks y DNA sequencing in Molecular Biology: Study
called nucleotides. The information in DNA is of genes, genotypes, and proteins; gene-
stored as a code made up of four chemical based studies of cancers; construction
bases: adenine (A), guanine (G), cytosine (C), of endonuclease maps; detection of
and thymine (T). mutations; construction of molecular
Sequencing of DNA means determining the evolution map, and transcriptome profiling.
order of the four chemical building blocks Elimination Technique: It is possible
- called “bases” - that make up the DNA to understand the causes of all human
molecule. The order, or sequence, of these diseases. Statement 2 is too extreme
bases, determines the information available and it can be eliminated. So, Option (c) is
for building and maintaining an organism. correct.
Application of DNA Sequencing:
y DNA sequencing in Evolutionary biology: 30. Answer: (a)
The ability to sequence the whole genome
of many related organisms has allowed Option (a) is correct: A genetically engineered
large-scale comparative genomics, and form of brinjal gives resistance against insects
phylogenetics to know the pedigree such as the Brinjal Fruit and Shoot Borer
of livestock and evolutionary studies. (Leucinodes orbonalis). It was developed by
(Statement 1 is correct) Maharashtra Hybrid Seeds Company (Mahyco).
The father of India’s Green Revolution, S
y DNA sequencing in Diagnostics and Swaminathan, had asked for an independent
Medicine: DNA sequencing has applications study on the long-term toxicity of Bt Brinjal
in screening the risk of genetic diseases, before approving it for commercial use.
gene therapy-based treatments, genetic
engineering, and gene manipulation. y Bt. Cotton is the only Genetically Modified
However, it cannot be claimed that it is (GM) crop approved in 2002 by the Genetic
possible to understand the causes of Engineering Appraisal Committee of
all human diseases. (Statement 2 is not the Ministry of Environment, Forest and
correct) Climate Change for commercial cultivation
in the Country and, therefore, cultivation of
y DNA sequencing in Forensic Science: DNA other unapproved GM crops are banned in
sequencing has applications in DNA profiling, India. Few incidences of suspected open
forensic sampling and identification, and cultivation of Bt. Brinjal and HT cotton were
paternity testing. reported in Maharashtra, Haryana, Punjab,
y DNA sequencing in Metagenomics: Shotgun Gujarat and Andhra Pradesh.

631 Biotechnology-Explanation
unacademy.com | Download the Unacademy app
Give your feedback here: Link
2 Defence Technology

1. Consider the following statements: (2023) 2. With reference to Agni-IV Missile, which of
1. Ballistic missiles are jet-propelled the following statements is/are correct?
at subsonic speeds throughout their (2014)
fights, while cruise missiles are rocket- 1. It is a surface-to-surface missile.
powered only in the initial phase of 2. It is fuelled by liquid propellant only.
flight.
3. It can deliver one-tonne nuclear
2. Agni-V is a medium-range supersonic warheads about 7500 km away.
cruise missile, while BrahMos is a solid-
fuelled intercontinental ballistic missile. Select the correct answer using the code
given below:
Which of the statements given above is/are
correct? (a) 1 only

(a) 1 only (b) 2 and 3 only

(b) 2 only (c) 1 and 3 only

(c) Both 1 and 2 (d) 1, 2 and 3

(d) Neither 1 nor 2

Defence Technology 632


unacademy.com | Download the Unacademy app
Give your feedback here: Link
2 Defence Technology-Explanation

2. Answer: (d) or moving slowly.


Statement 1 is incorrect: Ballistic missiles y Agni IV is a surface-to-surface missile. It
are rocket-powered throughout their flight, is an indigenously developed sophisticated
reaching high speeds including supersonic two-stage missile having 4,000 km strike
and hypersonic velocities. Cruise missiles, on range. It is 20 metres long with a weight of
the other hand, are typically jet-propelled and 17 tonnes. (Statement 1 is correct)
fly at subsonic speeds, maintaining sustained y Agni IV has two stages of Solid Propulsion.
flight within the atmosphere. The state-of-the-art missile is equipped
Statement 2 is incorrect: Agni-V is an with modern and compact avionics to
intercontinental ballistic missile (ICBM) with provide a high level of reliability and
a long range, while BrahMos is a supersonic precision. It is equipped with advanced
cruise missile with a shorter range. They are Avionics, 5th generation on-board Computer
different in terms of their propulsion systems, and distributed avionics architecture and
ranges, and intended purposes. high-speed reliable communication bus
and a full Digital Control System used
to control and guide the missile to the
1. Answer: (a) target. (Statement 2 is not correct)
A surface-to-surface missile (SSM) or ground- y AGNI IV has a range of about 4000 km with
to-ground missile (GGM) is a missile designed 1-tonne nuclear head. (Statement 3 is not
to be launched from the ground or the sea correct)
and strike targets on land or at sea. They may
be fired from hand-held, or vehicle-mounted Elimination Technique: Agni-V, the longest
devices, from fixed installations, or from a ship. of the Agni series can deliver over a range
They are often powered by a rocket motor or of 5,000 km. Statement 3 is eliminated.
sometimes fired by an explosive charge since Thus, Option (a) is correct.
the launching platform is typically stationary

633 Defence Technology-Explanation


unacademy.com | Download the Unacademy app
Give your feedback here: Link
3 Space Science

1. Consider the following pairs: (2023) of an equilateral triangle that has sides one
million kilometers long, with lasers shining
Objects in Description
between the craft.” The experiment in
space
question refers to (2020)
1. Cepheids : Giant clouds of dust and
(a) Voyager-2
gas in space
(b) New Horizons
2. Nebulae : Stars which brighten and
dim periodically (c) LISA Pathfinder
3. Pulsars : Neutron stars that are (d) Evolved LISA
formed when massive
stars run out of fuel and
4. For the measurement/estimation of which
collapse
of the following are satellite images and
How many of the above pairs are correctly remote sensing data is used? (2019)
matched?
1. Chlorophyll content in the vegetation of
(a) Only one a specific location.
(b) Only two 2. Greenhouse gas emission from rice
(c) All three paddies of a specific location.
(d) None 3. Land surface temperature of a specific
location.
Select the correct answer using the code
2. Which one of the following statements best
given below:
reflects the idea behind the “Fractional
Orbital Bombardment System” often talked (a) 1 only
about in the media? (2022) (b) 2 and 3 only
(a)
A hypersonic missile is launched (c) 3 only
into space to counter the asteroid
(d) 1, 2 and 3
approaching the Earth and explode it in
space.
(b) A spacecraft lands on another planet 5. Recently, scientists observed the merger
after making several orbital motions. of giant ‘blackholes’ billions of light-
years away from the Earth. What is the
(c) A missile is put into a stable orbit
significance of this observation? (2019)
around the Earth and deorbits over a
target on the Earth. (a) ‘Higgs boson particles’ were detected.

(d) A spacecraft moves along a comet with (b) ‘Gravitational waves’ were detected.
the same speed and places a probe on (c) Possibility of intergalactic space travel
its surface. through ‘wormhole’ was confirmed.
(d) It enabled scientists to understand
3. The experiment will employ a trio of ‘singularity’.
spacecraft flying in formation in the shape

Space Science 634


unacademy.com | Download the Unacademy app
Give your feedback here: Link
6. In which of the following areas, can GPS second and fourth stages using liquid
technology be used? (2018) rocket engines.
1. Mobile phone operations Which of the statements given above is/are
2. Banking operations correct?

3. Controlling the power grids (a) 1 only

Select the correct answer using the codes (b) 2 and 3 only
given below: (c) 1 and 2 only
(a) 1 only (d) 3 only
(b) 2 and 3 only
(c) 1 and 3 only 9. With reference to the Indian Regional
(d) 1, 2 and 3 Navigation Satellite System (IRNSS),
consider the following statements: (2018)
1. IRNSS has three satellites in
7. Consider the following phenomena: (2018)
geostationary and four satellites in
1. Light is affected by gravity. geosynchronous orbits.
2. The Universe is constantly expanding. 2. IRNSS covers the entire India and about
3. Matter warps its surrounding space- 5500 sq. km beyond its borders.
time. 3. India will have its own satellite navigation
Which of the above is/are the predictions/ system with full global coverage by the
predictions of Albert Einstein's General middle of 2021.
Theory of Relativity, often discussed in the Which of the statements given above is/are
media? correct?
(a) 1 and 2 only (a) 1 only
(b) 3 only (b) 1 and 2 only
(c) 1 and 3 only (c) 2 and 3 only
(d) 1, 2 and 3 (d) None

8. With reference to India’s satellite 10.


With reference to ‘AstroSat’, the
launch vehicles, consider the following astronomical observatory launched by
statements: (2018) India, which of the following statements
1. PSLVs launch the satellites useful for is/are correct? (2016)
Earth resources monitoring whereas 1. Other than the USA and Russia, India
GSLVs are designed mainly to launch is the only country to have launched a
communication satellites. similar observatory into space.
2. Satellites launched by PSLV appear to 2. AstroSat is a 2000 kg satellite placed in
remain permanently fixed in the same an orbit at 1650 km above the surface
position in the sky, as viewed from a of the Earth.
particular location on Earth.
Select the correct answer using the code
3. GSLV Mk III is a four-staged launch given below.
vehicle with the first and third stages
(a) 1 only
l using solid rocket motors; and the

635 Space Science


unacademy.com | Download the Unacademy app
Give your feedback here: Link
(b) 2 only (b) regions inside the Earth-like planets in
(c) Both 1 and 2 outer space

(d) Neither 1 nor 2 (c) search for the Earth-like planets in


outer space
(d)
search for meteorites containing
11. Consider the following statements: (2016)
precious metals
The Mangalyaan launched by ISRO
1. is also called the Mars Orbiter Mission
14. In the context of modern scientific research,
2. made India the second country to have consider the following statements about
a spacecraft orbit the Mars after USA ‘IceCube’, a particle detector located at
3. made India the only country to be South Pole, which was recently in the
successful in making its spacecraft news: (2015)
orbit the Mars in its very first attempt 1. It is the world’s largest neutrino detector,
Which of the statements given above is/are encompassing a cubic kilometre of ice.
correct? 2. It is a powerful telescope to search for
(a) 1 only dark matter.

(b) 2 and 3 only 3. It is buried deep in the ice.

(c) 1 and 3 only Which of the statements given above is/are


correct?
(d) 1, 2 and 3
(a) 1 only
(b) 2 and 3 only
12. In which of the following activities are
Indian Remote Sensing (IRS) satellites (c) 1 and 3 only
used? (2015) (d) 1, 2 and 3 only
1. Assessment of crop productivity
2. Locating groundwater resources 15. Which of the following pairs is/are correctly
3. Mineral exploration matched? (2014)

4. Telecommunications Spacecraft Purpose


5. Traffic studies 1. Cassini- Orbiting the Venus
Huygens and transmitting data
Select the correct answer using the code
to the Earth
given below:
2. Messenger Mapping and
(a) 1, 2 and 3 only
investigating the
(b) 4 and 5 only Mercury
(c) 1 and 2 only 3. Voyager 1 and Exploring the outer
(d) 1, 2, 3, 4 and 5 2 solar system
Select the correct answer using the code
given below:
13. The term ‘Goldilocks Zone’ is often seen in
the news in the context of (2015) (a) 1 only

(a) the limits of habitable zone above the (b) 2 and 3 only
surface of the Earth (c) 1 and 3 only

Space Science 636


unacademy.com | Download the Unacademy app
Give your feedback here: Link
(d) 1, 2 and 3 3. Movement of asteroids in space
4. Occurrence of supernova explosions in
16.
A team of scientists at Brookhaven space
National Laboratory including those from Select the correct answer using the code
India created the heaviest antimatter given below:
(anti-helium nucleus). What is/are the (a) 1 and 2
implication/ implications of the creation of
antimatter? (2012) (b) 2 only

1. It will make mineral prospecting and oil (c) 1, 3 and 4


exploration easier and cheaper. (d) None of the above can be cited as
2. It will help probe the possibility of the evidence
existence of stars and galaxies made of
antimatter. 18.
Satellites used for telecommunication
3. It will help understand the evolution of relay are kept in a geostationary orbit. A
the universe. satellite is said to be in such as orbit when:
Select the correct answer using the code (2011)
given below: 1. The orbit is geosynchronous.
(a) 1 only 2. The orbit is circular.
(b) 2 and 3 only 3. The orbit lies in the plane of the earth’s
(c) 3 only equator.

(d) 1, 2 and 3 4. The orbit is at an altitude of 22,236 km.

17. Which of the following is/are cited by Which the correct answer using the code
the scientists as evidence/evidences for given below:
the continued expansion of the universe? (a) 1, 2 and 3 only
(2012) (b) 1, 3 and 4 only
1. Detection of microwaves in space (c) 2 and 4 only
2. Observation of redshift phenomenon in (d) 1, 2, 3 and 4
space

637 Space Science


unacademy.com | Download the Unacademy app
Give your feedback here: Link
3 Space Science-Explanation

1. Answer: (a) 3. Answer: (d)


Pair 1 is incorrectly matched: Cepheids are a Option (d) is correct: e-LISA (Evolved Laser
class of variable stars that have played a crucial Interferometer Space Antenna) is a plan of
role in measuring astronomical distances and setting into space three spacecraft (trio of
advancing our understanding of the universe. spacecraft), a mother and two daughter
Cepheids are characterised by their regular spacecraft, which will fly in a triangular
and predictable variations in brightness. The formation (has sides one million kilometers
primary factor that causes their variability is long of an equilateral triangle), trailing the earth
pulsation—the expansion and contraction of in its orbit around the sun at a distance of over
the star’s outer layers. 50 million km. It has lasers shining between
Pair 2 is incorrectly matched: Nebulae are the craft.
vast clouds of gas, dust, and other interstellar
material scattered throughout galaxies. These
beautiful and often intricate structures can be
found in various shapes, sizes, and colours.
Nebulae play a fundamental role in the
formation and evolution of stars and galaxies.
Pair 3 is correctly matched: Pulsars are a
specific type of neutron star that emits
beams of radiation, typically in the form of
radio waves, but also in other parts of the
electromagnetic spectrum. As the neutron star
rotates rapidly, the emission beams sweep
across space, resembling the sweeping motion
of a lighthouse beam.

2. Answer: (c)
Option (c) is correct: The Fractional Orbital
Bombardment System (FOBS) was developed
by the Soviet Union in the 1960s. The system
is used to launch nuclear warheads from
space. In 2021, Frank Kendal III (the US
Secretary of Air Force) stated that the Chinese
eLISA aims to measure gravitational waves
Government is developing and testing FOBS.
in the frequency range from 0.1mHz to about
The idea behind FOBS is, a warhead is put into
100 mHz. To do this, it is necessary for the
a stable orbit and it deorbits over the target.
interferometers to have an arms length of a
If the target and the launch position is lined
million kilometers and that is impossible to
up and the warhead keeps going round, it will
achieve with an earth-based setup. Hence, it is
complete a circle.
necessary to have this elaborate experiment in
space. This frequency range, from 0.1 MHz to 100
MHz, or even 1Hz, is expected to be sensitive

Space Science-Explanation 638


unacademy.com | Download the Unacademy app
Give your feedback here: Link
to a rich set of sources of gravitational waves. gravitational-wave detectors have recorded
These sources are such that understanding gravitational waves from a total of 10 black
them can throw light on many problems of hole mergers and one merger of neutron stars.
interest in cosmology and astrophysics and
also make possible stringent tests of the
General Theory of Relativity. 6. Answer: (d) 
Global Positioning System (GPS) is a satellite-
based navigation system made up of at least
4. Answer: (d) 24 satellites covering Earth from space. Core
Statement 1 is correct: The satellite images Applications of GPS technology are Navigation,
obtained are widely used in the field of Timing, Tracking, Mapping and Location.
research nowadays. Chlorophyll content can Option (d) is correct:  GPS is used for mobile
be calculated by using multispectral images. phone operations (Example: Cell towers use
Multi-spectral image is a satellite image which it to route phone calls), banking operations
includes limited spectral bands, each with (Example: ATMs and cash registers use it for
different wavelengths. transactions) and controlling the power grids
Statement 2 is correct: Satellite remote sensing (Example: electrical grids use it to send power
data can provide spatially explicit information to house). 
with regard to greenhouse gas emissions of GPS is also used to identify, locate, and maintain
a specific location. Rice paddies have been contact reports with one or more fleet vehicles
mapped from multi-temporal Synthetic in real-time. Astronomy, Automated vehicle,
Aperture Radar (SAR) data using backscatter Cartography, Clock synchronisation, Disaster
change thresholds for many regions in Asia. relief/emergency services, Radio occultation for
Statement 3 is correct: Land Surface weather and atmospheric science applications. 
Temperature (LST) can be calculated through
thermal infrared remote sensing data. It can Elimination Technique: GPS is a very potent
record the apparent temperature of the Earth’s technology that has wide applications.
surface by measuring the radiant energy exiting Applications of GPS are often in the news.
its surface. With the advent of satellite images Thus, Option (d) is correct.
and digital image processing software, now it
is possible to calculate LST. 7. Answer: (d)
Statement 1 is correct: Einstein’s General
5. Answer: (b) Theory of Relativity is based on how we think
A black hole is a place in space where gravity gravity governs the behaviour of the Universe.
pulls so much that even light cannot get out. According to Einstein’s theory, light, just like
Gravity is so strong because matter has been any other form of matter, is affected by gravity.
squeezed into a tiny space. This can happen That is, the light also “falls” in a gravitational
when a star is dying. Because no light can get field. 
out, people cannot see black holes. They are Statement 2 is correct: Steady-state theory,
invisible. in cosmology the universe is always expanding
Option (b) is correct: Scientists have made four but maintaining a constant average density,
new detections of gravitational waves, ripples with the matter being continuously created
in the fabric of space and time emanating to form new stars and galaxies at the same
from separate black hole mergers. So far, rate that old ones become unobservable as a
the US-based LIGO and Europe-based VIRGO consequence of their increasing distance and
velocity of recession.

639 Space Science-Explanation


unacademy.com | Download the Unacademy app
Give your feedback here: Link
Statement 3 is correct: Hubble Space Telescope Orbit (GTO) whereas PSLVs launch satellites
captured the image of a phenomenon called useful for Earth resources monitoring.
the Einstein Ring. Einstein in his theory of Statement 2 is not correct: PSLV is designed
general relativity had suggested that a massive mainly to deliver the “earth-observation” or
object would warp space and time. “remote-sensing” satellites with a lift-off mass
of up to about 1750 Kg to Sun-Synchronous
8. Answer: (a) circular polar orbits of 600-900 Km altitude.
Satellites launched by PSLV do not appear to
To develop the technology needed to launch remain permanently fixed in the same position
satellites, Satellite Launch Vehicles or SLV, in the sky, as viewed from a particular location
a small lift launch vehicle project started in on Earth.
the early 1970s by the Indian Space Research
Organisation. Statement 3 is not correct: GSLV Mk III is a
three-stage heavy lift (not a four-stage) launch
vehicle developed by ISRO. First stage uses
Statement 1 is correct: The GSLV is designed solid fuel, the second stage liquid fuel and the
mainly to deliver the communication-satellites third stage cryogenic technology with the use
to the highly elliptical Geosynchronous Transfer of liquid hydrogen and oxygen.

9.  Answer: (a)

Space Science-Explanation 640


unacademy.com | Download the Unacademy app
Give your feedback here: Link
Option (a) is correct: IRNSS with an operational with a compact object accreting matter
name of NAVIC (Navigation with Indian from companion stars.
Constellation) is an independent regional y It will help to study star birth regions and
navigation satellite system being developed high energy processes in star systems lying
by India. It is designed to provide accurate beyond our galaxy
position information service to users in India
y It will help to detect new transient X-ray
as well as the region extending up to 1500 km
sources in the sky
from its boundary, which is its primary service
area.  y It will help to perform a limited deep-field
survey of the Universe in the Ultraviolet
Statement 1 is correct: Three of the seven
region.
satellites in the constellation are located in
geostationary orbit (GEO) and four in inclined
geosynchronous orbit (GSO).  11. Answer: (c)
Statement 2 is not correct: It covers India and a India’s Mangalyaan mission is aimed at studying
region extending 1,500 km around it, with plans the Martian atmosphere. Its objective is to
for further extension. The system at present explore Martian surface features, mineralogy,
consists of a constellation of seven satellites, morphology and atmosphere using indigenous
with two additional satellites on the ground as scientific instruments. A crucial objective
stand-by. of the mission was to develop technologies
Statement 3 is not correct: The constellation is required in planning, designing, management
already in orbit and the system was expected and operations of an interplanetary mission. 
to be operational in early 2018 after a system Statement 1 is correct: The Mangalyaan is also
check. known as the Mars Orbiter Mission (MOM),
launched in 2013 by the Indian Space Research
10. Answer: (d)  Organisation (ISRO). Mangalyaan was India’s
first interplanetary mission. 
Statement 1 is not correct:  ASTROSAT is
India’s first dedicated multi-wavelength Statement 2 is not correct: The mission made
space observatory. This scientific satellite India the fourth in the world after Roscosmos,
mission endeavours for a more detailed NASA, and the European Space Agency, to get
understanding of our universe. It enables to the planet. 
simultaneous multi-wavelength observations Statement 3 is correct: India is the first Asian
of various astronomical objects with a single nation to reach Mars orbit, and the first nation
satellite. Other than the USA and Russia, India in the world to do so in its first attempt. In
is not the only country to have launched such an September 2014, MOM controllers published
observatory.  The European Space Agency and the spacecraft’s first global view of Mars. The
Japan have also launched such observatories image was captured by the Mars Colour Camera
in space besides India. (MCC). Over the years, MCC has captured over
Statement 2 is not correct: ASTROSAT had a 980 images that were released to the public.
lift-off mass of about 1513 kg (not 2000 kg) and In September 2018, MOM completed four years
it was launched into a 650 km orbit inclined at in its orbit around Mars, although the designed
an angle of 6 deg to the equator by PSLV-C30. mission life was only six months. ISRO now
plans to develop and launch a follow-up
The objectives of AstroSat Mission:
mission called Mars Orbiter Mission 2 (MOM-2
y It will help to understand the high energy or Mangalyaan-2) in 2024.
processes in interacting binary systems

641 Space Science-Explanation


unacademy.com | Download the Unacademy app
Give your feedback here: Link
12. Answer: (a) y Use of Indian Remote Sensing Satellites
Remote sensing is the process of detecting for the Assessment of Crop Productivity:
and monitoring the physical characteristics Information on crop statistics is required
of an area by measuring its reflected and for planning and decision-making purposes,
emitted radiation at a distance (typically from such as distribution and storage of food
satellite or aircraft). Special cameras installed grains, Govt. policies, pricing, procurement,
in satellites collect remotely sensed images, and food security and so on. The Ministry
which help researchers “sense” things about of Agriculture and Farmers’ Welfare
the Earth. effectively uses contemporary techniques
of satellite remote sensing in such decision
Option (a) is correct: Indian Remote Sensing
making. Space data is used in addressing
(IRS) Satellites have a variety of applications,
many critical aspects, such as crop
such as in agriculture (for the assessment
area estimation, crop yield & production
of crop productivity), forestry, geology
estimation, crop condition, deriving basic
(mineral exploration), and hydrology (locating
soil information, cropping system studies,
groundwater resources), etc. Apart from
experimental crop insurance, etc.
this, they are also used for natural resources
monitoring, ocean and atmospheric studies y Use of Indian Remote Sensing Satellites
and cartographic applications. Indian for Mineral Exploration: The application
Remote Sensing Satellites are not used for of remote sensing techniques to geology
telecommunications and traffic studies yet. and mineral exploration started with
aerial photography. With the use of aerial
INSAT (Indian National Satellite System)
photography in Indian Remote Sensing
satellites have been traditionally supporting
Satellites, mineral deposits can be traced.
telecommunication applications for providing
Spaceborne data, particularly the LANDSAT
voice and data communications and there are
have been used in mineral exploration
no specific Indian satellites for traffic studies.
in two ways, viz identifying features
y Use of Indian Remote Sensing Satellites directly related to mineralisation such as
for Locating Ground Water Resources: alteration zones, gossans, specific host
Timely and reliable information on the rock or characteristic mineral association
occurrence and movement of groundwater and secondly as delineation of features
is a prerequisite for meeting its growing favourable for localisation of mineralisation
demand for drinking, domestic and such as folds, ‘faults and fractures.
industrial sectors. Being a sub-surface
feature, the detection of groundwater
relies heavily on controlling factors, namely 13. Answer: (c) 
lithology, geomorphology, structures, and Option (c) is correct: The National Aeronautics
precipitation, etc. Spaceborne spectral and Space Administration (NASA) reported the
measurements hold great promise in discovery of an Earth-size planet named TOI
providing such information in a timely, 700 d, orbiting its star in the “habitable zone”,
reliable, and cost-effective manner. In also called the “Goldilocks zone”. Goldilocks
India, initially, the coarse resolution data zone is the area around a star where it is not
from Indian Remote Sensing Satellite (IRS- too hot and not too cold for liquid water to
1A/-1B LISS-II) & Landsat-TM have been exist on the surface of surrounding planets.
operationally used by the Department of The newest such planet was found by NASA’s
Space mainly for identifying and mapping Transiting Exoplanet Survey Satellite (TESS)
potential groundwater zones for the entire mission, which it launched in 2018. Very few
country. such Earth-size planets have been found so

Space Science-Explanation 642


unacademy.com | Download the Unacademy app
Give your feedback here: Link
far, including some by NASA’s Kepler mission, to Saturn, launched on October 15, 1997. The
and this one is the first such discovery by TESS. mission consisted of NASA’s Cassini orbiter,
TOI 700 d measures 20% larger than Earth. It which was the first space probe to orbit Saturn,
orbits its star once every 37 days and receives and the European Space Agency ’s Huygens
an amount of energy that is equivalent to 86% probe, which landed on Titan, Saturn’s largest
of the energy that the Sun provides to Earth. moon.
The star, TOI 700, is an “M dwarf” located Pair 2 is correctly matched: Messenger was
just over 100 light-years away in the southern a NASAs robotic space probe that orbited
constellation Dorado, is roughly 40% of our the planet Mercury between 2011 and 2015,
Sun’s mass and size, and has about half its studying Mercury’s chemical composition,
surface temperature. geology, and magnetic field.
Pair 3 is correctly matched: Voyager 1 and 2
14. Answer: (d)  both are space probes that were launched by
NASA in 1977 as part of the Voyager program to
Statement 1 is correct: IceCube is the first
study the outer Solar System.
gigaton neutrino detector ever built and was
primarily designed to observe neutrinos from Elimination Technique: Cassini-Huygens,
the most violent astrophysical sources in our was a space mission to Saturn. Pair 1 is
universe. Neutrinos, almost massless particles not correctly matched. Thus, Option (b) is
with no electric charge, can travel from their correct.
sources to Earth with essentially no attenuation
and no deflection by magnetic fields. IceCube,
the South Pole neutrino observatory, is a 16. Answer: (b)
cubic-kilometer particle detector made of Antimatter composed of antiparticles, which
Antarctic ice and located near the Amundsen- possess the same mass as particles of ordinary
Scott South Pole Station. matter but have opposite charge and quantum
Statement 2 is correct: The IceCube telescope spin.
is a powerful tool to search for dark matter and Statement 1 is not correct: The reaction of
could reveal the physical processes associated matter-antimatter releases a huge amount
with the enigmatic origin of the highest energy of energy and can be used as a potent fuel
particles in nature. In addition, exploring the source specially for interplanetary travels. The
background of neutrinos produced in the energy density of antimatter is higher than that
atmosphere, IceCube studies the neutrinos of conventional fuels.
themselves; their energies far exceed those Statement 2 is correct: Astronomers have
produced by accelerator beams.  detected vast clouds of antimatter in our own
Statement 3 is correct: It is buried beneath Milky Way, generated largely by black holes
the surface (deep in ice), extending to a depth and neutron stars grinding up their binary
of about 2,500 meters. A surface array, IceTop, companions.
and a denser inner sub-detector, DeepCore, Statement 3 is correct: The Big Bang should
significantly enhance the capabilities of the have produced matter and antimatter in equal
observatory, making it a multipurpose facility. quantities. The asymmetry of matter and
antimatter in the visible universe is one of
15. Answer: (b) the greatest unsolved problems in physics. It
is assumed that, very early in the life of the
Pair 1 Is not correctly matched: Cassini- universe in a process known as baryogenesis,
Huygens, the U.S.- European space mission massive numbers of particles and antiparticles

643 Space Science-Explanation


unacademy.com | Download the Unacademy app
Give your feedback here: Link
were created and annihilated each other. direction we look at: all the distant galaxies are
going away from us. This can only be due to
the fact that the Universe is expanding.
17. Answer: (a)
The discovery of the 2.7 K cosmic microwave
A research team during the 1990s, Saul
background radiation was convincing evidence
Perlmutter, Brian Schmidt and Adam Riess
that the universe originated 13.8 billion years
found that the light from more than 50
ago from a very dense and hot state in the big
distant exploding stars was far weaker than
bang. As the theory goes, when the universe
they expected, meaning that galaxies had to
was born it underwent rapid inflation and
be racing away from each other at increasing
expansion. Redshift and blueshift describe
speed. The acceleration is driven by what
how light shifts toward shorter or longer
scientists call dark energy, a cosmic force that
wavelengths as objects in space (such as stars
is one of the great mysteries of the universe.
or galaxies) move closer or farther away from
The Nobel-winning discovery implies instead us. The concept is key to charting the universe’s
that the universe will get increasingly colder as expansion.
matter spreads across ever-vaster distances in
space. Galaxies that are 3 million light years
away from Earth move at a speed of around 70 18. Answer: (a) 
km per second. Galaxies that are 6 million light Option (a) is Correct: Satellites used for
years away move twice as fast. The research telecommunication relays are kept in a
implies that billions of years from now, the geostationary orbit. A satellite is said to be in
universe will become “a very, very large, but such an orbit when the orbit is geosynchronous,
very cold and lonely place. the orbit is circular, and the orbit lies in the
Option (a) is correct: Scientists found evidence plane of the Earth’s equator.
for the continued expansion of the universe Earth-orbiting satellite is a geostationary
when they analysed the microwave radiation satellite placed at an altitude of approximately
left over from the big bang that still bathes 35,786 km (not 22,236 Km) above the mean sea
the universe. It is also observed that light from level. The orbit lies in the plane of the Earth’s
distant objects in the universe is redshifted equator. An object in such an orbit has an
(shift in the frequency of light towards red orbital period equal to the Earth’s rotational
colour), which tells us that the objects are all period and thus appears motionless.
receding away from us. This is true in whatever

Space Science-Explanation 644


unacademy.com | Download the Unacademy app
Give your feedback here: Link
4 IT & Communication Technology

19. Which one of the following countries has its (SaaS)”, consider the following statements
own Satellite Navigation System? (2023) : (2022)
(a) Australia 1. SaaS buyers can customise the user
(b) Canada interface and can change data fields.

(c) Israel 2. SaaS users can access their data


through their mobile devices.
(d) Japan
3. Outlook, Hotmail and Yahoo! Mail are
forms of SaaS.
1. Consider the following communication Which of the statements given above are
technologies : (2022) correct?
1. Closed-circuit Television (a) 1 and 2 only
2. Radio Frequency Identification (b) 2 and 3 only
3. Wireless Local Area Network (c) 1 and 3 only
Which of the above are considered Short- (d) 1, 2 and 3
Range devices/technologies ?
(a) 1 and 2 only
4. Which one of the following is the context
(b) 2 and 3 only in which the term “qubit” is mentioned?
(c) 1 and 3 only (2022)
(d) 1, 2 and 3 (a) Cloud Services
(b) Quantum Computing
2. With reference to Web 3.0, consider the (c)
Visible Light Communication
following statements : (2022) Technologies
1. Web 3.0 technology enables people to (d) Wireless Communication Technologies
control their own data.
2. In the Web 3.0 world, there can be 5. Consider the following : (2022)
blockchain based social networks.
1. Aarogya Setu
3. Web 3.0 is operated by users collectively
rather than a corporation. 2. CoWIN

Which of the statements given above are 3. DigiLocker


correct? 4. DIKSHA
(a) 1 and 2 only Which of the above are built on top of
(b) 2 and 3 only open-source digital platforms ?

(c) 1 and 3 only (a) 1 and 2 only

(d) 1, 2 and 3 (b) 2, 3 and 4 only


(c) 1, 3 and 4 only

3. With reference to “Software as a Service (d) 1, 2, 3 and 4

645 IT & Communication Technology


unacademy.com | Download the Unacademy app
Give your feedback here: Link
6. In India, the term “Public Key Infrastructure” (c) 2 only
is used in the context of (2020)
(d) 1 and 3 only
(a) Digital security infrastructure
(b) Food security infrastructure
9. In the context of wearable technology,
(c) Health care and education infrastructure which of the following tasks is/are
(d) Telecommunication and transportation accomplished by wearable devices? (2019)
infrastructure 1. Location identification of a person.
7.
With reference to visible light 2. Sleep monitoring of a person.
communication (VLC) technology, which
3. Assisting the hearing-impaired person.
of the following statements are correct?
(2020) Select the correct answer using the code
given below:
1. VLC uses electromagnetic spectrum
wavelengths 375 to 780nm (a) 1 only

2. VLC is known as long-range optical (b) 2 and 3 only


wireless communication. (c) 3 only
3. VLC can transmit large amounts of data (d) 1, 2 and 3
faster than Bluetooth
4. VLC has no electromagnetic interference 10. Consider the following statements about a
Select the correct answer using the code digital signature: (2019)
given below 1. An electronic record that identifies the
(a) 1, 2 and 3 only certifying authority issuing it.
(b) 1, 2 and 4 only 2. Used to serve as a proof of identity of an
(c) 1, 3 and 4 only individual to access information from a
server on the Internet.
(d) 2, 3 and 4 only
3. An electronic method of signing an
electronic document and ensuring that
8. With reference to “Blockchain Technology”, the original content is unchanged.
consider the following statements: (2020)
Which of the statements given above is/are
1. It is a public ledger that everyone correct?
can inspect but which no single user
(a) 1 only
controls.
(b) 2 and 3 only
2. The structure and design of the
blockchain is such that all the data in it (c) 3 only
are about cryptocurrency only. (d) 1, 2 and 3
3. Applications that depend on basic
features of blockchain can be developed 11. In the context of digital technologies for
without anybody’s permission. entertainment, consider the following
Which of the statements given above is/are statements: (2019)
correct? 1. In Augmented Reality (AR), a simulated
(a) 1 only environment is created, and the physical
(b) 1 and 2 only world is completely shut out.

IT & Communication Technology 646


unacademy.com | Download the Unacademy app
Give your feedback here: Link
2. In Virtual Reality (VR), images generated take some groceries from your refrigerator
from the computer are projected onto for making breakfast, it recognizes the
real life objects or surroundings. shortage of stock in it and places an order
3. AR allows individuals to be present in for the supply of fresh grocery items.
the world and improves the experience When you step out of your house and lock
using the camera of a smart-phone or the door, all lights, fans, geysers and AC
PC. machines get switched off automatically.
On your way to the office, your car warns
4. VR closes the world, and transposes
you about traffic congestion ahead and
an individual, providing a complete
suggests an alternative route, and if you
immersion experience.
are late for a meeting, it sends a- message
Which of the statements given above is/are to your office accordingly. (2018)
correct?
In the context of emerging communication
(a) 1 and 2 only technologies, which one of the following
(b) 3 and 4 only terms best applies to the above scenario?
(2018)
(c) 1, 2 and 3 only
(a) Border Gateway Protocol
(d) 4 only
(b) Internet of Things
(c) Internet Protocol
12.
With reference to communication
technologies, what is/are the difference/ (d) Virtual Private Network
differences between LTE (Long-Term
Evolution) and VoLTE (Voice over Long-
14. With reference to ‘LiFi’, recently in the
Term Evolution)? (2019)
news, which of the following statements
1. LTE is commonly marketed as 3G is/are correct? (2016)
and VoLTE is commonly marketed as
1. It uses light as the medium for high-
advanced 3G.
speed data transmission.
2. LTE is data-only technology and VoLTE
2. It is wireless technology and is several
is voice-only technology.
times faster than ‘WiFi’.
Select the correct answer using the code
Select the correct answer using the codes
given below:
given below:
(a) 1 only
(a) 1 only
(b) 2 only
(b) 2 only
(c) Both 1 and 2
(c) Both 1 and 2
(d) Neither 1 nor 2
(d) Neither 1 nor 2

13. When the alarm of your smartphone rings


15.
With reference to ‘Near Field
in the morning, you wake up and tap it to
Communication (NFC) Technology’, which
stop the alarm which causes your geyser
of the following statements is/are correct?
to be switched on automatically. The
(2015)
smart mirror in your bathroom shows the
day's weather and also indicates the level 1. It is a contactless communication
of water in your overhead tank. After you technology that uses electromagnetic

647 IT & Communication Technology


unacademy.com | Download the Unacademy app
Give your feedback here: Link
radio fields. band, whereas Wi-Fi can use 2.4 GHz or
2. NFC is designed for use by devices which 5 GHz frequency band.
can be at distance of even a metre from (b) Bluetooth is used for wireless local area
each other. networks (WLAN) only, whereas Wi-Fi is
3. NFC can use encryption when sending used for wireless wide area networks
sensitive information. (WWAN) only.

Select the correct answer using the code (c)


When information is transmitted
given below: between two devices using blue-tooth
technology, the devices have to be in
(a) 1 and 2 only
the line of sight of each other, but when
(b) 3 only Wi-Fi technology is used the devices
(c) 1 and 3 only need not be in the line of sight of each
other.
(d) 1, 2 and 3
(d) The statements (a) and (b) given above
are correct in this context.
16. In addition to fingerprint scanning, which of
the following can be used in the biometric
identification of a person? (2014) 18. What is a “Virtual Private Network”? (2011)

1. Iris scanning (a) It is a private computer network of an


organisation where the remote users
2. Retinal scanning
can transmit encrypted information
3. Voice recognition through the server of the organisation.
Select the correct answer using the code (b) It is a computer network across a
given below: public internet that provides users
(a) 1 only access to their organisation's network
while maintaining the security of the
(b) 2 and 3 only
information transmitted.
(c) 1 and 3 only
(c) It is a computer network in which users
(d) 1, 2 and 3 can access a shared pool of computing
resources through a service provider.

17. What is the difference between Bluetooth (d) None of the statements (a), (b) and (c)
and Wi-Fi devices? (2011) given above is the correct description
of virtual private networks.
(a) Bluetooth uses 2.4 GHz radio frequency

IT & Communication Technology 648


unacademy.com | Download the Unacademy app
Give your feedback here: Link
IT & Communication Technology-
4 Explanation
1. Answer: (d) y Metering devices
Japan has its own satellite navigation system y Remote control
called the Quasi-Zenith Satellite System y Radio frequency identification (RFID)
(QZSS). QZSS is a satellite-based positioning
y Road Transport Telematics
and navigation system developed by the
Japanese government to provide reliable and y Telemetry.
accurate positioning information in Japan and Short range devices often benefit from a
the surrounding regions. relaxed regulatory regime compared with other
The QZSS constellation consists of multiple radio communications equipment.
satellites in geosynchronous orbit and inclined
orbit, allowing for increased visibility and
3. Answer: (d)
improved accuracy in urban areas where
tall buildings and other obstructions may Web 3.0 is the upcoming third generation of
affect satellite signals. The system works the internet where websites and apps will be
in conjunction with other global navigation able to process information in a smart human-
satellite systems like GPS (Global Positioning like way through technologies like machine
System) to enhance positioning accuracy and learning (ML), Big Data, decentralized ledger
availability. technology (DLT), etc. Web 3.0 was originally
called the Semantic Web by World Wide Web
inventor Tim Berners-Lee, and was aimed at
2. Answer: (d) being a more autonomous, intelligent, and
Option (d) is correct: Short-range open internet.
radio device is intended to cover radio Statement 1 is correct: Web 3.0 gives users
transmitters which provide either control of their data. Web 3.0 allows individuals
unidirectional or bidirectional communication. to not only own and control their data but also
Short Range Devices (SRD) are radio devices to get reimbursed for their online time. Users
that offer a low risk of interference with can engage in the management of internet
other radio services, usually because their protocols directly, rather than using free tech
transmitted power, and hence their range is platforms in exchange for their personal data,
low. The definition ‘Short Range Device’ may which is the case with platforms like Facebook
be applied to many different types of wireless and Instagram.
equipment, including various forms of:
Statement 2 is correct: Web 3.0 is also refers to
y Access control (including door and gate a decentralised internet based on blockchain,
openers) as they race to develop their own brands.
y Alarms and movement detectors Statement 3 is correct: With Web 3.0 being
y Closed-circuit television (CCTV) based on the incorporation of blockchain
technology into its operations, the new World
y Cordless audio devices, including wireless
Wide Web it will create will be decentralized,
microphones Industrial control
trustless (Web 3.0 takes out the middleman),
y Local Area Networks and permissionless (anybody may join and no
y Medical implants one can ever be prohibited).

649 IT & Communication Technology-Explanation


unacademy.com | Download the Unacademy app
Give your feedback here: Link
4. Answer: (d) the government is walking the talk, instead of
Software-as-a-Service (SaaS)—also known raising suspicion. The third part is security, if
as cloud-based software is now mainstream. there are any security lapses in the code, the
SaaS is a way of delivering applications over government and open-source community can
the Internet—as a service. Instead of installing collectively address the issue.
and maintaining software, you simply access Many solutions launched by the government
it via the Internet, freeing yourself from including Digilocker, Diksha, Aarogya Setu, and
complex software and hardware management. the Covid-19 vaccination platform CoWIN —
Statement 1 is correct: SaaS buyers can built on top of open-source digital platforms —
customize the user interface (UI) to change have benefited from valuable inputs provided
the look and feel of the program, as well as by volunteer open-source developers.
modify specific areas, such as data fields, to
alter what data appears.
7. Answer: (a)
Statement 2 is correct: Software as a Service
is used not only for desktop or laptop devices Option (a) is correct: Public Key Infrastructure
but also on various mobile devices. (PKI) is the combination of software, encryption
technologies, and services that enable entities
Statement 3 is correct: If you have used a web- to protect the security of their communications
based email service such as Outlook, Hotmail and business transactions on networks. Using
or Yahoo! Mail, then you have already used a a combination of private (e.g., secret) key
form of SaaS. With these services, you log into and public key cryptography, PKI enables a
your account over the Internet, often from a number of other security services, including
web browser. The email software is located data confidentiality, data integrity and non-
on the service provider’s network and your repudiation. PKI integrates digital certificates,
messages are stored there as well. You can public key cryptography, and certification
access your email and stored messages from authorities into one complete network security
a web browser on any computer or Internet- architecture. PKI essentially is a digital security
connected device. infrastructure.

5. Answer: (b) 8. Answer: (c)


Option (b) is correct: A qubit is a quantum bit, Visible Light Communication (VLC) technology
the counterpart in quantum computing to the is a wireless communication technology which
binary digit or bit of classical computing. Just utilizes light to transmit data and position
as a bit is the basic unit of information in a between devices.
classical computer, a qubit is the basic unit of
information in a quantum computer. Statement 1 is correct: Li-Fi is a Visible Light
Communications (VLC) system. Li-Fi standard-
compliant devices operate from 380 nm to 780
6. Answer: (d) nm wavelength. These devices operate on one
Option (d) is correct: Open source as the or multiple visible light frequency bands.
name implies is open. It means that the code Statement 2 is not correct: Long-range links are
is publicly accessible and hence anyone can provided by optical fibre (also known as fibre
edit and modify, and distribute it. Making the optic cables), and links from the long-range
code public has three key advantages. One, it end-points to end users are accomplished by
will make it accessible to the wider public for RF wireless, not by VLC.
scrutiny. Two, this will also bring in trust that Statement 3 is correct: VLC or Li-Fi that

IT & Communication Technology-Explanation 650


unacademy.com | Download the Unacademy app
Give your feedback here: Link
transmits data via light is said to be 100 times advantage of being useful in electromagnetic-
faster than Wi-Fi & Bluetooth as it can reach sensitive areas such as in aircraft cabins,
a speed of 224 Gbps. The Bluetooth standard’s hospitals, and nuclear power plants.
maximum data rate is 3 Mbps or about 2.1 after
protocol overheads take their cut. Wi-Fi, on Elimination Technique: VLC is known
the other hand, maxes out anywhere from 54 as short-range optical wireless
to 1300+ Mbps. communication. Statement 2 is eliminated.
Thus, Option (c) is correct.
Statement 4 is correct: Without causing
electromagnetic interference VLC has the

9. Answer: (d)
Statement 1 is correct: Blockchain is a distributed ledger technology (DLT) that allows data to be
stored globally on thousands of servers – while letting anyone on the network see everyone else’s

651 IT & Communication Technology-Explanation


unacademy.com | Download the Unacademy app
Give your feedback here: Link
entries in near real-time. That makes it difficult move and eat better.
for one user to gain control of, or game, the
network. It is a public ledger that everyone can
inspect but which no single user controls. 11. Answer: (d)

Statement 2 is not correct: Its application is not Option (d) is correct: Digital signature is a
only of cryptocurrency. It has a wide range of digital code, generated and authenticated by
applications in health, education, governance, public key encryption, which is attached to
etc. an electronically transmitted document to
verify its contents and the sender’s identity.
Statement 3 is correct: As per NITI Aayog, the Digital signatures are based on public key
majority of viable use cases for blockchain will cryptography, also known as asymmetric
be permissioned ones, not public blockchains. cryptography.
“Public blockchains like Bitcoin, have no central
authority and are regarded as enablers of total It serves as proof of the identity of an individual
disruptive disintermediation. Permissioned for a certain purpose; for example, a driver’s
blockchains are hosted on private computing licence identifies someone who can legally
networks, with controlled access and editing drive in a particular country. Likewise, a digital
rights, that is there are still central authorities certificate can be presented electronically to
with admin rights”. Thus, applications that prove one’s identity, to access information
depend on the basic features of blockchain can or services on the Internet or to sign certain
be developed without anybody’s permission. documents digitally.

Elimination Technique: The structure and


design of the blockchain is such that all the 12. Answer: (b)
data in it are about cryptocurrency only. Statements 1 and 2 are not correct: Augmented
Statement 2 is too narrow as blockchain reality (AR) is the superimposition of computer-
technology has wide applications and it generated images on existing environments to
can be eliminated. Option (d) is correct. make them more interactive. It is utilised in
the form of applications for mobile devices.
Whereas Virtual reality (VR) is a computer-
10. Answer: (d) generated simulation of real-life environments
Wearable technologies are electronic devices that are primarily achieved with the use of
that are physically worn by individuals in order headsets. When worn, the user’s vision and
to track, analyse and transmit personal data. hearing are stimulated to provide realistic
Option (d) is correct: Wearable devices can experiences.
track biometric data such as heart rate, Statements 3 and 4 are correct: VR completely
sleeping pattern, and location of a person puts the user in a simulated reality, AR blends
and also assists the hearing-impaired person. the virtual and real. Like VR, an AR experience
Wearable technologies use the application of involves some sort of goggles through which we
the Internet of Things (IoT) and are becoming can view a physical reality whose elements are
popular across various industries, especially in augmented (or supplemented) by computer-
the medical and fashion industries. generated sensory input such as sound, video,
Additional Information: graphics or GPS data.

Digital watches are classic examples of Elimination Technique: Virtual Reality (VR)
wearable technology. JawBone UP is the most and Augmented Reality are buzzwords in
popular fitness band around, Jawbone UP today’s world. Using our presence of mind,
helps people to understand their sleep cycles, we can see both terms are interchanged.

IT & Communication Technology-Explanation 652


unacademy.com | Download the Unacademy app
Give your feedback here: Link
Statement 2 is correct: LiFi is considered more
Thus, statements 1 and 2 are not correct. superior to WiFi. Data transmission tests have
Statement 3 and Statement 4 are correct. reached a speed of up to 224 Gbps, roughly
Thus, Option (b) is correct. 100 times faster than WiFi. LiFi is a wireless
technology that holds the key to solving
challenges faced by 5G. LiFi can transmit at
13. Answer: (d)
multiple gigabits and is more reliable, virtually
Option (d) is correct: Both LTE and VoLTE interference free and uniquely more secure
operate in 4G networks. LTE is a high-speed than radio technology such as Wi-Fi or cellular.
data communication system. It may or may
Radio frequency communication requires radio
not support data and voice call services at the
circuits, antennas, and complex receivers,
same time. It turns off the data connection
whereas LiFi is much simpler and uses direct
while making voice calls. Moreover, call
modulation methods similar to those used in
connection between two users is also slower.
low-cost infrared communications devices
On the contrary, VoLTE is an improved version
such as remote-control units. LED light bulbs
of LTE which provides much faster data
have high intensities and therefore can achieve
communication. It always supports data and
very large data rates.
voice call services at the same time.

16. Answer: (c)


14. Answer: (b)
Statement 1 is correct: Near Field
Option (b) is correct: Internet of Things (IoT)
Communication (NFC), is a short-range
is the interconnection via the internet of
wireless connectivity standard (Ecma-340,
computing devices embedded in everyday
ISO/IEC 18092) that uses an electromagnetic
objects, enabling them to send and receive
radio field to enable communication between
data.
devices. The technology behind NFC allows
It is the wireless sensor network (WSN) of a device, known as a reader, interrogator, or
physical devices, vehicles, home appliances active device, to create a radio frequency
and other items embedded with electronics, current that communicates with another NFC-
software, sensors, actuators, and connectivity compatible device or a small NFC tag holding
that enables these things to connect and the information the reader wants. Passive
exchange data, creating opportunities for devices, such as the NFC tag in smart posters,
more direct integration of the physical world store information and communicate with the
into computer-based systems, resulting in reader but do not actively read other devices.
efficiency improvements, economic benefits Peer-to-peer communication through two
and reduced human intervention. active devices is also a possibility with NFC.
This allows both devices to send and receive
15. Answer: (c) information.

Statement 1 is correct: LiFi is referred to as Statement 2 is not correct: NFC-enabled


Light Fidelity, a mobile wireless technology smartphones or NFC-enabled devices to
that uses light rather than radio frequencies establish radio communication with each other
to transmit data. The technology is supported by touching the devices together or bringing
by a global ecosystem of companies driving them into proximity to a distance of typically
the adoption of LiFi, the next generation of 10 cm (3.9 in) or less. The communication
wireless that is ready for seamless integration between the devices can not be established at
into the 5G core. a distance of a meter.

653 IT & Communication Technology-Explanation


unacademy.com | Download the Unacademy app
Give your feedback here: Link
Statement 3 is correct: It establishes a secure communication through Radio Frequency
channel and uses encryption when sending has many advantages as it does not require
sensitive information such as credit card any line of sight connection between the
numbers to ensure security. transmitter and receiver while the line of sight
connection exists in infrared communication.
Both Bluetooth and Wi-Fi use radio frequency
17. Answer: (d) for communication, the connecting devices
Option (d) is correct: (whether Bluetooth-enabled, or Wi-Fi-enabled)
Biometric identification is a technique that do not have to be in the line of sight of each
uses unique human characteristics to identify other.
an individual. There are mainly two categories y Line of sight (LoS) is a type of propagation
of biometrics identification: physiological that can transmit and receive data only
characteristics and behavioural characteristics. where transmit and receive stations are
y Physiological biometrics has to do with the in view of each other without any sort of
physical traits of a person. For example, obstacle between them.
fingerprint, a physiological characteristic,
does not usually change except for 19. Answer: (b)
accidents or illness.
Option (b) is correct: A virtual private network,
y Behavioural biometrics has to do with or VPN, is an encrypted connection over the
the things that can change with the Internet from a device to a network. The
environment. For example, signatures, encrypted connection helps ensure that
speech patterns, voice, and keystrokes are sensitive data is safely transmitted. It prevents
behavioural biometrics. unauthorised people from eavesdropping on
y Iris scanning, retinal scanning, voice the traffic and allows the user to conduct work
recognition, etc., are used in the biometric remotely.
identification of a person. Over the Internet from a device to a network
virtual private network acts as an encrypted
18. Answer: (a) connection. In corporate environments, VPN
technology is widely used. This encrypted
Option (a) is correct: Bluetooth uses the
connection helps us to ensure that sensitive
microwave radio frequency spectrum in the
data is safely transmitted. It also prevents
2.402 GHz to 2.480 GHz range. Wi-Fi is based
unauthorised people from eavesdropping on
upon IEEE 802.11 standards that uses 2.4, 3.6
the traffic and allows the user to conduct work
and 5 GHz, frequency bands. IEEE 802.11 is
remotely.
for implementation of the wireless local area
network (WLAN) using the above frequencies
and makes the basis of Wi-Fi wireless networks.
Option (b) is not correct: Bluetooth is also used
for Personal Area Network (PAN) and not only
used for wireless local area networks (WLAN).
Whereas Wi-Fi is used for Local Area Network
(LAN).
Option (c) is not correct: Wireless

IT & Communication Technology-Explanation 654


unacademy.com | Download the Unacademy app
Give your feedback here: Link
5 Energy

1. India is an important member of the that generates electricity by direct


‘International Thermonuclear Experimental conversion of light into electricity, while
Reactor’. If this experiment succeeds, what 'Solar Thermal' is a technology that
is the immediate advantage for India? utilises the Sun's rays to generate heat
(2016) which is further used in the electricity
(a) It can use thorium in place of uranium generation process.
for power generation 2. Photovoltaics generates Alternating
(b) It can attain a global role in satellite Current (AC), while Solar Thermal
navigation generates Direct Current (DC).

(c) It can drastically improve the efficiency 3. India has a manufacturing base for
of its fission reactors in power Solar Thermal Technology but not for
generation photovoltaics.

(d) It can build fusion reactors for power Which of the statements given above is/are
generation correct?
(a) 1 only

2. With reference to ‘fuel cells’ in which (b) 2 and 3 only


hydrogen-rich fuel and oxygen are used to (c) 1, 2 and 3
generate electricity. Consider the following (d) None
statements: (2015)
1. If pure hydrogen is used as a fuel, the
fuel cell emits heat and water as by- 4. To meet its rapidly growing energy demand,
products. some opine that India should pursue
research and development on thorium as
2. Fuel cells can be used for powering the future fuel of nuclear energy. In this
buildings and not for small devices like context, what advantage does thorium
laptop computers. hold over uranium? (2012)
3. Fuel cells produce electricity in the 1. Thorium is far more abundant in nature
form of Alternating Current (AC). than uranium.
Which of the statements given above is/are 2. On the basis of per unit mass of mined
correct? mineral, thorium can generate more
(a) 1 only energy compared to natural uranium.
(b) 2 and 3 only 3. Thorium produces less harmful waste
(c) 1 and 3 only compared to uranium.

(d) 1, 2 and 3 Which of the statements given above is/are


correct?
(a) 1 only
3. With reference to technology for solar
power production, consider the following (b) 2 and 3 only
statements: (2014) (c) 1 and 3 only
1. 'Photovoltaics' is a technology (d) 1, 2 and 3

655 Energy
unacademy.com | Download the Unacademy app
Give your feedback here: Link
5. Microbial fuel cells are considered a source (a) 1 only
of sustainable energy. Why? (2011)
(b) 2 and 3 only
1. They use living organisms as catalysts
(c) 1 and 3 only
to generate electricity from certain
substrates. (d) 1, 2 and 3

2. They use a variety of inorganic materials


as substrates. 6. The function of heavy water in a nuclear
3. They can be installed in wastewater reactor is to? (2011)
treatment plants to cleanse water and (a) Slow down the speed of neutrons.
produce electricity. (b) Increase the speed of neutrons.
Which of the following statements given (c) Cooldown the reactor.
above is/are correct?
(d) Stop the nuclear reaction.

Energy 656
unacademy.com | Download the Unacademy app
Give your feedback here: Link
5 Energy-Explanation

1. Answer: (d) as a hydrogen, is fed to the anode, and the air


Option (d) is correct: ‘International is fed to the cathode. 
Thermonuclear Experimental Reactor’ (ITER) is Elimination Technique: Fuel cells produce
one of the most ambitious energy projects in electricity in the form of direct current
the world today. Through ITER, many nations (DC). Statement 3 is eliminated. Thus,
are collaborating to build the world’s largest Option (a) is correct.
tokamak, a magnetic fusion device that has
been designed to prove the feasibility of
fusion as a large-scale and carbon-free source Fuel cells produce electricity in the form of
of energy based on the same principle that direct current (DC)
powers our Sun and stars.
3. Answer: (a)
2. Answer: (a) Photovoltaics is the direct conversion of light
Statement 1 is correct: A fuel cell uses the into electricity at the atomic level. Some
chemical energy of hydrogen or another fuel materials exhibit a property known as the
to cleanly and efficiently produce electricity. photoelectric effect that causes them to
If hydrogen is the fuel, electricity, water, and absorb photons of light and release electrons.
heat are the only products. Fuel cells work like When these free electrons are captured, an
batteries, but they do not run down or need electric current results that can be used as
recharging. They produce electricity and heat electricity. Solar Thermal energy is one of the
as long as fuel is supplied. In a hydrogen fuel most promising renewable energy resources.
cell, a catalyst at the anode separates hydrogen y Solar Thermal technologies convert solar
molecules into protons and electrons, which radiation (utilises sun rays) into heat that
take different paths to the cathode. The either can be directly utilised for various
electrons go through an external circuit, applications or can be transformed into
creating a flow of electricity. electricity to serve any purpose as deemed
Statement 2 is not correct: Fuel cells are from conventional electricity. (Statement 1
unique in terms of the variety of their potential is correct)
applications; they can provide power for y Both Photovoltaic cells and Solar Thermal
systems as large as a utility power station and generate direct current (DC). (Statement 2
as small as a laptop computer. is not correct)
Statement 3 is not correct: Alternating Current y India has a manufacturing base for both
(AC) is produced by an electric generator which technologies- Solar Thermal technology
consists of a magnet and a loop of wire which and photovoltaics. (Statement 3 is not
rotates in the magnetic field of the magnet. It correct)
is the type of electric current generated by the
vast majority of power plants and used by most
4. Answer: (d)
power distribution systems. A fuel cell consists
of two electrodes: a negative electrode (or Statement 1 is correct: Thorium is a useful
anode) and a positive electrode (or cathode) fuel of a nuclear reactor. It was discovered in
sandwiched around an electrolyte. A fuel, such 1828 by Jons Jacob Berzelius. Thorium (Th) is

657 Energy-Explanation
unacademy.com | Download the Unacademy app
Give your feedback here: Link
a radioactive element of the actinoid series of and energy by a population of bacteria each of
the periodic table. Its atomic number is 90. It which is roughly 1 micron in size. Consumption
is four times more abundant in nature than of these compounds is a normal part of the
uranium and is widely distributed throughout microorganism’s metabolism; in a microbial fuel
the Earth’s crust. cell part of the energy liberated is harvested in
Statement 2 is correct: Thorium fuel generates the form of electricity.
no new bomb-usable material in the waste Microbial fuel cells have various practical
profile. Thorium fuel waste consists of the applications such as in breweries, domestic
radioisotope Uranium-233, or U233, which is wastewater treatment, desalination plants,
virtually impossible to weaponize/ Thorium hydrogen production, remote sensing, and
fuel will generate more energy per unit of mass pollution remediation, and they can be used as
than uranium fuel by a factor of approximately a remote power source.
30.
Statement 3 is correct: Thorium is silvery white 6. Answer: (a) 
but turns gray or black on exposure to air.
According to the UN nuclear agency IAEA There Option (a) is correct: Nuclear reactors are the
is limited radioactive debris when thorium is heart of a nuclear power plant. They contain and
used. And in terms of chemical stability and control nuclear chain reactions that produce
resistance to radioactivity thorium is a safer heat through a physical process called fission.
alternative compared to uranium. That heat is used to make steam that spins
a turbine to create electricity. In a nuclear
reactor, heavy water is used to slow down
5. Answer: (c) the neutrons. A neutron hitting a deuterium
Microbial fuel cells (MFCs) are a new nucleus slows down quickly. A neutron hitting
bioelectrochemical process that aims to a deuterium nucleus slows down quickly.
produce electricity by using the electrons Heavy water is deuterium oxide or D20. The
derived from biochemical reactions catalysed world’s second-largest producer of heavy
by bacteria. water in India. Heavy water is used as a
Option (c) is correct: Microbial fuel cells are low- coolant as well as a neutron moderator. It is
power devices that use microbes (single-celled chemically the same as normal water (H20) but
living organisms) to convert wastewater into the hydrogen atoms are of the heavy isotope
electricity. A huge range of organic compounds deuterium in which the nucleus contains a
can be converted into carbon dioxide, water, neutron in addition to protons found in the
nucleus of any hydrogen atom. 

Energy-Explanation 658
unacademy.com | Download the Unacademy app
Give your feedback here: Link
Additional Information: This is called a chain reaction.
A nuclear reactor is driven by the splitting The fissioning of atoms in the chain reaction
of atoms, a process called fission, where a also releases a large amount of energy as heat.
particle (a ‘neutron’) is fired at an atom, which The generated heat is removed from the reactor
then fissions into two smaller atoms and some by a circulating fluid, typically water. This heat
additional neutrons. Some of the neutrons can then be used to generate steam, which
that are released then hit other atoms, causing drives turbines for electricity production.
them to fission too and release more neutrons.

659 Energy-Explanation
unacademy.com | Download the Unacademy app
Give your feedback here: Link
6 Miscellaneous

1. Consider the following statements : (2022) surface pumps and not for submersible
1. Other than those made by humans, pumps,
nanoparticles do not exist in nature. 2. Solar power can be used for running
2. Nanoparticles of some metallic oxides centrifugal pumps and not the ones
are used in the manufacture of some with piston.
cosmetics. Which of the statements given above is/are
3. Nanoparticles of some commercial correct?
products which enter the environment (a) 1 only
are unsafe for humans. (b) 2 only
Which of the statements given above is/are (c) Both 1 and 2
correct ?
(d) Neither 1 nor 2
(a) 1 only
(b) 3 only
5. With reference to the carbon nanotubes,
(c) 1 and 2 consider the following statement: (2020)
(d) 2 and 3 1. They can be used as the carriers of
drugs and antigens in the human body.
2. Bisphenol A (BPA), a cause of concern, 2. They can be made into artificial blood
is a structural/key component in the capillaries for an injured part of the
manufacture of which of the following human body.
kinds of plastics? (2021) 3. They can be used in biochemical
(a) Low-density polyethylene sensors.
(b) Polycarbonate 4. Carbon nanotubes are biodegradable.
(c) Polyethylene terephthalate Which of the statements given above are
(d) Polyvinyl chloride correct?
(a) 1 and 2 only

3. ‘Triclosan’, consider harmful when exposed (b) 2, 3 and 4 only


to high levels for a long time, is most likely (c) 1, 3 and 4 only
present in which of the following? (2021) (d) 1, 2, 3 and 4
(a) Food preservatives
(b) Fruit-ripening substances 6. Consider the following activities: (2020)
(c) Reused plastic containers 1. Spraying pesticides on a crop field
(d) Toiletries 2. Inspecting the craters of active
volcanoes
4. With reference to solar water pumps, 3. Collecting breath samples from
consider the following statements:(2020) spouting whales for DNA analysis
1. Solar power can be used for running At the present level of technology, which

Miscellaneous 660
unacademy.com | Download the Unacademy app
Give your feedback here: Link
of the above activities can be successfully Appellate Board.
carried out by using drones?
3. Plant varieties are not eligible to be
(a) 1 and 2 only patented in India.
(b) 2 and 3 only Which of the statements given above is/are
(c) 1 and 3 only correct?

(d) 1, 2 and 3 (a) 1 and 3 only


(b) 2 and 3 only

7. With the present state of development, (c) 3 only


Artificial Intelligence can effectively do (d) 1, 2 and 3
which of the following? (2020)
1. Bring down electricity consumption in
10. "3D printing" has applications in which of
industrial units.
the following? (2018)
2. Create meaningful short stories and
1. Preparation of confectionery items
songs.
2. Manufacture of bionic ears
3. Disease diagnosis.
3. Automotive industry
4. Text-to-speech conversion.
4. Reconstructive surgeries
5. Wireless transmission of electrical
energy. 5. Data processing technologies

Select the correct answer using the code Select the correct answer using the code
given below: given below:

(a) 1, 2, 3 and 5 only (a) 1, 3 and 4 only

(b) 1, 3, and 4 only (b) 2, 3 and 5 only

(c) 2, 4, and 5 only (c) 1 and 4 only

(d) 1, 2, 3, 4 and 5 (d) 1, 2, 3, 4 and 5

8. The word ‘Denisovan’ is sometimes 11. Consider the following pairs: (2018)
mentioned in media in reference to: (2019) Terms sometimes Context/Topic
(a) fossil of a kind of dinosaur. seen in news
(b) an early human species. 1. Belle II experiment Artificial
Intelligence
(c) a cave system found in North-East
India. 2. Blockchain Digital/
technology Cryptocurrency
(d) a geological period in the history of the
Indian subcontinent. 3. CRISPR-Cas9 Particle Physics
Which of the pairs given above is/are
correctly matched?
9. Consider the following statements: (2019)
(a) 1 and 3 only
1. According to the Indian Patent Act, a
biological process to create a seed can (b) 2 only
be patented in India. (c) 2 and 3 only
2. In India, there is no Intellectual Property (d) 1, 2 and 3

661 Miscellaneous
unacademy.com | Download the Unacademy app
Give your feedback here: Link
12. Consider the following pairs: (2017) 2. Nanotechnology can largely contribute
to gene therapy.
Commonly Unwanted or
used/ controversial chemicals Select the correct answer using the code
consumed likely to be found in given below:
materials them (a) 1 only
1. Lipstick Lead (b) 2 only
2. Soft drinks Brominated vegetable (c) Both 1 and 2
oils
(d) Neither 1 nor 2
3. Chinese fast Monosodium glutamate
food
15. The efforts to detect the existence of Higgs
Which of the pairs given above is/are
boson particles have become frequent
correctly matched?
news in the recent past. What is /are the
(a) 1 only importance/importance of discovering this
(b) 2 and 3 only particle? (2013)
(c) 1 and 3 only 1. It will enable us to understand why
(d) 1, 2 and 3 elementary particles have mass.
2. It will enable us in the near future to
develop the technology to transfer
13. Organic Light-Emitting Diodes (OLEDs) matter from one point to another
are used to create digital displays in many without traversing the physical space
devices. What is/are the advantage of between them.
OLED displays over Liquid Crystal displays?
(2017) 3. It will enable us to create better fuels
for nuclear fission.
1. OLED displays can be fabricated on
flexible plastic substrates. Select the correct answer using the code
given below:
2. Roll-up displays embedded in clothing
can be made using OLEDs. (a) 1 only

3. Transparent displays are possible using (b) 2 and 3 only


OLEDs. (c) 1 and 3 only
Select the correct answer using the codes (d) 1, 2 and 3
given below:
(a) 1 and 3 only 16. Graphene is frequently in the news recently.
(b) 2 only What is its importance? (2012)
(c) 1, 2 and 3 1. It is a two-dimensional material and
(d) None of the above statements is correct has good electrical conductivity.
2. It is one of the thinnest but strongest
materials tested so far.
14. With reference to the use of nanotechnology
in the health sector, which of the following 3. It is entirely made of silicon and has
statements is/are correct? (2015) high optical transparency.

1. Targeted drug delivery is made possible 4. It can be used as ‘conducting electrodes’


by nanotechnology. required for touch screens, LCDs and
organic LEDs.

Miscellaneous 662
unacademy.com | Download the Unacademy app
Give your feedback here: Link
Which of the statements given above are 2. Compared to a DVD, the Blu-ray disc
correct? format has several times more storage
(a) 1 and 2 only capacity.

(b) 3 and 4 only 3. Thickness of Blu-ray discs is 2.4 mm


while that of DVD is 1.2 mm.
(c) 1, 2 and 4 only
Which of the statements given above is/are
(d) 1, 2, 3 and 4 correct?
(a) 1 only
17. A new optical disc format known as the (b) 1 and 2 only
Blu-ray disc is becoming popular. In what
way is it different from the traditional (c) 2 and 3 only
DVD? (2011) (d) 1, 2 and 3.
1. DVD supports standard definition
video while Blu-ray disc supports high-
definition video.

663 Miscellaneous
unacademy.com | Download the Unacademy app
Give your feedback here: Link
6 Miscellaneous-Explanation

1. Answer: (d) the protective internal epoxy resin coatings


Statement 1 is not correct: Nature itself is of canned foods and from consumer products
an excellent nanotechnologist. It provides us such as polycarbonate tableware, food storage
with a range of fine particles, from inorganic containers, water bottles, and baby bottles
ash, soot, sulfur and mineral particles found
in the air or in wells, to sulfur and selenium 3. Answer: (d)
nanoparticles produced by many bacteria
Option (d) is correct: Triclosan is an anti-
and yeasts. These nanomaterials are entirely
microbial chemical. Triclosan can stop the
natural.
growth of unwanted microorganisms, which
Statement 2 is correct: Among metal and metal may degrade the product and decrease
oxide nanoparticles (NPs) potentially present in its shelf life. Triclosan is used in personal
cosmetics, those containing titanium dioxide care products such as soap, toothpaste,
and zinc oxide are common ingredients added mouthwash, shampoo, body washes and
to obtain sufficient sun protection. some cosmetics. It’s also an ingredient in
Statement 3 is correct: The nanoparticle may some cleaning products, paints, etc. It is
be unsafe: Due to their immensely small size, used to prevent bacterial contamination, very
these materials may exhibit different physical, commonly used in wet products to increase
chemical, and biological properties, and shelf life. It is known to cause skin irritation.
penetrate cells more easily.

Elimination Technique: Other than those 4. Answer: (d)


made by humans, nanoparticles do not Solar water pumps, a relatively new concept in
exist in nature. Statement 1 is too extreme. mechanics, are commonly seen in residential
Nanoparticles of some commercial and commercial uses, as well as for irrigation
products which enter the environment are of agricultural land. A typical solar-powered
unsafe for humans. Statement 3 is correct pumping system consists of a solar panel array
as the harmful effects of nanoparticles are that powers an electric motor, which in turn
in the news. Thus, Option (d) is correct. powers a bore or surface pump.
Statement 2 is not correct: Solar power can be
2. Answer: (b) used for both running centrifugal pumps and
Option (b) is correct: An industrial chemical that ones with pistons.
has been around since the 1960s, Bisphenol A When it comes to stand -alone solar pumping
(BPA), is used to make polycarbonate plastics. systems, the main types include rotating and
Polycarbonate plastic is used to make hard positive displacement pumps.
plastic items, such as baby bottles, reusable Centrifugal pumps are the common choice
water bottles, food containers, pitchers, for rotation and are designed for fixed-head
tableware and other storage containers. applications. They have increased output in
Bisphenol A (BPA) is also found in epoxy resins, proportion to their speed of rotation. A positive
which act as a protective lining on the inside displacement (PD) pump moves a fluid by
of some metal-based food and beverage repeatedly enclosing a fixed volume and
cans. Bisphenol A can leach into food from moving it mechanically through the system.
The pumping action is cyclic and can be driven

Miscellaneous-Explanation 664
unacademy.com | Download the Unacademy app
Give your feedback here: Link
by pistons, screws, gears, rollers, diaphragms, based on their placement (underwater and
or vanes. above the waterline). Solar power can be
Statement 1 is not correct: Also, pumps are used for both running centrifugal pumps and
classified as submersible and surface pumps, ones with pistons.

5. Answer: (d) genes, vaccines, antibodies, biosensors, etc.).


A carbon nanotube (CNT) is a miniature Hence, they can be made into artificial blood
cylindrical carbon structure that has hexagonal capillaries for an injured part of the human
graphite molecules attached at the edges. body. Other applications of CNTs have been
Carbon nanotubes have the potential to be for tissue regeneration, biosensor diagnosis,
used as semiconductors, potentially replacing enantiomer separation of chiral drugs, and
silicon in a wide variety of computing devices. extraction and analysis of drugs and pollutants.

Statement 1 is correct: In case of Cancer, CNTs Statement 3 is correct: New materials CNTs are
as drug delivery vehicles (carriers) have shown promising building blocks for biosensors due to
potential in targeting specific cancer cells with their unique electronic and optical properties.
a dosage lower than conventional drugs used. Carbon nanotubes are rolled-up cylinders of
carbon monolayers (graphene).
Statement 2 is correct: CNTs have been
successfully applied in pharmacy and medicine Statement 4 is correct: Microbes including
due to their high surface area that is capable bacteria and fungi have the ability to degrade
of adsorbing or conjugating with a wide variety carbon nanotubes (CNTs), graphene (GRA), and
of therapeutic and diagnostic agents (drugs, their derivatives.

665 Miscellaneous-Explanation
unacademy.com | Download the Unacademy app
Give your feedback here: Link
6. Answer: (d) been in practice for quite some time. David
Drones are Unmanned aerial vehicles, aircraft Bowie helped develop an app called the
that flies without a human pilot aboard. Verbasizer in the ‘90s, which took the literary
source material and randomly reordered the
Statement 1 is correct: To surmount the words to create new combinations that could
shortage of farm labour and rising labour be used as lyrics.
charges, some progressive farmers are trying
to harness drone technology to find a lasting Statement 3 is correct: Artificial Intelligence is
solution to their recurring problems in every being used to diagnose various diseases, such
cropping season. Thus, it can be helpful in as Alzheimer’s disease early by reviewing brain
spraying pesticides on a crop field. scans. In the Italy University of Bari and Istituto
Nazionale di Fisica Nucleare have developed a
Statement 2 is correct: Drones have become new machine learning AI technology that helps
a critical tool for professionals working in a identify Alzheimer’s a decade before doctors
large variety of different sectors such as public usually can, by way of non-invasive MRI brain
safety, search and rescue, infrastructure scans.
inspection, surveying and mapping, wildlife
conservation and many more. It is also useful Statement 4 is correct: An easy-to-use text-
in Inspecting the craters of active volcanoes. to-speech conversion tool powered by AI is
Lister which is available in the market. It helps
Statement 3 is correct: Marine researcher to convert all texts into a speech with human-
Vanessa Pirotta of Sydney’s Macquarie sounding voices.
University, says a drone has been used for
the first time to collect whale mucus from Statement 5 is correct: Wireless smart control,
humpback whales at sea in a technique that or smart off-grid, or wireless transmission of
could help monitor the health of whales around electrical energy is going to be what enables
the world. this big shift in the power industry. Big Data,
advanced analytics, and artificial intelligence
Elimination Technique: Drone technology (AI) will also have a material impact on
is developing at a rapid rate and being performance and capabilities of off-grid
used in a variety of applications. All these systems.
activities are possible through the present
level of drone technology.
8. Answer: (b)
Note: Always watch out for new headlines
in development in the science and Option (b) is correct: Denisovans are an extinct
technology field. species of hominid and a close relative to
modern humans. Denisovans may have ranged
from Siberia to Southeast Asia during the last
7. Answer: (d) Ice Age. DNA evidence suggests Denisovans
Statement 1 is correct: Artificial Intelligence are related to both Neanderthals and modern
will be the brain of the future smart grid. humans.
The technology will continuously collect and Neanderthals were an extinct relative of
synthesize overwhelming amounts of data modern humans once found across Europe,
from millions of smart sensors nationwide to extending into Central and Southwest Asia.
make timely decisions on how to best allocate
energy resources. It will help in bringing down
electricity consumption in industrial units. 9. Answer: (c)

Statement 2 is correct: To create meaningful The Indian Patent Act, 1970 strikes a balance
short stories and songs using AI as a tool has between the rights of the applicant and his

Miscellaneous-Explanation 666
unacademy.com | Download the Unacademy app
Give your feedback here: Link
obligation to the society granting the rights. in the anatomic geometry of a human ear
Some salient features of the Act include in America.
product and process patent, term of the patent y Industrial 3D printers have opened new
as 20 years, patent examination conducted paths at each stage of the production of
on request, fast track mechanism for quick motor vehicles in the automotive industry.
disposal of appeals, pre-grant and post-grant y  It has been used in reconstructive surgery
opposition allowed, protection of biodiversity to help repair damaged tissue; for example
and traditional knowledge, and publication of - in cases of birth defects.
applications after 18 months of date of filing of
the patent application. y 3D scanning is the process of analysing
a real-world object or environment to
Statement 1 is not correct: Under Section 3(j) collect data on its shape and possibly its
of India’s Patents Act, a seed or a plant, or a appearance (e.g., colour) then the collected
biological process to create a seed or plant data can be used to Construct or Print
cannot be patented. digital 3D models, hence, it is useful in data
Statement 2 is not correct: Intellectual processing technologies as well.
Property Appellate Board has been constituted
by the Government of India in the Ministry of
Commerce and Industry in September 2003 11. Answer: (b)
to hear appeals against the decisions of the Pair 1 is not correctly matched: The Belle II
Registrar under the Trademarks Act, 1999 experiment is a particle physics experiment
and the Geographical Indications of Goods designed to study the properties of B mesons
(Registration and Protection) Act, 1999. (heavy particles containing a bottom quark).
Statement 3 is correct: Any of the variety Pair 2 is correctly matched: Blockchain is a
as mentioned are not inventions, hence not system which helps in recording information.
patentable: “plants and animals in whole or in The information is recorded in such a way
any part thereof other than microorganisms; that it makes it difficult for anyone to hack or
but including seeds, varieties, and species, and cheat the system. Unlike physical currencies,
essentially biological processes for production blockchain cryptocurrency works on digital
or propagation of plants and animals”. channels and are often adhered to strong
cryptography to secure financial transactions
that happen online. Blockchain technology -
10. Answer: (d)  Digital/ Cryptocurrency.
3D printing is additive manufacturing Pair 3 is not correctly matched: CRISPR- Cas9 is
technology where a 3D object is created by a Gene-editing tool that can be used to modify
laying down successive layers of material. From DNA in cells. CRISPR-Cas9 was adapted from
construction to medical technology, 3D printing a naturally occurring genome editing system in
is being used across a range of industries. Its bacteria. The bacteria capture snippets of DNA
use in the food industry continues to grow. from invading viruses and use them to create
Option (d) is correct: DNA segments known as CRISPR arrays. The
y 3D printing technology has applications in CRISPR arrays allow the bacteria to “remember”
the preparations of confectionery items, the viruses (or closely related ones).
focused primarily on the chocolate and Elimination Technique: CRISPR- Cas9
hard sugar candy industry. is a Gene-editing tool ( it is not related
y 3D-printed bionic ears were generated via to Particle Physics). Pair 3 is eliminated.
3D printing of a cell-seeded hydrogel matrix Thus, Option (b) is correct.

667 Miscellaneous-Explanation
unacademy.com | Download the Unacademy app
Give your feedback here: Link
12. Answer:   technology is used for heads-up displays.
Pair 1 is correctly matched: Over the years, there Elimination Technique: Any technologically
have been many reports alleging dangerous intensive question that is associated with
levels of lead to be used in lipstick. Research the word ‘can’ should be true in most
has shown that 61 per cent of lipsticks contain cases. Thus, Option (c) is correct.
lead, with levels ranging up to 0.65 parts per
million.
Pair 2 is correctly matched: Brominated 14. Answer: (c) 
vegetable oil (BVO) is a complex mixture Nanotechnology is the manipulation and
of plant-derived triglycerides. Brominated manufacture of materials and devices on the
vegetable oil is used primarily to help emulsify scale of atoms or small groups of atoms. The
citrus-flavoured soft drinks, preventing them word Nano means very small and the size of
from separating during distribution. Brominated Nanometer. Nanotechnology has a wide range
vegetable oil has been used by the soft drink of applications in the field of electronics,
industry since 1931, generally at a level of about medical, biotechnology, etc.
eight ppm. In the health sector, nanomaterials are used
Pair 3 is correctly matched: Monosodium for the diagnosis, treatment, control, and
glutamate (MSG) is a flavour enhancer prevention of diseases. Nanoparticles lead to
commonly added to Chinese food, canned the development of better and safer medicine,
vegetables, soups and processed meats. tissue-targeted actions and custom-made
The Food and Drug Administration (FDA) has nanomedicines. 
classified MSG as a food ingredient that’s Statement 1 is correct: Nanotechnology drug
“generally recognized as safe,” but its use delivery applications occur through the use
remains controversial. of designed nanomaterials as well as forming
delivery systems from nanoscale molecules
13. Answer: (c)  such as liposomes. Targeted drug delivery is
made possible by nanotechnology. It means
Organic Light-Emitting Diodes (OLED) is a delivery of a drug to the site of action not to all
flat light emitting technology, which can be organs by somehow nano-tagging the drug. In
created by placing a series of organic thin films drug delivery, nanotechnology is just beginning
between two conductors. to make an impact. Many of the current “nano”
Statement 1 and statement 2 are correct: drug delivery systems, however, are remnants
OLED displays are fabricated on flexible plastic of conventional drug delivery systems that
substrates. It leads to the possible fabrication happen to be in the nanometre range, such as
of flexible organic light-emitting diodes for liposomes, polymeric micelles, nanoparticles,
other new applications—for example,  roll- dendrimers, and nanocrystals.
up displays embedded in fabrics or clothing. Statement 2 is correct: Nanotechnology can
If a substrate like polyethylene terephthalate largely contribute to gene therapy also. Gene
(PET) is used, the displays are produced therapy by means of nanotechnology has the
inexpensively. potential of helping the body treat its diseases
Statement 3 is  correct: Transparent OLED by means of its own genes and cells.
(T-OLED) has transparent components like
substrate, cathode and anode and, when Elimination Technique: Statements like
turned off, are up to 85 percent as transparent 1 and 2 are generally true because the
as to their substrate. When it is turned on, it negation of like these statements would
allows light to pass in both directions. This be very difficult, especially in science and

Miscellaneous-Explanation 668
unacademy.com | Download the Unacademy app
Give your feedback here: Link
one of the most pliable. It is also called “The
technology questions. Thus, Option (c) is Wonder Material”.
correct.
Statement 3 is not correct: Graphene is made of
a single layer of carbon atoms (it is not entirely
15. Answer: (a)  made of Silicon) that are bonded together in
a repeating pattern of hexagons. Graphene is
Option (a) is correct: The Higgs boson was
one million times thinner than paper. Carbon is
proposed in 1964 by Peter Higgs, François
an incredibly versatile element. Depending on
Englert, and four other theorists to explain
how atoms are arranged, it can produce hard
why certain particles have mass. Scientists
diamonds or soft graphite.
confirmed its existence in 2012 through the
ATLAS and CMS experiments at the Large Statement 4 is correct: Graphene is a promising
Hadron Collider (LHC) at CERN in Switzerland. next-generation conducting material with
This discovery led to the 2013 Nobel Prize in the potential to replace traditional electrode
Physics being awarded to Higgs and Englert. materials such as indium tin oxide in electrical
and optical devices. It combines several
The Higgs boson is the fundamental particle
advantageous characteristics including low
associated with the Higgs field, a field that
sheet resistance, high optical transparency, and
gives mass to other fundamental particles
excellent mechanical properties. Graphene can
such as electrons and quarks. A particle’s mass
be used as ‘conducting electrodes’ required for
determines how much it resists changing its
touch screens, LCDs and organic LEDs. However,
speed or position when it encounters a force.
recent research has coincided with increased
Not all fundamental particles have mass. The
interest in the application of graphene as an
photon, which is a particle of light and carries
electrode material in transistors, light-emitting
electromagnetic force, has no mass at all.
diodes, solar cells and flexible devices.

Elimination Technique: Graphene is


16. Answer: (c)
made of a single layer of carbon atoms.
Graphene, the world’s slenderest material, Statement 3 is eliminated. It is a good
could help shape the next generation of conductor of electricity. So, it can be used
computers that would make their existing as electrode material in organic LEDs.
counterparts seem like stone-age relics. Statement 4 is correct. Thus, Option (c) is
Graphene, a single layer of carbon atoms, correct.
whose discoverers shared the 2010 Nobel Prize
for physics for work on the material.
17. Answer: (b)
Statement 1 is correct: It is a two-dimensional
(2D) material composed of carbon atoms linked Statement 1 is correct: Blu-ray disc is an
in a hexagonal lattice extracted from graphite. optical disc format that is mainly used to
Graphene has good electrical conductivity as enable recording and rewriting, play high-
it permits flow of electrons at much higher definition video, and store large amounts of
speeds than they do in silicon, the substance data. Whereas, the DVD, commonly known as
that existing computer chips rely on. Moreover, Digital Video Disc is a digital optical disc storage
Due to its exceptional quantum properties, it format used to store operating systems and
has applications in nanoelectronics. other data including standard-definition video
(not high-definition video).
Statement 2 is correct: Graphene is the
strongest (200 times tougher than steel), and Statement 2 is correct: Blue-ray disc format
thinnest material known to exist. It is not only has more storage capacity than a DVD. A typical
the hardest material in the world but also Blu-ray disc has more than 5 times the storage

669 Miscellaneous-Explanation
unacademy.com | Download the Unacademy app
Give your feedback here: Link
capacity of a DVD. It can hold 25 GB on single- Statement 3 is not correct: HD DVD also has
layer discs and 50GB on double-layer discs (i.e., a disc thickness of 1.2mm (not simple DVD)
both-sided discs). Blu-ray disc was developed which is equal to Blu Ray Disc – 1.2mm (not
to record, rewrite and playback high-definition 2.4 mm).
video as well as store large amounts of data.

Miscellaneous-Explanation 670
unacademy.com | Download the Unacademy app
Give your feedback here: Link
7 Physics

1. Which one of the following is a reason why (a) 3 only


astronomical distances are measured in (b) 2 only
light-years? (2021)
(c) 1 and 3 only
(a) Distances among stellar bodies do not
change (d) 1, 2 and 3

(b)
Gravity of stellar bodies does not
change. 4. Ball bearings are used in bicycles, cars,
(c) Light always travels in a straight line. etc. because (2013)

(d) Speed of light is always same. (a) The actual area of contact between the
wheel and axle is increased
(b) The effective area of contact between
2. In a pressure cooker, the temperature at the wheel and axle is increased
which the food is cooked depends mainly
upon which of the following? (2021) (c) The effective area of contact between
the wheel and axle is reduced
1. Area of the hole in the lid
(d) None of the statements is correct
2. Temperature of the flame
3. Weight of the lid
5. Consider the following phenomena: (2013)
Select the correct answer using the code
given below. 1. Size of the sun at dusk

(a) 1 and 2 only 2. Colour of the sun at dawn

(b) 2 and 3 only 3. Moon being visible at dawn

(c) 1 and 3 only 4. Twinkle of stars in the sky

(d) 1, 2 and 3 5. Polestar being visible in the sky


Which of the above are optical illusions?

3. With reference to street-lighting, how (a) 1, 2 and 3


do sodium lamps differ from LED lamps? (b) 3, 4 and 5
(2021) (c) 1, 2 and 4
1. Sodium lamps produce light in 360 (d) 2, 3 and 5
degrees but it is not so in the case of
LED lamps.
6. Rainbow is produced when sunlight falls
2. As street-lights, sodium lamps have a
on drops of rain. (2013)
longer lifespan than LED lamps.
Which of the following physical phenomena
3. The spectrum of visible light from
are responsible for this?
sodium lamps is almost monochromatic
while LED lamps offer significant colour 1. Dispersion
advantages in street-lighting. 2. Refraction
Select the correct answer using the code 3. Internal reflection
given below.

671 Physics
unacademy.com | Download the Unacademy app
Give your feedback here: Link
Select the correct answer using the code 2. one would not be able to use a straw to
given below: consume a soft drink
(a) 1 and 2 only 3. the blotting paper would paper would
(b) 2 and 3 only fail to function

(c) 1 and 3 only 4. the big trees that we see around would
not have grown on the earth
(d) 1, 2 and 3
Which of the following statements given
above are correct?
7. The known forces of nature can be
(a) 1, 2 and 3 only
divided into four classes, viz., gravity,
electromagnetism, weak nuclear force (b) 1, 3 and 4 only
and strong nuclear force. With reference (c) 2 and 4 only
to them, which one of the following
(d) 1, 2, 3 and 4
statements is not correct? (2013)
(a) Gravity is the strongest of the four
10. An artificial satellite orbiting around the
(b) Electromagnetism act only on particles
Earth does not fall down. This is so because
with an electric charge
the attraction of Earth (2011)
(c) Weak nuclear force causes radioactivity
(a) does not exist at such a distance.
(d) Strong nuclear force holds protons and
(b) is neutralised by the attraction of the
neutrons inside the nucleus of an atom
moon.
(c) provides the necessary speed for its
8. What is the role of ultraviolet (UV) radiation steady motion.
in the water purification systems? (2012)
(d) provides the necessary acceleration for
1. It inactivates/kills the harmful its motion.
microorganisms in water.
2. It removes all the undesirable odours
11. The surface of a lake is frozen in severe
from the water.
winter, but the water at its bottom is still
3. It quickens the sedimentation of liquid. What is the reason? (2011)
solid particles, removes turbidity and
(a) Ice is a bad conductor of heat.
improves the clarity of water.
(b) Since the surface of the lake is at the
Which of the statements given above is/are
same temperature as the air, no heat is
correct?
lost.
(a) 1 only
(c) The density of water is maximum at 4°c.
(b) 2 and 3 only
(d) None of the statements (a), (b) and (c)
(c) 1 and 3 Only given is correct.
(d) 1, 2 and 3
12. What is the difference between a CFL and a
9. Consider the following statements: (2012) LED lamp? (2011)

If there were no phenomenon of capillarity 1. To produce light, a CFL uses mercury


vapour and phosphor while an LED
1. it would be difficult to use a kerosene
lamp uses semiconductor material.
lamp

Physics 672
unacademy.com | Download the Unacademy app
Give your feedback here: Link
2. The average lifespan of a CFL is much 13.
A layer in the Earth’s atmosphere
longer than that of a LED lamp. called the ionosphere facilitates radio
3. A CFL is less energy efficient as communication. Why? (2011)
compared to an LED lamp. 1. The presence of ozone causes the
Which of the statements given above is/are reflection of radio waves to earth.
correct? 2. Radio waves have a very long wavelength.
(a) 1 only Which of the statements given above is/are
(b) 2 and 3 only correct?

(c) 1 and 3 only (a) 1 only

(d) 1, 2 and 3 (b) 2 only


(c) Both 1 and 2
(d) Neither 1 nor 2

673 Physics
unacademy.com | Download the Unacademy app
Give your feedback here: Link
7 Physics-Explanation

1. Answer: (d) a pressure cooker by increasing the pressure.


Option (d) is correct: The speed of light is Hence cooking is faster.
constant throughout the universe and is
known to have high precision. In a vacuum, 3. Answer: (c)
light travels at 670,616,629 mph (1,079,252,849
Statement 1 is correct: Sodium vapour lights
km/h). The only thing absolute in this universe,
are omnidirectional. Omnidirectional lights
according to Einstein, is the speed of light; the
produce light in 360 degrees. This is a large
rest of everything is relative. Light travels at the
system inefficiency because at least half of the
speed of about 300,000 km per second, and it
light needs to be reflected and redirected to
is always the same throughout the universe. To
the desired area being illuminated. Whereas,
find the distance of a light-year, multiply this
whereas LEDs are extremely energy efficient
speed by the number of hours in a year (8,766).
relative to every other commercially available
One light-year equals 5,878,625,370,000 miles
lighting technology. There are several reasons
(9.5 trillion km). At first glance, this may seem
for this, including the fact they waste very
like an extreme distance, but the enormous
little energy in the form of infrared radiation
scale of the universe dwarfs this length. One
(much different than most conventional lights,
estimate puts the diameter of the known
including fluorescent lights), and they emit
universe at 28 billion light-years in diameter.
light directionally (over 180 degrees versus
360 degrees which means there are far fewer
2. Answer: (c) losses from the need to redirect or reflect
A pressure cooker is a sealed chamber that light). The Low-Pressure Sodium lamp was
traps the steam generated as its contents are the first sodium lamp to be developed. It is
heated. known for its signature monochromatic yellow
colour. It is mostly used in Europe since it did
Statements 1 and 3 are correct: In a pressure not appeal in other markets due to its poor CRI
cooker, cooking is done by producing a pressure or colour rendering.
higher than atmospheric pressure. Due to the
high pressure, the water in the cooker can be Statement 2 is not correct: The benefit of
heated to a temperature of more than 100°C High-Pressure Sodium is that they are the only
because the boiling point of water is higher at light source with similar efficiency to LEDs and
higher pressure. Hence, in a pressure cooker, maintain luminescence fairly well. They also do
the temperature at which the food is cooked have great life spans – around 24,000 hours
depends mainly upon the Area of the hole in – though this is significantly less than LEDs.
the lid and the Weight of the lid. The benefits of using LEDs are extensive. The
most significant benefits are extremely long
Statement 2 is not correct: In a pressure lifespans (50,000 to 100,000 hours or more),
cooker, the temperature at which the food very high energy efficiency, extremely high light
is cooked does not actually directly depends quality and little to no maintenance costs.
on the temperature of the flame. It depends
upon pressure. At high altitudes, atmospheric Statement 3 is correct: The spectrum of
pressure is lower, reducing the boiling point of visible light from sodium lamps is almost
water as compared to that at sea level. On the monochromatic as they are known for their
other hand, the boiling point is increased inside warm yellow glow. On the other hand, LED
lamps offer significant colour advantages in

Physics-Explanation 674
unacademy.com | Download the Unacademy app
Give your feedback here: Link
street lighting. LEDs offer a variety of colour at dawn during the entire half of the cycle
options providing better lighting solutions than of phases from Full Moon to just before
their High-Pressure Sodium counterparts. New Moon. It is not an optical illusion.
(Statement 3 is not correct)

4. Answer: (c)  y Stars twinkle because as light from those


stars passes through our atmosphere, it
Option (c) is correct: Rolling friction is smaller is bent and distorted by winds, as well
than sliding friction, (therefore) sliding is as varying temperatures and densities of
replaced in most machines by rolling with air. (Statement 4 is correct)
the use of ball bearings. The Rolling friction
is minimal as compared to static friction y It is the rotation of the earth which creates
and dynamic friction. So, ball bearings allow an illusion of the movement of stars across
any direction of movement, and the friction the sky. But the polestar appears to be
associated is the least and the effort required fixed at its position. This happens because
to move them is also very less.  The Loss of the pole star is in line with the axis of the
energy in moving the ball bearings is also the earth. Polestar is for real and not an optical
least. Therefore, due to reduced friction and illusion. (Statement 5 is not correct)
a less effective area of contact, less friction
force is applied. Friction is directly proportional 6. Answer: (d)
to the effective surface area.
Option (d) is correct: A rainbow is a natural
spectrum of sunlight in the form of bows
5. Answer: (c)  appearing in the sky when the sun shines on
Optical illusions (Visual illusions) are cool raindrops. Water droplets act as prisms. It is
images perceived in a manner that differs from the combined result of reflection, refraction
and dispersion of sunlight from water droplets
objective reality.  in the atmosphere. Dispersion of light occurs
y Dusk generally refers to the transition from when white light is separated into its different
day to night and is often used as another constituent colours because of refraction.  The
word for evening twilight. The sun does droplet causes a deviation in the path of light
not appear bigger or smaller in size, it as it enters and exits the drop. 
just appears oval or flattened at sunset or
sunrise because at sunset and sunrise the
sun is near to the horizon, and the rays of
light coming from the upper and lower edge
of the sun bend unequally while travelling
through the atmosphere due to refraction
of light. (Statement 1 is correct)
y The real colour of the Sun is actually white.
Dawn is the time that marks the beginning
of twilight before sunrise. Colour of the sun
at dawn is recognized by the appearance of
indirect sunlight being scattered in Earth’s
atmosphere when the centre of the Sun’s
disc has reached 18° below the observer’s
horizon. (Statement 2 is correct)
y The moon is visible somewhere in the sky

675 Physics-Explanation
unacademy.com | Download the Unacademy app
Give your feedback here: Link
7. Answer: (a) neutron by emitting a positron. Gamma decay
Gravitational force, weak nuclear force, normally happens after alpha or beta decay
electromagnetic force, and strong nuclear when the nucleus is in an excited state. Beta
force are the four fundamental forces, and they radioactivity is the best-known manifestation
govern everything that happens in the universe, of the weak force. It’s a slow process as it
from walking on the street to launching a occurs with difficulty. Unlike nuclear forces
rocket into space, to sticking a magnet on a and electromagnetism, the weak force can
refrigerator. transform a neutron into a proton or vice versa,
thus changing the composition of a nucleus.
Statement (a) is not correct: The gravitational
force was described systematically by Isaac Statement (d) is correct: The strong nuclear
Newton in the 17th century. Newton stated that force, also called the strong nuclear interaction,
the gravitational force acts between all objects is the strongest of the four fundamental forces
having mass (including objects ranging from of nature. It is six thousand trillion times
atoms and photons to planets and stars) and stronger than the force of gravity and that is
is directly proportional to the masses of the because it binds the fundamental particles
bodies and inversely proportional to the square of matter together to form larger particles. It
of the distance between the bodies. Since holds together the quarks that makeup protons
energy and mass are equivalent, all forms of and neutrons, and part of the strong force also
energy (including light) also cause gravitation keeps the protons and neutrons of an atom’s
and are under the influence of it. Objects with nucleus together.
more mass have more gravity. Gravity also gets
weaker with distance. So, the closer objects are 8. Answer: (a)
to each other, the stronger their gravitational
pull is. The gravitational force is weaker than Option (a) is correct: Ultraviolet water
the other four fundamental forces. purification is the most effective method for
disinfecting bacteria from the water. Ultraviolet
Statement (b) is correct: The electromagnetic (UV) rays penetrate harmful pathogens in water
force is the force responsible for all and destroy illness-causing microorganisms
electromagnetic processes. It acts between by attacking their genetic core (DNA). This
electrically charged particles. It is an infinite- is extremely efficient in eliminating their
ranged force, much stronger than the ability to reproduce. Disinfecting your water
gravitational force, and obeys the inverse with Ultraviolet light is exceptionally simple,
square law, but neither electricity or magnetism effective, and environmentally safe. UV systems
adds up in the way that gravitational force destroy 99.99% of harmful microorganisms
does. Since there are positive and negative without adding chemicals or changing your
charges (poles), these charges tend to cancel water’s taste or odour. UV water purification is
each other out. Electromagnetism includes usually used with other forms of filtration such
the electrostatic force acting between charged as reverse osmosis systems or carbon block
particles at rest, and the combined effect of filters.
electric and magnetic forces acting between
charged particles moving relative to each other.
Statement (c) is correct: There are three forms 9. Answer: (b)
of radioactivity, alpha, beta, and gamma. Alpha Option (b) is correct: Capillary action is the
decay only occurs in heavier elements which movement of water molecules through the gaps
give up energy when the nucleus is divided. Beta in the porous material due to cohesive force,
decay is where a neutron turns into a proton adhesive force, and surface tension. Capillarity
by emitting an electron or a proton turns into a can also be defined as the tendency of a liquid

Physics-Explanation 676
unacademy.com | Download the Unacademy app
Give your feedback here: Link
in a capillary tube or absorbent material to rise 12. Answer: (c) 
or fall as a result of surface tension. Compact Fluorescent Light (CFL) is a light bulb
Few Examples of capillary actions: which uses fluorescence for producing light
y The oil in the wick of a lamp rises due to whereas the Light Emitting Diode (LED) uses
the capillary action of threads in the wick. the semiconductor diode for producing visible
light.
y A blotting paper soaks ink by the capillary
action of the pores in the blotting paper. Statement 1 is correct: CFL uses mercury
vapour and phosphor to produce light while
y The root-hairs of plants draw water from
LED lamps use semiconductor material. Due
the soil through capillary action. Water is
to the use of mercury, power consumption in
retained in a piece of sponge on account of
CFL is more (as more power is required for
a capillary.
ionisation) than in LED.
y The action of a towel in soaking up moisture
Statement 2 is not correct: The average lifespan
from the body is due to the capillary action
of LED lamps (not CFLs) is much longer than
of cotton in the towel.
that of CFLs. LEDs are better than CFLs in
y No capillary action in using a straw to every aspect. LED lamps save power, the cost
consume the soft drink as we applied is very less, recyclable and lifespan are also
additional pressure from the mouth. higher than CFL lamps.
Statement 3 is correct: A CFL is less energy
10. Answer: (d)  efficient as compared to a LED lamp. A CFL can
save energy costs up to 70% whereas a LED
Option (d) is correct: An artificial satellite
lamp can save energy costs up to 80%.
moving around the Earth in a circular orbit
possesses an acceleration which is “constant” Additional Information:
in magnitude but “changing in direction”. An y The destruction of CFL bulbs is difficult
artificial satellite revolves around the earth because it contains toxic mercury vapour.
under centripetal acceleration. Acceleration is This mercury vapour has harmful effects on
the rate at which velocity changes with time, in the health of humans and the environment
terms of both speed and direction. An artificial whereas the destruction of LEDs is easy
satellite maintains its orbit by balancing two because it is free from toxic metals.
factors: its velocity, that is the speed it takes
to travel in a straight line and the gravitational
pull that Earth has on it. An artificial satellite 13. Answer: (d)
orbiting closer to the Earth requires more Option (d) is correct: Edward Appleton first
velocity to resist the stronger gravitational pull. discovered that radio waves were broadcast
around the world. Radio waves reflected back
from the ionosphere are the highest electrified
11. Answer: (c)
layers of the atmosphere that contain large
Option (c) is correct: The density of water is concentrations of charged particles (ions) and
greatest at 4˚C and in cold weather ice forms free electrons. Electromagnetic waves that are
at the top of the water. But after some time, sent from radio transmitters to outer space
it acts as an insulating barrier, preventing the are reflected back to every corner of the Earth
water underneath from getting much colder. after hitting this gas and plasma layer that is
Frozen lake surface acts like an insulator composed of charged particles. Thus, radio
preventing heat from the water from escaping, and radio telephone communication is made
keeping the heat of the water trapped.  possible for the benefit of human beings.

677 Physics-Explanation
unacademy.com | Download the Unacademy app
Give your feedback here: Link
8 Chemistry

1. Water can dissolve more substances than 1. Crystallization of sodium chloride


any other liquid because (2021) 2. Melting of ice
(a) it is dipolar in nature 3. Souring of milk
(b) it is a good conductor of heat Select the correct answer using the code
(c) it has high value of specific heat given below:
(d) it is an oxide of hydrogen (a) 1 and 2 only
(b) 3 only
2. Which of the following is/are the example/ (c) 1, 2 and 3
examples of chemical change? (2014) (d) None

Chemistry 678
unacademy.com | Download the Unacademy app
Give your feedback here: Link
8 Chemistry-Explanation

1. Answer: (a) in our lives. All new substances are formed as


Option (a) is correct: Water is called the a result of chemical changes. For example, if a
universal solvent because more substances metal is to be extracted from an ore, such as iron
dissolve in water than in any other chemical. from iron ore, we need to carry out a series of
This has to do with the polarity of each water chemical changes. Medicine is the end product
molecule. Water molecules have a polar of a chain of chemical reactions. Useful new
arrangement of the oxygen and hydrogen materials, such as plastics and detergents, are
atoms where one side (hydrogen) has a produced by chemical reactions.
positive electrical charge, and the other side y Crystallization is the physical process of
(oxygen) has a negative charge. This allows the solidification of a liquid substance into a
water molecule to become attracted to many highly structured solid whose atoms or
other different types of molecules. The oxygen molecules are placed in a well-defined
atom in a water molecule is slightly more 3D crystal lattice. Example: Crystallization
electronegative than a hydrogen atom. In other of sodium chloride. (Statement 1 is not
words, the oxygen atom is negatively charged, correct)
and the hydrogen atoms are positively charged, y A physical process that takes place in the
making the entire water molecule dipolar in phase transition of a substance from a
nature. solid to a liquid is called Melting, or Fusion.
Example: Melting of ice. (Statement 2 is not
2. Answer: (b) correct)

A change in which one or more new substances y Souring of milk is a chemical change due to
are formed is called a chemical change. A this process, acidification takes place and
chemical change is also called a chemical soured milk is produced. (Statement 3 is
reaction. Chemical changes are very important correct)

679 Chemistry-Explanation
unacademy.com | Download the Unacademy app
Give your feedback here: Link
9 Biology

1. Consider the following statements in (d) 2, 3 and 4 only


respect of probiotics : (2022)
1. Probiotics are made of both bacteria 3. Which of the following statements are
and yeast. correct regarding the general difference
2. The organisms in probiotics are found between plant and animal cells? (2020)
in foods we ingest but they do not 1. Plant cells have cellulose cell walls
naturally occur in our gut. whilst animal cells do not.
3. Probiotics help in the digestion of milk 2. Plant cells do not have plasma
sugars. membranes unlike animal cells which
Which of the statements given above is/are do.
correct? 3. Mature plant cell has one large vacuole
(a) 1 only whilst an animal cell has many small
(b) 2 only vacuoles.

(c) 1 and 3 Select the correct answer using the code


given below:
(d) 2 and 3
(a) 1 and 2 only
(b) 2 and 3 only
2. With reference to the current trends in the
cultivation of sugarcane in India, consider (c) 1 and 3 only
the following statements: (2020) (d) 1, 2 and 3
1. A substantial saving in seed material is
when ‘bud chip settlings’ are raised in 4. Which of the following statements is not
a nursery and transplanted in the main correct? (2019)
field.
(a) Hepatitis B virus is transmitted much
2. When direct planting of seeds is done, like HIV.
the germination percentage is better
with single-budded sets as compared (b) Hepatitis B, unlike Hepatitis C, does not
to sets with many buds. have a vaccine.

3. If bad weather conditions prevail when (c) Globally, the number of people infected
seeds are directly planted, single- with Hepatitis B and C viruses are
budded setts have better survival as several times more than those infected
compared to large sets. with HIV.

4. Sugarcane can be cultivated using (d) Some of those infected with Hepatitis
settlings prepared from tissue culture. B and C viruses do not show the
symptoms for many years.
Which of the statements given above is/are
correct?
(a) 1 and 2 only 5. Which of the following are the reasons for
the occurrence of multidrug resistance in
(b) 3 only microbial pathogens in India? (2019)
(c) 1 and 4 only

Biology 680
unacademy.com | Download the Unacademy app
Give your feedback here: Link
1. Genetic predisposition of some people. 8. Consider the following diseases (2014)
2. Taking incorrect doses of antibiotics to 1. Diphtheria
cure diseases.
2. Chickenpox
3. Using antibiotics in livestock farming.
3. Smallpox
4. Multiple chronic diseases in some
Which of the above diseases has/have been
people.
eradicated in India?
Select the correct answer using the code
(a) 1 and 2 only
given below:
(b) 3 only
(a) 1 and 2 only
(c) 1, 2 and 3
(b) 2 and 3 only
(d) None
(c) 1, 3 and 4 only
(d) 2, 3 and 4 only
9. Consider the following statements: (2014)
1. Maize can be used for the production of
6. Consider the following: (2018)
starch.
1. Birds
2. Oil extracted from maize can be a
2. Dust blowing feedstock for biodiesel.
3. Rain 3. Alcoholic beverages can be produced
4. Wind blowing by using maize.

Which of the above spread plant diseases? Which of the statements given above is/are
correct?
(a) 1 and 3 only
(a) 1 only
(b) 3 and 4 only
(b) 1 and 2 only
(c) 1, 2 and 4 only
(c) 2 and 3 only
(d) 1, 2, 3 and 4
(d) 1, 2 and 3

7. Which of the following statements is/are


correct? (2016) 10. Which of the following statements is/are
correct regarding vegetative propagation
Viruses can infect
of plants? (2014)
1. bacteria
1. Vegetative propagation produces a
2. fungi clonal population.
3. plants 2. Vegetative propagation helps in
Select the correct answer using the code eliminating the virus.
given below. 3. Vegetative propagation can be practised
(a) 1 and 2 only most of the year.

(b) 3 only Select the correct answer using the code


given below:
(c) 1 and 3 only
(a) 1 only
(d) 1, 2 and 3
(b) 2 and 3 only
(c) 1 and 3 only

681 Biology
unacademy.com | Download the Unacademy app
Give your feedback here: Link
(d) 1, 2 and 3 transplantation
(d)
leaves get damaged during
11. Consider the following pairs: (2014) transplantation

Vitamin Deficiency disease


1. Vitamin C Scurvy 15. Which of the following statements is/are
correct? (2013)
2. Vitamin D Rickets
1. Viruses lack enzymes necessary for the
3. Vitamin E Night blindness generation of energy.
Which of the pairs given above is/are 2. Viruses can be cultured in any synthetic
correctly matched? medium.
(a) 1 and 2 only 3. Viruses are transmitted from one
(b) 3 only organism to another by biological
(c) 1, 2 and 3 vectors only.

(d) None Select the correct answer using the code


given below:
(a) 1 only
12. Lichens, which are capable of initiating
ecological succession even on a bare rock, (b) 2 and 3 only
are actually a symbiotic association of (c) 1 and 3 only
(2014) (d) 1, 2 and 3
(a) algae and bacteria
(b) algae and fungi 16. Which of the following diseases can be
(c) bacteria and fungi transmitted from one person to another
(d) fungi and mosses through tattooing? (2013)
1. Chikungunya

13. Fruits stored in a cold chamber exhibit 2. Hepatitis B


longer storage life because (2013) 3. HIV-AIDS
(a) exposure to sunlight is prevented Select the correct answer using the code
(b) concentration dioxide in the is increased given below:
of carbon environment (a) 1 only
(c) rate of respiration is decreased (b) 2 and 3 only
(d) there is an increase in humidity (c) 1 and 3 only
(d) 1, 2 and 3
14. Many transplanted seedlings do not grow
because (2013) 17. Consider the following minerals (2013)
(a) the new soil does not contain favourable 1. Calcium
minerals
2. Iron
(b) most of the root hairs grip the new soil
too hard 3. Sodium

(c) most of the root hairs are lost during Which of the minerals given above is/
are required by the human body for the

Biology 682
unacademy.com | Download the Unacademy app
Give your feedback here: Link
contraction of muscles? 20. Aspartame is an artificial sweetener sold in
(a) 1 only the market. It consists of amino acids and
provides calories like other amino acids.
(b) 2 and 3 only
Yet, it is used as a low-calorie sweetening
(c) 1 and 3 only agent in food items. What is the basis of
(d) 1, 2 and 3 this use? (2011)
(a) Aspartame is as sweet as table sugar,
but unlike table sugar, it is not readily
18. Regular intake of fresh fruits and vegetables
oxidised in the human body due to lack
is recommended in the diet since they are
of requisite enzymes.
a good source of antioxidants. How do
antioxidants help a person maintain health (b)
When aspartame is used in food
and promote longevity? (2011) processing, the sweet taste remains,
but it becomes resistant to oxidation.
(a) They activate the enzyme necessary for
vitamin synthesis in the body and help (c) Aspartame is as sweet as sugar, but after
prevent vitamin deficiency. ingestion into the body, it is converted
into metabolites that yield no calories.
(b) They prevent excessive oxidation of
carbohydrates, fats and protein in (d) Aspartame is several times sweeter than
the body and help avoid unnecessary table sugar, hence food items made
wastage of energy. with small quantities of aspartame yield
fewer calories on oxidation.
(c) They neutralise the free radical produced
in the body during metabolism.
(d) They activate certain genes in the cells 21. A married couple adopted a male child.
of the body and help delay the ageing A few years later, twin boys were born to
process. them. The blood group of the couple is AB
positive and O negative. The blood group of
the three sons is A positive, B positive, and
19.
A company marketing food product O positive. The blood group of the adopted
advertises that its items do not contain son is? (2011)
trans-fats. What does this campaign signify
(a) O positive
to the customers? (2011)
(b) A positive
1. The food products are not made out of
hydrogenated oils. (c) B positive

2. The food products are not made out of (d) Cannot be determined on the basis of
animal fats/oils. the given data

3. The oils used are not likely to damage


the cardiovascular health of the 22. When the bark of a tree is removed in a
consumers. circular fashion all around near its base, it
Which of the statements given above is/are gradually dries up and dies because? (2011)
correct? (a) Water from soil cannot rise to aerial
(a) 1 only parts.

(b) 2 and 3 only (b) Roots are starved of energy.

(c) 1 and 3 only (c) Tree is infected by soil microbes.

(d) 1, 2 and 3 (d)


Roots do not receive oxygen for
respiration.

683 Biology
unacademy.com | Download the Unacademy app
Give your feedback here: Link
9 Biology-Explanation

1. Answer: (c) compared to single-budded setts.


Statement 1 is correct: Common probiotic Statement 4 is correct: Tissue culture or
bacteria can include Lactobacillus and vegetative propagation can be used to
Bifidobacterium. The most common yeast germinate and grow these settlings which can
found in probiotics is Saccharomyces boulardii. be transplanted in the field later. Sugarcane
Probiotics are defined as living bacteria that, can be cultivated using settlings prepared from
when administered in adequate amounts, tissue culture.
confer a health benefit on the host.
Statement 2 is incorrect: Probiotics 3. Answer: (c)
occur naturally in fermented foods and
cultured milk, but we can also find Statement 1 is correct: The plant cell wall is
manufactured probiotic supplements. mainly composed of cellulose. Cellulose is a
Statement 3 is correct: Recent studies suggest complex substance and provides structural
that probiotics in yogurt drinks and non-dairy strength to plants. When a living plant cell
probiotic supplements could help aid the loses water through osmosis there is shrinkage
digestion of lactose (milk sugar). or contraction of the contents of the cell away
from the cell wall. This phenomenon is known
as plasmolysis. Animal cells do not have
2. Answer: (c) cellulose.
Conventional method of sugarcane planting Statement 2 is not correct: Plant cells, in
requires 7 to 8 tonnes of seed cane per hectare, addition to the plasma membrane, have
and this is the main reason for the slow rate another rigid outer covering called the cell wall
of seed and varietal replacement. Sugarcane which lies outside the plasma membrane. All
being a long-duration crop and heavy biomass animal cells are multicellular. Animal cells are
producer requires about 1500 to 2500 mm also surrounded by a plasma membrane, and
water. it contains the nucleus and organelles that are
Statement 1 is correct: Bud chip technology membrane-bound.
reduces the mass and enables quick Statement 3 is correct: Most mature plant
multiplication of seeds. This method has cells have one large vacuole that occupies
proved to be more economical and convenient more than 30% of the cell’s volume, and that
than the traditional method of planting two can occupy as much as 80% of the volume for
to three bud setts. The returns are relatively certain cell types and conditions. An animal
better, with substantial savings on the seed cell has many small vacuoles.
material used for planting.
Elimination Technique: The plasma
Statement 2 is not correct: When direct membrane, also called the cell membrane,
planting of setts is done, the germination is the membrane found in all cells that
percentage is better with setts with many buds separates the interior of the cell from
as compared to single-budded setts. the outside environment. Statement 2 is
Statement 3 is not correct: If bad weather eliminated. Thus, Option (c) is correct.
conditions prevail when setts are directly
planted, large setts have better survival as

Biology-Explanation 684
unacademy.com | Download the Unacademy app
Give your feedback here: Link
Option (d) is correct: Signs and symptoms of
hepatitis B range from mild to severe. They
usually appear about one to four months after
infection. Some people, usually young children,
may not have any symptoms.

5. Answer: (b)
Multidrug resistance is also called cross-
resistance. It is a phenomenon seen in
some malignant cell lines. Malignant cells
are the cells that have developed natural
4. Answer: (b) resistance to a single cytotoxic compound
Hepatitis B is a serious liver infection caused and are also resistant to structurally unrelated
by the hepatitis B virus (HBV). For some people, chemotherapy agents. Whereas microbial
hepatitis B infection becomes chronic, meaning pathogens are microorganisms that are capable
it lasts more than six months. Having chronic of producing disease.
hepatitis B, increases risk of developing liver Statement 1 is not correct: Genetic
failure, liver cancer or cirrhosis (a condition predisposition is an increased chance that
that permanently scars the liver). one will develop a certain disease based
Option (a) is correct: HIV (human on genetic makeup. This can be identified
immunodeficiency virus) is a virus that by one’s family history and/or genetic
attacks the body’s immune system. It can be alterations. A predisposition contributes to the
transmitted via the exchange of a variety of development of disease but is not the reason
body fluids from infected people, such as blood, for the occurrence for multidrug resistance in
breast milk, semen, and vaginal secretions. The microbial pathogens.
Hepatitis B virus also spreads through contact Statement 2 is correct: The overuse and
with the blood, semen, or other body fluid of a inappropriate consumption and application of
person who has HBV, much like HIV. antibiotics is one of the major reasons for the
Option (b) is not correct: Different strains of occurrence of multidrug resistance.
hepatitis exist, including hepatitis A, B, C, and Statement 3 is correct: Antibiotics are a type
D. Hepatitis B and hepatitis C are both viral of antimicrobial designed to target bacterial
infections that attack the liver, and they have infections in people and animals. Antibiotics
similar symptoms. People can be effectively increase the efficiency of animal growth
protected against Hepatitis B through by inhibiting the growth of microbes in the
vaccination. In fact, India is the largest producer gastrointestinal tract which triggers immune
of recombinant hepatitis B vaccine but there is responses in the host. But indiscriminate
no effective Hepatitis C vaccine till date. use of antibiotics in livestock farming is also
Option (c) is correct: As per the World Health the reason for the occurrence of multidrug
Organization (WHO), Hepatitis B and C is a resistance in microbial pathogens.
leading infectious killer and affects 325 million Statement 4 is not correct: A chronic disease is
people worldwide, leading to about 1.4 million a disease that persists for a long time. People
deaths a year. It is the second major killer with multiple chronic diseases often receive
infectious disease after tuberculosis, and nine high doses of antimicrobials. This encourages
times more people are infected with hepatitis the spread of Antimicrobial Resistance (AMR)
than HIV. microbes. So, the treatment of chronic

685 Biology-Explanation
unacademy.com | Download the Unacademy app
Give your feedback here: Link
diseases may cause an AMR resistance based by the bacterium corynebacterium diphtheria,
on the medication and schedule followed by which affects the mucous membranes of the
the patient. But it is not always necessary. throat and nose. It is a communicable disease.
The usual symptoms of this disease are fever,
a loud cough, swollen neck, and a sore throat.
6. Answer: (d)
Diphtheria is a purely vaccine-preventable
Plant diseases are caused by a pathogenic disease and effective vaccines are also
organism such as a fungus, bacterium, available. But national immunization coverage
mycoplasma, virus, viroid, nematode, or of triple antigen DPT (Diphtheria, Pertussis,
parasitic flowering plant. Plant diseases can Tetanus) is only 50 to 60 percent. It should be
be classified as endemic diseases (prevalent in 100 percent otherwise there are chances of
and confined to a particular locality), epidemic disease eradication being less. Diphtheria has
diseases (occur incidentally and occasionally not been fully eradicated in India yet.
in a particular locality) and sporadic diseases
Statement 2 is not correct: Chickenpox is
(occur at very irregular intervals and locations).
a highly contagious disease caused by the
Option (d) is correct: Plant diseases spread varicella-zoster virus (VZV). It can cause an
through direct transmission (the pathogen is itchy, blister-like rash. The rash appears first
carried externally or internally on the seed or on the chest, back, and face, and then spreads
planting material like cuttings, sets, tubers, over the entire body. It primarily occurs in
bulbs, etc.) or through indirect transmission children under 10 years of age but can affect
(the pathogen spreads itself by way of its adults as well. The disease can be severe in
persistent growth or certain structures of the adults. In India, Chickenpox has not been fully
pathogen carried independently by natural eradicated yet.
agencies like wind, water (rain), animals,
Statement 3 is correct: It was a serious
insects, mites, nematodes, birds, etc.
infectious disease caused by the variola virus.
It had a contagious meaning, it spread from one
7. Answer: (d) person to another. People who had smallpox
had a fever and a distinctive, progressive skin
Option (d) is correct: Viruses are minuscule
rash. Most people with smallpox recover, but
infectious particles composed of a protein
about 3 out of every 10 people with the disease
coat and a nucleic acid core. They exist in a
died. India has eradicated smallpox.
huge variety of forms and infect practically all
living creatures: animals, plants, insects and
bacteria. Viruses can be considered as mobile 9. Answer: (d)
genetic particles, containing instructions for
Its percentage varies with the plant and also in
reproducing themselves using foreign cellular
different parts of the same plant. Corn (maize),
resources.
sorghum grain wheat, rice, potato, tapioca,
The number of viruses that exist in the arrowroot and sago are among the important
biosphere is enormous, varying in their virion sources of natural starches. The grains of
shapes, genomes, and lifestyles. They can barley, rye, oat, and millet are also employed
infect bacteria, fungi, plants, etc. in the production of starches.
y Corn Starch is made from maize kernels,
8. Answer: (b) which are high in starch, and used as
thickening agents in soups. (Statement 1 is
Statement 1 is not correct: Diphtheria is a
correct)
serious bacterial infection. It is primarily caused
y Maize has oil extracts that can be converted

Biology-Explanation 686
unacademy.com | Download the Unacademy app
Give your feedback here: Link
to ethanol or biodiesel. (Statement 2 is Pair 3 is not correctly matched: Vitamin E
correct) deficiency in humans is a very rare condition,
y An alcoholic drink is a drink that contains occurring as a consequence of abnormalities
ethanol, usually sourced from maize and in dietary fat absorption or metabolism rather
other grains. (Statement 3 is correct) than from a diet low in Vitamin E. Deficiency of
Vitamin A causes night blindness.

10. Answer: (c)


12. Answer: (b)
Vegetative propagation is a form of asexual
reproduction in plants. It is a process by which Option (b) is correct: Lichen refers to a
new organisms arise without the production of symbiotic association of two components,
seeds. a fungus, the ‘mycobiont’, and at least one
photosynthetic organism, the ‘photobiont’,
y It involves mitosis. This makes sure that the
consisting of a microalgae (usually a green
genetic information, in the DNA of vegetative
alga) or cyanobacterium, or both. Lichen cover
progeny child, is same as in the mother
8 percent of the land surface, including some
plant and can be practised throughout the
of the most extreme environments on Earth.
year. It helps in the development of clones.
(Statement 1 and statement 3 are correct) Lichens are found from the Arctic to Antarctic,
deserts to tropics, littoral zones to mountain
y But it does not help in elimination of viruses
peaks. Some prefer a single type of habitat
as a plant once systematically infected
such as limestone or sand dunes, while others
with a virus, usually remains infected for
are widespread and grow on different trees,
its lifetime. Any vegetative parts taken for
soil and rocks.
propagation remain infected. (Statement 2
is not correct) Additional Information:
y Lichens on the basis of the substrate on
which they grow can be categorised in to:
11. Answer: (a)
Corticolous (on bark of trees), Lignicolous (on
Deficiency diseases are diseases that are dead wood), Foliicolous (on living evergreen
caused by the lack of certain essential leaves), Terricolous (on soil), Saxicolous (on
nutrients, especially vitamins and minerals, in rocks and boulders), Muscicolous (found
one’s diet over a prolonged period of time. over mosses); Ramicolous (growing on the
Pair 1 is correctly matched: The classic Vitamin twigs); as well as on man-made substrates
C deficiency disease is Scurvy. Early signs of like plastic (called Plasticolous) and over
the disease are bleeding gums and bleeding Iron poles, fences, etc.
under the skin, causing tiny pinpoint bruises. y Lichens are categorised into four basic
The deficiency can progress to the point that growth forms viz. Crustose, Foliose,
it causes poor wound healing, anaemia, and Fruticose and Squamulose.
impaired bone growth.
y Lichens are very useful to people in diverse
Pair 2 is correctly matched: Vitamin D is cultures, especially as a source of drugs,
essential for strong bones because it helps the medicines, perfumery, food stuff, dyes, bio-
body use calcium from the diet. Traditionally, monitoring and other useful compounds.
vitamin D deficiency has been associated with
Rickets, a disease in which the bone tissue
does not properly mineralize, leading to soft 13. Answer: (c)
bones and skeletal deformities. Option (c) is correct: Fruits and vegetables
continue their metabolic activity after harvest.

687 Biology-Explanation
unacademy.com | Download the Unacademy app
Give your feedback here: Link
Metabolic activity is manifested by respiration. rotavirus which are common causes of viral
Reduction of temperature is an effective means gastroenteritis, are transmitted by the faecal
of reducing the rate of respiration. oral route passed by hand-to-mouth contact
y Most food products would benefit from or in food or water.
a reduction in the storage temperature Elimination Technique: Viral diseases like
provided that no freezing occurs. Reduced flu, Common Cold is spread by droplets
temperature reduces the rate of chemical generated through coughing or sneezing.
reactions that deteriorate the product and Statement 3 is eliminated. Thus, Option
also reduces microbiological activity. (a) is correct.
y However, the perishable foods not Note: Always pay extra attention to only-
specifically prepared for frozen storage, the statements
freezing temperature should be the lowest
acceptable limit for storage.
16. Answer: (b)

14. Answer: (c) Tattooing is a process of pricking and ingraining


an indelible pigment or by raising scars a
Option (c) is correct: During a transplant permanent mark or design made on the skin.
Seedling, most fine roots are destroyed either
by being cut to keep the root ball size down, by Statement 1 is not correct: Chikungunya diseases
drying out or simply by being jarred and jostled are spread by mosquitoes. The symptoms of
around in the move. Due to this, the plant is not chikungunya are similar to those of dengue
able to get the amount of moisture it requires and Zika. Tattooing also can lead to keloids —
to grow. This results in an imbalance between raised areas caused by an overgrowth of scar
the moisture demands of the top growth and tissue-Bloodborne diseases. If the equipment
the reduced ability of the roots to meet those used to create a tattoo is contaminated with
demands manifesting as transplant shock. infected blood. Various bloodborne diseases
include methicillin-resistant Staphylococcus
aureus (MRSA), hepatitis B and hepatitis C.
15. Answer: (a) Statement 2 is correct: Hepatitis B is an
Statement 1 is correct: For normal metabolic infectious disease caused by the hepatitis
processes viruses lack necessary enzymes B virus (HBV) that affects the liver. It is
they require other cell enzymes to replicate. transmitted when blood, semen or another
For a virus they must attach and penetrate a bodily fluid from an infected person enters the
host cell to replicate and synthesise proteins. body of another individual. Hepatitis B is one of
Once the virus genetic material is inserted the diseases that can be transmitted from one
into the host cell, to perform the necessary person to another through tattooing.
processes they will begin to instruct the host Statement 3 is correct: HIV virus that damages
cell’s organelles. our immune system. Untreated HIV affects and
Statement 2 is not correct: Viruses can be kills CD4 cells, which are a type of immune cell
grown on cultures of bacteria. Viruses cannot called T cell. Without its treatment, a person
be grown on synthetic media, for example, with HIV is likely to develop a serious condition
agar, a type of gelatine. called acquired immunodeficiency syndrome,
Statement 3 is not correct: Viruses can spread known as AIDS. HIV Virus is transmitted through
through disease-bearing organisms known bodily fluids that include blood, semen, vaginal
as vectors. Influenza viruses are spread and rectal fluids, breast milk. While tattooing,
by coughing and sneezing. Norovirus and if a single tattoo needle is used then there will

Biology-Explanation 688
unacademy.com | Download the Unacademy app
Give your feedback here: Link
be chances of transmission of HIV through 18. Answer: (c)
blood if the contacting person is infected with Antioxidants are man-made or natural
HIV. substances that may prevent some types
of cell damage. Diets high in vegetables and
17. Answer: (d) fruits, which are good sources of antioxidants,
have been found to be healthy. Examples of
Option (d) is correct: In Our body muscle
antioxidants include vitamins (C and E), and
contraction is triggered when a signal is sent by
minerals (copper, zinc, selenium, etc.)
the central nervous system through the motor
neuron. The minerals Calcium, Iron, Potassium, Option (c) is correct: Antioxidants are
Magnesium, Sodium, etc., are required by the molecules that fight free radicals in our body.
body for the contraction of muscles. Free radicals are compounds that can cause
harm if their levels become too high in our body.
Calcium is important for healthy bones and
They are linked to multiple illnesses, including
teeth; helps muscles relax and contract;
diabetes, heart disease, and cancer. Our body
important in nerve functioning, blood clotting,
has its own antioxidant defences to keep free
blood pressure regulation, and immune system
radicals in check. Antioxidants are also found in
health. Sources of Calcium are milk and milk
food, especially in fruits, vegetables, and other
products; canned fish with bones (salmon,
plant-based, whole foods, which may prevent
sardines); fortified tofu and fortified soy milk;
some of the damage caused by free radicals
greens (broccoli, mustard greens); legumes.
by neutralising them. Antioxidant preservatives
Sodium is needed for proper fluid balance, also play a crucial role in food production by
nerve transmission, and muscle contraction. increasing shelf life.
The sources of Sodium are Table salt, soy
Free Radicals:
sauce; large amounts in processed foods;
small amounts in milk, breads, vegetables, and Free radicals are constantly being formed in
unprocessed meats. In muscle contraction your body. Without antioxidants, free radicals
Sodium plays essential roles due to their would cause serious harm very quickly,
importance in nerve function. Sodium helps eventually resulting in death.
our nerve cells send electrical signals, called However, free radicals also serve important
action potentials that signal for your muscles functions that are essential for health. For
to contract. example, our immune cells use free radicals to
Iron also helps muscles contract. Each muscle fight infections. As a result, your body needs to
contraction uses up energy as well as oxygen. maintain a certain balance of free radicals and
Iron helps support continued muscle function antioxidants.
by supporting energy production in muscle When free radicals outnumber antioxidants,
cells, so that muscle fibres always have the it can lead to a state called oxidative stress.
energy they need to contract properly. Iron Prolonged oxidative stress can damage your
also helps muscles store oxygen to fuel muscle DNA and other important molecules in your
contractions and promotes healthy circulation body. Sometimes it even leads to cell death.
so that muscles can get additional oxygen from Several lifestyle, stress, and environmental
the bloodstream. Men need 8 milligrams of iron factors are known to promote excessive free
each day to prevent a deficiency, according to radical formation and oxidative stress, including
the Linus Pauling Institute, while women need air pollution, cigarette smoke, alcohol intake,
18 milligrams. toxins, etc.

689 Biology-Explanation
unacademy.com | Download the Unacademy app
Give your feedback here: Link
19. Answer: (c) 21. Answer: (a)
Statement 1 is correct: Trans fats come in both Option (a) is correct: One of the parent’s
natural and man-made forms. The process blood group is AB⁺ to form single chromosome
of adding hydrogen to oil to make it thicker, cells his cell will undergo division during
and to increase its shelf life is Hydrogenation. reproduction either A⁺ cell or B⁺ cell. Other
Vanaspati Ghee, a type of shortening made parent has blood group O⁻ which will remain
from hydrogenated vegetable oils used as a same during reproduction.
cheaper alternative to ghee and butter.
Statement 2 is not correct: A limited amount
of trans fats present naturally in some food
items such as dairy products and meat and
processed and baked food such as cakes,
biscuits, pastries, some packaged popcorn
and chips, pre-mixes such as pancake mixes
contain industrially-manufactured trans-fat.
Statement 3 is correct: World Health
Organisation’s (WHO) call for eliminating trans
fats globally by 2023. This aims to reduce the
incidence of heart ailments in the country,
The following configuration the offspring thus
with one in four deaths in India due to cardiac
produced may have:
diseases. In light of the COVID-19 pandemic
this becomes even more vital which saw most A⁺ and O⁻ results in A⁺
deaths in patients who had comorbidities, B⁺ and O⁻ results in B⁺
including heart ailments. Their offspring can have only either A⁺ or B⁺
blood groups. So, the adopted son’s blood
20. Answer: (d) group will be O positive.

Option (d) is correct: Aspartame is a low-


calorie artificial sweetener used to sweeten a 22. Answer: (b)
wide variety of low- and reduced-calorie foods Option (b) is correct: The innermost layer of
and beverages. Aspartame is metabolised the bark is Phloem, and its main function is
by the human body into two constituent’s to do the transport of soluble organic material
methanol and amino acids then hydrolysis made during photosynthesis which is called
products are managed by our body in the same translocation. If water or minerals produced in
way as L-Phenylalanine, aspartic acid, and the leaves are not sent to the parts such as
methanol from other consumed foods. These roots, they will be starved of energy and die.
components yield no calories and add nothing
new to the diet.

Biology-Explanation 690
unacademy.com | Download the Unacademy app
Give your feedback here: Link
12 CURRENT AFFAIRS
AND MISCELLANEOUS
1 Current Affairs: India

1. Consider the following statements in (b) Only two


relation to Janani Suraksha Yojna: (2023) (c) Only three
1. It is safe motherhood intervention of (d) All four
the State Health Departments.
2. Its objective is to reduce maternal and
neonatal mortality among poor pregnant 3. Consider the following statements: (2023)
women. Statement-I:
3. It aims to promote institutional delivery According to the United Nations’ 'World
among poor pregnant women. Water Development Report, 2022’, India
4. Its objective includes providing public extracts more than a quarter of the world's
health facilities to sick infants up to one groundwater withdrawal each year.
year of age. Statement-II:
How many of the statements given above India needs to extract more than a quarter
are correct? of the world’s groundwater each year to
(a) Only one satisfy the drinking water and sanitation
needs of almost 18% of world’s population
(b) Only two living in its territory.
(c) Only three Which one of the following is correct in
(d) All four respect of the above statements?
(a) Both Statement-I and Statement-II are
2. Consider the following statements in the correct and Statement-II is the correct
context of interventions being undertaken explanation for Statement-I
under Anaemia Mukt Bharat Strategy: (b)
Both Statement-I and Statement-II
(2023) are correct and Statement-II is not the
1. It provides prophylactic calcium correct explanation for Statement-I
supplementation for pre-school (c) Statement-I is correct but Statement-II
children, adolescents and pregnant is incorrect
women. (d) Statement-I is incorrect but Statement-
2. It runs a campaign for delayed cord II is correct
clamping at the time of child-birth.
3. It provides for periodic deworming to 4. Consider the following statements: (2023)
children and adolescents.
Statement-I:
4. It addresses non-nutritional causes of
anaemia in endemic pockets with special India accounts for 3.2% of global exports of
focus on malaria, hemoglobinopathies goods.
and fluorosis. Statement-II:
How many of the statements given above Many local companies and some foreign
are correct? companies operating in India have taken
(a) Only one advantage of India’s ‘Production-linked

692 Current Affairs: India


unacademy.com | Download the Unacademy app
Give your feedback here: Link
Incentive’ scheme. 6. Consider the following statements in
Which one of the following is correct in respect of the 44th Chess Olympiad, 2022:
respect of the above statements? (2023)

(a) Both Statement-I and Statement-II are 1. It was the first time that Chess Olympiad
correct and Statement-II is the correct was held in India.
explanation for Statement-I 2. The official mascot was named ‘Thambi’.
(b)
Both Statement-I and Statement-II 3. The trophy for the winning team in the
are correct and Statement-II is not the open section is the Vera Menchik Cup.
correct explanation for Statement-I
4. The trophy for the winning team in
(c) Statement-I is correct but Statement-II the women’s section is the Hamilton-
is incorrect Russell Cup.
(d) Statement-I is incorrect but Statement- How many of the statements given above
II is correct are correct?
(a) Only one
5. Consider the following pairs with regard to (b) Only two
sports awards: (2023)
(c) Only three
1. Major Dhyan For the most (d) All four
Chand Khel spectacular and
Ratna Award : o u t s t a n d i n g
performance by a 7. Consider the following statements : (2022)
sportsperson over 1. Gujarat has the largest solar park in
period of last four India.
years
2. Kerala has a fully solar powered
2. Arjuna Award For the lifetime International Airport.
: achievement by a
3. Goa has the largest floating solar
sportsperson
photovoltaic project in India.
3. Dronacharya To honor eminent
Which of the statements given above is/are
Award : coaches who have
correct ?
successfully trained
sportspersons or (a) 1 and 2
teams (b) 2 only
4. Rashtriya To recognize the (c) 1 and 3
Khel contribution made by
(d) 3 only
Protsahan sports persons even
Puraskar : after their retirement
How many of the above pairs are correctly 8. With reference to Ayushman Bharat Digital
matched? Mission, consider the following statements
: (2022)
(a) Only one
1. Private and public hospitals must adopt
(b) Only two
it.
(c) Only three
2. As it aims to achieve universal health
(d) All four coverage, every citizen of India should
be part of it ultimately.

Current Affairs: India 693


unacademy.com | Download the Unacademy app
Give your feedback here: Link
3. It has seamless portability across the to minimize the loss in case of cyber
country. extortion.
Which of the statements given above is/are 4. Cost of defence in the court of law if
correct? any third-party files a suit.
(a) 1 and 2 only Select the correct answer using the code
(b) 3 only given below:

(c) 1 and 3 only (a) 1, 2 and 4 only

(d) 1, 2 and 3 (b) 1, 3 and 4 only


(c) 2 and 3 only

9. In India, what is the role of the Coal (d) 1, 2, 3 and 4


Controller’s Organization (CCO)? (2022)
1. CCO is the major source of Coal 11. In India, why are some nuclear reactors
Statistics in Government of India. kept under “IAEA Safeguards” while others
2. It monitors progress of development of are not? (2020)
Captive Coal/Lignite blocks. (a) Some use uranium and others use
3. It hears any objection to the thorium
Government’s notification relating to (b) Some use imported uranium and others
acquisition of coal-bearing areas. use domestic supplies
4. It ensures that coal mining companies (c)
Some are operated by foreign
deliver the coal to end users in the enterprises and others are operated by
prescribed time. domestic enterprises
Select the correct answer using the code (d) Some are State-owned, and others are
given below : privately-owned
(a) 1, 2 and 3
(b) 3 and 4 only 12. Recently, there was a growing awareness
(c) 1 and 2 only in our country about the importance of
Himalayan Nettle (Girardinia Diversifolia)
(d) 1, 2 and 4 because it is found to be a sustainable
source of: (2019)
10.
In India, under cyber insurances for (a) Anti-malarial drug
individuals, which of the following benefits (b) Biodiesel
are generally covered, in addition to
payment for the loss of funds and other (c) Pulp for paper industry
benefits? (2020) (d) Textile fibre
1. Cost of restoration of the computer
system in case of malware disrupting 13. Atal Innovation Mission is set up under the:
access to one’s computer. (2019)
2. Cost of a new computer if some (a) Department of Science and Technology
miscreant wilfully damages it, if proved
so. (b) Ministry of Labour and Employment

3. Cost of hiring a specialized consultant (c) NITI Aayog


(d)
Ministry of Skill Development and

694 Current Affairs: India


unacademy.com | Download the Unacademy app
Give your feedback here: Link
Entrepreneurship citizens known as ‘General Data Protection
Regulation’ in April 2016 and started
implementation of it from 25th May, 2018?
14. Consider the following statements: (2019)
(2019)
As per the Industrial Employment (Standing
(a) Australia
Orders) Central (Amendment) Rules,
2018 (b) Canada

1. If rules for fixed-term employment are (c) The European Union


implemented, it becomes easier for the (d) The United States of America
firms/companies to lay off workers.
2. No notice of termination of employment 17. Consider the following statements: (2019)
shall be necessary in the case of a
temporary workman. The Reserve Bank of India’s recent directives
relating to ‘Storage of Payment System Data,
Which of the statements given above is/are popularly known as data diktat, command
correct? the payment system providers that
(a) 1 only 1. They shall ensure that entire data
(b) 2 only relating to payment systems operated
(c) Both 1 and 2 by them are stored in a system only in
India.
(d) Neither 1 nor 2
2. They shall ensure that the systems are
owned and operated by public sector
15. Consider the following statements about enterprises.
Particularly Vulnerable Tribal Groups
3. They shall submit the consolidated
(PVTGs) in India: (2019)
system audit report to the Comptroller
1. PVTGs reside in 18 States and one Union and Auditor General of India by the end
Territory. of the calendar year.
2. A stagnant or declining population is Which of the statements given above is/are
one of the criteria for determining PVTG correct?
status.
(a) 1 only
3. There are 95 PVTGs officially notified in
(b) 1 and 2 only
the country so far.
(c) 3 only
4. Irular and Konda Reddi tribes are
included in the list of PVTGs. (d) 1, 2 and 3

Which of the statements given above are


correct? 18. Which of the following statements is/are
(a) 1, 2 and 3 only correct regarding the Maternity Benefit
(Amendment) Act, 2017? (2019)
(b) 2, 3 and 4 only
1. Pregnant women are entitled for three
(c) 1, 2 and 4 only
months pre-delivery and three months
(d) 1, 3 and 4 only post-delivery paid leave.
2. Enterprises with creches must allow
16. Which of the following adopted a law the mother a minimum six creche visits
on data protection and privacy for its daily.

Current Affairs: India 695


unacademy.com | Download the Unacademy app
Give your feedback here: Link
3. Women with two children get reduced 21. Which one of the following best describes
entitlements. the term "Merchant Discount Rate"
Select the correct answer using the code sometimes seen in news? (2018)
given below: (a) The incentive given by a bank to a
(a) 1 and 2 only merchant for accepting payments
through debit cards pertaining to that
(b) 2 only
bank.
(c) 3 only
(b) The amount paid back by banks to their
(d) 1, 2 and 3 customers when they use debit cards
for financial transactions for purchasing
goods or services.
19. Consider the following statements: (2019)
(c) The charge to a merchant by a bank for
1. Petroleum and Natural Gas Regulatory
accepting payments from his customers
Board (PNGRB) is the first regulatory
through the bank's debit cards.
body set up by the Government of India.
(d) The incentive given by the Government
2. One of the tasks of PNGRB is to ensure
to merchants for promoting digital
competitive markets for gas.
payments by their customers through
3. Appeals against the decisions of PNGRB Point of Sale (PoS) machines and debit
go before the Appellate Tribunals for cards.
Electricity.
Which one of the statements given above
22. Consider the following statements: (2018)
is correct?
1. Aadhaar card can be used as a proof of
(a) 1 and 2 only
citizenship or domicile.
(b) 2 and 3 only
2. Once issued, Aadhaar number cannot
(c) 1 and 3 only be deactivated or omitted by the Issuing
(d) 1, 2 and 3 Authority.
Which of the statements given above is/are
correct?
20. What was the purpose of the Intercreditor
Agreement signed by Indian banks and (a) 1 only
financial institutions recently? (2019) (b) 2 only
(a) To lessen the Government of India’s (c) Both 1 and 2
perennial burden of fiscal deficit and
(d) Neither 1 nor 2
current account deficit.
(b) To support the infrastructure projects
of Central and State Governments. 23. The identity platform `Aadhaar' provides
open "Application Programming Interfaces
(c) To act as independent regulator in case
(APIs)". What does it imply? (2018)
of applications for loans of Rs 50 crore
or more. 1. It can be integrated into any electronic
device.
(d) To aim at faster resolution of stressed
assets of Rs 50 crore or more which are 2. Online authentication using iris is
under consortium lending. possible.
Which of the statements given above is/are

696 Current Affairs: India


unacademy.com | Download the Unacademy app
Give your feedback here: Link
correct? (b) 2 only
(a) 1 only (c) Both 1 and 2
(b) 2 only (d) Neither 1 nor 2
(c) Both 1 and 2
(d) Neither 1 nor 2 26.
With reference to the provisions made
under the National Food Security Act, 2013,
consider the following statements: (2018)
24. Consider the following statements: (2018)
1. The families coming under the category
1. As per the right to education (RTE) Act, to
of 'below poverty line (BPL)' only are
be eligible for appointment as a teacher
eligible to receive subsidised food
in a state, a person would be required to
grains.
possess the minimum qualification laid
down by the concerned State Council of 2. The eldest woman in a household, of
Teacher education. age 18 years or above, shall be the head
of the household for the purpose of
2. As per the RTE Act, for teaching primary
issuance of a ration card.
classes, a candidate is required to pass
a Teacher Eligibility Test conducted in 3. Pregnant women and lactating mothers
accordance with the National Council of are entitled to a 'take-home ration' of
Teacher Education guidelines. 1600 calories per day during pregnancy
and for six months thereafter.
3. In India, more than 90% of teacher
education institutions are directly under Which of the statements given above is/are
the State Governments. correct?

Which of the statements given above is/are (a) 1 and 2 only


correct? (b) 2 only
(a) 1 and 2 only (c) 1 and 3 only
(b) 2 only (d) 3 only
(c) 1 and 3 only
(d) 3 only 27. Which of the following is/are the aims of
“Digital India” Plan of the Government of
India? (2018)
25. Consider the following statements: (2018)
1. Formation of India’s own Internet
1. The Food Safety and Standards Act,
companies like China did.
2006 replaced the Prevention of Food
Adulteration Act, 1954. 2. Establish a policy framework to
encourage overseas multinational
2. The Food Safety and Standards Authority
corporations that collect Big Data to
of India (FSSAI) is under the charge of
build their large data centres within our
Director General of Health Services in
national geographical boundaries.
the Union Ministry of Health and Family
Welfare. 3. Connect many of our villages to the
Internet and bring Wi-Fi to many of our
Which of the statements given above is/are
schools, public places and major tourist
correct?
centres.
(a) 1 only
Select the correct answer using the code
given below:

Current Affairs: India 697


unacademy.com | Download the Unacademy app
Give your feedback here: Link
(a) 1 and 2 only (2017)
(b) 3 only 1. Under NSQF, a learner can acquire
(c) 2 and 3 only the certification for competency only
through formal learning.
(d) 1, 2 and 3
2. An outcome expected from the
implementation of NSQF is the mobility
28. With reference to Pradhan Mantri Kaushal between vocational and general
Vikas Yojana, consider the following education.
statements: (2018)
Select the correct answer using the code
1. It is the flagship scheme of the Ministry given below:
of Labour and Employment.
(a) 1 only
2. It, among other things, will also impart
(b) 2 only
training in soft skills, entrepreneurship,
financial and digital literacy. (c) Both 1 and 2

3. It aims to align the competencies of the (d) Neither 1 nor 2


unregulated workforce of the country
to the National Skill Qualification 31. Consider the following in respect of
Framework. ‘National Career Service’: (2017)
Which of the statements given above is/are 1. National Career Service is an initiative
correct? of the Department of Personnel and
(a) 1 and 3 only Training, Government of India.
(b) 2 only 2. National Career Service has been
(c) 2 and 3 only launched in a Mission Mode to improve
the employment opportunities to
(d) 1, 2, and 3
uneducated youth of the country.
Which of the above statements is/are
29. With reference to 'National Investment and correct?
Infrastructure Fund', which of the following
(a) 1 only
statements is/are correct? (2017)
(b) 2 only
1. It is an organ of NITI Aayog.
(c) Both 1 and 2
2. It has a corpus of Rs. 4,00,000 Crore at
present. (d) Neither 1 nor 2

Select the correct answer using the code


given below: 32. Which of the following statements best
(a) 1 only describes the term 'Scheme for Sustainable
Structuring of Stressed Assets (S4A)',
(b) 2 only
recently seen in the news? (2017)
(c) Both 1 and 2
(a)
It is a procedure for considering
(d) Neither 1 nor 2 ecological costs of developmental
schemes formulated by the Government.

30.
With reference to 'National Skills (b) It is a scheme of RBI for reworking the
Qualification Framework (NSQF)', which of financial structure of big corporate
the statements given below is/are correct? entities facing genuine difficulties.

698 Current Affairs: India


unacademy.com | Download the Unacademy app
Give your feedback here: Link
(c) It is a disinvestment plan of the (b) 3 only
Government regarding Central Public (c) 2 and 3 only
Sector Undertakings.
(d) 1, 2 and 3
(d)
It is an important provision in
'The Insolvency and Bankruptcy
Code' recently implemented by the 35. Consider the following statements: (2017)
Government. 1. National Payments Corporation of India
(NPCI) helps in promoting financial
33. Which of the following are the objectives inclusion in the country.
of the ‘National Nutrition Mission’? (2017) 2. NCPI has launched RuPay, a card
1. To create awareness relating to payment scheme.
malnutrition among pregnant women Which of the statements given above is/are
and lactating mothers. correct?
2. To reduce the incidence of anaemia (a) 1 only
among young children, adolescent girls, (b) 2 only
and women.
(c) Both 1 and 2
3. To promote the consumption of millets,
(d) Neither 1 nor 2
coarse cereals, and unpolished rice.
4. To promote the consumption of poultry
eggs. 36. In India, it is legally mandatory for which
of the following to report on cybersecurity
Select the correct answer using the code
incidents? (2017)
given below:
1. Service providers
(a) 1 and 2 only
2. Data centres
(b) 1, 2 and 3 only
3. Body corporate
(c) 1, 2 and 4 only
Select the correct answer using the code
(d) 3 and 4 only
given below:
(a) 1 only
34. Consider the following statements: (2017)
(b) 1 and 2 only
The nation-wide ‘Soil Health Card Scheme’
(c) 3 only
aims at
(d) 1, 2 and 3
1. expanding the cultivable area under
irrigation.
2. enabling the banks to assess the 37. What is the purpose of ‘Vidyanjali Yojana’?
quantum of loans to be granted to (2017)
farmers on the basis of soil quality. 1. To enable the famous foreign educational
3. checking the overuse of fertilisers in institutions to open their campuses in
farmlands. India.
Which of the above statements is/are 2. To increase the quality of education
correct? provided in government schools by
taking help from the private sector and
(a) 1 and 2 only
the community.

Current Affairs: India 699


unacademy.com | Download the Unacademy app
Give your feedback here: Link
3. To encourage voluntary monetary 40. The term 'Domestic Content Requirement'
contributions from private individuals is sometimes seen in the news with
and organisations so as to improve the reference to: (2017)
infrastructure facilities for primary and (a) Developing solar power production in
secondary schools. our country
Select the correct answer using the code (b) Granting licenses to foreign T.V. channels
given below: in our country
(a) 2 only (c) Exporting our food products to other
(b) 3 only countries
(c) 1 and 2 only (d) Permitting foreign educational
(d) 2 and 3 only institutions to set up their campuses in
our country
41. Who among the following can join the
38. What is the aim of the programme ‘Unnat
National Pension System (NPS)? (2017)
Bharat Abhiyan’? (2017)
(a) Resident Indian citizens only.
(a) Achieving 100% literacy by promoting
collaboration between voluntary (b) Persons of age from 21 to 55 only.
organisations, the government’s (c)
All-State Government employees
education system and local joining the services after the date of
communities. notification by the respective State
(b)
Connecting institutions of higher Governments.
education with local communities (d) All Central Governments Employees
to address development challenges including those of Armed Forces joining
through appropriate technologies. the services on or after 1st April 2004.
(c) Strengthening India’s scientific research
institutions in order to make India a 42. Consider the following statements: (2017)
scientific and technological power.
1. The Standard Mark of the Bureau of
(d) Developing human capital by allocating Indian Standards (BIS) is mandatory for
special funds for health care and automotive tyres and tubes.
education of rural and urban poor
and organising skill development 2. AGMARK is a quality Certification Mark
programmes and vocational training for issued by the Food and Agriculture
them. Organisation (FAO).
Which of the statements given above is/are
correct?
39.
Recently there was a proposal to
translocate some of the lions from their (a) 1 only
habitat in Gujarat to which one of the (b) 2 only
following sites? (2017) (c) Both 1 and 2
(a) Corbett National Park (d) Neither 1 nor 2
(b) Kuno Palpur Wildlife Sanctuary
(c) Mudumalai Wildlife Sanctuary 43. With reference to ‘Quality Council of India
(d) Sariska National Park (QCI)’, consider the following statements:
(2017)

700 Current Affairs: India


unacademy.com | Download the Unacademy app
Give your feedback here: Link
1. QCI was set up jointly by the Government 46. Recognition of 'Prior Learning Scheme' is
of India and the Indian Industry. sometimes mentioned in the news with
2. The Chairman of QCI is appointed by the reference to: (2017)
Prime Minister on the recommendations (a)
Certifying the skills acquired by
of the industry to the Government. construction workers through traditional
Which of the above statements is/are channels.
correct? (b) Enrolling the persons in Universities for
(a) 1 only distance learning programmes.

(b) 2 only (c) Reserving some skilled jobs to rural


and urban poor in some public sector
(c) Both 1 and 2
undertakings.
(d) Neither 1 nor 2
(d) Certifying the skills acquired by trainees
under the National Skill Development
44. What is the purpose of setting up Small Programme.
Finance Banks (SFBs) in India? (2017)
1. To supply credit to small business units 47. Which of the following statements is/are
2. To supply credit to small and marginal correct regarding Smart India Hackathon
farmers 2017? (2017)

3. To encourage young entrepreneurs to 1. It is a centrally sponsored scheme for


set up a business, particularly in rural developing every city of our country
areas into Smart Cities in a decade.

Select the correct answer using the code 2. It is an initiative to identify new digital
given below: technology innovations for solving the
many problems faced by our country.
(a) 1 and 2 only
3. It is a programme aimed at making all
(b) 2 and 3 only
the financial transactions in our country
(c) 1 and 3 only completely digital in a decade.
(d) 1, 2 and 3 Select the correct answer using the code
given below:

45. Which of the following is a most likely (a) 1 and 3 only


consequence of implementing the 'Unified (b) 2 only
Payments Interface (UPI)'? (2017)
(c) 3 only
(a) Mobile wallets will not be necessary for
(d) 2 and 3 only
online payments.
(b)
Digital currency will totally replace
physical currency in about two decades. 48. With reference to the ‘Prohibition of Benami
Property Transactions Act, 1988 (PBPT
(c) FDI inflows will drastically increase.
Act)’, consider the following statements:
(d) Direct transfer of subsidies to poor (2017)
people will become very effective.
1. A property transaction is not treated as
a Benami transaction if the owner of the
property is not aware of the transaction.
2. Properties held by Benami are liable for

Current Affairs: India 701


unacademy.com | Download the Unacademy app
Give your feedback here: Link
confiscation by the Government. (c) 1 and 3 only
3. The Act provides for three authorities (d) 1, 2 and 3
for investigations but does not provide
for any appellate mechanism.
51. On which of the following can you find the
Which of the statements given above is/are Bureau of Energy Efficiency Star Label?
correct? (2016)
(a) 1 only 1. Ceiling fans
(b) 2 only 2. Electric geysers
(c) 1 and 3 only 3. Tubular fluorescent lamps
(d) 2 and 3 only Select the correct answer using the code
given below.
49. Rashtriya Garima Abhiyaan is a national (a) 1 and 2 only
campaign to: (2016) (b) 3 only
(a) rehabilitate the homeless and destitute (c) 2 and 3 only
persons and provide them with suitable
sources of livelihood. (d) 1, 2 and 3

(b) release the sex workers from their


practice and provide them with 52.
Regarding DigiLocker, sometimes seen
alternative sources of livelihood. in the news, which of the following
(c)
eradicate the practice of manual statements is/are correct? (2016)
scavenging and rehabilitate the manual 1. It is a digital locker system offered by
scavengers. the Government under Digital India
(d) release the bonded labourers from their Programme.
bondage and rehabilitate them. 2. It allows you to access your e-documents
irrespective of your physical location.

50. Regarding ‘Atal Pension Yojana’, which of Select the correct answer using the code
the following statements is/are correct? given below.
(2016) (a) 1 only
1. It is a minimum guaranteed pension (b) 2 only
scheme mainly targeted at unorganized (c) Both 1 and 2
sector workers.
(d) Neither 1 nor 2
2. Only one member of a family can join
the scheme.
53. Recently, linking of which of the following
3. Same amount of pension is guaranteed
rivers was undertaken? (2016)
for the spouse for life after the
subscriber’s death. (a) Cauvery and Tungabhadra
Select the correct answer using the code (b) Godavari and Krishna
given below. (c) Mahanadi and Son
(a) 1 only (d) Narmada and Tapti
(b) 2 and 3 only

702 Current Affairs: India


unacademy.com | Download the Unacademy app
Give your feedback here: Link
54. Recently, for the first time in our country, 2. It spearheads the river conservation
which of the following States has declared efforts at the national level.
a particular butterfly as ‘State Butterfly’? 3. One of the Chief Ministers of the
(2016) States through which the Ganga flows
(a) Arunachal Pradesh becomes the Chairman of NGRBA on
(b) Himachal Pradesh rotation basis.

(c) Karnataka Select the correct answer using the code


given below.
(d) Maharashtra
(a) 1 and 2 only
(b) 2 and 3 only
55. Recently, which of the following States has
explored the possibility of constructing an (c) 1 and 3 only
artificial inland port to be connected to sea (d) 1, 2 and 3
by a long navigational channel? (2016)
(a) Andhra Pradesh 58.
Why does the Government of India
(b) Chhattisgarh promote the use of ‘Neem Coated Urea’ in
(c) Karnataka agriculture? (2016)

(d) Rajasthan (a)


Release of Neem oil in the soil
increases nitrogen fixation by the soil
microorganisms.
56. With reference to Pradhan Mantri Fasal
(b) Neem coating slows down the rate of
Bima Yojana, consider the following
dissolution of urea in the soil.
statements: (2016)
(c) Nitrous oxide, which is a greenhouse
1. Under this scheme, farmers will have to
gas, is not at all released into the
pay a uniform premium of two percent
atmosphere by crop fields.
for any crop they cultivate in any season
of the year. (d) It is a combination of a weedicide and a
fertiliser for particular crops.
2. This scheme covers post-harvest
losses arising out of the cyclones and
unseasonal rains. 59. With reference to ‘Stand Up India Scheme’
Which of the statements given above is/are which of the following statements is/are
correct? correct? (2016)

(a) 1 only 1. Its purpose is to promote


entrepreneurship among SC/ST and
(b) 2 only
women entrepreneurs.
(c) Both 1 and 2
2. It provides for refinancing through
(d) Neither 1 nor 2 SIDBI.
Select the correct answer using the code
57. Which of the following are the key features given below.
of ‘National Ganga River Basin Authority (a) 1 only
(NGRBA)’? (2016)
(b) 2 only
1. River basin is the unit of planning and
(c) Both 1 and 2
management.
(d) Neither 1 nor 2

Current Affairs: India 703


unacademy.com | Download the Unacademy app
Give your feedback here: Link
60.
‘Mission Indradhanush’ launched by the (b) 2 and 3 only
Government of India pertains to: (2016) (c) 1 and 2 only
(a) Immunisation of children and pregnant (d) 1, 2 and 3
women
(b) Construction of smart cities across the
country 63. Recently, India’s first ‘National Investment
and Manufacturing Zone’ was proposed to
(c) India’s own search for the Earth-like be set up in: (2016)
planets in outer space
(a) Andhra Pradesh
(d) New Educational Policy
(b) Gujarat
(c) Maharashtra
61. ‘Net metering’ is sometimes seen in the
news in the context of promoting the: (d) Uttar Pradesh
(2016)
(a) production and use of solar energy by 64. ‘SWAYAM’, an initiative of the Government
the households/consumers of India, aims at: (2016)
(b) use of piped natural gas in the kitchens (a) promoting the Self-Help Groups in rural
of households areas
(c) installation of CNG kits in motorcars (b) providing financial and technical
(d) installation of water meters in urban assistance to young start-up
households entrepreneurs
(c) promoting the education and health of
adolescent girls
62. With reference to ‘Initiative for Nutritional
Security through Intensive Millets (d)
providing affordable and quality
Promotion’, which of the following education to the citizens for free
statements is/are correct? (2016)
1. This initiative aims to demonstrate the 65. Pradhan Mantri MUDRA Yojana is aimed at:
improved production and post-harvest (2016)
technologies, and to demonstrate value (a) bringing the small entrepreneurs into
addition techniques, in an integrated formal financial system
manner, with cluster approach.
(b) providing loans to poor farmers for
2. Poor, small, marginal and tribal farmers cultivating particular crops
have a larger stake in this scheme.
(c) providing pensions to old and destitute
3. An important objective of the scheme persons
is to encourage farmers of commercial
(d) funding the voluntary organizations
crops to shift to millet cultivation
involved in the promotion of skill
by offering them free kits of critical
development and employment
inputs of nutrients and micro irrigation
generation.
equipment.
Select the correct answer using the code
given below. 66. With reference to pre-packaged items in
India, is it mandatory to the manufacturer
(a) 1 only
to put the following information on the

704 Current Affairs: India


unacademy.com | Download the Unacademy app
Give your feedback here: Link
main label, as per the Food Safety and 69. The Government of India has established
Standards (Packaging and Labelling) NITI Aayog to replace the: (2015)
Regulations, 2011? (2016) (a) Human Rights Commission
1. List of ingredients including additives (b) Finance Commission
2. Nutrition information (c) Law Commission
3. Recommendations, if any, made by the (d) Planning Commission
medical profession about the possibility
of any allergic reactions
4. Vegetarian/non-vegetarian 70. With reference to two non-conventional
energy sources called 'coalbed methane'
Select the correct answer using the code and 'shale gas', consider the following
given below. statements: (2014)
(a) 1, 2 and 3 1. Coalbed methane is the pure methane
(b) 2, 3 and 4 gas extracted from coal seams, while
(c) 1, 2 and 4 shale gas is a mixture of propane and
butane only that can be extracted from
(d) 1 and 4 only
fine-grained sedimentary rocks.
2. In India, abundant coalbed methane
67. With reference to the Indian Renewable sources exist, but so far, no shale gas
Energy Development Agency Limited sources have been found.
(IREDA), which of the following statements
Which of the statements given above is/are
is/are correct? (2015)
correct?
1. It is a Public Limited Government
(a) 1 only
Company.
(b) 2 only
2. It is a Non–Banking Financial Company.
(c) Both 1 and 2
Select the correct answer using the code
given below: (d) Neither 1 nor 2

(a) 1 only
(b) 2 only 71.
With reference to consumer’s rights/
privileges under the provision of law in
(c) Both 1 and 2
India, which of the following statements
(d) Neither 1 nor 2 is/are correct? (2012)
1. Consumer are empowered to take
68. The term ‘IndARC’ sometimes seen in the samples for food testing
news, is the name of? (2015) 2. When a consumer files a complaint in
(a) an indigenously developed radar system any consumer forum, no fee is required
inducted into Indian Defence to be paid.

(b) India’s satellite to provide services to 3. In case of death of a consumer, his/her


the countries of Indian Ocean Rim legal heir can file a complaint in the
consumer forum on his/her behalf.
(c) a scientific establishment set up by
India in Antarctic region Select the correct answer using the code
given below:
(d)
India’s underwater observatory to
scientifically study the Arctic region (a) 1 only

Current Affairs: India 705


unacademy.com | Download the Unacademy app
Give your feedback here: Link
(b) 2 and 3 only pregnancy and confinements
(c) 1 and 3 only Which of the statements given above is/are
(d) 1, 2 and 3 correct
(a) 1 and 2 only

72. With reference to the National Rural Health (b) 2 only


Mission, which of the following are the (c) 3 only
jobs of ‘ASHA”, trained community health (d) 1, 2 and 3
workers? (2012)
1. Accompanying women to the health
facility for antenatal care check-up 74.
In the Union Budget 2011-12, a full
exemption from the basic customs duty
2. Using pregnancy test kits for early was extended to the bio-based asphalt
detection pregnancy (bioasphalt). What is the importance of
3. Providing information on nutrition and this material? (2011)
immunization 1. Unlike traditional asphalt, bio-asphalt is
4. Conducting the delivery of baby. not based on fossil fuels.
Select the correct answer using the code 2. Bioasphalt can be made from non-
given below: renewable resources.
(a) 1, 2 and 3 only 3. Bioasphalt can be made from organic
(b) 2 and 3 only waste materials.

(c) 1 and 3 only 4. It is eco-friendly to use bioasphalt for


surfacing of the roads.
(d) 1, 2, 3 and 4
Which of the statements given above are
correct?
73. The endeavour of Janani Suraksha yojana (a) 1, 2 and 3 only
programme is: (2012)
(b) 1, 3 and 4 only
1. to promote institutional deliveries
(c) 2 and 4 only
2. to provide monetary assistance to the
mother to meet the cost of delivery (d) 1, 2, 3 and 4

3. to provide for wage loss due to

706 Current Affairs: India


unacademy.com | Download the Unacademy app
Give your feedback here: Link
1 Current Affairs: India-Explanation

1. Answer: (b) for delayed cord clamping during childbirth.


Statement 1 is not correct: Janani Suraksha Delayed cord clamping refers to the practice
Yojana was launched in April 2005 by modifying of allowing the umbilical cord to pulsate for a
the National Maternity Benefit Scheme certain duration before it is clamped and cut.
(NMBS). Janani Suraksha Yojana (JSY) is a safe This technique helps to improve iron levels and
motherhood intervention under the National reduce the risk of anemia in newborns.
Health Mission (NHM). It is not the intervention Statement 3 is correct: The Anaemia Mukt
of the State Health Departments. Bharat Strategy includes deworming initiatives
Statement 2 is correct:The primary objective as part of its interventions. Periodic deworming
of the Janani Suraksha Yojana is to reduce is an essential measure to address parasitic
maternal and neonatal mortality rates, worm infections, which can contribute to
particularly among economically disadvantaged anaemia among children and adolescents.
pregnant women. Statement 4 is correct: The Anaemia Mukt
Statement 3 is correct:One of the key goals Bharat Strategy recognizes that anaemia
of the Janani Suraksha Yojana is to encourage can have non-nutritional causes as well.
and facilitate institutional deliveries among In endemic pockets where specific factors
economically vulnerable pregnant women. like malaria, hemoglobinopathies (genetic
The program provides financial assistance disorders affecting haemoglobin), and fluorosis
and incentives to pregnant women to opt (excessive fluoride intake) contribute to
for institutional deliveries instead of home anaemia, the strategy focuses on addressing
deliveries, which can help ensure access to these factors to combat anaemia effectively.
skilled medical care during childbirth.
Statement 4 is incorrect:The Janani Suraksha 3. Answer: (c)
Yojana primarily focuses on maternal health Statement 1 is correct: India is the largest
and promoting institutional deliveries. It does groundwater user in the world, with an
not include providing public health facilities estimated usage of around 251 bcm per year,
specifically for sick infants up to one year more than a quarter of the global total. India
of age. The program’s main emphasis is on uses 89 per cent of the total groundwater
reducing maternal and neonatal mortality. abstracted per year for agriculture — also the
highest in the world.
2. Answer: (c) Statement 2 is not correct: India uses about
Statement 1 is incorrect:The Anaemia Mukt 89% of this groundwater for irrigation. Major
Bharat Strategy does not include the provision of share of total groundwater extracted by India
prophylactic calcium supplementation instead is utilized for irrigation purposes and not for
Prophylactic Iron Folic Acid supplementation drinking and sanitation purposes.
(not Prophylactic calcium) is given to children,
adolescents, women of reproductive age and 4. Answer: (d)
pregnant women as part of the comprehensive
Statement 1 is incorrect: As per Economic
approach to combat anaemia.
Survey 2023 India’s share in global export is
Statement 2 is correct: The Anaemia Mukt 2.2% (2021).
Bharat Strategy promotes and advocates

Current Affairs: India-Explanation 707


unacademy.com | Download the Unacademy app
Give your feedback here: Link
Statement 2 is correct: local companies and contribute to promotion of sports
some foreign companies operating in India events after their retirement.
have taken advantage of India’s ‘Production-
linked Incentive’ (PLI) scheme. The PLI scheme
was introduced by the Government of India to 6. Answer: (b)
promote domestic manufacturing and boost Statement 1 is correct: The 44th Chess
the competitiveness of various sectors. Under Olympiad is being held in Chennai from July 28
this scheme, eligible companies are provided to August 9, 2022. The prestigious competition,
with financial incentives based on their which has been organized since 1927, is being
incremental production or value addition. The hosted in India for the first time and in Asia
scheme aims to attract investments, encourage after 30 years.
the development of advanced technologies, Statement 2 is correct: Thambi is a brown
and enhance the country’s manufacturing horse who is the official mascot of the 44th
capabilities. Chess Olympiad.
Statement 3 is incorrect: The trophy for
5. Answer: (b) the winning team in the open section is the
1. ‘Major Dhyan Chand Khel Ratna Award’ Hamilton-Russell Cup, which was offered
is given for the spectacular and most by the English magnate Frederick Hamilton-
outstanding performance in the field Russell as a prize for the 1st Olympiad (London
of sports by a sportsperson over the 1927). The cup is kept by the winning team
period of the previous four years. until the next event, when it is consigned to
the next winner.
2. ‘Arjuna Award for outstanding
performance in Sports and Games’ Statement 4 is incorrect: The trophy for the
is given for good performance over a winning team at the Women’s Chess Olympiad
period of the previous four years and is named the Vera Menchik Cup in her honor.
for showing qualities of leadership,
sportsmanship and a sense of discipline. 7. Answer: (b)
3. ‘Dronacharya Award for outstanding Statement 1 is not correct: India’s Bhadla Solar
coaches in Sports and Games’ is given Park is the largest solar power park in the
to coaches for doing outstanding and world. Bhadla Solar Park is located in Bhadla,
meritorious work on a consistent basis a dry and sandy region in Rajasthan, and spans
and for enabling sportspersons to 14,000 acres. There are over 10 million solar
excel in International events. ‘Rashtriya panels at the park, which contribute to an
Khel Protsahan Puruskar’ is given to operational capacity of 2245MW.
corporate entities (both in private and
Statement 2 is correct: Kerala’s Cochin
public sector), Sports Control Boards,
International Airport Ltd (CIAL) is the first
NGOs, including sports bodies at the
airport in the world that would be running fully
State and National level, who have
on solar power.
played a visible role in the area of sports
promotion and development. Statement 3 is not correct: Bharat Heavy
Electricals Limited (BHEL) has successfully
4. ‘Dhyan Chand Award for Lifetime
commissioned India’s largest Floating Solar
achievement in Sports and Games’
PV plant. Located at NTPC Simhadri in Andhra
is given to honor sportspersons who
Pradesh, the 25 MW floating SPV project covers
have contributed to sports by their
an area of 100 acres. Besides producing clean
performance and who continue to
power, the project will also reduce water

708 Current Affairs: India-Explanation


unacademy.com | Download the Unacademy app
Give your feedback here: Link
evaporation by providing shade to the covered Statement 3 is correct: Under this act, the
area. It will also have a higher yield as compared Coal Controller has the jurisdiction to hear
to conventional ground-mounted projects due any objections to the Central Government’s
to the cooling effect. Notification relating to the acquisition of
coal-bearing land and to report to the Central
Government.
8. Answer: (b)
Statement 4 is incorrect: Coal Controller’s
Statement 1 is incorrect: Ayushman Bharat Organization (CCO) does not ensure that coal
Digital Mission Sandbox, created as a part of the mining companies deliver the coal to end users
Mission, will act as a framework for technology in the prescribed time.
and product testing that will help organizations,
including private players, intending to be a
part of the National Digital Health Ecosystem 10. Answer: (b)
become a Health Information Provider or Cybercrimes are rapidly increasing. Data/
Health Information User or efficiently link with security breaches continue to affect millions
building blocks of Ayushman Bharat Digital and the reports of breaches are also
Mission. dramatically rising. Unauthorized access is a
Statement 2 is incorrect: The Ayushman Bharat common example of a security breach. This is
Digital Mission (ABDM) aims to develop the where the role of cybercrime insurance comes
backbone necessary to support the integrated into play.
digital health infrastructure of the country. Option (b) is correct: Cyber insurance offers
Statement 3 is correct: It has seamless compensation of expenses, fees and legal costs
portability across the country. It will bridge arising due to cyber breaches. It also includes
the existing gap among different stakeholders the loss or theft of client/employee information
of the Healthcare ecosystem through digital due to the hacking of the organization’s data.
highways. It broadly covers, cost of restoration of the
computer system in case of malware disrupting
Elimination Technique: Participation in access to one’s computer, the cost of hiring a
ABDM is voluntary including for citizens. specialized consultant to minimize the loss in
Participation of a healthcare facility or case of cyber extortion and cost of defense in
an institution is also voluntary and shall the court of law if any third-party files a suit.
be taken by the respective management
(government or private management). For availing the benefit of cost of new computer
Statement 1 is eliminated. Thus, Option (b) under wilful damage by some miscreant
is correct. a hardware-specific insurance would be
required. This benefit is not covered under
cyber insurance for individuals.
9. Answer: (a)
Statement 1 is correct: In terms of coal and 11. Answer: (b)
lignite data, the Coal Controller has been
designated as the statistical authority. Charged Option (b) is correct: The nuclear reactors in
with conducting an annual coal and lignite India are placed under IAEA safeguards only
survey and issuing Provisional Coal Statistics if they are fuelled by uranium procured from
and the Indian Coal Directory. abroad. There are at present 22 operational
reactors, of which 14 are under the International
Statement 2 is correct: The Coal Controller’s Atomic Energy Agency (IAEA) safeguards as they
Organisation (CCO) is tasked with maintaining use imported fuel. Some of them use imported
control over captive mines.

Current Affairs: India-Explanation 709


unacademy.com | Download the Unacademy app
Give your feedback here: Link
uranium and others use domestic supplies. Additional Information:
India currently imports uranium from Russia, Girardinia diversifolia is largely used in
Kazakhstan, and Canada. By placing the traditional medicine for the treatment of several
reactors under the IAEA safeguards, India gives diseases such as gastric disorders, chest pain,
the international nuclear energy watchdog rheumatism, tuberculosis, headache, joint
access to them. This step was taken by the aches, diabetes, asthma, gastritis, headache,
country in 2014 to demonstrate that its nuclear joint pain, tuberculosis, gonorrhea and delivery
energy programme was for peaceful purposes. problems. Other traditional uses are related to
It was a necessary step under the Indo -US the treatment of bone fracture, internal injury
nuclear deal. and blood purification.
It is also famous for the use of the bast fibre of
12. Answer: (d) its bark, for making varieties of clothing, ropes,
mats, sacks and other domestic implements.
Option (d) is correct: Recently, Girardinia
diversifolia (Himalayan nettle), a fibre-yielding
plant, has become an important livelihood option 13. Answer: (c)
for people living in the remote mountainous Option (c) is correct: Atal Innovation Mission is
villages of the Hindu Kush Himalaya. There is a flagship scheme by the Government of India,
a community in Khar, a hamlet in Darchula set up under NITI Aayog (National Institute
district in far-western Nepal, which produces for Transforming India), erstwhile Planning
fabrics from Himalayan nettle. Commission, to create and promote a culture
of innovation and entrepreneurship across the
country.
Its objective is to develop new programmes
and policies for fostering innovation in different
sectors of the economy, provide platforms
and collaboration opportunities for different
stakeholders, and create an umbrella structure
to oversee the innovation and entrepreneurship
ecosystem of the country.

710 Current Affairs: India-Explanation


unacademy.com | Download the Unacademy app
Give your feedback here: Link
14. Answer: (c) probationers or badli workmen.
Recently, the Ministry of Labour and
Employment notified fixed term employment 15. Answer: (c)
for all sectors through an amendment to the
Industrial Employment (Standing Orders) Statement 1 is correct and 3 is not correct:
PVTGs constitute the most vulnerable section
Central Rules, 1946. Under the rules, a fixed
term employment workman is engaged on the among the tribal, isolated inhabited, remote
and difficult areas in small and scattered
basis of a written contract of employment for
hamlets/ habitats. They are economically
a fixed period.
backward and have low levels of literacy. There
Statement 1 is correct: The changes introduced are a total of 75 such groups of tribals found in
in Industrial Employment (Standing Orders) 18 states and in 1 union territory.
Act, 1946 have made it easy for the firms to
hire and fire employees by introducing fixed Statement 2 is correct: The criteria followed
for determination of PVTGs are as under:
term employment in all sectors.
Statement 2 is correct: As the Amendment y A pre-agriculture level of technology;
provides that subject to the provisions of y A stagnant or declining population;
the Industrial Disputes Act, 1947, no notice of y Extremely low literacy; and
termination of employment shall be necessary
y A subsistence level of economy.
in the case of temporary workmen whether
monthly rated, weekly rated or piece rated and Statement 4 is correct: PVTGs in Andhra

Current Affairs: India-Explanation 711


unacademy.com | Download the Unacademy app
Give your feedback here: Link
Pradesh (including Telangana): Bodo Gadaba,
Bondo Poroja, Chenchu, Dongria Khond, Gutob Elimination Technique: There are 75(Not
Gadaba, Khond Poroja, Kolam, Kondareddis, 95) PVTGs officially notified in the country
Konda Savaras, Kutia Khond, Parengi Poroja so far. Statement 3 is eliminated. Thus,
and Thoti. Option (c) is correct.

PVTGs in Tamilnadu: Kattu Nayakans, Kotas, Note: Give extra attention to numbers in
Kurumbas, Irulas, Paniyans and Todas. statements.

16. Answer: (c)


Option (c) is correct: The General Data
Protection Regulation (GDPR) was passed by
the European Union (a political and economic
union of 27 member states) in April 2016. GDPR
is a legal framework that sets guidelines for
the collection and processing of personal
information from individuals who live in the
Additional Information: European Union.

PVTGs have been identified in only one Union This Regulation protects the fundamental
Territory i.e. Andaman & Nicobar Islands. Great rights and freedoms of natural persons and
Andamanese, Jarawas, Onges, Sentinelese and in particular their right to the protection of
ShomPens are identified as PVTGs in Andaman personal data. Moreover, the free movement of
& Nicobar Islands. personal data within the Union shall be neither
restricted nor prohibited.

712 Current Affairs: India-Explanation


unacademy.com | Download the Unacademy app
Give your feedback here: Link
17. Answer: (a) Benefit Act of 1961 (MBA), has been welcomed
Statement 1 is correct: The RBI’s recent by employees and employers. The key changes
directives command payment system providers include:
that they shall ensure that entire data relating y Increased paid maternity leave from 12
to payment systems operated by them are weeks to 26 weeks for women employees,
stored in a system only in India. unless they have two or more surviving
Statement 2 is not correct: The systems are children. This maternity benefit should not
owned and operated by International payments be availed before eight weeks (earlier six
services providers including foreign banks that weeks) from the date of expected delivery.
have a small number of wholesale branches in (Statement 1 is not correct and 3 is correct)
India and switch providers. y Recognition of the rights of an adopting
Statement 3 is not correct: The consolidated mother and of a commissioning mother
system audit report should be submitted to the (using a surrogate to bear a child) for the
Reserve Bank of India not to the Comptroller first time, who may claim paid maternity
and Auditor General of India. The audit would leave for 12 weeks.
be conducted by CERT-IN empanelled auditors y A “work from home” option that may be of
to certify that these payment system providers benefit after the maternity leave expires.
have complied with the above-mentioned y Mandatory crèche (daycare) facilities for
directive. every establishment employing 50 or more
employees, including the right of mothers
18. Answer: (c) to visit the crèche four times per day.
(Statement 2 is not correct)
The 2017 landmark amendment to the Maternity

Current Affairs: India-Explanation 713


unacademy.com | Download the Unacademy app
Give your feedback here: Link
19. Answer: (b) It is aimed at the resolution of loan accounts
Statement 1 is not correct: The Petroleum and with a size of Rs. 50 crore and above under
Natural Gas Regulatory Board (PNGRB) was consortium lending (an arrangement where
established under the Petroleum and Natural two or more lenders come together to finance
Gas Regulatory Board Act, 2006. It is not the a single borrower).
first regulatory body of the Government of Additional Information:
India. Forward Market Commission was the For loans ranging from 50 crores to 500
first regulatory body setup in 1953 under the crores, it was recommended that a Bank
Forward Contracts (Regulation) Act, 1952. Led Resolution Approach (BLRA) should be
Later, it was merged with the Securities and followed, wherein the financial institutions
Exchange Board of India (SEBI). involved in consortium lending will enter into
Statement 2 is correct: The Act provides inter-creditor agreements to authorise the
for the establishment of Petroleum and Lead bank to implement a resolution plan for
Natural Gas Regulatory Board to protect the the bad loans.
interests of consumers and entities engaged
in specified activities relating to petroleum,
petroleum products and natural gas and to
promote competitive markets and for matters
connected therewith or incidental thereto.
Statement 3 is correct: Subject to the provisions
of the Act, the appeals against the decisions
of the Board go before the Appellate Tribunal
established under Section 110 of the Electricity
Act, 2003. The said Appellate Tribunal shall
exercise the jurisdiction, powers and authority
conferred on it by or under this Act. 21. Answer: (c)

Elimination Technique: Forward Market Option (c) is correct: Merchant Discount Rate
Commission was the first regulatory body (alternatively referred to as the Transaction
set up in 1953. Statement 1 is eliminated. Discount Rate) is the charge to a merchant
Thus, Option (b) is correct. by a bank for accepting payments from his
customers through the bank’s debit as well as
credit cards.
20. Answer: (d) Merchant Discount Rate (alternatively referred
Option (d) is correct: An Intercreditor Agreement to as the Transaction Discount Rate or TDR)
(ICA) is an agreement between one or more is the sum total of all the charges and taxes
creditors (lenders), who have shared interests that a digital payment entails. For instance,
in a particular borrower. The agreement is part the MDR includes bank charges, which a bank
of project ‘Sashakt’, which was approved under charges customers and merchants for allowing
the recommendations of the Sunil Mehta payments to be made digitally. Similarly, MDR
Committee to address the bad loan problem also includes the processing charges that a
of India. The objective is to use ICA for faster payments aggregator has to pay to online or
facilitation of resolution of stressed assets. mobile wallets or indeed to banks for their
service.

714 Current Affairs: India-Explanation


unacademy.com | Download the Unacademy app
Give your feedback here: Link
other’s capabilities.
Statement 1 is correct: Aadhaar provides open
Application Programming Interfaces, which can
be integrated easily into any electronic device.
Its open application programming interface
(API) layers known as “India Stack” set Aadhaar
apart from other biometric ID programmes.
India Stack APIs, which include the Unified
Payment Interface (UPI) and Aadhaar e-KYC,
The rate is expressed as a percentage of allow applications to be built on a top App
transaction amount and is shared in a pre- (for example, the Bharat Interface for Money
agreed ratio between the bank and the or BHIM app) and enable identity-driven
merchant. transactions.
Statement 2 is correct: The Aadhaar
authentication Application programming
22. Answer: (d) interface provides various ways for an Aadhaar
Statement 1 is not correct: Section 9 of the holders to authenticate themselves. At a higher
Aadhaar Act states that “The Aadhaar number level, this authentication takes place using
or the authentication, thereof shall not, by itself, Demographics data or Biometric data, and OTP.
confer any right of, or be proof of, citizenship Moreover, online authentication using iris is
or domicile in respect of an Aadhaar number also possible.
holder.” Conversely, it implies that citizenship
itself does not imply that an Aadhaar card has
been validly obtained. 24. Answer: (b)

Statement 2 is not correct: The Unique Statement 1 is not correct: In 2011, the
Identification Authority of India (UIDAI) on National Council for Teacher Education (NCTE)
its website mentioned following provisions laid down the teacher recruitment norms for
for deactivation of Aadhaar: If within 2 years all government schools in India irrespective
of attaining age 5, the child’s biometrics are of the State. According to it, the stipulated
not updated in the Aadhaar database, his/ qualification for teaching Class 6 to Class 8
her Aadhaar number will be deactivated is either a four-year Bachelor of Elementary
(no authentication permissible). It will be Education or a B.Ed. degree after senior
reactivated once biometrics are updated secondary; or a two-year B.Ed. after graduation;
in the database. If a resident has not been or a two-year Diploma in Education (D.Ed.) plus
biometrically authenticated in 5 years, his/her a graduate degree.
One Time Password (OTP) based authentication Statement 2 is correct: The Teacher Eligibility
services will be deactivated. They will be Test, known as TET, is the minimum qualification
reactivated once the resident is biometrically required in India for a person to be eligible
authenticated. for appointment as a teacher for Classes I to
VIII. The test is mandatory for teaching jobs in
Indian government schools.
23. Answer: (c)
Statement 3 is not correct: The NCTE is the apex
Application Programming Interface (API) body that manages the teacher’s education
is Computer Program code that lets two institutions in the country. The key role of the
applications interact with each other. It helps NCTE is to ‘achieve planned and coordinated
in letting software exchange data and use each development of the teacher education system

Current Affairs: India-Explanation 715


unacademy.com | Download the Unacademy app
Give your feedback here: Link
throughout the country. However, more than Statement 1 is not correct: The Act provided
90% of the teacher education institutions for coverage of up to 75% of the rural and
across the country are privately managed. 50% of the urban population to receive highly
subsidized food grains under Targeted Public
Distribution System. Under this Act, two-
25. Answer: (a) thirds of the total population of the country or
The Food Safety and Standards Act was approximately 81 crore persons are estimated
enacted in 2006 as an Act to consolidate the to receive benefits under the Targeted Public
laws relating to food and to establish the Distribution System. Hence, the families
Food Safety and Standards Authority of India coming under the category of ‘below poverty
for laying down science-based standards line (BPL)’ are not the only eligible people to
for articles of food and to regulate their receive subsidised food grains.
manufacture, storage, distribution, sale, and Statement 2 is correct: As a step towards
import, to ensure availability of safe and women’s empowerment, the eldest woman
wholesome food for human consumption and of the household of age 18 years or above is
for matters connected therewith or incidental mandated to be the head of the household for
thereto. the purpose of issuing ration cards under the
Statement 1 is correct: The Food Safety and Act.
Standard Act, 2006 replaced various Acts Statement 3 is not correct: One of the guiding
and Orders like the Edible Oils Packaging principles of the Act is its life-cycle approach
(Regulation) Order 1998, Fruit Products Order wherein special provisions have been made for
(FPO) 1955, Meat Food Products Order (MFPO), pregnant women and lactating mothers and
Prevention of Food Adulteration Act, 1954, etc. children in the age group of 6 months to 14
Statement 2 is not correct: The Ministry years. Pregnant women and lactating mothers
of Health & Family Welfare, Government of are entitled to a nutritious “take-home ration”
India is the administrative Ministry for the of 600 Calories and a maternity benefit of at
implementation of FSSAI. The Chairperson least Rs 6,000 for six months.
and Chief Executive Officer of Food Safety
and Standards Authority of India (FSSAI) is
27. Answer: (b)
appointed by the Government of India, who
holds the rank of Secretary to the Government Digital India campaign was launched in
of India. The authority acts in an independent July 2015, with the aim of ensuring that the
capacity. government’s services are made available
to citizens electronically by improving online
infrastructure and by increasing Internet
26. Answer: (b) connectivity or making the country digitally
The National Food Security Act, 2013 provides empowered in the field of technology.
a legal right to persons belonging to “eligible Option (b) is correct: The initiative includes
households” to receive food grains at a connecting many of our villages to the internet
subsidised price. It has the objective to and bringing Wi-Fi to many of our schools,
provide for food and nutritional security in public places, and major tourist centres. Digital
the human life cycle approach, by ensuring India consists of three main components:
access to adequate quantities of quality food The development of secure and stable digital
at affordable prices to people to live a life with infrastructure, Delivering government services
dignity. digitally, and Universal digital literacy.

716 Current Affairs: India-Explanation


unacademy.com | Download the Unacademy app
Give your feedback here: Link
entrepreneurship, financial and digital literacy.
Elimination Technique: Digital mission is
Statement 3 is correct: Recognition of Prior
simply, about the empowerment of people
Learning is a subcomponent under the scheme
through digital connectivity. Statement
wherein the existing competencies of the
2 and Statement 3 are about creating
individuals through the way of their knowledge/
competitive digital prowess in India
skill set and experience which have been
compared to other countries, so these
gained either through formal or informal means
statements can be eliminated. So, Option
is assessed and recognised. The scheme aims
(b) is correct.
to align the competencies of the unregulated
workforce of the country to the National Skill
28. Answer: (c) Qualification Framework.
Statement 1 is not correct: Pradhan Mantri
Kaushal Vikas Yojana is the flagship scheme 29. Answer: (d)
of the Ministry of Skill Development &
National Investment and Infrastructure Fund
Entrepreneurship (MSDE) implemented by
Limited (NIIFL) is a collaborative investment
National Skill Development Corporation. The
platform for international and Indian
objective of this Skill Certification Scheme is
investors anchored by the Government of
to enable a large number of Indian youths to
India. NIIFL invests across asset classes such
take up industry-relevant skill training that will
as infrastructure, private equity and other
help them in securing a better livelihood.
diversified sectors in India, with the objective
Statement 2 is correct: The training programme to generate attractive risk-adjusted returns for
under the scheme comes under the National its investors. NIIFL thinks long-term, believes
Skill Qualification Framework (NSQF) wherein in generating returns through efficiently
various training centres provide skills operating its investments through economic
and training in areas such as soft skills,

Current Affairs: India-Explanation 717


unacademy.com | Download the Unacademy app
Give your feedback here: Link
cycles, and is committed to sustainable y Master Fund: India’s largest core
investing principles. infrastructure fund, with a focus on creating
Statement 1 is not correct: It is not an organ scalable sectoral platforms in collaboration
of NITI Aayog. It has its own governing council with strong and reputed operating and
chaired by the Finance Minister of India. financial partners.

Statement 2 is not correct: NIIFL has a planned y Fund of Funds: A fund focused on anchoring
corpus of Rs. 40,000 Crore. and backing credible and reputed third-
party managers across diversified sectors
Additional Information: and strategies with a strong track record of
NIIFL currently manages three separate funds, delivering returns over a cycle.
which are registered as Alternative Investment y Strategic Opportunities Fund: A Private
Fund (AIF) with SEBI. Equity fund that aims to build scalable
These three funds are as follows: businesses across a range of opportunity
long but capital-short sectors.

718 Current Affairs: India-Explanation


unacademy.com | Download the Unacademy app
Give your feedback here: Link
30. Answer: (b) various other services. This portal facilitates
The National Skill Qualification Framework registration of Job Seekers, Employers, Skill
organizes and places qualification according to Providers, Career Counsellors, Local Service
the levels of knowledge, skills and aptitude. Providers (LSP’s), Career Centers, Placement
Organisations, Households (for availing
Statement 1 is not correct: Various levels of
the services of the LSP’s) and Government
qualifications are determined based on learning
Departments. This portal does not help
outcomes which a learner possesses regardless
uneducated or illiterate people.
of the way they were attained, whether formal,
non-formal or informal learning.
Statement 2 is correct: It is a nationally 32. Answer: (b)
integrated education and competency-based Option (b) is correct: The Scheme for
skill framework that will provide for multiple Sustainable Structuring of Stressed Assets
pathways, horizontal as well as vertical, both (S4A) is the latest measure taken by the RBI
within vocational education and vocational to tackle stressed assets. Under this scheme,
training and among vocational education, large ticket loans are restructured by separating
vocational training, general education and a sustainable loan from an unsustainable loan.
technical education, thus linking one level of The main aims of S4A are:
learning to another higher level. y Strengthen the lenders’ ability to deal with
stressed assets.
31. Answer: (d) y Put real assets back on track of entities
Statement 1 is not correct: National Career facing genuine difficulties by providing an
Service is a Five Year Mission Mode Project avenue for reworking financial structure.
launched by the Hon’ble Prime Minister on 20th Accounts having loans exceeding Rs 500 Crores
July 2015. The project is being implemented (big corporate entities) can be restructured
by the Directorate General of Employment, by allowing lenders to acquire equity in the
Ministry of Labour & Employment. National stressed projects.
Career Service (NCS) is a one-stop solution Additional Information:
that provides a wide array of employment and
career-related services to the citizens of India. Under this scheme, the Lender bank will
It works towards bridging the gap between determine the amount of debt that is
jobseekers and employers, candidates seeking sustainable, that is which can be serviced in
training and career guidance, and agencies future even with the current levels of inflows.
providing training and career counselling. The lender will bifurcate the outstanding
debt into sustainable debt and equity-based
Statement 2 is not correct: The NCS project instruments.
reaches out to the people of this country
through its three essential pillars i.e., a well-
designed ICT-based portal which is NCS portal, 33. Answer: (a)
Country wide set up of Model Career Centers Statement 1 is correct: National Nutrition
and Interlinkage with all the states through Mission, also known as POSHAN Abhiyaan.
employment exchanges. The digital centralised It is a flagship programme of the Ministry of
portal provides a wide range of career related Women and Child Development (MWCD), which
services including job search, job matching, rich aims to achieve improvement in the nutritional
career content, career counselling, information status of Children from 0-6 years, Adolescent
on Job Fairs, services of local service providers Girls, Pregnant Women and Lactating Mothers
like drivers, plumbers, etc. for households and in a time-bound manner during the next three

Current Affairs: India-Explanation 719


unacademy.com | Download the Unacademy app
Give your feedback here: Link
years beginning 2017-18. Statement 3 and statement 4 are not correct:
Statement 2 is correct: The mission targets Promotion of the consumption of millets,
to reduce stunting, under-nutrition, anaemia coarse cereals and unpolished rice and the
(among young children, women, and adolescent consumption of poultry eggs are not objectives
girls) and reduce low birth weight by 2%, 2%, of the National Nutrition Mission.
3% and 2% per annum, respectively, by 2022.

34. Answer: (b) printed report that a farmer will be handed


‘Soil Health Card’ (SHC) is a Government of India over for each of his holdings. It will contain the
scheme which is promoted by the Department status of his soil with respect to 12 parameters,
of Agriculture & Cooperation under the Ministry namely N, P, K (Macronutrients); S (Secondary-
of Agriculture and Farmers’ Welfare. It is a nutrient); Zn, Fe, Cu, Mn, Bo (Micronutrients);

720 Current Affairs: India-Explanation


unacademy.com | Download the Unacademy app
Give your feedback here: Link
and pH, EC, OC (Physical parameters). show recommendations on the dosage of
Option (b) is correct: The scheme will also different nutrients needed. Further, it will
indicate fertiliser recommendations and soil advise the farmer on the fertilisers and their
amendment required for the farm. The card quantities he should apply, thus checking the
will contain an advisory based on the soil overuse of fertilisers in farmlands, and also the
nutrient status of a farmer’s holding. It will soil amendments that he should undertake so
as to realise optimal yields.

35. Answer: (c)


National Payments Corporation of India (NPCI), an umbrella organization for operating retail payments
and settlement systems in India, is an initiative of the Reserve Bank of India (RBI) and Indian Banks’

Current Affairs: India-Explanation 721


unacademy.com | Download the Unacademy app
Give your feedback here: Link
Association (IBA) under the provisions of the and any other person. Further, CERT rule 12 also
Payment and Settlement Systems Act, 2007, places an obligation on them to mandatorily
for creating a robust Payment & Settlement report the different kinds of incidents as early
Infrastructure in India. as possible.
Option (c) is correct: The stated Mission of
NPCI is “to touch every Indian with one or 37. Answer: (a)
other Payment Services’’, and it promotes
Vidyanjali Yojana, launched by the Ministry of
its’ mission of Financial Inclusion through
Education (erstwhile, the Ministry of Human
initiatives like RuPay, Aadhar Payment Bridge
Resource and Development) which focuses
Systems, Aadhar Enabled Payment Systems,
on the improvement of literacy by offering
Unified Payments Interface, etc. RuPay is an
volunteer teachers in government schools.
indigenously designed payment system by
Many children in India nowadays are unable to
the NPCI– designed to meet the expectation
study because of various reasons, in particular
and needs of the Indian consumer, banks and
poverty. Even though they try to join government
merchant ecosystem. RuPay supports the
schools nearby their residence, the lack of
issuance of debit, credit and prepaid cards by
teachers in government schools obstructs
banks in India and thereby supports the growth
their way towards education. The Yojana has
of retail electronic payments in India.
become the bridge between underprivileged
children and the educated society.
36. Answer: (d) Option (a) is correct: The scheme primarily
Section 70-B of the Information Technology aims to increase the quality of education
Act, 2000 gives the Central Government the provided in government schools by taking help
power to appoint an agency of the government from the private sector and the community.
to be called the Indian Computer Emergency Some of the features of the scheme are:
Response Team (CERT), which will serve as the y No particular educational qualification for
national agency for performing the following teaching is needed to apply for this scheme.
functions in the area of cybersecurity: Anyone from any service can join the team
y Collection, analysis, and dissemination of and start teaching the children.
information on cyber incidents. y This scheme is entirely volunteering based.
y Forecast and alerts of cybersecurity People who are retired or homemakers can
incidents. move forward and accede to the scheme to
y Emergency measures for handling uplift the Indian education system.
cybersecurity incidents. y Ex-servicemen, retired school teachers,
y Coordination of cyber incidents response homemakers, ex-defence officers and other
activities. government officials can join the scheme
as volunteer teachers.
y Issue guidelines, advisories, vulnerability
notes and whitepapers relating to y All the government schools from a total
information security practices, procedures, of 20 great states will be involved in this
presentation, response, and reporting of program. One can join any of the schools
cyber incidents. near their district or place and start
teaching the students.
Option (d) is correct: For carrying functions
mentioned above, CERT may call for information
and give direction to the service providers, 38. Answer: (b)
intermediaries, data centres, body corporate Option (b) is correct: The Unnat Bharat

722 Current Affairs: India-Explanation


unacademy.com | Download the Unacademy app
Give your feedback here: Link
Abhiyan program was launched by the Ministry Madhya Pradesh would take all necessary
of Education (erstwhile, the Ministry Human scientific measures thereafter for maintaining
Resource Development) in November 2014. It the genetic stability of the lion population as
is inspired by the vision of transformational per International Union for Conservation of
change in rural development processes by Nature (IUCN) guidelines on translocation.
leveraging knowledge institutions to help build Additional Information:
the architecture of an Inclusive India.
Asiatic lions: Asiatic lions were once distributed
The goals of Unnat Bharat Abhiyan are: up to the state of West Bengal in the east
y To build an understanding of the and Rewa in Madhya Pradesh in central India.
development agenda within institutes At present, Gir National Park and Wildlife
of Higher Education and an institutional Sanctuary is the only abode of the Asiatic lion.
capacity and training relevant to national Asiatic lions are slightly smaller than African
needs, especially those of rural India. lions. They are listed in Schedule I of Wildlife
y To re-emphasize the need for fieldwork, (Protection) Act 1972, in Appendix I of CITES
stakeholder interactions and design for and as Endangered on the IUCN Red List.
societal objectives as the basis of higher
education. 40. Answer: (a)
y To stress on rigorous reporting and useful Option (a) is correct: The Domestic Content
outputs as central to developing new Requirement (DCR) is the mandate for developing
professions. solar projects (solar power production) in India
y To provide rural India and regional to utilize domestically manufactured solar
agencies with access to the professional modules and cells, instituted in the Jawaharlal
resources of the institutes of higher Nehru National Solar Mission (JNNSM). This is
education, especially those that have an effort of the Government of India to create a
acquired academic excellence in the field healthy and robust indigenous manufacturing
of science, engineering and technology, base and to elevate India’s status as a solar
and management. To improve development hub.
outcomes as a consequence of this DCR dispute: The World Trade Organization’s
research. To develop new professions and (WTO) Appellate Body has declared DCRs in
new processes to sustain and absorb the India’s Jawaharlal Nehru National Solar Mission
outcomes of research. (JNNSM) as illegal. In 2015, the WTO disputes
y To foster a new dialogue within the larger panel also ruled that India’s subsidies for
community on science, society, and the solar power contravene WTO trade rules, and
environment and to develop a sense of India must remove the subsidies or face trade
dignity and collective destiny. sanction.
The reason behind the dispute: DCR violates
39. Answer: (b) the National Treatment principle of WTO. This
principle prohibits discrimination between
Option (b) is correct: A Central expert
imported and domestically produced goods
committee recommended the translocation of
with respect to internal taxation or other
Asiatic lions from Gir National Park in Gujarat
government regulation.
to Kuno Sanctuary in Madhya Pradesh. The
Supreme Court in 2013 ruled in favour of Additional Information:
translocation of the Asiatic lion to Kuno in the Jawaharlal Nehru National Solar Mission:
interest of the genetic stability of the species. The objective of the Mission is to establish

Current Affairs: India-Explanation 723


unacademy.com | Download the Unacademy app
Give your feedback here: Link
India as a global leader in solar energy by deploying 20,000 MW of grid-connected solar
creating the policy conditions for its diffusion power by 2022, which was revised to 1,00,000
across the country as quickly as possible. MW by 2022 during June 2015.
It focuses on setting up an enabling environment
for solar technology penetration in the country 41. Answer: (c)
both at a centralized and decentralized level.
The National Pension System (NPS) was
The Mission will adopt a 3 - phase approach, launched in 2004, which aims to enhance
Phase 1 (up to 2012 - 13), Phase 2 (2013 - 17) social security in the country by encouraging
and Phase 3 (2017 - 22). people to save and invest in a pension account
The Mission has set the ambitious target of during the course of their employment.

724 Current Affairs: India-Explanation


unacademy.com | Download the Unacademy app
Give your feedback here: Link
Applicability of NPS: Statement 1 is correct: Bureau of India
y Individual: Unlike any other pension scheme, Standards (BIS) certification for automotive
all Indian residents, as well as non-resident tyres and tubes was made mandatory in 2010.
Indians, can subscribe or join NPS. (Option Statement 2 is not correct: AGMARK is a
(a) is not correct) certification mark employed on agricultural
y For Joining NPS, persons between the age products in India, assuring that they conform to
group of 18 and 60 years (not 21 to 55 years) a set of standards approved by the Directorate
are only eligible. (Option (b) is not correct) of Marketing and Inspection an attached Office
of the Department of Agriculture, Cooperation
y State Government Employees: It applies to
and Farmers Welfare under the Ministry of
all the employees of State Governments,
Agriculture & Farmers Welfare an agency of the
State Autonomous Bodies joining services
Government of India.
after the date of notification by the
respective State Governments. (Option (c)
is correct) 43. Answer: (c)
y Central Government Employees: NPS Statement 1 is correct: The Quality Council of
applies to all new employees of Central India, an autonomous body set up by the Ministry
Government service (except Armed Forces) of Commerce and Industry, Government of India
and Central Autonomous Bodies joining jointly with the Indian Industry represented
Government service on or after 1st January by the three premier industry associations,
2004. (Option (d) is not correct) i.e., Associated Chambers of Commerce and
y Unorganized Sector Workers: A citizen of Industry of India (ASSOCHAM), Confederation
India between the age of 18 and 60 years of Indian Industry (CII) and Federation of Indian
who belongs to the unorganized sector or Chambers of Commerce and Industry (FICCI),
is not in regular employment of the Central to establish and operate national accreditation
or a state government, or an autonomous structure and promote quality through National
body/ public sector undertaking of the Quality Campaign.
Central or state government, can open NPS Statement 2 is correct: Quality Council of India
-Swavalamban account. (QCI) is registered as a non-profit society with
its own Memorandum of Association and is
governed by a Council with equal representations
42. Answer: (a)
of government, industry, and consumers. The
Bureau of Indian Standards (BIS) is the National Council plays a pivotal role at the national
Standard Body of India established under the level in propagating, adoption, and adherence
BIS Act 2016 for the harmonious development to quality standards in all important spheres
of the activities of standardization, marking and of activities including education, healthcare,
quality certification of goods and for matters environment protection, governance, social
connected therewith or incidental thereto. BIS sectors, infrastructure sector and such
has been providing traceability and tangibility other areas of organised activities that have
benefits to the national economy in a number significant bearing in improving the quality
of ways – providing safe, reliable quality goods; of life and wellbeing of the citizens of India.
minimizing health hazards to consumers; The Chairman of QCI is appointed by the
promoting exports and imports substitute; Prime Minister on the recommendation of the
control over proliferation of varieties, etc. industry to the government.
through standardization, certification and
testing.

Current Affairs: India-Explanation 725


unacademy.com | Download the Unacademy app
Give your feedback here: Link
44. Answer: (a) Development Corporation. The objective of
Small Finance Banks (SFBs) are the financial this programme is to enable a large number of
institutions that provide financial services Indian youths to take up industry-relevant skill
to the unserved and unbanked region of the training that will help them in securing a better
country. SFBs are licensed and governed by livelihood. The scheme is underway across
the provisions of the Banking Regulation Act, construction sites in five states to certify
1949. skills acquired by workers through traditional
learning channels.
Option (a) is correct: The purpose of setting up
Small Finance Banks will be to boost financial Individuals with prior learning experience or
inclusion by (i) provision of savings vehicles skills will also be assessed and certified under
and (ii) supply of credit to small business Recognition of Prior Learning (RPL), but this is
units; small and marginal farmers; micro and not applicable to trainees.
small industries; and other unorganized sector
entities, through high technology-low cost 47. Answer: (b)
operations. Encouraging young entrepreneurs
to set up a business, particularly in rural areas, Option (b) is correct: Smart India Hackathon
is not a purpose for setting up SFBs. 2017 is a pan India 36-hour nonstop digital
programming competition. This initiative aims
to build Digital India and to engage the youth
45. Answer: (a) of the nation directly with nation-building. It is
Option (a) is correct: Unified Payments an initiative to identify new digital technology
Interface (UPI) is a system that powers innovations for solving real-time challenges
multiple bank accounts into a single mobile faced by the nation.
application (of any participating bank), merging It will also harness creativity among millions of
several banking features, seamless fund bright young minds. For the first time in India,
routing & merchant payments into one hood. the government departments were directly
It also caters to the “Peer to Peer” collection engaging with students and challenging them
request, which can be scheduled and paid to build digital solutions to improve their
as per requirement and convenience. It is an efficiency, and plug revenue leakages and
instant payment system developed by the corruption.
National Payments Corporation of India (NPCI).
UPI transfers money directly between the bank Elimination Technique: It is a programme
accounts of the two parties, and customers aimed at making all the financial
cannot link a wallet to UPI; only bank accounts transactions in our country completely
can be added. Hence, mobile wallets will not digital in a decade. Statement 3 is too
be necessary for online payments. extreme, so it can be eliminated. Thus,
Option (b) is correct.

46. Answer: (a)


48. Answer: (b)
Recognition of the ‘Prior Learning Scheme’ is
a crucial element under the Pradhan Mantri The ‘Prohibition of Benami Property Transactions
Kaushal Vikas Yojana (PMKVY) of the Ministry Act, 1988 (PBPT Act) defines Benami Property
of Skill Development and Entrepreneurship transactions as:
(MSDE). y A transaction or an arrangement where a
Option (a) is correct: It is a skill certification property is transferred to or is held by, a
programme implemented by National Skill person, and the consideration for such
property has been provided, or paid by,

726 Current Affairs: India-Explanation


unacademy.com | Download the Unacademy app
Give your feedback here: Link
another person; and the property is held of a PIL (Public Interest Litigation) filed in the
for the immediate or future benefit, direct Supreme Court for abolition and rehabilitation
or indirect, of the person who has provided of all manual scavengers across India. In 2013
the consideration. the government of India finally announced a
y A transaction or an arrangement in respect new law - The Prohibition of Employment as
of a property carried out or made in a Manual Scavengers and their Rehabilitation,
fictitious name. 2013.

y A transaction or an arrangement in
respect of a property where the owner 50. Answer: (c)
of the property is not aware of or denies Atal Pension Yojana is a flagship social security
knowledge of such ownership. scheme of the Government of India, launched
y A transaction or an arrangement in respect in 2015. with an objective of delivering old age
of a property where the person providing income security, particularly to the workers in
the consideration is not traceable or is the unorganised sector.
fictitious. y The scheme provides for a minimum
Option (b) is correct: This act was amended in guaranteed fixed pension for the
2016 and now referred to as Benami Transaction beneficiaries ranging from Rs 1,000 per
(Prohibition) Amendment Act, 2016. It gives month to Rs 5,000 per month after attaining
the government power to confiscate Benami the age of 60 years. (Statement 1 is correct)
properties through competent authority. No y The scheme is open to all citizens of India
compensation will be paid for such properties. who are in the age group of 18-40 years and
Also, it will not be possible to recover the not limited to only one member of a family,
confiscated property. The act now also provides those with a bank account and currently
for an appellate tribunal whose orders will be not covered under any other social security
open to appeal in High Courts. benefit scheme. (Statement 2 is not
Elimination Technique: The Act provides correct)
for three authorities for investigations y After the death of the subscriber, the same
but does not provide for any appellate pension amount will be paid to the spouse
mechanism. Statement 3 is not correct of the subscriber. On the demise of both
and can be eliminated. Statement 2 is subscriber and spouse, the accumulated
correct. So, Option (b) is correct. pension as accumulated till the age 60
years of the subscriber will be returned to
the nominee. (Statement 3 is correct)
49. Answer: (c)
Option (c) is correct: Rashtriya Garima Abhiyan Elimination Technique: The scheme is
is National Campaign for dignity and Eradication open to all citizens of India and not limited
of Manual Scavenging. The initiative is the to one family member. So, we are left with
brainchild of Ashif Shaikh, social activist and Option (a) and Option (c).
founder of nonprofit organisation Jan Sahas.
Rashtriya Garima Abhiyan used a positive 51. Answer: (d)
transformation approach to empower manual Energy ‘labelling’ is one of the most cost-
scavengers, village by village, to take collective effective tools for improving energy efficiency
action, burn their cane baskets, demolish dry and lowering the energy cost of appliances/
latrines and stand united in opposition to equipment for consumers. The program
caste-based exploitation. Ashif was also part has been developed in a collaborative and

Current Affairs: India-Explanation 727


unacademy.com | Download the Unacademy app
Give your feedback here: Link
consensus-driven approach with active the use of physical documents. Using this
participation from all the stakeholders. facility, one can store important documents
The Standards & Labelling (S& L) Programme such as PAN cards, mark sheets, insurance
is one of the major thrust areas of the Bureau policies, certificates, driving licence, etc. in an
of Energy Efficiency. A key objective of this electronic format.
scheme is to provide the consumer an informed
choice about energy saving and thereby the 53. Answer: (b)
cost-saving potential of the relevant marketed
product. The scheme targets display of energy Option (b) is correct: Pattiseema Lift Irrigation
performance labels on high-energy end-use Project is a river interlinking project which
equipment & appliances and lay down minimum connects Godavari River to Krishna River.
energy performance standards. The linking of the two rivers was originally
Option (d) is correct: Presently, S&L program envisaged as part of the river-linking
covers star rating for 26 appliances/equipment programme initiated by the Atal Bihari Vajpayee
such as ceiling fans, electric geysers, tubular government. And the Polavaram project,
fluorescent lamps, induction motors, variable now accorded a national project status,
capacity inverter, Microwave, deep freezer, etc. was supposed to achieve it by taking the 80
thousand million cubic feet (TMC) of surplus
water from the Godavari to Krishna through
52. Answer: (c) its right bank canal. The major benefit of this
Statement 1 is correct: Digilocker is an initiative interlinking would be to the farmers and people
under the Digital India Program by the Ministry of Rayalaseema.
of Electronics & Information Technology
(MeitY) Government of India where citizens
can get authentic documents/certificates in
digital format from original issuers of these
certificates. It aims at eliminating or minimising
the use of physical documents and will enhance
the effectiveness of service delivery, making
these hassle-free and friendly for the citizens.

Statement 2 is correct: DigiLocker is a platform


for the issuance and verification of documents
and certificates in a digital way, thus eliminating

728 Current Affairs: India-Explanation


unacademy.com | Download the Unacademy app
Give your feedback here: Link
Additional Information: 55. Answer: (d)
The main water divide in Peninsular India is Option (d) is correct: A proposal was received
formed by the Western Ghats, which runs from from the Government of Rajasthan to develop
north to south close to the western coast. an artificial inland Shipping Port at Jalore
Most of the major rivers of the Peninsula such which would be connected to the Arabian
as the Mahanadi, the Godavari, the Krishna and Sea by developing a channel along the Kutch
the Kaveri flow eastwards and drain into the Creek. The shipping port will promote the
Bay of Bengal. transportation of minerals through a waterway.
These rivers make deltas at their mouths. There The Inland Waterways Authority of India (IWAI)
are numerous small streams flowing west of under the Ministry of Shipping proposes to
the Western Ghats. The Narmada and the Tapi guide and support the Rajasthan government
are the only long rivers, which flow west and in building an inland shipping port at Jalore.
make estuaries. The drainage basins of the The port and the terminal that are to be created
peninsular rivers are comparatively small in will help develop inland navigation facilities in
size. western Rajasthan and will also bring about
socioeconomic development of the region.
54. Answer: (d)
Option (d) is correct: Maharashtra was the first 56. Answer: (b)
state that declared Blue Mormon as its state Pradhan Mantri Fasal Bima Yojana (PMFBY) was
butterfly. launched in Kharif 2016 with an aim to support
Maharashtra is home to 225 varieties of production in agriculture by providing an
butterflies and accounts for 15 percent of the affordable crop insurance product to ensure
country’s estimated butterfly population. comprehensive risk cover for crops of farmers
The Blue Mormon is the second largest in against all non-preventable natural risks from
size among all butterflies, second only to the pre-sowing to the post-harvest stage. The
Southern Birdwing. It has a velvet like black Ministry of Agriculture and Farmer Welfare
wings with bright blue spots. The lower end of (MoA&FW) is the nodal Ministry implementing
the wings is black while the body has some red the scheme.
spots on one side. It is found only in Sri Lanka, y The premium rate for Kharif Crops is two
Western Ghats of Maharashtra, and the coastal percent, for Rabi Crops it is 1.5 percent and
belts of India. for annual commercial and horticulture
Additional Information: crops, it is five percent. (Statement 1 is not
correct)
State State butterfly y The scheme under its ambit covers Yield
Losses, prevention of sowing due to bad
Maharashtra Blue Mormon
weather, post-harvest losses and localized
Uttarakhand Common peacock problems like a landslide, hailstorms, etc.
(Statement 2 is correct)
Karnataka Southern birdwings
Earlier, under the Pradhan Mantri Fasal Bima
Kerala Malabar banded Yojana, it was mandatory for loanee farmers
peacock to obtain crop insurance under the scheme.
However, in 2020 obtaining crop insurance
Tamil Nadu Tamil Yeoman
under the scheme has been made optional.

Current Affairs: India-Explanation 729


unacademy.com | Download the Unacademy app
Give your feedback here: Link
Elimination Technique: Under this scheme, y Elimination Technique: The Prime
farmers will have to pay a uniform premium Minister is the ex-officio Chairperson
of two percent for any crop they cultivate of the Authority. Statement 3 is
in any season of the year. Statement eliminated. Thus, Option (a) is correct.
1 is extreme and it can be eliminated.
Statement 2 is correct. So, Option (b) is 58. Answer: (b)
correct.
Option (b) is correct: The use of Neem Coated
Urea has been found to improve the uptake
57. Answer: (a) of nitrogen, phosphorus and potassium
National Ganga River Basin Authority (NGRBA) significantly. Neem-coating makes it more
has started the Mission Clean Ganga with difficult for black marketers to divert urea
a changed and comprehensive approach to to industrial consumers. Neem-coating also
champion the challenges posed to Ganga benefits farmers by reducing nitrogen losses
through four different sectors, namely, from the soil by providing greater nutrients to
of wastewater management, solid waste the crop. As a result, farmers need less urea to
management, industrial pollution, and achieve the same effect. In Neem Coated Urea,
riverfront development. every granule of urea has a coating of neem
oil which slows down the rate of dissolution
Statement 1 is correct: The Central of urea in the soil and thus increases the
Government set up the ‘National Ganga availability of nitrogen for crops.
River Basin Authority’ (NGRBA) in 2009 as an
empowered planning, financing, monitoring, Benefits of the use of Neem-coated Urea:
and coordinating authority for the Ganga River, y Increase in the crop yield.
to ensure effective abatement of pollution and y Improvement in soil fertility.
conservation of the river Ganga by adopting a
holistic approach with the river basin as the y Optimum utilisation of limited water
unit of planning. resources.

Statement 2 is correct: National Ganga River y Decrease the cost of cultivation.


Basin Authority (NGRBA) will spearhead river y Better nutrition for the family.
conservation efforts at the national level. y Slow release of nitrate into the soil.
Implementation will be by the State Agencies
y Improves groundwater quality.
and Urban Local Bodies.
y Lesser quantity of urea is required, saving
Statement 3 is not correct: The Ministry of
precious foreign exchange.
Water Resources, River Development and
Ganga Rejuvenation (MoWR, RD & GR) is the
nodal Ministry for the NGRBA. The authority is 59. Answer: (c)
chaired by the Prime Minister and has as its
The ‘Stand Up India’ Scheme was launched
members the Union Ministers concerned, the
by the Government of India in 2015. It aims
Chief Ministers of the States through which
to promote entrepreneurship at the grass-
Ganga flows, viz., Uttarakhand, Uttar Pradesh,
root level for economic empowerment and job
Bihar, Jharkhand and West Bengal, among
creation.
others. This initiative is expected to rejuvenate
the collective efforts of the Centre and the y Statement 1 is correct: The scheme seeks
States for cleaning the river. to leverage the institutional credit structure
to reach out to the underserved sector of
people such as Scheduled Caste, Scheduled

730 Current Affairs: India-Explanation


unacademy.com | Download the Unacademy app
Give your feedback here: Link
Tribe and Women entrepreneurs to enable the Micro, Small and Medium Enterprise
them to participate in the economic growth (MSME) sector as well as for coordination
of the nation. of functions of institutions engaged in
y Statement 2 is correct: The scheme similar activities.
seeks to provide financial aid for Working
Capital needs through the Small Industries 60. Answer: (a)
Development Bank of India (SIDBI).
Option (a) is correct: The Government of India
Additional Information: launched “Mission Indradhanush” in December
y Small Industries Development Bank of 2014, to strengthen and re-energize the already
India (SIDBI), set up on 2nd April 1990 running immunisation programme and achieve
under an Act of Indian Parliament, acts full immunisation coverage for all children and
as the Principal Financial Institution for pregnant women at a rapid pace.
Promotion, Financing and Development of

Current Affairs: India-Explanation 731


unacademy.com | Download the Unacademy app
Give your feedback here: Link
The ultimate goal of Mission Indradhanush is post-harvest technologies, and to demonstrate
to ensure full immunisation with all available value-addition techniques, in an integrated
vaccines for children up to two years of age manner, with a cluster approach.
and pregnant women. The Government has Statement 2 is correct: Poor, small, marginal
identified 201 high-focus districts across and tribal farmers have a larger stake in this
28 states in the country that have the scheme.
highest number of partially immunised and
unimmunized children. Statement 3 is not correct: The scheme
proposed to bring 0.5 million hectares (ha)
under millet cultivation. A key feature of
61. Answer: (a) INSIMP is giving input kits, comprising urea
Option (a) is correct: Net metering is a and pesticides depending on the type of crop;
mechanism which allows domestic or and seed kits, comprising hybrid seeds to
commercial users who generate their own the farmers. These kits are supplied by nodal
electricity using solar panels or photovoltaic agencies in a state, and are, in turn, procured
systems to export their surplus energy back to from various manufacturers. The other key
the grid. aspect of the scheme is the post-harvest
handling of millets, involving the establishment
of units for processing and value-addition.
There is no such provision of encouraging
farmers of commercial crops to shift to millet
cultivation.

63. Answer: (a)


Option (a) is correct: National Investment and
Manufacturing Zone (NIMZs) are envisaged
In India, net metering was introduced as an as large areas of developed land with the
initiative to make renewable energy more requisite eco-system for promoting world-
economical and accessible with the regulations class manufacturing activity. They are based on
differing on a state-to-state basis. Few the principle of industrial growth in partnership
advantages of net metering are extra financial with States and focus on manufacturing growth
credit, eliminates the need for battery storage and employment generation.
and backup generator, inexpensive and low y India’s first NIMZ was proposed to be set
maintenance. up in Andhra Pradesh in Prakasam District.
The NIMZs are envisaged as integrated
industrial townships with state-of-the-art
62. Answer: (c)
infrastructure; land use based on zoning;
The Initiative for Nutritional Security through clean and energy-efficient technology;
Intensive Millet Promotion (INSIMP) was necessary social infrastructure; skill
launched by the Government of India in 2011- development facilities etc., to provide
12 to promote millets as “nutri-cereals”. The a productive environment for persons
scheme aims to catalyse increased production transitioning from the primary to the
of millets in the country. In doing so, it aims to secondary and tertiary sectors.
enhance India’s nutritional security.
Statement 1 is correct: The initiative aims to
64. Answer: (d)
demonstrate the improved production and
Option (d) is correct: SWAYAM (Study Webs

732 Current Affairs: India-Explanation


unacademy.com | Download the Unacademy app
Give your feedback here: Link
of Active Learning for Young Aspiring Minds) size of the loans granted.
is an integrated MOOCs (massive open online ○ Shishu Loan: Loans of up to Rs fifty
course) platform for distance education that thousand.
is aimed at providing affordable and quality
education to citizens for free. It offers all ○ Kishore Loan: Loans ranging from Rs
the courses from school (Class 9) to post- fifty thousand to five lakh.
graduation level. The platform has been ○ Tarun Loan: Loans ranging from Rs five
developed collaboratively by MHRD (Ministry lakh to Rs ten lakh.
of Human Resource Development) and AICTE
(All India Council for Technical Education) with
66. Answer: (c)
the help of Microsoft and is capable of hosting
2,000 courses. Option (c) is correct: As per the Food Safety
and Standards (Packaging and Labelling)
Major Features of SWAYAM include:
Regulations, 2011, it is mandatory to the
y Course type and duration: There are two manufacturer to put:
types of courses offered through SWAYAM
y The name of the food which shall include
– Credit and Non-Credit.
trade name or description of food contained
y Course components: Courses hosted on in the package.
SWAYAM are available for students in four
y List of ingredients, except for single-
components: video lectures, downloadable/
ingredient foods, a list of ingredients shall
printable reading material, self-assessment
be declared on the label.
tests through quizzes and tests, and an
online discussion forum for clearing doubts. y Nutritional information or nutritional facts
energy value in kcal.
y Course fee: Courses offered via SWAYAM
are free of cost for residents of India. y Declaration regarding Vegetarian or Non-
However, in order to obtain a certificate for vegetarian.
a particular course, students need to pay a Every package of “non-vegetarian” food shall
nominal fee. bear a declaration to this effect made by a
symbol and colour code to indicate that the
product is Non-Vegetarian Food. The symbol
65. Answer: (a)
shall consist of a brown colour filled circle
Option (a) is correct: MUDRA stands for Micro having a diameter not less than the minimum
Units Development and Refinance Agency. size specified.
Under the Pradhan Mantri MUDRA Yojana, non-
corporate, non-farm small/micro enterprises
can avail of loans of up to Rs ten lakh. It aimed 67. Answer: (c)
at bringing small entrepreneurs into a formal Indian Renewable Energy Development Agency
financial system. Limited (IREDA) is a Mini Ratna (Category –
y MUDRA is a refinance scheme where the I) Government of India Enterprise under the
authority formed under the MUDRA Yojana administrative control of Ministry of New and
does not directly lend to the borrowers; Renewable Energy (MNRE).
instead, the loans are provided by various Option (c) is correct: IREDA is a Public Limited
financial institutions like Commercial Government Company established as a Non-
Banks, NBFCs, etc. Banking Financial Institution in 1987 engaged in
y Loans provided under the MUDRA Yojana promoting, developing, and extending financial
are classified into three types based on the assistance for setting up projects relating to

Current Affairs: India-Explanation 733


unacademy.com | Download the Unacademy app
Give your feedback here: Link
new and renewable sources of energy and 69. Answer: (d)
energy efficiency/conservation with the motto: Option (d) is correct: On the 13th of August
“ENERGY FOR EVER”. 2014, the Government of India scrapped
the 65-year-old Planning Commission and
68. Answer: (d) announced that it would be replaced by a new
body. Accordingly, on January 1, 2015, the NITI
Option (d) is correct: IndARC is India’s first Aayog (National Institution for Transforming
underwater moored observatory in the Arctic India) was established as the successor to the
region. It was deployed in 2014 at Kongsfjorden Planning Commission.
fjord, Svalbard, Norway which is midway
between Norway and the North Pole. Its y However, it must be noted here that the NITI
research goal is to study the Arctic climate and Aayog, like that of the Planning Commission,
its influence on the monsoon. was also created by an executive resolution.
It is neither a Constitutional body nor a
Statutory body.

734 Current Affairs: India-Explanation


unacademy.com | Download the Unacademy app
Give your feedback here: Link
70. Answer: (d) Statement 2 is not correct: A written complaint
The conventional sources of energy are usually can be filed by the complainant, or any person
non-renewable sources of energy, which have authorised by him, along with the supporting
been used for a long time. These sources of documents, if any. There is no need to engage
energy are being used extensively in such a any advocate. A small fee proportionate to the
manner that their known reserves have been compensation claimed is to be paid.
exploited to a great extent. The sources of The forums where a complaint can be filed
energy which are being produced continuously are the district forum (if the claim is up to Rs.
in nature and are inexhaustible are called non- 20 Lakhs), the State Commission (if the claim
conventional energy (or) renewable sources of is between Rs. 20 lakhs and Rs. 1 crore) and
energy. Some Examples of nonconventional the National Commission (if the claim exceeds
energy (or) renewable sources of energy Rs. 1 crore). These forums are quasi-judicial
include- Solar Energy, Wind Energy, Bio Energy, bodies, consisting of judicial as well as non-
Hydro Energy, Ocean Energy and Energy from judicial members and headed by a person from
waste. a legal background.
y Shale gas is a mixture of Methane, Propane, Statement 3 is correct: In case of the death
Nitrogen, Carbon monoxide and many other of a consumer, his/her legal heir can file a
gasses. (Statement 1 is not correct) complaint in the consumer forum on his/her
y Abundant shale reserves occur in India. To behalf. Moreover, one or more consumers on
strengthen its hydrocarbon resource base, behalf of numerous consumers having common
India has identified six basins as areas for interest also file a complaint. Other than
shale gas exploration: Cambay (Gujarat), this, any consumers’ association (registered),
Assam-Arakan (Northeast), Gondwana and the central government or any state
(Central India), Krishna Godavari onshore government can also file a complaint in the
(East Coast), Cauvery onshore, and Indo- consumer forum.
Gangetic basins. (Statement 2 is not
correct) 72. Answer: (a)
Option (a) is correct: One of the key components
71. Answer: (c) of the National Rural Health Mission is to
People participate both as a consumer as provide every village in the country with a
well as a producer in the marketplace, while trained female community health activist ASHA
acting as a consumer they may be vulnerable or Accredited Social Health Activist.
to certain exploitative exposures such as: ASHA will be trained to work as an interface
unfair trade practices (like adulteration, wrong between the community and the public health
measurement, etc.) by the producer, evasion system.
of responsibility by the producer post sales of Key responsibilities of ASHA workers:
the product and passing off false information
y Identifying and registering new
about the product through media, etc. The
pregnancies, births and deaths.
Government of India enacted Consumer
Protection Act 1986, popularly known as y Mobilizing, counseling and supporting the
COPRA. community to demand and seek health
services.
Statement 1 is correct: The existing law
governing consumer rights/privileges y Identifying, managing or referring cases of
empowered consumers to take samples for illness.
food tasting. y Supporting health services delivery through

Current Affairs: India-Explanation 735


unacademy.com | Download the Unacademy app
Give your feedback here: Link
home visits, first-aid and immunization some women would access accredited private
sessions. institutions for antenatal care, they would
y Maintaining data and participating in require some financial support to get at least
community-level health planning. 3 ANCs including the TT injections. In such
cases, at least three-fourth (3/4) of the cash
The National Rural Health Mission (NRHM)
assistance under JSY should be paid to the
was launched on 12th April 2005, to provide
beneficiary in one go, importantly, at the time
accessible, affordable, and quality health of delivery.
care to the rural population, especially the
vulnerable groups. NRHM seeks to provide Statement 3 is not correct: The scheme does
equitable, affordable, and quality health care to not have any provisions on to provide for wage
the rural population, especially the vulnerable loss due to pregnancy and confinements.
groups
74. Answer: (b)
73. Answer: (a) Statement 1 is correct: Bioasphalt is a greener
Janani Suraksha Yojana (JSY) was launched in alternative to asphalt made from non-Petroleum
2005 and is being implemented in all states materials based on renewable resources
and UTs with special focus on low-performing (biomass containing lignin). Bioasphalt has a
states. function, among others, used as an alternative
to asphalt replacement, can reduce the use of
The scheme focuses on the poor pregnant natural resources that cannot be renewed, and
woman with special dispensation for states can increase the use of biomass materials. In
having low institutional delivery rates namely addition, bioasphalt raw materials are cheaper
the states of Uttar Pradesh, Uttaranchal, Bihar, and available in sufficient quantities.
Jharkhand, Madhya Pradesh, Chhattisgarh,
Assam, Rajasthan, Orissa and Jammu and Statement 2 is not correct: Bioasphant can
Kashmir. While these states have been named only be made from renewable sources. It is
as Low Performing States (LPS), the remaining made from organic waste and is considered as
states have been named as High performing eco-friendly.
States (HPS). Statement 3 is correct: These sources include
Statement 1 is correct: Janani Suraksha Yojana sugar, molasses and rice, corn and potato
(JSY) is a safe motherhood intervention under starches, natural tree and gum resins, natural
the National Rural Health Mission (NRHM) latex rubber and vegetable oils, lignin, cellulose,
being implemented with the objective of palm oil waste, coconut waste, peanut oil
reducing maternal and neo-natal mortality by waste, canola oil waste, dried sewage effluent
promoting institutional delivery among poor and so on. Bitumen can also be made from
pregnant women. waste vacuum tower bottoms produced in the
process of cleaning used motor oils, which are
Statement 2 is correct: JSY is a 100 % centrally normally burned or dumped into landfills.
sponsored scheme, and it integrates cash
assistance with delivery and post-delivery Statement 4 is correct: Non-petroleum based
care. For pregnant women going to a public bitumen binders can be coloured, which can
health institution for delivery, entire cash reduce the temperatures of road surfaces and
entitlement should be disbursed to her in one reduce the Urban heat islands.
go, at the health institution. Considering that

736 Current Affairs: India-Explanation


unacademy.com | Download the Unacademy app
Give your feedback here: Link
2 Current Affairs: World

1. Which of the following countries has been this they are trying to bring technological
suffering from decades of civil strife and progress and physical productivity under
food shortages and was in news in the their control.
recent past for its very severe famine? Which one of the following is correct in
(2023) respect of the above statements?
(a) Angola (a) Both Statement-I and Statement-II are
(b) Costa Rica correct and Statement-II is the correct
(c) Ecuador explanation for Statement-I

(d) Somalia (b)


Both Statement-I and Statement-II
are correct and Statement-II is not the
correct explanation for Statement-I
2. Consider the following statements: (2023)
(c) Statement-I is correct but Statement-II
Statement-I: is incorrect
Switzerland is one of the leading exporters (d) Statement-I is incorrect but Statement-
of gold in terms of value. II is correct
Statement-II:
Switzerland has the second largest gold 4. Consider the following statements: (2023)
reserves in the world.
The ‘Stability and Growth Pact’ of the
Which one of the following is correct in European Union is a treaty that
respect of the above statements?
1. limits the levels of the budgetary deficit
(a) Both Statement-I and Statement-II are of the countries of the European Union
correct and Statement-II is the correct
2. makes the countries of the European
explanation for Statement-I
Union to share their infrastructure
(b)
Both Statement-I and Statement-II facilities
are correct and Statement-II is not the
3. enables the countries of the European
correct explanation for Statement-I
Union to share their technologies
(c) Statement-I is correct but Statement-II
How many of the above statements are
is incorrect
correct?
(d) Statement-I is incorrect but Statement-
(a) Only one
II is correct
(b) Only two
(c) All three
3. Consider the following statements: (2023)
(d) None`
Statement-I:
Recently, the United States of America (USA)
and the European Union (EU) have launched 5. Consider the following statements: (2023)
the ‘Trade and Technology Council’. 1. Recently, all the countries of the United
Statement-II: Nations have adopted the first-ever
compact for international migration, the
The USA and the EU claim that through

Current Affairs: World 737


unacademy.com | Download the Unacademy app
Give your feedback here: Link
‘Global Compact for Safe, Orderly and Sudan civil war live in Bidibidi.
Regular Migration (GCM)’.
3. Some people who fled from civil war in
2. The objectives and commitments Somalia live in Dadaab refugee complex
stated in the GCM are binding on the in Kenya.
UN member countries.
Which of the statements given above is/are
3. The GCM addresses internal migration correct?
or internally displaced people also in its
(a) 1 and 2
objectives and commitments.
(b) 2 only
How many of the above statements are
correct? (c) 2 and 3

(a) Only one (d) 3 only

(b) Only two


(c) All three 8. "Which one of the following best describes
the term “greenwashing” ? (2022)
(d) None
(a) Conveying a false impression that a
company’s products are eco-friendly
6. Consider the following statements: (2023) and environmentally sound
Statement-I: (b)
Non-inclusion of ecological/
Israel has established diplomatic relations environmental costs in the Annual
with some Arab States. Financial Statements of a country

Statement-II: (c)
Ignoring the disastrous ecological
consequences while undertaking
The ‘Arab Peace Initiative’ mediated by
infrastructure development
Saudi Arabia was signed by Israel and Arab
League. (d)
Making mandatory provisions for
environmental costs in a government
Which one of the following is correct in
project/programme
respect of the above statements?
(a) Both Statement-I and Statement-II are
correct and Statement-II is the correct 9. Consider the following statements : (2022)
explanation for Statement-I 1. Vietnam has been one of the fastest
(b)
Both Statement-I and Statement-II growing economies in the world in the
are correct and Statement-II is not the recent years.
correct explanation for Statement-I 2. Vietnam is led by a multi-party political
(c) Statement-I is correct but Statement-II system.
is incorrect 3. Vietnam’s economic growth is linked to
(d) Statement-I is incorrect but Statement- its integration with global supply chains
II is correct and focus on exports.
4. For a long time Vietnam’s low labour
costs and stable exchange rates have
7. Consider the following statements : (2022)
attracted global manufacturers.
1. Bidibidi is a large refugee settlement in
5. Vietnam has the most productive
north-western Kenya.
e-service sector in the Indo-Pacific
2. Some people who fled from South region.

738 Current Affairs: World


unacademy.com | Download the Unacademy app
Give your feedback here: Link
Which of the statements given above are (2021)
correct ? 1. American golfer Tiger Woods was the
(a) 2 and 4 first winner of this award.
(b) 3 and 5 2. The award was received mostly by
(c) 1, 3 and 4 ‘Formula One’ players so far.

(d) 1 and 2 3. Roger Federer received this award the


maximum number of times compared
to others.
10.
Consider the following statements in Which of the above statements are correct?
respect of the ICC Word Test Championship:
(2021) (a) 1 and 2 only

1. The finalists were decided by the (b) 2 and 3 only


number of matches they won. (c) 1 and 3 only
2. New Zealand was ranked ahead of (d) 1, 2 and 3
England because it won more matches
than England.
13. Consider the following statements: (2020)
Which of the above statements is/are
correct? 1. The value of Indo-Sri Lanka trade has
consistently increased in the last
(a) 1 only decade.
(b) 2 only 2. “Textile and Textile articles” constitute
(c) Both 1 and 2 an important item of trade between
(d) Neither 1 nor 2 India and Bangladesh.
3. In the last five years, Nepal has been
the largest trading partner of India in
11. Consider the following statements in South Asia.
respect of the 32nd Summer Olympics:
(2021) Which of the statements given above is/are
correct?
1. The official motto for this Olympics is ‘A
New World’. (a) 1 and 2 only

2. Sport Climbing, Surfing, Skateboarding, (b) 2 only


Karate and Baseball are included in this (c) 3 only
Olympics. (d) 1, 2 and 3
Which of the above statements is/are
correct?
14.
International agreement/set-up Subject
(a) 1 only (2020)
(b) 2 only
1. Alma-Ata Healthcare of the
(c) Both 1 and 2 Declaration people
(d) Neither 1 nor 2 2. Hague Convention Biological and
chemical weapons
12.
Consider the following statements in 3. Talanoa Dialogue Global climate
respect of the Laureus World Sports Award change
which was instituted in the year 2000:

Current Affairs: World 739


unacademy.com | Download the Unacademy app
Give your feedback here: Link
4. Under2 Coalition Child rights (b) Morocco and Tunisia

Which of the pairs given above is/are (c) Venezuela and Colombia
correctly matched? (d) Yemen and South Sudan
(a) 1 and 2 only
(b) 4 only 19. The term “Two state Solution” is sometimes
(c) 1 and 3 only mentioned in the new in the context of the
affairs of: (2018)
(d) 2, 3 and 4 only
(a) China
(b) Israel
15.
The Global Competitiveness Report is
published by the: (2019) (c) Iraq

(a) International Monetary Fund (d) Yemen

(b) United Nations Conference on Trade


and Development 20.
What is "Terminal High Altitude Area
(c) World Economic Forum Defense (THAAD)", sometimes seen in the
news? (2018)
(d) World Bank
(a) An Israeli radar system
(b) India's indigenous anti-missile program
16. Which one of the following is not a sub-
index of the World Bank’s 'Ease of Doing (c) An American anti-missile system
Business Index'? (2019) (d) A defence collaboration between Japan
(a) Maintenance of law and order and South Korea

(b) Paying taxes


(c) Registering property 21. The Global Infrastructure Facility is a/an:
(2017)
(d) Dealing with construction permits
(a)
ASEAN initiative to upgrade
infrastructure in Asia and financed by
17. “Rule of Law Index” is released by which of credit from the Asian Development
the following? (2018) Bank.
(a) Amnesty International (b) World Bank collaboration that facilitates
(b) International Court of Justice the preparation and structuring
of complex infrastructure Public-
(c) The Office of UN Commissioner for
Private Partnerships (PPPs) to enable
Human Rights
mobilization of the private sector and
(d) World Justice Project institutional investor capital.
(c) Collaboration among the major banks
18. Recently, in which of the following countries of the world working with the OECD
having lakhs of people either suffered from and focused on expanding the set of
severe famine/acute malnutrition or died infrastructure projects that have the
due to starvation caused by war/ethnic potential to mobilize private vestment.
conflicts? (2018) (d) UNCTAD funded an initiative that seeks
(a) Angola and Zambia to finance and facilitate infrastructure
development in the world.

740 Current Affairs: World


unacademy.com | Download the Unacademy app
Give your feedback here: Link
22. With reference to the role of UN-Habitat correct?
in the United Nations Programme working (a) 1 only
towards a better urban future, which of the
statements is/are correct? (2017) (b) 2 only

1. UN-Habitat has been mandated by the (c) Both 1 and 2


United Nations General Assembly to (d) Neither 1 nor 2
promote socially and environmentally
sustainable towns and cities to provide
25. Which of the following gives 'Global Gender
adequate shelter for all.
Gap Index' ranking to the countries of the
2. Its partners are either governments or world? (2017)
local urban authorities only.
(a) World Economic Forum
3. UN-Habitat contributed to the overall
(b) UN Human Rights Council
objective of the United Nations system
to reduce poverty and to promote (c) UN Women
access to safe drinking water and basic (d) World Health Organization
sanitation.
Select the correct answer using the code
26. The term 'Digital Single Market Strategy'
given below:
seen in the news refers to: (2017)
(a) 1, 2 and 3
(a) ASEAN
(b) 1 and 3 only
(b) BRICS
(c) 2 and 3 only
(c) EU
(d) 1 only
(d) G20

23.
'Broad-based Trade and Investment
27.
The term ‘Regional Comprehensive
Agreement (BTIA)' is sometimes seen in
Economic Partnership’ often appears in
the news in the context of negotiations
the news in the context of the affairs of a
held between India and: (2017)
group of countries known as: (2016)
(a) European Union
(a) G20
(b) Gulf Cooperation Council
(b) ASEAN
(c) Organization for Economic Cooperation
(c) SCO
and Development
(d) SAARC
(d) Shanghai Cooperation Organization

28. Consider the following statements: (2016)


24. Consider the following statements: (2017)
1. The International Solar Alliance was
1. The Nuclear Security Summits are
launched at the United Nations Climate
periodically held under the aegis of the
Change Conference in 2015.
United Nations.
2. The Alliance includes all the member
2. The International Panel on Fissile
countries of the United Nations.
Materials is an organ of the International
Atomic Energy Agency. Which of the statements given above is/are
correct?
Which of the statements given above is/are

Current Affairs: World 741


unacademy.com | Download the Unacademy app
Give your feedback here: Link
(a) 1 only Which of the pairs given above is/are
(b) 2 only correctly matched?

(c) Both 1 and 2 (a) 1 and 2

(d) Neither 1 nor 2 (b) 2 only


(c) 2 and 3

29. ‘European Stability Mechanism’, sometimes (d) 3 only


seen in the news, is an: (2016)
(a) Agency created by the EU to deal with 32. With reference to ‘Organisation for the
the impact of millions of refugees Prohibition of Chemical Weapons (OPCW)’,
arriving from the Middle East. consider the following statements: (2016)
(b) Agency of the EU that provides financial 1. It is an organisation of the European
assistance to eurozone countries. Union in working relation with NATO
(c) Agency of the EU to deal with all the and WHO.
bilateral and multilateral agreements 2. It monitors the chemical industry to
on trade. prevent new weapons from emerging.
(d) Agency of the EU to deal with the 3. It provides assistance and protection
conflicts arising among the member to States (Parties) against chemical
countries. weapons threats.
Which of the statements given above is/are
30. Which of the following is/are the indicator/ correct?
indicators used by IFPRI to compute the (a) 1 only
Global Hunger Index Report? (2016) (b) 2 and 3 only
1. Undernourishment (c) 1 and 3 only
2. Child stunting (d) 1, 2 and 3
3. Child mortality
Select the correct answer using the code 33. With reference to the ‘Trans-Pacific
given below. Partnership’, consider the following
(a) 1 only statements: (2016)
(b) 2 and 3 only 1. It is an agreement among all the Pacific
(c) 1, 2 and 3 Rim countries except China and Russia.

(d) 1 and 3 only 2. It is a strategic alliance for the purpose


of maritime security only.
Which of the statements given above is/are
31. Consider the following pairs: (2016) correct?
Community sometimes In the affairs of (a) 1 only
mentioned in the news:
(b) 2 only
1. Kurd Bangladesh
(c) Both 1 and 2
2. Madhesi Nepal
(d) Neither 1 nor 2
3. Rohingya Myanmar

742 Current Affairs: World


unacademy.com | Download the Unacademy app
Give your feedback here: Link
34. Consider the following statements: (2016) (d) Organisation for Economic Cooperation
The India-Africa Summit and Development

1. held in 2015 was the third such Summit.


2. was actually initiated by Jawaharlal 38.
‘Beijing Declaration and Platform for
Nehru in 1951. Action’ often seen in the news, is: (2015)

Which of the statements given above is/are (a)


a strategy to tackle the regional
correct? terrorism, an outcome of a meeting of
the Shanghai Cooperation Organization.
(a) 1 only
(b)
a plan of action for sustainable
(b) 2 only
economic growth in the Asia-Pacific
(c) Both 1 and 2 Region, an outcome of deliberations of
(d) Neither 1 nor 2 the Asia-Pacific Economic Forum.
(c) an agenda for women’s empowerment,
an outcome of a World Conference
35. Consider the following statements: (2016)
convened by the United Nations.
1. The New Development Bank has
(d) a strategy to combat wildlife trafficking,
been set up by Asia-Pacific Economic
a declaration of the East Asia Summit.
Cooperation (APEC).
2. The headquarters of New Development
Bank is in Shanghai. 39.
With reference to 'Forest Carbon
Partnership Facility', which of the following
Which of the statements given above is/are
statements is/are correct? (2015)
correct?
1. It is a global partnership of governments,
(a) 1 only
businesses, civil society and indigenous
(b) 2 only peoples.
(c) Both 1 and 2 2. It provides financial aid to universities,
(d) Neither 1 nor 2 individual scientists and institutions
involved in scientific forestry research
to develop eco-friendly and climate
36. Which of the following is not a member of adaptation technologies for sustainable
‘Gulf Cooperation Council’? (2016) forest management.
(a) Iran 3. It assists the countries in their 'REDD+
(b) Saudi Arabia (Reducing Emissions from Deforestation
+)' efforts by providing them with
(c) Oman
financial and technical assistance.
(d) Kuwait
Select the correct answer using the code
given below:
37.
‘Global Financial Stability Report’ is (a) 1 only
prepared by the: (2016)
(b) 2 and 3 only
(a) European Central Bank
(c) 1 and 3 only
(b) International Monetary Fund
(d) 1, 2 and 3
(c) International Bank for Reconstruction
and Development

Current Affairs: World 743


unacademy.com | Download the Unacademy app
Give your feedback here: Link
40.
In the Mekong-Ganga Cooperation, (c) South-East Asia
an initiative of six countries, which of (d) Central Africa
the following is/are not a participant/
participants? (2015)
1. Bangladesh 44.
The terms ‘Agreement on Agriculture’,
‘Agreement on the Application of Sanitary
2. Cambodia and Phytosanitary Measures’ and Peace
3. China Clause’ appear in the news frequently in
4. Myanmar the context of the affairs of the: (2015)

5. Thailand (a) Food and Agriculture Organisation

Select the correct answer using the code (b) United Nations Framework Conference
given below: on Climate Change

(a) 1 only (c) World Trade Organisation

(b) 2, 3 and 4 only (d) United Nations Environment Programme

(c) 1 and 3 only


(d) 1, 2 and 5 only 45.
Which one of the following issues the
‘Global Economic Prospects’ report
periodically? (2015)
41. ‘BioCarbon Fund Initiative for Sustainable (a) The Asian Development Bank
Forest Landscapes’ is managed by the:
(2015) (b) The European Bank for Reconstruction
and Development
(a) Asian Development Bank
(c) The US Federal Reserve Bank
(b) International Monetary Fund
(d) The World Bank
(c) United Nations Environment Program
(d) World Bank
46. Among the following which were frequently
mentioned in the news for the outbreak of
42. Amnesty International is: (2015) Ebola virus recently? (2015)
(a) an agency of the United Nations to help (a) Syria and Jordan
refugees of civil wars. (b) Guinea, Sierra Leone and Liberia
(b) a global Human Rights Movement. (c) Philippines and Papua New Guinea
(c) a non-governmental voluntary (d) Jamaica, Haiti and Surinam
organisation to help very poor people.
(d) an inter-governmental agency to cater
to medical emergencies in war-ravaged 47. Which of the following statements is/
regions. are correct regarding National Innovation
Foundation-India (NIF)? (2015)
1. NIF is an autonomous body of the
43.
The area known as 'Golan Heights' Department of Science and Technology
sometimes appears in the news in the under the Central Government
context of the events related to: (2015)
2. NIF is an initiative to strengthen the
(a) Central Asia highly advanced scientific research in
(b) Middle East India’s premier scientific institutions

744 Current Affairs: World


unacademy.com | Download the Unacademy app
Give your feedback here: Link
in collaboration with highly advanced (b) 2 and 3 only
foreign scientific institutions. (c) 1 and 3 only
Select the correct answer using the code (d) 1, 2 and 3
given below:
(a) 1 only
51. In the context of global oil prices, "Brent
(b) 2 only crude oil" is frequently referred to in the
(c) Both 1 and 2 news. What does this term imply? (2011)
(d) Neither 1 nor 2 1. It is a major classification of crude oil.
2. It is sourced from the North Sea.
48. The ‘Fortaleza Declaration’ recently in the 3. It does not contain sulphur.
news, is related to the affairs of: (2015) Which of the statements given above is/are
(a) ASEAN correct?
(b) BRICS (a) 2 only
(c) OECD (b) 1 and 2 only
(d) WTO (c) 1 and 3 only
(d) 1, 2 and 3
49. Recently, a series of uprisings of people
referred to as 'Arab Spring' originally 52. Southeast Asia has captivated the attention
started from: (2014) of the global community over space and
(a) Egypt time as a geo strategically significant
(b) Lebanon region. Which among the following is the
most convincing explanation for this global
(c) Syria perspective? (2011)
(d) Tunisia (a) It was the hot theatre during the Second
World War.
50. Recently there has been a concern over the (b) Its location is between the Asian powers
short supply of a group of elements called of China and India.
rare earth metals. Why? (2012) (c)
It was the arena of superpower
1. China, which is the largest producer confrontation during the Cold War
of these elements, has imposed some period.
restrictions on their export. (d) Its location between the Pacific and
2. Other than China, Australia, Canada, Indian Oceans and its preeminent
Chile, these elements are not found in maritime character.
any country.
3. Rare earth metals are essential for 53. The "New START" treaty was in the news.
the manufacture of various kinds of What is this treaty? (2011)
electronic items and there is growing
demand for these elements. (a) It is a bilateral strategic nuclear arms
reduction treaty between the USA and
Select the correct answer using the code the Russian Federation.
given below:
(b) It is a multilateral energy security
(a) 1 only

Current Affairs: World 745


unacademy.com | Download the Unacademy app
Give your feedback here: Link
cooperation treaty among the members
of the East Asia Summit.
(c) It is a treaty between the Russian
Federation and the European Union for
energy security cooperation.
(d) It is a multilateral cooperation treaty
among the BRICS countries for the
promotion of trade.

746 Current Affairs: World


unacademy.com | Download the Unacademy app
Give your feedback here: Link
Current Affairs: World-
2 Explanation
1. Answer: (d) cooperation, address shared challenges, and
Somalia has been experiencing prolonged promote economic growth through innovation
periods of civil strife, political instability, and digital transformation.
and food shortages, which have resulted in Statement 2 is incorrect: The EU-US Trade and
humanitarian crises, including severe famines. Technology Council serves as a forum for the
The country has faced significant challenges for United States and European Union to coordinate
several decades, which have had a detrimental approaches to key global trade, economic, and
impact on its people and infrastructure. technology issues and to deepen transatlantic
The civil strife in Somalia dates back to the late trade and economic relations based on these
1980s, when the central government collapsed, shared values.
leading to a prolonged period of civil war and
internal conflicts. These conflicts, along with 4. Answer: (a)
other factors such as droughts, limited access
The Stability and Growth Pact (SGP) is an
to healthcare, and economic challenges, have
agreement among the member states of the
contributed to recurring food shortages and
European Union (EU) aimed at promoting
widespread hunger in the country.
fiscal discipline and coordination of economic
policies. Its primary focus is on maintaining
2. Answer: (c) stability and sustainable economic growth
Statement 1 is correct: Switzerland is the within the Eurozone. The SGP primarily targets
leading exporter of gold in the world. As per budgetary discipline and sets rules for member
the Observatory of Economic Complexity (OEC) states to limit their government budget deficits
data, in 2021, Switzerland exported $86.7B in and public debt levels. The pact requires
Gold, making it the 1st largest exporter of Gold member states to strive for budget deficits
in the world. The main destinations of Gold below 3% of their GDP and aims to ensure that
exports from Switzerland are: India ($29.3B), public debt remains below 60% of their GDP.
China ($16B), United States ($8.13B), Germany The SGP does not involve sharing infrastructure
($5.8B), and Hong Kong ($4.67B). facilities or technologies among the countries
Statement 2 is incorrect: Switzerland has the of the European Union. Its main objective is
seventh largest gold reserves in the world. The to promote fiscal responsibility and stability in
top three countries with largest gold reserves the Eurozone. (Hence Statements 2 and 3 are
in the world are the United States of America incorrect)
(USA) followed by Germany, Italy.
5. Answer: (d)
3. Answer: (c) Statement 1 is incorrect: NOT all the countries
Statement 1 is correct: The United States of of the United Nations have adopted the first-
America (USA) and the European Union (EU) ever compact for international migration, the
have indeed launched the ‘Trade and Technology ‘Global Compact for Safe, Orderly and Regular
Council’ (TTC). The TTC is a platform for the Migration (GCM)’.
USA and the EU to collaborate on trade and Statement 2 is incorrect: The Global Compact
technology-related issues, aiming to enhance for Safe, Orderly and Regular Migration (GCM)
is a non-legally binding agreement. It means

Current Affairs: World-Explanation 747


unacademy.com | Download the Unacademy app
Give your feedback here: Link
that the objectives and commitments outlined relations with Israel, following
in the GCM are not legally enforceable on negotiations mediated by the United
UN member countries. The GCM is intended States. This marked a significant
to serve as a framework for international diplomatic shift for Sudan.
cooperation on migration issues, but it does Statement 2 is incorrect: Israel has not signed
not create legally binding obligations. the Arab Peace Initiative. The Arab Peace
Statement 3 is incorrect: The primary focus Initiative, also known as the “Saudi Initiative,”
of the GCM is on international migration, not was first proposed by Saudi Arabia in 2002
internal migration or internally displaced and later endorsed by the Arab League. The
people. While the GCM recognizes the initiative outlines a comprehensive plan for
importance of protecting the rights of all Arab states to normalize relations with Israel
migrants, regardless of their migration status, in exchange for Israel’s withdrawal from the
its main aim is to address the challenges and occupied territories and the establishment of
opportunities associated with cross-border a Palestinian state.
migration.

7. Answer: (c)
6. Answer: (c) Statement 1 is not correct & Statement 2 is
Statement 1 is correct: In recent years, Israel correct: The U.N. refugee agency now says that
has established diplomatic relations with Bidi Bidi hosts more than 270,000 refugees —
several Arab states, marking a significant shift making it the world’s largest refugee camp.
in regional dynamics. It is now bigger than Kenya’s Dadaab camp,
1. Egypt: Egypt was the first Arab country which has been receiving Somali refugees for
to establish diplomatic relations with more than 20 years. As soon as you set foot
Israel in 1979 under the Camp David in any of the refugee camps along the South
Accords, which led to a peace treaty Sudan border in Uganda, vast human suffering
between the two nations. becomes easily apparent. What began as
a dispute between the president and vice
2. Jordan: Jordan followed suit and
president has turned into a brutal civil war
signed a peace treaty with Israel in
fueled by ethnic tensions.
1994, normalizing diplomatic relations
between the two countries. Statement 3 is correct: The first camp was
established in 1991 when refugees fleeing the
3. United Arab Emirates (UAE): In August
civil war in Somalia started to cross the border
2020, the UAE and Israel signed the
into Kenya. A second large influx occurred in
historic Abraham Accords, which
2011, when some 130,000 refugees arrived,
included the establishment of full
fleeing drought and famine in southern Somalia.
diplomatic ties between the two
countries. This marked a significant
development in regional relations. 8. Answer: (a)
4. Bahrain: Also in August 2020, Bahrain Option (a) is correct: Greenwashing is the
and Israel signed the Abraham Accords, process of conveying a false impression or
establishing diplomatic relations and providing misleading information about how a
normalization of ties between the two company’s products are more environmentally
countries. sound. Greenwashing is considered an
5. Sudan: In October 2020, Sudan unsubstantiated claim to deceive consumers
announced its intention to normalize into believing that a company’s products are
environmentally friendly.

748 Current Affairs: World-Explanation


unacademy.com | Download the Unacademy app
Give your feedback here: Link
9. Answer: (c) foremost sports competition and include
Statement 1 is correct: According to a February athletes from all over the world.
2017 prediction by PricewaterhouseCoopers, Statement 1 is not correct: On 20 July 2021,
Vietnam’s economy could be the fastest- the International Olympic Committee approved
growing in the world, with a potential annual a change in the Olympic motto that recognises
GDP growth rate of roughly 5.1 percent, making the unifying power of sport and the importance
it the world’s 10th-largest by 2050. of solidarity. The new Olympic motto now
Statement 2 is incorrect: Vietnam is a one- reads: “Faster, Higher, Stronger – Together”.
party communist state, not a parliamentary Statement 2 is correct: The International
democracy. The ruling Communist Party treats Olympic Committee (IOC) added baseball/
the National Assembly as a rubber stamp and softball, karate, skateboard, sports climbing
the electoral process is neither free nor fair. and surfing to the sports programme in the
Statement 3 is correct: Vietnam’s economic Olympic Games held in Tokyo.
development is dependent on its integration
into global supply chains and its export- 12. Answer: (c)
oriented strategy.
The Laureus World Sports Awards is the
Statement 4 is correct: Low labour costs premier global sporting award. First held in
and stable exchange rates attract foreign 2000, the annual event honours the greatest
investment to Vietnam. and most inspirational sporting triumphs of
the year. At the inaugural Laureus World Sports
10. Answer: (d) Awards in 2000, Nelson Mandela made his
famous speech about sports having the power
The International Cricket Council (ICC) is the
to change the world.
global governing body for cricket. The ICC World
Test Championship is a league competition for Statement 1 is correct: The inaugural winner of
Test cricket run by the International Cricket the award was the American golfer Tiger Woods
Council (ICC). who finished the 1999 season with eight wins.

Statement 1 is not correct: As per the Statement 2 is not correct: The award was
current regulations for the ICC World Test received mostly by tennis players.
Championship (WTC), the finalists are decided Statement 3 is correct: Roger Federer received
by the percentage of points (PCT) earned by this award the maximum number of times
individual teams. PCT is the percentage of compared to others.
points won out of the total number of points
contested by each team.
13. Answer: (b)
Statement 2 is not correct: New Zealand has
ranked ahead of England in WTC 2019-21 Statement 1 is not correct: The value of Indo-
because the percentage of points won by New Sri Lanka trade has not consistently increased
Zealand was more than England. They won 70 in the last decade. It registered a downfall
percent as against England who won 68.7 post 2014-15. While the import value almost
remains stagnant or at a very slow growth rate,
the sharp decline in exports value has brought
11. Answer: (b) down the total value of the trade in the past
The Olympic Games are an athletic festival that decade.
originated in ancient Greece and was revived Statement 2 is correct: “Textile and Textile
in the late 19th century. They are the world’s articles” such as the import of readymade

Current Affairs: World-Explanation 749


unacademy.com | Download the Unacademy app
Give your feedback here: Link
garments from Bangladesh and export of fabric
from India constitute an important link item in Pair 4 is not correctly matched. Thus,
the Indo-Bangladesh trade. Option (c) is correct.
Statement 3 is not correct: Bangladesh has
been India’s largest trading partner in South 15. Answer: (c)
Asia followed by Nepal in the last five years. Option (c) is correct: The Global Competitiveness
Report is published by the World Economic
Forum on a yearly basis. It ranks economies
14. Answer: (c)
based upon their performance on the twelve
Pair 1 is correctly matched: The Alma-Ata pillars of competitiveness which include:
Declaration of 1978 is a major milestone in the Institutions, Infrastructure, ICT adoption,
field of public health, and it identified primary Macroeconomic stability, Health, Skills,
healthcare as the key to the attainment to the Product market, Labour market, Financial
goal of healthcare of the people. system, Market size, Business dynamism and
Pair 2 is not correctly matched: The Hague Innovation capability.
Convention on the Protection of Children India ranked 68th in the 2019 edition of this
and Co-operation in Respect of Intercountry report, down from 58th in the previous year
Adoption is an international agreement to (2018).
safeguard intercountry adoptions. Another
Additional Information:
Hague Convention which is on the Civil Aspects
of International Child Abduction, provides The World Economic Forum was established
an expeditious method to return a child in 1971 as a not-for-profit foundation and
internationally abducted by a parent from one- is headquartered in Geneva, Switzerland.
member country to another. The Forum engages the foremost political,
business, cultural and other leaders of society
Pair 3 is correctly matched: Talanoa is a
to shape global, regional and industry agendas.
traditional word used in Fiji and across the
Pacific to reflect a process of inclusive, Some other important reports published by
participatory, and transparent dialogue. The WEF are:
Talanoa Dialogue of UNFCCC is to take stock y Global Environmental Performance Index
of the collective efforts of Parties in relation
y Social Mobility Index
to progress towards the long-term goal of
the Paris Climate Agreement and to inform y Global Gender Gap Report
the preparation of nationally determined y Human Capital Index
contributions. It is related to global climate
y Travel and Tourism Competitiveness Index
change.
y Enabling Trade Report
Pair 4 is not correctly matched: The Under2
Coalition is a global community of state
and regional governments committed to 16. Answer: (a)
ambitious climate action in line with the Paris
The ‘Ease of Doing Business Index’ is published
Agreement. Signatories commit to keeping
by the World Bank every year. The index has
global temperature rises to well below 2°C
been created to evaluate the impact of the
with efforts to reach 1.5°C.
economic policies or reforms implemented by
Elimination Technique: Talanoa Dialogue the government in the respective countries to
and Under2 Coalition are related to climate promote and make it easier to do business.
change. Pair 3 is correctly matched and Option (a) is not correct: This report is

750 Current Affairs: World-Explanation


unacademy.com | Download the Unacademy app
Give your feedback here: Link
published after evaluating the performance of It is a man-made famine, stemming from
countries under various parameters or sub- rapacious power struggles, is threatening the
indexes which are Starting a Business, Dealing lives of innocent civilians, especially children,
with Construction Permits, Getting Electricity, who, even if they survive, are likely to be
Registering Property, Getting Credit, Protecting mentally or physically stunted for the rest of
Minority Investors, Paying Taxes, Trading across their lives.
Borders, Enforcing Contracts and Resolving
Insolvency.
19. Answer: (b)
Maintenance of law and order is not a sub-
index of the World Bank’s Ease of Doing Option (b) is correct: The two-state solution
Business Index. would establish an independent Palestinian
state alongside Israel — two states for two
peoples. This is a proposed framework for
17. Answer:(d) resolving the Israeli-Palestinian conflict by
Option (d) is correct: The Rule of Law Index is establishing two states for two peoples:
released by the World Justice Project. The Index Israel for the Jewish people and Palestine
is a quantitative assessment tool designed to for the Palestinian people. In 1993 the Israeli
offer a comprehensive picture of the extent government and the Palestine Liberation
to which countries adhere to the rule of law. Organization (PLO) agreed on a plan to
The scores of the Index can be interpreted as implement a two-state solution as part of the
degrees of compliance of a legal system to a Oslo Accords, leading to the establishment of
definition of Rule of Law. the Palestinian Authority (PA).

The index measures countries’ rule of law


performance across eight factors: Constraints 20. Answer: (c)
on Government Powers, Absence of Corruption, Option (c) is correct: THAAD is a relatively
Open Government, Fundamental Rights, Order recent addition to the United States’ anti-
and Security, Regulatory Enforcement, Civil ballistic missile/interceptor toolkit. THAAD
Justice, and Criminal Justice. is particularly well-suited to intercept and
destroy short, medium, and intermediate-
18. Answer: (d) range ballistic missiles in their terminal phase.

The United Nations (UN) defines famine as a THAAD is able to intercept incoming missiles
crisis where at least 20 percent of a region’s at endo- and exo-atmospheric altitudes, with
population does not have sufficient food to be a maximum engagement altitude of roughly 93
healthy, more than 30 percent of children under miles above the earth’s surface. In South Korea,
the age of five are severely malnourished, and the THAAD missile defence system is operated
two in 10,000 people or four in 10,000 children by the US army stationed in the country.
die due to extreme hunger every day. The US had previously announced that the
deployment of this missile defence system was
Option (d) is correct: Presently, around 20
a countermeasure against potential attacks by
million people, including 1.4 million children,
North Korea, particularly after the country had
are inching closer to this state of living as
engaged in testing ballistic missiles.
the spectre of famine looms large over South
Sudan, Somalia, Northeast Nigeria, and Yemen.
People in these regions either suffered from 21. Answer: (b)
severe famine/acute malnutrition or died due Option (b) is correct: The Global Infrastructure
to starvation caused by war/ethnic conflicts. Facility (GIF), a G20 initiative, is a global

Current Affairs: World-Explanation 751


unacademy.com | Download the Unacademy app
Give your feedback here: Link
collaboration platform that integrates efforts Statement 3 is correct: UN-Habitat promotes
to boost private investment in sustainable, urbanization as a positive transformative
quality infrastructure projects in developing force for people and communities, reducing
countries and emerging markets. The GIF inequality, discrimination, and poverty. UN-
is co-chaired by the World Bank Group. It Habitat contributed to the overall objective of
enables collective action among a wide range the United Nations system to reduce poverty
of partners – including donors, development and to promote access to safe drinking water
finance institutions, country governments, and basic sanitation.
with inputs of private sector investors and
financiers – to leverage resources and
expertise and find solutions to build bankable 23. Answer: (a)
pipelines of infrastructure projects that attract Option (a) is correct: The ‘Broad-based Trade
private financing. Quality infrastructure drives and Investment Agreement (BTIA)’ is a trade
economic growth, social progress, and climate negotiation between India and the European
action. Nowhere are low-carbon, climate- Union. These negotiations are pursuant to the
resilient, sustainable infrastructure needs commitment made by political leaders at the
greater than in emerging markets, where 7th India-EU Summit held in Helsinki in 2006
infrastructure deficits are large at best and to move towards negotiations for a broad-
staggering at worst. based trade and investment agreement on the
basis of the report of the India-EU High-Level
Technical Group.
22. Answer: (b)
India and the EU expect to promote bilateral
Statement 1 is correct: The United Nations trade by removing barriers to trade in goods
Human Settlements Programme, UN-Habitat, and services and investment across all sectors
is the United Nations agency for human of the economy. The negotiations cover Trade in
settlements. It is mandated by the UN Goods, Trade in Services, Investment, Sanitary
General Assembly to promote socially and and Phytosanitary Measures, Technical Barriers
environmentally sustainable towns and cities to Trade, Trade Remedies, Rules of Origin,
with the goal of providing adequate shelter for Customs and Trade Facilitation, Competition,
all. Trade Defence, Government Procurement,
Statement 2 is not correct: UN-Habitat is the Dispute Settlement, Intellectual Property Rights
coordinating agency within the United Nations and Geographical Indications, Sustainable
System for human settlement activities and in Development.
collaboration with governments responsible
for promoting and consolidating collaboration
with all partners, including local authorities, 24. Answer: (d)
private and non-governmental organisations The Nuclear Security Summit provides a forum
in the implementation of the Sustainable for leaders to engage with each other and
Development Goals (SDGs), in particular, Goal reinforce their commitment at the highest
11 of “Making cities and human settlements levels to securing nuclear materials. The
inclusive, safe, resilient and sustainable”. first Nuclear Security Summit was held in
UN-Habitat is a focal point for the monitoring, Washington, DC, in 2010 and was followed by
evaluation and implementation of the New additional Summits in Seoul in 2012 and the
Urban Agenda adopted during the United Hague in 2014. These Summits have achieved
Nations Conference on Housing and Sustainable tangible improvements in the security of
Urban Development (Habitat III) in Ecuador, nuclear materials and stronger international
Quito, 2016. institutions that support nuclear security.

752 Current Affairs: World-Explanation


unacademy.com | Download the Unacademy app
Give your feedback here: Link
Statement 1 is not correct: In 2009 Prague growth.
speech, then President of the USA, Obama y Global Social Mobility Index: The index
stated that nuclear terrorism “is the most focuses on drivers of relative social mobility.
immediate and extreme threat to global It benchmarks progress on social mobility
security.” To mitigate this threat, the President and makes effective comparisons across
urged that “we act with purpose and without regions as well as generations. The ten
delay,” announcing “a new international effort pillars of the index are Health, Education
to secure vulnerable nuclear material around access, Education quality and equity,
the world” that would begin with “a Global Lifelong Learning, Access to Technology,
Summit on Nuclear Security that the United Work Opportunities, Fair wages, Work
States will host.” This is not held under the Conditions, Social Protection, Efficient and
aegis of the United Nations. Inclusive Institutions.
Statement 2 is not correct: The International y Global Risk Report: It analyses the risks
Panel on Fissile Materials (IPFM) was founded from societal fractures manifested through
in January 2016 and is an independent group persistent and emerging risks to human
of arms-control and non-proliferation experts health, rising unemployment, widening
from both nuclear weapon and non-nuclear digital divides, youth disillusionment, and
weapon states. It is an organ of the International geopolitical fragmentation.
Atomic Energy Agency.
y Travel and Tourism Competitiveness Report:
The mission of the IPFM is to analyse the The index is a measurement of the factors
technical basis for practical and achievable that make it attractive to develop business
policy initiatives to secure, consolidate, and in the travel and tourism industry of
reduce stockpiles of highly enriched uranium individual countries, rather than a measure
and plutonium. These fissile materials are of a country’s attractiveness as a tourist
the key ingredients in nuclear weapons, and destination.
their control is critical to nuclear weapons
disarmament, halting the proliferation of
nuclear weapons and ensuring that terrorists 26. Answer: (c)
do not acquire nuclear weapons. Option (c) is correct: The Digital Single Market
Strategy was adopted by the European Union
25. Answer: (a) (EU) in 2015. This strategy is based on three
broad pillars,
Option (a) is correct: The Global Gender
Gap Index report is published by the World y Improving access to digital goods and
Economic Forum. The report captures the services: The Digital Single Market
magnitude of gender-based disparities and strategy seeks to ensure better access for
tracks their progress over time. consumers and businesses to online goods
and services across Europe, for example,
The report benchmarks countries on four by removing barriers to cross-border
dimensions, namely: Economic Participation e-commerce and access to online content
and Opportunity, Educational Attainment, while increasing consumer protection.
Health and Survival and Political Attainment.
y An environment where digital networks
Some of the other reports published by the and services can prosper: The Digital Single
World Economic Forum are: Market aims to create the right environment
y Global Competitiveness Report: It provides for digital networks and services by providing
an annual assessment of the drivers of high-speed, secure and trustworthy
productivity and long-term economic infrastructures and services supported

Current Affairs: World-Explanation 753


unacademy.com | Download the Unacademy app
Give your feedback here: Link
by the right regulatory conditions. Key 29. Answer: (b)
concerns include cybersecurity, data Option (b) is correct: The European Stability
protection/e-privacy, and the fairness and Mechanism (ESM) was set up as an International
transparency of online platforms. Financial Institution by the euro area Member
y Digital as a driver for growth: The Digital States to help euro area countries in severe
Single Market Strategy aims at maximising financial distress. It provides emergency loans
the growth potential of the European Digital but in return, countries must undertake reform
Economy so that every European can fully programmes. Together with its predecessor,
enjoy its benefits – notably by enhancing the European Financial Stability Facility (EFSF),
digital skills, which are essential for an it can lend a total of €700 billion.
inclusive digital society. With a paid-in capital of more than €80 billion,
the ESM is one of the largest International
27. Answer: (b) Financial Institutions in the world. The ESM
is the only official institution of the euro area.
Option (b) is correct: The Regional
Combined with the EFSF, it has disbursed €250
Comprehensive Economic Partnership (RCEP) is
billion in loans during the crisis, more than
a regional free trade agreement signed in 2020
three times what the IMF disbursed globally
between ASEAN and five countries namely
during that period. It is one of the largest
China, Japan, South Korea, Australia, and New
issuers of euro-denominated debt in the world.
Zealand. RCEP negotiations were launched
in November 2012 between the Association
of Southeast Asian Nations (ASEAN includes 30. Answer: (c)
Brunei, Cambodia, Indonesia, Laos, Malaysia, The Global Hunger Index (GHI) report is
Myanmar, Philippines, Singapore, Thailand, and released by the International Food Policy
Vietnam) and ASEAN’s free trade agreement Research Institute (IFPRI) annually. The report
partners (Australia, China, India, Japan, New comprehensively measures and tracks hunger
Zealand, and Republic of Korea). RCEP formed at the global, regional, and country levels.
the world’s largest trading bloc, covering nearly GHI scores are calculated each year to assess
a third of the global economy. progress and setbacks in combating hunger.
There are four indicators used to compute the
28. Answer: (a) Global Hunger Index, which are:
Statement 1 is correct. The International Solar y Undernourishment: The share of the
Alliance (IAS) was launched at the United population that is undernourished (whose
Nations Climate Change Conference in 2015. The caloric intake is insufficient). (Option 1 is
ISA is a treaty-based on an intergovernmental correct)
organisation working to create a global market y Child Wasting: The share of children under
system to tap the benefits of solar power and the age of five who are wasted (who have
promote clean energy applications. low weight for their height, reflecting acute
Statement 2 is not correct. The ISA is a joint undernutrition).
initiative of France and India. It will be open y Child Stunting: The share of children under
to all UN members following an amendment the age of five who are stunted (who have
to its framework agreement, which is currently low height for their age, reflecting chronic
limited to 122 intertropical countries. The ISA undernutrition). (Option 2 is correct)
has 122 sunbelt countries that lie between
the two tropics as its prospective member y Child Mortality: The mortality rate of
countries. children under the age of five (in part, a

754 Current Affairs: World-Explanation


unacademy.com | Download the Unacademy app
Give your feedback here: Link
reflection of the fatal mix of inadequate which prohibits the use, stockpiling, or transfer
nutrition and unhealthy environments). of chemical weapons by signatory states. It
(Option 3 is correct) closely monitors the chemical industry to
International Food Policy Research Institute prevent new weapons from emerging.
(IFPRI): IFPRI was established in 1975. It Statement 3 is correct: The Chemical Weapons
provides research-based policy solutions to Convention codified an international norm
sustainably reduce poverty and end hunger and against chemical weapons, to which all States
malnutrition in developing countries. IFPRI’s must adhere. Protecting this norm requires
vision is a world free of hunger and malnutrition. constant vigilance. While the work of the
Its mission is to provide research-based policy OPCW is largely focused on preventing the use
solutions that sustainably reduce poverty and of chemical weapons through its verification
end hunger and malnutrition. activities, destruction of existing stockpiles,
and capacity building in Member States.

31. Answer: (c)


Pair 1 is not correctly matched: The Kurds 33. Answer: (d)
are one of the indigenous peoples of the Statement 1 is not correct: The Trans-Pacific
Mesopotamian plains. Kurds inhabit in south- Partnership (TPP) was a proposed free trade
eastern Turkey, north-eastern Syria, northern agreement between the United States and
Iraq, north-western Iran and south-western eleven other countries that border the Pacific
Armenia. Ocean: Australia, Brunei, Canada, Chile,
Pair 2 is correctly matched: Madhesi is an Japan, Malaysia, Mexico, New Zealand, Peru,
indigenous ethnic group of Nepal’s Terai region. Singapore, the United States, and Vietnam. It is
The Madhesis have castes and ethnicity similar not an agreement between all the Pacific Rim
to Bihar and eastern UP, with frequent inter- countries except China and Russia. Only 12 out
marriages between families on either side of of more than 50 Pacific Rim countries are part
the border. of it.

Pair 3 is correctly matched: The Rohingya are Statement 2 is not correct: The Trans-Pacific
an ethnic Muslim minority who practice a Partnership’ is a trade bloc and is not related to
Sufi-inflected variation of Sunni Islam. Major a strategic alliance for the purpose of maritime
population of Rohingya inhabited in Myanmar’s security. In the United States, the deal was
Rakhine State. viewed in the broader context of the military
and diplomatic “pivot” toward East Asia.

32. Answer: (b)


34. Answer: (a)
Statement 1 is not correct: The Organisation
for the Prohibition of Chemical Weapons Statement 1 is correct: India–Africa Forum
(OPCW) is an autonomous intergovernmental Summit (IAFS) is the official platform for
organisation and the implementing body for the African Indian relations. The Heads of State
Chemical Weapon Convention. Its headquarters and Government and Heads of Delegation
is located in Hague, Netherlands. It is not an representing the continent of Africa, the
organisation of the European Union. African Union (AU) and its Institutions, and the
Prime Minister of the Republic of India, met in
Statement 2 is correct: OPCW was established
New Delhi, India on 29 October 2015 for the
by the Chemical Weapons Convention (adopted
third India Africa Forum Summit; the theme of
1992, entered into force 1997) to implement and
the summit was Partners in Progress: Towards
enforce the terms of the international treaty,
a Dynamic and Transformative Development

Current Affairs: World-Explanation 755


unacademy.com | Download the Unacademy app
Give your feedback here: Link
Agenda. 37. Answer: (b)
Statement 2 is not correct: The first such Option (b) is correct: The Global Financial
summit was held from April 4 to April 8, 2008, Stability Report is a semi-annual report
in New Delhi, India and it was initiated by Dr. published by the International Capital Markets
Manmohan Singh. This is a historic Summit division of the International Monetary Fund
between India and countries representing the (IMF).
AU and the Regional Economic Communities of y The report provides an assessment of the
Africa. global financial system and markets and
addresses emerging market financing in a
35. Answer: (b) global context.

Statement 1 is not correct: At the fourth BRICS y It focuses on current market conditions,
Summit in New Delhi (2012), the leaders of highlighting systemic issues that could
Brazil, Russia, India, China, and South Africa pose a risk to financial stability and
considered the possibility of setting up a sustained market access by emerging
New Development Bank (NDB) to mobilise market borrowers.
resources for infrastructure and sustainable y Some of the other reports published by
development projects in BRICS and other the IMF are the ‘World Economic Outlook’
emerging economies, as well as in developing report, Fiscal Monitor, etc.
countries. During the sixth BRICS Summit
in Fortaleza (2014), the leaders signed the
Agreement establishing the New Development 38. Answer: (c)
Bank. NDB was not set up by Asia-Pacific Option (c) is correct: In 1995, 189 governments
Economic Cooperation (APEC). were joined by 17,000 participants and 30,000
Statement 2 is correct: The headquarters of non-governmental activists in Beijing for
New Development Bank is located in Shanghai, the opening of the Fourth World Conference
China. on Women. For two weeks, government
representatives worked on producing a
The Bank has an initial authorised capital of document of agreed written targets towards
US$ 100 billion. The initial subscribed capital achieving gender equality. The outcome of
shall be US$ 50 billion, equally shared among this momentous conference was the Beijing
founding members. Declaration and Platform for Action, a holistic
document for women empowerment and
36. Answer: (a) gender-neutral society.

Option (a) is correct: Gulf Cooperation Council


(GCC), a political and economic alliance of six 39. Answer: (c)
Middle Eastern countries: Saudi Arabia, Kuwait, Statement 1 is correct: The Forest Carbon
the United Arab Emirates, Qatar, Bahrain, and Partnership Facility (FCPF) is a global
Oman. Iran is not a member of the GCC. partnership of governments, businesses, civil
The GCC was established in Riyadh, Saudi society, and Indigenous peoples, which focused
Arabia, in May 1981. The purpose of the GCC on reducing emissions from deforestation
is to achieve unity among its members based and forest degradation, forest carbon stock
on their common objectives and their similar conservation, the sustainable management of
political and cultural identities, which are forests, and the enhancement of forest carbon
rooted in Arab and Islamic cultures. Presidency stocks in developing countries, activities
of the council rotates annually. commonly referred to as REDD+.

756 Current Affairs: World-Explanation


unacademy.com | Download the Unacademy app
Give your feedback here: Link
Statement 2 is not correct: Launched in India and five Mekong countries (Cambodia,
2008, the FCPF now works with 47 developing Lao PDR, Myanmar, Thailand, and Vietnam),
countries across Africa, Asia, and Latin America launched in 2000 at Vientiane, Lao PDR for
and the Caribbean, along with 17 donors that cooperation in tourism, culture, education, as
have made contributions and commitments well as transport and communications. Both
totalling $1.3 billion. It provides grants to the Ganga and the Mekong are civilizational
developing countries, not universities. rivers, and the MGC aims to build on the long-
Statement 3 is correct: One of the four standing civilizational linkages between the
objectives of the FCPF is to assist countries people inhabiting these two major river basins.
in their REDD+ efforts by providing them with Bangladesh and China are not participants.
financial and technical assistance in building y The Mekong region is very important for
their capacities to benefit from possible future India’s ‘Act East Policy’ for strengthening its
systems of positive incentives for REDD+. economic integration with Southeast and
Other Objectives are: East-Asian countries.

y To pilot a performance-based payment y India has accorded high priority to economic


system for REDD+ activities, with a view engagement with the MGC countries,
to ensuring equitable benefit sharing and working towards establishing seamless
promoting future large-scale positive physical and digital connectivity, as well
incentives for REDD+. as capacity building under the Initiative
for ASEAN Integration and Narrowing the
y Within the approach to REDD+, to test ways
Development Gap.
to sustain or enhance livelihoods of local
communities and to conserve biodiversity.
y To disseminate broadly the knowledge 41. Answer: (d)
gained in the development of the Facility Option (d) is correct: The BioCarbon Fund
and the implementation of Readiness Initiative for Sustainable Forest Landscapes
Preparation Proposals (RPPs) and Emission initiative, a public-private program housed
Reductions Programs (ERPs). within and managed by the World Bank that
Additional Information: mobilises finance for activities that sequester
or conserve carbon emissions in forest and
The FCPF supports REDD+ efforts through two
agricultural systems.
separate but complementary funds.
y The BioCarbon Fund Initiative for
y The FCPF Readiness Fund, which helps
Sustainable Forest Landscapes (ISFL)
countries set up the building blocks to
collaborates with countries around the
implement REDD+.
world to reduce emissions from the land
y The FCPF Carbon Fund, is a results- sector through smarter land-use planning,
based payment to countries that have policies, and practices.
advanced through REDD+ readiness
y ISFL aims to help rural communities
and implementation and have achieved
address poverty and develop sustainably
verifiable emission reductions in their
while simultaneously reducing land-based
forest and broader land-use sectors.
greenhouse gas emissions. To accomplish
these goals, ISFL programs seek to build
40. Answer: (c) on experience, leverage partnerships,
incentivize results, and emphasise working
Option (c) is correct: Mekong-Ganga Cooperation
at scale.
(MGC) is an initiative of the Government of

Current Affairs: World-Explanation 757


unacademy.com | Download the Unacademy app
Give your feedback here: Link
42. Answer: (b)
Amnesty International is an international Non-
governmental Organization (NGO) founded in
London on May 28, 1961, that seeks to publicize
violations by governments and other entities of
rights recognized in the Universal Declaration
of Human Rights (1948), especially freedom of
speech and of conscience and the right against
torture.
Option (b) is correct: Amnesty International is
a global movement campaigning for a world
where human rights are enjoyed by all. It
actively seeks the release of political prisoners
and the relief, when necessary, of their families.
It also works with intergovernmental human
rights bodies to expand and enforce human
rights protections in international law.

43. Answer: (b)


Option (b) is correct: The Golan Heights, a rocky
plateau in south-western Syria, (Middle East
region) has a political and strategic significance 44. Answer: (c)
which belies its size. Option (c) is correct: The WTO Agreement on
y Israel seized the Golan Heights from Syria in Agriculture, which came into force in 1995,
the closing stages of the 1967 Six-Day War. represents a significant step towards reforming
Most of the Syrian Arab inhabitants fled the agricultural trade and making it fairer and more
area during the conflict. Israel unilaterally competitive. Article 13 (“due restraint”) of the
annexed the Golan Heights in 1981. Agriculture Agreement protects countries using
y The Golan is a hilly, 1,200-square-kilometre subsidies which comply with the agreement
(460-square-mile) plateau overlooking from being challenged under other WTO
Lebanon, Syria and the Jordan Valley. agreements.
The Golan catchment area feeds into the y Without this “peace clause”, countries
Jordan River and Sea of Galilee, both major would have greater freedom to take action
sources of water for Israel. against each other’s subsidies, under the
Subsidies and Countervailing Measures
Agreement and related provisions.
y The Agreement on the Application of
Sanitary and Phytosanitary Measures (the
“SPS Agreement”) entered into force with
the establishment of the World Trade
Organization on 1 January 1995. It concerns
the application of food safety and animal
and plant health regulations.

758 Current Affairs: World-Explanation


unacademy.com | Download the Unacademy app
Give your feedback here: Link
45. Answer: (d) y Ebola virus disease (EVD), formerly known
Option (d) is correct: Global Economic as Ebola haemorrhagic fever, is a rare but
Prospects is a flagship report of the World severe, often fatal illness in humans.
Bank Group that examines global economic y The virus is transmitted to people from
developments and prospects, with a special wild animals and spreads in the human
focus on emerging markets and developing population through human-to-human
economies. It is issued twice a year. transmission.
Some other reports published by the World y The average EVD case fatality rate is around
Bank are: 50%. Case fatality rates have varied from
y Ease of Doing Business Report 25% to 90% in past outbreaks.

y World Development Report y Community engagement is key to


successfully controlling outbreaks.
y Ease of Living Index
y Good outbreak control relies on applying
y The Service Trade Restriction Index
a package of interventions, namely case
Additional Information: management, infection prevention and
y The World Bank is a multilateral institution control practices, surveillance and contact
established in 1944. It comprises two tracing, a good laboratory service, safe and
institutions: The International Bank for dignified burials and social mobilisation.
Reconstruction and Development (IBRD)
and the International Development
47. Answer: (a)
Association (IDA).
Statement 1 is correct: The National Innovation
y The World Bank is different from the
Foundation (NIF) – India, an autonomous
World Bank Group. The World Bank Group
body, was set up in March 2000 with the
comprises three more institutions along
assistance of the Department of Science and
with the IBRD and IDA, which are:
Technology, Government of India. It is India’s
1. International Finance Corporation (IFC) national initiative to strengthen grassroots
2. Multilateral Investment Guarantee Agency technological innovations and outstanding
(MIGA) traditional knowledge.

3. International Centre for Settlement of Statement 2 is not correct: Its mission is to help
Investment Disputes (ICSID) India become a creative and knowledge-based
society by expanding policy and institutional
space for grassroots technological innovators.
46. Answer: (b) NIF supports grassroots innovations developed
Option (b) is correct: The 2014 outbreak of by individuals and local communities in
Ebola virus disease in West Africa was the any technological field, helping in human
“largest, most severe and most complex survival without any help from the formal
Ebola epidemic” in history, according to the sector. NIF helps grassroots innovators and
World Health Organization. More than 28,000 outstanding traditional knowledge holders get
people were infected, and over 11,000 people due recognition, respect and reward for their
died before the international public health innovations.
emergency ended in June 2016. The outbreak
started in Guinea and then moved across land
48. Answer: (b)
borders to Sierra Leone and Liberia.
Option (b) is correct: The Fortaleza Declaration
Ebola:
of heads of state from Brazil, Russia, India,

Current Affairs: World-Explanation 759


unacademy.com | Download the Unacademy app
Give your feedback here: Link
China, and South Africa (BRICS countries) is the limiting exports, by establishing quotas and
latest evidence in the rise of regional assertion banning the sale of some products outside the
in the global field. country altogether due to the risk of a shortage
y The leaders of the BRICS nations through of rare earth metals which would be a serious
the “Fortaleza Declaration” have pulled problem for businesses around the world and
off a coup of sorts in announcing the could lead to higher prices for many consumer
establishment of a New Development Bank goods.
(NDB) with an initial subscribed capital of Statement 2 is not correct: During the 1990s
$50 billion. and for much of the past decade, China was
y Significantly, they have chosen to share able to produce rare earth more cheaply than
the capital equally among themselves. other countries, leading to the closure of
The capital base is to be used for mines elsewhere, notably in Australia and the
funding infrastructure and “sustainable US. Therefore, China is not the only country
development” projects in the BRICS where these rare earth metals are found.
countries initially. Statement 3 is correct: Cerium, for example,
is an abrasive used in the manufacture
of flat screen televisions. Lanthanum is a
49. Answer: (d) catalyst much prized by the oil industry, while
Option (d) is correct: The Arab Spring began neodymium is found in computer hard drives.
with the self-immolation of Mohamed Bouazizi
on 17 December 2010. They led to the ousting
of Ben Ali on 14 January 2011, when he officially 51. Answer: (b)
resigned after fleeing to Saudi Arabia, ending The price of crude oil like any other commodities
his 23 years in power. in the free market economies largely depends
y The key driving force behind the protests on its demand and supply conditions in the
was a pan-Arabist anger against the old market.
system. Also, the economic model based Option (b) is correct: There are over 160 different
on patronage was crumbling in those types of crude oil traded on the market, like
countries. Western Texas Intermediate (WTI), Brent Crude,
y The Arab Spring is a revolutionary wave of OPEC Reference Basket, Dubai Crude, Bonny
demonstrations, protests and civil wars in Light and Urals etc. When it comes to physical
the Arab world that spread throughout the oil, there are different grades or classes. The
countries of the Arab League. It originally most popular traded grades are Brent crude
started from Tunisia. North Sea Crude (commonly known as Brent
crude) and West Texas Intermediate (commonly
known as WTI). Brent refers to oil that is
50. Answer: (c) produced in the Brent oil fields and other sites
Lanthanum, cerium, and neodymium are the in the North Sea. This grade is described as
“rare earth” metals which play a vital role in light because of its relatively low density, and
many modern technologies. sweet because of its low sulphur content.

Statement 1 is correct: China is the largest Additional Information:


producer of Lanthanum, Cerium and Brent Crude: Brent Crude is the benchmark for
Neodymium and the bulk of them mined in the African, European and Middle Eastern crude
mountains of Inner Mongolia. China controlled oils. This grade dictates nearly two-thirds of
almost 97% of the market for these “rare earth” the world’s crude oil production. OPEC also
metals. But in recent years, China has been uses Brent crude as reference price.

760 Current Affairs: World-Explanation


unacademy.com | Download the Unacademy app
Give your feedback here: Link
WTI: It is the benchmark for crude oil for the mainland (insular Southeast Asia). Extending
United States. It, however, continues to be the some 700 miles (1,100 km) southward from
main benchmark oil consumed in the United the mainland and insular Southeast Asia is the
States. Malay Peninsula.
Option (d) is correct: Southeast Asia is at the
52. Answer: (d) crossroads of the Pacific Ocean and the Indian
Ocean hosting some of the most important
Southeast Asia, a vast region of Asia situated maritime trade routes in the world. It also has
east of the Indian subcontinent and south of been of great importance that Southeast Asia,
China. It consists of two dissimilar portions: which is the most easily accessible tropical
a continental projection (commonly called region in the world, lies strategically astride
mainland Southeast Asia) and a string of the sea passage between East Asia and the
archipelagoes to the south and east of the Middle Eastern–Mediterranean world.

Current Affairs: World-Explanation 761


unacademy.com | Download the Unacademy app
Give your feedback here: Link
53. Answer: (a) is “Measures for the further reduction and
Option (a) is correct: New START is an agreement Limitation of Strategic Offensive Arms”. The
for nuclear arms reduction between the United Treaty was signed to reduce nuclear arms
States and Russian Federation, signed in production and utilisation by the US and
2010 in Prague (capital of Czech Republic). It Russia.
entered into force in 2011. It established a limit y Under the Treaty, the countries will reduce
on deployed strategic warheads. The Treaty their strategic nuclear missile launchers by
limits the United States and Russia on the half. The treaty established an inspection
deployments of warheads. and verification regime replacing the Treaty
y The formal name of the New START Treaty of Moscow (SORT).

762 Current Affairs: World-Explanation


unacademy.com | Download the Unacademy app
Give your feedback here: Link
3 GK/Persons in News

1. With reference to coal-based thermal according to the Flag Code of India, 2002:
power plants in India, consider the (2023)
following statements: (2023) Statement-I:
1. None of them uses seawater. One of the standard sizes of the National
2. None of them are set up in water- Flag of India is 600 mm × 400 mm.
stressed districts. Statement-II:
3. None of them is privately owned. The ratio of the length to the height (width)
How many of the above statements are of the Flag shall be 3 : 2.
correct? Which one of the following is correct in
(a) Only one respect of the above statements?
(b) Only two (a) Both Statement-I and Statements-II are
(c) All three correct and Statement-II is the correct
explanation for Statement-I
(d) None
(b) Both Statement-I and Statements-II
are correct and Statement-II is not the
2. Consider the following statements: (2023) correct explanation for Statement-I
Statement-I: (c) Statement-I is correct but Statement-II
7th August is declared as the National is incorrect
Handloom day. (d) Statement-I is incorrect but Statement-
Statement-II: II is correct

It was in 1905 that the Swadeshi movement


was launched on the same day. 4. A recent movie titled The Man Who Knew
Which one of the following is correct in Infinity is based on the biography of: (2016)
respect of the above statements? (a) S. Ramanujan
(a) Both Statement-I and Statement-II are (b) S. Chandrasekhar
correct and Statement-II is the correct (c) S. N. Bose
explanation for Statement-I
(d) C. V. Raman
(b)
Both Statement-I and Statement-II
are correct and Statement-II is not the
correct explanation for Statement-I 5. H1N1 virus is sometimes mentioned in the
(c) Statement-I is correct but Statement-II news with reference to which one of the
is incorrect following diseases? (2016)

(d) Statement-I is incorrect but Statement- (a) AIDS


II is correct (b) Bird flu
(c) Dengue
3. Consider the following statements in (d) Swine flu
respect of the National Flag of India

GK/Persons in News 763


unacademy.com | Download the Unacademy app
Give your feedback here: Link
6. Indira Gandhi Peace Prize for Peace, 7. Karl Marx explained the process of class
Disarmament and Development for 2014 struggle with the help of which one of the
was given to which of the following? (2015) following theories? (2011)
(a) Bhabha Atomic Research Centre (a) Empirical liberalism
(b) Indian Institute of Science (b) Existentialism
(c) Indian Space Research Organisation (c) Darwin's theory of evolution
(d) Tata Institute of Fundamental Research (d) Dialectical materialism

764 GK/Persons in News


unacademy.com | Download the Unacademy app
Give your feedback here: Link
3 GK/Persons in News-Explanation

1. Answer: (d) 3. Answer: (d)


Statement 1 is not correct: Coal-based thermal Statement-I is incorrect:
power plants in India also use sea water.
The Mundra Thermal Power Plant employs a
closed-cycle induced draft circulating cooling
water system that utilizes seawater.
Statement 2 is not correct: According to
research by the World Resources Institute
(WRI), about 40% of India’s thermal power
plants in India are situated in the region
experiencing a significant water crisis. he
scarcity of water is already causing disruptions
in electricity generation in these areas, with 14 Statement-II is correct: According to the
out of India’s 20 largest thermal utilities having official guidelines for the Indian national flag,
experienced at least one shutdown between the ratio of its length to its height (width)
2013 and 2016 due to water shortages. should be 3:2. This means that the length of
Statement 3 is not correct: In India, a total of the flag should be 1.5 times its height. The
269 Thermal Power Plants are there, with 138 flag’s dimensions are designed to maintain the
of them being owned by the public sector and proper proportions and ensure its aesthetic
the remaining 131 owned by the private sector. appeal.

2. Answer: (a) 4. Answer: (a)


Statement-I is correct: 7th August is declared Option (a) is correct: “The Man Who Knew
as National Handloom Day by the Government Infinity,” is a biopic about the early-20th-
of India to commemorate the Swadeshi century mathematician Srinivasa Ramanujan,
Movement launched in 1905 and to honour who found the divine in integers. The film is
the handloom weavers of the country. The based on the 1991 book of the same name by
Swadeshi Movement played a significant role Robert Kanigel. The film stars Dev Patel as
in promoting Indian handloom and indigenous Srinivasa Ramanujan.
textiles as a symbol of India’s cultural heritage
and self-reliance.
Statement-II is correct: The Swadeshi
Movement was launched on 7th August 1905.
The Swadeshi Movement was a significant
socio-political movement during India’s
struggle for independence from British rule. It
aimed to promote the use of indigenous goods
and products and boycott British-made goods,
particularly textiles.

GK/Persons in News-Explanation 765


unacademy.com | Download the Unacademy app
Give your feedback here: Link
5. Answer: (d) Swine flu symptoms include fever, cough, sore
Option (d) is correct: H1N1 virus, originally throat, runny nose, etc. Like the regular flu,
referred to as “swine flu” because laboratory swine flu can lead to more serious problems,
testing showed that many of the genes in the including pneumonia, a lung infection, and
virus were very similar to influenza viruses that other breathing problems. H1N1 was spreading
normally occur in pigs (swine) in North America. fast around the world in 2009 so the World
Health Organization called it a pandemic.

6. Answer: (c) its path-breaking achievements, culminating


The Indira Gandhi Prize for Peace, Disarmament in the Mars Orbiter Mission; its significant
and Development was instituted in the memory contributions in strengthening international
of the former Prime Minister by a trust in her cooperation in the peaceful use of outer space,
name in 1986. It consists of a monetary award etc.
of Rs 25 lakh along with a citation.
Option (c) is correct: The Indira Gandhi Prize 7. Answer: (d)
for Peace, Disarmament and Development Option (d) is correct: Karl Marx made class
for 2014 was presented to the Indian Space struggle the central fact of social evolution.
Research Organisation (ISRO) in recognition of “The history of all hitherto existing human

766 GK/Persons in News-Explanation


unacademy.com | Download the Unacademy app
Give your feedback here: Link
society is the history of class struggles.” In important classes are grouped, oppose each
Marx’s view, the dialectical nature of history is other in the capitalist system: the owners of
expressed in the class struggle. He called this the means of production, or the bourgeoisie,
phenomenon dialectical materialism. With the and the workers, or the proletariat. “The
development of capitalism, the class struggle bourgeoisie produces its own grave-diggers.
takes an acute form. The fall of the bourgeoisie and the victory of
Two basic classes, around which other less the proletariat are equally inevitable.”

GK/Persons in News-Explanation 767


unacademy.com | Download the Unacademy app
Give your feedback here: Link
4 Miscellaneous

1. Consider the following statements: (2023) (b) Converting crop residues into packing
1. Carbon fibres are used in the material
manufacture of the components used (c) Producing biodegradable plastics
in automobiles and aircrafts. (d) Producing biochar from thermochemical
2. Carbon fibres once used cannot be conversion of biomass
recycled.
Which of the statements given above is/are 4. With reference to Home Guards, consider
correct? the following statements: (2023)
(a) 1 only 1. Home Guards are raised under the
(b) 2 only Home Guards Act and Rules of the
(c) Both 1 and 2 Central Government.

(d) Neither 1 nor 2 2. The role of the Home Guards is to serve


as an auxiliary force to the police in
maintenance of internal security.
2. Consider the following actions: (2023) 3. To prevent infiltration on the
1. Detection of car crash/collision which international border/coastal areas, the
results in the deployment of airbags Border Wing Home Guards Battalions
almost instantaneously have been raised in some States.
2. Detection of accidental free fall of How many of the above statements are
a laptop towards the ground which correct?
results in the immediate turning off of (a) Only one
the hard drive.
(b) Only two
3. Detection of the tilt of the smartphone
which results in the rotation of display (c) All three
between portrait and landscape mode (d) None
How many of the above actions is the
function of the accelerometer required? 5. The term ‘West Texas Intermediate’,
(a) Only one sometimes found in news, refers to a grade
(b) Only two of: (2020)

(c) All three (a) Crude oil

(d) None (b) Bullion


(c) Rare earth elements

3. ‘Wolbachia method’ is sometimes talked (d) Uranium


about with reference to which one of the
following? (2023) 6. The terms ‘Wanna Cry, Petya and Eternal
(a) Controlling the viral diseases spread by Blue' sometimes mentioned in the news
mosquitoes recently are related to: (2018)

768 Miscellaneous
unacademy.com | Download the Unacademy app
Give your feedback here: Link
(a) Exoplanets 10. ‘Project Loon’, sometimes seen in the news,
(b) Crypto currency is related to: (2016)

(c) Cyber attacks (a) Waste management technology

(d) Minisatellites (b) wireless communication technology


(c) solar power production technology

7. What is the purpose of the 'evolved Laser (d) water conservation technology
Interferometer Space Antenna (eLISA)'
project? (2017)
11. Recently, our scientists have discovered a
(a) To detect neutrinos new and distinct species of banana plant
(b) To detect gravitational waves which attains a height of about 11 metres
and has orange-coloured fruit pulp. In
(c) To detect the effectiveness of missile
which part of India has it been discovered?
defence system
(2016)
(d) To study the effect of solar flares on our
(a) Andaman Islands
communication systems
(b) Anaimalai Forests
(c) Maikal Hills
8. Consider the following statements: (2017)
(d) Tropical rain forests of northeast
1. In tropical regions, Zika virus disease is
transmitted by the same mosquito that
transmits dengue. 12. Which one of the following is the best
2. Sexual transmission of Zika virus disease description of ‘INS Astradharini’, that was
is possible. in the news recently? (2016)

Which of the statements given above is/are (a) Amphibious warfare ship
correct? (b) Nuclear-powered submarine
(a) 1 only (c) Torpedo launch and recovery vessel
(b) 2 only (d) Nuclear-powered aircraft carrier
(c) Both 1 and 2
(d) Neither 1 nor 2 13. What is ‘Greased Lightning-10 (GL-10)’,
recently in the news? (2016)

9. The terms 'Event Horizon', 'Singularity', (a) Electric plane tested by NASA
‘String Theory' and 'Standard Model' are (b) Solar-powered two-seater aircraft
sometimes seen in the news in the context designed by Japan
of: (2017)
(c) Space observatory launched by China
(a) Observation and understanding of the
(d) Reusable rocket designed by ISRO
Universe
(b) Study of the solar and the lunar eclipses
14. ‘Doctors Without Borders (Medecins Sans
(c) Placing satellites in the orbit of the
Frontieres)’, often in the news, is: (2016)
Earth
(a) A division of World Health Organisation
(d) Origin and evolution of living organisms
on the Earth (b)
A non-governmental international
organisation

Miscellaneous 769
unacademy.com | Download the Unacademy app
Give your feedback here: Link
(c)
An inter-governmental agency 17. Which one of the following was given
sponsored by European Union classical language status recently? (2015)
(d) A specialised agency of the United (a) Odia
Nations (b) Konkani
(c) Bhojpuri
15. The term ‘Base Erosion and Profit Shifting’ (d) Assamese
is sometimes seen in the news in the
context of: (2016)
(a)
mining operation by multinational 18. Which of the following brings out the
companies in resource-rich but ‘Consumer Price Index’ Number for the
backward areas Industrial Workers’? (2015)

(b)
curbing of the tax evasion by (a) The Reserve Bank of India
multinational companies (b) The Department of Economic Affairs
(c) exploitation of genetic resources of a (c) The Labour Bureau
country by multinational companies (d)
The department of Personnel and
(d) lack of consideration of environmental Training
costs in the planning and implementation
of developmental projects
19. With reference to bio-toilets used by the
Indian Railways, consider the following
16. Which of the following has/have been statements: (2015)
accorded ‘Geographical Indication’ status? 1. The decomposition of human waste in
(2016) the bio-toilets is initiated by a fungal
1. Banaras Brocades and Sarees inoculum.
2. Rajasthani Dal-Bati-Churma 2. Ammonia and water vapour are the only
3. Tirupati Laddu end products in this decomposition
which are released into the atmosphere.
Select the correct answer using the code
given below: Which of the statements given above is/are
correct?
(a) 1 only
(a) 1 only
(b) 2 and 3 only
(b) 2 only
(c) 1 and 3 only
(c) Both 1 and 2
(d) 1, 2 and 3
(d) Neither 1 nor 2

770 Miscellaneous
unacademy.com | Download the Unacademy app
Give your feedback here: Link
4 Miscellaneous-Explanation

1. Answer: (a) vice versa.


Statement 1 is correct: Carbon fibre is
increasingly celebrated as a wonder material 3. Answer: (a)
for the clean economy. Its unique combination The “Wolbachia method” is a biological
of high strength and low weight has helped approach used to control mosquito populations,
drive the wind power revolution and make specifically those that transmit diseases like
planes more fuel efficient. Auto makers are dengue, Zika, and chikungunya. Wolbachia
also waking up to the material’s potential to is a type of bacteria that naturally infects a
make lighter and more efficient vehicles. wide range of insects, including mosquitoes.
Statement 2 is not correct: The carbon fibre The method involves introducing Wolbachia
can be recycled. There is a need to divert bacteria into the mosquito population to
carbon fibre from landfill, they could open the reduce their ability to transmit diseases.
gates for use of recycled carbon fibre in cars, When female mosquitoes carrying Wolbachia
bikes and for dozens of other applications. bacteria mate with male mosquitoes that do
They could also save a lot of energy. not carry Wolbachia, the resulting eggs often
fail to develop or hatch. This leads to a decrease
2. Answer: (c) in the overall mosquito population. In addition,
the presence of Wolbachia can also interfere
1. Accelerometers are used in vehicles to
with the replication of certain disease-causing
detect sudden changes in acceleration
viruses within the mosquito, further reducing
or deceleration. When a car crash or
their ability to transmit diseases.
collision occurs, the accelerometer
senses the rapid change in motion and
triggers the deployment of airbags to 4. Answer: (b)
protect the occupants. Statement 1 is incorrect: Home Guards are
2.
Laptops often incorporate raised under the Home Guards Act and Rules of
accelerometers as a part of their the States/Union Territories. They are recruited
motion sensing systems. When the from amongst all classes of people and
laptop detects a sudden fall or change walks of life, who give their spare time to the
in orientation, the accelerometer organisation for betterment of the community.
measures the change in acceleration Amenities and facilities given to Home
and triggers actions like parking the Guards include free uniform, duty allowances
hard drive heads or shutting down the and award for gallantry, distinguished and
device to prevent damage. meritorious services

3. Accelerometers are commonly found in Statement 2 is correct: The role of the Home
smartphones and other portable devices. Guards is to serve as an auxiliary force to the
They detect changes in orientation and police in maintenance of internal security for
motion. When a smartphone is tilted, preventing infiltration on the international
the accelerometer senses the change border/coastal areas, guarding of VA/VPs and
in position and provides the necessary lines of communication in vulnerable areas at
input for the device to adjust the display the time of external aggression.
from portrait to landscape mode, or Statement 3 is correct: Fifteen Border Wing

Miscellaneous-Explanation 771
unacademy.com | Download the Unacademy app
Give your feedback here: Link
Home Guards (BWHG) Battalions have been grades or benchmarks (reference price for
raised in the border States viz. Punjab (6 Bns.), buyers and sellers), namely, Brent Crude and
Rajasthan ( 4 Bns.), Gujarat (2 Bns.) and one West Texas Intermediate (WTI).
each Battalion for Meghalaya, Tripura and Brent Crude: It is the most popular traded
West Bengal to serve as an auxiliary to Border grade, produced in the Brent oil fields and other
Security Force. sites in the North Sea. It is the benchmark for
African, European and Middle Eastern crude
5. Answer: (a) oils. This grade dictates nearly two-thirds of
the world’s crude oil production.
Option (a) is correct: West Texas Intermediate
is a particular grade of crude oil. This oil is WTI: It is the benchmark for crude oil for the
primarily sourced from Texas and serves as a United States. It, however, continues to be the
global benchmark for setting global oil prices. main benchmark oil consumed in the United
States.
Crude oil mainly comes under two different

772 Miscellaneous-Explanation
unacademy.com | Download the Unacademy app
Give your feedback here: Link
6. Answer: (c) noise: in this sense, eLISA will complement
Option (c) is correct: WannaCry is ransomware the efforts of ground-based gravitational wave
which is used for cyberattacks worldwide. detectors.
It targets computers running the Microsoft
Windows operating system by encrypting data
and demanding ransom payments in Bitcoin
cryptocurrency.
The Petya ransomware not only encrypts
files, but it also locks the entire disk, making
it basically unusable until the infection is
removed. It shuts down the system and asks
for a ransom of $300 in bitcoins on rebooting.
Code known as “Eternal Blue,” which cyber
security experts widely believe was stolen
from the US National Security Agency (NSA)
and was also used in a ransomware attack,
named “WannaCry.”
8. Answer: (c)
Statement 1 is correct: Zika virus disease is
7. Answer: (b)
caused by a virus transmitted primarily by
Option (b) is correct: The evolved Laser Aedes mosquitoes (responsible for transmitting
Interferometer Space Antenna (eLISA) is a dengue fever and chikungunya), which bite
mission aiming at exploring the Gravitational during the day. Symptoms are generally mild
Universe from space. and include fever, rash, conjunctivitis, muscle,
The eLISA mission consists of a “Mother” and and joint pain, malaise, or headache. Symptoms
two “Daughter” spacecraft. These will orbit typically last for 2–7 days. Most people with
the Sun in a triangular configuration. The three Zika virus infection do not develop symptoms.
satellites will form a precision interferometer, Statement 2 is correct: Zika can be passed
with the two Daughter spacecraft connected through sexual transmission from a person
to the mother one by 1 million km long laser with Zika to his or her partners. Sex includes
beams. This interferometer will be capable of vaginal, anal, and oral sex and the sharing of
detecting gravitational waves at frequencies in sex toys. Zika can be passed through sex even
the range of 0.1 MHz to 1 Hz. in a committed relationship. The timeframes
Such a frequency interval is not accessible that men and women can pass Zika through
on Earth due to arm length limitations and to sex are different because Zika virus can stay in
noise caused by the terrestrial gravity gradient semen longer than in other body fluids.

Miscellaneous-Explanation 773
unacademy.com | Download the Unacademy app
Give your feedback here: Link
9. Answer: (a) and are identical to one another.
Option (a) is correct: ‘Event Horizon’, ‘Singularity’, y The Standard Model is a theory that
‘String Theory’ and ‘Standard Model’ are used displays modern ideas about the original
for the observation and understanding of the basic material for building the Universe.
Universe. This model describes how matter is formed
y Singularity and Event Horizons are from its basic components; what interaction
concerned with Black Holes. Singularities forces exist between its components.
were first predicted as Einstein’s Theory
of General Relativity, which gave the
theoretical existence of black holes. In this,
any star that reaches beyond a certain point
in its mass would exert a gravitational force.
Event Horizon is the boundary defining the
region of space around a black hole from
which nothing (not even light) can escape.
y String Theory and ‘Standard Model’ try
to observe and understand the universe. 10. Answer: (b)
String Theory states that everything in our Option (b) is correct: Project Loon is a pilot
Universe is made up of tiny vibrating strings. project developed by Google LLC with an aim
These strings are one-dimensional objects of providing economic internet access across

774 Miscellaneous-Explanation
unacademy.com | Download the Unacademy app
Give your feedback here: Link
the world. It comprises a network of balloons 12. Answer: (c)
that would float above in the stratosphere Option (c) is correct: INS Astradharini is the
higher than the airplanes and the weather. Indian Navy’s first indigenously designed and
Project Loon targets such parts of the world and built torpedo launch and recovery vessel. The
others that have been hit by natural calamities unique design of the ship was a collaborative
and allows them internet connectivity. It uses effort of Naval Science and Technological
wireless communication technology to provide Laboratory, IIT Kharagpur and Shoft Shipyard.
internet services. The catamaran hull configuration significantly
reduces its power requirement.
INS Astradharini will be used to carry out the
technical trials of underwater weapons and
systems developed by the city-based NSTL.
It is an advanced replacement for Astravahini
which was decommissioned on July 17, 2015

11. Answer: (a)


Option (a) is correct: Scientists at the Botanical
Survey of India (BSI) have discovered a new
species of banana from a remote tropical
rainforest on the Little Andaman Islands. The
species, Musa indandamanensis, was located
about 16 km inside the Krishna Nalah Forest in
the island.
Wild banana species are largely distributed in 13. Answer: (a)
some tropical rainforests, wet evergreen forests
Option (a) is correct: In 2015, NASA Langley
to deciduous forests of low rainfall zones. The
researchers completed a carbon fiber hybrid-
major centres for these wild bananas can be
electric Vertical Take-Off and Landing (VTOL)
found from India to Indonesia. The Andaman
drone prototype named Greased Lightning (or
and Nicobar Islands in the Indian Ocean are
GL-10) and was successfully remotely piloted
little known.
from a hover to wing-borne flight back to
landing. The purpose of this research project
was to take advantage of new technologies,
electric engine propulsion and affordable drone
flight controllers (closed loop controllers) and
make an aircraft which would take on the best
aspects of an airplane and helicopter and fly
with superb reliability, affordably and excellent
stability.

Miscellaneous-Explanation 775
unacademy.com | Download the Unacademy app
Give your feedback here: Link
15. Answer: (b)
Base Erosion and Profit Shifting are the
techniques by which firms operating in
multiple jurisdictions shift their profits from
the jurisdiction of one nation to another by
exploiting tax laws and taking advantage of
lower tax rates. Through ways such as transfer
pricing, firms often manipulate their purchase
prices from their own related concerns in other
jurisdictions so that the profits accrue to the
related concern that is operating in a lower tax
jurisdiction.
Option (b) is correct: Governments have been
Greased Lightning-10 (GL-10) is an aircraft
worrying about such practices and take steps
configuration that combines the characteristics
to plug loopholes so that such practices
of a cruise efficient airplane with the ability to
cannot take place and curb tax evasion by
perform vertical take-off and landing (VTOL).
multinational companies. Advance pricing
This aircraft has been designed, fabricated
agreements form a way out of this.
and flight tested at the small unmanned aerial
system (UAS) scale.
16. Answer: (c)
14. Answer: (b) A Geographical Indication (GI) is a status given
to certain products which correspond to a
Option (b) is correct: Doctors Without
specific geographical location or origin (e.g.,
Borders/ Médecins Sans Frontieres (MSF)
a town, region, or country). The use of a GI
is an international, independent, medical
may act as a certification that the product
humanitarian organisation that delivers
possesses certain qualities, is made according
emergency aid to people affected by armed
to traditional methods, or enjoys a certain
conflict, epidemics, natural disasters and
reputation, due to its geographical origin.
exclusion from healthcare. MSF was established
in 1971 by a small group of French doctors who y Banarasi Brocade has been in existence
had worked in Biafra. Upon their return, they since the Mughal era and can be identified
were determined to find a way to respond rapidly with a narrow fringe-like pattern, called
and effectively to public health emergencies, Jhalar, found along the inner and outer
with complete independence from political, border. This fringe resembles a string of
economic and religious influences. leaves. Banarasi Brocade is a GI-protected
item. As per the GI registry, brocade sarees
Today, MSF is one of the world’s leading
made only in the districts of Varanasi,
independent international medical relief
Chandauli, Mirzapur, Jaunpur, Bhadohi
organisations, working in more than 70
and Azamgarh in Uttar Pradesh can be
countries worldwide and with operational
authentically identified as Banarasi saree
centres and national offices in 19 countries.
or brocade. [Option 1 is correct]
MSF offers assistance to people based on
need, irrespective of race, religion, gender or
political affiliation.

776 Miscellaneous-Explanation
unacademy.com | Download the Unacademy app
Give your feedback here: Link
17. Answer: (a)
Option (a) is correct: Odia became the sixth
language of the country to get classical language
status on 20th February 2014 after Malayalam
(2013), Telugu (2008), Kannada (2008), Sanskrit
(2005) and Tamil (2004). Sahitya Akademi is
the authority that gives the status of classical
languages.
According to information provided by the
Ministry of Culture in the Rajya Sabha in
February 2014, the guidelines for declaring a
language as ‘Classical’ are:
y High antiquity of its early texts/recorded
history over a period of 1500-2000 years.
y A body of ancient literature/texts, which
is considered a valuable heritage by
generations of speakers.
y The literary tradition is original and not
y Rajasthani Dal-Bati-Churma is the most borrowed from another speech community.
popular dish of Rajasthani cuisine. This meal
y The classical language and literature being
is an esteemed part of every Rajasthani
distinct from modern, there may also
household. This dish has not been accorded
be a discontinuity between the classical
‘Geographical Indication’ status yet. [Option
language and its later forms or its offshoots.
2 is not correct]
Once a language is notified as a classical
y India’s richest temple, the Venkateshwara
language, it will have the following benefits:
(Tirupati) temple, continues to have the
patent of its famous ‘Laddu’. It is the y Two major annual international awards for
decision by the Geographical Indications scholars of eminence in classical Indian
Registry which also granted Laddu the languages.
status of Geographical Indicator of the y A Centre of Excellence for Studies in
Tirupati region. [Option 3 is correct] Classical Languages is set up.
y The University Grants Commission is
requested to create, to start with at least in
the Central Universities, a certain number
of Professional Chairs for the Classical
languages so declared.

18. Answer: (c)


There are different consumer price indices
compiled by different organizations for different
segments of consumers, namely, industrial
workers (IW), Agricultural Labourers (AL) and
Rural Labourers (RL), CPI for Rural, Urban and
combined.

Miscellaneous-Explanation 777
unacademy.com | Download the Unacademy app
Give your feedback here: Link
Option (c) is correct: The Consumer Price
index (CPI) for Industrial Workers is released
by the Labour Bureau, Ministry of Labour
and Employment. CPI for Industrial Workers
measures the changes in prices over a period
of time for a specified basket of commodities
consumed by the Industrial Workers.
At all India levels, the Labour Bureau,
Chandigarh/Shimla, has been compiling the
CPI-IW with base 2001=100. The index is
released every month. Regulation of dearness
allowance in respect of all the workers and
employees belonging to private and public Statement 2 is not correct: Carbon dioxide and
sectors industries as well as Central and State Methane (not Ammonia and water vapour) are
Government employees are decided based on the end products released into the atmosphere.
CPI-IW.

19. Answer: (d)


Statement 1 is not correct: In the bio-toilets the
decomposition of human waste is initiated by a
bacterial inoculum. When human excreta come
in contact with bacteria, it gets converted into
methane and water through a series of steps of
anaerobic digestion—hydrolysis, acidogenesis,
acetogenesis and methanogenesis.

778 Miscellaneous-Explanation
unacademy.com | Download the Unacademy app
Give your feedback here: Link

You might also like